{"Type":"TOSS-UP\n\nMath - Short Answer","Prompt":"What is the determinant of the 2 by 2 matrix with first row -2, -3 and second row 8, -4?","Completion":"32"} {"Type":"TOSS-UP\n\nBiology - Multiple Choice","Prompt":"Which of the following types of microscopy require the specimen to be subjected to a vacuum?\n\nW) Fluorescence\n\nX) Transmission electron\n\nY) Scanning tunneling\n\nZ) Dark field","Completion":"X) TRANSMISSION ELECTRON"} {"Type":"TOSS-UP\n\nEarth and Space - Multiple Choice","Prompt":"How does the synodic period of the moon relate to its sidereal period?\n\nW) Its synodic period is two days longer\n\nX) Its synodic period is one day shorter\n\nY) Its synodic period is one day longer\n\nZ) Its synodic period is two days shorter","Completion":"W) ITS SYNODIC PERIOD IS TWO DAYS LONGER"} {"Type":"TOSS-UP\n\nPhysics - Short Answer","Prompt":"A small ball is fired horizontally off the edge of a table at 3 meters per second, taking 0.4 seconds to hit the ground below. When the ball hits the floor, how far, in meters, is it horizontally from the edge of the table?","Completion":"1.2"} {"Type":"TOSS-UP\n\nChemistry - Multiple Choice","Prompt":"Change in which of the following quantities must be positive for a spontaneous process?\n\nW) Gibbs free energy of the system\n\nX) Entropy of the system\n\nY) Gibbs free energy of the universe\n\nZ) Entropy of the universe","Completion":"Z) ENTROPY OF THE UNIVERSE"} {"Type":"BONUS\n\nEnergy - Multiple Choice","Prompt":"In a nuclear power plant, the fission reaction directly yields all but which of the following?\n\nW) Neutrons\n\nX) Fission products\n\nY) Fission gamma rays\n\nZ) Electrical current","Completion":"Z) ELECTRICAL CURRENT"} {"Type":"TOSS-UP\n\nBiology - Multiple Choice","Prompt":"Most cells are exposed to extracellular glucose concentrations higher than those inside the cell. Glucose is taken up by these cells via which of the following?\n\nW) Osmosis\n\nX) Active transport\n\nY) Exocytosis\n\nZ) Facilitated diffusion","Completion":"Z) FACILITY DIFFUSION"} {"Type":"TOSS-UP\n\nChemistry - Short Answer","Prompt":"Identify all of the following three metals that have soluble chlorides: Silver; Iron; Zinc.","Completion":"2 AND 3"} {"Type":"TOSS-UP\n\nEnergy - Multiple Choice","Prompt":"Which U.S. state received over 70% of its electricity from hydroelectric power in 2013?\n\nW) Maine\n\nX) Texas\n\nY) Nebraska\n\nZ) Washington","Completion":"Z) WASHINGTON"} {"Type":"TOSS-UP\n\nMath - Short Answer","Prompt":"What is the complex conjugate of -4 - 36?","Completion":"-4 + 36i"} {"Type":"TOSS-UP\n\nEnergy - Short Answer","Prompt":"What type of machine, used by both hydroelectric plants and uranium nuclear power plants, converts kinetic energy to electrical energy?","Completion":"GENERATOR (DO NOT ACCEPT: TURBINE or TURBINE-GENERATOR)"} {"Type":"BONUS\n\nEnergy - Short Answer","Prompt":"Characterized by its \"rotten-egg smell,\" what gas released by geothermal plants is of great environmental concern?","Completion":"HYDROGEN SULFIDE (ACCEPT: H\\({}{2}\\)S)"} {"Type":"TOSS-UP\n\nEarth and Space - Multiple Choice","Prompt":"Which of the following is true about radiometric dating?\n\nW) All parent material in a mineral is converted to the daughter product\n\nX) When the quantities of parent and daughter products are equal, two half-lives have transpired\n\nY) The number of radioactive atoms that decay during one half life is always the same\n\nZ) The percentage of radioactive atoms that decay during one half life is always the same","Completion":"Z) THE PERCENTAGE OF RADIOACTIVE ATOMS THAT DECAY DURING ONE HALF LIFE IS ALWAYS THE SAME"} {"Type":"TOSS-UP\n\nBiology - Multiple Choice","Prompt":"The size of most mammalian cells is closest to which of the\n\nfollowing?\n\nW) 1 picometer\n\nX) 1 micrometer\n\nY) 1 millimeter\n\nZ) 1 centimeter","Completion":"X) 1 MICROMETER"} {"Type":"TOSS-UP\n\nMath - Short Answer","Prompt":"What is the derivative with respect to \\(x\\) of -5 divided by \\(x^{6}\\)?","Completion":"30\/\\(x^{7}\\) (DO NOT ACCEPT: 30\\(x^{-7}\\))"} {"Type":"BONUS\n\nMath - Short Answer","Prompt":"What are the linear factors of the polynomial \\(2ab+3a-8b-12\\)?","Completion":"\\(a-4\\) AND \\(2b+3\\)"} {"Type":"TOSS-UP\n\nChemistry - Multiple Choice","Prompt":"Which of the following statements is NOT true regarding alloys?\n\nW) Alloys are generally more brittle than the metals from which they are made\n\nX) Alloys are generally harder than the metals from which they are made\n\nY) Alloys are generally better electric conductors than the metals from which they are made\n\nZ) Alloys are generally worse thermal conductors than the metals from which they are made","Completion":"Y) ALLOYS ARE GENERALLY BETTER ELECTRIC CONDUCTORS THAN THE METALS FROM WHICH THEY ARE MADE"} {"Type":"BONUS\n\nChemistry - Multiple Choice","Prompt":"Which of the following techniques would be most useful for determining the change in enthalpy of a reaction?\n\nW) Visible light spectroscopy\n\nX) Calorimetry [cal-or-IH-metree]\n\nY) Crystallography\n\nZ) Gravimetry [graa-VIH-metree]","Completion":"X) CALORIMETRY"} {"Type":"TOSS-UP\n\nPhysics - Short Answer","Prompt":"What general term refers to lighting that uses LEDs as sources of illumination rather than electrical filaments, plasma, or gas?","Completion":"SOLID-STATE LIGHTING (ACCEPT: SSL)"} {"Type":"TOSS-UP\n\nChemistry - Short Answer","Prompt":"A cylinder has an internal diameter of 3 centimeters and a piston is depressed 20 centimeters with a pressure of 2 atmospheres. Assuming that atmospheric pressure is 1 x 10\\({}^{5}\\) pascals, to two significant figures and in joules, how much work is done during the compression?","Completion":"28"} {"Type":"TOSS-UP\n\nPhysics - Short Answer","Prompt":"What particular type of Raman scattering occurs when the emitted photon is of higher wavelength than the incident one?","Completion":"STOKES"} {"Type":"TOSS-UP\n\nMath - Multiple Choice","Prompt":"Let f(x) be a continuous and differentiable function on the closed interval [3,6]. Which of the following could be the x-value guaranteed by the Mean Value Theorem for this situation?\n\nW) 2\n\nX) 3\n\nY) 5\n\nZ) 7","Completion":"Y) 5"} {"Type":"BONUS\n\nMath - Short Answer","Prompt":"One dimension of a cube is increased by 1, another is decreased by 1, and the other is left unchanged. If the new solid has a volume 5 less than that of the original cube, what was the volume of the original cube?","Completion":"125"} {"Type":"TOSS-UP\n\nBiology - Short Answer","Prompt":"The selective loss of dopaminergic [dope-ah-min-UR-jik] neurons in the substantia nigra [NYE-grahl] is characteristic of what neurodegenerative disease?","Completion":"PARKINSON'S DISEASE (ACCEPT: PARKINSONISM)"} {"Type":"BONUS\n\nBiology - Short Answer","Prompt":"Chondrocytes are associated with what structural material found in joints and spinal disks?","Completion":"CATILAGE"} {"Type":"TOSS-UP\n\nEnergy - Short Answer","Prompt":"If a solar power farm doesn't operate for 28% of the time in a month, what is its month-averaged capacity factor?","Completion":"72% (ACCEPT: 0.72)"} {"Type":"TOSS-UP\n\nChemistry - Short Answer","Prompt":"What allotrope of phosphorous is thermodynamically stable at room temperature and pressure and is structured as puckered sheets of linked atoms connected in 6-membered rings?","Completion":"BLACK PHOSPHORUS"} {"Type":"TOSS-UP\n\nPhysics - Multiple Choice","Prompt":"Which of the following statements correctly summarizes the zeroth law of thermodynamics?\n\nW) If each of two systems are in thermal equilibrium with a third system, they must be in thermal equilibrium with each other\n\nX) When energy passes as work, heat, or with matter, into or out of a system, its internal energy changes in accord with the law of conservation of energy\n\nY) In a natural thermodynamic process, the sum of the entropies of the participating thermodynamic systems increases\n\nZ) The entropy of a system approaches a constant value as the temperature approaches absolute zero","Completion":"W) IF EACH OF TWO SYSTEMS ARE IN THERMAL EQUILIBRIUM WITH A THIRD SYSTEM, THEY MUST BE IN THERMAL EQUILIBRIUM WITH EACH OTHER"} {"Type":"TOSS-UP\n\nEarth and Space - Multiple Choice","Prompt":"Which of the following low pressure systems typically has the weakest sustained wind speed?\n\nW) Tropical storm\n\nX) Tropical depression\n\nY) Hurricane\n\nZ) Tropical wave","Completion":"Z) TROPICAL WAVE"} {"Type":"TOSS-UP\n\nMath - Short Answer","Prompt":"How many points of intersection are there for the graphs of \\(y=3^{x}\\) and \\(y=-x^{2}+5\\)?","Completion":"2"} {"Type":"TOSS-UP\n\nChemistry - Short Answer","Prompt":"What general type of bond is formed when two lobes of an orbital of one atom overlap two lobes of an orbital of the other atom?","Completion":"PI BOND"} {"Type":"TOSS-UP\n\nEnergy - Multiple Choice","Prompt":"Which of the following percentages is closest to the percent of energy used in Texas in 2014 that was generated by wind?\n\nW) 10\n\nX) 20\n\nY) 30\n\nZ) 40","Completion":"W) 10"} {"Type":"TOSS-UP\n\nBiology - Short Answer","Prompt":"Identify all of the following three macromolecules that are digested primarily in the stomach: Starches; Nucleic acids; Proteins.","Completion":"3 (ACCEPT: PROTEINS)"} {"Type":"TOSS-UP\n\nEarth and Space - Multiple Choice","Prompt":"Which of the following is NOT an example of an ichnofossil?\n\nW) Shell\n\nX) Burrow\n\nY) Corpolite\n\nZ) Urolite","Completion":"W) SHELL"} {"Type":"TOSS-UP\n\nPhysics - Short Answer","Prompt":"An ionization chamber smoke detector uses the radiation from a small amount of radioactive material to detect the presence of smoke. What type of radiation is it that ionizes oxygen and nitrogen molecules, providing a steady current in the chamber, and then when impeded by smoke, disrupts the current, triggering the alarm?","Completion":"ALPHA (ACCEPT: ALPHA PARTICLES OR ALPHA DECAY)"} {"Type":"BONUS\n\nEarth and Space \\(-\\) Short Answer","Prompt":"What formation of porous sand and gravel, covering 450,000 square kilometers just east of the Rocky Mountains, is the largest aquifer in the United States?","Completion":"OGALLALA FORMATION (ACCEPT: OGALLALA AQUIFER)"} {"Type":"TOSS-UP\n\nBiology \\(-\\) Short Answer","Prompt":"During DNA replication, what protein is responsible for negatively supercoiling the DNA to relieve strain?","Completion":"TOPOISOMERASE (ACCEPT: DNA TOPOISOMERASE)"} {"Type":"BONUS\n\nBiology \\(-\\) Short Answer","Prompt":"What type of muscle contraction is NOT associated with a change in length?","Completion":"ISOMETRIC"} {"Type":"TOSS-UP\n\nMath - Short Answer","Prompt":"Identifify all of the four quadrants in which cosecant is positive.","Completion":"1 and 2 (ACCEPT: FIRST and SECOND)"} {"Type":"BONUS\n\nMath - Short Answer","Prompt":"What is the image of the point (0, when it is reflected across the line \\(y=x-1\\)?","Completion":"\\((1,-1)\\)"} {"Type":"TOSS-UP\n\nPhysics - Short Answer","Prompt":"A 100-kilogram box filled with coffee is sitting on your concrete garage floor. You decide to move the box, but you're going to push the box rather than lift it. How much force, in newtons to the nearest integer, must you exert on the box in order to get it moving, if the coefficient of static friction is 0.25?","Completion":"245"} {"Type":"BONUS\n\nPhysics - Multiple Choice","Prompt":"An oscillating system has a characteristic frequency of 10 radians per second and an effective spring constant of 200 Newtons per meter. What is the effective mass, in kilograms, of the oscillator?\n\nW) 1\n\nX) 2\n\nY) 5\n\nZ) 10","Completion":"X) 2"} {"Type":"TOSS-UP\n\nChemistry - Multiple Choice","Prompt":"Which of the following values, in volts, is closest to the standard\n\nemf [E-M-F] produced by the Daniell cell?\n\nW) 0.6\n\nX) 0.8\n\nY) 1.1\n\nZ) 1.5","Completion":"Y) 1.1"} {"Type":"TOSS-UP\n\nEnergy - Multiple Choice","Prompt":"Which of the following is the International Energy Agency's\n\n stance on energy subsidies?\n\nW) They should be phased out\n\nX) There should be greater international collaboration on subsidies\n\nY) Subsides are necessary for energy sustainability\n\nZ) Subsides are necessary to reduce greenhouse emissions","Completion":"W) THEY SHOULD BE PHASED OUT"} {"Type":"TOSS-UP\n\nEarth and Space - Short Answer","Prompt":"What is the plutonic equivalent of andesite?","Completion":"DIORITE"} {"Type":"BONUS\n\nEarth and Space - Short Answer","Prompt":"Identify all of the following four objects around which accretion disks are sometimes present: White dwarfs; Protostars; Stellar mass black holes; Supermassive black holes.","Completion":"ALL OF THEM"} {"Type":"TOSS-UP\n\nBiology - Short Answer","Prompt":"Tandem mass spectrometry is a technique that can be used to determine the primary structure of a protein. What two amino acids can mass spectrometry NOT differentiate?","Completion":"ISOLEUCINE AND LEUCINE"} {"Type":"BONUS\n\nBiology - Short Answer","Prompt":"A glutamate to valine mutation is well-known to be responsible for what disease that is associated with resistance to malaria?","Completion":"SICKLE CELL ANEMIA"} {"Type":"TOSS-UP\n\nMath - Short Answer","Prompt":"If \\(n\\) is an odd integer, how many lines of symmetry does a regular n-gon have?","Completion":"\\(n\\)"} {"Type":"TOSS-UP\n\nChemistry - Short Answer","Prompt":"By what process can a molecule in the singlet excited state relax to the ground state with concomitant [Kon-koh-MIT-ent] emission of light?","Completion":"FLUORESCENCE"} {"Type":"TOSS-UP\n\nPhysics - Short Answer","Prompt":"An RC circuit contains a 10-ohm resistor and a 20-millifarad capacitor. How long, in milliseconds, would it take for the voltage to fall by a factor of e\\({}^{-2}\\)?","Completion":"400"} {"Type":"BONUS\n\nPhysics - Short Answer","Prompt":"To one significant figure, how fast, in meters per second, would you be moving if you started at rest and accelerated at a constant rate of one meter per second squared for an entire year?","Completion":"30 MILLION"} {"Type":"TOSS-UP\n\nBiology - Multiple Choice","Prompt":"Your teacher has a passion for studying human immunoglobulins, but they have to be dimers. What immunoglobulin does your teacher study?\n\nW) IgD\n\nX) IgA\n\nY) IgM\n\nZ) IgE","Completion":"X) IgA"} {"Type":"TOSS-UP\n\nEnergy - Short Answer","Prompt":"After a nuclear reactor is shut down and fission stops, the fuel continues to produce a lower, but significant, amount of heat for several years. What is the term for this residual heat energy?","Completion":"DECAY HEAT"} {"Type":"TOSS-UP\n\nMath - Multiple Choice","Prompt":"Which of the following best describes the type of conic section represented by the equation \\(x^{2}+10x+y^{2}\\ +16\\) equals zero?\n\nW) Circle\n\nX) Ellipse\n\nY) Hyperbola\n\nZ) Parabola","Completion":"W) CIRCLE"} {"Type":"TOSS-UP\n\nChemistry - Short Answer","Prompt":"What class of molecules is characterized by the functional group containing atoms C-O-O-H?","Completion":"CARBOXYLIC ACID (DO NOT ACCEPT: ORGANIC ACID)"} {"Type":"TOSS-UP\n\nMath - Multiple Choice","Prompt":"Which of the following is equivalent to cosine56\\({}^{\\circ}\\)cosine23\\({}^{\\circ}\\) - sine23\\({}^{\\circ}\\)sine56\\({}^{\\circ}\\)?\n\nW) sine 33\\({}^{\\circ}\\)\n\nX) cosine 33\\({}^{\\circ}\\)\n\nY) sine 79\\({}^{\\circ}\\)\n\nZ) cosine 79\\({}^{\\circ}\\)","Completion":"Z) COSINE 79\\({}^{\\circ}\\)"} {"Type":"TOSS-UP\n\nEarth and Space - Short Answer","Prompt":"What fast-flowing, narrow currents form above sharp air mass boundaries in the atmospheres of rotating planets, including Earth?","Completion":"JET STREAMS"} {"Type":"TOSS-UP\n\nBiology - Short Answer","Prompt":"Identify all of the following three structures that can be found in both bacteria and plants: Nucleus; Peroxisome [per-OX-ih-sowm]; Mitochondndrion.","Completion":"NONE OF THEM"} {"Type":"TOSS-UP\n\nEnergy - Multiple Choice","Prompt":"Which of the following experimental batteries is able to charge in less than a minute?\n\nW) Copper ion\n\nX) Chromium ion\n\nY) Aluminum ion\n\nZ) Silicon","Completion":"Y) ALUMINUM ION"} {"Type":"TOSS-UP\n\nEarth and Space - Multiple Choice Which of the following is NOT true of the lunar maria?\n\nW) They formed within the first one billion years of the moon's history\n\nX) They cover about 50% of the moon's surface\n\nY) They are composed of iron-rich basalt\n\nZ) They are more abundant on the near side of the moon\n\nAnswer: X) THEY COVER ABOUT 50% OF THE MOON'S SURFACE\n\nBonus\n\nEarth and Space - Short Answer","Prompt":"Order the following three groups of photosynthetic\n\norganisms by their time of appearance on Earth, from first to last: Cyanobacteria [sye-ANNO-bacteria]; C4 plants; Ferns.","Completion":"1, 3, 2 (CYANOBACTERIA, FERNS, C4 PLANTS)"} {"Type":"TOSS-UP\n\nEnergy - Short Answer","Prompt":"The Oak Ridge National Laboratory revealed an electric vehicle,\n\npatterned after a Shelby Cobra, at the 2015 North American International Auto Show that was built\n\nusing what process?","Completion":"3D PRINTING (ACCEPT: ADDITIVE MANUFACTURING, ADDITIVE PROCESS)"} {"Type":"## BONUS\n\nBiology \\(-\\) Short Answer","Prompt":"What amino acid possesses a secondary amine functionality and as a result, has special constraints on its secondary structure?","Completion":"PROLINE"} {"Type":"## BONUS\n\nMath \\(-\\) Multiple Choice","Prompt":"According to the rational root theorem, which of the following is NOT a possible solution to the equation \\(x^{3}-2x^{2}+x-2=0\\)?","Completion":"W) -1 X) \\( icefrac{{1}}{{2}}\\) Y) 1 Z) 2 X) \\( icefrac{{1}}{{2}}\\)"} {"Type":"TOSS-UP\n\nPhysics - Multiple Choice","Prompt":"In a DC\/DC converter, the electrical energy is stored in which elements of the circuit?\n\nW) Capacitors and inductors\n\nX) Resistive elements\n\nY) Switching element\n\nZ) Diodes and protection circuitry","Completion":"W) CAPACITORS AND INDUCTORS"} {"Type":"TOSS-UP\n\nBiology - Multiple Choice","Prompt":"p53 is a protein associated with which of the following processes?\n\nW) Cell cycle\n\nX) Transcription\n\nY) Electron transport chain\n\nZ) Heme synthesis","Completion":"W) CELL CYCLE"} {"Type":"TOSS-UP\n\nChemistry \\(-\\) Short Answer","Prompt":"How many valence electrons does atomic silicon have?","Completion":"FOUR"} {"Type":"BONUS\n\nChemistry \\(-\\) Short Answer","Prompt":"Identify all of the following three quantities that increase when the electron in a hydrogen atom transitions from 1s to 2s: Energy of the electron; Value of azimuthal quantum number; Value of principal quantum number.","Completion":"1 AND 3"} {"Type":"TOSS-UP\n\nMath \\(-\\) Short Answer","Prompt":"What is the 4th term of the geometric series with first term 400 and second term 120?","Completion":"10.8 (ACCEPT: 10 4\/5, 54\/5)"} {"Type":"BONUS\n\nMath \\(-\\) Short Answer","Prompt":"Simplify the following expression involving the complex number \\(i\\): \\(i+2i^{2}+3i^{3}+4i^{4}\\)","Completion":"\\(2-2i\\)23) Earth and Space - Multiple Choice Which of the following metamorphic rocks cannot originate from shale?"} {"Type":"TOSS-UP\n\nChemistry - Short Answer","Prompt":"Order the following three solvents from least to greatest by the pKa of acetic acid when dissolved in that solvent: DMSO; Water; Methanol.","Completion":"2, 3, 1"} {"Type":"BONUS\n\nChemistry - Short Answer","Prompt":"Order the following three colors of light from least energetic to most energetic: Yellow; Violet; Red.","Completion":"3, 1, 2"} {"Type":"TOSS-UP\n\nEnergy - Short Answer","Prompt":"Identify all of the following three statements that illustrate disadvantages of hydroelectricity plants: Water used to run the plant may be reliant on precipitation; The plants can produce a lot of pollution; Aquatic habitats near the plants may be destroyed.","Completion":"1 AND 3"} {"Type":"BONUS\n\nEnergy - Multiple Choice","Prompt":"Which of the following establishes renewable energy use goals for the U.S. by a specific date?\n\nW) Net Metering Agreement\n\nX) National Renewable Energy Target Contract\n\nY) Solar Guerrillas Pact\n\nZ) Renewable Energy Portfolio Standards","Completion":"Z) RENEWABLE ENERGY PORTFOLIO STANDARDS"} {"Type":"TOSS-UP\n\nMath - Short Answer","Prompt":"What is the sum of the largest prime number less than 50 and the largest prime number less than 100?","Completion":"144"} {"Type":"BONUS\n\nMath - Short Answer","Prompt":"The bob on a pendulum swings through an arc 6 feet long when the string swings through an angle of 0.5 radians. What is the length of the pendulum in feet?","Completion":"12"} {"Type":"TOSS-UP\n\nEarth and Space - Multiple Choice","Prompt":"Which of the following is always true during the month of July?\n\nW) The Earth is near its orbital perihelion\n\nX) The Southern Hemisphere is tilted towards the Sun\n\nY) The sun does not rise over the Drake Passage\n\nZ) The sun does not set over Svalbard","Completion":"Z) THE SUN DOES NOT SET OVER SVALBARD"} {"Type":"BONUS\n\nEarth and Space - Short Answer","Prompt":"What is the resolving power, in arc seconds to one decimal place, of a 56.0 centimeter diffraction-limited telescope observing in the V band?","Completion":"0.2"} {"Type":"TOSS-UP\n\nPhysics - Short Answer","Prompt":"What fundamental force mediates the decay of a neutron into a proton, an electron, and a neutrino?","Completion":"WEAK FORCE (ACCEPT: WEAK NUCLEAR FORCE; WEAK NUCLEAR; WEAK)"} {"Type":"TOSS-UP\n\nBiology - Short Answer","Prompt":"What group of biotech-relevant enzymes, derived from bacteria, often recognize specific palindromic sequences of nucleic acids and introduce a double-stranded break?","Completion":"RESTRICTION ENZYMES (ACCEPT: RESTRICTION ENDONUCLEASES)"} {"Type":"## BONUS\n\nChemistry - Short Answer","Prompt":"Identify all of the following three statements that are true regarding carbon and graphite: Carbon atoms in diamond are sp3 hybridized; Graphite is a better conductor of electricity than diamond; Graphite carbon is tetrahedral.","Completion":"1 AND 2"} {"Type":"## BONUS\n\nEarth and Space - Short Answer","Prompt":"Identify all of the following three statements that are TRUE regarding calderas: Calderas can form atop shield volcanoes; Basalt-filled calderas tend to result from explosive eruptions; Calderas can be found on Venus and Io.","Completion":"1 AND 3"} {"Type":"TOSS-UP\n\nPhysics - Short Answer","Prompt":"What thermodynamic cycle, most often found in jet engines, uses adiabatic compression and expansion processes as well as isobaric heating and cooling processes?","Completion":"BRAYTON CYCLE"} {"Type":"BONUS\n\nPhysics - Multiple Choice","Prompt":"A former Russian service officer died in November 2006 after being diagnosed with acute radiation syndrome. During the investigation into his death, authorities were able to trace his activities, in part, because his body had excreted trace amounts of which of the following radioactive isotopes, which he had recently ingested?\n\nW) Polonium-210 with a half-life of 138 days\n\nX) Bismuth-213 with a half-life of 45 minutes\n\nY) Technetium-99 with a half-life of 6 hours\n\nZ) Uranium-238 with half-life of 4.5 billion years","Completion":"W) POLONIUM-210 WITH A HALF-LIFE OF 138 DAYS"} {"Type":"TOSS-UP\n\nEnergy - Short Answer","Prompt":"What environmental secondary pollutant results from sulfur dioxide that is produced during the combustion of fossil fuels?","Completion":"ACID PRECIPITATION (ACCEPT: ACID RAIN, SULFURIC ACID, HzSO4)"} {"Type":"BONUS\n\nEnergy - Short Answer","Prompt":"During operation of a nuclear power plant, some non-fuel uranium-238 present in the fuel is converted to a different element, which then acts as fuel to provide about 1\/3 of the total energy. What is this second element?","Completion":"PLUTONIUM"} {"Type":"TOSS-UP\n\nMath - Multiple Choice","Prompt":"In the conversion of the rectangular coordinates (-8, into polar coordinates, to the nearest whole number, what is the degree measure of the angle?\n\nW) 127\n\nX) 143\n\nY) 307\n\nZ) 323","Completion":"X) 143"} {"Type":"TOSS-UP\n\nBiology - Short Answer","Prompt":"What enzyme, found in the human stomach, is activated by hydrochloric acid and breaks down proteins?","Completion":"PEPSIN"} {"Type":"## BONUS\n\nChemistry - Short Answer","Prompt":"Identify all of the following three statements that are TRUE concerning colligative properties: Boiling point elevation is proportional to the molality of particles in a solution; The Van't Hoff factor for calcium chloride is close to 2; Adding a solid solute often increases the vapor pressure of a solution.","Completion":"1"} {"Type":"## BONUS\n\nMath - Short Answer","Prompt":"What is the remainder when \\(3x^{3}+6x^{2}-7x+2\\) is divided by the binomial \\(x+4\\)?","Completion":"-6617) Physics - Multiple Choice What is the name of the postulate of quantum mechanics that requires that quantum mechanical systems behave classically as quantum numbers approach infinity?"} {"Type":"TOSS-UP\n\nEarth and Space - Short Answer","Prompt":"What region of the H-R diagram, named after a Japanese astrophysicist, is occupied by protostars of fewer than 3 solar masses?","Completion":"HAYASHI TRACK"} {"Type":"TOSS-UP\n\nMath - Short Answer","Prompt":"A special coin has a 1\/3 probability of landing on heads, and a 2\/3 probability of landing on tails. In a series of three coin flips, what is the probability of the outcome sequence heads, heads, tails?","Completion":"2\/27"} {"Type":"TOSS-UP\n\nEarth and Space - Multiple Choice","Prompt":"If the ambient air cools 8 degrees Celsius for each kilometer of vertical elevation you gain as you ascend a low mountain in Tennessee, which of the following most likely describes the stability of the local atmosphere?\n\nW) Absolutely unstable\n\nX) Conditionally unstable\n\nY) Neutrally stable\n\nZ) Absolutely stable","Completion":"X) CONDITIONALLY UNSTABLE"} {"Type":"TOSS-UP\n\nBiology \\(-\\) Short Answer","Prompt":"To prevent the body from metabolizing Drug G too quickly, a pharmaceutical company develops Substance Q to be taken concurrently with Drug G. If Substance Q binds to the enzyme catalyzing the metabolism of Drug G and alters its active site, what type of inhibition is occuring?","Completion":"NON-COMPETITIVE INhibition"} {"Type":"TOSS-UP\n\nChemistry \\(-\\) Short Answer","Prompt":"Consider a gaseous mixture of helium and molecular hydrogen that is allowed to equilibrate to a constant internal temperature. What is the ratio of the root-mean-square speed of hydrogen molecules to that of helium atoms?","Completion":"SQUARE ROOT OF 2 (ACCEPT: \\(\\sqrt{2}\\) TO 1)"} {"Type":"TOSS-UP\n\nPhysics - Short Answer","Prompt":"By what process can a quantum particle cross potential energy hills without expending any energy?","Completion":"QUANTUM TUNNELING (ACCEPT: TUNNELING)"} {"Type":"TOSS-UP\n\nEarth and Space \\(-\\) Short Answer","Prompt":"What class of barred spiral galaxies has the most tightly wound arms?","Completion":"SBa (ACCEPT: a)"} {"Type":"BONUS\n\nEarth and Space \\(-\\) Multiple Choice","Prompt":"Which of the following rock types would be most susceptible to chemical weathering by tropical rainfall?\n\nW) Granite\n\nX) Andesite\n\nY) Dacite\n\nZ) Basalt","Completion":"Z) BASALT"} {"Type":"TOSS-UP\n\nEnergy \\(-\\) Short Answer","Prompt":"What energy use comprises the greatest percentage of energy used in commercial buildings in the US?","Completion":"HEATING"} {"Type":"BONUS\n\nEnergy \\(-\\) Short Answer","Prompt":"Uranium-235 and plutonium-239 can be used as fuel in a reactor because they are fissile. Uranium-238, the most abundant form of natural uranium, is NOT fissile, but can be converted to plutonium by absorbing neutrons. What term describes material that can be converted to fuel in this way?","Completion":"FERTILE"} {"Type":"TOSS-UP\n\nPhysics - Short Answer","Prompt":"Identify all of the following four elements that are most commonly used as semiconductor material in integrated circuits: Silicon; Copper; Germanium; Indium.","Completion":"1, 3 (ACCEPT: SILICON AND GERMANIUM)"} {"Type":"TOSS-UP\n\nBiology - Multiple Choice","Prompt":"Which of the following provides the best example of a situation in which genetic drift would play a significant role within a population?\n\nW) Non-random mating among plants with purple flowers leads to inbreeding and homozygous lethal conditions that prevent seed germination\n\nX) In a large population of plants with purple flowers, one individual exhibits white flowers caused by a mutation\n\nY) In a small population of plants with purple flowers, three white-flowered individuals exist. A deer eats two of the three white flowered plants, thus removing them from the population\n\nZ) In a small population of plants where some individuals have white and some have purple flowers, bees strongly prefer the purple flowers, thus more seeds are produced from the purple individuals.","Completion":"Y) IN A SMALL POPULATION OF PLANTS WITH PURPLE FLOWERS, THREE WHITE-FLOWERED INDIVIDUALS EXIST. A DEER EATS TWO OF THE THREE WHITE FLOWERED PLANTS, THUS REMOVING THEM FROM THE POPULATION"} {"Type":"TOSS-UP\n\nMath - Multiple Choice","Prompt":"Let f(\\(x\\)) be a polynomial function with real coefficients and at least one real root. Which of the following must be true?\n\nW) f(\\(x\\)) has an even number of non-real complex roots\n\nX) f(\\(x\\)) is of odd degree\n\nY) f(\\(x\\)) is an even function\n\nZ) The graph of f(\\(x\\)) has a horizontal asymptote","Completion":"W) f(\\(x\\)) HAS AN EVEN NUMBER OF NON-REAL COMPLEX ROOTS"} {"Type":"TOSS-UP\n\nChemistry - Short Answer","Prompt":"Molten cryolite is used in what industrial process?","Completion":"HALL-HEROUT PROCESS"} {"Type":"TOSS-UP\n\nEarth and Space - Multiple Choice","Prompt":"Which of the following clay minerals forms from the weathering of feldspar and is the main component of most porcelain?\n\nW) Kaolinite\n\nX) Chlorite\n\nY) Saponite\n\nZ) Vermiculite","Completion":"W) KaOLINITE"} {"Type":"TOSS-UP\n\nPhysics - Short Answer","Prompt":"What force carrying particle holds quarks together inside of hadrons?","Completion":"GLUONS"} {"Type":"TOSS-UP\n\nBiology - Short Answer","Prompt":"Chlorophylll is a pigment that chelates what metal center?","Completion":"MAGNESIUM"} {"Type":"TOSS-UP\n\nMath - Multiple Choice","Prompt":"Which of the following equations does NOT include the half-line \\(y\\)\\(=\\)\\(x\\), where \\(x\\) and \\(y\\) are positive, in its solution set?\n\nW) \\(x^{y}=y^{x}\\)\n\nX) cosine of the product \\(xy\\) equals sine of the product \\(xy\\)\n\nY) \\(x\\) natural log of \\(y=y\\) natural log of \\(x\\)\n\nZ) \\(2^{x}+3^{y}=2^{y}+3^{x}\\)","Completion":"X) COSINE OF THE PRODUCT \\(xy\\) EQUALS SINE OF THE PRODUCT \\(xy\\)"} {"Type":"TOSS-UP\n\nChemistry \\(-\\) Short Answer","Prompt":"Order the following three ions from smallest to largest in terms of ionic radii: Cl minus; Br minus; O two minus.","Completion":"3, 1, 2"} {"Type":"TOSS-UP\n\nEnergy \\(-\\) Multiple Choice","Prompt":"Which of the following policies allows users with renewable energy sources to sell excess electricity generated to the utility grid and receive compensation?\n\nW) Renewable Energy Portfolio Standards\n\nX) Feed-in Tariff\n\nY) Net Metering Agreement\n\nZ) Carbon Cap and Trade","Completion":"Y) NET METERING AGREEMENT"} {"Type":"TOSS-UP\n\nMath - Multiple Choice","Prompt":"Given two sets A and B with A union B containing exactly 3 elements, which of the following must be true about sets A and B?\n\nW) Each set contains at least 2 elements\n\nX) Each set contains at most 2 elements\n\nY) Each set contains at least 3 elements\n\nZ) Each set contains at most 3 elements","Completion":"Z) EACH SET CONTAINS AT MOST 3 ELEMENTS"} {"Type":"TOSS-UP\n\nBiology - Short Answer","Prompt":"What plant-signaling molecule, similar in structure to aspirin, is thought to be produced around infection sites and carried by the phloem throughout the plant?","Completion":"METHYLSALICYLIC ACID (ACCEPT: METHYL SALICYLATE, WINTERGREEN OIL)"} {"Type":"TOSS-UP\n\nPhysics - Short Answer","Prompt":"In the field of electrostatics, integrals of electric field over a closed surface yield electric flux. What is the term for such a closed surface?","Completion":"GAUSSIAN SURFACE (ACCEPT: GAUSSIAN)"} {"Type":"TOSS-UP\n\nMath - Short Answer","Prompt":"What is the log base 3 of 729?","Completion":"6"} {"Type":"TOSS-UP\n\nPhysics - Multiple Choice","Prompt":"Which of the following quarks does NOT have negative charge?\n\nW) Charm\n\nX) Strange\n\nY) Down\n\nZ) Bottom","Completion":"W) CHARM"} {"Type":"## 18 BONUS\n\nPhysics - Multiple Choice","Prompt":"Which of the following best describes the electromotive force produced in a circuit by a changing flux?\n\nW) Ampere's Law\n\nX) Faraday's Law\n\nY) Gauss' Law\n\nZ) Ohm's Law","Completion":"X) FARADAY'S LAW"} {"Type":"TOSS-UP\n\nEarth and Space - Multiple Choice","Prompt":"In a landlocked area with multiple plays and sparse low-lying vegetation, what type of research would best reveal how the local climate has changed over the past several thousand years?\n\nW) Palynology\n\nX) Scherochronology [sklair-oh-kraw-NAW-lah-jee]\n\nY) Dendoclimatology [den-droh-kye-math-TALL-ahjee]\n\nZ) Speleology [speed-ee-AWL-ah-jee]","Completion":"W) PALYNOLOGY"} {"Type":"BONUS\n\nEarth and Space - Short Answer","Prompt":"A fictional moon in a faraway galaxy has the same mass\n\nand atmospheric composition as the Earth, but its radius is twice as large. To one decimal place,\n\nin kelvins per kilometer, what would be the dry adiabatic lapse rate in this moon's atmosphere?","Completion":"2.5"} {"Type":"TOSS-UP\n\nBiology - Short Answer","Prompt":"Identify all of the following four cellular attachments that can be found in animal cells: Tight junction; Desmosome; Plasmodesmata [plazz-moh-dez-MAH-tah]; Adherens junction.","Completion":"1, 2, AND 4"} {"Type":"BONUS\n\nBiology - Short Answer","Prompt":"Identify all of the following three statements that are TRUE\n\nregarding E. coli: Gram negative; Forms spores; Rod-shaped.","Completion":"1 AND 3 (ACCEPT: GRAM NEGATIVE; ROD SHAPED)"} {"Type":"TOSS-UP\n\nMath - Multiple Choice","Prompt":"Which of the following is the contrapositive of the conditional statement: If p, then q?\n\nW) If q, then p\n\nX) If not p, then q\n\nY) If not p, then not q\n\nZ) If not q, then not p","Completion":"Z) IF NOT Q, THEN NOT P"} {"Type":"BONUS\n\nMath - Short Answer","Prompt":"What is the area of a triangle with sides of length 11, 8, and 5?","Completion":"4\\(\\backslash\\)21"} {"Type":"TOSS-UP\n\nChemistry - Short Answer","Prompt":"What is the molecular geometry of SOCl?","Completion":"TRIGONAL PYRAMIDAL (ACCEPT: TRIPOD; DO NOT ACCEPT: PYRAMIDAL, TETRAHERDAL)"} {"Type":"BONUS\n\nChemistry - Short Answer","Prompt":"The lattice enthalpy of a solid cannot be measured directly, but it can be measured through application of Hess's law to a series of physical and chemical changes. What is the term for this closed path of steps?","Completion":"BORN-HABER CYCLE7) Math - Short Answer If theta is an angle in the third quadrant and the cosine of theta equals negative five-thirteenths, what is the sine of theta?"} {"Type":"TOSS-UP\n\nBiology - Short Answer","Prompt":"Dynein [DYE-nee-in] and kinesin [kin-EE-sin] are proteins that bind to what cytoskeletal features?","Completion":"MICROTUBULES"} {"Type":"TOSS-UP\n\nEnergy - Multiple Choice","Prompt":"Biogas is comprised primarily of methane and what other gas?\n\nW) Carbon dioxide\n\nX) Carbon monoxide\n\nY) Nitrogen\n\nZ) Oxygen","Completion":"W) CARBON DIOXIDE"} {"Type":"TOSS-UP\n\nChemistry - Short Answer","Prompt":"One of the most important industrial uses of vanadium\n\n[vuh-NAY-dee-um] pentoxide is as a catalyst for the production of what acid?","Completion":"SULFURIC"} {"Type":"TOSS-UP\n\nEarth and Space - Multiple Choice","Prompt":"What bright features extending from the Sun's\n\nphotosphere, often loop-shaped, are composed of relatively cool plasma that emits visible light?\n\nW) Sunspots\n\nX) Flares\n\nY) Prominences\n\nZ) Van Allen belts","Completion":"Y) PROMINENCES"} {"Type":"TOSS-UP\n\nBiology - Short Answer","Prompt":"What is the term for malignant tumors of the connective tissues?","Completion":"SARCOMAS"} {"Type":"BONUS\n\nBiology - Multiple Choice","Prompt":"The mechanisms involved in aligning homologous chromosomes in meiosis are also useful for which of the following?\n\nW) Repairing nicks\n\nX) Repairing single-stranded breaks\n\nY) Repairing double-stranded breaks\n\nZ) Repairing point mutations","Completion":"Y) REPAIRING DOUBLE STANDED BREAKS"} {"Type":"TOSS-UP\n\nChemistry - Short Answer","Prompt":"The ideal gas constant is another form of what other constant, but written in energy per unit temperature per mole rather than energy per unit temperature per particle?","Completion":"BOLTZMANN CONSTANT"} {"Type":"BONUS\n\nChemistry - Short Answer","Prompt":"What is the molecular geometry about the nitrogen atoms in hydrazine [HIGH-drah-zeen]?","Completion":"TRIGONAL PYRAMIDAL (ACCEPT: TRIPOD; DO NOT ACCEPT: PYRAMIDAL, TETRAHERDAL)17) Energy - Short Answer The Department of Energy's advanced reactor research supports new designs for liquid metal-cooled fast reactors. In these new designs, what metal is used to cool the reactor fuel?"} {"Type":"## BONUS\n\nEnergy - Short Answer","Prompt":"What US Environmental Protection Agency regulation mandates that gasoline contain a minimal amount of ethanol or other biofuel?","Completion":"RENEWABLE FUEL STANDARD PROGRAM"} {"Type":"## BONUS\n\nMath - Short Answer","Prompt":"What is the standard deviation of the following list of 4 numbers: 3, 3, 9, 17?","Completion":"SQUARE ROOT OF 3319) Physics - Short Answer In an optical microscope, lenses of plastic and glass direct and focus the light. What is the name of the distance between the specimen being viewed and the front lens of the objective?"} {"Type":"TOSS-UP\n\nChemistry - Multiple Choice","Prompt":"Which of the following statements best explains why triethylamine [try-ethil-ah-MEEN] is more basic than ammonia?\n\nW) Alkyl groups elevate the energy of nitrogen's nonbonding HOMO\n\nX) Alkyl groups lower the energy of nitrogen's nonbonding HOMO\n\nY) Alkyl groups elevate the energy of nitrogen's nonbonding LUMO\n\nZ) Alkyl groups lower the energy of nitrogen's nonbonding LUMO","Completion":"W) ALKYL GROUPS ELEVATE THE ENERGY OF NITROGEN'S NONBONDING HOMO"} {"Type":"TOSS-UP\n\nEarth and Space - Short Answer","Prompt":"What type of rotating neutron stars are classified into rotation-powered, accretion-powered, and magnetars?","Completion":"PULSARS"} {"Type":"TOSS-UP\n\nMath - Short Answer","Prompt":"What is the dot product of \\(4\\mathbf{i}-6\\mathbf{j}\\) and \\(-3\\mathbf{i}+\\mathbf{j}\\)?","Completion":"-18"} {"Type":"TOSS-UP\n\nPhysics - Multiple Choice","Prompt":"Which of the following characteristics makes Xenon-135 an effective neutron poison in nuclear reactors?\n\nW) Cherenkov radiation from beta decay of Xenon-135\n\nX) Full valence shell\n\nY) High thermal neutron capture cross-section\n\nZ) Unstable with half-life of 9.2 hours","Completion":"Y) HIGH THERMAL NEUTRON CAPTURE CROSS-SECTION"} {"Type":"BONUS\n\nPhysics - Short Answer","Prompt":"What fraction of the speed of light must a person drive in order that incoming 450 terahertz red light would appear to be 540 terahertz green light?","Completion":"11\/61"} {"Type":"TOSS-UP\n\nMath - Short Answer","Prompt":"What is the sum of the infinite geometric series with first term 1\/4 and second term 1\/16?","Completion":"1\/3"} {"Type":"BONUS\n\nMath - Multiple Choice","Prompt":"A regular decagon is inscribed in a circle with radius 8. Which of the following is the length of one of its sides?\n\nW) 8 sine of 36\\({}^{\\circ}\\)\n\nX) 8 cosine of 36\\({}^{\\circ}\\)\n\nY) 16 sine of 18\\({}^{\\circ}\\)\n\nZ) 16 cosine of 18\\({}^{\\circ}\\)","Completion":"Y) 16 sine of 18\\({}^{\\circ}\\)"} {"Type":"TOSS-UP\n\nPhysics - Short Answer","Prompt":"What law of electromagnetism states that the total electric flux through a closed surface is equal to the total electric charge inside the surface divided by the permittivity of free space?","Completion":"GAUSS'S LAW (ACCEPT: GAUSS'S)"} {"Type":"BONUS\n\nPhysics - Short Answer","Prompt":"When a material transitions into a superconducting state, it actively excludes magnetic fields from its interior. What is the name of this electromagnetic\n\nphenomenon?","Completion":"MEISSNER EFFECT"} {"Type":"TOSS-UP\n\nEarth and Space - Short Answer","Prompt":"Identify all of the following three minerals that are formed by oxidative weathering: Hematite; Limonite; Goethite.","Completion":"ALL OF THEM"} {"Type":"BONUS\n\nEarth and Space - Multiple Choice","Prompt":"As Earth's temperature rises, sea levels will also rise due to the melting of continental ice. Why won't this effect be canceled by increased evaporation of seawater from the oceans?\n\nW) The thermohaline [thermo-HAY-line] circulation will slow down\n\nX) When ice sheets melt, they release latent heat\n\nY) Saltwater evaporates much more slowly than freshwater\n\nZ) The atmosphere is a much smaller water reservoir than the ocean","Completion":"Z) THE ATMOSPHERE IS A MUCH SMALLER WATER RESERVOIR THAN"} {"Type":"TOSS-UP\n\nBiology - Short Answer","Prompt":"What cytoskeletal component is composed of actin?","Completion":"MICROFILAMENTS (ACCEPT: CYTOSKEETAL FILAMENTS, THIN FILAMENTS)"} {"Type":"BONUS\n\nBiology - Multiple Choice","Prompt":"Which of the following is true regarding the differences between New World and Old World monkeys?\n\nW) New World monkeys evolved from prosimians, while Old World monkeys evolved from anthropoids\n\nX) New World monkeys are all arboreal, while Old World monkeys include some ground-\n\ndwelling species\n\nY) New World monkeys lack prehensile tails, while Old World monkeys have prehensile tails\n\nZ) New World monkeys have downward pointed noses, while Old World monkeys have flat, spreading noses","Completion":"X) NEW WORLD MONKEYS ARE ALL ARBOREAL, WHILE OLD WORLD MONKEYS INCLUDE SOME GROUND-DWELLING SPECIES"} {"Type":"TOSS-UP\n\nChemistry - Multiple Choice","Prompt":"According to VSEPR theory, which of the following could NOT be the molecular geometry of a compound with a trigonal bipyramidal electron-pair geometry?\n\nW) Trigonal bipyramidal [BYE-peer-ah-MID-all]\n\nX) Trigonal pyramidal\n\nY) Linear\n\nZ) See-saw","Completion":"X) TRIGONAL PYRAMIDAL"} {"Type":"BONUS\n\nChemistry - Short Answer","Prompt":"Consider the reaction A plus B yields C, which is second-order in the reactant A and first-order in the reactant B. If the concentrations of both A and B are doubled, then by what factor is the reaction rate multiplied?","Completion":"8"} {"Type":"TOSS-UP\n\nEnergy - Multiple Choice","Prompt":"Breader reactors use uranium-238. Based on the abundance of this isotope, which of the following best estimates how many years breeder reactors could operate with the Earth's supply of uranium-238?\n\nW) 500\n\nX) 5,000\n\nY) 5 million\n\nZ) 5 billion","Completion":"Z) 5 BILLION"} {"Type":"BONUS\n\nEnergy - Short Answer","Prompt":"Identify all of the following three statements that are true regarding carbon sequestration practices on farms: Groundcover is used to prevent soil from being washed or blown away; Groundcover is used as protection from the Sun to attract microbes; Livestock are only allowed to graze in one spot for 3-4 days at a time.","Completion":"1, 2, 3 (ACCEPT: ALL OF THEM)7) Math - Multiple Choice Consider a restaurant menu with nine meals. A table at the restaurant has \\(n\\) diners, all of whom order different meals. For which of the following values of \\(n\\) could the table add a diner without changing the number of combinations of meals the table can order?"} {"Type":"TOSS-UP\n\nEnergy - Multiple Choice","Prompt":"Which of the following is a major concern introduced with demand response programs and smart grids in general:\n\nW) Increased flow of energy in distribution and power systems.\n\nX) Decrease in system reliability due to intermittency of generation of renewable energy sources\n\nY) Increased rates of cost of electricity\n\nZ) Hackers can gain access to smart grid control software","Completion":"Z) Hackers CAN GAIN ACCESS TO SMART GRID CONTROL SOFTWARE"} {"Type":"TOSS-UP\n\nChemistry - Multiple Choice","Prompt":"Which of the following alkaline metal hydroxides has the\n\nhighest solubility in water?\n\nW) Magnesium hydroxide\n\nX) Calcium hydroxide\n\nY) Strontium hydroxide\n\nZ) Barium hydroxide","Completion":"Z) Barium HYDROXIDE"} {"Type":"TOSS-UP\n\nBiology - Multiple Choice","Prompt":"A deletion mutation that results in loss of the final 3 nucleotides of an organism's histidine tRNA gene will impair what function of the tRNA?\n\nW) Folding into functional tertiary structure\n\nX) Codon-anticodon base pairing with mRNA\n\nY) Binding to histidine\n\nZ) Transcription of the gene encoding histidine","Completion":"Y) BINDING TO HISTIDINE"} {"Type":"## BONUS\n\nEnergy - Short Answer","Prompt":"Identify all of the following three statements that are TRUE regarding biodiesel: B20 is commonly used in diesel equipment without modifications; Water is a common biodiesel additive to reduce corrosion; Biodiesel is produced via the Fischer-Tropsch process.","Completion":"1"} {"Type":"## BONUS\n\nPhysics - Multiple Choice","Prompt":"Two loudspeakers, A and B, are placed 4 meters apart and facing each other. The loudspeakers produce sound at 171 hertz in phase with one another. A microphone is moved with varying speed along a straight line that joins A and B. Which of the following statements is true?\n\nW) The microphone detects sound with no amplitude fluctuations as it moves\n\nX) The microphone detects sound with amplitude fluctuations that increase as its speed decreases\n\nY) The microphone detects sound with amplitude fluctuations that increase as its speed increases\n\nZ) The microphone detects no sound","Completion":"Y) THE MICROPHONE DETECTS SOUND WITH AMPLITUDE"} {"Type":"TOSS-UP\n\nPhysics - Multiple Choice","Prompt":"According to the equipartition theorem, by what factor of the Boltzmann constant does each translational degree of freedom of an ideal gas contribute to the heat capacity?\n\nW) 1\/2\n\nX) 1\n\nY) 2\n\nZ) 3","Completion":"W) 1\/2"} {"Type":"BONUS\n\nPhysics - Short Answer","Prompt":"What particles are thought to interact with standard particles via a force similar in strength to the weak force and are among the leading candidates for the composition of dark matter?","Completion":"WIMPS (ACCEPT: WEAKLY INTERACTING MASSIVE PARTICLES)"} {"Type":"TOSS-UP\n\nBiology - Multiple Choice","Prompt":"Helicobacter pylori [HEEL-ik-oh-bak-tur pie-LOR-ee] is associated with which of the following disorders?\n\nW) SARS\n\nX) Tuberculosis\n\nY) Stomach ulcers\n\nZ) Anemia","Completion":"Y) STOMACH ULCERS"} {"Type":"BONUS\n\nBiology - Multiple Choice","Prompt":"Vitamin K is important to which of the following processes?\n\nW) Metabolism\n\nX) Calcium absorption\n\nY) Night vision\n\nZ) Blood clotting","Completion":"Z) BLOOD CLOTTING"} {"Type":"TOSS-UP\n\nChemistry - Short Answer","Prompt":"What is the bond order of N\\({}{2}\\)+?","Completion":"2.5"} {"Type":"BONUS\n\nChemistry - Short Answer","Prompt":"Identify all of the following three statements concerning chemical kinetics that are TRUE: Zero-order rate constants have units of inverse seconds; First-order reactions have half-lives that are dependent on time; Catalysts cannot appear in the rate law for a reaction.","Completion":"NONE OF THEM"} {"Type":"TOSS-UP\n\nPhysics - Multiple Choice","Prompt":"Taking the acceleration due to gravity to be 10 meters per second squared, how much power, in watts, is required to lift a 2 kilogram mass at a constant speed of 2 meters per second?\n\nW) 2\n\nX) 4\n\nY) 40\n\nZ) 80","Completion":"Y) 40"} {"Type":"TOSS-UP\n\nEnergy - Multiple Choice","Prompt":"Which of the following would not be considered a balance-of-system component for a stand-alone wind power system?\n\nW) Battery\n\nX) Circuit breaker\n\nY) Inverter\n\nZ) Turbine","Completion":"Z) TURBINE"} {"Type":"TOSS-UP\n\nChemistry - Multiple Choice","Prompt":"Which of the following ions is most similar in ionic radius to the cation Na+?\n\nW) K\\({}^{+}\\)\n\nX) Rb\\({}^{+}\\)\n\nY) Mg\\({}^{2+}\\)\n\nZ) Ca\\({}^{2+}\\)","Completion":"Z) Ca\\({}^{2+}\\)"} {"Type":"BONUS\n\nChemistry - Short Answer","Prompt":"Identify all of the following three organic acids that would be more acidic in water than in acetic acid: Chloroacetic acid; Formic acid; Propanoic acid.","Completion":"1 AND 2 (ACCEPT: CHLOROACETIC ACID AND FORMIC ACID)"} {"Type":"TOSS-UP\n\nMath - Multiple Choice","Prompt":"Which of the following describes the solutions for the equation \\(x\\) to the \\(x\\) power = \\(x\\) to the \\(x+1\\) power?\n\nW) -1 and 1\n\nX) 0\n\nY) 1\n\nZ) No solution exists","Completion":"Y) 1"} {"Type":"BONUS\n\nMath - Short Answer","Prompt":"What is the sum of the first 6 terms in the geometric sequence with first term 3 and second term 12?","Completion":"40957) Physics - Multiple ChoiceA disc originally at rest moves through 6 radians as it undergoes a constant angular acceleration about its central axis. If the time taken to accelerate is 2 seconds, what is the magnitude of the acceleration in radians per square second?"} {"Type":"TOSS-UP\n\nBiology - Multiple Choice","Prompt":"Which of the following lengths of DNA would be least likely to be unwound by a passive helicase?\n\nW) ATATAT\n\nX) AAATTT\n\nY) TTTGGG\n\nZ) GGCGC","Completion":"Z) GGCGCGC"} {"Type":"BONUS\n\nBiology - Short Answer","Prompt":"Bacteria use what cellular structure for motility?","Completion":"FLAGELLUM"} {"Type":"TOSS-UP\n\nEarth and Space - Short Answer","Prompt":"Identify all of the following three phenomena that are formed by aeolian processes: Yardangs; Ventifacts; Dunes.","Completion":"ALL OF THEM"} {"Type":"BONUS\n\nEarth and Space - Short Answer","Prompt":"Identify all of the following three events that occurred during the recombination epoch of the Big Bang: Formation of neutral hydrogen; Neutrino decoupling; Star formation.","Completion":"1 ONLY"} {"Type":"TOSS-UP\n\nEnergy - Multiple Choice","Prompt":"Lithium cobalt oxide is most commonly used for what component of a lithium-ion battery?\n\nW) Electrolyte\n\nX) Cathode\n\nY) Enclosure\n\nZ) Anode","Completion":"X) CATHODE"} {"Type":"BONUS\n\nEnergy - Multiple Choice","Prompt":"What country used 100% renewable energy for 75 consecutive days in 2015?\n\nW) Lesotho [leh-SOH-toh]\n\nX) Costa Rica\n\nY) Norway\n\nZ) Sweden","Completion":"X) COSTA RICA"} {"Type":"TOSS-UP\n\nMath - Short Answer","Prompt":"The point \\((3,-\\) is reflected through the \\(y\\)-axis, with the result then translated 2 units left. What are the coordinates of the final image?","Completion":"\\((-5,-7)\\)"} {"Type":"BONUS\n\nMath - Short Answer","Prompt":"What is the distance between the two points with polar coordinates of \\(r=4\\) and theta = 60 degrees, and \\(r=6\\) and theta = 0 degrees?","Completion":"2\\(\\sqrt{7}\\)13) Biology - Short Answer Identify all of the following three conditions that negatively impact short term memory: 1) Post traumatic stress disorder; 2) Aphasia [ah-FAY-sjah]; 3) Alzheimer's."} {"Type":"TOSS-UP\n\nMath - Short Answer","Prompt":"What is the average of the integers from 1 to 21, inclusive?","Completion":"11"} {"Type":"BONUS\n\nChemistry - Short Answer","Prompt":"Identify all of the following three statements that are TRUE concerning crystal field theory: Carbon monoxide is a strong-field ligand; Tetrahedral splitting is larger than octahedral splitting; Square planar compounds can be chiral.","Completion":"1"} {"Type":"TOSS-UP\n\nEnergy - Multiple Choice","Prompt":"What biological carbon sequestration method uses iron and requires a better understanding of iron cycling in marine systems to determine its long-term impacts?\n\nW) Iron geo-reduction\n\nX) Iron sequestration\n\nY) Iron filtering\n\nZ) Iron fertilization","Completion":"Z) IRON FERTILIZATION"} {"Type":"BONUS\n\nEnergy - Short Answer","Prompt":"Recent work at Lawrence Berkeley National Laboratory has linked what metal to the health of the brain, showing that improper oxidation is not only linked to Alzheimer's disease, but also that it modulates spontaneous activity in developing circuits?","Completion":"COPPER17) Physics - Multiple Choice The current in a cathode ray tube is 10 nanoamps. Which of the following is closest to the number of electrons striking the face of the glass tube each second?"} {"Type":"TOSS-UP\n\nEarth and Space - Short Answer","Prompt":"Refracting telescopes have a severe limitation in that each wavelength of light is refracted differently, making it impossible to focus on more than one wavelength of light at a time. What is this type of optical aberration called?","Completion":"CHROMATIC ABERRATION (ALSO ACCEPT: CHROMATIC DISTORITION, SPHEROCHROMATISM)"} {"Type":"TOSS-UP\n\nBiology - Multiple Choice","Prompt":"Which of the following nucleobases is NOT a pyrimidine\n\n[pur-RIM-ih-deen]?\n\nW) Guanine [GWAH-neen]\n\nX) Cytosine [SYE-toh-seen]\n\nY) Thymine [THIGH-meen]\n\nZ) Uracil [YUR-ah-sil]","Completion":"W) GUANINE"} {"Type":"TOSS-UP\n\nPhysics - Short Answer","Prompt":"How much energy, in joules, is in a single kilo-watt hour?","Completion":"3.6 MILLION (ACCEPT: 3 MILLION 6 HUNDRED THOUSAND)"} {"Type":"BONUS\n\nPhysics - Short Answer","Prompt":"Studies of neutral kaon oscillations resulted in the discovery of what symmetry violation in particle physics?","Completion":"CP VIOLATION"} {"Type":"TOSS-UP\n\nChemistry - Short Answer","Prompt":"Given the percentages by mass of different elements in a compound, identify all of the following three characteristics of the compound that can be uniquely determined:The empirical formula; The molecular formula; The structure","Completion":"1"} {"Type":"TOSS-UP\n\nBiology - Short Answer","Prompt":"The webbed hand of a human embryo develops into five separate fingers by a process of programmed cell death. What is the term for this process?","Completion":"APOPTOSIS"} {"Type":"TOSS-UP\n\nMath - Multiple Choice","Prompt":"Matrices with which of the following pairs of dimensions cannot be multiplied together?\n\nW) 3 rows, 2 columns times 2 rows, 3 columns\n\nX) 6 rows, 5 columns times 6 rows, 5 columns\n\nY) 1 row, 2 columns times 2 rows, 2 columns\n\nZ) 1,000 rows, 2,000 columns times 2,000 rows, 4,000 columns","Completion":"X) 6 ROWS, 5 COLUMNS TIMES 6 ROWS, 5 COLUMNS"} {"Type":"TOSS-UP\n\nEarth and Space - Multiple Choice","Prompt":"What layer of the Earth's atmosphere contains approximately 75% of its air mass?\n\nW) Troposphere\n\nX) Mesosphere\n\nY) Stratosphere\n\nZ) Thermosphere","Completion":"W) TROPSPHERE"} {"Type":"TOSS-UP\n\nEnergy - Short Answer","Prompt":"Uranium pellets that are sealed into fuel rods are composed of what specific uranium compound?","Completion":"URANIUM DIOXIDE (ACCEPT: URANIUM(IV) OXIDE, URANOUS OXIDE, URANIA; DO NOT ACCEPT: URANIUM OXIDE)"} {"Type":"BONUS\n\nEnergy - Multiple Choice","Prompt":"What technology provides the vast majority of the worldwide grid energy storage capacity?\n\nW) Liquid or compressed air\n\nX) Lead-acid, nickel-cadmium and sodium-sulfur batteries\n\nY) Hydroelectric pump storage\n\nZ) Superconducting magnetic energy storage","Completion":"Y) HYDROELECTRIC PUMP STORAGE"} {"Type":"TOSS-UP\n\nBiology - Multiple Choice","Prompt":"Which of the following best describes how oxygen is stored in the muscles?\n\nW) Spread throughout\n\nX) Bound to hemoglobin\n\nY) Bound to calmodulin\n\nZ) Bound to myoglobin","Completion":"Z) BOUND TO MYOGLOBIN"} {"Type":"BONUS\n\nBiology - Multiple Choice","Prompt":"Which of the following examples is NOT evidence for evolution from a common ancestor?\n\nW) All living things have basic cell processes that are similar and use similar molecules\n\nX) Hox genes, or homeotic genes, control vertebrate body plans and are homologous in all\n\nvertebrates\n\nY) Even though snakes don't have legs, some snake embryos have limb buds during development similar to other reptile embryos\n\nZ) Bats and birds both have wings that are adaptations for flying","Completion":"Z) BATS AND BIRDS BOTH HAVE WINGS THAT ARE ADAPTATIONS FOR FLYING"} {"Type":"TOSS-UP\n\nPhysics - Multiple Choice","Prompt":"Which of the following is NOT true of laser light?\n\nW) It is highly coherent\n\nX) It is highly dispersive\n\nY) It is highly monochromatic\n\nZ) It is highly directional","Completion":"X) IT IS HIGHLY DISPERSIVE"} {"Type":"TOSS-UP\n\nChemistry - Multiple Choice A molecule with the molecular formula C\\({}{5}\\)H\\({}{12}\\) cannot form which of the following structural orientations?\n\nW) Straight-chain alkane\n\nX) Branched alkane\n\nY) Cycloalkane\n\nZ) Normal alkane\n\nAnswer: Y) CYCLOALKANE\n\nBonus\n\nChemistry - Short Answer","Prompt":"In the balanced equation for the combustion of octane, what are the coefficients for carbon dioxide and water respectively?","Completion":"16; 1811) Math - Short Answer There are 8 siblings in a family, and when ranked from youngest to oldest, all differ from the next oldest in age by the same number of years. If the youngest is 8 and the oldest is 29, what is the common difference in their ages?"} {"Type":"TOSS-UP\n\nBiology - Multiple Choice","Prompt":"As the average temperature dropped during the last glacial period, a species of cave bears evolved a layer of thicker fur. What kind of selection is this an example of?\n\nW) Speciation\n\nX) Disruptive selection\n\nY) Stabilizing selection\n\nZ) Directional selection","Completion":"Z) DIRECTIONAL SELECTION"} {"Type":"BONUS\n\nBiology - Multiple Choice","Prompt":"What substance composes the frustule [FRUH-stool] of a diatom?\n\nW) Silica\n\nX) Cellulose\n\nY) Keratin\n\nZ) Calcium carbonate","Completion":"W) SILICA"} {"Type":"TOSS-UP\n\nEnergy - Multiple Choice","Prompt":"Which of the following is the most common usage of syngas?\n\nW) Electricity production\n\nX) Transportation\n\nY) Water heating\n\nZ) Production of ethanol","Completion":"W) ELECTRICITY PRODUCTION"} {"Type":"BONUS\n\nEnergy - Short Answer","Prompt":"What is the most commonly used process for desulfurizing gas?","Completion":"CLAUS PROCESS15) Physics - Short AnswerA helicopter uses a main rotor as propulsion and control, but requires a tail rotor to counteract torque. The balance of forces resulting from this configuration is represented best by which of Newton's laws?"} {"Type":"BONUS\n\nChemistry - Multiple Choice","Prompt":"Consider both the hydrogenation [high-draw-jen-AY-shun] and the bromination [broh-min-AY-shun] of alkenes using elemental bromine. Which of the following statements concerning these reactions is TRUE?\n\nW) Both reactions are syn-additions\n\nX) Hydrogenation is a syn-addition, but bromination is an anti-addition\n\nY) Hydrogenation is an anti-addition, but bromination is a syn-addition\n\nZ) Both reactions are anti-additions","Completion":"X) HYDROGENATION IS A SYN-ADDITION, but Bromination IS AN ANTI ADDITION"} {"Type":"TOSS-UP\n\nMath - Short Answer","Prompt":"What are the critical numbers for the function \\(y=3t^{3}-t\\)?","Completion":"1\/3, -1\/3 (ACCEPT: \\(\\pm 1\/3\\))"} {"Type":"BONUS\n\nMath - Multiple Choice","Prompt":"Each day, Adam eats 20% of the cookies left in a jar. At the end of the 2\\({}^{\\rm nd}\\) day, there are 32 cookies left in the jar. How many cookies were in the jar originally?\n\nW) 40\n\nX) 48\n\nY) 50\n\nZ) 60","Completion":"Y) 5019) Chemistry - Short Answer One confirmed transformation of ununnotium [un-un-AWK-tee-um] is the decay of ununoctium-294 into livermorium [liver-MOOR-ee-um]-290. What type of decay process is this?"} {"Type":"BONUS\n\nChemistry - Multiple Choice","Prompt":"Which of the following IR spectrum wavenumber ranges are characteristic of alcohols, terminal alkynes, and amines?\n\nW) 3700 - 3200\n\nX) 3200 - 2700\n\nY) 2300 - 2100\n\nZ) 1750 - 1650","Completion":"W) 3700 - 3200"} {"Type":"TOSS-UP\n\nPhysics - Multiple Choice","Prompt":"Which of the following isotopes is fissile?\n\nW) Thorium-232\n\nX) Uranium-233\n\nY) Uranium-238\n\nZ) Plutonium-238","Completion":"X) URANIUM-233"} {"Type":"BONUS\n\nPhysics - Short Answer","Prompt":"Two resistors in parallel have an equivalent resistance of 6 ohms. One of the resistors has a resistance of 8 ohms. What is the resistance, in ohms, of the second resistor?","Completion":"24"} {"Type":"TOSS-UP\n\nEarth and Space - Multiple Choice","Prompt":"Hydraulic fracturing is used to obtain which of the following resources?\n\nW) Coal\n\nX) Natural gas\n\nY) Uranium ore\n\nZ) Water","Completion":"X) NATURAL GAS"} {"Type":"TOSS-UP\n\nBiology - Multiple Choice","Prompt":"Which of the following is NOT true about plant physiology?\n\nW) Meristematic [meh-rih-stuh-MAT-ic] tissue located at the tips of stems and roots forms the apical [A-pik-ul] meristens [MEH-rih-stems]\n\nX) Some plants, like shrubs, have lateral meristens\n\nY) Vascular and cork cambium [KAM-bee-um] are produced by apical meristens\n\nZ) Interaclay [In-TER-cah-lair-ee] meristems allow grass to grow even after it has been moved","Completion":"Y) VASCULAR AND CORK CAMBIUM ARE PRODUCED BY APICAL MERISTEMS"} {"Type":"TOSS-UP\n\nMath - Short Answer","Prompt":"How many lines of symmetry does a square have?","Completion":"4"} {"Type":"BONUS\n\nMath - Short Answer","Prompt":"What are the coordinates of the vertex of the graph of\n\n\\(y=-2x^{2}-8x+7\\)?","Completion":"(-2,15)"} {"Type":"TOSS-UP\n\nEarth and Space \\(-\\) Short Answer","Prompt":"What is a tornado called when it moves or develops over water?","Completion":"WATERSPOUT"} {"Type":"BONUS\n\nEarth and Space \\(-\\) Multiple Choice","Prompt":"As the Atlantic Ocean began to open around 200 million years ago, which of the following regions was most directly adjacent to the crust that now underlies the eastern margin of the United States?\n\nW) The British Isles\n\nX) The southwest margin of Scandinavia\n\nY) The northwest margin of Africa\n\nZ) The Walvis Ridge","Completion":"Y) THE NORTHWEST MARGIN OF AFRICA"} {"Type":"TOSS-UP\n\nBiology \\(-\\) Multiple Choice","Prompt":"In which of the following organelles does transcription NOT occur?\n\nW) Nucleus\n\nX) Peroxisome [purr-OX-ih-zohm]\n\nY) Chloroplast [KLOR-oh-plast]\n\nZ) Mitochondria [my-low-KON-dree-on]","Completion":"X) PEROXISOME"} {"Type":"BONUS\n\nBiology \\(-\\) Short Answer","Prompt":"A commonly-used method of classifying animals is to find a shared ancestral characteristic and use it to group organisms as either within the group or outside the group. Identify all of the following three animals that are inside the group for the hinged jaw characteristic: Lamprey;\n\nTurtle; Bass.","Completion":"2 AND 3"} {"Type":"TOSS-UP\n\nMath - Short Answer","Prompt":"Solve the following equation for \\(x\\): 4 divided by open parenthesis \\(x-6\\) close parenthesis equals 5 divided by open parenthesis \\(x-1\\) close parenthesis.","Completion":"26"} {"Type":"TOSS-UP\n\nChemistry - Multiple Choice","Prompt":"Which of the following elements is most electronegative according to the Pauling scale?\n\nW) Carbon\n\nX) Iron\n\nY) Arsenic\n\nZ) Tellurium","Completion":"W) CARBON"} {"Type":"TOSS-UP\n\nEnergy - Short Answer","Prompt":"Scientists at Lawrence Berkeley National Lab recently used molybdenum\n\n[muh-LIB-den-um] disulfide to generate a one-nanometer transistor gate that is resistant to what quantum-mechanical process, where electrons transition from one side of a potential barrier to another?","Completion":"TUNNELING (ACCEPT: QUANTUM TUNNELING)"} {"Type":"BONUS\n\nEnergy - Short Answer","Prompt":"A team of Berkeley Lab researchers recently published a study using a thin monolayer of an sp\\({}^{2}\\)-hybridized material to image electric fields. What is the common name of the material they used?","Completion":"GRAPHENE"} {"Type":"TOSS-UP\n\nPhysics - Short Answer","Prompt":"Given a graph with velocity as the \\(y\\) axis and time as the \\(x\\) axis, what physical quantity does the slope of the graph represent?","Completion":"ACCELERATION"} {"Type":"BONUS\n\nPhysics - Short Answer","Prompt":"By what factor is the frequency of a spring-mass oscillator multiplied if the mass of the block doubles?","Completion":"SQUARE ROOT 2 OVER 2"} {"Type":"TOSS-UP\n\nBiology \\(-\\) Multiple Choice","Prompt":"Which of the following human proteins, when fully assembled, contains exactly four subunits?\n\nW) Actin\n\nX) Myoglobin\n\nY) Thrombin\n\nZ) Hemoglobin","Completion":"Z) HEMOGLOBIN"} {"Type":"TOSS-UP\n\nEarth and Space \\(-\\) Multiple Choice","Prompt":"Which of the Milankovitch [mil-AYN-koh-vitch] cycles account for climate changes over a 41,000 year period?\n\nW) Eccentricity\n\nX) Obliquity [oh-BLIH-kwih-tee]\n\nY) Procession of the equinox\n\nZ) Apsidal [AP-sih-dul] precession","Completion":"X) OBLIQUITY"} {"Type":"TOSS-UP\n\nEnergy - Short Answer","Prompt":"William Robert Grove proposed the idea that if water can be broken down to hydrogen and oxygen with electricity, the opposite reaction should produce electricity. What device did he then go on to invent based on this idea?","Completion":"FUEL CELL (ACCEPT: HYDROGEN FUEL CELL)"} {"Type":"BONUS\n\nEnergy - Short Answer","Prompt":"The Planck and BICEP2 [BY-sep two] experiments, partially supported by the Department of Energy, were constructed to attempt to confirm what event in the early history of the universe?","Completion":"COSMIC INFLATION (ACCEPT: INFLATION, INFLATIONARY EPOCH; DO NOT ACCEPT: BIG BANG)"} {"Type":"TOSS-UP\n\nMath - Short Answer","Prompt":"Find the zeroes of the function f of \\(x\\) equals 6 over open parenthesis \\(x^{2}-6x+5\\) close parenthesis.","Completion":"NONE"} {"Type":"BONUS\n\nMath - Short Answer","Prompt":"What is the derivative of the function \\(x^{2}e^{x}\\) at \\(x=1\\)?","Completion":"3e11) Chemistry - Short Answer What quantum number is responsible for the energy of the orbital and corresponds to the shell of the electrons?"} {"Type":"TOSS-UP\n\nEarth and Space \\(-\\) Short Answer","Prompt":"Triton is a moon of what gas giant?","Completion":"NSPTUNE"} {"Type":"TOSS-UP\n\nMath \\(-\\) Short Answer","Prompt":"Simplify \\(x\\) raised to the 0.8 power times \\(x\\) raised to the 1.2 power.","Completion":"\\(x^{2}\\)"} {"Type":"TOSS-UP\n\nBiology - Short Answer","Prompt":"In animal cells, what type of cellular junction is formed by connexons [kah-NEX-uns] and enables molecules to pass from cell to cell?","Completion":"GAP JUNCTIONS"} {"Type":"BONUS\n\nBiology - Short Answer","Prompt":"A pea farmer starts with two parent plants, one dominant plant being true-breeding for yellow and round seeds, while the recessive plant is true-breeding for green and wrinkled seeds. If the F1 generation is self-pollinated, what fraction of the F2 generation will have yellow and round seeds?","Completion":"9\/16"} {"Type":"TOSS-UP\n\nPhysics - Short Answer","Prompt":"What is the ratio of the intensity of a 30-decibel sound to that of a 20-decibel sound?","Completion":"10 (ACCEPT: 10 TO 1)"} {"Type":"BONUS\n\nPhysics - Short Answer","Prompt":"A 4.5-centimeter tall object is placed 15 centimeters from a double convex lens with a 9-centimeter focal length. In centimeters, what is the image distance?","Completion":"22.5"} {"Type":"TOSS-UP\n\nChemistry - Short Answer","Prompt":"A student places a small amount of \\(n\\)-octane into a constant-volume reaction vessel and combusts the sample in excess oxygen. The heat given off in this reaction is a direct measure of the change in what state function between products and reactants?","Completion":"INTERNAL ENERGY"} {"Type":"BONUS\n\nChemistry - Short Answer","Prompt":"Identify all of the following three molecules that have an empirical formula consisting of only one type of atom: Charcoal; Diamond; Hexane.","Completion":"2"} {"Type":"TOSS-UP\n\nChemistry - Multiple Choice","Prompt":"Steven plots temperature versus time as he heats water while adding energy at a constant rate. Which of the following statements best explains why the heating curve reaches a plateau at 100 degrees Celsius?\n\nW) Energy is required to decrease the entropy [EN-trouh-pee] of the water\n\nX) Energy is required to decrease the enthalpy [EN-thul-pee] of the water\n\nY) Energy is required to break the hydrogen-oxygen bonds in water\n\nZ) Energy is required to break the hydrogen bonds between water molecules","Completion":"Z) ENERGY IS REQUIRED TO BREAK THE HYDROGEN BONDS BETWEEN WATER MOLECULES"} {"Type":"BONUS\n\nChemistry - Short Answer","Prompt":"How many distinct carbon signals are in the 13C NMR spectrum for ethyl isobutyl ether, also known as 1-ethoxy-2-methylpropane?","Completion":"5"} {"Type":"TOSS-UP\n\nBiology - Short Answer","Prompt":"How many chambers are in a fish heart?","Completion":"2"} {"Type":"BONUS\n\nBiology - Short Answer","Prompt":"Identify all of the following three types of plants for which the gametophyte [gah-MEET-oh-file] is the dominant stage of the plant life cycle: Mosses; Ferns; Gymnosperms [JIM-no-sperms].","Completion":"1"} {"Type":"TOSS-UP\n\nPhysics - Short Answer","Prompt":"Electric eels can generate a 500-volt potential difference that they use to stun their prey. If the potential difference is generated by a network of cells all connected in series, each of which can generate a 100-millivolt potential difference, how many cells are required to generate the total voltage?","Completion":"5000"} {"Type":"TOSS-UP\n\nEnergy - Short Answer","Prompt":"Scientists at Fermilab participate in the NOvA [nova] collaboration to understand the physics of neutrinos [new-TREE-noze]. One of their many projects is to understand the differences between the three flavor states of neutrinos. What are these three types?","Completion":"ELECTRON, MUON, AND TAU (ACCEPT: ELECTON, MUON, AND TAUON)"} {"Type":"TOSS-UP\n\nMath - Multiple Choice","Prompt":"In the conversion of the rectangular coordinates (-5, to polar coordinates, to the nearest whole number, what is the degree measure of the angle?\n\nW) 113\n\nX) 157\n\nY) 293\n\nZ) 337","Completion":"W) 113"} {"Type":"TOSS-UP\n\nEarth and Space - Short Answer","Prompt":"Relative to continental land masses, tropical cyclones typically occur to which cardinal direction?","Completion":"EAST"} {"Type":"TOSS-UP\n\nMath - Multiple Choice","Prompt":"With respect to \\(x\\), which of the following has -8x\\({}^{3}\\) as an antiderivative?\n\nW) -24\\(x^{2}\\)\n\nX) 24\\(x^{2}\\)\n\nY) -2\\(x^{4}\\)\n\nZ) 2\\(x^{4}\\)","Completion":"W) -24x\\({}^{2}\\)"} {"Type":"TOSS-UP\n\nBiology - Multiple Choice","Prompt":"In a tree, older, nonfunctioning xylem [ZYE-lun] is called what?\n\nW) Springwood\n\nX) Sapwood\n\nY) Summer wood\n\nZ) Heartwood","Completion":"Z) HEARTWOOD"} {"Type":"TOSS-UP\n\nEnergy - Short Answer","Prompt":"Scientists at the Oak Ridge Titan supercomputer are simulating the\n\ninteraction of electrons and lattice vibrations in copper-oxide materials. What property of these materials\n\nare they studying?","Completion":"SUPERCONDUCTIVITY (ACCEPT: HIGH-TEMPERATURE SUPERCONDUCTIVITY)"} {"Type":"TOSS-UP\n\nEarth and Space - Multiple Choice","Prompt":"Which of these is not an example of a Milankovich [mil-AYN-koh-vitch] cycle?\n\nW) Shifting tilt of the Earth's axis\n\nX) Axial \"wobble\"\n\nY) Sunspot cycle\n\nZ) Change in shape of orbit","Completion":"Y) SUNSPOT CYCLE"} {"Type":"BONUS\n\nEarth and Space - Short Answer","Prompt":"Identify all of the following three statements that are true of binary star systems: It is impossible to tell if a given star is actually an astrometric binary; For a visual binary, the masses of both stars can be determined; For a spectroscopic [spek-trou-SCAW-pik] binary, the semimajor axis can be determined.","Completion":"2 AND 3"} {"Type":"TOSS-UP\n\nChemistry - Short Answer","Prompt":"What is the systematic name of tert-butyl [turt-BYOO-til] chloride?","Completion":"2-CHLORO-2-METHYLPROPANE"} {"Type":"BONUS\n\nChemistry - Short Answer","Prompt":"Identify all of the following three compounds that can be considered soluble in water: Ammonium chloride; Silver bromide; Lead (II) sulfate.","Completion":"1"} {"Type":"TOSS-UP\n\nMath - Short Answer","Prompt":"A special coin has a 1\/3 probability of landing on heads, and a 2\/3 probability of landing on tails. In a series of three coin flips, what is the probability of the outcome sequence heads, tails, tails?","Completion":"4\/27"} {"Type":"BONUS\n\nMath - Short Answer","Prompt":"The perimeters of two squares differ by 16 inches and their areas differ by 92 square inches. What is the perimeter, in inches, of the larger square?","Completion":"54"} {"Type":"TOSS-UP\n\nEarth and Space - Short Answer","Prompt":"What sea in the North Atlantic Ocean, named for a genus of brown seaweed, is characterized by notably warm, clear, and nutrient-poor surface waters?","Completion":"SARGASSO SEA"} {"Type":"BONUS\n\nEarth and Space - Multiple Choice","Prompt":"Which of the following characteristics would make a species a poor index fossil?\n\nW) Globally-distributed\n\nX) Highly-conserved physical characteristics\n\nY) Short-lived in terms of geologic time\n\nZ) Ability to form mineral hard parts","Completion":"X) HIGHLY-CONSERVED PHYSICAL CHARACTERISTICS"} {"Type":"TOSS-UP\n\nPhysics - Multiple Choice","Prompt":"A mixture of steam and liquid water is being heated at an increasing rate. Which of the following best describes the temperature of the mixture?\n\nW) Increases at a constant rate\n\nX) Increases at an increasing rate\n\nY) Increases at a decreasing rate\n\nZ) Remains constant","Completion":"Z) REMAINS CONSTANT"} {"Type":"TOSS-UP\n\nEnergy - Short Answer","Prompt":"In general, nickel-cadmium batteries have fallen out of use and been replaced with another type of nickel-containing battery. What general type of nickel-containing battery replaced nickel-cadmium batteries?","Completion":"NICKEL-METAL HYDRIDE"} {"Type":"TOSS-UP\n\nEarth and Space \\(-\\) Short Answer","Prompt":"If a newly-discovered elliptical galaxy appears to be spherical, what Hubble classification will be given to it?","Completion":"E-ZERO"} {"Type":"TOSS-UP\n\nBiology \\(-\\) Short Answer","Prompt":"Flatworms, such as tapeworms, belong to what phylum [FYE-lum]?","Completion":"PLATYHELMINTHES [plat-ih-hell-MIN-thees]"} {"Type":"TOSS-UP\n\nPhysics - Short Answer","Prompt":"According to the Standard Model, what elementary boson [BOH-sawn] acts as the exchange particle for the strong nuclear force?","Completion":"GLUONS"} {"Type":"TOSS-UP\n\nBiology - Short Answer","Prompt":"When fermentation occurs in humans, pyruvate [pie-ROO-vate] is reduced by NADH to form what molecule?","Completion":"LACTATE (ACCEPT: LACTIC ACID)"} {"Type":"BONUS\n\nBiology - Short Answer","Prompt":"What flexible amino acid is present on nearly every third residue of collagen, due to its ability to fit on the inside of the triple helix?","Completion":"GLYCINE"} {"Type":"TOSS-UP\n\nChemistry - Short Answer","Prompt":"What is the hybridization [HIGH-brih-dye-ZAY-shun] of the central atom in bromine [BROH-meen] trifluoride?","Completion":"SP3D (ACCEPT: DSP3)"} {"Type":"BONUS\n\nChemistry - Short Answer","Prompt":"Consider the combustion reaction of solid graphite with gaseous [gah-shus] oxygen to generate gaseous carbon monoxide. Identify all of the following three changes that would shift the equilibrium of this reaction to the right: Adding more solid carbon; Increasing the temperature; Compressing the reaction vessel.","Completion":"NONE"} {"Type":"TOSS-UP\n\nMath \\(-\\) Multiple Choice","Prompt":"According to the rational root theorem, which of the following is NOT a possible solution to the equation \\(8x^{3}-2x^{2}+x-12=0\\)?\n\nW) 2\n\nX) 4\n\nY) 6\n\nZ) 8","Completion":"Z) 8"} {"Type":"TOSS-UP\n\nEarth and Space \\(-\\) Short Answer","Prompt":"What isotope accounts for approximately 7 parts per trillion of the Earth's atmosphere, and can be formed from the radioactive decay of tritium [TRIH-tee-um]?","Completion":"HELIUM-3"} {"Type":"TOSS-UP\n\nEnergy - Multiple Choice","Prompt":"Scientists at Brookhaven National Lab are studying molecular [moh-LEK-yoo-lur] wires as a potential polymeric replacement for silicon-based technologies. Which of the following polymers could behave as a molecular wire without any other functional groups?\n\nW) Polyester\n\nX) Polyphenylene [poly-FEH-nil-een] vinylene\n\nY) Polyamide [poly-AM-ide]\n\nZ) Polystyrene [poly-STYE-reen]","Completion":"X) POLYPHENYLENE VINYLENE"} {"Type":"BONUS\n\nEnergy - Short Answer","Prompt":"To better understand matter-antimatter asymmetry in the universe, scientists at SLAC [stack] are studying the CP-violating decays of what particle in the BaBar experiment?","Completion":"B MESON (ACCEPT: B PARTICLE)"} {"Type":"TOSS-UP\n\nBiology - Short Answer","Prompt":"What part of a myelinated [MY-ah-lin-ay-did] neuron is covered by myelin sheaths?","Completion":"AXON"} {"Type":"BONUS\n\nBiology - Short Answer","Prompt":"When triggered by inositol triphosphate, ions of what element are released as second messengers?","Completion":"CALCIUM"} {"Type":"TOSS-UP\n\nEarth and Space \\(-\\) Short Answer","Prompt":"The core of what planet in our solar system constitutes the largest fraction of the planet's volume?","Completion":"MERCURY"} {"Type":"TOSS-UP\n\nMath \\(-\\) Short Answer","Prompt":"An arithmetic sequence has fourth term 9 and ninth term 29. What is the positive difference between consecutive terms for the sequence?","Completion":"4"} {"Type":"TOSS-UP\n\nChemistry - Multiple Choice","Prompt":"A student prepares a saturated sodium chloride solution and then adds a few drops of 12-molar hydrochloric acid. Which of the following will occur?\n\nW) No visible changes will occur\n\nX) A large amount of gas will be evolved\n\nY) A hydrogen chloride precipitate will form\n\nZ) A sodium chloride precipitate will form","Completion":"Z) A SODIUM CHLORIDE PRECIPITATE WILL FORM"} {"Type":"TOSS-UP\n\nEarth and Space - Multiple Choice","Prompt":"Which of the following correctly describes the weather\n\nphenomenon rime?\n\nW) Frozen fog deposit\n\nX) A tornado that spins clockwise in the Northern Hemisphere\n\nY) High altitude lightning\n\nZ) Out-of-season hurricane","Completion":"W) FROZEN FOG DEPOSIT"} {"Type":"TOSS-UP\n\nBiology - Multiple Choice","Prompt":"The common dandelion, which is fluorescent under UV light, is pollinated by what?\n\nW) Bees\n\nX) Wind\n\nY) Butterflies\n\nZ) Moths","Completion":"W) BEES"} {"Type":"TOSS-UP\n\nEnergy - Short Answer","Prompt":"Researchers at Lawrence Berkeley National Lab recently demonstrated that certain pore complexes can recognize and block aberrant strands of mRNA from entering the cytosol [SIGH-toh-sol]. In what membrane do these pore complexes reside?","Completion":"NUCLEAR MEMBRANE (ACCEPT: NUCLEAR LAMIN)"} {"Type":"TOSS-UP\n\nMath - Short Answer","Prompt":"What is the complex conjugate of -7i?","Completion":"7\\(i\\) (ACCEPT: 0 + 7i)"} {"Type":"TOSS-UP\n\nPhysics - Short Answer","Prompt":"If the multiplicity of a system increases from 10 to 1000, by what factor is the entropy [EN-trou-peer] of the system multiplied?","Completion":"3"} {"Type":"TOSS-UP\n\nEarth and Space \\(-\\) Short Answer","Prompt":"What constellation contains the celestial South Pole?","Completion":"OCTANS (ACCEPT: L'OCTANS DE REFLEXION; OCTANS HADLEIANUS)"} {"Type":"TOSS-UP\n\nChemistry \\(-\\) Short Answer","Prompt":"What is the IUPAC name for the compound with the chemical formula\n\nK\\({}{2}\\)[Cu(CN)\\({}{4}\\)]?","Completion":"POTASSIUM TETRACYANOCUPRATE (II) [TWO]"} {"Type":"TOSS-UP\n\nBiology - Multiple Choice","Prompt":"When viewed in cross-section, in which plant organ would you find a star-shaped zone of xylem [ZYE-lum]?\n\nW) Monocot stem\n\nX) Gymnosperm [JIM-no-sperns] stem\n\nY) Dicot [DYE-kawf] root\n\nZ) Dicot stem","Completion":"Y) DICOT ROOT"} {"Type":"BONUS\n\nBiology - Short Answer","Prompt":"Identify all of the following three locations in a dNTP that, when radioactively labeled, will be retained in the final DNA molecule: Alpha phosphate; Beta phosphate; Gamma phosphate.","Completion":"1"} {"Type":"TOSS-UP\n\nPhysics - Short Answer","Prompt":"In orbital mechanics, what are the points called where a small object can maintain a constant position relative to two larger massive bodies?","Completion":"LAGRANGIAN POINTS (ACCEPT: L POINTS)"} {"Type":"BONUS\n\nPhysics - Short Answer","Prompt":"Order the following four materials from most negative to most positive in terms of their charge affinity when they undergo triboelectric [try-boh-electric] contact charging: Paper; Silver; Rabbit fur; PVC plastic.","Completion":"4, 2, 1, 3 (ACCEPT: PVC PASTIC, SILVER, PAPER, RABBIT FUR)"} {"Type":"TOSS-UP\n\nEarth and Space - Multiple Choice","Prompt":"Which of the following is closest to the average speed of the Earth's tectonic [tek-TAWN-ik] plates in centimeters of relative motion per year?\n\nW) 0.1\n\nX) 3\n\nY) 50\n\nZ) 200","Completion":"X) 3"} {"Type":"BONUS\n\nEarth and Space - Multiple Choice","Prompt":"The ocean floor can be mapped by altimeters [all-TIH-meh-durs] on satellites. Which of the following best describes how this method functions?\n\nW) The altimeter maps the ocean surface with radar, which is affected by underlying topography\n\nX) The altimeter maps the floor using sonar beams attached to the body of the altimeter\n\nY) The altimeter communicates with an underwater remotely operated vehicle, which maps the ocean floor beneath it\n\nZ) The altimeter communicates with a manned submersible, which maps the ocean floor beneath it","Completion":"W) THE ALTIMETER MAPS THE OCEAN SURFACE WITH RADAR, WHICH IS AFFECTED BY UNDERLYING TOPOGRAPHY"} {"Type":"TOSS-UP\n\nEnergy - Short Answer","Prompt":"Los Alamos National Lab scientists recently observed a special type of Bose [bohz]-Einstein condensate in a two-dimensional lattice. This room-temperature condensate was composed of what quasiparticles that carry vibrational energy in the lattice?","Completion":"PHONON"} {"Type":"BONUS\n\nEnergy - Short Answer","Prompt":"Scientists at Lawrence Berkeley National Lab recently constructed a genome [JEE-none]-wide map of more than 80,000 cis-acting DNA elements relevant to human heart development. What is the name of these eukaryotic [YOU-care-ee-AW-tic] DNA elements that are outside gene promoters and contain binding sites for transcription factors?","Completion":"ENHANCERS"} {"Type":"TOSS-UP\n\nChemistry - Short Answer","Prompt":"What is the molecular [moh-LEK-yoo-lur] geometry of phosphorus pentachloride?","Completion":"TRIGONAL BIPYRAMIADAL"} {"Type":"BONUS\n\nChemistry - Short Answer","Prompt":"Identify all of the following three hydrogen atoms that could be deprotonated [dee-PRO-tuh-nated] by sodium amide [AM-ide]: Hydrogen on terminal alkyne; Hydrogen alpha to ketone [KEE-tone]; Hydrogen on tri-substituted alkene.","Completion":"1 AND 2"} {"Type":"TOSS-UP\n\nMath \\(-\\) Multiple Choice","Prompt":"Which of the following equations is that of a line perpendicular to the line with equation \\(4x-5y=20\\)?\n\nW) \\(4x-5y=40\\)\n\nX) \\(4x+5y=40\\)\n\nY) \\(5x-4y=40\\)\n\nZ) \\(5x+4y=40\\)","Completion":"Z) \\(5x+4y=40\\)"} {"Type":"TOSS-UP\n\nBiology \\(-\\) Short Answer","Prompt":"What stage in animal development follows the blastula and is associated with the embryo folding inward?","Completion":"GASTRULA (ACCEPT: GASTRULATION)"} {"Type":"TOSS-UP\n\nPhysics - Multiple Choice","Prompt":"An object is placed 17 centimeters to the left of the vertex of a concave spherical mirror with a radius of curvature of 30 centimeters. Which of the following pairs of properties describe the image formed?\n\nW) Real and upright\n\nX) Real and inverted\n\nY) Virtual and upright\n\nZ) Virtual and inverted","Completion":"X) REAL AND INVERTED"} {"Type":"BONUS\n\nPhysics - Short Answer","Prompt":"A softball player exerts a constant 10-newton force to accelerate a 180-gram ball from rest to 30 meters per second. In seconds, how long does she take to throw the ball?","Completion":"0.54 (ACCEPT: 27\/50)"} {"Type":"TOSS-UP\n\nMath - Multiple Choice","Prompt":"For a twice-differentiable function \\(f(x),\\ f(\\) is 5, f prime of 4 is 0 and f\n\ndouble prime of 4 is -6. Which of the following is true of the graph of \\(y=f(x)\\) at the point (4, ?\n\nW) There is a local minimum\n\nX) There is a local maximum\n\nY) There is an inflection point\n\nZ) The graph is concave up","Completion":"X) THERE IS A LOCAL MAXIMUM"} {"Type":"BONUS\n\nMath - Short Answer","Prompt":"One dimension of a cube is increased by 3, another is decreased by 3, and the other is left unchanged. If the new solid has a volume 45 less than that of the original cube, what was the volume of the original cube?","Completion":"125"} {"Type":"TOSS-UP\n\nChemistry - Short Answer","Prompt":"Chlorine gas is prepared via the electrolysis [elec-TRAWL-ih-sis] of aqueous [AY-kwee-us] sodium chloride. This process also produces what strong base?","Completion":"SODIUM HYDROXIDE (ACCEPT: NaOH)"} {"Type":"BONUS\n\nChemistry - Short Answer","Prompt":"Rank the following three elements in increasing order of first ionization [eye-on-ih-ZAY-shun] energy: Beryllium [beh-RILL-ee-un]; Boron; Carbon.","Completion":"2, 1, 3"} {"Type":"BONUS\n\nBiology - Short Answer","Prompt":"What specific type of glial cell serves as the immune response in the central nervous system?","Completion":"MICROGLIA [micro-GLEE-ah]"} {"Type":"TOSS-UP\n\nPhysics - Multiple Choice","Prompt":"Which of the following distributions best describes the emission spectrum of cosmic microwave background radiation?\n\nW) Gaussian\n\nX) Fermi-Dirac\n\nY) Maxwell-Boltzmann\n\nZ) Blackbody","Completion":"Z) BLACKBODY"} {"Type":"BONUS\n\nPhysics - Short Answer","Prompt":"A 5-kilogram block is stationary on an inclined plane that is tilted 30 degrees from the horizontal. Increasing the incline by any amount causes the block to slip. To the nearest tenth, what is the coefficient of static friction between the block and the plane?","Completion":"0.6"} {"Type":"TOSS-UP\n\nChemistry - Multiple Choice","Prompt":"If the standard change in Gibbs free energy of a reaction is negative, which of the following MUST be true at standard temperature and pressure?\n\nW) Q is greater than one\n\nX) K is greater than one\n\nY) Q is less than one\n\nZ) K is less than one","Completion":"X) K IS GREATER THAN ONE"} {"Type":"BONUS\n\nChemistry - Short Answer","Prompt":"Identify all of the following three functional groups that are present in nucleotides: Ether; Alcohol; Thiol [THIGH-aw].","Completion":"1 and 215) Energy \\(-\\) Short AnswerIdentify all of the following three statements that are true of primary lithium cells: 1) The negative electrode is lithium; 2) They can be recharged; 3) They require no electrolyte."} {"Type":"TOSS-UP\n\nMath \\(-\\) Short Answer","Prompt":"A college class contains 25 freshmen, 35 sophomores, and 28 juniors. One of the students receives a phone call during the middle of the class. What is the probability that the ringing phone belongs to a junior?","Completion":"7\/22"} {"Type":"BONUS\n\nMath \\(-\\) Short Answer","Prompt":"What is the sixteenth power of \\(1+\\)\\(i\\)?","Completion":"256 (ACCEPT: \\(256+0i\\))17) Earth and Space \\(-\\)Short Answer What rock is transitional between igneous and metamorphic rocks, in which the felsic minerals have melted and recrystallized but the mafic [MAA-fik] minerals have not?"} {"Type":"BONUS\n\nEarth and Space \\(-\\) Short Answer","Prompt":"Identify all of the following three oxide minerals in which all of the iron is in the three plus oxidation state: Ferrihydrite [fair-ih-HIGH-drite]; Magnetite; Goethite [GUR-ite].","Completion":"1 AND 3"} {"Type":"TOSS-UP\n\nBiology \\(-\\) Short Answer","Prompt":"Chromatin [CROW-mah-tin] is a complex structure consisting of DNA wrapped around what proteins?","Completion":"HISTONES"} {"Type":"BONUS\n\nBiology \\(-\\) Short Answer","Prompt":"The somatosensory cortex is located in what lobe of the human cerebral [seh-REE-brul] cortex?","Completion":"PARIETAL"} {"Type":"TOSS-UP\n\nMath - Short Answer","Prompt":"If an exterior angle adjacent to a base angle of an isosceles triangle measures 124 degrees, what is the degree measure of the vertex angle?","Completion":"68"} {"Type":"TOSS-UP\n\nEarth and Space - Short Answer","Prompt":"If the moon is setting around midnight, then it is closest to what phase of the moon?","Completion":"FIRST QUARTER"} {"Type":"TOSS-UP\n\nChemistry - Short Answer","Prompt":"What is the term for copolymers [koh-polymers] that contain continuous segments of single monomers, rather than a random assortment of single monomers one after another?","Completion":"BLOCK POLYMER (ACCEPT: BLOCK COPOLYMER)"} {"Type":"BONUS\n\nChemistry - Short Answer","Prompt":"The osmotic pressure of a 0.5 molar solution of glucose is measured to be 12 atmospheres. To the nearest whole atmosphere, what is the osmotic pressure of a 2-molar solution of glucose?","Completion":"48"} {"Type":"TOSS-UP\n\nPhysics - Short Answer","Prompt":"In beta plus decay, what specific virtual particle mediates the conversion of a proton into a neutron, positron [PAW-zih-trawn], and neutrino [new-TREE-no]?","Completion":"W PLUS BOSON"} {"Type":"BONUS\n\nPhysics - Short Answer","Prompt":"Two objects each carry one coulomb [KOO-lawml] of excess charge, with one being positive and the other being negative. To the nearest integer multiple of one thousand meters, how far apart must they be so that they have a 22.5-newton attraction between them?","Completion":"20,000"} {"Type":"TOSS-UP\n\nBiology - Short Answer","Prompt":"Identifify all of the following three types of genetic material that viruses are known to carry: dsDNA; ssDNA; dsRNA.","Completion":"ALL"} {"Type":"TOSS-UP\n\nEnergy - Multiple Choice","Prompt":"Researchers at the DOE's Joint Genome [JEE-name] Institute recently performed a transcriptomic [trans-krip-TAWN-ik] analysis on mycorrhizal [MY-koh-RYE-zul] fungi [fun-jil] to understand their mechanisms of drought adaptation. Which of the following procedures was integral to this analysis?\n\nW) Measured numbers of RNAs for all of the fungal [FUN-gul] genes\n\nX) Sequenced the fungi's [fun-jize] DNA\n\nY) Measured the levels of every fungal protein\n\nZ) Sequenced all of the fungi's [fun-jize] proteins","Completion":"W) MEASURED NUMBERS OF RNAS FOR ALL OF THE FUNGAL GENES"} {"Type":"TOSS-UP\n\nPhysics - Multiple Choice","Prompt":"One vector has a magnitude of six meters, and the other has a magnitude of three meters. The magnitude of their vector product is 0.556. Which of the following is true of this system?\n\nW) The angle between the vectors is 33.7 degrees\n\nX) The angle between the vectors is 146.3 degrees\n\nY) The scalar product is 19.9 meters\n\nZ) There is not enough information to determine the angle or the scalar product","Completion":"Z) THERE IS NOT ENOUGH INFORMATION TO DETERMIINE THE ANGLE OR THE SCALAR PRODUCT"} {"Type":"TOSS-UP\n\nMath - Short Answer","Prompt":"What is the sum of the largest prime number less than 50 and the smallest prime number greater than 50?","Completion":"100"} {"Type":"BONUS\n\nMath - Multiple Choice","Prompt":"A regular 15-sided polygon is inscribed in a circle with radius 6. Which of the following is the length of one of its sides?\n\nW) 6 sine 24\\({}^{\\circ}\\)\n\nX) 6 cosine 24\\({}^{\\circ}\\)\n\nY) 12 sine 12\\({}^{\\circ}\\)\n\nZ) 12 cosine 12\\({}^{\\circ}\\)","Completion":"Y) 12 SINE 12\\({}^{\\circ}\\)"} {"Type":"TOSS-UP\n\nBiology - Short Answer","Prompt":"What cells in plants function similarly to embryonic [em-bree-ON-ic] stem cells in animals?","Completion":"MERISTEMATIC (ACCEPT: TOTIPOTENT)"} {"Type":"BONUS\n\nBiology - Short Answer","Prompt":"The adenovirus [AD-an-oh-virus] capsid is shaped like what platonic solid?","Completion":"ICOSAHEDRON (ACCEPT: REGULAR ICOSAHEDRON)"} {"Type":"TOSS-UP\n\nChemistry - Multiple Choice","Prompt":"Nylon is a polymer formed by condensation reactions of diamines [DYE-ah-meens] and dicarboxylic [dye-car-box-IHL-ik] acids. What functional group is formed by this reaction?\n\nW) Alcohol\n\nX) Nitrile\n\nY) Ester\n\nZ) Amide [AM-ide]","Completion":"Z) AMIDE"} {"Type":"TOSS-UP\n\nEarth and Space - Multiple Choice","Prompt":"Which of the following methods of determining distances to celestial objects is useful for the greatest distances?\n\nW) Stellar parallax\n\nX) Hubble's law\n\nY) Variable stars\n\nZ) Radar ranging","Completion":"X) HUBBLE'S LAW"} {"Type":"TOSS-UP\n\nEarth and Space - Multiple Choice","Prompt":"Which of the following is NOT true of stellar evolution and the HR diagram?\n\nW) White dwarfs occupy the lower left region of the HR diagram\n\nX) The sun falls on the main sequence\n\nY) The instability strip falls below the main sequence\n\nZ) The Hayashi track is a pre-main-sequence phase of stellar evolution","Completion":"Y) THE INSTABILITY STRIP FALLS BELOW THE MAIN SEQUENCE"} {"Type":"TOSS-UP\n\nBiology - Short Answer","Prompt":"In what phylum [FYE-lum] is an octopus classified?","Completion":"MOLLUSCA"} {"Type":"TOSS-UP\n\nPhysics - Short Answer","Prompt":"Doug pushes a 7-kilogram box up a 30-degree ramp onto the bed of a truck, which is 3 meters above the ground. In joules and rounded to the nearest 10, how much work did he do?","Completion":"210"} {"Type":"BONUS\n\nPhysics - Short Answer","Prompt":"According to Noether's [Noh-turz] Theorem, the conservation of angular momentum is related to what kind of spatial symmetry?","Completion":"ROTATIONAL (ACCEPT: ROTATIONAL INVARIANCE)"} {"Type":"TOSS-UP\n\nChemistry - Multiple Choice","Prompt":"A student observes the dimerization [dye-mur-ih-ZAY-shun] of a solution of cyclopentadiene [sigh-kloh-penta-DYE-een]. Assuming the reaction is second-order in cyclopentadiene, which of the following will she observe as she continues to monitor the reaction?\n\nW) The rate increases\n\nX) The rate remains constant\n\nY) The half-life increases\n\nZ) The half-life remains constant","Completion":"Y) THE HALF-LIFE INCREASES"} {"Type":"BONUS\n\nChemistry - Multiple Choice","Prompt":"Consider a gas-phase reaction where delta-H is less than delta-U. If the reaction is at equilibrium, which of the following changes will ALWAYS shift the reaction mixture to the right?\n\nW) Increasing the temperature\n\nX) Decreasing the temperature\n\nY) Increasing the volume\n\nZ) Decreasing the volume","Completion":"Z) DECREASING THE VOLUME"} {"Type":"TOSS-UP\n\nBiology - Short Answer","Prompt":"What property is exhibited by genes that have multiple phenotypic [FEE-no-TIP-ik] effects?","Completion":"PLEIOROPY [plee-AW-trou-pee]"} {"Type":"BONUS\n\nBiology - Multiple Choice","Prompt":"White flowers with a long, narrow corolla tube that flower at night are most likely pollinated by what animals?\n\nW) Beetles\n\nX) Birds\n\nY) Bats\n\nZ) Moths","Completion":"Z) MOTHS"} {"Type":"TOSS-UP\n\nEnergy - Short Answer","Prompt":"EXO-200 is a SLAC [slack]-supported experiment attempting to determine if anti-neutrinos [new-TREE-nozel] and neutrinos are the same particle. They are doing this by observing the double beta [BAY-tah] decay of what noble gas?","Completion":"XENON [ZE-non]"} {"Type":"BONUS\n\nEnergy - Short Answer","Prompt":"Scientists at Oak Ridge National Lab are using computer simulations to better understand the stability of calcium-52, which is inadequately described by current theories. Most surprisingly, despite having completely filled a nucleon shell, calcium-52 does not appear to behave like what set of nuclei?","Completion":"MAGIC NUCLEI (ACCEPT: MAGIC NUMBER NUCLEI)"} {"Type":"TOSS-UP\n\nEarth and Space - Multiple Choice","Prompt":"Which of the following pairs of tectonic [tek-TAWN-ik] plates does NOT constitute a primarily convergent boundary?\n\nW) North American and Cocos plates\n\nX) South American and Nazca plates\n\nY) African and Eurasian plates\n\nZ) Pacific and Antarctic plates","Completion":"Z) PACIFIC AND ANTACTIC PLATES"} {"Type":"TOSS-UP\n\nPhysics - Multiple Choice","Prompt":"The statement that magnetic monopoles do not exist in classical physics is most equivalent to which of the following statements?\n\nW) The magnetic field has a divergence of zero\n\nX) The electric field has a divergence related to total charge density\n\nY) A changing magnetic field will induce an electromotive force in a closed loop\n\nZ) A moving charge will induce a magnetic field","Completion":"W) THE MAGNETIC FIELD HAS A DIVERGENCE OF ZERO"} {"Type":"TOSS-UP\n\nEarth and Space \\(-\\) Short Answer","Prompt":"What mineral can be found on the continuous branch of Bowen's reaction series?","Completion":"PLAGIOCLASE FELDSPAR (ACCEPT: PLAGIOCLASE; DO NOT ACCEPT: FELDSPAR)"} {"Type":"BONUS\n\nEarth and Space \\(-\\) Short Answer","Prompt":"Identify all of the following three statements that are true regarding winds: In general, winds at the equator flow west; Air rising from the equator sinks around 5 degrees north and south latitudes; At 30 degrees south latitude, the sinking air generally yields clear skies.","Completion":"1 AND 3"} {"Type":"TOSS-UP\n\nMath \\(-\\) Short Answer","Prompt":"What is the discriminant of the quadratic function \\(y=x^{2}-5x+9\\)?","Completion":"-11"} {"Type":"BONUS\n\nMath \\(-\\) Short Answer","Prompt":"In a circle with radius 6, what is the area of a circular segment defined by an arc of \\(60^{\\circ}\\)?","Completion":"\\(6\\pi-9\\surd 3\\)23) Physics - Short AnswerA propeller is accelerating at 2.5 radians per second squared. After 4 seconds, it has rotated through 80 radians. What was the initial angular velocity, in radians per second, at the beginning of the 4 seconds?"} {"Type":"## 20 BONUS\n\nPhysics - Short Answer","Prompt":"A beam of light is traveling inside a solid cube of a material with an index of refraction of 1.414. If the cube is in air, what is the measure, to the nearest degree, of the minimum angle to the normal inside the glass so that the beam of light will not enter the air?","Completion":"45"} {"Type":"TOSS-UP\n\nEarth and Space \\(-\\) Short Answer","Prompt":"During a solar eclipse, an observer's view of the Sun is completely blocked by the moon. The observer is therefore standing in what region of the Moon's shadow?","Completion":"UMBRA"} {"Type":"BONUS\n\nEarth and Space \\(-\\) Multiple Choice","Prompt":"Which of the following statements best reflects our current understanding of the past and present of Mars' atmosphere?\n\nW) It has always been about as thick Earth's atmosphere today\n\nX) It has always been much thinner than Earth's atmosphere today\n\nY) It was thick shortly after formation, but has since thinned due to solar wind\n\nZ) It was thin shortly after formation, but has since thickened due to volcanic outgassing","Completion":"Y) IT WAS THICK SHORTLY AFTER FORMATION, BUT HAS SINCE THINNED DUE TO SOLAR WIND"} {"Type":"TOSS-UP\n\nChemistry \\(-\\) Short Answer","Prompt":"Identify all of the following three terms that can describe the behavior of water: Nucleophile [NU-klee-oh-file]; Lewis base; Bronsted [BRON-sted] acid.","Completion":"ALL"} {"Type":"BONUS\n\nChemistry \\(-\\) Short Answer","Prompt":"Identify all of the following three statements that are true of crystal field theory: \\(d\\) orbitals in a coordinated metal can be higher in energy than in the isolated metal ion; All \\(d\\) orbitals are elevated in energy equally when a metal is coordinated; In a coordinated metal ion, the \\(d\\) orbitals are degenerate.","Completion":"1"} {"Type":"TOSS-UP\n\nEnergy - Multiple Choice","Prompt":"DOE scientists working on the Dark Energy Survey at Fermilab\n\nserendipitously discovered a potential dwarf planet orbiting the Sun at 92 AU. Which of the following is closest, in Earth-years, to the orbital period of this object?\n\nW) 110\n\nX) 1,100\n\nY) 11,000\n\nZ) 110,000","Completion":"X) 1,100"} {"Type":"BONUS\n\nEnergy - Short Answer","Prompt":"Researchers supported by Ames Lab have recently provided spectroscopic\n\n[spek-troln-SCAW-pik] evidence for a type of material in which the electron and hole bands are connected at Weyl [wile] points, which are in turn connected by Fermi [FUR-mee] arcs. These materials are a specific type of what general group of materials in electronic band theory?","Completion":"SEMIMETAL"} {"Type":"TOSS-UP\n\nPhysics - Multiple Choice","Prompt":"Which of the following thermodynamic quantities is a path function?\n\nW) Energy\n\nX) Enthalpy [EN-thul-pee] Y) Entropy [EN-troln-pee] Z) Heat","Completion":"Z) HEAT"} {"Type":"BONUS\n\nPhysics - Short Answer","Prompt":"A solid uniform cylindrical rod has moment of inertia \\(I\\) when rotating about its center like a propeller. In terms of \\(I\\), what is the moment of inertia of a second rod of the same density but twice the length, when rotated about its symmetry axis?","Completion":"8 I"} {"Type":"TOSS-UP\n\nMath - Multiple Choice","Prompt":"Let f(x) be a continuous and differentiable function on the closed interval [2,6]. Which of the following could be the x-value guaranteed by the mean value theorem for this situation?\n\nW) 0\n\nX) 3\n\nY) 6\n\nZ) 9","Completion":"X) 3"} {"Type":"TOSS-UP\n\nBiology - Short Answer","Prompt":"Identify all of the following three classes that contain organisms that produce amniotic eggs: Reptilia; Aves [AY-veez]; Mammalia.","Completion":"ALL"} {"Type":"TOSS-UP\n\nBiology - Short Answer","Prompt":"What gene-editing method relies on synthetic guide RNAs to direct Cas nucleases [NEW-klee-aces] to target sites for gene editing?","Completion":"CRISPR (ACCEPT: CRISPR\/CAS, CRISPR\/CAS9)"} {"Type":"BONUS\n\nBiology - Multiple Choice","Prompt":"In a typical mouse mammalian research model, which type of epithelial [eh-ph-THEEL-ee-ul] cell would be most prevalent lining kidney tubules?\n\nW) Simple cuboidal [kyoo-BOY-dul] X) Simple columnar\n\nY) Simple squamous [SKWAY-mus] Z) Stratified columnar","Completion":"W) SIMPLE CUBOIDAL"} {"Type":"TOSS-UP\n\nMath - Short Answer","Prompt":"What is the largest prime factor of 165?","Completion":"11"} {"Type":"BONUS\n\nMath - Short Answer","Prompt":"What is the standard deviation of the following list of 4 numbers: 7, 7, 9, 17?","Completion":"\\(\\surd 17\\)"} {"Type":"TOSS-UP\n\nEnergy \\(-\\) Multiple Choice","Prompt":"Scientists at Oak Ridge National Lab are simulating the breakdown of lignocellulosic [Iigno-sellyoo-LOH-sik] biomass in the presence of tetrahydrofuran [tetrah-high-droh-FYOOR-an], or THF. What purpose does THF serve in this reaction?\n\nW) Catalyst\n\nX) Solvent\n\nY) Reactant\n\nZ) Chaotropic [kay-ah-TROH-pik] agent","Completion":"X) SOLVENT"} {"Type":"BONUS\n\nEnergy \\(-\\) Short Answer","Prompt":"There is significant interest in thorium as a next-generation fuel for nuclear fission [FIH-zhun] reactors. Identify all of the following three statements that are true regarding thorium:\n\nThorium can be found in nature as one nonfissile isotope; Thorium-232 can undergo neutron capture;\n\nThorium-232 is radioactive with a half-life of 21 minutes.","Completion":"1 AND 2"} {"Type":"TOSS-UP\n\nChemistry \\(-\\) Multiple Choice","Prompt":"A student is interested in the optical properties of chiral [KYE-ruhl] molecules, so she successively performs a chlorination, bromination [broh-min-AY-shun], and an\n\niodination [eye-oh-dah-NAY-shun] of methane. However, she is disappointed to find out that the resulting product demonstrates no optical rotation of polarized light. Which of the following best explains this?\n\nW) She prepared a racemic [ray-SEEM-ik] mixture\n\nX) Only certain chiral [KYE-ruhl] molecules demonstrate optical rotation\n\nY) Geometric isomers demonstrate optical rotation, not stereoisomers [stereo-EYE-soh-murs]\n\nZ) Only larger, more complex chiral molecules demonstrate optical rotation","Completion":"W) SHE PREPARED A RACEMIC MIXTURE"} {"Type":"BONUS\n\nChemistry \\(-\\) Multiple Choice","Prompt":"Consider the reaction of 1-bromopentane with sodium hydroxide.\n\nWhich of the following pairs of characteristics apply to this reaction?\n\nW) S\\({}{\\rm N}\\)1 at low temperatures and E1 at high temperatures\n\nX) E1 at low temperatures and S\\({}{\\rm N}\\)1 at high temperatures\n\nY) S\\({}{\\rm N}\\)2 at low temperatures and E2 at high temperatures\n\nZ) E2 at low temperatures and S\\({}{\\rm N}\\)2 at high temperatures","Completion":"Y) S\\({}{\\rm N}\\)2 AT LOW TEMPERATURES AND E2 AT HIGH TEMPERATURES"} {"Type":"TOSS-UP\n\nBiology - Multiple Choice","Prompt":"In the lac [lack] operon, allolactose can be classified as what?\n\nW) Repressor\n\nX) Inducer\n\nY) Promoter\n\nZ) Transcription factor","Completion":"X) INDUCER"} {"Type":"TOSS-UP\n\nMath - Short Answer","Prompt":"Point \\(P\\) is one of the vertices of a 42-sided convex polygon. How many diagonals of the polygon have point \\(P\\) as an endpoint?","Completion":"39"} {"Type":"BONUS\n\nEarth and Space \\(-\\) Short Answer","Prompt":"Identify all of the following three eras that belong to the Phanerozoic [fah-NAIR-oh-zoh-ik] con: Paleozoicoic [PAY-leo-ZO-ik]; Mesozoic [MEH-zoh-ZOH-ik]; Cenozoic [SEN-oh-ZOH-ik].","Completion":"ALL"} {"Type":"TOSS-UP\n\nChemistry \\(-\\) Multiple Choice","Prompt":"Which of the following atoms has an atomic radius closest to the atomic radius of sodium?\n\nW) Beryllium [beh-RILL-ee-um]\n\nX) Magnesium\n\nY) Calcium\n\nZ) Strontium [STRAWNCH-ee-um]","Completion":"Y) CALCIUM"} {"Type":"BONUS\n\nChemistry \\(-\\) Short Answer","Prompt":"Arrange the following three classes of organic compounds in increasing order of electrophilicity: Ketone [KEE-tone]; Ester; Amide [AM-ide].","Completion":"3, 2, 1"} {"Type":"TOSS-UP\n\nMath - Short Answer","Prompt":"What is the dot product of \\(4i-7j\\) and \\(5i+2j\\)?","Completion":"6"} {"Type":"TOSS-UP\n\nEarth and Space - Short Answer","Prompt":"On the planet Jupiter, what term describes the cooler, dark bands of latitude that correspond to atmospheric downwelling?","Completion":"BELTS"} {"Type":"TOSS-UP\n\nPhysics - Short Answer","Prompt":"In order to solve a physics problem, a student calculates the surface integral of the normal component of an electric field over a particular surface area. What physical quantity is the student determining?","Completion":"ELECTRIC FLUX"} {"Type":"BONUS\n\nPhysics - Short Answer","Prompt":"What is the resonant frequency, in radians per second, of a parallel LC circuit with an inductance of 2 millihenries and a capacitance of 80 microfarads [micro-FAIR-ad]?","Completion":"2,500"} {"Type":"TOSS-UP\n\nChemistry - Short Answer","Prompt":"P orbitals can form what two types of bonds?","Completion":"SIGMA AND PI"} {"Type":"BONUS\n\nChemistry - Short Answer","Prompt":"Identify all of the following three molecules that, according to molecular orbital theory, have non-zero bond orders: H-two [H-2]; He-two [H-E-2];\n\nHe-two plus [H-E-2-plus].","Completion":"1 AND 323) Biology - Short Answer In fungal [FUN-gul] reproduction, what stage is characterized by the haploid [HAP-loid] nuclei [NEW-klee-eye] of different parents fusing into a single diploid [DIH-polyd] nucleus?"} {"Type":"## BONUS\n\nBiology - Short Answer","Prompt":"Identify all of the following four phyla [FYE-lah] that possess a segmented body plan: Arthropoda ; Mollusca [mull-US-kah]; Nematoda; Chordata [kor-DAH-tah].","Completion":"1 AND 4"} {"Type":"TOSS-UP\n\nChemistry - Multiple Choice","Prompt":"Adding which of the following solutes to hexane would form a solution that most closely obeys Raoult's [rah-OOLZ] law?\n\nW) Acetone\n\nX) Benzene\n\nY) Pentane\n\nZ) Ethanol","Completion":"Y) PENTANE"} {"Type":"BONUS\n\nChemistry - Multiple Choice","Prompt":"A student isolates a natural product from a tropical frog and takes a proton NMR in deuterated chloroform. They notice that there is a sharp peak at 10.5 parts per million in the compound. This suggests the presence of what functional group?\n\nW) Nitrile\n\nX) Alcohol\n\nY) Aldehyde\n\nZ) Ether","Completion":"Y) ALDEHYDE"} {"Type":"TOSS-UP\n\nPhysics - Short Answer","Prompt":"In classical electromagnetic theory, the speed of light in a vacuum can be written exactly in terms of the permittivity of free space and what other fundamental physical constant?","Completion":"PERMEABILITY OF FREE SPACE (ACCEPT: PERMEABILITY, MU-0, MU-NAUGHT)"} {"Type":"BONUS\n\nPhysics - Short Answer","Prompt":"Identify all of the following three statements that are true regarding the relationship between momentum and energy: For a photon, if momentum is doubled, kinetic energy is doubled; For a neutron, if momentum is doubled, kinetic energy is doubled; For an alpha particle, if momentum is doubled, kinetic energy is doubled.","Completion":"1"} {"Type":"TOSS-UP\n\nEarth and Space \\(-\\) Multiple Choice","Prompt":"An igneous [IG-nee-us] rock sample is characterized as being composed of crystals of olivine, calcium-rich plagioclase, and augite, and has a phaneritic [fan-er-IT-ik] texture. Which of the following four rocks is it?\n\nW) Basalt [bah-SALT]\n\nX) Andesite\n\nY) Gabbro\n\nZ) Granite","Completion":"Y) GABBRO"} {"Type":"BONUS\n\nEarth and Space \\(-\\) Multiple Choice","Prompt":"You are mapping a flat erosional surface of an anticline fold in which no rock layers have been overturned. Which of the following describes the change in age of the rock layers in the anticline as one walks from the hinge to the limb?\n\nW) The layers get older\n\nX) The layers get younger\n\nY) The age of the layers is constant\n\nZ) Relative age cannot be determined","Completion":"X) THE LAYERS GET YOUNGER"} {"Type":"TOSS-UP\n\nMath \\(-\\) Short Answer","Prompt":"What is the partial derivative with respect to \\(x\\) of the expression \\(x^{3}y^{2}\\)?","Completion":"\\(3x^{2}y^{2}\\)"} {"Type":"BONUS\n\nMath \\(-\\) Short Answer","Prompt":"Teams A and B meet in a best-of-three series. If Team A wins any given game with probability 3\/5, what is the probability that Team A wins the series?","Completion":"81\/125 (ACCEPT: 0.648, 64.8%)5) Biology - Short Answer The Weddell seal is able to stay underwater for extraordinarily long periods of time due its ability to store twice as much oxygen per kilogram of body mass than humans. This ability is due to the seal having a high concentration of what protein in its muscles?"} {"Type":"### 11 TOSS-UP\n\nPhysics - Short Answer","Prompt":"A guitar string is plucked, producing the single A5 tone at 880 hertz. To two significant figures and in milliseconds, what is the period of vibration of this string?","Completion":"1.1"} {"Type":"TOSS-UP\n\nMath \\(-\\) Short Answer","Prompt":"The end of a pendulum swings through an arc 4 feet long when the string swings through an angle of 0.8 radians. In feet, what is the length of the pendulum?","Completion":"5"} {"Type":"TOSS-UP\n\nBiology \\(-\\) Short Answer","Prompt":"Which of the three meninges [meh-NIN-jeez] of the brain is closest to the cerebral [seh-REE-brull] cortex?","Completion":"PIA MATER"} {"Type":"TOSS-UP\n\nEnergy \\(-\\) Multiple Choice","Prompt":"When Oak Ridge National Lab physicists use the term \"doubly magic,\" it describes atomic nuclei [NEW-klee-eye] that have greater stability than their neighbors due to having shells that are fully occupied by both protons and neutrons. What nuclear property is maximized to create this effect?\n\nW) Neutron degeneracy [dee-JEN-er-ah-see] pressure\n\nX) Quantum spin\n\nY) Electron-degeneracy pressure\n\nZ) Nucleon binding energy","Completion":"Z) NUCLEON BINDING ENERGY"} {"Type":"BONUS\n\nEnergy \\(-\\) Multiple Choice","Prompt":"Researchers at Argonne National Lab are developing smart materials that can self-assemble and break down in response to external stimuli. They recently demonstrated self-assembly of nickel microparticles in the presence of what?\n\nW) Changing electrical field\n\nX) Changing magnetic field\n\nY) Changing temperature\n\nZ) Changing particle concentration","Completion":"X) CHANGING MAGNETIC FIELD"} {"Type":"TOSS-UP\n\nEarth and Space \\(-\\) Short Answer","Prompt":"For how many planets in the solar system is hydrogen the most abundant atmospheric gas?","Completion":"FOUR"} {"Type":"BONUS\n\nEarth and Space \\(-\\) Short Answer","Prompt":"An alien astronomer is observing our Sun as a flat disk with a temperature of 5800 kelvins when Jupiter passes in front of it. Given that the Sun has a radius of approximately 430,000 miles and Jupiter has a radius of approximately 43,000 miles and that Jupiter contributes negligible brightness to the system, then, to one significant figure, by what percentage does the brightness of the disk drop when Jupiter passes in front of the Sun?","Completion":"1"} {"Type":"TOSS-UP\n\nBiology - Multiple Choice","Prompt":"Most angiosperm [AN-jee-oh-sperns] species can be classified in which of the following clades?\n\nW) Basal [BAY-zul] X) Magnoliids [mag-NO-lee-idz] Y) Monocots","Completion":"Z) Eudicots [YOU-dih-kawts] Z) EUDICOTS"} {"Type":"BONUS\n\nBiology - Multiple Choice","Prompt":"Which of the following hormones is primarily responsible for maintaining pregnancy in most mammalian species?\n\nW) Estrogen\n\nX) Lutenizing hormone\n\nY) Progesterone","Completion":"Z) Prostaglandin [PRAW-stah-glan-din] Y) PROGESTERONE"} {"Type":"TOSS-UP\n\nPhysics - Multiple Choice","Prompt":"The liquid-drop model of the nucleus was originally proposed based on what observation?\n\nW) Rutherford gold-foil experiment\n\nX) All nuclei [NEW-klee-eyel] have similar densities\n\nY) Emission lines vary between elements\n\nZ) Millikan oil-drop experiment","Completion":"X) ALL NUCLEI HAVE SIMILAR DENSITIES"} {"Type":"BONUS\n\nPhysics - Short Answer","Prompt":"A square surface with side length 3 meters is placed in a constant magnetic field of 2 teslas such that the normal to the surface forms a 45-degree angle with the field lines. In webers [WEH-burz]to the nearest whole number, what is the magnetic flux through the surface?","Completion":"13"} {"Type":"TOSS-UP\n\nChemistry - Multiple Choice","Prompt":"Which of the following carboxylic [CAR-box-IHL-ik] acid derivatives would be most reactive with a nucleophilic [NU-klee-oh-file]?\n\nW) Amide [AM-ide]\n\nX) Acid chloride\n\nY) Ester\n\nZ) Acid anhydride [an-HIGH-dried]","Completion":"X) ACID CHLORIDE"} {"Type":"BONUS\n\nChemistry - Multiple Choice","Prompt":"Methylamine [meh-thil-ah-MEEN] reacts with 1-bromobutane [one-broh-mo-BYOO-tanel]by donating electrons into what orbital of the carbon at the one position of bromobutane?\n\nW) C-C sigma bonding orbital\n\nX) C-C sigma antibonding orbital\n\nY) C-Br sigma bonding orbital\n\nZ) C-Br sigma antibonding orbital","Completion":"Z) C-Br SIGMA ANTIBONDING ORBITAL"} {"Type":"TOSS-UP\n\nEarth and Space - Multiple Choice","Prompt":"What object may be formed by the collision of two galaxies, each containing a large black hole at the core?\n\nW) Quasar\n\nX) Pulsar\n\nY) Neutron star\n\nZ) Supercluster","Completion":"W) QUASAR"} {"Type":"BONUS\n\nEarth and Space - Short Answer","Prompt":"Place the following three Galilean satellites of Jupiter in increasing order of distance from Jupiter: Europa; Ganymede [GAN-ih-meed]; lo [EYE-oh].","Completion":"3, 1, 2"} {"Type":"TOSS-UP\n\nMath - Short Answer","Prompt":"What is the integer part of the log base 5 of 1000?","Completion":"4"} {"Type":"TOSS-UP\n\nEnergy - Short Answer","Prompt":"Due to environmental concerns with lead perovskites [purr-OFF-skites] as solar-cell materials, researchers at Ames Lab are studying what same-row, period 6 metal as an alternative thin-film material?","Completion":"BISMUTH (ACCEPT: BI)"} {"Type":"TOSS-UP\n\nPhysics - Short Answer","Prompt":"A concave mirror has a radius of curvature of 54 centimeters. In centimeters, what is its focal length?","Completion":"27"} {"Type":"TOSS-UP\n\nMath - Short Answer","Prompt":"The point \\((3,-\\) is reflected through the \\(x\\)-axis, with the result then translated 4 units down. What are the coordinates of the final image?","Completion":"(3, 1) (ACCEPT: \\(x=3\\) AND \\(y=1\\))"} {"Type":"TOSS-UP\n\nBiology - Short Answer","Prompt":"A 19th-century Danish scientist classified bacteria based on the ability of their cell walls to retain crystal violet, which is due to the abundance of what polymer?","Completion":"PEPTIDOGLYCAN [peptihdo-GLY-can]"} {"Type":"BONUS\n\nBiology - Short Answer","Prompt":"Parathyroid [para-THIGH-royd] hormone is necessary for human survival because it controls the blood concentration of what?","Completion":"CALCIUM (ACCEPT: Ca\\({}^{2+}\\))"} {"Type":"TOSS-UP\n\nChemistry - Short Answer","Prompt":"Identify all of the following three statements that are true of cyclohexane:\n\nIt is miscible with benzene; The bond angles in the ring are sixty degrees; It is more strained than cyclooctane.","Completion":"1"} {"Type":"BONUS\n\nChemistry - Multiple Choice","Prompt":"Lithium aluminum hydride would likely be used by a synthetic chemist for what type of reaction?\n\nW) Amide [AM-ide] bond formation\n\nX) Nucleophilic substitution\n\nY) Reduction\n\nZ) Aromatic substitution","Completion":"Y) REDUCTION"} {"Type":"TOSS-UP\n\nMath - Short Answer","Prompt":"A bag contains three red and three blue marbles. One marble is drawn, and then a second, without replacement. What is the probability of drawing one marble of each color?","Completion":"3\/5 (ACCEPT: 0.6, 60%)"} {"Type":"BONUS\n\nMath - Short Answer","Prompt":"Solve the following system of three equations in three unknowns:\n\n\\(x+y=4\\), \\(y+z=1\\), and \\(x+z=13\\)?","Completion":"\\(x=8\\); \\(y=-4\\); \\(z=5\\) (ACCEPT: (8, \\(-4\\), 5))"} {"Type":"TOSS-UP\n\nBiology \\(-\\) Multiple Choice","Prompt":"In humans, hyperfunctioning of the thyroid gland can result from which of the following conditions?\n\nW) Cushing's syndrome\n\nX) Addison's disease\n\nY) Cretinism [KREE-tin-izm]\n\nZ) Graves' disease","Completion":"Z) GRAVES' DISEASE"} {"Type":"BONUS\n\nBiology \\(-\\) Short Answer","Prompt":"Identify all of the following three statements that are true of lancelets:\n\nThey possess a post-anal tail; They possess pharyngeal [fair-in-JEE-ul] slits; Their musculature\n\nderives from mesoderm.","Completion":"ALL"} {"Type":"TOSS-UP\n\nEarth and Space \\(-\\) Multiple Choice","Prompt":"Widmanstatten [WTHD-mahn-STAH-ten] patterns are most commonly found in which of the following objects?\n\nW) Stony meteorites\n\nX) Iron meteorites\n\nY) Moon rocks\n\nZ) Carbonaceous [car-bon-AY-shus] chondrites [KON-drytes]","Completion":"X) IRON METEORITES"} {"Type":"BONUS\n\nEarth and Space \\(-\\) Multiple Choice","Prompt":"Lobate scarps on the surface of Mercury are a result of which of the following processes?\n\nW) Cooling\n\nX) Plate tectonics [tek-TAWN-iks]\n\nY) Volcanism\n\nZ) Land tides","Completion":"W) COOLING"} {"Type":"TOSS-UP\n\nPhysics - Short Answer","Prompt":"To the nearest whole number of joules, how much work is done by a 4-newton-meter torque applied over one full revolution to a 3-kilogram wheel with radius 7 meters?","Completion":"25"} {"Type":"TOSS-UP\n\nEnergy - Short Answer","Prompt":"Researchers at SLAC [slack] lab are operating and collecting data from the orbiting Fermi [FUR-mee] telescope, which captures light from what region of the electromagnetic spectrum that is useful for studying Type II supernovae [SUper-noh-vee] and pulsars?","Completion":"GAMMA RAYS"} {"Type":"BONUS\n\nEnergy - Multiple Choice","Prompt":"Scientists at Ames Lab have recently released a study of the various parameters of superconductors, including the superconducting transition temperature and the London Penetration Depth. The London Penetration Depth best measures the penetration of which of the following quantities?\n\nW) Current\n\nX) External magnetic field\n\nY) Alternating current\n\nZ) An external electric field","Completion":"X) EXTERNAL MAGNETIC FIELD"} {"Type":"TOSS-UP\n\nMath - Short Answer","Prompt":"What is 69 times 13?","Completion":"897"} {"Type":"TOSS-UP\n\nPhysics - Short Answer","Prompt":"An astronaut, who is 60 kilograms and initially at rest, is approached by a\n\ncrate of supplies that is moving at 4 meters per second. When the crate is close, the astronaut pushes against it, accelerating him to 2 meters per second and slowing the crate down to 2.5 meters per second. What is the mass, in kilograms, of the crate?","Completion":"80"} {"Type":"TOSS-UP\n\nEarth and Space \\(-\\) Multiple Choice","Prompt":"Tephrochronology [TEF-roh-krah-NAWL-ah-jee] deals with age-dating what type of material?\n\nW) Carbonate shells\n\nX) Dead organisms\n\nY) Volcanic ash\n\nZ) Ice-rafted debris","Completion":"Y) VOLCANIC ASH"} {"Type":"BONUS\n\nEarth and Space \\(-\\) Short Answer","Prompt":"Identify all of the following three statements that are true of fossil fuels: Good reservoir rocks have high permeability; Good cap rocks have low permeability; Natural gas tends to form at shallower depths than oil.","Completion":"1 AND 2"} {"Type":"TOSS-UP\n\nChemistry \\(-\\) Multiple Choice","Prompt":"Methanol has a more negative standard change in enthalpy [EN-thul-peel] of formation than methane. Which of the following is true regarding their relative chemical stabilities?\n\nW) Methanol is more stable than methane\n\nX) Methanol is less stable than methane\n\nY) Because both are composed of the same atoms, they are equally stable\n\nZ) Their stabilities cannot be compared with standard enthalpy [EN-thul-peel] changes","Completion":"Z) THEIR STABILITIES CANNOT BE COMPARED WITH STANDARD ENTHALPY CHANGES"} {"Type":"BONUS\n\nChemistry \\(-\\) Multiple Choice","Prompt":"Acetone cyanohydrin [sigh-ANN-oh-HIGH-drin] is formed when a cyanide ion reacts with acetone. What atom is the electrophile in this reaction?\n\nW) Nitrogen on the cyanide ion\n\nX) Carbon at the 2 position on acetone\n\nY) Carbon at the 1 position on acetone\n\nZ) Oxygen on acetone","Completion":"X) CARBON AT THE 2 POSITION ON ACETONE"} {"Type":"TOSS-UP\n\nEnergy - Multiple Choice","Prompt":"De novo phasing is a technique that structural biologists at SLAC [slack] use to gather information about a new protein's 3D shape. This technique involves the replacement of a single atom inside the protein with a traditionally non-proteinogenic [pro-teen-oh-JEN-ik] atom with similar chemical behavior. In one example, the researchers replaced a sulfur atom with what?\n\nW) Nitrogen\n\nX) Selenium\n\nY) Phosphorus\n\nZ) Arsenic","Completion":"X) SELENIUM"} {"Type":"BONUS\n\nEnergy - Multiple Choice","Prompt":"Los Alamos National Lab scientists are studying how microbes in the soil can slow down the spread of what long-lived radioisotope that is readily absorbed in the thyroid?\n\nW) Strontium-90 [STRAWNCH-ee-um-90]\n\nX) Iodine-131\n\nY) Cesium-137 [SEEZ-ee-um-one-thirty-seven]\n\nZ) Iodine-129","Completion":"Z) IODINE-129"} {"Type":"TOSS-UP\n\nBiology - Multiple Choice","Prompt":"Which of the following forms of nitrogenous [nye-TRAW-jen-us] waste excretion is most energetically costly to produce?\n\nW) Ammonia\n\nX) Urea\n\nY) Ammonium\n\nZ) Uric acid","Completion":"Z) URIC ACID"} {"Type":"BONUS\n\nBiology - Multiple Choice","Prompt":"During Western blotting, a blocking solution is often prepared. What is the primary function of the blocking solution?\n\nW) Conditioning of the PVDF membrane\n\nX) Limiting non-specific binding of antibodies\n\nY) Preserving antibody-protein interactions\n\nZ) Quenching protein denaturation","Completion":"X) LIMITING NON-SPECIFIC BINDING OF ANTIBODIES"} {"Type":"TOSS-UP\n\nEarth and Space - Multiple Choice","Prompt":"What class of meteorites is composed of undifferentiated, spherical particles that are believed to contain primitive material from which rocky planets formed?\n\nW) Achondrites [ay-KAWN-dries]\n\nX) Chondrites [KON-drite]\n\nY) Iron meteorites\n\nZ) Pallasites","Completion":"X) CHONDRITES"} {"Type":"TOSS-UP\n\nMath - Short Answer","Prompt":"If the graph of \\(y\\) equals open parenthesis\\(x+4\\) close parenthesis squared\\(+3\\) is translated 2 units to the right and 4 units down, what are the coordinates of the vertex of the image?","Completion":"(-2, -1) (ACCEPT: \\(x=-2\\) AND \\(y=-1\\))"} {"Type":"TOSS-UP\n\nBiology - Multiple Choice","Prompt":"What type of phagocytic cell can primarily be found under mucosal [myoo-KOH-sul] surfaces and plays a primary role in responding to multicellular pathogens?\n\nW) Neutrophphils [NEW-toth-fill]\n\nX) Eosinophilis [ee-oh-SIN-oh-filz]\n\nY) Dendritic cells [den-DRIH-dik]\n\nZ) Macrophages [MACRO-fay-jiz]","Completion":"X) EOSINOPHILS"} {"Type":"BONUS\n\nBiology - Short Answer","Prompt":"A scientist is separating proteins using size-exclusion chromatography [crow-mah-TAW-grah-fee]. Identify the relative order of elution from earliest to latest of the following proteins from the column: Titin; Myoglobin [MY-oh-glow-bin]; Hemoglobin; Cytochrome [SIGH-tow-krome] c.","Completion":"1, 3, 2, 4"} {"Type":"TOSS-UP\n\nChemistry - Multiple Choice","Prompt":"Which of the following functional groups can be found in sucrose?\n\nW) Alcohol\n\nX) Amine\n\nY) Aminal\n\nZ) Carboxyl","Completion":"W) ALCOHOL"} {"Type":"BONUS\n\nChemistry - Multiple Choice","Prompt":"Which of the following techniques would most unambiguously determine the structure of a compound that contains stereocenters?\n\nW) Mass spectrometry [spek-TRAW-metry]\n\nX) X-ray crystallography [crystal-AW-graphy]\n\nY) 1D NMR spectroscopy [spek-TRAW-scuh-pee]\n\nZ) Atomic absorption spectroscopy","Completion":"X) X-RAY CRYSTALLOGRAPHY"} {"Type":"TOSS-UP\n\nMath - Short Answer","Prompt":"What is the determinant of the 2-by-2 matrix with first row \\(-3,-5\\) and second row \\(8\\), \\(-2\\)?","Completion":"46"} {"Type":"TOSS-UP\n\nBiology - Short Answer","Prompt":"Fairy rings are associated with what division of fungi?","Completion":"BASIDIOMYCOTA (ACCEPT: BASIDIOMYCETES)"} {"Type":"TOSS-UP\n\nPhysics - Short Answer","Prompt":"A 9-volt battery with internal resistance of 1 ohm is connected to a lightbulb. If the potential drop across the lightbulb is 6 volts, then in watts, how much power is dissipated by the lightbulb?","Completion":"18"} {"Type":"BONUS\n\nPhysics - Short Answer","Prompt":"Water flows through a horizontally-level pipe at 4 meters per second, until it encounters a section where its static pressure decreases by 10 kilopascals. To the nearest whole number of meters per second, what is its new flow speed?","Completion":"6"} {"Type":"TOSS-UP\n\nEarth and Space - Short Answer","Prompt":"Identify all of the following three rocks that would be classified as\n\naphanitic [AFF-in-it-ik]: Andesite [ANN-deh-sitel]; Basalt [bah-SALT]; Diorite.","Completion":"1 AND 2"} {"Type":"BONUS\n\nEarth and Space - Short Answer","Prompt":"Place the following three rock types in increasing order of distance\n\nfrom shore at the time of deposition in a typical sedimentary sequence: Shale; Limestone;\n\nSandstone.","Completion":"3, 1, 2"} {"Type":"TOSS-UP\n\nChemistry - Short Answer","Prompt":"Aspartame is an artificial sweetener composed of amino acids linked by what type of chemical bond?","Completion":"AMIDE (ACCEPT: PEPTIDE)"} {"Type":"BONUS\n\nChemistry - Short Answer","Prompt":"Identify all of the following three chemical moieties that are capable of having cis [sis] -trans isomerism [eye-SAW-mer-izm]: Tetrahedral [letrah-HEE-drul] complex; Alkene; Square planar [PLAY-nar] complex.","Completion":"2 AND 3"} {"Type":"TOSS-UP\n\nEarth and Space \\(-\\) Multiple Choice","Prompt":"Which of the following ocean currents does NOT flow from higher latitudes toward the equator?\n\nW) The California Current\n\nX) The Peru Current\n\nY) The Brazil Current\n\nZ) The Benguela [ben-GWAY-la] Current","Completion":"Y) THE BRAZIL CURRENT"} {"Type":"BONUS\n\nEarth and Space \\(-\\) Multiple Choice","Prompt":"Which of the following events occurred in the Late Mississippi?\n\nW) Breakup of Rodinia\n\nX) Collision of Gondwana with Laurasia\n\nY) Collision of Laurentia with Baltic\n\nZ) Breakup of Pangaea","Completion":"X) COLLISION OF GONDWANA WITH LAURASIA"} {"Type":"TOSS-UP\n\nMath \\(-\\) Short Answer","Prompt":"What are the \\(x\\)-coordinates of the intersections of the graphs of the equations \\(y=9x^{2}+3\\) and \\(y=-27x+3\\)?","Completion":"0 AND -3"} {"Type":"BONUS\n\nMath \\(-\\) Short Answer","Prompt":"What is the cube of the complex number \\(3+5i\\)?","Completion":"\\(-198+10i\\)"} {"Type":"TOSS-UP\n\nChemistry - Multiple Choice","Prompt":"A student wishes to convert an aldehyde to a secondary alcohol. Which of the following reagents would be most useful?\n\nW) Alkyl [AL-keel] Grignard [GRIN-yard] reagent\n\nX) Lithium aluminum hydride\n\nY) Sodium borohydride\n\nZ) Sodium hydride","Completion":"W) ALKYL GRIGNARD REAGENT"} {"Type":"BONUS\n\nChemistry - Short Answer","Prompt":"Andrew synthesizes a compound that consists of a benzene ring with an alcohol group in the one position and a primary amine in the four position. Identify all of the following three names that unambiguously represent the structure of this compound: P-aminophenol;\n\n4-aminophenol; 4-hydroxyaniline.","Completion":"ALL"} {"Type":"TOSS-UP\n\nPhysics - Multiple Choice","Prompt":"Planck resolved the ultraviolet catastrophe for black bodies by postulating the quantization of photon energies to derive an expression showing that the modal energies obey which of the following statistical distributions?\n\nW) Fermi-Dirac\n\nX) Maxwell-Boltzmann\n\nY) Bose-Einstein\n\nZ) Gaussian","Completion":"Y) BOSE-EINSTEIN"} {"Type":"BONUS\n\nPhysics - Short Answer","Prompt":"A spinning dancer draws her arms inwards until she rotates at 3 times her original velocity. In terms of her original kinetic energy \\(E\\), how much energy did this maneuver require?","Completion":"\\(2E\\)"} {"Type":"TOSS-UP\n\nBiology - Short Answer","Prompt":"Identify all of the following four phyla [FYE-lah] that exhibit radial symmetry: Porifera [poor-IH-fer-ah]; Cnidaria [nye-DARE-ee-ah]; Mollusca [null-US-kah]; Echinodermata [eh-kye-noh-der-MAH-tah].","Completion":"2, 4"} {"Type":"BONUS\n\nBiology - Short Answer","Prompt":"In humans, circadian [sir-KAY-dee-an] rhythms are controlled by a specific group of neurons in the hypothalamus [high-poh-THAL-ah-mus] called what?","Completion":"SCN (ACCEPT: SUPRACTIASMATIC NUCLEUS)"} {"Type":"TOSS-UP\n\nEnergy - Multiple Choice","Prompt":"Scientists at Lawrence Berkeley National Lab recently captured images of a Type one-A supernova in a galaxy four billion light-years away that appeared 50 times brighter than normal and in four different places in the sky. Which of the following did the scientists discover?\n\nW) A Type-one-A supernova with a progenitor that was 20-50 times more massive than normal\n\nX) An anomalous type-two supernova with a light curve resembling type Ia\n\nY) A light echo of a type-one-A supernova\n\nZ) A type-one-A supernova whose image was strongly gravitationally lensed","Completion":"Z) A TYPE-ONE-A SUPERNOVA WHOSE IMAGE WAS STRONGLY GRAVITIGNALLY LENSED"} {"Type":"BONUS\n\nEnergy - Multiple Choice","Prompt":"One of the most promising generation IV nuclear reactor designs involves the use of supercritical water as the coolant. What is the main reason to use supercritical water?\n\nW) It has a higher neutron absorption cross section\n\nX) It has a lower neutron absorption cross section\n\nY) It is fissile\n\nZ) It allows for a higher thermal efficiency","Completion":"Z) IT ALLOWS FOR A HIGHER THERMAL EFFICIENCY"} {"Type":"TOSS-UP\n\nEarth and Space \\(-\\) Multiple Choice","Prompt":"Which of the following drainage patterns would likely be visible on jointed, rocky terrain?\n\nW) Dendritic [den-DRIH-dik]\n\nX) Trellis\n\nY) Radial\n\nZ) Rectangular","Completion":"Z) RECTANGULAR"} {"Type":"TOSS-UP\n\nBiology \\(-\\) Multiple Choice","Prompt":"A patient is demonstrating loss in bone density. Which of the following hormones would be most useful in treating this symptom?\n\nW) Parathyroid hormone\n\nX) Vasopressin\n\nY) Calcitonin [kal-sinh-TOE-nin]\n\nZ) Mineralocorticoid [mineralo-CORE-deh-KOYD]","Completion":"Y) CALCITONIN"} {"Type":"TOSS-UP\n\nEnergy - Multiple Choice","Prompt":"DOE scientists are using the Advanced Light Source to study atmospheric reentry heat shields, including PICA [IPEE-kahl], the type currently used by NASA and SpaceX. Which of the following best describes PICA?\n\nW) A silicone-based heating pad\n\nX) A highly-thermally-conductive epoxy\n\nY) A carbon fiber that slowly ablates\n\nZ) Titanium shingles with heat sinks","Completion":"Y) A CARBON FIBER THAT SLOWLY ABLATES"} {"Type":"TOSS-UP\n\nMath - Short Answer","Prompt":"What is the product of the three different prime factors of 315?","Completion":"105"} {"Type":"TOSS-UP\n\nPhysics - Short Answer","Prompt":"In terms of the elementary charge \\(e\\), what is the electric charge of an up quark?","Completion":"POSITIVE (2\/3)\\(e\\) (ACCEPT: (2\/3)\\(e\\), 2\\(e\\)\/3)"} {"Type":"BONUS\n\nPhysics - Short Answer","Prompt":"An excess charge is distributed over the surface of a solid metal sphere that has a radius of one meter. Identify all of the following three distances from the center of the sphere at which the electrical potential relative to infinity is maximized: 50 centimeters; 100 centimeters; 150 centimeters.","Completion":"1 AND 2"} {"Type":"TOSS-UP\n\nChemistry - Multiple Choice","Prompt":"The precipitation reaction caused by the addition of potassium chloride to aqueous [AY-kwee-us] silver nitrate is an example of which of the following types of reactions?\n\nW) Metathesis [meh-TATH-uh-sis] X) Decomposition Y) Synthesis Z) Redox","Completion":"W) METATHESIS"} {"Type":"BONUS\n\nChemistry - Multiple Choice","Prompt":"Copper (II) hydroxide is insoluble in water. However, upon the addition of excess ammonia, it dissolves and changes to a deep blue color. Which of the following statements best explains this observation?\n\nW) The rise in pH solubilized the copper hydroxide\n\nX) The drop in pH solubilized the copper hydroxide\n\nY) The copper reacted with ammonia, forming a soluble species\n\nZ) The ammonia neutralized the hydroxide, releasing soluble copper (II)","Completion":"Y) THE COPPER REACTED WITH AMMONIA, FORMING A SOLUBLE SPECIES13) Biology - Short Answer The five-prime cap added to mRNA is a modified version of what nucleotide base?"} {"Type":"TOSS-UP\n\nEnergy - Multiple Choice","Prompt":"Lithium-ion batteries are generally built in the discharged state, then recharged after construction. Which of the following best explains why this is done?\n\nW) The materials in the discharged state can be exposed to the atmosphere safely\n\nX) In the charged state, the raw materials pose a risk of dendrite formation\n\nY) Lithium dendrites cannot form in the discharged state\n\nZ) Lithiated manganese materials are particularly unstable in the presence of humidity","Completion":"W) THE MATERIALS IN THE DISCHARGED STATE CAN BE EXPOSED TO THE ATMOSPHERE SAFELY"} {"Type":"TOSS-UP\n\nChemistry - Multiple Choice","Prompt":"Which of the following substances is more soluble in toluene than in\n\nwater?\n\nW) Carbon tetrachloride\n\nX) Sodium chloride\n\nY) Acetic acid\n\nZ) Sodium methoxide","Completion":"W) CARBON TETRACHLORIDE"} {"Type":"BONUS\n\nPhysics - Short Answer","Prompt":"Identify all of the following three quantities that are dimensionless:\n\nTensile [TEN-sile] stress; Tensile strain; Young's Modulus.","Completion":"2"} {"Type":"TOSS-UP\n\nMath - Short Answer","Prompt":"How many terms does the binomial expansion of open parenthesis \\(8x^{3}+5y^{2}\\) close parenthesis to \\(18^{\\rm th}\\) power contain?","Completion":"19"} {"Type":"BONUS\n\nMath - Short Answer","Prompt":"What are the coordinates of the image of the point (-1, when it is rotated about the point (3, by 90 degrees counterclockwise?","Completion":"(1, -2) (ACCEPT: \\(x=1\\) AND \\(y=-2\\))"} {"Type":"TOSS-UP\n\nBiology - Short Answer","Prompt":"What is the name of the jelly-like fluid that fills the human eye?","Completion":"VITREOUS HUMOR (ACCEPT: VITREOUS BODY)"} {"Type":"BONUS\n\nBiology - Short Answer","Prompt":"Identify all of the following three vitamins that are considered fat-soluble:\n\nBiotin; Cobalamin [koh-BALL-ah-min]; Niacin.","Completion":"NONE"} {"Type":"TOSS-UP\n\nChemistry - Short Answer","Prompt":"Identify all of the following three statements that are true regarding the\n\ndentity of ligands [LIH-gundz]: Bromine [BROH-meen] is a monodentate ligand;\n\nDiethylenetriamine [dye-ethileen-tri-ah-meen] is a bidentate ligand; EDTA is a quatradentate [kwah-trah-DEN-tate] ligand.","Completion":"1"} {"Type":"BONUS\n\nChemistry - Short Answer","Prompt":"Identify all of the following three molecules that have the chemical\n\nformula C\\({}{4}\\)H\\({}{8}\\)O: Acetone; Butanone; Butyraldehyde.","Completion":"2 AND 3"} {"Type":"TOSS-UP\n\nEarth and Space \\(-\\) Short Answer","Prompt":"What spectral class of stars possesses the strongest Balmer lines?","Completion":"A (ACCEPT: A-ZERO)"} {"Type":"TOSS-UP\n\nChemistry - Multiple Choice","Prompt":"A student is holding two balloons, one containing one mole of hydrogen gas and the other containing one mole of helium gas. What is the ratio of the volume of the helium balloon to that of the hydrogen balloon?\n\nW) 1\/2\n\nX) 1\n\nY) 2\n\nZ) 4","Completion":"X) 1"} {"Type":"BONUS\n\nChemistry - Short Answer","Prompt":"Rank the following three elements in increasing order of atomic radius:\n\nCalcium; Chlorine; Magnesium.","Completion":"2, 3, 1"} {"Type":"TOSS-UP\n\nPhysics - Short Answer","Prompt":"The radioactive decay of magnesium-23 to sodium-23 is mediated by which fundamental force?","Completion":"WEAK FORCE (ACCEPT: WEAK NUCLEAR FORCE)"} {"Type":"BONUS\n\nPhysics - Short Answer","Prompt":"A block rests on an inclined plane angled at 30 degrees from the horizontal.\n\nWhat is the ratio of the normal force to the force down the surface of the plane?","Completion":"\\(\\forall\\)3 (ACCEPT: \\(\\forall\\)3 TO 1)"} {"Type":"TOSS-UP\n\nEnergy - Short Answer","Prompt":"Researchers at Lawrence Berkeley National Lab are studying a tunable, hypersensitive form of MRI that could present a breakthrough in cancer diagnostics. Typically, MRI machines use radio frequency pulses to measure the distribution of what element?","Completion":"HYDROGEN"} {"Type":"BONUS\n\nEnergy - Short Answer","Prompt":"Identify all of the following three fuels that could be used in a fuel cell:\n\nMethane; Hydrogen; Carbon monoxide.","Completion":"ALL"} {"Type":"TOSS-UP\n\nBiology - Short Answer","Prompt":"Glucagon [GLUE-kah-gon] is primarily manufactured in the alpha cells of what human organ?","Completion":"PANCREAS"} {"Type":"BONUS\n\nBiology - Short Answer","Prompt":"A molecular biologist wishes to move a section of DNA from one plasmid to another. After cutting the first plasmid with restriction enzymes and isolating the band of DNA of interest, she mixes together the second, already-cleaved plasmid and the DNA of interest. What enzyme must she add to finish incorporating the gene into the plasmid?","Completion":"LIGASE (ACCEPT: DNA LIGASE, RECOMBINASE)"} {"Type":"TOSS-UP\n\nMath - Short Answer","Prompt":"In the expansion of the cube of \\(a+2b\\), what is the coefficient of the \\(ab^{2}\\) term?","Completion":"12"} {"Type":"TOSS-UP\n\nEarth and Space - Short Answer","Prompt":"What technique is employed by astronomers to study the ways in which matter interacts with or emits electromagnetic radiation?","Completion":"SPECTROSCOPY"} {"Type":"TOSS-UP\n\nEnergy - Short Answer","Prompt":"The Deep Underground Neutrino [new-TREE-no] Experiment is an effort at Fermilab to send neutrinos between two points separated by a large distance to give them time to undergo what unique behavior of neutrinos?","Completion":"OSCILLATION"} {"Type":"BONUS\n\nEnergy - Multiple Choice","Prompt":"Which of the following is the most important difference between fast nuclear reactors and thermal nuclear reactors?\n\nW) They use different fuel configurations\n\nX) Unlike thermal reactors, fast reactors do not use a moderator\n\nY) Thermal reactors have a higher neutron flux than fast reactors\n\nZ) Thermal reactors run at higher temperatures than fast reactors","Completion":"X) UNLIKE THERMAL REACTORS, FAST REACTORS DO NOT USE A MODERATOR"} {"Type":"TOSS-UP\n\nMath - Short Answer","Prompt":"What is the arc length of a sector of a circle with radius 2 and central angle pi over three?","Completion":"2\\(\\pi\\)\/3 (ACCEPT: (2\/3)\\(\\pi\\))"} {"Type":"BONUS\n\nMath - Short Answer","Prompt":"A student has a paper cup in the shape of a right circular cone, with a height of 5 inches and an opening diameter of 3 inches. How many of these paper cups of water will it take the student to fill a cylindrical container of diameter 3 inches and height 10 inches?","Completion":"6"} {"Type":"TOSS-UP\n\nEarth and Space \\(-\\) Short Answer","Prompt":"What is the term for the point of origin within the Earth of seismic waves in an earthquake?","Completion":"FOCUS (ACCEPT: HYPOCENTER)"} {"Type":"BONUS\n\nEarth and Space \\(-\\) Short Answer","Prompt":"What name is given to the boundary between the Earth's crust and mantle where P-wave velocity jumps from approximately 6 kilometers per second to approximately 8 kilometers per second?","Completion":"MOHO (ACCEPT: MOHORVICIC DISCONTINUITY)"} {"Type":"TOSS-UP\n\nBiology \\(-\\) Multiple Choice","Prompt":"Beadle and Tatum's one gene-one enzyme hypothesis was formed from the results of their investigation of which of the following biological processes?\n\nW) Operon inhibition\n\nX) Amino acid biosynthesis\n\nY) DNA replication\n\nZ) Limited proteolysis [pro-tee-AW-lih-sis]","Completion":"X) AMINO ACID BIOSYNTHESIS"} {"Type":"BONUS\n\nBiology \\(-\\) Short Answer","Prompt":"Identify all of the following three plant nutrients that are required for the light reactions of photosynthesis to occur: Iron; Nickel; Molybdenum [muh-LIB-den-um].","Completion":"1"} {"Type":"TOSS-UP\n\nMath - Short Answer","Prompt":"If \\(f\\) of \\(x\\) is equal to \\(5x^{2}-6x+2\\), what is \\(f\\) of 3?","Completion":"29"} {"Type":"BONUS\n\nMath - Short Answer","Prompt":"What is the limit as \\(x\\) approaches 0 of the fraction with numerator\\(x+\\mbox{sine}\\ x\\) and denominator\\(e\\) to the \\(x\\) power minus 1?","Completion":"2"} {"Type":"TOSS-UP\n\nPhysics - Short Answer","Prompt":"What physical quantity is defined as the rate of change of a particle's energy with respect to time?","Completion":"POWER"} {"Type":"BONUS\n\nPhysics - Short Answer","Prompt":"A linear spring is hung from the ceiling. When a 1-kilogram block is hung from the spring, the spring's length increases by 2 meters. If another 1-kilogram block is hung from 8 similar springs connected in parallel, then, in meters, what will be the change in length of each of the springs?","Completion":"0.25 (ACCEPT: \\(\\forall\\)a)"} {"Type":"TOSS-UP\n\nEnergy \\(-\\) Short Answer","Prompt":"The first useful nuclear-generated electricity was produced in Idaho in 1951. The nuclear reactor was designed and built by what DOE national laboratory?","Completion":"ARGONNE NATIONAL LAB (ACCEPT: ARGONNE)"} {"Type":"BONUS\n\nEnergy \\(-\\) Short Answer","Prompt":"The electrodes in hydrogen fuel cells are generally composed of what catalytically-active element?","Completion":"PLATINUM"} {"Type":"TOSS-UP\n\nEarth and Space \\(-\\) Multiple Choice","Prompt":"Which of the following minerals would be found in least abundance in a typical gabbro [gab-roh]?\n\nW) Muscovite\n\nX) Plagioclase [PLAY-jee-oh-klaze]\n\nY) Augite [AW-jite]\n\nZ) Hornblende","Completion":"W) MUSCOVITE"} {"Type":"BONUS\n\nEarth and Space \\(-\\) Short Answer","Prompt":"Identify all of the following three features that are associated with karst landscapes. Disappearing streams; Stalagmites [stah-LAG-mites]; Sinkholes.","Completion":"ALL17) Chemistry - Short Answer What is the hybridization [HIGH-brih-dye-ZAY-shun] of the carbon atoms in polyethylene?"} {"Type":"TOSS-UP\n\nBiology - Short Answer","Prompt":"Sand dollars belong to what phylum [FYE-lum]?","Completion":"ECHINODERMATA"} {"Type":"TOSS-UP\n\nMath - Short Answer","Prompt":"What is the slope of the line that passes through the points (6, and (2, ?","Completion":"1\/4 (ACCEPT: 0.25)"} {"Type":"TOSS-UP\n\nPhysics - Multiple Choice","Prompt":"Which of the following length scales best describes the spatial distortion resolution of the Laser Interferometer Gravitational-wave Observatory?\n\nW) Length of the interferometer arm\n\nX) Wavelength of the helium-neon laser\n\nY) Radius of a silver atom\n\nZ) Thousandth of a proton diameter","Completion":"Z) Thousandth of a proton diameter"} {"Type":"TOSS-UP\n\nEarth and Space \\(-\\) Short Answer","Prompt":"What is the brightest star in the constellation Orion?","Completion":"RIGEL"} {"Type":"TOSS-UP\n\nBiology \\(-\\) Short Answer","Prompt":"In order for tropomyosin [troh-poh-MY-oh-sin] to be removed from the myosin-binding sites on actin, troponin [TROH-poh-min] must bind to what second messenger?","Completion":"CALCIUM (ACCEPT: CALCIUM TWO PLUS)"} {"Type":"TOSS-UP\n\nChemistry - Short Answer","Prompt":"According to VSEPR theory, what is the electron-pair geometry of the central atom in the compound iodine trichloride?","Completion":"TRIGONAL BIPYRAMIDAL"} {"Type":"TOSS-UP\n\nEnergy - Multiple Choice","Prompt":"Two years after carbon dioxide was sequestered by the Pacific Northwest National Lab near Wallula, Washington, the material was converted into mineral substances resembling what type of sedimentary rock?\n\nW) Rock gypsum\n\nX) Limestone\n\nY) Coal\n\nZ) Rock salt","Completion":"X) LIMESTONE"} {"Type":"TOSS-UP\n\nEarth and Space - Short Answer","Prompt":"What star has the lowest apparent magnitude as observed from Earth?","Completion":"THE SUN (ACCEPT: SOL)"} {"Type":"TOSS-UP\n\nBiology - Short Answer","Prompt":"Before amino acids can be used in translation, the free amino acid must be bonded to what class of molecules?","Completion":"tRNA"} {"Type":"TOSS-UP\n\nChemistry - Multiple Choice","Prompt":"Which of the following terms best describes the relationship between 1-butanol [one-BYOO-tan-all] and 4-butanol?\n\nW) Structural isomers\n\nX) Same compound\n\nY) Diastereomers [dye-ah-STARE-ee-oh-murz]\n\nZ) Enantiomers [en-AN-tee-oh-mirs]","Completion":"X) SAME COMPOUND"} {"Type":"TOSS-UP\n\nPhysics - Short Answer","Prompt":"A 1,000-kilogram block of density 800 kilograms per cubic meter is placed in a fluid of density 1,000 kilograms per cubic meter. What percentage of the block's total volume floats above the surface of the fluid?","Completion":"20"} {"Type":"BONUS\n\nPhysics - Multiple Choice","Prompt":"A lightning bolt strikes a power station, transmitting a current of 30,000 amps for 100 microseconds into it, which is currently charging a 2,000-milliampere-hour battery. Assuming that the charge from the lightning bolt is perfectly transmitted into the battery, which of the following percentages is closest to the percentage of the battery that it would charge?\n\nW) Less than 1\n\nX) 25\n\nY) 50\n\nZ) 75","Completion":"W) LESS THAN 1"} {"Type":"TOSS-UP\n\nMath - Short Answer","Prompt":"What value of \\(x\\) minimizes the function \\(f\\) of \\(x\\) equals \\(x^{2}-6x+2\\)?","Completion":"3"} {"Type":"BONUS\n\nMath - Multiple Choice If \\(f(x)=\\) the square root of open parenthesis \\(x+4\\) close parenthesis and \\(g(x)=\\) the square root of open parenthesis \\(x-4\\) close parenthesis, what is the domain of the function \\(f(x)\\) divided by \\(g(x)\\)?\n\nW) All real numbers greater than 4\n\nX) All real numbers greater than or equal to 4\n\nY) All real numbers greater than or equal to \\(-4\\)\n\nZ) All real numbers between \\(-4\\) and 4\n\nAnswer: W) All real numbers greater than Biology - Short Answer","Prompt":"Under limited nutrient conditions Bacillus [bah-SIL-us] subtilis [SUB-dil-iss] initiates a morphological change, notably documented by Robert Koch [coke], that results in the formation of what structure?","Completion":"ENDOSPORE (ACCEPT: SPORE)"} {"Type":"TOSS-UP\n\nPhysics - Short Answer","Prompt":"The grand unified theory joins together all of the fundamental forces except which one?","Completion":"GRAVITY"} {"Type":"TOSS-UP\n\nEarth and Space - Short Answer","Prompt":"To what geologic era would you travel if you wanted to observe living dinosaurs?","Completion":"MESOZOIC"} {"Type":"TOSS-UP\n\nMath - Short Answer","Prompt":"What is 27 raised to the 4\/3 power?","Completion":"81"} {"Type":"TOSS-UP\n\nEnergy - Short Answer","Prompt":"SLAC [slack] researchers recently published a report on the development of a new, cheap catalyst for the oxygen-evolving reaction relevant to fuel production. On what molecule do these catalysts act to evolve oxygen?","Completion":"WATER (ACCEPT: H\\({}{2}\\)O)"} {"Type":"TOSS-UP\n\nChemistry - Multiple Choice","Prompt":"When performing bomb calorimetry [cal-or-IH-metree], which of the following physical quantities is being held constant?\n\nW) Pressure\n\nX) Volume\n\nY) Temperature\n\nZ) Entropy [EN-trou-pee]","Completion":"X) VOLUME"} {"Type":"BONUS\n\nChemistry - Short Answer","Prompt":"Rank the following three molecules in terms of increasing dipole [DYE-pole] moment: O-dihydroxybenzene [dye-high-droxe-BEN-zeen]; M-dihydroxybenez;\n\nP-dihydroxybenzene.","Completion":"3, 2, 1"} {"Type":"TOSS-UP\n\nMath - Multiple Choice","Prompt":"Which of the following is true about the function \\(f\\) of \\(x\\) = cosine of x?\n\nW) It is neither even nor odd\n\nX) It has the horizontal asymptote \\(y\\) = 1\n\nY) It has period 2 pi\n\nZ) It is not differentiable at \\(x\\) = 0","Completion":"Y) IT HAS PERIOD 2 PI"} {"Type":"BONUS\n\nMath - Short Answer","Prompt":"Simplify the following expression involving the complex number i:\n\n\\(4i+3i^{2}+2i^{3}+i^{4}\\)","Completion":"-2 + 2\\(i\\) (DO NOT ACCEPT: 2\\(i\\) - 2)15) Energy - Short Answer SLAC [slack] scientists recently developed a new technique for probing the electronic structure of solid materials via high harmonic generation, or HHG. In this technique, light is produced via the oscillations of what subatomic particle?"} {"Type":"TOSS-UP\n\nEarth and Space - Multiple Choice","Prompt":"Which of the following is the term for the set of planets that will exhibit a full range of phases as observed from Earth?\n\nW) Terrestrial\n\nX) Jovian\n\nY) Inferior\n\nZ) Superior","Completion":"Y) INFERIOR"} {"Type":"TOSS-UP\n\nBiology - Short Answer","Prompt":"Identify all of the following three Linnaean [lin-AY-in] classifications that apply to the leopard: Carnivora [car-NIV-or-ah]; Chordata [kor-DAH-tah]; Canidae [KAN-ih-dee].","Completion":"1 AND 2"} {"Type":"TOSS-UP\n\nBiology - Short Answer","Prompt":"In the citric acid cycle, how many molecules of NADH are generated from the conversion of citrate to isocitrate?","Completion":"ZERO"} {"Type":"TOSS-UP\n\nPhysics - Short Answer","Prompt":"The function describing an object's position is transformed into a function describing velocity by what operation from calculus?","Completion":"DERIVATIVE (ACCEPT: DIFFERENTIATION, DERIVATIVE OPERATION)"} {"Type":"BONUS\n\nPhysics - Short Answer","Prompt":"Two 6-kilogram masses are connected by a spring with a force constant of 300 newtons per meter. If the spring is compressed by five centimeters, what is the initial acceleration, in meters per second squared, of one of the masses?","Completion":"2.5"} {"Type":"TOSS-UP\n\nChemistry - Short Answer","Prompt":"Identify all of the following three chemicals whose addition will increase the percent ionization [eye-on-ih-ZAY-shun] of acetic acid in a solution: Water; Concentrated acetic acid; Concentrated hydrochloric acid.","Completion":"1"} {"Type":"BONUS\n\nChemistry - Short Answer","Prompt":"Identify all of the following three reactions that would favor reactants over products: A reaction with K\\({}{\\rm eq}\\) of 3; A reaction with positive delta G; An endothermic reaction with negative delta S.","Completion":"2 AND 3"} {"Type":"TOSS-UP\n\nEarth and Space - Short Answer","Prompt":"What type of well has water coming out of the ground freely because the pressure surface is above ground level?","Completion":"FLOWING ARTESIAN (DO NOT ACCEPT: ARTESIAN)"} {"Type":"TOSS-UP\n\nMath - Short Answer","Prompt":"An artist creates the background for a painting by placing a circle in a square frame so that their centers coincide. If the radius of the circle is 7 inches, and the minimum distance from the circle to the square is 5 inches, what is the perimeter, in inches, of the inside of the frame?","Completion":"96"} {"Type":"TOSS-UP\n\nChemistry - Short Answer","Prompt":"How many electrons does the beryllium [beh-RILL-ee-um] two plus cation [CAT-eye-on] have in its valence [VAY-lence] shell?","Completion":"TWO"} {"Type":"TOSS-UP\n\nEnergy - Short Answer","Prompt":"Fermilab's [FUR-mee-labs] MicroBooNE collaboration is currently operating a 170-ton Liquid Argon Time Projection Chamber that is used to detect what class of leptons?","Completion":"NEUTRINO"} {"Type":"BONUS\n\nEnergy - Short Answer","Prompt":"Scientists at Los Alamos National Lab are currently using computational models to study the astrophysical processes that produced the lanthanide and actinide elements. Identify all of the following three situations that produce rare-earth elements: Big-bang nucleosynthesis; The core of a four solar-mass main-sequence star; The collision of two neutron stars.","Completion":"3"} {"Type":"TOSS-UP\n\nBiology - Multiple Choice","Prompt":"Bacteria at deep sea vents obtain energy by oxidizing hydrogen sulfide. They would be classified as which of the following?\n\nW) Photoautotrophic [photo-auto-TROH-fik]\n\nX) Chemoautotrophic [kee-moh-auto-TROH-fik]\n\nY) Photoheterotrophic [photo-hetero-TROW-fik]\n\nZ) Chemoheterotrophic [kee-moh-hetero-TROH-fik]","Completion":"X) CHEMOAUTTORPHIC"} {"Type":"BONUS\n\nBiology - Short Answer","Prompt":"Human vision occurs due to the photoisomerization [photo-eye-saw-mer-ih-ZAY-shun] of what molecule?","Completion":"RETINAL (ACCEPT: RETINALDEHYDE, DO NOT ACCEPT: RETINOL - if you can tell the difference)7) Chemistry - Short Answer How many aluminum atoms are in one formula unit of the compound aluminum sulfate?"} {"Type":"TOSS-UP\n\nBiology - Multiple Choice","Prompt":"What is the function of the operculum [oh-PURR-kyoo-lum] in fish?\n\nW) Propulsion\n\nX) Protection of gills\n\nY) Sensing predators\n\nZ) Decreasing drag when swimming","Completion":"X) PROTECTION OF GILLS"} {"Type":"TOSS-UP\n\nEnergy - Short Answer","Prompt":"Lead-acid batteries are commonly found in cars. In these batteries, what substance serves as the electrolyte?","Completion":"SULFURIC ACID (ACCEPT: H\\({}{2}\\)SO\\({}{4}\\))"} {"Type":"BONUS\n\nEarth and Space \\(-\\) Short Answer","Prompt":"What is the term for a spherically-shaped, dense group of up to a million stars that are thought to share a common origin?","Completion":"GLOBULAR CLUSTER"} {"Type":"TOSS-UP\n\nPhysics \\(-\\) Short Answer","Prompt":"What scientist is famous for discovering X-rays?","Completion":"WILHELM ROENTGEN (ACCEPT: ROENTGEN)"} {"Type":"BONUS\n\nPhysics \\(-\\) Short Answer","Prompt":"A meteor strikes the Earth and ejects a rock to a final speed of 8 kilometers per second, accelerating it over the course of 5 meters. Assuming the acceleration is constant and in kilometers per second squared, what is the magnitude of the acceleration upon the rock?","Completion":"6,400"} {"Type":"TOSS-UP\n\nBiology - Short Answer","Prompt":"What animal phylum [FYE-lumi] has the greatest number of identified species?","Completion":"ARTHROPODA"} {"Type":"BONUS\n\nBiology - Multiple Choice","Prompt":"Which of the following is not a characteristic of paedomorphic [ped-oh-MOOR-fik] amphibians?\n\nW) Delayed sexual maturity\n\nX) Aquatic life\n\nY) External gills\n\nZ) Tadpole-like tail","Completion":"W) DELAYED SEXUAL MATURITY"} {"Type":"TOSS-UP\n\nEarth and Space - Short Answer","Prompt":"What is the term for the steady downhill flow of soil?","Completion":"CREEP"} {"Type":"BONUS\n\nEarth and Space - Short Answer","Prompt":"What is the name for the area of low pressure encircling Earth near the equator where the northeast and southeast trade winds come together?","Completion":"INERTROPICAL CONVERGENCE ZONE (ACCEPT ITCZ)15) Energy - Short Answer Researchers at the Advanced Photon Source have been studying a class of high-temperature superconductors based on a barium-arsenic crystal structure. These materials can demonstrate certain hallmarks of superconductivity when the material is ferromagnetic. What is the term for the transition temperature at which these materials will become paramagnetic?"} {"Type":"TOSS-UP\n\nMath - Short Answer","Prompt":"What is the second derivative with respect to \\(x\\) of \\(6x^{2}-15\\) evaluated at \\(x=7\\)?","Completion":"12"} {"Type":"TOSS-UP\n\nPhysics - Multiple Choice","Prompt":"Which of the following is an example of an action that demonstrates negative work?\n\nW) Lifting a book\n\nX) Holding a book\n\nY) Throwing a book to the ground\n\nZ) Gently lowering a book to the ground","Completion":"Z) GENTLY LOWERING A BOOK TO THE GROUND"} {"Type":"TOSS-UP\n\nEarth and Space - Short Answer","Prompt":"How many known objects in our solar system have a radius that is larger than that of Earth?","Completion":"FIVE"} {"Type":"TOSS-UP\n\nChemistry - Short Answer","Prompt":"In the Clemmensen reduction, zinc amalgam is used to reduce ketones [KEE-tones] and aldehydes to alkanes. In the preparation of zinc amalgam, in what substance is zinc dissolved?","Completion":"MERCURY"} {"Type":"BONUS\n\nChemistry - Short Answer","Prompt":"Identify all of the following three statements that are true regarding enthalpies of reaction: For the formation of water from oxygen and hydrogen, the enthalpy [EN-thulpeel] change depends on the final phase of the water; For the combustion of methane, the forward reaction and the reverse reaction have the same enthalpy; Enthalpy is an intensive property.","Completion":"1"} {"Type":"TOSS-UP\n\nPhysics - Multiple Choice","Prompt":"Hubble's law states that there is a direct correlation between the distance to a galaxy and which of the following quantities:\n\nW) Recessional velocity\n\nX) Total mass\n\nY) Blue-shift wavelength\n\nZ) Precession period","Completion":"W) RECESSIONAL VELOCITY"} {"Type":"BONUS\n\nPhysics - Short Answer","Prompt":"A track star runs exactly one lap around a circular track at a constant speed. Identify all of the following three statements that are true of this system: Her average speed is greater than the magnitude of her average velocity; Her velocity is constant; The magnitude of her average velocity after half of her journey is equal to her average speed at that time.","Completion":"1"} {"Type":"TOSS-UP\n\nMath - Short Answer","Prompt":"Solve the following equation for \\(x\\): the absolute value of open parenthesis \\(x-3\\) close parenthesis is equal to 5.","Completion":"-2 AND 8"} {"Type":"BONUS\n\nMath - Short Answer","Prompt":"What is the derivative evaluated at \\(x=5\\) of the fraction with numerator \\(x^{3}-8\\) and denominator \\(x-2\\)?","Completion":"12"} {"Type":"TOSS-UP\n\nMath - Multiple Choice","Prompt":"The tangent of open parenthesis pi + theta close parenthesis is equal to which of the following?\n\nW) Tangent theta\n\nX) Negative tangent theta\n\nY) Cotangent theta\n\nZ) Negative cosecant theta","Completion":"W) Tangent Theta"} {"Type":"BONUS\n\nMath - Short Answer","Prompt":"Special water for a 650-liter aquarium is only sold in 42-liter containers. What is the minimum number of these 42-liter containers you would need to buy to be able to completely fill the tank?","Completion":"16 (DO NOT ACCEPT: 15 10\/21 or 325\/21)"} {"Type":"TOSS-UP\n\nBiology - Short Answer","Prompt":"What is the term for infections or diseases that are naturally transmissible from vertebrate animals to humans?","Completion":"ZOONOSES (ACCEPT: ZOONOTIC)"} {"Type":"BONUS\n\nBiology - Short Answer","Prompt":"Identify all of the following three plants that associate parasitically with other plants: Vanilla; Mistletoe; Venus flytrap.","Completion":"2"} {"Type":"TOSS-UP\n\nEarth and Space - Multiple Choice","Prompt":"Which of the following is NOT among the criteria that define a substance as a mineral?\n\nW) Specific chemical composition\n\nX) Specific crystal structure\n\nY) Specific melting point\n\nZ) Naturally-occurring","Completion":"Y) SPECIFIC MELTING POINT"} {"Type":"TOSS-UP\n\nPhysics - Short Answer","Prompt":"The conversion of carbon-14 into nitrogen-14 is an example of what type of radioactive decay?","Completion":"BETA DECAY (ACCEPT: BETA MINUS DECAY)"} {"Type":"TOSS-UP\n\nChemistry - Multiple Choice","Prompt":"In which of the following compounds is the oxidation state of oxygen equal to zero?\n\nW) CuO\n\nX) H\\({}{2}\\)O\n\nY) O\\({}{2}\\)\n\nZ) H\\({}{3}\\)PO\\({}{4}\\)","Completion":"Y) O\\({}{2}\\)"} {"Type":"TOSS-UP\n\nEnergy - Short Answer","Prompt":"Scientists at Lawrence Berkeley National Lab recently studied the effect of\n\nvariation in genes regulating genomic [jeh-NOH-mik] integrity in a myriad of cancers. What specific\n\ncomplex of proteins, to which spindle fibers attach during mitosis, was coded for by many of these genes?","Completion":"KINETOCHORE"} {"Type":"TOSS-UP\n\nPhysics - Short Answer","Prompt":"When used as adjectives, Fresnel [fresh-NELL] and Fraunhofer [FR4WN-hofer] are examples of what property of waves that results in them bending around obstacles?","Completion":"DIFFRACTION"} {"Type":"TOSS-UP\n\nMath - Multiple Choice","Prompt":"Which of the following is true regarding the graph of \\(y=-x^{5}\\) at \\(x=-4\\)?\n\nW) Increasing and concave up\n\nX) Increasing and concave down\n\nY) Decreasing and concave up\n\nZ) Decreasing and concave down","Completion":"Y) DECREASING AND CONCAVE UP"} {"Type":"TOSS-UP\n\nEarth and Space - Short Answer","Prompt":"What type of tide occurs when the Moon is at first or third quarter?","Completion":"NEAP"} {"Type":"BONUS\n\nEarth and Space - Multiple Choice","Prompt":"A moon with a small mass is orbiting a large planet of unknown mass with an orbital period of ten to the fifth seconds and with a semimajor axis of ten to the eighth meters. Which of the following values is closest to the mass, in kilograms, of the planet?\n\nW) 6 times 10 to the 15\\({}^{\\rm th}\\)\n\nX) 6 times 10 to the 20\\({}^{\\rm th}\\)\n\nY) 6 times 10 to the 25\\({}^{\\rm th}\\)\n\nZ) 6 times 10 to the 30\\({}^{\\rm th}\\)","Completion":"Y) 6 TIMES 10 TO THE 25TH"} {"Type":"TOSS-UP\n\nEnergy - Short Answer","Prompt":"In the uranium fuel cycle, mined ore is milled and uranium is precipitated using solvent extraction or trituration. The precipitate is formed into bricks known as what?","Completion":"YELLOWCAKE (ACCEPT: URANIA)"} {"Type":"BONUS\n\nEnergy - Short Answer","Prompt":"The Dark Energy Camera was built at Fermilab to image the universe from an observatory in Chile. Identify all of the following three statements that are true of the Dark Energy Camera: It uses charge-coupled devices to detect light; It is better at detecting red and near-infrared light than violet and ultraviolet light; It is part of an interferometer.","Completion":"1 AND 2"} {"Type":"TOSS-UP\n\nChemistry - Short Answer","Prompt":"Identify all of the following three acids that are polyprotic [poly-PRO-tic]: Phosphorous acid; Hydroiodic [HIGH-droh-eye-AW-dikj acid; Selenic acid.","Completion":"1 AND 3"} {"Type":"BONUS\n\nChemistry - Short Answer","Prompt":"Consider the dehydration of isopropanol by sulfuric acid to form propene. What is the mechanism of this reaction?","Completion":"E1 (ACCEPT: ELIMINATION UNIMOLEULAR UNIMOLEULAR ELIMINATION, ELIMINATION 1)"} {"Type":"TOSS-UP\n\nBiology - Short Answer","Prompt":"What structures at the ends of eukaryotic [YOU-care-ee-AW-tic] chromosomes prevent their degradation?","Completion":"TELOMERES"} {"Type":"BONUS\n\nBiology - Multiple Choice","Prompt":"In DNA replication, which of the following components is necessary for the synthesis of the lagging strand but not the leading strand?\n\nW) Helicase [HEEL-ih-case] X) Ligase [LYE-gase] Y) Primase\n\nZ) Single-strand binding proteins","Completion":"X) LIGASE13) Physics - Short Answer What is the thermodynamic efficiency of a Carnot [CAR-noh] engine whose hot source is at 2000 kelvins and whose cold sink is at 250 kelvins?"} {"Type":"TOSS-UP\n\nBiology \\(-\\) Short Answer","Prompt":"The neurons found in humans require nourishment from supporting cells\n\nknown as what?","Completion":"GLIA (ACCEPT: NEUROGLIA, GLIAL CELLS)"} {"Type":"TOSS-UP\n\nMath - Short Answer","Prompt":"What is the distance between the points (0, and (4, ?","Completion":"4\\(\\backslash\\)2"} {"Type":"TOSS-UP\n\nEnergy - Short Answer","Prompt":"Gaseous [gah-shus] diffusion is currently the most common method used in the United States for the enrichment of uranium. What is the chemical formula of the gaseous uranium compound in this stage?","Completion":"UF\\({}{6}\\)"} {"Type":"BONUS\n\nEarth and Space \\(-\\) Short Answer","Prompt":"The Kirkwood gaps in the asteroid belt are due to orbital resonance with what body?","Completion":"JUPITER"} {"Type":"TOSS-UP\n\nChemistry \\(-\\) Short Answer","Prompt":"A reaction has a rate constant with units of liters to the 3rd, moles to the negative 3rd, and seconds to the negative 1. What is the order of this reaction?","Completion":"4"} {"Type":"BONUS\n\nChemistry \\(-\\) Short Answer","Prompt":"Identify all of the following three statements that are true about intermolecular [inter-moh-LEK-yoo-lur] forces and phase diagrams: Dinitrogen has a high critical temperature relative to water; Substances with higher vapor pressures have lower boiling points; Substances with stronger intermolecular forces have lower vapor pressures.","Completion":"2 AND 3"} {"Type":"TOSS-UP\n\nMath - Short Answer","Prompt":"What is the slope of the line with equation \\(4x-7\\mbox{y}=8\\)?","Completion":"4\/7"} {"Type":"TOSS-UP\n\nBiology - Short Answer","Prompt":"One common method of cloning DNA is incorporating a small loop of DNA containing the gene of interest into E. coli [ee COLE-eye]. What is the term for these small loops?","Completion":"PLASMIDS"} {"Type":"TOSS-UP\n\nPhysics - Short Answer","Prompt":"What set of rules states that for an electron configuration, the term with maximum multiplicity has the lowest energy?","Completion":"HUND'S RULES"} {"Type":"TOSS-UP\n\nEnergy - Multiple Choice","Prompt":"The National Energy Technology Lab is leading a project that gasifies coal and coal-biomass blends to form syngas, which is primarily a mixture of hydrogen and what other chemical compound?\n\nW) Carbon monoxide\n\nX) Carbon dioxide\n\nY) Methane\n\nZ) Sulfur dioxide","Completion":"W) CARBON MONOXIDE"} {"Type":"TOSS-UP\n\nEarth and Space - Short Answer","Prompt":"In what corner of a Hertzsprung-Russell diagram can one find an O-class supergiant?","Completion":"TOP LEFT"} {"Type":"TOSS-UP\n\nMath - Short Answer","Prompt":"Giving your answer in standard form, what is the equation of the circle with center at (0, and diameter 28?","Completion":"\\(x^{2}+y^{2}=196\\)"} {"Type":"BONUS\n\nMath - Short Answer","Prompt":"What is the \\(x\\)-coordinate of the solution of the following system of two equations in two unknowns: \\(5x-3y=6\\) and \\(4x-7y=5\\)?","Completion":"27\/23 (ACCEPT: 1 4\/23)"} {"Type":"TOSS-UP\n\nBiology - Short Answer","Prompt":"Deficiency of what vitamin is linked to the connective-tissue disorder scurvy?","Completion":"VITAMIN C"} {"Type":"BONUS\n\nBiology - Short Answer","Prompt":"Identify all of the following three statements that are true of eukaryotic [YOU-care-ee-4W-tic] cell structures: The cilia [SILL-ee-ah] has a characteristic 9 plus 2 arrangement of microtubules [micro-TUBE-yules]; Lysosomes [LYE-soh-solums]are responsible for assembling ribosomes; The endoplasmic reticulum [reh-TIK-yoo-lum] plays a role in synthesizing lipids.","Completion":"1 AND 3"} {"Type":"TOSS-UP\n\nChemistry - Short Answer","Prompt":"What is the oxidation state of copper in Cu\\({}{2}\\)O?","Completion":"1+"} {"Type":"BONUS\n\nChemistry - Multiple Choice","Prompt":"Di-iodine gas can dissociate into two atoms of iodine gas. At 1200\n\nkelvins, the forward reaction has an equilibrium constant of approximately 10. In kiljoules [KILL-oh-jools] per mole, which of the following is closest to the standard change in Gibbs free energy of this reaction?\n\nW) -20\n\nX) -2\n\nY) +2\n\nZ) +20","Completion":"W) -20"} {"Type":"TOSS-UP\n\nPhysics - Short Answer","Prompt":"What is the term for the point on a pressure-temperature diagram at which a substance can exist in three phases of matter simultaneously?","Completion":"TRIPLE POINT"} {"Type":"BONUS\n\nPhysics - Short Answer","Prompt":"A circuit consists of a 10-volt battery connected to a parallel branch of three 90-ohm resistors. Expressing your answer in amperes and as a decimal with one significant figure, what is the total current flow through the circuit?","Completion":"0.37) Energy - Short Answer Scientists at Ames Lab are developing methods to encapsulate catalysts inside porous glass nanoparticles to offer exquisite control of stability and reactivity. What is the chemical formula of the encapsulating material they are using?"} {"Type":"TOSS-UP\n\nChemistry - Short Answer","Prompt":"What constant, which appears in the denominator of the Nernst equation, is the amount of charge present in one mole of electrons?","Completion":"FARADAY (ACCEPT: F)"} {"Type":"BONUS\n\nChemistry - Short Answer","Prompt":"For the gas-phase reaction of hydrogen and bromine [BROH-meen] to generate HBr, identify all of the following three changes that would shift the equilibrium towards products: Adding more hydrogen gas; Increasing the volume of the container; Adding a catalyst to the reaction.","Completion":"1"} {"Type":"TOSS-UP\n\nBiology - Multiple Choice","Prompt":"A plant loses the greatest amount of water through what structures?\n\nW) Root cuticle\n\nX) Leaf cuticle\n\nY) Lenticels\n\nZ) Stomata","Completion":"Z) STOMATA"} {"Type":"BONUS\n\nBiology - Multiple Choice","Prompt":"A flower that has four stamens, one pistil, four petals, and no sepals would be considered which of the following?\n\nW) Complete\n\nX) Incomplete\n\nY) Monoecious\n\nZ) Imperfect","Completion":"X) INCOMPLETE"} {"Type":"TOSS-UP\n\nMath - Short Answer","Prompt":"What is the expected value when rolling a single ten-sided die?","Completion":"5.5 (ACCEPT: 11\/2, 5\\(\\%\\))"} {"Type":"BONUS\n\nMath - Short Answer","Prompt":"If the third derivative of a position function can be described by the function 6t\\({}^{3}\\), what is the coefficient of the highest-order term in the position function?","Completion":"1\/20 (ACCEPT: 0.05)"} {"Type":"TOSS-UP\n\nPhysics - Short Answer","Prompt":"Currents induced inside conductors due in the presence of a time-varying magnetic field are called what?","Completion":"EDDY CURRENTS (ACCEPT: FOUCAULT CURRENTS)"} {"Type":"BONUS\n\nPhysics - Multiple Choice","Prompt":"Which of the following mathematical representations is commonly used to model AC voltages and currents as a rotating vector in the complex plane?\n\nW) Wavefunction\n\nX) Phasor\n\nY) Cosine vector\n\nZ) Tangent vector","Completion":"X) PHASOR13) Energy \\(-\\) Short Answer Alkaline batteries are a common type of commercialized battery that involves the reaction of manganese [MAYN-gan-eez] dioxide and what other metal?"} {"Type":"TOSS-UP\n\nMath - Short Answer","Prompt":"What is the antilog base 9 of 2?","Completion":"81"} {"Type":"BONUS\n\nMath - Short Answer","Prompt":"Each day, Sue cats 25% of the cookies left in a jar. At the end of the second day, there are 36 cookies left in the jar. How many cookies were in the jar originally?","Completion":"64"} {"Type":"TOSS-UP\n\nChemistry - Multiple Choice","Prompt":"Which of the following statements is true regarding a gas-phase reaction at chemical equilibrium?\n\nW) The forward and reverse reactions stop at equilibrium\n\nX) Adding reactant will increase the concentration of product\n\nY) Adding reactant will increase the equilibrium constant\n\nZ) The reverse reaction only begins at equilibrium","Completion":"X) ADDING REACTANT WILL INCREASE THE CONCENTRATION OF PRODUCT"} {"Type":"BONUS\n\nChemistry - Short Answer","Prompt":"How many three-center two-electron bonds are present in the compound with formula B\\({}{2}\\)H\\({}{6}\\)?","Completion":"2"} {"Type":"TOSS-UP\n\nEarth and Space - Short Answer","Prompt":"What substance serves as a lubricant in glacier basal slip?","Completion":"WATER"} {"Type":"TOSS-UP\n\nPhysics - Short Answer","Prompt":"The uranium-238 decay series terminates at the 206 isotope of what element?","Completion":"LEAD"} {"Type":"TOSS-UP\n\nMath - Short Answer","Prompt":"What is the sum of the two vectors \\(8i+3j\\) and \\(-6i-7j\\)?","Completion":"\\(2i-4j\\)"} {"Type":"TOSS-UP\n\nChemistry - Short Answer","Prompt":"What is the name given to a special mixture of liquids whose composition cannot be changed by simple distillation, such as 95% ethanol to 5% water?","Completion":"AZEOTROPES"} {"Type":"TOSS-UP\n\nBiology - Short Answer","Prompt":"Pneumatophores [new-MAT-ah-forz] and beets are modified versions of what plant organ?","Completion":"ROOTS"} {"Type":"TOSS-UP\n\nBiology - Short Answer","Prompt":"In what organelle does the Calvin cycle occur?","Completion":"CHLORPOLAST"} {"Type":"BONUS\n\nBiology - Short Answer","Prompt":"In humans, the inactive \\(X\\) chromosome condenses into a compact structure called what?","Completion":"BARR BODY"} {"Type":"TOSS-UP\n\nEarth and Space - Short Answer","Prompt":"What is the term for areas of the ocean floor that are relatively flat and found 5 to 6 kilometers below sea level?","Completion":"ABYSSAL PLAINS"} {"Type":"BONUS\n\nEarth and Space - Multiple Choice","Prompt":"If a metamorphic rock was subjected to high temperature and pressure deep in the crust long enough to reach equilibrium with its environment, which of the following would be most useful for determining the temperature and pressure range at which it formed?\n\nW) Elemental analysis\n\nX) Mineralogical analysis\n\nY) Grain-size analysis\n\nZ) Stress-strain analysis","Completion":"X) MINERALOGICAL ANALYSIS"} {"Type":"TOSS-UP\n\nEnergy - Short Answer","Prompt":"Scientists at the National Energy Technology Lab have been working with ION in Norway to evaluate the \"Advanced Liquid Absorbent System\" that works to capture and sequester what chemical produced by combustion?","Completion":"CARBON DIOXIDE (ACCEPT: CO\\({}{2}\\))"} {"Type":"BONUS\n\nEnergy - Short Answer","Prompt":"Scientists at Oak Ridge National Lab are working on a novel accelerated membrane-based gas separation process that will be used to separate what molecule from flue gas?","Completion":"CARBON DIOXIDE (ACCEPT: CO\\({}{2}\\))"} {"Type":"TOSS-UP\n\nChemistry - Multiple Choice","Prompt":"Which of the following elements is LEAST likely to violate the octet rule?\n\nW) Carbon\n\nX) Sulfur\n\nY) Phosphorous\n\nZ) Xenon [ZEE-non]","Completion":"W) CARBON"} {"Type":"BONUS\n\nChemistry - Short Answer","Prompt":"A chemist prepares a 10%-by-mass aqueous [AY-kwee-us] solution of a salt with a formula unit mass of 200 grams per mole. To two significant digits and in molal [moh-LAHL], what is the molality of this solution?","Completion":"0.56"} {"Type":"TOSS-UP\n\nPhysics - Short Answer","Prompt":"What principle of quantum mechanics states that two electrons cannot have the same quantum numbers?","Completion":"PAULI EXCLUSION PRINCIPLE"} {"Type":"TOSS-UP\n\nMath - Short Answer","Prompt":"A pair of ten-sided dice, each with the numbers one through 10 printed on the faces, is rolled. What is the probability that the two numbers rolled are different?","Completion":"9\/10 (ACCEPT: 0.9, 90%)"} {"Type":"TOSS-UP\n\nEarth and Space \\(-\\) Multiple Choice","Prompt":"The Richter scale is a base-10 logarithmic scale. What feature of an earthquake is multiplied by a factor of 10 for a unit increase on the Richter [RIK-tur] scale?\n\nW) Total energy radiated by an earthquake\n\nX) Fault rupture velocity\n\nY) Seismic [SIZE-mic] wave amplitude\n\nZ) Seismic moment","Completion":"Y) SEISMIC WAVE AMPLITUDE"} {"Type":"BONUS\n\nEarth and Space \\(-\\) Short Answer","Prompt":"Identify all of the following three minerals that possess two planes of cleavage: Mica; Pyroxene [pie-ROX-een]; Olivine.","Completion":"2"} {"Type":"TOSS-UP\n\nBiology \\(-\\) Multiple Choice","Prompt":"Chlorophyll \\(a\\) and \\(b\\) most strongly absorb which of the following colors of light?\n\nW) Red\n\nX) Yellow\n\nY) Green\n\nZ) Blue","Completion":"Z) BLUE"} {"Type":"BONUS\n\nBiology \\(-\\) Short Answer","Prompt":"Identify all of the following three groups of plants that have nitrogen-fixing root nodules: Peanuts; Soybean; Alfalfa.","Completion":"ALL"} {"Type":"TOSS-UP\n\nPhysics - Short Answer","Prompt":"What mechanical property of a material characterizes its stiffness as a ratio of its stress to strain?","Completion":"YOUNG'S MODULUS (ACCEPT: ELASTIC MODULUS)"} {"Type":"TOSS-UP\n\nMath - Short Answer","Prompt":"If a universal set contains 24 elements, how many elements are in the complement of a set with 9 elements?","Completion":"15"} {"Type":"TOSS-UP\n\nChemistry - Short Answer","Prompt":"What is the name for the functional group in organic chemistry that consists of a carbon atom double-bonded to an oxygen atom, and is a component of aldehydes and ketones [KEE-tones]?","Completion":"CARBONYL [carboh-NEEL]"} {"Type":"BONUS\n\nChemistry - Short Answer","Prompt":"Identify all of the following three atoms that have a higher first ionization [eye-on-ih-ZAY-shun] energy than lithium: Sodium; Beryllium [beth-RILL-ee-un]; Magnesium.","Completion":"2 AND 3"} {"Type":"TOSS-UP\n\nEnergy - Short Answer","Prompt":"Berkeley lab scientists are studying \"gateway\" proteins associated with the nuclear pore complex, which are responsible for blocking aberrant strands of what RNA-polymerase [paw-LIM-er-ace] II-synthesized macromolecule?","Completion":"MESSENGER RNA (ACCEPT: mRNA)"} {"Type":"BONUS\n\nEnergy - Short Answer","Prompt":"Scientists at SLAC [slack] recently used the SSRL to study the protein complex Cascade that is associated with the bacterial immune system and is of recent interest due to potential uses for gene editing. With what prokaryotic [PRO-care-ee-AW-tic] DNA segments is Cascade associated?","Completion":"CRISPR (ACCEPT: CLUSTERED REGULARLY INTERSPACED SHORT PALINDROMIC REPEATS)"} {"Type":"TOSS-UP\n\nEarth and Space - Multiple Choice","Prompt":"Which of the following is closest to the average length of time an individual sunspot will persist on the surface of the sun?\n\nW) 1 hour\n\nX) 50 days\n\nY) 10 years\n\nZ) 500 years","Completion":"X) 50 DAYS"} {"Type":"BONUS\n\nEarth and Space - Multiple Choice","Prompt":"Which of the following most accurately describes the strength of Mercury's magnetic field at the planet's surface?\n\nW) Not detectable\n\nX) About the same as that of the Moon\n\nY) About one-hundreth that of the Earth\n\nZ) About half that of the Earth","Completion":"Y) ABOUT ONE-HUNDRETH THAT OF THE EARTH"} {"Type":"TOSS-UP\n\nChemistry - Short Answer","Prompt":"In the Lewis structure of the cyanide anion [AN-eye-on], how many electrons belong SOLELY to the nitrogen atom?","Completion":"TWO"} {"Type":"BONUS\n\nChemistry - Short Answer","Prompt":"Identify all of the following three compounds that have a hybridization [HIGH-brih-dye-ZAY-shun] scheme requiring d-orbitals in VSEPR theory: Xenon [ZEE-non] tetrafluoride; Xenon tetroxide [teh-TROX-ide]; Xenon difluoride.","Completion":"1 AND 3"} {"Type":"TOSS-UP\n\nMath - Short Answer","Prompt":"What is the cosine of 240 degrees?","Completion":"-1\/2 (ACCEPT: -0.5)"} {"Type":"BONUS\n\nMath - Short Answer","Prompt":"The shortest side of a 3-4-5 right triangle lies on the \\(x\\)-axis. If the triangle is rotated about the \\(x\\)-axis, what is the volume of the resulting shape?","Completion":"16\\(\\pi\\)"} {"Type":"TOSS-UP\n\nPhysics - Short Answer","Prompt":"A 60-kilogram sprinter accelerates horizontally off the starting block at 18 meters per second squared. What horizontal force, in newtons, does the sprinter exert on the block?","Completion":"1080"} {"Type":"BONUS\n\nPhysics - Short Answer","Prompt":"A 9-kilogram ball traveling at 5 meters per second undergoes an elastic collision with a 6-kilogram block that is stationary before the collision. What is the kinetic energy, in joules, of the block after the collision?","Completion":"10817) Biology - Short Answer What is the term for paired chromosomes that carry genes controlling the same set of inherited characteristics?"} {"Type":"TOSS-UP\n\nEnergy - Multiple Choice","Prompt":"Scientists at Brookhaven National Lab have developed a method to stimulate the anabolic [AN-ah-BAWL-ik] reactions in the smooth endoplasmic [EN-doh-PLAZ-mik] reticulum [reh-TIK-yoo-lum] of plant leaves. These findings could result in better feedstock for production of which of the following?\n\nW) Biofuel\n\nX) Fertilizer\n\nY) Paper\n\nZ) Lumber","Completion":"W) BIOFUEL"} {"Type":"TOSS-UP\n\nChemistry - Short Answer","Prompt":"Rank the following three elements in increasing order of electron affinity:\n\nLithium; Oxygen; Sulfur.","Completion":"1, 3, 2"} {"Type":"TOSS-UP\n\nEarth and Space - Short Answer","Prompt":"Neptune appears a shade of blue because of what atmospheric gas mixed in with hydrogen and helium?","Completion":"METHANE (ACCEPT: CH\\({}{4}\\))"} {"Type":"TOSS-UP\n\nBiology - Short Answer","Prompt":"Sickle cell anemia is an example of heterozygote [hetero-ZYE-goat]\n\nadvantage because it protects against what disease?","Completion":"MALARIA"} {"Type":"TOSS-UP\n\nMath - Short Answer","Prompt":"Log base 100 of 50 is equal to log base 10 of 50 divided by log base 10 of what number?","Completion":"100"} {"Type":"BONUS\n\nMath - Short Answer","Prompt":"A bag contains 22 socks: 6 are red, 8 are white, and 8 are blue. How many socks must you take out to be certain that you will have at least one pair of blue socks?","Completion":"16"} {"Type":"TOSS-UP\n\nChemistry - Short Answer","Prompt":"What is the molecular [moh-LEK-yoo-lur] geometry of methane?","Completion":"TETRAHERDAL"} {"Type":"BONUS\n\nChemistry - Short Answer","Prompt":"A student mixes together aqueous [AY-kwee-us] equimolar solutions of calcium chloride and ammonium phosphate. Which two of the component ions stay in solution?","Completion":"AMMONIUM AND CHLORIDE (ACCEPT IN ANY ORDER)"} {"Type":"TOSS-UP\n\nEnergy - Multiple Choice","Prompt":"Which of the following is a renewable, primary energy source?\n\nW) Nuclear\n\nX) Fossil\n\nY) Geothermal\n\nZ) Electricity","Completion":"Y) GEOTHERMAL"} {"Type":"TOSS-UP\n\nPhysics - Short Answer","Prompt":"If one wants to double the period of a simple pendulum, what ratio of new-to-old length is required?","Completion":"4 (ACCEPT: 4 TO 1)"} {"Type":"TOSS-UP\n\nEarth and Space \\(-\\) Short Answer","Prompt":"What part of the nitrogen cycle can be performed by lightning?","Completion":"NITROGEN FIXATION"} {"Type":"TOSS-UP\n\nBiology \\(-\\) Multiple Choice","Prompt":"In the Griffith experiment, nonpathogenic bacteria were found to be able to cause disease after being exposed to heat-killed virulent bacteria. Which of the following substances caused this transformation?\n\nW) Toxins\n\nX) Proteins\n\nY) RNA\n\nZ) DNA","Completion":"Z) DNA"} {"Type":"TOSS-UP\n\nEnergy - Multiple Choice","Prompt":"The Peter Auger cosmic ray observatory is operated by DOE scientists from Fermilab. Which of the following statements is TRUE of how cosmic rays are detected by this observatory?\n\nW) The observatory detects cosmic rays by measuring the local electric field\n\nX) The observatory directly observes cosmic rays interacting with water\n\nY) The observatory detects the shower of energetic particles produced by a cosmic ray\n\nZ) The observatory uses interferometry to detect cosmic rays","Completion":"Y) THE OBSERVATORY DETECTS THE SHOWER OF ENERGETIC PARTICLES PRODUCED BY A COSMIC RAY"} {"Type":"BONUS\n\nEnergy - Short Answer","Prompt":"Scientists at SLAC [slack] have been studying the iron sulfur cluster in a group of liver-detoxifying enzymes to determine the source of their high activity towards a wide variety of substrates. What is the general name for this group of enzymes?","Completion":"CYTOCHROME P450"} {"Type":"TOSS-UP\n\nChemistry - Short Answer","Prompt":"The total amount of useful work extractable from a chemical reaction at constant pressure and temperature is equal to the negative change in what state function?","Completion":"GIBBS FREE ENERGY (ACCEPT: G)"} {"Type":"BONUS\n\nChemistry - Short Answer","Prompt":"Identify all of the following three molecules that are matched correctly to their hybridization: Xenon dilluoride and dsp\\({}^{3}\\); Nitrogen tri-iodide and sp\\({}^{2}\\); Sulfur tetrafluoride and d\\({}^{2}\\)sp\\({}^{3}\\).","Completion":"1"} {"Type":"TOSS-UP\n\nBiology - Multiple Choice","Prompt":"Which of the following best describes the result of the first step of glycolysis [glye-KAWL-eh-sis] in a fibroblast?\n\nW) Glucose-6-phosphate is fully committed to the glycolysis [glye-KAWL-eh-sis] pathway\n\nX) Fructose-1,6-bisphosphate [biss-FAWS-fatel] is split into two three-carbon sugars\n\nY) ATP is extracted from glucose-6-phosphate\n\nZ) Glucose-6-phosphate can no longer exit the cell","Completion":"Z) GLUCOSE-6-PHOSPHATE CAN NO LONGER EXIT THE CELL"} {"Type":"TOSS-UP\n\nMath - Short Answer","Prompt":"What is the greatest common factor of 36 and 92?","Completion":"4"} {"Type":"TOSS-UP\n\nEarth and Space - Multiple Choice","Prompt":"Which of the following is closest to the ratio of the absolute temperature at the Sun's core to that of its surface?\n\nW) 25\n\nX) 250\n\nY) 2,500\n\nZ) 25,000","Completion":"Y) 2,500"} {"Type":"TOSS-UP\n\nBiology - Short Answer","Prompt":"What specific quantity on a reaction diagram does the addition of an enzyme change?","Completion":"ACTIVATION ENERGY (ACCEPT: TRANSITION STATE ENERGY)"} {"Type":"TOSS-UP\n\nPhysics - Short Answer","Prompt":"Identifify all of the following three quantities that are vector quantities:\n\nImpulse; Momentum; Energy.","Completion":"1 AND 2"} {"Type":"BONUS\n\nPhysics - Short Answer","Prompt":"A 3-kilogram block slides along a horizontal surface, and the coefficient of kinetic friction between them is 0.5. The block's initial velocity is 20 meters per second. To the nearest second, how long does it take for the block to come to a stop?","Completion":"4"} {"Type":"TOSS-UP\n\nChemistry - Short Answer","Prompt":"Mendeleev famously predicted the existence of eka-silicon in 1871 as well as a list of properties this theoretical element would have, many of which turned out to be correct when the element was discovered in 1886. What is the IUPAC name of this element?","Completion":"GERMANIUM"} {"Type":"BONUS\n\nChemistry - Short Answer","Prompt":"Identifify all of the following three quantities that are positive for the gas-phase decomposition reaction of carbon dioxide to form carbon monoxide and molecular oxygen at\n\n300 kelvins: Delta-H; Delta-S; Delta-G.","Completion":"ALL"} {"Type":"TOSS-UP\n\nMath - Short Answer","Prompt":"How many exterior angles does an octagon have?","Completion":"16 (DO NOT ACCEPT: 8)"} {"Type":"BONUS\n\nMath - Short Answer","Prompt":"What are the coordinates of the vertex of the graph of\n\n\\(y=-3x^{2}-12x-7\\)?","Completion":"(-2, 5) (ACCEPT: \\(x=-2\\) AND \\(y=5\\))"} {"Type":"TOSS-UP\n\nEnergy - Multiple Choice","Prompt":"Researchers at SLAC [slack] are currently working on mining methods to expand proven resources of uranium. Which of the following contains the largest amount of uranium?\n\nW) Ocean water\n\nX) Salt mines\n\nY) Gold deposits\n\nZ) Coal mines","Completion":"W) OCEAN WATER"} {"Type":"BONUS\n\nEnergy - Short Answer","Prompt":"Scientists at Lawrence Berkeley National Lab discovered a protein important in DNA repair called XPG, whose function was often lost in breast, ovarian, and other cancer types. XPG is in what class of cancer-preventing genes, examples of which also include APC and p53?","Completion":"TUMOR SUPPRESSOR19) Earth and Space - Short Answer In a stream, what is the term for the sediment load that slides and rolls along the stream bed?"} {"Type":"BONUS\n\nEarth and Space - Short Answer","Prompt":"Rank the following three materials in increasing order of speed of P-waves when traveling through them: Ultramafic; Felsic; Mafic.","Completion":"2, 3, 1"} {"Type":"TOSS-UP\n\nPhysics - Multiple Choice","Prompt":"Which of the following adjacent pairs of thermodynamic processes are a part of a Carnot [CAR-noh] cycle?\n\nW) Isothermal expansion and isentropic [eye-sen-TROH-pik] expansion\n\nX) Isentropic compression and isochoric [eye-soh-KOR-ik] heating\n\nY) Isentropic compression and isobaric [eye-soh-BARE-ik] heating\n\nZ) Isothermal compression and isochoric cooling","Completion":"W) ISOTHERMAL EXPANSION AND ISENTRropic EXPANSION"} {"Type":"BONUS\n\nPhysics - Short Answer","Prompt":"The force on an object was kept constant for 1 second and then changed to a new value for each successive second, for a total of five force values over a five-second time interval. All five forces were in the same direction and had magnitudes of 0 newtons, 5 newtons, 15 newtons, 5 newtons, and 0 newtons. What was the magnitude of the total impulse, in newton seconds, exerted on the object during this time?","Completion":"25"} {"Type":"TOSS-UP\n\nChemistry - Multiple Choice","Prompt":"Which of the following is not a Lewis acid?\n\nW) Aluminum chloride\n\nX) Ammonia\n\nY) Carbon dioxide\n\nZ) Magnesium ion","Completion":"X) AMMONIA"} {"Type":"TOSS-UP\n\nMath - Short Answer","Prompt":"If the probability of it raining on a given day is 0.21, what is the probability of it NOT raining?","Completion":"0.79 (ACCEPT: 79\/100)"} {"Type":"TOSS-UP\n\nMath - Multiple Choice","Prompt":"Which of the following is NOT true?\n\nW) 37 is congruent to 6 mod 5\n\nX) 71 is congruent to 5 mod 3\n\nY) 43 is congruent to 8 mod 7\n\nZ) 17 is congruent to 2 mod 3","Completion":"W) 37 IS CONGRUENT TO 6 MOD 5"} {"Type":"BONUS\n\nMath - Short Answer","Prompt":"Find the \\(y\\)-intercept of the line passing through the point \\((4,-\\) that is perpendicular to the line \\(3x-7y=12\\).","Completion":"19\/3 (ACCEPT: 6 1\/3, (0, 19\/3), (0, 6 1\/3))"} {"Type":"TOSS-UP\n\nEnergy - Multiple Choice","Prompt":"At the National Renewable Energy Lab, scientists are using sunlight to split what chemical compound during photoelectrochemical processes that employ stacks of light-absorbing semiconductors immersed in an acid-water solution?\n\nW) Methane\n\nX) Ammonia\n\nY) Hydrogen Peroxide\n\nZ) Water","Completion":"Z) WATER"} {"Type":"BONUS\n\nEnergy - Short Answer","Prompt":"Identify all of the following three statements that are true of secondary batteries: They are not rechargeable; They require a constant source of energy; The electricity-producing reaction in them is chemically reversible.","Completion":"3"} {"Type":"TOSS-UP\n\nChemistry - Multiple Choice","Prompt":"Addition of which of the following compounds will increase the solubility of calcium carbonate in water?\n\nW) Sodium carbonate\n\nX) Phosphoric acid\n\nY) Sodium hydroxide\n\nZ) Calcium chloride","Completion":"X) PHOSPHORIC ACID"} {"Type":"TOSS-UP\n\nEarth and Space - Multiple Choice","Prompt":"Which of the following numbers corresponds to the approximate temperature in kelvins of red stars?\n\nW) 300\n\nX) 3,000\n\nY) 30,000\n\nZ) 300,000","Completion":"X) 3,000"} {"Type":"TOSS-UP\n\nBiology - Multiple Choice","Prompt":"Cataracts occur in what part of the eye?\n\nW) Retina\n\nX) Posterior chamber\n\nY) Cornea\n\nZ) Lens","Completion":"Z) LENS"} {"Type":"TOSS-UP\n\nPhysics - Short Answer","Prompt":"The henry is the SI unit corresponding to what property of an electric circuit?","Completion":"INDUCTANCE"} {"Type":"TOSS-UP\n\nEarth and Space \\(-\\) Multiple Choice","Prompt":"If the total energy flux from a star quadruples, by what factor is its temperature multiplied?\n\nW) \\(\\surd\\)2 [square root of 2]\n\nX) 2\n\nY) 4\n\nZ) 16","Completion":"W) \\(\\surd\\)"} {"Type":"TOSS-UP\n\nBiology \\(-\\) Short Answer","Prompt":"Fungal cell walls are primarily composed of what compound?","Completion":"CHITIN"} {"Type":"TOSS-UP\n\nChemistry - Multiple Choice","Prompt":"The pKa [p.k.a.] of deprotonation [dee-pro-tun-AY-shun] for most primary amines [ah-MEENS] is between 35 and 45. Which of the following is closest to the pKa of cyanamide [sigh-ANN-ah-mide], a primary amine in which the R group is a cyano group?\n\nW) 1\n\nX) 15\n\nY) 35\n\nZ) 45","Completion":"W) 1"} {"Type":"BONUS\n\nChemistry - Multiple Choice","Prompt":"Some compounds feature two-atom covalent [koh-VAY-lent] bonds in which both electrons forming the bond are donated by one of the atoms, also known as a dative bond.\n\nWhich of the following bonds is most likely to be a dative bond?\n\nW) Carbon-carbon\n\nX) Sodium-fluorine [FLOOR-een]\n\nY) Nitrogen-boron\n\nZ) Hydrogen-chlorine","Completion":"Y) NITROGEN-BORON"} {"Type":"TOSS-UP\n\nPhysics - Short Answer","Prompt":"According to the Standard Model of Particle Physics, what lepton [LEP-tawn]is the most massive?","Completion":"TAU (ACCEPT: TAUON)"} {"Type":"BONUS\n\nPhysics - Short Answer","Prompt":"A particle undergoing a constant acceleration starts at rest at position \\(x\\) equals zero. Identify all of the following three pairs of positions it could be at after one second and then after two seconds, respectively: \\(x\\) equals zero, then \\(x\\) equals zero; \\(x\\) equals three, then \\(x\\) equals twelve; \\(x\\) equals two, then \\(x\\) equals sixteen.","Completion":"1 AND 2"} {"Type":"TOSS-UP\n\nMath - Short Answer","Prompt":"A bag contains six uniquely marked marbles. One marble is drawn, and then a second, without replacement. How many different permutations are possible?","Completion":"30"} {"Type":"TOSS-UP\n\nEnergy - Short Answer","Prompt":"In order to better design materials that can tolerate the extreme pressures and temperatures of controlled fusion reactions, researchers at LANL are simulating the behavior of bubbles of what gas embedded in tungsten?","Completion":"HELIUM"} {"Type":"TOSS-UP\n\nChemistry - Short Answer","Prompt":"What is the systematic name of propyl iodide?","Completion":"1-IODOPROPANE"} {"Type":"TOSS-UP\n\nPhysics - Short Answer","Prompt":"What is the equivalent resistance, in ohms, of a circuit composed of a 2-ohm resistor connected in parallel to a series branch with two 3-ohm resistors?","Completion":"3\/2 (ACCEPT 1 \\(\\vee{2}\\) or 1.5)"} {"Type":"TOSS-UP\n\nEarth and Space \\(-\\) Multiple Choice","Prompt":"Which of the following minerals would scratch all of the others?\n\nW) Quartz\n\nX) Calcite\n\nY) Halite\n\nZ) Topaz","Completion":"Z) TOPAZ"} {"Type":"TOSS-UP\n\nEnergy \\(-\\) Short Answer","Prompt":"The U.S. Department of Energy's Thomas Jefferson National Accelerator Facility is studying the components of protons and mesons. The quarks inside these particles are held together by interactions mediated by what boson [BOH-sawn]?","Completion":"GLUON"} {"Type":"TOSS-UP\n\nBiology - Short Answer","Prompt":"In humans, what gland makes melatonin?","Completion":"PINEAL"} {"Type":"TOSS-UP\n\nEarth and Space - Short Answer","Prompt":"What type of organized, low-pressure weather system is an intermediate stage between a tropical depression and a hurricane?","Completion":"TROPICAL STORM"} {"Type":"TOSS-UP\n\nMath - Short Answer","Prompt":"Given \\(f\\) of \\(x=4x-6\\) and \\(g\\) of \\(x=3x\\), what is \\(g\\) of \\(f\\) of x?","Completion":"12\\(x\\) - 18"} {"Type":"TOSS-UP\n\nPhysics - Short Answer","Prompt":"Unpolarized light with an intensity of \\(I\\) passes through a filter that makes a 50-degree angle with respect to the horizontal. In terms of \\(I\\), what is the intensity of the light after it passes through the filter?","Completion":"0.5 \\(I\\)"} {"Type":"TOSS-UP\n\nChemistry - Short Answer","Prompt":"Hans Bethe is famous for the development of what theory of coordination in which metal-ligand [LIH-gund] interactions are assumed to be completely ionic?","Completion":"CRYSTAL FIELD THEORY (ACCEPT: CFT)"} {"Type":"BONUS\n\nChemistry - Short Answer","Prompt":"A student constructs a ball and stick model of a linear molecule. Identify all of the following statements that could be valid for this molecule: The central atom has zero lone pairs; The central atom has two lone pairs; The central atom has three lone pairs.","Completion":"1 AND 3"} {"Type":"TOSS-UP\n\nBiology - Short Answer","Prompt":"What reflex describes the profound decrease in heart rate of marine mammals upon feeling cold water on the face?","Completion":"DIVE (ACCEPT: DIVE REFLEX)"} {"Type":"BONUS\n\nBiology - Short Answer","Prompt":"Identify all of the following three amino acids that are considered nonpolar: Asparagine [lass-PAIR-ah-jeen]; Tryptophan [TRIP-tow-fan]; Glutamine.","Completion":"2"} {"Type":"TOSS-UP\n\nMath - Short Answer","Prompt":"The complex number \\(3i\\) lies on the boundary between what two quadrants in the complex plane?","Completion":"1 AND 2 (ACCEPT: FIRST AND SECOND)"} {"Type":"BONUS\n\nMath - Short Answer","Prompt":"On a calculus pop quiz, 10% of the students scored 6 points, 45% scored 8 points, 20% scored 9 points, and the rest scored 10 points. What is the average score, rounded to the nearest tenth, on the quiz?","Completion":"8.5"} {"Type":"TOSS-UP\n\nBiology - Short Answer","Prompt":"What is the term for the enlarged end of a stolon that is used for storing food?","Completion":"TUBER"} {"Type":"TOSS-UP\n\nPhysics - Short Answer","Prompt":"When beryllium-8 emits an alpha particle, what isotope is formed?","Completion":"HELIUM-4"} {"Type":"BONUS\n\nPhysics - Short Answer","Prompt":"A toaster is composed of 4 compartments connected in parallel to a standard 120-volt RMS wall socket. If each compartment is modeled by a 4-kilo-ohm resistor, what is the peak power, in watts, dissipated by the toaster when 4 waffles are being toasted simultaneously?","Completion":"28.8"} {"Type":"TOSS-UP\n\nEnergy - Multiple Choice","Prompt":"Researchers at SLAC National Accelerator Lab have developed a new electron-gun that produces near-continuous high-energy electron beams. The electron gun uses a chamber of niobium cooled to 4 kelvins, which has what property?\n\nW) Piezoelectricity [PEE-zoh-electricity]\n\nX) Extremely high magnetic susceptibility\n\nY) Superfluidity\n\nZ) Superconductivity","Completion":"Z) SUPERCONDUCTIVITY"} {"Type":"BONUS\n\nEnergy - Short Answer","Prompt":"A team of scientists from Oak Ridge National Laboratory have been studying the enzyme RuBisCO [roo-BIS-kohl] using neutron scattering and X-ray diffraction techniques. What key atom can neutron-scattering methods directly detect that X-ray methods cannot?","Completion":"HYDROGEN"} {"Type":"TOSS-UP\n\nMath - Multiple Choice","Prompt":"Consider the polynomial equation \\(6x^{3}-24x+25=0.\\) According to the rational root theorem, which of the following is NOT a possible rational root of this equation?\n\nW) -3\n\nX) 1\/3\n\nY) 5\/6\n\nZ) 5\/3","Completion":"W) -3"} {"Type":"BONUS\n\nMath - Short Answer","Prompt":"If two sides of a right triangle have lengths 15 and 7, then, rounded to the nearest whole numbers, what are the possible lengths of the third side?","Completion":"13 and 17"} {"Type":"TOSS-UP\n\nEarth and Space - Multiple Choice","Prompt":"Which of the following is closest to the pre-industrial concentration of carbon dioxide in the atmosphere?\n\nW) 800 parts per billion\n\nX) 275 parts per billion\n\nY) 275 parts per million\n\nZ) 8 parts per million","Completion":"Y) 275 PARTS PER MILLION"} {"Type":"BONUS\n\nEarth and Space - Short Answer","Prompt":"A subsurface magma chamber cools until fifty percent of its mass has crystallized. Identify all of the following three statements that are true of the magma-rock system: The magma is differentiated; The rock and the magma will be chemically identical; The newly-formed rock will have a glassy texture.","Completion":"17) Physics - Short Answer What is the term for frictional forces between adjacent layers within a fluid that prevent them from flowing freely?"} {"Type":"TOSS-UP\n\nBiology - Short Answer","Prompt":"What type of meristem is primarily responsible for secondary growth?","Completion":"LATERAL"} {"Type":"TOSS-UP\n\nEnergy - Multiple Choice","Prompt":"Scientists at Pacific Northwest National Lab have constructed the Shallow Underground Laboratory at a depth of 12 meters to reduce background from which of the following sources?\n\nW) Neutrinos\n\nX) Cosmic rays\n\nY) Top quarks\n\nZ) Gravitational waves","Completion":"X) COSMIC RAYS"} {"Type":"## BONUS\n\nMath - Short Answer","Prompt":"A custom mixture of nuts is composed of cashews worth $8 a pound and almonds worth $6 a pound. Mae purchases 11 pounds of the mixture for $75. How many of these 11 pounds are cashews?","Completion":"4.5 (ACCEPT: 4\\(\\%\\), 9\/2)"} {"Type":"## BONUS\n\nEarth and Space - Multiple Choice","Prompt":"Which of the following locations in a spiral galaxy possesses the highest rate of star formation?\n\nW) Halo\n\nX) Core\n\nY) Spiral arm\n\nZ) Area between spiral arms","Completion":"Y) SPIRAL ARM"} {"Type":"TOSS-UP\n\nPhysics - Multiple Choice","Prompt":"The energy potential of a system scales quadratically with respect to distance away from a point. Which of the following could represent this system?\n\nW) Gravitational orbit\n\nX) Electrostatic orbit\n\nY) Hookean spring\n\nZ) Double pendulum","Completion":"Y) HOOKEAN SPRING"} {"Type":"BONUS\n\nPhysics - Short Answer","Prompt":"In an experiment of the photoelectric effect, a metal is found to emit electrons when exposed to strong blue light, but not to weak yellow light. Identify all of the following four forms of light that will eject electrons from the metal: Weak red light;\n\nStrong red light; Weak violet light; Strong violet light.","Completion":"3, 4"} {"Type":"TOSS-UP\n\nEnergy - Multiple Choice","Prompt":"Idaho National Lab researchers are studying rocks that form near rhyolitic [rye-oh-LIT-ik] eruptions with x-ray fluorescence. Which of the following statements is true regarding the magma from eruptions?\n\nW) It contains little silica compared to basaltic magma\n\nX) It is cooler in temperature than basaltic magma\n\nY) It is low-viscosity compared to basaltic magma\n\nZ) It is high in iron content relative to basaltic magma","Completion":"X) IT IS COOLER IN TEMPERATURE THAN BASALIC MAGMA"} {"Type":"BONUS\n\nEnergy - Short Answer","Prompt":"Scientists at SLAC National Accelerator Lab are developing improvements to scanning tunneling electron microscopy. In this technique, an electron beam interacts with what shell of electrons in the sample?","Completion":"VALENCE SHELL"} {"Type":"TOSS-UP\n\nChemistry - Multiple Choice","Prompt":"Which of the following is NOT a Lewis acid?\n\nW) Boron tribromide\n\nX) Aluminum trichloride\n\nY) Carbon tetrachloride\n\nZ) Sulfur trioxide","Completion":"Y) CARBON TETRACHLORIDE"} {"Type":"BONUS\n\nChemistry - Short Answer","Prompt":"Rank the following three indicators in terms of increasing pKa:\n\nPhenolphthalein [fee-nawf-THAY-leen]; Methyl red;\n\nBromothymol [BROH-mo-THIGH-mawl] blue.","Completion":"2, 3, 1"} {"Type":"TOSS-UP\n\nBiology - Multiple Choice","Prompt":"What is the name of the largest family of human cell surface receptors that is characterized by a 7-transmembrane domain motif?\n\nW) G-protein coupled receptor\n\nX) Ionotropic receptor\n\nY) Receptor tyrosine kinase\n\nZ) Catalytic receptor","Completion":"W) G-PROTEIN COUPLED RECEPTOR"} {"Type":"BONUS\n\nBiology - Multiple Choice","Prompt":"Which of the following is NOT a seedless, vascular plant?\n\nW) Ferns\n\nX) Liverworts\n\nY) Horsetails\n\nZ) Club mosses","Completion":"X) LIVERWORTS"} {"Type":"TOSS-UP\n\nChemistry - Multiple Choice","Prompt":"Which of the following would cause the electron distribution in a molecule to change the least?\n\nW) A nearby ion\n\nX) A far-away dipole\n\nY) An external magnetic field\n\nZ) An external electric field","Completion":"X) A FAR-AWAY DIPOLE"} {"Type":"TOSS-UP\n\nMath - Short Answer","Prompt":"What is the slope of a line perpendicular to the line with equation\n\n7\\(x-6y=21\\)?","Completion":"-6\/7"} {"Type":"TOSS-UP\n\nEarth and Space \\(-\\) Short Answer","Prompt":"What letter denotes the spectral type of the Sun?","Completion":"G (ACCEPT G2, G2V)"} {"Type":"TOSS-UP\n\nBiology \\(-\\) Short Answer","Prompt":"A pea plant with yellow, round seeds that is heterozygous [hetero-ZYE-gus] for both traits is allowed to self-pollinate. What percentage of the offspring plants will be true-breeding for both yellow and wrinkled seeds?","Completion":"6.25 (ACCEPT: 6\\(\\%\\), 25\/4)"} {"Type":"TOSS-UP\n\nBiology - Short Answer","Prompt":"Two groups of the same species of monkey develop different mating rituals and eventually become reproductively incompatible. If this occurs while they occupy the same geographic region, what is the adjective for the type of speciation that has occurred?","Completion":"SYMPATRIC"} {"Type":"TOSS-UP\n\nChemistry - Multiple Choice","Prompt":"Which of the following elements has the largest first ionization energy?\n\nW) Sodium\n\nX) Aluminum\n\nY) Argon\n\nZ) Chlorine","Completion":"Y) ARGON"} {"Type":"TOSS-UP\n\nMath - Short Answer","Prompt":"What is the 5\/3 power of -8?","Completion":"-32"} {"Type":"BONUS\n\nMath - Short Answer","Prompt":"A jar contains 9 coins: 8 fair coins and 1 double-headed coin. If a random coin is selected from the jar, is flipped, and comes up heads, what is the probability that the double-headed coin was the one selected?","Completion":"1\/5 (ACCEPT: 0.20, 20%)"} {"Type":"TOSS-UP\n\nEarth and Space - Multiple Choice","Prompt":"Where on the H-R diagram can one find red dwarf stars?\n\nW) Top right\n\nX) Bottom right\n\nY) Bottom left\n\nZ) Top left","Completion":"X) BOTOM RIGHT"} {"Type":"BONUS\n\nEarth and Space - Short Answer","Prompt":"Identify all of the following three positions on the celestial sphere that have a declination of zero: Summer solstice; Autumnal equinox; Winter solstice.","Completion":"2"} {"Type":"TOSS-UP\n\nPhysics - Multiple Choice","Prompt":"Which of the following best describes the shape of the graph of potential energy as a function of position for a constant force?\n\nW) Linear\n\nX) Quadratic\n\nY) Logarithmic\n\nZ) Exponential","Completion":"W) LINEAR"} {"Type":"TOSS-UP\n\nChemistry - Short Answer","Prompt":"Cyclohexene undergoes hydrogenation with hydrogen gas. What is the molecular formula of the product of the reaction?","Completion":"C6H12"} {"Type":"TOSS-UP\n\nEnergy - Multiple Choice","Prompt":"Oak Ridge National Lab scientists are studying bacteria that have penicillin resistance. Penicillin and similar drugs inhibit what process in the bacteria?\n\nW) DNA replication\n\nX) Translation\n\nY) Cell wall biosynthesis\n\nZ) Ribosome assembly","Completion":"Y) CELL WALL BIOSYNTHESIS"} {"Type":"BONUS\n\nEnergy - Short Answer","Prompt":"Scientists at Pacific Northwest National Lab are studying the production of lipid metabolites by fungi. These metabolites are produced by the oxidation of simpler lipids by cytochrome [SIGH-tow-krohm] P450 enzymes in what double-membrane-bound organelle?","Completion":"SMOOTH ENDOPLASMIC REticulUM (ACCEPT: ENDOPLASMIC REticulUM, ER; DO NOT ACCEPT: ROUGH ER)"} {"Type":"TOSS-UP\n\nBiology - Multiple Choice","Prompt":"What mammalian characteristic do Echidnas [eh-KID-nahs] lack?\n\nW) Hair\n\nX) Milk production\n\nY) Nipples\n\nZ) Temporal fenestra","Completion":"Y) NIPLES"} {"Type":"BONUS\n\nBiology - Multiple Choice","Prompt":"Spiracles on the surface of insects function as part of which of the following organ systems?\n\nW) Circulatory\n\nX) Digestive\n\nY) Reproductive\n\nZ) Respiratory","Completion":"Z) RESPIRATORY11) Earth and Space \\(-\\)Multiple Choice Increasing the silica content in rock melts results in an increase in the melt viscosity. Which of the following SI derived units would be used to describe melt viscosity?"} {"Type":"TOSS-UP\n\nEarth and Space - Short Answer","Prompt":"A large number of Kuiper [KYE-pur] Belt objects possess a 3-to-2 orbital resonance with what planet?","Completion":"NEPTUNE"} {"Type":"BONUS\n\nEarth and Space - Short Answer","Prompt":"Acid rain can result from gases released to the atmosphere by natural or anthropogenic[an-throw-poh-JEN-ic] activity. Identify all of the following three acids that are commonly found in acid rain: Hydrochloric acid; Nitric acid; Sulfuric acid.","Completion":"2, 3"} {"Type":"TOSS-UP\n\nChemistry - Short Answer","Prompt":"Rank the following four single bonds in increasing order of polarity: Carbon-carbon; Carbon-hydrogen; Nitrogen-hydrogen; Fluorine-hydrogen.","Completion":"1, 2, 3, 4"} {"Type":"BONUS\n\nChemistry - Short Answer","Prompt":"Identify all of the following three molecules that would be expected to have a delocalized pi [pie] bond system: Benzene;\n\n1,4-pentadiene [one-four-penta-DYE-een]; 1,3-butadiene [one-three-BYU-tah-DYE-een].","Completion":"1, 3"} {"Type":"TOSS-UP\n\nBiology - Short Answer","Prompt":"The bacterial cell wall is primarily composed of what polymer that consists of sugars cross-linked by short amino acid chains?","Completion":"PEPTIDOGLYCAN (ACCEPT: MUREIN)"} {"Type":"BONUS\n\nBiology - Short Answer","Prompt":"For a protein secreted by a eukaryotic cell, put the following three locations in chronological order from synthesis to secretion: Rough ER; Cytoplasm; Extracellular space.","Completion":"2, 1, 3"} {"Type":"TOSS-UP\n\nEnergy - Short Answer","Prompt":"Physicists at Oak Ridge National Lab have developed a spectroscopic [spek-roth-SCAW-pik] thermometer for nanomaterials and have mapped the temperature of a boron nitride [NYE-traide] nanocrystal to nanometer resolution. What hybridization shared by boron and nitrogen atoms in boron nitride gives it its unique hexagonal [hex-AG-uh-null] shape?","Completion":"SP\\({}^{2}\\)"} {"Type":"TOSS-UP\n\nChemistry - Short Answer","Prompt":"Rank the following three bonds in terms of increasing polarity:\n\nF-H; N-H; P-H.","Completion":"3, 2, 1"} {"Type":"TOSS-UP\n\nMath - Short Answer","Prompt":"Counting itself, what is the third-largest integer factor of 700?","Completion":"175"} {"Type":"TOSS-UP\n\nBiology - Short Answer","Prompt":"A common injury seen in soccer or football is a tear to the anterior cruciate ligament, also known as the ACL. What joint does this injury most directly affect?","Completion":"KNEE"} {"Type":"TOSS-UP\n\nEarth and Space - Short Answer","Prompt":"The Orion Nebula is an example of what type of nebula?","Completion":"EMISSION (ACCEPT: REFLECTION, EMISSION-REFLECTION)"} {"Type":"BONUS\n\nEarth and Space - Multiple Choice","Prompt":"Which of the following best describes why an active galaxy could become a normal galaxy?\n\nW) It stops rotating\n\nX) Star-formation ceases in the central bulge\n\nY) The supermassive black hole runs out of fuel\n\nZ) It collides with another galaxy","Completion":"Y) THE SUPERMASSIVE BLACK HOLE RUNS OUT OF FUEL"} {"Type":"TOSS-UP\n\nEnergy - Multiple Choice","Prompt":"Pacific Northwest National Lab scientists are depolymerizing lignin. In order to prevent the newly-cleaved lignin monomers from repolymerizing, they scavenge them with what type of molecule?\n\nW) Amine\n\nX) Alcohol\n\nY) Alkane\n\nZ) Alkyne [AL-kine]","Completion":"X) ALCOHOL"} {"Type":"BONUS\n\nEnergy - Short Answer","Prompt":"Scientists at the National Renewable Energy Lab are studying low-cost, highly absorptive solar cells that are made from thin films of compounds like\n\nmethylammonium [meth-il-ammonium] lead trihalides [try-HAY-lieds], a member of a group of materials having crystal structures resembling CaTiO\\({}{3}\\). What is this group of materials called?","Completion":"PEROVSKITES"} {"Type":"TOSS-UP\n\nPhysics - Multiple Choice","Prompt":"When radium-226 decays into radon-222, it emits an alpha particle as well as a gamma ray. Which of the following best describes this nuclear decay process?\n\nW) A unique type of double alpha-gamma decay\n\nX) The first alpha decay leaves the radon nucleus in an excited state\n\nY) Alpha emissions are always accompanied by gamma emissions\n\nZ) The alpha particle undergoes beta decay, emitting a photon","Completion":"X) THE FIRST ALPHA DECAY LEAVES THE RADON NUCLEUS IN AN EXCITED STATE"} {"Type":"BONUS\n\nPhysics - Short Answer","Prompt":"A 5-kilogram mass is attached to a spring, and its motion is\n\ndescribed by the function \\(x\\) of \\(t\\) equals cosine of \\(1.5t\\), where \\(t\\) is expressed in seconds. In newtons per meter, what is the spring constant of the spring?","Completion":"11.25"} {"Type":"TOSS-UP\n\nMath - Multiple Choice","Prompt":"A square can be cut into two congruent parts by how many\n\ndifferent coplanar lines?\n\nW) 2\n\nX) 4\n\nY) 8\n\nZ) An infinite number","Completion":"Z) AN INFINITE NUMBER"} {"Type":"BONUS\n\nMath - Short Answer","Prompt":"To the nearest degree, what is the measure of each interior angle of\n\na regular 17-sided polygon?","Completion":"159"} {"Type":"TOSS-UP\n\nChemistry \\(-\\) Short Answer","Prompt":"What is the oxidation state of nickel in nickel tetracarbonyl!\n\n[tetra-carbon-EEL]?","Completion":"0"} {"Type":"TOSS-UP\n\nBiology \\(-\\) Short Answer","Prompt":"In coelomates [SEEL-oh-mates], the body cavity is lined with tissue from what germ layer?","Completion":"MESODERM"} {"Type":"TOSS-UP\n\nPhysics - Multiple Choice","Prompt":"The proportionality [proportion-AL-ih-tee] constant in the ideal gas equation turns out to be almost equal for all real gases in which of the following conditions?\n\nW) High temperature, high pressure\n\nX) High temperature, low pressure\n\nY) Low temperature, high pressure\n\nZ) Low temperature, low pressure","Completion":"X) HIGH TEMPERATURE, LOW PRESSURE"} {"Type":"BONUS\n\nPhysics - Short Answer","Prompt":"Consider a yo-yo that is made by winding a string around a uniform disc of radius 1 centimeter. If the disc is allowed to fall vertically and unroll the string without slipping, then in terms of the acceleration \\(g\\) due to gravity, what is the acceleration of the yo-yo?","Completion":"2\\(g\\)\/3 (ACCEPT: 2\/3 \\(g\\))"} {"Type":"TOSS-UP\n\nChemistry - Multiple Choice","Prompt":"Which of the following best describes the orbital interactions when a cyanide ion performs nucleophilic addition into an aldehyde group?\n\nW) The HOMO of cyanide overlaps with the LUMO of the aldehyde on the carbon side\n\nX) The HOMO of cyanide overlaps with the LUMO of the aldehyde on the oxygen side\n\nY) The LUMO of cyanide overlaps with the HOMO of the aldehyde on the carbon side\n\nZ) The LUMO of the cyanide overlaps with the HOMO of the aldehyde on the oxygen side","Completion":"W) THE HOMO OF CYANIDE OVERLAPS WITH THE LUMO OF THE ALDEHYDE ON THE CARBON SIDE"} {"Type":"BONUS\n\nChemistry - Multiple Choice","Prompt":"A galvanic [gal-VAN-ik] cell has a copper anode and a silver cathode. Which of the following would decrease the voltage across the two electrodes?\n\nW) Increasing the concentration of Cu\\({}^{2+}\\) ions near the anode\n\nX) Increasing the concentration of Ag\\({}^{+}\\) ions near the cathode\n\nY) Increasing the area of the copper electrode\n\nZ) Increasing the electrolyte pH","Completion":"W) INCREASING THE CONCENTRATION OF Cu\\({}^{2+}\\) IONS NEAR THE ANODE15) Energy - Short Answer Scientists at Lawrence Berkeley National Lab are studying elements in eukaryotic DNA that bind to transcription factors, but are often thousands of base pairs away from the gene they control. What is the term for these elements?"} {"Type":"BONUS\n\nEnergy - Short Answer","Prompt":"Scientists at Princeton Plasma Physics Lab are growing nanotubes made of what element as potential transistor components?","Completion":"CARBON"} {"Type":"TOSS-UP\n\nMath - Short Answer","Prompt":"What is the y-intercept of the line with slope 4 that contains the point (3, -?","Completion":"-13 (ACCEPT: (0, -13))"} {"Type":"BONUS\n\nMath - Short Answer","Prompt":"Consider an infinite geometric series with sum equal to 11\/3 and first term 6\/5. What is the value of the common ratio \\(r\\) for this series?","Completion":"37\/5517) Earth and Space \\(-\\)Multiple Choice Which of the following is the most likely next event in the evolution of Betelgeuse [BAY-tel-jews]?"} {"Type":"## BONUS\n\nMath - Short Answer","Prompt":"Consider the graph given by the parametric equations \\(x=t\\)\\({}^{2}-2t\\) and \\(y=t+1\\). At what values of \\(y\\) will \\(x\\) equal 8?","Completion":"-1 AND 5"} {"Type":"## BONUS\n\nEarth and Space - Short Answer","Prompt":"Identify all of the following three plate boundary types that are present at mid-ocean ridges: Convergent; Divergent; Transform.","Completion":"2, 3"} {"Type":"TOSS-UP\n\nChemistry - Short Answer","Prompt":"Identify all of the following three compounds that contain both covalent and ionic bonds: Sodium nitrate; Hydrogen peroxide; Lithium chloride.","Completion":"1"} {"Type":"BONUS\n\nChemistry - Short Answer","Prompt":"What is the average kinetic energy, to the nearest tenth of a kilojoule per mole, of argon at 300 kelvins?","Completion":"3.7"} {"Type":"TOSS-UP\n\nPhysics - Short Answer","Prompt":"In 1932, Carl Anderson noticed a set of tracks in his cloud chamber that were similar to those created by electrons, but curving the other way. These particles are now called what?","Completion":"POSITRONS (ACCEPT: ANTI-ELECTRONS)"} {"Type":"BONUS\n\nPhysics - Short Answer","Prompt":"A \\(2i+3j\\) force acts at a position -\\(i\\) - \\(4j\\) relative to a point P. In vector form, what is the torque [tork] at point P?","Completion":"\\(5k\\)23) Biology - Short Answer C4 photosynthesis is thought to be an adaptation that bypasses what pathway, by which RuBisCO [foo-BIS-koh] oxidizes RuBP [R-U-B-P] rather than carboxolating it?"} {"Type":"TOSS-UP\n\nMath - Short Answer","Prompt":"To the nearest whole number, what is the slant height of a right circular cone with radius 7 and height 6?","Completion":"9"} {"Type":"TOSS-UP\n\nBiology - Short Answer","Prompt":"Diastole [dye-ASS-tuh-lee] and systole [SIS-tuh-lee] correspond to the phases of what organ?","Completion":"HEART"} {"Type":"TOSS-UP\n\nChemistry - Short Answer","Prompt":"Rank the following three aqueous solutions in terms of increasing electrolyte strength: One-molar sucrose; One-molar hydrochloric acid; One-molar acetic acid.","Completion":"1, 3, 2"} {"Type":"TOSS-UP\n\nEarth and Space - Multiple Choice","Prompt":"Arkose contains quartz and at least 25 percent of what other mineral group?\n\nW) Carbonate\n\nX) Feldspar\n\nY) Mica [MY-kah]\n\nZ) Clay","Completion":"X) FELDSPAR"} {"Type":"## BONUS\n\nEnergy - Multiple Choice","Prompt":"Scientists at Ames Lab have developed solvent-free techniques to synthesize alkali metal hydrides. Which of the following techniques do they use to unambiguously determine that the hydrogen atoms have bound to the metal?\n\nW) Gravimetric analysis\n\nX) Infrared spectroscopy\n\nY) UV\/Vis spectroscopy\n\nZ) Solid-state NMR spectroscopy","Completion":"Z) SOLID-STATE NMR SPECTROSCOPY"} {"Type":"TOSS-UP\n\nMath - Short Answer","Prompt":"If the definite integral from 2 to 6 of \\(f\\)of \\(x\\) d\\(x\\) = 26, then what is the definite integral from 2 to 6 of \\(f\\)of \\(x\\) minus 4 d\\(x\\)?","Completion":"10"} {"Type":"## BONUS\n\nMath - Short Answer","Prompt":"Rounded to the nearest integer, what is the geometric mean of 6 and 85?","Completion":"239) Earth and Space - Multiple Choice Which of the following values, in kelvins, is closest to the temperature of the Sun's photosphere?"} {"Type":"TOSS-UP\n\nBiology - Short Answer","Prompt":"Of what class are sharks and rays members?","Completion":"CHONDRICHTHYES [KON-dirik-theez]"} {"Type":"TOSS-UP\n\nBiology - Short Answer","Prompt":"In rod cells, what protein is photobleached when exposed to light?","Completion":"RHODOPSIN"} {"Type":"BONUS\n\nBiology - Short Answer","Prompt":"Several important evolutionary features only evolved once in history. Identify all of the following three features that are known to have evolved a single time: C4 photosynthesis; Ammiotic egg; Feathers.","Completion":"2, 3"} {"Type":"TOSS-UP\n\nEnergy - Short Answer","Prompt":"Scientists at Fermi National Accelerator Lab are attempting to detect WIMPs as a part of the SuperCDMS experiment. WIMPs are particles that are thought to primarily interact with ordinary matter via what two fundamental forces?","Completion":"GRAVITY AND WEAK (ACCEPT: GRAVITY AND WEAK NUCLEAR, GRAVITY AND WEAK INTERACTION)"} {"Type":"BONUS\n\nEnergy - Short Answer","Prompt":"Scientists at Pacific Northwest National Lab are developing methods to engineer the rhizosphere [RYE-zoh-feger]. This is the area of soil that interacts with what part of the plant?","Completion":"ROOTS15) Earth and Space \\(-\\)Multiple Choice A stream's competence is a measure of which of the following?"} {"Type":"TOSS-UP\n\nPhysics \\(-\\) Multiple Choice","Prompt":"Which of the following capacitor configurations for four identical capacitors would yield the largest stored charge for a given voltage?\n\nW) All four capacitors connected in series\n\nX) Two parallel branches of two series capacitors\n\nY) Three capacitors in parallel, in series to a fourth\n\nZ) All four capacitors connected in parallel","Completion":"Z) ALL FOUR CAPACITORS CONNECTED IN PARALLEL"} {"Type":"TOSS-UP\n\nChemistry - Short Answer","Prompt":"What is the bond order of the diatomic species N\\({}{2}\\) two minus?","Completion":"TWO"} {"Type":"BONUS\n\nChemistry - Short Answer","Prompt":"Rank the following three nuclides in terms of increasing binding energy per nucleon: Deuterium [doo-TEER-ee-um]; Helium-4; Lithium-6.","Completion":"1, 3, 2"} {"Type":"TOSS-UP\n\nMath - Short Answer","Prompt":"The science and chess clubs at a school have a total of 60 students in them. If 37 are in the science club and 28 are in the chess club, how many are in both?","Completion":"5"} {"Type":"BONUS\n\nMath - Short Answer","Prompt":"What is the slope-intercept equation of the oblique asymptote of the function \\(f\\) of \\(x\\) equals the fraction with numerator \\(2x^{2}\\) - 5 and denominator \\(x+3\\)?","Completion":"\\(y=2x-6\\)"} {"Type":"TOSS-UP\n\nBiology - Short Answer","Prompt":"In the lac operon in E. coli, CAP acts as an activator protein. What second messenger must bind to CAP to activate transcription of the DNA?","Completion":"CYCLIC AMP (ACCEPT: cAMP)"} {"Type":"BONUS\n\nBiology - Multiple Choice","Prompt":"John Gurdon studied toitpotency [toh-TIH-path-ten-see] by implanting a nucleus from a fully-differentiated frog cell into enucleated egg cells. Which of the following best describes what he observed about the egg?\n\nW) It continued to divide, but stopped development prior to forming a tadpole\n\nX) It continued to divide and developed into a tadpole as normal\n\nY) It did not begin to divide after nucleation\n\nZ) It continued to divide, but skipped the tadpole stage and formed a frog","Completion":"W) IT CONTINUED TO DIVIDE, BUT STOPPED DEVELOPMENT PRIOR TO FORMING A TADPOLE"} {"Type":"TOSS-UP\n\nChemistry - Short Answer","Prompt":"What named pericyclic reaction is classically the reaction of a di-ene [DYE-een] with a dienophile [dye-EENOH-file] in a single, concerted step?","Completion":"DIELS-ALDER REACTION"} {"Type":"BONUS\n\nChemistry - Short Answer","Prompt":"Identify all of the following three changes to an organic reaction that would increase its rate constant: Decreasing the energy of the nucleophile HOMO; Decreasing the energy of the electrophile LUMO; Decreasing the overlap between nucleophile HOMO and electrophile LUMO.","Completion":"2"} {"Type":"TOSS-UP\n\nEarth and Space - Short Answer","Prompt":"Astronomers sometimes notice that spectral lines can be broadened due to the presence of strong magnetic fields. What is the name for this effect?","Completion":"ZEEMAN EFFECT"} {"Type":"BONUS\n\nEarth and Space - Short Answer","Prompt":"Arrange the following three atmospheric boundaries in order of increasing pressure: Mesopause; Tropopause; Stratopause.","Completion":"1, 3, 2"} {"Type":"TOSS-UP\n\nMath - Short Answer","Prompt":"The line with equation \\(y=2x-7\\) is reflected across the \\(y\\)-axis. What is the slope-intercept equation of its image?","Completion":"\\(y=-2x-7\\)"} {"Type":"BONUS\n\nMath - Short Answer","Prompt":"How many two-digit positive integers have the tens digit strictly less than the ones digit?","Completion":"3623) Physics - Short Answer What principle posits the requirement that the laws of physics must be equivalent in all inertial [in-UR-shul] reference frames?"} {"Type":"TOSS-UP\n\nEarth and Space - Short Answer","Prompt":"What was the most abundant atmospheric gas during the Proterozoic [proh-tair-oh-ZOH-ik] Eon?","Completion":"NITROGEN (ACCEPT: DINITROGEN, N\\({}{2}\\))"} {"Type":"BONUS\n\nEarth and Space - Multiple Choice","Prompt":"A transgressive stratigraphic sequence can be related to which of the following events?\n\nW) Sea-level rise\n\nX) Orogeny [or-4W-jeh-nee]\n\nY) Oxbow lake formation\n\nZ) Significant rainfall events","Completion":"W) SEA-LEVEL RISE"} {"Type":"TOSS-UP\n\nChemistry - Short Answer","Prompt":"Marie places 4.4 grams of dry ice in a vacuum-sealed 5-liter bag at standard temperature. After the dry ice reaches standard temperature, then, to the nearest tenth of a liter, how much volume will it fill?","Completion":"2.2"} {"Type":"BONUS\n\nChemistry - Short Answer","Prompt":"Identify all of the following three molecules that typically undergo an Sn1 reaction in the presence of a nucleophile: Methyl-bromide;\n\n2-bromo-propane; Tert-butyl-amine [turt-BYOO-til-ah-MEEN].","Completion":"2"} {"Type":"TOSS-UP\n\nEnergy - Multiple Choice","Prompt":"Scientists at Los Alamos National Lab are measuring the positron flux in Earth's upper atmosphere. Which of the following is NOT TRUE concerning positrons?\n\nW) They decay into muons\n\nX) They have a positive charge\n\nY) They are first-generation leptons\n\nZ) They can interact with electrons to generate two gamma rays","Completion":"W) THEY DECAY INTO MUONS"} {"Type":"BONUS\n\nEnergy - Multiple Choice","Prompt":"Lawrence Berkeley National Lab scientists are studying the teeth of parrotfish due to their unique structure. Parrotfish can be classified best as which of the following?\n\nW) Lobe-finned fish\n\nX) Hagfish\n\nY) Ray-finned fish\n\nZ) Sharks","Completion":"Y) RAY-FINNED FISH"} {"Type":"TOSS-UP\n\nPhysics - Short Answer","Prompt":"Consider a circuit in which two light bulbs with equal resistance values are connected in parallel. If one of the light bulbs burns out, by what factor is the brightness of the other bulb multiplied?","Completion":"ONE"} {"Type":"BONUS\n\nPhysics - Short Answer","Prompt":"Consider a 200-newton-meter torque [tork] that acts on a solid disk of radius one meter and mass 100 kilograms. What is the angular velocity of the disk after 5 seconds, in radians per second?","Completion":"20"} {"Type":"TOSS-UP\n\nMath - Multiple Choice","Prompt":"For a conic section, which of the following values of eccentricity would indicate that the conic section is an ellipse?\n\nW) 0\n\nX) 0.5\n\nY) 1\n\nZ) 1.5","Completion":"X) 0.5"} {"Type":"BONUS\n\nMath - Short Answer","Prompt":"What are the coordinates of the image of the point (-1, when it is reflected through the line \\(y=x-4\\)?","Completion":"(9, -5) (ACCEPT: \\(x=9\\) AND \\(y=-5\\))"} {"Type":"TOSS-UP\n\nBiology - Short Answer","Prompt":"What type of connective tissue links muscles to bones?","Completion":"TENDON"} {"Type":"BONUS\n\nBiology - Short Answer","Prompt":"Identify all of the following three statements that are true of SDS-PAGE: It requires addition of antioxidant to the inner gel chamber; It requires protein denaturation using urea; It relies on SDS binding proportional to protein mass.","Completion":"1, 3"} {"Type":"TOSS-UP\n\nEarth and Space - Short Answer","Prompt":"What type of sub-stellar object can fuse deuterium [doo-TEER-ee-um] but not hydrogen?","Completion":"BROWN DWARF"} {"Type":"BONUS\n\nEarth and Space - Short Answer","Prompt":"An asteroid revolves around the Sun every 64 Earth-years. To the nearest whole astronomical unit, what is the semimajor axis of the asteroid?","Completion":"16"} {"Type":"TOSS-UP\n\nPhysics - Short Answer","Prompt":"Similar to his famous gold foil experiment, Rutherford bombarded nitrogen-14 atoms with alpha particles, and observed that each event released a proton. Into what isotope was the nitrogen converted?","Completion":"OXYGEN-17"} {"Type":"BONUS\n\nPhysics - Short Answer","Prompt":"A certain steel guitar has strings of length 1 meter and cross sectional area 2 times \\(10^{-6}\\) square meter. If the Young's modulus of steel is 2 times \\(10^{11}\\) newtons per square meter, then in newtons, what is the tension in the string when it is stretched 0.02 meter for tuning?","Completion":"8000"} {"Type":"TOSS-UP\n\nBiology - Multiple Choice","Prompt":"The organisms that form arbuscular mycorrhizalae are part of what group of fungi?\n\nW) Chytrids [KIH-trids]\n\nX) Glomeromycetes [glo-mair-oh-MY-seets]\n\nY) Basidiomycetes [bah-sid-ee-oh-MY-seet]\n\nZ) Ascomycetes","Completion":"X) GLOMEROMYCETES"} {"Type":"BONUS\n\nBiology - Short Answer","Prompt":"Starting with the fixation of atmospheric nitrogen and ending with the assimilation of nitrogen compounds by plants, order the following three ions of the nitrogen cycle chronologically: Ammonium; Nitrate; Nitrite.","Completion":"1, 3, 2"} {"Type":"TOSS-UP\n\nMath - Multiple Choice","Prompt":"The graph of a continuous function \\(f\\) contains the points (3, -and (8, -. The intermediate value theorem guarantees an x-value such that \\(f\\) of \\(x\\) equals which of the following?\n\nW) -5\n\nX) -2\n\nY) 2\n\nZ) 5","Completion":"W) -5"} {"Type":"BONUS\n\nMath - Short Answer","Prompt":"How many zeroes occur at the right of the number of permutations of 60 objects taken 20 at a time?","Completion":"511) Energy - Short Answer Researchers at Pacific Northwest National Lab are studying O-linked glycosylation [glove-law-sil-AY-shun] of the tau protein. This post-translational process plays a role in the pathogenesis [path-oh-JEN-eh-sis] of what human disease?"} {"Type":"TOSS-UP\n\nBiology - Multiple Choice","Prompt":"Which of the following organisms is the primary infection target of the virus T2?\n\nW) E. coli\n\nX) S. cerevisiae[sarah-VTH-see-ay]\n\nY) D. melanogaster[meh-LAN-oh-gas-tur]\n\nZ) M. musculus","Completion":"W) E. COLI"} {"Type":"BONUS\n\nBiology - Short Answer","Prompt":"What microsatellite genetic markers do forensic scientists typically use for crime scene DNA analysis?","Completion":"STRs (ACCEPT: SHORT TANDEM REPEATS)"} {"Type":"TOSS-UP\n\nMath - Short Answer","Prompt":"Find the value of \\(a\\) for which the following vectors are perpendicular: \\(9i-3j\\) and \\(ai+36j\\).","Completion":"12"} {"Type":"BONUS\n\nMath - Short Answer","Prompt":"The graph of \\(f\\) of \\(x=\\) the fraction with numerator \\(x^{2}-2x-24\\) and denominator \\(x^{2}-16\\) has a removable discontinuity at \\(x=a\\) for some real number \\(a\\). What is the limit as \\(x\\) approaches \\(a\\) of \\(f\\) of \\(x\\)?","Completion":"5\/4 (ACCEPT: 1\\(\\%\\), 1.25)15) Energy - Short Answer Scientists at Brookhaven National Lab use the Relativistic Heavy Ion Collider to study a state of matter thought to have also existed in the early universe. This matter is composed of quarks and what other strongly-interacting fundamental particles?"} {"Type":"## BONUS\n\nEnergy - Short Answer","Prompt":"Researchers at Princeton Plasma Physics Lab are studying the Alpha Centauri star system. This system contains how many stars?","Completion":"THREE"} {"Type":"## BONUS\n\nPhysics - Short Answer","Prompt":"One mole of an ideal monatomic gas has a net energy change of 1245 joules. To one significant figure and in degrees Celsius, how much does its temperature change?","Completion":"10017) Earth and Space \\(-\\)Multiple Choice What type of drainage pattern will develop on bedrock that is highly jointed?"} {"Type":"BONUS\n\nEarth and Space \\(-\\) Multiple Choice","Prompt":"In which of the following locations would histosols most likely be found?\n\nW) Desert\n\nX) Peat bog\n\nY) Boreal [BOAR-ee-ul] forest\n\nZ) Subhumid climate","Completion":"X) PEAT BOG"} {"Type":"TOSS-UP\n\nChemistry \\(-\\) Multiple Choice","Prompt":"X-rays with a wavelength of 150 picometers are scattered at an angle of 15 degrees by a crystal. Assuming that this was first-degree scattering, which of the following is closest to the spacing in this crystal, in picometers?\n\nW) 145\n\nX) 155\n\nY) 290\n\nZ) 589","Completion":"Y) 290"} {"Type":"BONUS\n\nChemistry \\(-\\) Short Answer","Prompt":"Identify all of the following three numbers of \\(\\pi\\)[pie]-electrons in a conjugated planar cyclic system that would make that system aromatic: Four; Five; Six.","Completion":"319) Math - Short Answer A prism with 24 edges has how many faces?"} {"Type":"## BONUS\n\nMath - Short Answer","Prompt":"For the arithmetic sequence with first term 2 and fifth term 22, what is the sum of the first 60 terms?","Completion":"8970"} {"Type":"TOSS-UP\n\nEarth and Space - Short Answer","Prompt":"What stellar spectral class tends to have the strongest singly-ionized helium absorption lines?","Completion":"O [Oh]"} {"Type":"## BONUS\n\nEarth and Space - Short Answer","Prompt":"The C-N-O process in massive stars is responsible for fusing what element into heavier nuclides?","Completion":"HYDROGEN"} {"Type":"TOSS-UP\n\nPhysics - Short Answer","Prompt":"What model of the nucleus, developed by George Gamow [GAMM-off], suggested that nucleons were held together primarily by short-range interactions?","Completion":"LIQUID DROP MODEL"} {"Type":"BONUS\n\nPhysics - Short Answer","Prompt":"As the number of degrees of freedom of motion of an ideal gas increases, what does its adiabatic [ad-ee-ah-BAT-ik] index asymptotically [ay-sim-TOT-ik-lee] tend to?","Completion":"ONE"} {"Type":"TOSS-UP\n\nChemistry - Short Answer","Prompt":"Identify all of the following three chemical reactions that are likely to be spontaneous at room temperature: Solid sodium combines with chlorine gas to form solid sodium chloride; Gaseous nitrogen combines with gaseous hydrogen to form gaseous ammonia;\n\nSolid potassium iodide dissolves into potassium and iodide ions.","Completion":"1, 3"} {"Type":"BONUS\n\nChemistry - Short Answer","Prompt":"Consider a reaction that is nonspontaneous at 300 kelvins, but becomes spontaneous when the temperature is raised to 400 kelvins. Identify all of the following three values that are positive for this reaction at 300 kelvins: Delta-H; Delta-S; Delta-G.","Completion":"1, 2, AND 3 (ACCEPT: ALL)"} {"Type":"TOSS-UP\n\nBiology - Short Answer","Prompt":"If the sinoatrial [SIGH-no-AY-tree-ul] node fails, what tissue will be responsible for setting the pace of the heart?","Completion":"AV NODE (ACCEPT: ATRIOVENTRICULAR NODE)"} {"Type":"BONUS\n\nBiology - Short Answer","Prompt":"Identify all of the following three statements that are true of eukaryotic mRNA processing: The 5-prime end is capped with a modified cytosine [sigh-toh-seen] residue; The 5-prime end is capped in the nucleus; The 3-prime end is adenylated in the cytosol.","Completion":"2"} {"Type":"TOSS-UP\n\nEarth and Space \\(-\\) Multiple Choice","Prompt":"Which of the following best describes the mass of Ceres [SEER-eez] in terms of a percentage of the mass of our Moon?\n\nW) 0.001\n\nX) 1\n\nY) 100\n\nZ) 200","Completion":"X) 1"} {"Type":"TOSS-UP\n\nMath \\(-\\) Multiple Choice","Prompt":"A and \\(B\\) are statements, and \\(B\\) is true. Which of the following is true regarding the statement \"A implies \\(B\\)?\"\n\nW) It is true regardless of the truth value of \\(A\\)\n\nX) It is false regardless of the truth value of \\(A\\)\n\nY) It is true only when \\(A\\) is true, and false otherwise\n\nZ) It is true only when \\(A\\) is false, and false otherwise","Completion":"W) IT IS TRUE REGARDLESS OF THE TRUTH VALUE OF \\(A\\)"} {"Type":"TOSS-UP\n\nBiology - Short Answer","Prompt":"What substance, which is produced by parietal [path-RYE-eth-ul] cells in the stomach, is needed for the uptake of vitamin B12?","Completion":"INTRINSIC FACTOR (ACCEPT: GASTRIC INTRINSIC FACTOR)"} {"Type":"BONUS\n\nBiology - Short Answer","Prompt":"Identify all of the following three surface receptors whose synthesis depends on VDJ recombination: Toll-like receptor; T-cell receptor; B-cell receptor.","Completion":"2, 3"} {"Type":"TOSS-UP\n\nEnergy - Multiple Choice","Prompt":"Department of Energy and Argonne National Lab researchers have recently found that termolecular [ter-mo-lecular] reactions may play a significant role in a number of processes. Why had chemists previously considered termolecular reactions to be insignificant?\n\nW) Termolecular reactions have high changes in entropy\n\nX) Termolecular reactions are usually endothermic\n\nY) Termolecular reactions are forbidden by the Woodward-Hoffmann rules\n\nZ) Termolecular reactions are statistically unlikely to occur","Completion":"Z) TERMOELULAR REACTIONS ARE STATISTICALLY UNLIKELY TO OCCUR"} {"Type":"BONUS\n\nEnergy - Multiple Choice","Prompt":"DOE researchers recently detected the simultaneous arrival on Earth of the gravitational waves and the light emitted from a distant neutron star merger. This simultaneous arrival implies that, if a graviton particle exists, it must have a mass most similar to which of the following particles?\n\nW) Top quark\n\nX) Proton\n\nY) Neutron\n\nZ) Photon","Completion":"Z) PHOTON7) Chemistry - Short AnswerArrange the following four diatomic molecules in order of increasing bond length: 1) N\\({}{2}\\); 2) Cl\\({}{2}\\); 3) F\\({}{2}\\); 4) O\\({}{2}\\)."} {"Type":"TOSS-UP\n\nMath - Short Answer","Prompt":"What is the dot product of the vectors \\(3\\mathbf{i}-2\\mathbf{j}+\\mathbf{k}\\) and \\(4\\mathbf{i}-\\mathbf{j}\\)?","Completion":"14"} {"Type":"TOSS-UP\n\nBiology - Short Answer","Prompt":"What stage of embryonic development that follows the morula [MOOR-yoo-lah] is characterized as a fluid-filled sphere?","Completion":"BLASTULA"} {"Type":"TOSS-UP\n\nBiology - Multiple Choice","Prompt":"Which of the following diseases is caused by an apicomplexan [appee-com-PLEX-en]?\n\nW) Tuberculosis\n\nX) Malaria\n\nY) Chagas [SHAH-gus] disease\n\nZ) AIDS","Completion":"X) MALARIA"} {"Type":"TOSS-UP\n\nPhysics - Short Answer","Prompt":"A light wave is propagating along the \\(x\\)-axis in the positive\n\ndirection. If the electric field is currently oriented toward the positive \\(z\\)-axis, then expressing\n\nyour answer as a sign and an axis, which way is the magnetic field oriented?","Completion":"NEGATIVE \\(y\\)-AXIS"} {"Type":"BONUS\n\nChemistry - Short Answer","Prompt":"Identify all of the following three statements that describe the dimerization [dye-mur-ih-Z4Y-shun] reaction of butadiene [byoo-tah-DYE-een]: It is first-order; It is entropically [en-TRAW-pik-ah-lee] favored; The half-life increases over time.","Completion":"3"} {"Type":"TOSS-UP\n\nEarth and Space - Multiple Choice","Prompt":"Which of the following statements is true of main sequence stars?\n\nW) Class A stars are the hottest\n\nX) Less-massive stars have longer lifetimes than more-massive stars\n\nY) Less-massive stars have higher luminosities than more-massive stars\n\nZ) Red supergiants are the coolest main-sequence stars","Completion":"X) LESS-MASSIVE STARS HAVE LONGER LIFETIMES THAN MORE-"} {"Type":"BONUS\n\nEarth and Space - Short Answer","Prompt":"A doublet of strong yellow D lines in the spectrum of the Sun are absorption lines stemming from what element?","Completion":"SODIUM17) Math - Short AnswerWhat is the limit as \\(x\\) approaches one of the fraction with numerator \\(x^{2}-9\\) and denominator \\(x^{3}-27\\)?"} {"Type":"TOSS-UP\n\nPhysics - Short Answer","Prompt":"Which of Maxwell's equations directly enforces the postulate that magnetic monopoles do not exist?","Completion":"GAUSS'S LAW FOR MAGNETISM (DO NOT ACCEPT: GAUSS'S LAW)"} {"Type":"BONUS\n\nPhysics - Short Answer","Prompt":"An object is undergoing one-dimensional motion at time \\(t=0\\). If its velocity equals 3 times the exponential of -2\\(t\\), then, as times increases, what value does the total displacement of the object approach?","Completion":"3\/2 (ACCEPT: 1.5, 1\\(\\%\\))"} {"Type":"TOSS-UP\n\nMath - Short Answer","Prompt":"What is the sum of the zeroes of the polynomial \\(2x^{3}+11x^{2}+2x-16\\)?","Completion":"-11\/2 (ACCEPT -5.5, -5 1\/2)"} {"Type":"BONUS\n\nMath - Short Answer","Prompt":"What is the largest prime factor of the quantity \\(5^{2016}+5^{2017}+5^{2018}+5^{2019}\\)?","Completion":"13"} {"Type":"TOSS-UP\n\nBiology - Short Answer","Prompt":"The synapsids and the reptiles were able to adapt to life on land due to what key evolutionary innovation?","Completion":"AMNIOTIC EGG"} {"Type":"TOSS-UP\n\nChemistry - Multiple Choice","Prompt":"In a polyelectronic atom, which of the following orbitals would have the lowest energy?\n\nW) 3s\n\nX) 4s\n\nY) 3p\n\nZ) 3d","Completion":"W) 3s"} {"Type":"TOSS-UP\n\nMath - Short Answer","Prompt":"What is \\(f\\) prime of 1 for the function \\(f\\) of \\(x=x\\) times \\(e^{x}\\)?","Completion":"2\\(e\\)"} {"Type":"TOSS-UP\n\nBiology - Short Answer","Prompt":"What structure in a snail is responsible for secreting its shell?","Completion":"MANTLE"} {"Type":"BONUS\n\nBiology - Short Answer","Prompt":"In a retrospective study comparing smoking to lung cancer, half of smokers had cancer, while only a quarter of non-smokers had cancer. What is the odds ratio of the association between smoking and cancer?","Completion":"3 (ACCEPT: THREE TO ONE)"} {"Type":"TOSS-UP\n\nEarth and Space - Short Answer","Prompt":"What is the name given to asteroids that share an orbit with a larger planet, and lead or trail the planet by 60 degrees?","Completion":"TROJANS"} {"Type":"BONUS\n\nEarth and Space - Short Answer","Prompt":"The rate of solar rotation was originally determined by observing the movement of what features?","Completion":"SUNSPOTS"} {"Type":"TOSS-UP\n\nPhysics - Short Answer","Prompt":"According to band gap theory, in an insulator at absolute zero, what is the highest band with any electrons in it?","Completion":"VALENCE"} {"Type":"BONUS\n\nPhysics - Short Answer","Prompt":"Nick, who has a mass of 40 kilograms, is inflating his bike tires with a foot-pedal pump that extends 50 centimeters at a 60-degree incline to the ground. If he steps so that his entire weight acts vertically onto the tip of the pedal, then in newton-meters, what is the magnitude of the initial torque [tork] on the pump?","Completion":"98"} {"Type":"TOSS-UP\n\nEnergy - Multiple Choice","Prompt":"Researchers at SLAC National Accelerator Laboratory have developed a device to produce electromagnetic radiation with a frequency in the terahertz range. Between what two classifications of radiation does terahertz radiation fall?\n\nW) Gamma rays and X-rays\n\nX) X-rays and ultraviolet light\n\nY) Ultraviolet light and infrared light\n\nZ) Infrared light and microwaves","Completion":"Z) INFRARED LIGHT AND MICROWAVES"} {"Type":"BONUS\n\nEnergy - Short Answer","Prompt":"Scientists at Brookhaven National Lab are studying Cooper pairs in iron-selenide superconductors. Cooper pairs are pairs of what subatomic particles?","Completion":"ELECTRONS7) Math - Short Answer In a quadruple-elimination tournament, each game is played by two teams, there are no ties, and teams are eliminated when they have lost 4 times. What is the maximum number of games played if 41 teams enter?"} {"Type":"BONUS\n\nEarth and Space - Short Answer","Prompt":"Identify all of the following three characteristics that describe a typical neritic zone in the ocean water column: Oxygen-poor; Photo;\n\nNear-shore.","Completion":"2, 3"} {"Type":"TOSS-UP\n\nChemistry - Multiple Choice","Prompt":"When an alkene [al-KEEN] acts as a nucleophile, what specific part of the alkene is nucleophilic?\n\nW) The higher-energy carbon atom\n\nX) The lower-energy carbon atom\n\nY) The carbon-carbon sigma bond\n\nZ) The carbon-carbon pi [pie] bond","Completion":"Z) THE CARBON-CARBON PI [pie] BOND"} {"Type":"BONUS\n\nChemistry - Short Answer","Prompt":"N\\({}{2}\\)O is in equilibrium with N\\({}{2}\\)O\\({}{4}\\) in a transparent cylinder\n\nfitted with a piston. Identify all of the following three changes that would increase the number of N\\({}{2}\\)O molecules in the cylinder: Depressing the piston; Increasing the temperature; Adding nitrogen gas.","Completion":"211) Energy - Multiple Choice Lawrence Berkeley National Lab scientists are studying the inflammasomeome, which is a structure that forms around flagellin. This protein is involved in the immune response to which of the following?"} {"Type":"TOSS-UP\n\nMath \\(-\\) Short Answer","Prompt":"The altitude to the hypotenuse of a right triangle divides the hypotenuse into segments of length 5 and 4. What is the length of the altitude?","Completion":"2\\(\\backslash\\)5"} {"Type":"TOSS-UP\n\nPhysics - Multiple Choice","Prompt":"The Chernobyl disaster occurred, in part, due to the buildup of xenon-135[ZEE-non one thirty five] in the core. Which of the following best describes the role xenon-135 played in the core?\n\nW) Neutron emitter\n\nX) Control rod\n\nY) Neutron poison\n\nZ) Catalyst","Completion":"Y) NEUTRON POISON"} {"Type":"BONUS\n\nPhysics - Short Answer","Prompt":"Identify all of the following three processes that would increase the entropy of an ideal gas: Reversible isothermal expansion; Reversible\n\nadiabatic [ad-ee-ah-BAT-ik] expansion; Free expansion against zero pressure.","Completion":"1, 3"} {"Type":"TOSS-UP\n\nEnergy - Short Answer","Prompt":"Scientists at Idaho National Lab have developed a new method to refine biodiesel. This novel process uses a carbon dioxide solvent in which the CO\\({}{2}\\) is in what specific phase?","Completion":"SUPERCRITICAL FLUID (ACCEPT: SUPERCRITICAL)"} {"Type":"BONUS\n\nEnergy - Short Answer","Prompt":"Scientists at Brookhaven National Lab are part of the ATLAS experiment at CERN, which recently observed light-by-light scattering, where light at high energies can be scattered by what particle?","Completion":"PHOTON"} {"Type":"TOSS-UP\n\nBiology - Multiple Choice","Prompt":"In embryonic development, the archenteron is a precursor to what?\n\nW) Gut\n\nX) Brain\n\nY) Skeletal musculature\n\nZ) Skin","Completion":"W) GUT"} {"Type":"TOSS-UP\n\nChemistry - Multiple Choice","Prompt":"Which of the following electron configurations is correct for the aqueous iron 2+ ion?\n\nW) [Ar] 4s\\({}^{2}\\) 3d\\({}^{4}\\)\n\nX) [Ar] 4s\\({}^{1}\\) 3d\\({}^{5}\\)\n\nY) [Ar] 3d\\({}^{6}\\)\n\nZ) [Ar] 3d\\({}^{5}\\) 4p\\({}^{1}\\)","Completion":"Y) [Ar] 3d\\({}^{6}\\)"} {"Type":"TOSS-UP\n\nMath - Short Answer","Prompt":"In base 7, what is 4 times 6?","Completion":"THREE-THREE (ACCEPT: 33)"} {"Type":"BONUS\n\nMath - Short Answer","Prompt":"What is the cross product of vectors \\(3\\mathbf{i}+2\\mathbf{j}+\\mathbf{k}\\) cross \\(\\mathbf{j}\\)?","Completion":"-\\(\\mathbf{i}+3\\mathbf{k}\\) (ACCEPT: -\\(\\mathbf{i}+0\\mathbf{j}+3\\mathbf{k},-1\\mathbf{i}+3\\mathbf{k},-1\\mathbf{i}+0\\mathbf{j}+3 \\mathbf{k}\\)"} {"Type":"TOSS-UP\n\nBiology - Short Answer","Prompt":"Identify all of the following three structures that are homologous [ha-MAWL-uh-gus] to a human hand: Whale flipper; Bat wing; Bird claw.","Completion":"1, 2"} {"Type":"BONUS\n\nBiology - Short Answer","Prompt":"Identify all of the following three groups of organisms that are protostomes: Snails; Sea cucumbers; Earthworms.","Completion":"1, 321) Physics - Short Answer Kaons [KAY-ons] and pions [PIE-ons] are classified as what type of composite particle that consists of a quark and an anti-quark?"} {"Type":"TOSS-UP\n\nChemistry - Short Answer","Prompt":"What are the two possible molecular geometries for a five-coordinate transition metal compound?","Completion":"TRIGONAL BIPYRAMIDAL AND SQUARE BIPYRAMIDAL"} {"Type":"TOSS-UP\n\nEarth and Space \\(-\\) Multiple Choice","Prompt":"Which of the following stars has a positive apparent magnitude?\n\nW) Polaris\n\nX) Sirius\n\nY) Canopus\n\nZ) The Sun","Completion":"W) POLARIS"} {"Type":"TOSS-UP\n\nMath \\(-\\) Short Answer","Prompt":"A set of test scores is normally distributed with mean 62 and standard deviation 8. What score corresponds to a Z-score of -1.25?","Completion":"52"} {"Type":"TOSS-UP\n\nChemistry - Short Answer","Prompt":"Rank the following three atoms in terms of increasing atomic radius: Argon; Fluorine; Lithium.","Completion":"2, 1, 3"} {"Type":"BONUS\n\nChemistry - Short Answer","Prompt":"Gripard [GRIN-yard] reagents are generally made in ether-based solvents due to the instability of the metal-organic species. Identify all of the following three solvents that are considered ethereal: Dioxane;\n\nTetrahydrofuran [tetrah-high-droh-FYOOR-an]; Dichloromethane.","Completion":"1, 2"} {"Type":"TOSS-UP\n\nPhysics - Multiple Choice","Prompt":"An electron is moving along the \\(x\\)-axis in the positive direction, within a uniform magnetic field that is oriented toward the positive \\(z\\)-axis. In which of the following directions does the Lorentz [Iohr-ENZ] force act?\n\nW) Positive \\(y\\)-axis\n\nX) Negative \\(y\\)-axis\n\nY) Positive \\(z\\)-axis\n\nZ) Negative \\(z\\)-axis","Completion":"W) POSITIVE \\(y\\)-AXIS"} {"Type":"BONUS\n\nPhysics - Short Answer","Prompt":"A hollow pipe closed at both ends has a fundamental frequency of 100 hertz. In meters, what is the length of the pipe?","Completion":"1.7"} {"Type":"TOSS-UP\n\nBiology - Short Answer","Prompt":"What iron-containing proteins are the main enzymes associated with the metabolism of drugs and toxins in the liver?","Completion":"CYTOCHROME P450 (ACCEPT: CYP)"} {"Type":"BONUS\n\nBiology - Multiple Choice","Prompt":"Which of the following amino acids is typically phosphorylated [faws-FOR-il-ated] by protein kinase A?\n\nW) Tyrosine\n\nX) Serine\n\nY) Tryptophan\n\nZ) Isoleucine","Completion":"X) SERINE"} {"Type":"TOSS-UP\n\nEnergy - Short Answer","Prompt":"Oak Ridge National Lab is contributing to the production of cancer radiation treatments by producing actinium-227, which undergoes two decay transitions to become the medically-useful isotope radium-223. What two types of decay does actinium-227 undergo to become radium-223?","Completion":"BETA AND ALPHA (ACCEPT: ALPHA AND BETA)"} {"Type":"BONUS\n\nEnergy - Short Answer","Prompt":"Scientists at Lawrence Berkeley National Lab have been studying crystallized biomolecules with electron microscopy. In one type of experiment, the samples are immersed in liquid nitrogen to form vitreous ice before imaging. What type of electron microscopy is this?","Completion":"CRYOELECTRON MICROSCOPY (ACCEPT: CRYO, CRYO-EM)7) Earth and Space - Multiple Choice Which of the following geologic phenomena led to the formation of submarine canyons?"} {"Type":"TOSS-UP\n\nBiology - Multiple Choice","Prompt":"Which of the following hormones most directly affects the metabolic rate of the body?\n\nW) Thyroid-stimulating hormone\n\nX) Throotropin-releasing hormone\n\nY) Triiodothyronine [try-eye-oh-doh-THIGH-roh-neen]\n\nZ) Testosterone","Completion":"Y) TRIODOUTYRONINE"} {"Type":"TOSS-UP\n\nEnergy - Short Answer","Prompt":"Scientists at Lawrence Berkeley National Lab are developing thermoelectric materials that can convert thermal energy to electrical energy via what effect?","Completion":"SEEBECK EFFECT (ACCEPT: PELTIER-SEEBECK EFFECT, DO NOT ACCEPT: PELTIER EFFECT)"} {"Type":"TOSS-UP\n\nMath - Short Answer","Prompt":"What is the absolute value of the complex conjugate of \\(-6+i\\)?","Completion":"\\(\\sqrt{3}\\)7"} {"Type":"TOSS-UP\n\nPhysics - Short Answer","Prompt":"A reading of zero on what two temperature scales refers to absolute zero?","Completion":"KELVIN AND RANKINE (IN ANY ORDER)"} {"Type":"BONUS\n\nPhysics - Short Answer","Prompt":"Identify all of the following three types of imaging that depend on ionizing radiation: Magnetic resonance imaging; Computerized tomography [tom-AW-grafeel]; Positron emission tomography.","Completion":"2, 3"} {"Type":"TOSS-UP\n\nChemistry - Short Answer","Prompt":"An aqueous-phase redox reaction has all of its stoichiometric [stoy-keeoh-MEH-tric] coefficients doubled. Identify all of the following three quantities that are exactly doubled: Equilibrium constant; Delta-G; Cell potential.","Completion":"2"} {"Type":"BONUS\n\nChemistry - Short Answer","Prompt":"Identify all of the following three statements that are true of bonding: Xenon dichloride has a bent molecular geometry; Sulfur dioxide and carbon disulfide have the same molecular geometry; Krypton tetrafluoride has no net dipole moment.","Completion":"313) Biology - Short Answer In seeds, what is the term for the layer of sporophyte tissue that envelops and protects the megasporangium [mega-spor-AYN-gee-um]?"} {"Type":"## BONUS\n\nBiology - Short Answer","Prompt":"If blood from an incompatible ABO group is mistakenly transfused, it can agglutinate. What class of antibodies mediates this process?","Completion":"IgM (ACCEPT: M, IMMUNOGLOBULIN M)"} {"Type":"## BONUS\n\nPhysics - Short Answer","Prompt":"The position of a particle in meters is described by the function \\(x\\) of \\(t\\) equals \\(t\\) cubed plus \\(4\\ t\\) squared plus \\(3\\ t\\), where \\(t\\) is expressed in seconds. What is the acceleration, in meters per second squared, at time \\(t\\) equals \\(5\\)?","Completion":"3815) Energy - Multiple Choice Scientists at Oak Ridge National Lab have designed a fuel rod cladding alloy made of iron, chromium, and aluminum to replace current zirconium alloys. Which of the following must be a property of this cladding?"} {"Type":"TOSS-UP\n\nMath - Short Answer","Prompt":"If a pyramid has 8 vertices, how many edges does it have?","Completion":"14"} {"Type":"BONUS\n\nMath - Short Answer","Prompt":"What is the smallest positive zero of the function with equation \\(y=2\\) sine open parenthesis \\(3x+\\underline{3\\pi}\\) over 5 close parenthesis?","Completion":"\\(2\\pi\/15\\) (ACCEPT: \\((2\/15)\\pi\\))"} {"Type":"TOSS-UP\n\nEarth and Space - Short Answer","Prompt":"What object, as a result of tidal heating, is the most volcanically active body in the solar system?","Completion":"IO"} {"Type":"BONUS\n\nEarth and Space - Multiple Choice","Prompt":"A galaxy is receding from an observer such that the hydrogen alpha line appears to be at 657 nanometers instead of 656.3 nanometers. Which of the following best describes the changes in the beta and gamma lines for this galaxy?\n\nW) They will both be shifted longer by 0.7 nanometers\n\nX) They will both be shifted by slightly less than 0.7 nanometers\n\nY) They will both be shifted by slightly more than 0.7 nanometers\n\nZ) They will both be shifted shorter by 0.7 nanometers","Completion":"X) THEY WILL BOTH BE SHIFTED BY SLIGHTLY LESS THAN 0.7"} {"Type":"TOSS-UP\n\nPhysics - Short Answer","Prompt":"What boson mediates the strong force?","Completion":"GLUON"} {"Type":"BONUS\n\nPhysics - Short Answer","Prompt":"Consider a configuration of two identical point charges that are 1 meter apart, and which has a potential energy of 1 joule. To two significant figures and in joules, what is the potential energy of a configuration where four such point charges are placed in a square with side length 1 meter?","Completion":"5.4"} {"Type":"TOSS-UP\n\nMath - Short Answer","Prompt":"In the integers mod 12, what is 7 times 11?","Completion":"5"} {"Type":"BONUS\n\nMath - Short Answer","Prompt":"If \\(a\\), \\(b\\), and \\(c\\) are the three zeroes of the polynomial\n\n\\(x^{3}-9x^{2}+26x-24\\), what is the product of the three numbers \\(1-a\\), \\(1-b\\), and \\(1-c\\)?","Completion":"-6"} {"Type":"TOSS-UP\n\nBiology - Short Answer","Prompt":"What is the term for small, circular RNA molecules that can infect plants?","Completion":"VIROIDS"} {"Type":"BONUS\n\nBiology - Short Answer","Prompt":"Identify all of the following three types of trees that are considered to be gymnosperms: Cycads [SIGH-cads]; Conifers; Ginkgos.","Completion":"ALL OF THEM"} {"Type":"TOSS-UP\n\nChemistry - Short Answer","Prompt":"What functional group is formed when hot sulfuric acid is used to dehydrate an alcohol?","Completion":"ALKENE [al-KEEN]"} {"Type":"BONUS\n\nChemistry - Short Answer","Prompt":"Pourbaix [poor-BAY] diagrams show the most stable species in an aqueous electrolyte as a function of what two variables?","Completion":"pH AND POTENTIAL (ACCEPT: pH AND VOLTAGE)23) Earth and Space - Short Answer Lapilli [[] [] and others are subtypes of what materials that are ejected from volcanoes?"} {"Type":"TOSS-UP\n\nEarth and Space - Multiple Choice","Prompt":"Which of the following best describes the luster of cut diamonds?\n\nW) Adamantine [ad-ah-MAN-teen]\n\nX) Vitreous [VIH-tree-us]\n\nY) Pitchy\n\nZ) Resinous [REH-zin-us]","Completion":"W) ADAMANTINE"} {"Type":"BONUS\n\nEarth and Space - Multiple Choice","Prompt":"The evolution of oxygenic photosynthesis marked a significant change in Earth history that led to permanent changes in the redox state of Earth's atmosphere and had lasting impacts on Earth's organisms. Which of the following best describes when the evolution of oxygenic photosynthesis occurred relative to the Cambrian Explosion?\n\nW) One billion years after the Cambrian Explosion\n\nX) The same time as the Cambrian Explosion\n\nY) Two billion years before the Cambrian Explosion\n\nZ) Three billion years before the Cambrian Explosion","Completion":"Y) TWO BILLION YEARS BEFORE THE CAMBRIAN EXPLOSION"} {"Type":"TOSS-UP\n\nBiology - Short Answer","Prompt":"The most common high-energy intermediate used to drive biological reactions contains what nucleobase?","Completion":"ADENINE"} {"Type":"BONUS\n\nBiology - Short Answer","Prompt":"What type of plant cell opens and closes stomata in leaves?","Completion":"GUARD CELLS"} {"Type":"TOSS-UP\n\nMath - Multiple Choice","Prompt":"When in standard position, which of the following angles has its terminal side in the fourth quadrant?\n\nW) 3\\(\\pi\\)\/2\n\nX) 5\\(\\pi\\)\/4\n\nY) 8\\(\\pi\\)\/5\n\nZ) 13\\(\\pi\\)\/6","Completion":"Y) 8\\(\\pi\\)\/5"} {"Type":"TOSS-UP\n\nChemistry - Short Answer","Prompt":"When performing a titration [tie-TRAY-shun], what point is defined as the point at which the indicator changes color?","Completion":"ENDPOINT (DO NOT ACCEPT: EQUIVALENCE POINT)"} {"Type":"TOSS-UP\n\nEnergy - Short Answer","Prompt":"Researchers at Lawrence Berkeley National Lab are using nanoscale imaging techniques to study a biological process that is analogous to a process plants use, but in which the water splitting is artificially produced. What biological process is this?","Completion":"PHOTOSYNTHESIS"} {"Type":"BONUS\n\nEnergy - Short Answer","Prompt":"Researchers at Los Alamos National Lab are studying how to convert sugars to fuels. One of the sets of reactions they are studying converts glucose into what three-carbon alkane?","Completion":"PROPANE"} {"Type":"TOSS-UP\n\nPhysics - Short Answer","Prompt":"A 5-kilogram mass is dropped from rest. In meters per second, how fast is it falling after 2 seconds?","Completion":"19.6"} {"Type":"BONUS\n\nPhysics - Short Answer","Prompt":"Given that the thermal expansion coefficient for steel is ten to the negative five per degree Celsius, then, in meters, how much will a 1200-meter bridge expand if the temperature rises from negative 3 degrees Celsius to 5 degrees Celsius?","Completion":"0.0967) Biology - Short Answer The skeletons of Chondrichthyans [kon-DRIK-thee-ans] are composed of what structural tissue?"} {"Type":"BONUS\n\nBiology - Short Answer","Prompt":"The cohesion-tension hypothesis attempts to explain the movement of liquid through what plant system?","Completion":"XYLEM"} {"Type":"TOSS-UP\n\nEarth and Space - Multiple Choice","Prompt":"Kepler's first law of planetary motion addresses which of the following aspects of orbiting bodies?\n\nW) The relationship between orbital period and distance from the Sun\n\nX) The relationship between mass and rotational period\n\nY) The shape of orbits\n\nZ) The area swept out by an orbiting body","Completion":"Y) THE SHAPE OF ORBITS"} {"Type":"BONUS\n\nEarth and Space - Short Answer","Prompt":"What type of reflecting telescope has a secondary mirror deflect the light by 90 degrees so that it is focused at an eyepiece on the side of the telescope?","Completion":"NEWTONIAN"} {"Type":"TOSS-UP\n\nEnergy - Short Answer","Prompt":"Researchers at Argonne National Lab are using, as a catalyst in the water-splitting reaction, a metal that has high resistance to corrosion and is found in higher abundance in meteorites than in the Earth's crust. What metal are they using?","Completion":"IRIDIUM (ACCEPT: IR)"} {"Type":"BONUS\n\nEnergy - Short Answer","Prompt":"Researchers at Oak Ridge National Lab have discovered a new solid catalyst to convert gaseous methane to methanol. Because the catalyst is not the same phase as the reactants, it can be said to be what type of catalyst?","Completion":"HETEROGENEOUS"} {"Type":"TOSS-UP\n\nMath - Multiple Choice","Prompt":"Riemann [REE-monn] sums are most directly related to which of the following?\n\nW) Average rate of change\n\nX) Derivatives\n\nY) Definite integrals\n\nZ) Indefinite integrals","Completion":"Y) DEFINITE INTEGRALS"} {"Type":"BONUS\n\nMath - Short Answer","Prompt":"What is the limit as \\(x\\) approaches zero of the fraction with numerator sine squared of \\(x\\) plus open parenthesis sine of \\(x^{2}\\) close parenthesis, and denominator \\(x^{2}\\)?","Completion":"2"} {"Type":"TOSS-UP\n\nChemistry - Multiple Choice","Prompt":"Which of the following pairs of compounds have the same empirical formula?\n\nW) Water and hydrogen peroxide\n\nX) Methane and ethane\n\nY) Glucose and formaldehyde\n\nZ) Carbon monoxide and carbon dioxide","Completion":"Y) GLUCOSE AND FORMALDEHYDE"} {"Type":"BONUS\n\nChemistry - Short Answer","Prompt":"Identify all of the following three statements that are always\n\nTRUE of chemical equilibrium: Adding a reactant always results in an equilibrium shift to the right; Decreasing the temperature of a reaction always results in a decrease in the equilibrium\n\nconstant \\(K\\); Adding a product to a reaction always results in a decrease in \\(K\\).","Completion":"NONE"} {"Type":"TOSS-UP\n\nPhysics - Short Answer","Prompt":"If a rigid body is experiencing no net torque [tork] and no net\n\nforces, it is said to be in what state?","Completion":"EQUILIBRIUM (ACCEPT: DYNAMIC EQUILIBRIUM, MECHANICAL EQUILIBRIUM; DO NOT ACCEPT: STATIC EQUILIBRIUM)"} {"Type":"BONUS\n\nPhysics - Short Answer","Prompt":"Consider a square ABCD where there is a one-ohm resistor on\n\neach side of the square. If there is also a one-ohm resistor bridging the AC diagonal, what is the\n\nequivalent resistance in ohms between points A and C?","Completion":"1\/2 (ACCEPT: 0.5)13) Earth and Space - Multiple Choice A location contains deposits of poorly-sorted mixtures of sediment and rocks, including large boulders. Which of the following geological processes is most likely responsible for these deposits?"} {"Type":"## BONUS\n\nEnergy - Multiple Choice","Prompt":"What property of carbon nanotubes contributes to their usefulness in electronics applications?\n\nW) Length\n\nX) Tensile Strength\n\nY) Conductivity\n\nZ) Flexibility","Completion":"Y) CONDUCTIVITY"} {"Type":"## TOSS-UP\n\nBiology - Short Answer","Prompt":"What is the adjective for a plant species that possesses two types of sporangia?","Completion":"HETEROSPOROUS"} {"Type":"## BONUS\n\nBiology - Short Answer","Prompt":"Identify all of the following three Cnidarians [nye-DARE-ee-enz] that do NOT experience a medusa phase in their life cycles: Sea wasp; Star coral; Sea anemone.","Completion":"2, 3"} {"Type":"TOSS-UP\n\nPhysics - Multiple Choice","Prompt":"Which of the following pairs of strains are most predominant in a bent beam?\n\nW) Pressure and shear\n\nX) Tension and shear\n\nY) Tension and compression\n\nZ) Shear and compression","Completion":"Y) Tension and Compression"} {"Type":"TOSS-UP\n\nChemistry - Short Answer","Prompt":"One major use of hydrogen industrially is in what industrial process that produces ammonia?","Completion":"HABER [HAH-bur] PROCESS (ACCEPT: HABER-BOSCH PROCESS)"} {"Type":"TOSS-UP\n\nBiology - Short Answer","Prompt":"What is the adjective for the type of enzyme inhibition that involves an inhibitor that mimics the shape of the substrate, thereby blocking the active site?","Completion":"COMPETITIVE"} {"Type":"BONUS\n\nBiology - Short Answer","Prompt":"Indole-acetic acid is an example of what class of plant hormones?","Completion":"AUXINS"} {"Type":"TOSS-UP\n\nPhysics - Short Answer","Prompt":"An electron in a particle accelerator passes through an electric field such that the change in its potential energy is 15 electron-volts. In volts, what is the magnitude of the potential difference in the field from the point where the electron exited to the point where it entered?","Completion":"15"} {"Type":"BONUS\n\nPhysics - Short Answer","Prompt":"Consider the two displacement vectors 3\\(i\\) meters and -4\\(j\\) meters. Identify all of the following three statements that are true of this system: Their dot product is zero; They are perpendicular to each other; Their cross product is zero.","Completion":"1, 2"} {"Type":"TOSS-UP\n\nChemistry - Short Answer","Prompt":"The equilibrium constant for a particular chemical reaction is 1.25 times ten to the negative two. What is the equilibrium constant for the reverse reaction?","Completion":"80"} {"Type":"BONUS\n\nChemistry - Short Answer","Prompt":"What is the systematic name of the compound NaOCl?","Completion":"SODIUM HYPOCHLORITE"} {"Type":"TOSS-UP\n\nEnergy - Short Answer","Prompt":"Fermi National Accelerator Lab scientists, as a part of the SENSEI [SEN-say] collaboration, are trying to detect dark matter particles with specialized \"skipper\" CCDs. CCDs are composed of what semiconductor element?","Completion":"SILICON"} {"Type":"TOSS-UP\n\nBiology - Short Answer","Prompt":"What is the general term for metabolic reactions that release energy by breaking down complex molecules?","Completion":"CATABOLIC (ACCEPT: CATABOLISM)"} {"Type":"TOSS-UP\n\nMath - Multiple Choice","Prompt":"Which of the following cannot be the rightmost two digits, in order, of the square of an integer?\n\nW) 24\n\nX) 27\n\nY) 56\n\nZ) 89","Completion":"X) 27"} {"Type":"TOSS-UP\n\nEarth and Space - Multiple Choice","Prompt":"Which of the following tectonic plates is largest?\n\nW) African\n\nX) Arabian\n\nY) Eurasian\n\nZ) Pacific","Completion":"Z) PACIFIC"} {"Type":"## BONUS\n\nChemistry - Short Answer","Prompt":"A weak acid has a K\\({}{\\rm a}\\) of 1.6 times ten to the negative fifth. Assuming that dissociation is small, what is the percent dissociation of a one-molar solution of this acid in water?","Completion":"0.4 (ACCEPT: 0.4 PERCENT)"} {"Type":"## BONUS\n\nPhysics - Short Answer","Prompt":"At a bottling factory, a robotic arm automatically raises a 4000-newton palette up 6 meters to load it into a canning machine every 5 minutes. In watts, how much average power is required to operate the robotic arm?","Completion":"80"} {"Type":"TOSS-UP\n\nEnergy - Short Answer","Prompt":"Scientists at Lawrence Berkeley National Lab have been studying a silencer protein known as PRC2. PRC2 silences genes by chemically modifying what proteins around which DNA is wound?","Completion":"HISTONES"} {"Type":"TOSS-UP\n\nMath - Multiple Choice","Prompt":"The function \\(y=f\\) of \\(x\\) has zeroes at 4 and 12. If its graph is vertically stretched using a factor of 2, what are the zeroes of the result?\n\nW) 2 and 6\n\nX) 4 and 12\n\nY) 6 and 14\n\nZ) 8 and 24","Completion":"X) 4 AND 12"} {"Type":"TOSS-UP\n\nEarth and Space - Short Answer","Prompt":"What is the most abundant gas in Mars' atmosphere?","Completion":"CARBON DIOXIDE (ACCEPT: CO2)"} {"Type":"BONUS\n\nEarth and Space - Short Answer","Prompt":"Neutron stars are generally created as remnants of what type of supernovae [super-NOH-veel]?","Completion":"TYPE TWO (ACCEPT: TWO, ONE B, ONE C)"} {"Type":"TOSS-UP\n\nBiology - Multiple Choice","Prompt":"What phylum [FYE- lum] of animals contains organisms that do not possess tissues?","Completion":"W) Cnidaria [nye-DARE-ee-ah] X) Porifera Y) Nematoda Z) Echinodermata [eh-keye-noh-der-MAH-tah] X) PORIFERA"} {"Type":"BONUS\n\nBiology - Short Answer","Prompt":"Pterosaurs [TERR-oh-sores] were the first tetrapods to evolve what key characteristic of their clade?","Completion":"FLIGHT (ACCEPT: POWERED FLIGHT, WINGED FLIGHT)"} {"Type":"TOSS-UP\n\nMath - Short Answer","Prompt":"Two cars leave from the same spot, driving directly away from each other on a straight road, at 20 and 30 miles per hour, respectively. How many minutes will it take for the two cars to be 10 miles apart?","Completion":"12"} {"Type":"TOSS-UP\n\nPhysics - Multiple Choice","Prompt":"Which of the following conditions is sufficient for describing a system in unstable equilibrium?\n\nW) Net force is zero\n\nX) Net force is at a minimum\n\nY) Potential energy is at a strict minimum\n\nZ) Potential energy is at a strict maximum","Completion":"Z) POTENTIAL ENERGY IS AT A STRICT MAXIMUM"} {"Type":"## BONUS\n\nEnergy - Short Answer","Prompt":"Scientists at Brookhaven National Lab are researching ways to increase oil production in the leaves of plants. In what plant structure do oils, such as soybean oil or cocoa butter, normally accumulate?","Completion":"SEEDS"} {"Type":"## BONUS\n\nEarth and Space - Short Answer","Prompt":"Identify all of the following three rocks that can have a glassy texture: Coal; Obsidian; Pumice.","Completion":"2, 3"} {"Type":"TOSS-UP\n\nChemistry - Short Answer","Prompt":"What is the primary industrial ore of aluminum?","Completion":"BAUXITE"} {"Type":"TOSS-UP\n\nBiology - Multiple Choice","Prompt":"Which of the following best describes how a serine protease catalyzes amide-bond-breaking reactions?\n\nW) The enzyme stabilizes the products\n\nX) The enzyme stabilizes the reactants\n\nY) The enzyme increases the activation energy\n\nZ) The enzyme provides a mechanism with lower-energy intermediates","Completion":"Z) THE ENZYME PROVIDES A MECHANISM WITH LOWER-ENERGY INTERMEDIATES"} {"Type":"## BONUS\n\nMath - Short Answer","Prompt":"If the product of cosine \\(x\\) and sine \\(x\\) is 2\/9, what is the value of open parenthesis cosine \\(x\\) minus sine \\(x\\) close parenthesis squared?","Completion":"5\/9"} {"Type":"TOSS-UP\n\nPhysics - Multiple Choice","Prompt":"Kepler's second law states that each planet moves so that a line from the sun to the planet sweeps out equal areas in equal periods of time. It is a direct result of the conservation of which of the following physical quantities?\n\nW) Linear momentum\n\nX) Linear kinetic energy\n\nY) Angular momentum\n\nZ) Angular kinetic energy","Completion":"Y) ANGULAR MOMENTUM"} {"Type":"## BONUS\n\nPhysics - Short Answer","Prompt":"Consider a spring-mass system that has an angular frequency of 6 radians per second. If the mass is 2 kilograms, then in newtons per meter, what is the value of the spring constant for this oscillator?","Completion":"7221) Earth and Space - Short Answer The Saros cycle describes the period of time between successive alignments of what three solar system bodies?"} {"Type":"BONUS\n\nEarth and Space - Short Answer","Prompt":"In stellar fusion, what isotope is the primary product of the proton-proton chain?","Completion":"HELIUM-4"} {"Type":"TOSS-UP\n\nBiology - Short Answer","Prompt":"Mature plant cells have the ability to de-differentiate and then give rise to all of the cell types found in the adult organism. What is the term for this potential?","Completion":"TOTIPOTENT (ACCEPT: TOTIPOTENCY; DO NOT ACCEPT: PLURIPOTENT)"} {"Type":"BONUS\n\nBiology - Short Answer","Prompt":"What is the adjective for organisms that lay eggs that hatch outside the mother's body?","Completion":"OVIPAROUS23) Chemistry - Short Answer Aldehydes [AL-deh-hides] and ketones have what functional group in common?"} {"Type":"BONUS\n\nChemistry - Short Answer","Prompt":"Identify all of the following three elements that are considered nonmetals: Radon; Europium; Americium.","Completion":"1"} {"Type":"TOSS-UP\n\nEarth and Space - Short Answer","Prompt":"Solar observatories sometimes spot sungrazers, which are manifestations of what type of object?","Completion":"COMETS"} {"Type":"TOSS-UP\n\nBiology - Short Answer","Prompt":"What virus is the target of vaccines that are designed to prevent cervical cancer?","Completion":"HPV (ACCEPT: HUMAN PAPILLOMAVIRUS)"} {"Type":"TOSS-UP\n\nChemistry - Short Answer","Prompt":"What term is used to describe a hydrocarbon that only possesses carbon-carbon single bonds?","Completion":"SATURATED (ACCEPT: ALKANE)"} {"Type":"TOSS-UP\n\nPhysics - Short Answer","Prompt":"Jan drops a football off a cliff and notes that it takes 4 seconds for it to hit the ground. She throws an identical football from the same height, but with a horizontal velocity of 3.4 meters per second. Assuming the ground is level, how far, in meters, from the base of the cliff will it land?","Completion":"13.6"} {"Type":"TOSS-UP\n\nMath - Short Answer","Prompt":"What is 4\/15 divided by 6\/25?","Completion":"10\/9 (ACCEPT: 1 1\/9)"} {"Type":"BONUS\n\nBiology - Multiple Choice","Prompt":"Alpha- and beta-adrenergic agonists are medications whose structures mimic which of the following hormones?\n\nW) Insulin\n\nX) Glucagon\n\nY) Acetylcholine\n\nZ) Epinephrine","Completion":"Z) EPINEPHRINE"} {"Type":"TOSS-UP\n\nChemistry - Multiple Choice","Prompt":"Which of the following mixtures of solvents would have a negative deviation from Raoult's [rah-OOLZ] law?\n\nW) Water and acetone\n\nX) Water and ethanol\n\nY) Hexane and acetone\n\nZ) Hexane and pentane","Completion":"W) WATER AND ACETONE"} {"Type":"BONUS\n\nChemistry - Short Answer","Prompt":"Rank the following three alkanes in increasing order of their melting points: Pentane; Ethane; Nonane.","Completion":"2, 1, 3"} {"Type":"TOSS-UP\n\nPhysics - Short Answer","Prompt":"Identify all of the following three statements that are true of specific gravity: It can be expressed in grams per liter; Some materials can have a negative specific gravity; Aluminum has a positive specific gravity.","Completion":"3"} {"Type":"TOSS-UP\n\nEarth and Space - Multiple Choice","Prompt":"Which of the following is NOT a geologic period?\n\nW) Devonian\n\nX) Jurassic\n\nY) Paleogene\n\nZ) Cenozoic [SEN-oh-ZOH-ik]","Completion":"Z) CENOZOIC"} {"Type":"TOSS-UP\n\nMath - Short Answer","Prompt":"What is the second derivative with respect to \\(x\\) of \\(2x^{3}+4x-7\\)?","Completion":"12x"} {"Type":"TOSS-UP\n\nPhysics - Short Answer","Prompt":"Planet X has the same mass as Earth but one-third of its surface gravitational acceleration. What is the ratio of planet X's radius to Earth's radius?","Completion":"\\(\\forall\\)3 (ACCEPT: \\(\\forall\\)3 TO ONE)"} {"Type":"BONUS\n\nPhysics - Short Answer","Prompt":"Identify all of the following three statements that are true of electric dipoles in an electric field: A non-uniform field can cause translational acceleration; In a uniform field, the net torque is directly proportional to the field strength; In a uniform field, the net force is zero.","Completion":"ALL OF THEM"} {"Type":"TOSS-UP\n\nChemistry - Short Answer","Prompt":"In organic chemistry, what is the term for a species with empty orbitals that tends to accept electrons in a chemical reaction?","Completion":"ELECTROPHILE"} {"Type":"BONUS\n\nChemistry - Short Answer","Prompt":"Rank the following three isotopes in terms of increasing number of neutrons; Cobalt-57; Iron-57; Manganese-57.","Completion":"1, 2, 323) Earth and Space - Short Answer Ishtar Terra is a surface province on what solar system object?"} {"Type":"TOSS-UP\n\nMath - Short Answer","Prompt":"A triangle has two sides of lengths 39 and 57. What is the largest possible integer length of the third side?","Completion":"95"} {"Type":"BONUS\n\nMath - Short Answer","Prompt":"The line with equation \\(y=3x+2\\) is reflected across the line \\(y=x\\). What are the slope and \\(y\\)-intercept of its image, respectively?","Completion":"\\(\\text{SLOPE}=1\/3\\), \\(y\\text{-INTERCEPT}=-2\/3\\)"} {"Type":"TOSS-UP\n\nChemistry - Short Answer","Prompt":"The fact that a solution of hydrofluoric acid will become less acidic if you add sodium fluoride to it is an example of what effect?","Completion":"COMMON ION EFFECT (ACCEPT: LE CHATELIER'S PRINCIPLE)"} {"Type":"BONUS\n\nChemistry - Short Answer","Prompt":"If a rate constant is expressed in hertz, what order must the reaction be?","Completion":"FIRST (ACCEPT: ONE)"} {"Type":"TOSS-UP\n\nEnergy - Short Answer","Prompt":"Scientists at Ames Lab are developing new techniques to study substances that lower the activation energy of chemical reactions. What is the general term for these substances?","Completion":"CATALYST"} {"Type":"TOSS-UP\n\nBiology - Short Answer","Prompt":"Phages are viruses that infect what group of organisms?","Completion":"BACTERIA"} {"Type":"TOSS-UP\n\nChemistry - Multiple Choice","Prompt":"Consider the first-order decomposition of N\\({}{2}\\)O\\({}{5}\\) into nitrogen dioxide and oxygen. In order to get a linear fit, which of the following parameters should be graphed against time?\n\nW) Natural log of the concentration of N\\({}{2}\\)O\\({}{5}\\)\n\nX) Concentration of N\\({}{2}\\)O\\({}{5}\\)\n\nY) Concentration of N\\({}{2}\\)O\\({}{5}\\) squared\n\nZ) One over the concentration of N\\({}{2}\\)O\\({}{5}\\)","Completion":"W) NATURAL LOG OF THE CONCENTRATION OF N\\({}{2}\\)O\\({}{5}\\)"} {"Type":"TOSS-UP\n\nMath - Multiple Choice","Prompt":"Given that the function \\(f\\) of \\(x\\) has a zero at \\(x\\) = 6, at what x-value must \\(f\\) of open parenthesis \\(x\\) over 2 close parenthesis have a zero?\n\nW) 3\n\nX) 6\n\nY) 8\n\nZ) 12","Completion":"Z) 12"} {"Type":"TOSS-UP\n\nBiology - Short Answer","Prompt":"Order the following three stages of development chronologically: Fetus; Embryo; Zygote.","Completion":"3, 2, 1"} {"Type":"BONUS\n\nBiology - Multiple Choice","Prompt":"Professional antigen-presenting cells such as macrophages and dendritic [den-DRIH-disk] cells typically present foreign antigens to which of the following types of cells?\n\nW) Helper T\n\nX) Cytotoxic T\n\nY) Memory B\n\nZ) Plasma","Completion":"W) HELPER T"} {"Type":"TOSS-UP\n\nEnergy - Short Answer","Prompt":"Scientists at Oak Ridge National Lab have been studying a bacterium that makes a new derivative of what cobalt-containing vitamin?","Completion":"B12 (ACCEPT: COBALAMIN)"} {"Type":"BONUS\n\nEnergy - Multiple Choice","Prompt":"Scientists at Brookhaven National Lab are studying medical imaging scintillators that absorb high-energy light and release lower-wavelength light that can be detected with a camera. In what type of imaging are these materials most likely used?\n\nW) X-ray imaging\n\nX) MRI\n\nY) Photoacoustic imaging\n\nZ) Ultrasound","Completion":"W) X-RAY IMAGING11) Earth and Space - Short Answer What is the adjective for a constellation that never appears to set in the night sky throughout the year?"} {"Type":"BONUS\n\nEarth and Space - Short Answer","Prompt":"Apollo 11 astronauts brought back primarily what type of rock from the Mare Tranquillitatis [MAH-ray Tran-quill-ih-TAH-tis]?","Completion":"BASALT"} {"Type":"TOSS-UP\n\nPhysics - Short Answer","Prompt":"Identify all of the following three physical quantities that are considered scalar quantities: Speed; Work; Acceleration.","Completion":"1, 2"} {"Type":"BONUS\n\nPhysics - Short Answer","Prompt":"Identify all of the following three vectors that must be constant for an object in an unperturbed orbit: Angular momentum; Linear momentum; Acceleration.","Completion":"1"} {"Type":"TOSS-UP\n\nBiology - Multiple Choice","Prompt":"Which of the following classes of macromolecules can be digested by salivary amylase?\n\nW) Proteins\n\nX) Carbohydrates\n\nY) Lipids\n\nZ) Nucleic acids","Completion":"X) CARBOHYDRTES"} {"Type":"TOSS-UP\n\nEarth and Space - Multiple Choice","Prompt":"Gabriela is studying a map showing rivers flowing through a mountainous region. She notices that the V-shaped river valleys are often much wider than the rivers themselves. Which of the following statements best explains her observation?\n\nW) The rivers carried more water in the past\n\nX) The valleys were formed by glaciers and were later replaced by rivers\n\nY) A series of cut-banks and point-bars formed as the rivers developed, widening the valley\n\nZ) Mass wasting and stream erosion have widened the valley","Completion":"Z) MASS WASTING AND STREAM Erosion HAVE WIDENED THE VALLEY"} {"Type":"## BONUS\n\nEnergy - Multiple Choice","Prompt":"Oak Ridge National Lab is developing a method to produce quantum particles in a controlled, deterministic manner to improve the speed and security of transmitting encrypted data over long fiber-optic cables. What specific particles are they using to transmit this data?\n\nW) Photons\n\nX) Phonons\n\nY) Electrons\n\nZ) Positrons","Completion":"W) PHOTONS"} {"Type":"TOSS-UP\n\nChemistry - Multiple Choice","Prompt":"Which of the following best describes the enthalpy and entropy changes associated with melting ice?\n\nW) Negative delta H, negative delta S\n\nX) Negative delta H, positive delta S\n\nY) Positive delta H, positive delta S\n\nZ) Positive delta H, negative delta S","Completion":"Y) POSITIVE DELTA H, POSITIVE DELTA S"} {"Type":"## BONUS\n\nChemistry - Short Answer","Prompt":"Rank the following three molecules in terms of increasing triple point temperature: Water; Carbon dioxide; Methane.","Completion":"3, 2, 1"} {"Type":"TOSS-UP\n\nMath - Short Answer","Prompt":"What is 2 times 5 squared?","Completion":"50 (DO NOT ACCEPT: 100)"} {"Type":"BONUS\n\nMath - Short Answer","Prompt":"How many sides does a regular polygon have if each interior angle measures 175 degrees?","Completion":"72"} {"Type":"TOSS-UP\n\nPhysics - Short Answer","Prompt":"Galileo's theory of gravity states that objects will fall at the same speed regardless of their weight. Due to the atmosphere, however, some objects fall faster than others. Rank the following three objects in increasing order of their speed one second after being dropped from a building: One-gram feather; One-gram solid-iron sphere; One-gram solid-iron cube.","Completion":"1, 3, 2"} {"Type":"BONUS\n\nPhysics - Short Answer","Prompt":"A pipe is carrying an incompressible liquid flowing at 8 meters per second. At a particular section, the center of the pipe is elevated by 3 meters. If the static pressure is unchanged, then what is the velocity to the nearest meter per second in the elevated section of pipe?","Completion":"219) Biology - Short Answer Identify all of the following three functional groups that can be found in alanine: 1) Carboxyl; 2) Amine; 3) Alkene [al-KEEN]."} {"Type":"## BONUS\n\nBiology - Multiple Choice","Prompt":"The estrus cycle in many mammals is responsible for which of the following processes?\n\nW) Sleep\n\nX) Digestion\n\nY) Reproduction\n\nZ) Migration","Completion":"Y) REPRODUCTION"} {"Type":"TOSS-UP\n\nEarth and Space - Short Answer","Prompt":"What is the name of the largest canyon on the planet Mars?","Completion":"VALLES MARINERIS (DO NOT ACCEPT: MARINER VALLEY)"} {"Type":"TOSS-UP\n\nChemistry - Short Answer","Prompt":"Standard reduction potentials are reported using what type of electrode as the zero point?","Completion":"STANDARD HYDROGEN ELECTRODE (ACCEPT: SHE)"} {"Type":"BONUS\n\nChemistry - Short Answer","Prompt":"Concentrated hydrochloric acid is sold as a 12-molar solution in water. How many milliliters of concentrated hydrochloric acid must be used to prepare 1.5 liters of a 100-millimolar solution of hydrochloric acid?","Completion":"12.5"} {"Type":"TOSS-UP\n\nPhysics - Short Answer","Prompt":"What is the name of the distance from an uncharged, stationary object at which the escape velocity equals the speed of light?","Completion":"SCHWARTZSCHILD RADIUS"} {"Type":"BONUS\n\nPhysics - Short Answer","Prompt":"Andrew is designing a bird-strike-resistant nose cone for an experimental aircraft that flies at 200 meters per second. If a typical bird is 0.5 kilograms and an impact lasts 2.5 milliseconds, then in newtons, what is the average impact force that the nose cone must be able to withstand?","Completion":"40,00023) Math - Short Answer In three-space, what is the distance from the point (1, 3, 6) to the origin?"} {"Type":"TOSS-UP\n\nMath - Short Answer","Prompt":"What is the square of the complex number 1 - 3i?","Completion":"-8 - 6\\(i\\)"} {"Type":"BONUS\n\nMath - Multiple Choice","Prompt":"A differentiable function \\(f\\) of \\(x\\) has domain the closed interval from -5 to 4 and range the closed interval from 3 to 12. If \\(f\\) prime of \\(x\\) is less than 0 for all \\(x\\) in the domain, what is \\(f\\) of -5?\n\nW) -5\n\nX) 3\n\nY) 4\n\nZ) 12","Completion":"Z) 12"} {"Type":"TOSS-UP\n\nBiology - Short Answer","Prompt":"What stain, performed with crystal violet dye, provides the most common way of classifying bacteria?","Completion":"GRAM"} {"Type":"BONUS\n\nBiology - Short Answer","Prompt":"What is the term for mobile genetic elements that can move from one location to another in a cell's genome?","Completion":"TRANSPOSONS (ACCEPT: JUMPING GENE)"} {"Type":"TOSS-UP\n\nEarth and Space - Multiple Choice","Prompt":"Which of the following surface reservoirs on Earth contains the most carbon?\n\nW) Soil\n\nX) Atmosphere\n\nY) Vegetation\n\nZ) Ocean","Completion":"Z) OCEAN"} {"Type":"TOSS-UP\n\nPhysics - Short Answer","Prompt":"An astronaut weighs 500 newtons on planet X, which has a gravitational acceleration of 4 meters per second squared. If he travels to planet Y, which has a gravitational acceleration of 12 meters per second squared, then in newtons, what is his new weight?","Completion":"1500"} {"Type":"TOSS-UP\n\nChemistry - Short Answer","Prompt":"What intermolecular force is responsible for the secondary structure of proteins?","Completion":"HYDROGEN BONDING"} {"Type":"BONUS\n\nChemistry - Short Answer","Prompt":"Rank the following three straight-chained alkanes in increasing order of their carbon-chain lengths: Propane; Butane; Methane.","Completion":"3, 1, 2"} {"Type":"TOSS-UP\n\nEnergy - Short Answer","Prompt":"Scientists at Oak Ridge National Lab are using the Titan supercomputer to model diesel engines. Unlike the Otto cycle, the Diesel cycle relies on what to ignite the fuel?","Completion":"COMPRESSION"} {"Type":"BONUS\n\nEnergy - Short Answer","Prompt":"Chemists at SLAC National Lab have developed a copper catalyst with a large surface area that converts carbon dioxide to ethanol in an acidic environment. Identify all of the following three statements that are TRUE of this process: Carbon dioxide is oxidized in this reaction; This is an example of homogenous catalysis; The reaction has many side products.","Completion":"37) Physics - Multiple Choice A torque [tork] integrated over an angle describes which of the following quantities?"} {"Type":"BONUS\n\nEarth and Space - Short Answer","Prompt":"Telescope A has 2 times the light gathering power of telescope B. If telescopes A and B are of the same type of telescope, what is the ratio of the diameter of telescope A to that of telescope B?","Completion":"\\(\\surd\\)2 (ACCEPT: \\(\\surd\\)2 TO ONE)"} {"Type":"TOSS-UP\n\nEnergy - Short Answer","Prompt":"Researchers at Ames Lab are developing a sub-diffraction Raman platform. In this case, sub-diffraction refers to the diffraction of what?","Completion":"LIGHT (ACCEPT: ELECTROMAGNETIC WAVES, PHOTONS)"} {"Type":"BONUS\n\nEnergy - Short Answer","Prompt":"The DES at Fermi National Accelerator Lab is studying the properties of what energy in the universe that is thought to counteract the effects of gravity?","Completion":"DARK ENERGY11) Chemistry - Short Answer Rank the following three species in terms of increasing basicity: 1) Chloride; 2) Cyanide; 3) Water."} {"Type":"BONUS\n\nChemistry - Short Answer","Prompt":"2-methylropene can react with hydrogen bromide to form 2-bromo-2-methyl-propane. Identify all of the following three species that act as nucleophiles in this reaction: Bromide; Pi [piel] bond; Sigma bond.","Completion":"1, 2"} {"Type":"TOSS-UP\n\nBiology - Multiple Choice","Prompt":"Which of the following plant hormones play a role in fruit ripening?\n\nW) Gibberellins [jib-er-ELL-ins]\n\nX) Abscisic [ab-SII-zik] acid\n\nY) Cytokins [sigh-toh-KYE-nins]\n\nZ) Brassinosteroids","Completion":"W) GIBBERELLINS"} {"Type":"BONUS\n\nBiology - Short Answer","Prompt":"Identify all of the following three substances that are able to pass through the glomerulus [glo-MAIR-yoo-lus] of a healthy nephron [NEF-ron] into the filtrate:\n\nAlbumin; Glucose; Red blood cell.","Completion":"213) Physics - Short Answer The mass of an oscillator [AWS-ih-later] is doubled. Identify all of the following three systems in which the resulting period of oscillation will NOT significantly change as a result of the change in mass: 1) Pendulum; 2) Spring; 3) Orbiting satellite."} {"Type":"TOSS-UP\n\nMath - Short Answer","Prompt":"Solve the following equation for \\(x\\): log base 2 of\n\nopen parenthesis \\(x+5\\) close parenthesis \\(=5\\)?","Completion":"27"} {"Type":"BONUS\n\nMath - Short Answer","Prompt":"An isosceles trapezoid with integer side lengths has bases of length 24 and 40. What is the minimum possible length of its shortest side?","Completion":"9"} {"Type":"TOSS-UP\n\nEnergy - Multiple Choice","Prompt":"Research from the Pacific Northwest National Lab indicates\n\nthat plant mortality in the moist tropics can be largely attributed to what two factors?\n\nW) Carbon glut and hydraulic failure\n\nX) Carbon starvation and hydraulic failure\n\nY) Carbon glut and overhydration\n\nZ) Carbon starvation and overhydration","Completion":"X) CARBON STARVATION AND HYDAULIC FAILURE"} {"Type":"BONUS\n\nEnergy - Multiple Choice","Prompt":"Fermi National Lab scientists demonstrated the accuracy of\n\nlattice quantum chromodynamics, which is a theory that, among other predictions, predicts\n\nwhich of the following experimental observations?\n\nW) Color confinement\n\nX) Electron capture\n\nY) Beta plus decay\n\nZ) Nuclear fission","Completion":"W) COLOR CONFINEMENT"} {"Type":"BONUS\n\nEarth and Space - Short Answer","Prompt":"What name is given to the regions of the asteroid belt that contain few asteroids and correspond to orbital resonances with Jupiter?","Completion":"KIRKWOOD GAPS"} {"Type":"TOSS-UP\n\nChemistry - Short Answer","Prompt":"Sodium metal is produced by the electrolysis of sodium chloride in what special type of cell?","Completion":"DOWNS CELL"} {"Type":"BONUS\n\nChemistry - Short Answer","Prompt":"What is the oxidation state of oxygen in hydrogen peroxide?","Completion":"NEGATIVE ONE"} {"Type":"TOSS-UP\n\nMath - Short Answer","Prompt":"If \\(f\\) of \\(x\\) equals \\(4x^{2}-x+3\\), what is \\(f\\) of -2?","Completion":"21"} {"Type":"BONUS\n\nMath - Short Answer","Prompt":"An airplane is traveling at 380 miles per hour with respect to the Earth. The Sun's reflection off the plane illuminates a small area on the surface of the Earth. A passenger on the plane observes this illuminated spot move between two houses in 15 seconds. Expressing your answer as a decimal to the nearest tenth of a mile, how far apart are the two houses?","Completion":"1.6"} {"Type":"TOSS-UP\n\nBiology - Short Answer","Prompt":"Stones made of precipitated bile products can sometimes cause problems in the digestive system. What organ can be surgically removed to remedy this?","Completion":"GALLBALODER"} {"Type":"BONUS\n\nBiology - Short Answer","Prompt":"Based on modern taxonomy, how many domains of life are there?","Completion":"323) Physics - Short Answer A Leyden [LAY-den] jar, which has a layer of metal coating the inside and outside of a glass jar, is an early form of what circuit component?"} {"Type":"## BONUS\n\nPhysics - Short Answer","Prompt":"Given that the electric constant is 8.9 times 10 -12 farads [FAIR-adz] per meter and a charge of 8 times 10 -11 coulombs is at the center of a spherical surface, then, to the nearest volt-meter, what is the electric flux that passes through the surface?","Completion":"9"} {"Type":"TOSS-UP\n\nEarth and Space - Multiple Choice","Prompt":"What is the name for slow and gradual displacement of rock layers that occurs without the accumulation of significant strain?\n\nW) Fault rupture\n\nX) Fault break\n\nY) Fault creep\n\nZ) Fault walk","Completion":"Y) FAULT CREEP"} {"Type":"TOSS-UP\n\nMath - Short Answer","Prompt":"What is the distance between the points (-1, and (-10, ?","Completion":"\\(\\sqrt{85}\\)"} {"Type":"TOSS-UP\n\nBiology - Short Answer","Prompt":"A sphygmomanometer [sfig-moh-man-AW-meh-tur] is a medical device that measures what vital sign?","Completion":"BLOOD PRESSURE"} {"Type":"TOSS-UP\n\nChemistry - Multiple Choice","Prompt":"Which of the following is closest to the percent by mass of chloroform that is carbon?\n\nW) 10\n\nX) 16\n\nY) 20\n\nZ) 25","Completion":"W) 10"} {"Type":"BONUS\n\nChemistry - Multiple Choice","Prompt":"A buffered aqueous system contains 0.02 molar ammonia and 0.2 molar ammonium. Which of the following is closest to the pH of this system?\n\nW) 8.3\n\nX) 9.3\n\nY) 10.3\n\nZ) 12.3","Completion":"W) 8.3"} {"Type":"TOSS-UP\n\nPhysics - Short Answer","Prompt":"In meters, how far does a 10-kilogram mass that is dropped from rest travel in 10 seconds?","Completion":"490"} {"Type":"BONUS\n\nPhysics - Short Answer","Prompt":"A positively charged particle is traveling in the positive direction along the line \\(y=x\\) in the presence of a constant \\(B\\) field pointing in the positive \\(x\\) direction. In what direction is the Lorentz [Iohr-ENZ] force on the particle pointing?","Completion":"NEGATIVE Z7) Energy - Multiple Choice Researchers at Oak Ridge National Lab use pulsed laser light to induce thermal stress. This enables them to study the behavior of what type of excitation that plays an important role in many of the physical properties of solids?"} {"Type":"## BONUS\n\nEnergy - Short Answer","Prompt":"Researchers at Lawrence Berkeley National Lab are revisiting the BaBar [BAH-bur] collaboration to look for data pertaining to the material that is thought to compose 85% of the mass in the universe. What is the term for this material?","Completion":"DARK MATTER"} {"Type":"## BONUS\n\nEarth and Space - Short Answer","Prompt":"In what layer of the Earth's atmosphere do auroras primarily occur?","Completion":"THERMOSPHERE"} {"Type":"TOSS-UP\n\nChemistry - Short Answer","Prompt":"What is the chemical formula of ethylene, the simplest alkene [al-KEEN]?","Completion":"C\\({}{2}\\)H\\({}{4}\\)"} {"Type":"TOSS-UP\n\nBiology - Short Answer","Prompt":"Dinoflagellates [DYE-no-FLAJ-ihl-ates] have been identified as one of the main sources of what deadly marine phenomenon?","Completion":"RED TIDE (ACCEPT: HARMFUL ALGAL BLOOM)"} {"Type":"TOSS-UP\n\nMath - Multiple Choice","Prompt":"Which of the following is equivalent to\n\nsine 47\\({}^{\\circ}\\)cosine 11\\({}^{\\circ}\\) - cosine 47\\({}^{\\circ}\\)sine 11\\({}^{\\circ}\\)?\n\nW) sine 36\\({}^{\\circ}\\)\n\nX) cosine 36\\({}^{\\circ}\\)\n\nY) sine 58\\({}^{\\circ}\\)\n\nZ) cosine 58\\({}^{\\circ}\\)","Completion":"W) SINE 36\\({}^{\\circ}\\)"} {"Type":"TOSS-UP\n\nPhysics - Short Answer","Prompt":"Winds flowing from high to low pressures on the Earth appear to be deflected to the right in the Northern hemisphere because of what effect?","Completion":"CORIOLIS"} {"Type":"## BONUS\n\nEnergy - Short Answer","Prompt":"Scientists at Princeton Plasma Physics Lab are building the world's largest stellarator, which is used to confine what phase of matter?","Completion":"PLASMA"} {"Type":"### TOSS-UP\n\nEarth and Space - Multiple Choice","Prompt":"Which of the following minerals is not typically found in granite?\n\nW) Olivine [AW-liv-een] X) Plagioclase [PLAY-jee-oh-klaze] Y) Quartz Z) Hornblende","Completion":"W) OLIVINE"} {"Type":"## BONUS\n\nEarth and Space - Short Answer","Prompt":"Arrange the following three colors of light in order of increasing depth of penetration in average seawater: Violet; Red; Yellow.","Completion":"2, 3, 115) Physics - Short Answer If silicon is doped with boron, it becomes what type of semiconductor?"} {"Type":"BONUS\n\nEarth and Space - Short Answer","Prompt":"What planet in our solar system is the only planet to have an orbital inclination to the ecliptic of greater than 4 degrees?","Completion":"MERCURY"} {"Type":"TOSS-UP\n\nPhysics - Short Answer","Prompt":"What physical quantity is defined as force integrated over a time interval?","Completion":"IMPULSE"} {"Type":"BONUS\n\nPhysics - Short Answer","Prompt":"Two capacitors, both with capacitance C, are initially charged such that they have voltages of 1.5 volts and 3.5 volts, respectively. If the positive plates are connected to each other, and the negative plates are also connected to each other, then, in volts, what is the total voltage across the system?","Completion":"2.5"} {"Type":"TOSS-UP\n\nMath - Short Answer","Prompt":"In the integers mod 8, what is 7 + 5?","Completion":"4"} {"Type":"BONUS\n\nMath - Short Answer","Prompt":"If the line tangent to the graph of the differentiable function \\(f\\) of \\(x\\) at the point (-3, passes through the point (4.5, -, then what is \\(f\\) prime of \\(x\\) when \\(x\\) = -3?","Completion":"-6\/5 (ACCEPT: -1 1\/5, -1.2)"} {"Type":"TOSS-UP\n\nChemistry - Short Answer","Prompt":"Benzene demonstrates what special type of conjugation due to its 6 pi [pie] electrons?","Completion":"AROMATICITY (ACCEPT: AROMATIC)"} {"Type":"BONUS\n\nChemistry - Short Answer","Prompt":"Which of the electron's quantum numbers corresponds to the size and energy of the orbital?","Completion":"PRINCIPAL (ACCEPT: N)"} {"Type":"TOSS-UP\n\nBiology - Short Answer","Prompt":"In female mammals, one of the two X chromosomes will be deactivated to form what structure?","Completion":"BARR BODY"} {"Type":"BONUS\n\nBiology - Short Answer","Prompt":"How many different human aminoacyl [ah-MEE-no-ASS-il]-tRNA synthetases are there to charge the 45 different tRNA molecules?","Completion":"TWENTY"} {"Type":"TOSS-UP\n\nBiology - Short Answer","Prompt":"Atherosclerotic [athero-sklair-AW-tic] plaques are composed of what lipid that is stored in LDL and HDL particles?","Completion":"CHOLESTEROL"} {"Type":"TOSS-UP\n\nEarth and Space - Multiple Choice","Prompt":"Horsts and grabens [GRAH-bens] are bounded by what type of fault?\n\nW) Strike-slip\n\nX) Reverse\n\nY) Thrust\n\nZ) Normal","Completion":"Z) NORMAL"} {"Type":"TOSS-UP\n\nEnergy - Short Answer","Prompt":"Scientists at Princeton Plasma Physics Lab are developing improvements to the fusion reactions occurring in inside a tokamak [TOW-kah-mak]. These fusion reactions are fueled by what isotopes of hydrogen?","Completion":"DEUTERIUM AND TRITIUM (ACCEPT: HYDROGEN-2 AND HYDROGEN-3)"} {"Type":"BONUS\n\nEnergy - Short Answer","Prompt":"Researchers at Idaho National Lab are developing practical methods to reduce the cost of biofuel production from biomass. What class of biofuel is made from vegetable oil or animal fat and consists of long-chain alkyl [AL-keel] esters?","Completion":"BIODIESEL (ACCEPT: DIESEL)"} {"Type":"TOSS-UP\n\nChemistry - Short Answer","Prompt":"What is the conjugate acid of ammonia?","Completion":"AMMONIUM (ACCEPT: NH\\({}{4}\\)\\({}^{+}\\))"} {"Type":"BONUS\n\nChemistry - Short Answer","Prompt":"What is the name of the aldehyde [AL-deh-hide] with only a single carbon atom?","Completion":"FORMALDEHYDE (ACCEPT: METHANAL)"} {"Type":"TOSS-UP\n\nPhysics - Multiple Choice","Prompt":"Kate is tuning her cello's A string by producing an A note on a string that is already tuned and comparing the sounds. Which of the following beat frequencies, in hertz, would indicate that the strings are closest to having the same pitch?\n\nW) 2\n\nX) 100\n\nY) 440\n\nZ) 880","Completion":"W) 2"} {"Type":"BONUS\n\nPhysics - Short Answer","Prompt":"Identify all of the following three statements that must be true of all satellites in geosynchronous [jee-oh-SINK-rin-us] orbits: They stay above the same point on Earth; Their altitude is affected by the mass of the satellite; They have an orbital period of one sidereal day.","Completion":"3"} {"Type":"TOSS-UP\n\nMath - Short Answer","Prompt":"Giving your answer as a decimal, increasing a number by 125% is equivalent to multiplying it by what?","Completion":"2.25"} {"Type":"BONUS\n\nMath - Short Answer","Prompt":"Three exterior angles, one at each vertex of a triangle, have measures in the ratio of 5 to 7 to 8. What is the degree measure of the triangle's smallest interior angle?","Completion":"367) Earth and Space - Short Answer RR Lyrae [LYE-ree] and Cepheid [SEF-ee-id] are classes of what type of star?"} {"Type":"TOSS-UP\n\nBiology - Short Answer","Prompt":"What is the common term for a basidiocarp [bah-SID-ee-oh-karpl], the fruiting body of a basidiomycete [bah-sid-ee-oh-MY-seet], when it is found above ground?","Completion":"MUSHROOM"} {"Type":"TOSS-UP\n\nPhysics - Short Answer","Prompt":"A remote control car drops from a cliff and travels 19.6 meters before hitting the ground. How long, in seconds, did it spend in the air?","Completion":"2"} {"Type":"BONUS\n\nPhysics - Short Answer","Prompt":"Identify all of the following three statements that are true of free-fall: Apparent weightlessness is the sensation that occurs when the gravitational force is negated by air friction; Satellites orbiting the earth are in near free-fall; Objects free-falling on Earth all reach terminal velocity.","Completion":"2"} {"Type":"TOSS-UP\n\nMath - Short Answer","Prompt":"Using the point names \\(A\\), \\(B\\), \\(C\\), \\(D\\), \\(E\\), \\(F\\), \\(G\\), and \\(H\\), how many different names are there for octagon ABCDEFGH?","Completion":"16"} {"Type":"BONUS\n\nMath - Short Answer","Prompt":"What are the coordinates of the vertex of the graph of \\(y=-4x^{2}-16x-9\\)?","Completion":"(-2, 7) (ACCEPT: \\(x=-2\\) AND \\(y=7\\))"} {"Type":"TOSS-UP\n\nChemistry - Multiple Choice","Prompt":"Kevin performs a titration [tie-TRAY-shun] using sodium hydroxide as the titrant. He finds that the pH at the equivalence point is above 7. What does this imply about the analyte?\n\nW) It is a strong acid\n\nX) It is a weak acid\n\nY) It is a strong base\n\nZ) It is a weak base","Completion":"X) IT IS A WEAK ACID"} {"Type":"TOSS-UP\n\nEnergy - Multiple Choice","Prompt":"Argonne National Lab scientists recently developed a computer model to calculate the ionization potential of liquid water. This property was previously only approximated using photoemission spectroscopy, a technique most dependent on which of the following physical effects?\n\nW) Photoelectric effect\n\nX) Faraday effect\n\nY) Two-photon absorption\n\nZ) Kinetic isotope effect","Completion":"W) PHOTOELECTRIC EFFECT"} {"Type":"BONUS\n\nEarth and Space - Short Answer","Prompt":"What set of prevailing winds is responsible for the curving of Atlantic hurricanes toward the northeast once they reach temperate latitudes?","Completion":"WESTERLIES (ACCEPT: ANITTRADES)"} {"Type":"TOSS-UP\n\nChemistry - Short Answer","Prompt":"Identify all of the following three metals that are transition metals: Lead; Silver; Aluminum.","Completion":"2"} {"Type":"BONUS\n\nChemistry - Short Answer","Prompt":"Consider the combustion reaction between solid carbon and gaseous oxygen to generate gaseous carbon dioxide. Identify all of the following three changes that would shift the reaction to the right: Decreasing the volume of the container; Adding an inert gas to the container; Decreasing the temperature of the container.","Completion":"3"} {"Type":"TOSS-UP\n\nMath - Short Answer","Prompt":"What is the discriminant of the quadratic function \\(y=x^{2}-5x+8\\)?","Completion":"-7"} {"Type":"BONUS\n\nMath - Short Answer","Prompt":"Evaluate the integral from \\(x=-10\\) to \\(0\\) of the square root of open parenthesis \\(100-x^{2}\\)close parenthesis \\(dx\\).","Completion":"\\(25\\pi\\)"} {"Type":"TOSS-UP\n\nPhysics - Short Answer","Prompt":"A particular mineral rod can withstand approximately 200 joules of energy input from impact without shattering. If the rod weighs 5 kilograms, then, to the nearest meter, how far can it be dropped without shattering?","Completion":"4"} {"Type":"BONUS\n\nPhysics - Short Answer","Prompt":"Vince drops a football from a cliff and notes that it takes 2 seconds for it to hit the ground. He throws an identical football from the same height, but with a horizontal velocity of 5 meters per second. Assuming the ground is level, then to the nearest whole meter, how far from the point of release will the ball land?","Completion":"2217) Biology - Short Answer What enzyme joins all of the various Okazaki fragments together in DNA replication?"} {"Type":"## BONUS\n\nMath - Short Answer","Prompt":"The sums of three whole numbers taken in pairs are 16, 17, and 23. What is the product of these three numbers?","Completion":"660"} {"Type":"## BONUS\n\nPhysics - Short Answer","Prompt":"A certain particle has a potential energy of zero at a given moment. Identify all of the following three statements that must be TRUE of the particle: The force on the particle is zero; The particle is at equilibrium; The particle has zero acceleration.","Completion":"NONE OF THEM"} {"Type":"TOSS-UP\n\nChemistry - Multiple Choice","Prompt":"Doping a silicon crystal with which of the following elements would produce an n-type semiconductor?\n\nW) Bismuth [BIZ-muth]\n\nX) Selenium [sih-LEE-nee-un]\n\nY) Sulfur\n\nZ) Tin","Completion":"W) BISMUTH"} {"Type":"BONUS\n\nChemistry - Short Answer","Prompt":"Rank the following three atoms in terms of increasing first ionization energy: Chlorine; Phosphorous; Silicon.","Completion":"3, 2, 1"} {"Type":"TOSS-UP\n\nEarth and Space - Short Answer","Prompt":"Hydrogen and helium make up approximately 97 percent of the Sun's mass. What element, making up about one percent of the Sun's mass, is the next most abundant?","Completion":"OXYGEN"} {"Type":"BONUS\n\nEarth and Space - Short Answer","Prompt":"Identify all of the following three aspects of planetary motion that were included in Copernicus' model of the solar system: The Sun is at the center; Planets have elliptical orbits; The Moon orbits Earth.","Completion":"1, 3"} {"Type":"TOSS-UP\n\nBiology - Short Answer","Prompt":"What technique did Rosalind Franklin use to determine the helical structure of DNA?","Completion":"X-RAY CRYSTALLOGRAPHY (ACCEPT: X-RAY DIFFRACTION, X-RAY DIFFRACTION CRYSTALLOGRAPHY)"} {"Type":"TOSS-UP\n\nMath - Multiple Choice","Prompt":"Which of the following best describes the relationship between the lines with equations \\(\\underline{5x+4v=20}\\) and \\(\\underline{16x+20v=80}\\)?\n\nW) Parallel\n\nX) Perpendicular\n\nY) Intersecting but not perpendicular\n\nZ) Coincident","Completion":"Y) INTERSECTING BUT NOT PERPENDICULAR"} {"Type":"TOSS-UP\n\nChemistry - Short Answer","Prompt":"What is the systematic name of the compound N\\({}{2}\\)O\\({}{4}\\)?","Completion":"DINITROGEN TETROXIDE"} {"Type":"TOSS-UP\n\nEnergy - Short Answer","Prompt":"Scientists at Los Alamos National Lab are studying a group of bacterial pumps known as RND efflux pumps, which improve bacteria's resistance to what?","Completion":"ANTIBIOTICS"} {"Type":"BONUS\n\nEnergy - Short Answer","Prompt":"Researchers at Los Alamos National Lab have applied machine learning to seismic monitoring to identify when an earthquake will occur. What is the term for the Earth surface location directly above an earthquake?","Completion":"EPICENTER"} {"Type":"TOSS-UP\n\nPhysics - Short Answer","Prompt":"Andrew applies an average acceleration of 5 meters per second squared to increase his vehicular velocity from 15 meters per second to 30 meters per second. How much acceleration, in meters per second squared, would he have to apply to perform the same velocity change in half the time?","Completion":"10"} {"Type":"BONUS\n\nPhysics - Short Answer","Prompt":"Given that the reference sound intensity is defined as ten to the negative twelfth watts per meter squared, what is the decibel value for a sound with an intensity of one watt per meter squared?","Completion":"120"} {"Type":"TOSS-UP\n\nEarth and Space - Multiple Choice","Prompt":"Which of the following statements regarding Earth's core is NOT true?\n\nW) It contains a liquid outer core and a solid inner core\n\nX) The inner core has a higher temperature than the outer core\n\nY) The inner core is responsible for Earth's magnetic field\n\nZ) The inner core has a greater density than the outer core","Completion":"Y) THE INNER CORE IS RESPONSIBLE FOR EARTH'S MAGNETIC FIELD"} {"Type":"TOSS-UP\n\nBiology - Short Answer","Prompt":"What human organ contains the semicircular canals as well as the cochlea [KOH-klee-ah]?","Completion":"EAR"} {"Type":"## BONUS\n\nPhysics - Short Answer","Prompt":"Kevin is standing on a scale in an elevator. As the elevator starts moving, he notices his apparent weight on the scale decrease. Identify all of the following three statements that are true of this system: The elevator is traveling upwards; Kevin's change in apparent weight is dependent on the acceleration of the elevator; Kevin's apparent weight will increase as the elevator stops moving.","Completion":"2, 3"} {"Type":"## BONUS\n\nEarth and Space - Short Answer","Prompt":"A telescope's objective lens has a focal length of 90 centimeters, and its eyepiece has a focal length of one-third of a centimeter. What is the telescopes magnifying power?","Completion":"270 (ACCEPT: 270 TIMES, 270X)"} {"Type":"TOSS-UP\n\nEnergy - Multiple Choice","Prompt":"Scientists at SLAC National Lab have developed a way to use diamond anvils to drive chemical reactions using which of the following?\n\nW) Temperature\n\nX) Conductivity\n\nY) Pressure\n\nZ) Malleability","Completion":"Y) PRESSURE"} {"Type":"TOSS-UP\n\nChemistry - Short Answer","Prompt":"What is the term for isomers that are non-superimposable mirror images of each other?","Completion":"ENANTIGMERS"} {"Type":"TOSS-UP\n\nBiology - Short Answer","Prompt":"How many different human codons code for\n\nmethionine [meth-IOH-neen]?","Completion":"ONE"} {"Type":"BONUS\n\nBiology - Multiple Choice","Prompt":"Operant and classical conditioning are both examples of which of the following forms of learning?\n\nW) Sensitization\n\nX) Habituation\n\nY) Associative\n\nZ) Episodic","Completion":"Y) ASSOCIATIVE"} {"Type":"TOSS-UP\n\nMath - Short Answer","Prompt":"If angle ABC is inscribed in a circle and measures 65 degrees, what is the degree measure of arc ABC?","Completion":"230"} {"Type":"BONUS\n\nMath - Short Answer","Prompt":"What is the sum of the distinct prime factors of 924?","Completion":"23"} {"Type":"TOSS-UP\n\nEarth and Space - Short Answer","Prompt":"Into what units of geologic time are periods divided?","Completion":"EPOCHS"} {"Type":"TOSS-UP\n\nBiology - Short Answer","Prompt":"A plant that is heterozygous [hetero-ZYE-gus] for flower color is\n\nself-pollinated. What percentage of the offspring will be true-breeding?","Completion":"50"} {"Type":"TOSS-UP\n\nChemistry - Short Answer","Prompt":"A balloon is inflated with 10 grams of helium. Under the same conditions, a second balloon is inflated to the same volume with nitrogen. To the nearest gram, what is the mass of nitrogen in the second balloon?","Completion":"70"} {"Type":"BONUS\n\nChemistry - Short Answer","Prompt":"What is the hybridization [HIGH-brih-dye-ZAY-shun] state of the central atom in oxygen difluoride?","Completion":"SP\\({}^{3}\\)"} {"Type":"TOSS-UP\n\nMath - Short Answer","Prompt":"Name one of the linear factors of \\(x^{2}-34x-72\\).","Completion":"\\(x+2\\) OR \\(x-36\\) (must give at least one)"} {"Type":"BONUS\n\nMath - Short Answer","Prompt":"What is the slope-intercept equation of the line tangent to the graph of \\(y=x^{3}+x+4\\) at the point (1, ?","Completion":"\\(y=4x+2\\)"} {"Type":"TOSS-UP\n\nBiology - Short Answer","Prompt":"What is the term for the \"bead\" in the \"beads on a string\" model of eukaryotic DNA?","Completion":"NUCLEOSOME"} {"Type":"BONUS\n\nBiology - Short Answer","Prompt":"Biopolymers, such as amino acid chains, are created via the removal of what molecule from two monomer subunits?","Completion":"WATER (ACCEPT: H\\({}{2}\\)O)"} {"Type":"TOSS-UP\n\nMath - Short Answer","Prompt":"What is the period of the function with equation \\(y=3\\) sine open\n\nparenthesis \\(12\\pi x+4\\) close parenthesis minus 7?","Completion":"1\/6"} {"Type":"TOSS-UP\n\nEnergy - Multiple Choice","Prompt":"Oak Ridge National Lab has demonstrated that 3D-printed\n\nmagnets made from recycled materials can outperform those created by traditional methods.\n\nPermanent magnets made from rare-earth lanthanoid [LAN-tha-noyd] elements have partially occupied shells of what type?\n\nW) S\n\nX) P\n\nY) D\n\nZ) F","Completion":"Z) F"} {"Type":"TOSS-UP\n\nChemistry - Short Answer","Prompt":"Identify all of the following three acids that are diprotic acids:\n\nHydrochloric acid; Sulfuric acid; Phosphoric acid.","Completion":"2"} {"Type":"TOSS-UP\n\nEarth and Space - Short Answer","Prompt":"When CO\\({}{2}\\) from the atmosphere dissolves in ocean water, it quickly reacts with water to create what neutral acidic species?","Completion":"CARBONIC ACID (ACCEPT H\\({}{2}\\)CO\\({}{3}\\))"} {"Type":"TOSS-UP\n\nBiology - Short Answer","Prompt":"What is the term for the plant growth response to touch stimuli?","Completion":"THIGMOTROPISM"} {"Type":"TOSS-UP\n\nPhysics - Short Answer","Prompt":"Truth, beauty, and charm are flavors of what type of particle?","Completion":"QUARK (ACCEPT: ANTI-QUARK)"} {"Type":"TOSS-UP\n\nChemistry - Short Answer","Prompt":"Identify all of the following three forces that play a role in the interparticle interactions for hydrogen chloride: Hydrogen bonding; London dispersion; Dipole-dipole.","Completion":"2, 3"} {"Type":"TOSS-UP\n\nPhysics - Short Answer","Prompt":"Birefringence [bye-ree-FRIN-jets] is a phenomenon seen in materials whose index of refraction is dependent on the direction of travel and what other property of light?","Completion":"POLARIZATION"} {"Type":"TOSS-UP\n\nMath - Short Answer","Prompt":"What is the partial derivative with respect to \\(y\\) of the expression \\(x^{3}y^{7}\\)?","Completion":"\\(7x^{3}y^{6}\\)"} {"Type":"BONUS\n\nMath - Short Answer","Prompt":"Other than 5, what is the smallest positive integer that, when divided by 6, 8, 10, or 14, has a remainder of 5?","Completion":"845"} {"Type":"TOSS-UP\n\nEnergy - Short Answer","Prompt":"Researchers at Pacific Northwest National Lab are studying the relationship between soil carbon and microbial activity in soils. Most soil carbon is stored inorganically as part of what large group of minerals?","Completion":"CARBONATES"} {"Type":"BONUS\n\nEnergy - Multiple Choice","Prompt":"Fermi National Accelerator scientists are looking for physics beyond the Standard Model in the form of decay pathways that do not conserve lepton flavor. Which of the following particles has already been observed violating this conservation principle?\n\nW) Electrons\n\nX) Neutrinos\n\nY) Protons\n\nZ) Bosons","Completion":"X) NEUTRINOS13) Chemistry - Short Answer How many electrons in one atom of elemental sulfur are located in s-orbitals?"} {"Type":"TOSS-UP\n\nBiology - Multiple Choice","Prompt":"Which of the following organisms is haploid?\n\nW) Female grasshopper\n\nX) Male grasshopper\n\nY) Female ant\n\nZ) Male ant","Completion":"Z) MALE ANT"} {"Type":"BONUS\n\nBiology - Short Answer","Prompt":"The metal-ligating section of chlorophyll is based on what group of organic macrocycles that contain four pyrrole [PEER-ohl] rings?","Completion":"POPHYRIN"} {"Type":"TOSS-UP\n\nEarth and Space - Short Answer","Prompt":"The Chandra [CHAN-drah] Observatory observes radiation in what region of the electromagnetic spectrum?","Completion":"X-RAY"} {"Type":"BONUS\n\nEarth and Space - Short Answer","Prompt":"Place the following three natural satellites in order of increasing size: Moon; Phobos; Deimos.","Completion":"3, 2, 119) Math - Short Answer A geometric sequence has second term -5 and third term 3. What is its first term?"} {"Type":"TOSS-UP\n\nChemistry - Short Answer","Prompt":"Identify all of the following three techniques that could determine the optical purity of a sample: Polarimetry [polar-IH-meh-tree]; Thin-layer chromatography; Mass spectrometry.","Completion":"1"} {"Type":"TOSS-UP\n\nBiology - Short Answer","Prompt":"In alcohol fermentation, pyruvate is first decarboxylated to form what molecule?","Completion":"ACETALDEHYDE"} {"Type":"TOSS-UP\n\nMath - Short Answer","Prompt":"What is the greatest integer that is less than or equal to the log base 2 of 2019?","Completion":"10"} {"Type":"BONUS\n\nMath - Short Answer","Prompt":"If the sides of a triangle have lengths 5, 6, and 9, what is the cosine of the largest angle?","Completion":"-1\/3"} {"Type":"TOSS-UP\n\nPhysics - Short Answer","Prompt":"What principle states that the net response at a given point caused by two waves is the sum of the responses that those waves would have caused individually?","Completion":"SUPERPOSITION"} {"Type":"TOSS-UP\n\nBiology - Multiple Choice","Prompt":"The citric acid cycle is a component of what kind of metabolic process?\n\nW) Aerobic respiration\n\nX) Anaerobic respiration\n\nY) Fermentation\n\nZ) Photosynthesis","Completion":"W) AEROBIIC RESPIRATION"} {"Type":"TOSS-UP\n\nChemistry - Multiple Choice","Prompt":"In dilute aqueous solution, H\\(\\beta\\)SO\\({}{4}\\) is classified as which of the following?\n\nW) Strong acid\n\nX) Weak acid\n\nY) Strong base\n\nZ) Weak base","Completion":"W) STRONG ACID"} {"Type":"BONUS\n\nChemistry - Multiple Choice","Prompt":"What signs for enthalpy and entropy changes, respectively, guarantee a spontaneous reaction at all temperatures?\n\nW) Positive, positive\n\nX) Positive, negative\n\nY) Negative, positive\n\nZ) Negative, negative","Completion":"Y) NEGATIVE, POSITIVE"} {"Type":"TOSS-UP\n\nMath - Short Answer","Prompt":"What is the second derivative with respect to \\(x\\) of \\(2x^{3}-12\\) evaluated at \\(x=3\\)?","Completion":"36"} {"Type":"BONUS\n\nMath - Short Answer","Prompt":"Solve the following equation for \\(y\\): \\(\\underline{3v}\\) minus open parenthesis 2 minus \\(2v\\) close parenthesis plus \\(5=15\\) minus \\(y\\).","Completion":"2"} {"Type":"TOSS-UP\n\nEnergy - Short Answer","Prompt":"What is the primary gaseous element of wood gas, a product of gasification?","Completion":"NITROGEN"} {"Type":"TOSS-UP\n\nBiology - Multiple Choice","Prompt":"Which of the following processes is used to produce additional ATP for the light-independent reactions of photosynthesis?\n\nW) Cyclic electron flow\n\nX) Lactic acid fermentation\n\nY) Glycolysis [glye-KAWL-eh-sis]\n\nZ) Beta oxidation","Completion":"W) CYCLIC ELECTRON FLOW"} {"Type":"TOSS-UP\n\nPhysics - Short Answer","Prompt":"For an airplane in steady, level flight, what resisting force is equal and opposite to the vehicle thrust?","Completion":"DRAG (ACCEPT: AIR RESISTANCE, FRICTION)"} {"Type":"TOSS-UP\n\nMath - Multiple Choice","Prompt":"If \\(n\\) is an integer, and \\(x^{n}\\) is positive for all nonzero real numbers \\(x\\), which of the following must be true about \\(n\\)?\n\nW) \\(n\\) is greater than zero\n\nX) \\(n\\) does not equal zero\n\nY) \\(n\\) is even\n\nZ) \\(n\\) is odd","Completion":"Y) \\(n\\) IS EVEN"} {"Type":"TOSS-UP\n\nChemistry - Multiple Choice","Prompt":"Which of the following functional groups is most likely to be formed when dodecane [doh-DECK-ane] undergoes \"cracking?\"\n\nW) Alcohol\n\nX) Aldehyde\n\nY) Alkene\n\nZ) Amide","Completion":"Y) ALKENE"} {"Type":"## BONUS\n\nPhysics - Short Answer","Prompt":"To the nearest whole number of joules, what is the work done by gravity on a 500-gram ball that has rolled 20 meters down a 30-degree incline?","Completion":"49"} {"Type":"## BONUS\n\nEnergy - Short Answer","Prompt":"What is the most commonly used material for the anode in commercial lithium-ion batteries?","Completion":"GRAPHITE19) Earth and Space - Short Answer What subsurface phenomenon, defined as soil that is perennially frozen, occurs beneath much of the state of Alaska and, as it melts, causes extensive damage of roads, trains, and pipelines?"} {"Type":"TOSS-UP\n\nPhysics - Short Answer","Prompt":"A Carnot engine operates between a hot temperature of 527 degrees Celsius and a cold temperature of 127 degrees Celsius. What is its efficiency?","Completion":"50% (ACCEPT: 0.5, 1\/2)"} {"Type":"BONUS\n\nPhysics - Short Answer","Prompt":"Three resistors are connected in parallel to a 10-amp DC current source. If the individual resistances are 5 ohms, 8 ohms and 20 ohms, what is the power, to the nearest watt, dissipated by the circuit?","Completion":"267"} {"Type":"TOSS-UP\n\nBiology - Short Answer","Prompt":"Identify all of the following three statements that are true regarding lysosomes: Lysosomal enzymes are active at acidic pH; Lysosomes are implicated in inclusion-cell disease; Lysosomes are formed from late endosomes.","Completion":"1, 2 AND 3"} {"Type":"BONUS\n\nBiology - Short Answer","Prompt":"The term capnophile, used to describe a microbe, refers to the microbe's preference for environments high in what?","Completion":"CARBON DIOXIDE (ACCEPT: CARBON DIOXIDE GAS or CO\\({}{2}\\) or CO\\({}{2}\\) GAS)23) Earth and Space - Short Answer What geologic con extends from around 542 million years ago through the present day?"} {"Type":"TOSS-UP\n\nMath - Short Answer","Prompt":"What is the greatest common factor of 18 and 142?","Completion":"2"} {"Type":"TOSS-UP\n\nEnergy - Multiple Choice","Prompt":"Which of the following countries was NOT a major petroleum exporter to the United States in 2015?\n\nW) Canada\n\nX) Saudi Arabia\n\nY) Iran\n\nZ) Venezuela","Completion":"Y) IRAN"} {"Type":"TOSS-UP\n\nChemistry - Multiple Choice","Prompt":"If an acid is added to an unbuffered, neutral aqueous solution, which of the following is true?\n\nW) The hydroxide ion concentration increases\n\nX) The hydronium ion concentration decreases\n\nY) The hydroxide ion concentration decreases\n\nZ) The concentrations of the hydronium and hydroxide ion concentrations remain unchanged","Completion":"Y) THE HYDROXIDE ION CONCENTRATION DECREASES"} {"Type":"TOSS-UP\n\nPhysics - Multiple Choice","Prompt":"In millimeters, what must the focal length of the objective lens of a telescope be in order to produce a magnification of 300 when used with a 3-millimeter eye piece?\n\nW) 100\n\nX) 300\n\nY) 900\n\nZ) 1200","Completion":"Y) 900"} {"Type":"TOSS-UP\n\nBiology - Short Answer","Prompt":"How many chromosomes does a human gamete contain?","Completion":"23"} {"Type":"TOSS-UP\n\nEarth and Space - Short Answer","Prompt":"What mass movement of ocean water is referred to as the ocean conveyer belt?","Completion":"THERMOHALINE CIRULATION"} {"Type":"TOSS-UP\n\nBiology - Multiple Choice","Prompt":"Which of the following electrophoresis methods is used to analyze RNA?\n\nW) Northern Blot\n\nX) Southern Blot\n\nY) Eastern Blot\n\nZ) Western Blot","Completion":"W) NORTHERN BLOT"} {"Type":"TOSS-UP\n\nEarth and Space - Short Answer","Prompt":"Crater Lake in Oregon occupies what type of volcanic feature?","Completion":"CALDERA"} {"Type":"TOSS-UP\n\nEnergy - Short Answer","Prompt":"In large coal and nuclear power plants, the plant's power rating is measured in either \"megawatts e\" or \"megawatts t.\" What does \"t\" stand for?","Completion":"THERMAL"} {"Type":"TOSS-UP\n\nPhysics - Multiple Choice","Prompt":"Which of the following circuit elements resists instantaneous changes in the voltage applied to it?\n\nW) Inductor\n\nX) Capacitor\n\nY) Resistor\n\nZ) Transformer","Completion":"X) CAPACITOR"} {"Type":"TOSS-UP\n\nMath - Short Answer","Prompt":"What is the discriminant of the quadratic function \\(y=x^{2}-4x+10\\)?","Completion":"-24"} {"Type":"BONUS\n\nMath - Short Answer","Prompt":"What is the derivative evaluated at \\(x=7\\) of the fraction with numerator \\(x^{3}-1\\) and denominator \\(x-1\\)?","Completion":"15"} {"Type":"TOSS-UP\n\nChemistry - Short Answer","Prompt":"At what temperature, in degrees Celsius, does the change in enthalpy for the oxidation of 1 mole of graphite under 1 atmosphere of oxygen to generate carbon dioxide exactly equal the standard enthalpy of formation of carbon dioxide?","Completion":"25"} {"Type":"BONUS\n\nChemistry - Short Answer","Prompt":"Identify all of the following three compounds that have sp\\({}^{2}\\)[S-P-2] hybridized central atoms: Nitrogen trichloride; Carbon dioxide; Hydrocyanic acid.","Completion":"NONE OF THEM"} {"Type":"## BONUS\n\nPhysics - Multiple Choice","Prompt":"Given that it relates the energy of a photon to its frequency, what are the SI units of Planck's constant?\n\nW) Joule seconds\n\nX) Watts\n\nY) Newton meters\n\nZ) Watts per second","Completion":"W) JOULE SECONDS"} {"Type":"## BONUS\n\nChemistry - Multiple Choice","Prompt":"A protein chemist isolates a new enzyme. Which of the following techniques, when performed alone, would be most useful for determining its size?\n\nW) NMR spectroscopy\n\nX) MALDI-TOF mass spectrometry\n\nY) Elemental analysis\n\nZ) Gravimetric analysis","Completion":"X) MALDI-TOF MASS SPECTROMETRY"} {"Type":"TOSS-UP\n\nEarth and Space - Multiple Choice","Prompt":"Which of the following is a sedimentary rock?\n\nW) Scoria\n\nX) Granite\n\nY) Basalt\n\nZ) Shale","Completion":"Z) SHALE"} {"Type":"TOSS-UP\n\nEnergy - Short Answer","Prompt":"What one-carbon alcohol can be used directly as fuel in flex-fuel cars due to its high octane rating?","Completion":"METHANOL"} {"Type":"TOSS-UP\n\nBiology - Multiple Choice","Prompt":"Topoisomerases [toh-poh-eye-SAW-mer-aces] are involved in which of the following processes?\n\nW) DNA transcription\n\nX) DNA replication\n\nY) RNA translation\n\nZ) RNA processing","Completion":"X) DNA Replication"} {"Type":"TOSS-UP\n\nMath - Short Answer","Prompt":"Evaluate the following expression: log base 3 of \\(27+\\log\\) base 5 of 25.","Completion":"5"} {"Type":"BONUS\n\nEarth and Space - Multiple Choice","Prompt":"Jupiter appears brightest in which of the following configurations relative to the Earth?\n\nW) Conjunction\n\nX) Quasi conjunction [KWAH-eye kon-JUNK-shun]\n\nY) Quadrature\n\nZ) Opposition","Completion":"Z) OPPOSITION"} {"Type":"TOSS-UP\n\nMath - Short Answer","Prompt":"What is the value of 60 factorial divided by open parenthesis 59 factorial times 3 factorial close parenthesis?","Completion":"10"} {"Type":"BONUS\n\nMath - Short Answer","Prompt":"Identify all of the following three integer operations that are commutative: Addition; Subtraction; Multiplication.","Completion":"1 AND 3"} {"Type":"TOSS-UP\n\nBiology - Multiple Choice","Prompt":"Which of the following hormones is a precursor of both estrogen and testosterone?\n\nW) Cortisol\n\nX) Ergosterol [ur-GAWS-tur-aw]\n\nY) Aldosterone [al-DOSS-ter-own]\n\nZ) DHEA","Completion":"Z) DHEA"} {"Type":"TOSS-UP\n\nPhysics - Short Answer","Prompt":"Carbon-10 has a half-life of approximately 19 seconds.\n\nApproximately how long, in seconds, will it take for a sample of carbon-10 to decay to one-sixteenth of its original size?","Completion":"76"} {"Type":"TOSS-UP\n\nChemistry - Multiple Choice","Prompt":"An aqueous solution contains silver ions, barium ions, iron (III) ions, and nitrate ions. Upon addition of sodium chloride, which of the following will precipitate out of the solution first?\n\nW) Iron (III) chloride\n\nX) Barium chloride\n\nY) Sodium nitrate\n\nZ) Silver chloride","Completion":"Z) SILVER CHLORIDE"} {"Type":"BONUS\n\nChemistry - Short Answer","Prompt":"Other than hydrogen and the noble elementsWhat non-noble element has the lowest energy valence orbitals?","Completion":"FLUORINE"} {"Type":"TOSS-UP\n\nEarth and Space - Multiple Choice","Prompt":"Which of the following geologic eons lasted the longest?\n\nW) Hadean\n\nX) Archean [ar-KEE-en]\n\nY) Proterozoic [proh-tair-oh-ZOH-ik]\n\nZ) Phanerozoic [fah-NAIR-oh-zh-ik]","Completion":"Y) PROTEROZOIC"} {"Type":"BONUS\n\nEarth and Space - Short Answer","Prompt":"Identify all of the following three factors that threaten\n\nCalifornia's water supplies: Slow reservoir recharge rates; High agricultural water\n\nconsumption; Strike-slip faults intersecting major aquducts.","Completion":"ALL OF THEM"} {"Type":"TOSS-UP\n\nBiology - Multiple Choice","Prompt":"In what organ are dietary fats primarily absorbed?\n\nW) Small intestine\n\nX) Large intestine\n\nY) Stomach\n\nZ) Liver","Completion":"W) SMALL INTEREST"} {"Type":"TOSS-UP\n\nChemistry - Multiple Choice","Prompt":"O\\({}{2}\\) and O\\({}{3}\\) are considered what type of molecules?\n\nW) Allotropes\n\nX) Structural isomers\n\nY) Isotopes\n\nZ) Geometrical isomers","Completion":"W) ALLORTOPES"} {"Type":"TOSS-UP\n\nEnergy - Short Answer","Prompt":"Order the following three lightbulb technologies from least to most efficient: Light-emitting diode; Incandescent; Compact fluorescent.","Completion":"2, 3, 1"} {"Type":"TOSS-UP\n\nPhysics - Multiple Choice","Prompt":"A rock on the end of a string is being swung in a circle above a child's head. The string breaks. Neglecting gravity, in what direction does the acceleration of the rock now point?\n\nW) Tangent to the circle\n\nX) Inwards towards the center of the circle\n\nY) Outwards away from the center of the circle\n\nZ) The acceleration is now zero","Completion":"Z) THE ACCELERATION IS NOW ZERO"} {"Type":"TOSS-UP\n\nMath - Short Answer","Prompt":"What is the midpoint of the segment with endpoints (1, and (5, -?","Completion":"(3, 2)"} {"Type":"BONUS\n\nPhysics - Short Answer","Prompt":"How many lumens does an isotropic source of 1 candela emit into the upper half plane?","Completion":"2\\(\\pi\\)"} {"Type":"TOSS-UP\n\nChemistry - Short Answer","Prompt":"Part of the reason that graphene demonstrates such high mechanical strength is that the carbon atoms are in what hybridization state?","Completion":"SP\\({}^{2}\\)"} {"Type":"BONUS\n\nChemistry - Short Answer","Prompt":"Order the following three gases from slowest to fastest in terms of their rate of effusion at 25 degrees Celcius: Helium; Argon; Neon.","Completion":"2, 3, 1"} {"Type":"TOSS-UP\n\nMath - Short Answer","Prompt":"Specifying whether your answer is open or closed, what is the interval of angles in degrees whose terminal sides are in the second quadrant?","Completion":"THE OPEN INTERVAL FROM 90 TO 180"} {"Type":"BONUS\n\nMath - Multiple Choice","Prompt":"Which of the following base 2 sums is the largest?\n\nW) \\(1+1111\\) [one plus one one one one one]\n\nX) \\(100+1001\\) [one zero zero plus one zero zero one]\n\nY) \\(110+1000\\) [one one zero plus one zero zero zero]\n\nZ) \\(111+111\\) [one one one plus one one one]","Completion":"W) \\(1+1111\\)"} {"Type":"TOSS-UP\n\nEnergy - Multiple Choice","Prompt":"Which of the following is NOT an advantage of green roofs?\n\nW) Improved waterproofing\n\nX) Improved lifespan of roof\n\nY) Reduced heating costs\n\nZ) Reduced cooling costs","Completion":"W) IMPROVED WATPROOFING"} {"Type":"BONUS\n\nEnergy - Multiple Choice","Prompt":"Which of the following is NOT a desirable quality in the material used to coat nuclear fuel rods?\n\nW) Corrosion resistant\n\nX) High neutron absorption\n\nY) High hardness\n\nZ) Low reactivity to water","Completion":"X) HIGH NEUTRON ABSORPTION"} {"Type":"TOSS-UP\n\nBiology - Multiple Choice","Prompt":"Which of the following structures is referred to as our body's internal gyroscope?\n\nW) Inner ears\n\nX) Eyes\n\nY) Outer ears\n\nZ) Nose","Completion":"W) INNER EARS"} {"Type":"TOSS-UP\n\nEarth and Space - Multiple Choice","Prompt":"Which of the following best describes the relationship between the ages of the Earth, the Sun, and the Universe?\n\nW) The Universe is about 3 times as old as the Sun, which is about 3 times as old as the Earth\n\nX) The Universe is the same age as the Sun, which is about 3 times as old as the Earth\n\nY) The Universe is about 3 times as old as the Sun and the Earth, which are the same age\n\nZ) The Universe, the Earth, and the Sun are all the same age","Completion":"Y) THE UNIVERSE IS ABOUT 3 TIMES AS OLD AS THE SUN AND THE"} {"Type":"TOSS-UP\n\nBiology \\(-\\) Multiple Choice","Prompt":"What material typically constitutes the cell wall in fungi?\n\nW) Cellulose\n\nX) Peptidoglycan\n\nY) Pectin\n\nZ) Chitin","Completion":"Z) CHITIN"} {"Type":"TOSS-UP\n\nPhysics \\(-\\) Multiple Choice","Prompt":"Which of the following is the equivalent resistance, in ohms, of a circuit in which a 5 ohm resistor is connected in series to two 4 ohm resistors which are connected in parallel?\n\nW) 10\/7\n\nX) 40\/13\n\nY) 7\n\nZ) 13","Completion":"Y) 7"} {"Type":"TOSS-UP\n\nEarth and Space - Short Answer","Prompt":"What is the nearest galaxy cluster?","Completion":"VIRGO"} {"Type":"TOSS-UP\n\nMath - Short Answer","Prompt":"What is the limit as \\(x\\) approaches 3 of the function with numerator \\(x\\) - 3 and denominator \\(x^{2}-9\\)?","Completion":"1\/6"} {"Type":"TOSS-UP\n\nChemistry - Multiple Choice","Prompt":"The bond angle in NH\\({}{3}\\) is closest to which of the following degree values?\n\nW) 90\n\nX) 110\n\nY) 120\n\nZ) 180","Completion":"X) 110"} {"Type":"TOSS-UP\n\nEnergy - Short Answer","Prompt":"The Athabasca oil sands, the largest bitumen deposit in the world, are located in what country?","Completion":"CANADA"} {"Type":"TOSS-UP\n\nMath - Short Answer","Prompt":"Given f(x) = 3\\(x\\) - 5 and g(x) = 4\\(x\\), what is \\(f\\) of \\(g\\) of x?","Completion":"12\\(x\\) - 5"} {"Type":"BONUS\n\nMath - Short Answer","Prompt":"What is the least common multiple of the first four composite numbers?","Completion":"72"} {"Type":"TOSS-UP\n\nChemistry - Multiple Choice","Prompt":"An atom is sp\\({}^{3}\\)[S-P-3] hybridized. Which of the following is NOT a possible molecular geometry about this atom?\n\nW) Bent\n\nX) Trigonal pyramidal\n\nY) Tetrahedral\n\nZ) Trigonal bipyramidal","Completion":"Z) TRIGONAL Bipyramidal"} {"Type":"BONUS\n\nChemistry - Short Answer","Prompt":"What region of the electromagnetic spectrum would be able to excite an electron from a 1s orbital to a 2p orbital in a helium atom?","Completion":"ULTRAVIOLET (ACCEPT: UV)"} {"Type":"TOSS-UP\n\nBiology - Multiple Choice","Prompt":"Which of the following describes antibody-dependent cell-mediated cytotoxicity?\n\nW) An invertebrate host defense mechanism\n\nX) Achieved via the classical complement pathway\n\nY) Mediated by Fc receptors on NK cells and myeloid leukocytes\n\nZ) The primary cytotoxic pathway mediated by T cells","Completion":"Y) MEDIATED BY FC RECEPTORS ON NK CELLS AND MYELOID"} {"Type":"TOSS-UP\n\nPhysics - Multiple Choice","Prompt":"A student at the edge of a merry-go-round travels along an arc of length 6.28 meters as the ride completes a pi\/4 angular rotation. To the nearest meter, what is the radius of the ride?\n\nW) 2\n\nX) 4\n\nY) 8\n\nZ) 16","Completion":"Y) 8"} {"Type":"TOSS-UP\n\nEarth and Space - Multiple Choice","Prompt":"Which of the following phenomena is a buildup of ocean water that is pushed towards land by a hurricane?\n\nW) Tsunami\n\nX) Surge tide\n\nY) Storm surge\n\nZ) Rogue wave","Completion":"Y) STORM SURGE"} {"Type":"TOSS-UP\n\nPhysics - Multiple Choice","Prompt":"What is the arc length, in miles, traveled by a rocket along a circular path whose angle measures 0.2 radians with a radius of 80 miles?\n\nW) 16\n\nX) 40\n\nY) 80\n\nZ) 400","Completion":"W) 16"} {"Type":"BONUS\n\nPhysics - Short Answer","Prompt":"What is the name for the amount of time it takes for a wave to complete a single cycle?","Completion":"PERIOD"} {"Type":"TOSS-UP\n\nEarth and Space - Multiple Choice","Prompt":"Which of the following planets has rings?\n\nW) Mars\n\nX) Mercury\n\nY) Neptune\n\nZ) Venus","Completion":"Y) NEPTUNE"} {"Type":"BONUS\n\nEarth and Space - Short Answer","Prompt":"Put the following four minerals in order from softest to hardest on the Mohs scale: Apatite; Fluorite; Quartz; Corundum.","Completion":"2, 1, 3, 4 (ACCEPT: FLUORITE, APATITE, QUARTZ, CORUNDUM)"} {"Type":"TOSS-UP\n\nEnergy - Short Answer","Prompt":"What type of renewable fuel source consists of long alkyl chain esters produced from biolipids?","Completion":"BIODIESEL"} {"Type":"TOSS-UP\n\nChemistry - Short Answer","Prompt":"What is the name of the oxyanion [ox-ee-AN-eye-on] of bromine in which bromine has a +7 oxidation state and formula BrO\\({}{4}\\)- [B-R-oh-4-negative]?","Completion":"PERBROMATE (ACCEPT: PERBROMATE ION)"} {"Type":"TOSS-UP\n\nBiology - Multiple Choice","Prompt":"Maple syrup is extracted from maple trees by tapping what part of the plant?\n\nW) Xylem [ZYE-lum]\n\nX) Phloem [FLOW-em]\n\nY) Cambium [KAM-bee-um]\n\nZ) Bark","Completion":"W) XYLEM"} {"Type":"TOSS-UP\n\nMath - Short Answer","Prompt":"The ratio of the measures of the three angles in a triangle is 2:3:4. What are the degree measures of the three angles?","Completion":"40, 60, 80"} {"Type":"TOSS-UP\n\nChemistry - Short Answer","Prompt":"What alcohol reactant is needed to perform a Fischer esterification on a carboxylic acid to yield a methyl ester?","Completion":"METHANOL"} {"Type":"TOSS-UP\n\nEnergy - Short Answer","Prompt":"What operating situation, characterized by a circuit with very little to no electrical impedence, can cause lithium ion batteries to generate a lot of heat, sometimes causing fires?","Completion":"SHORT CIRCUIT"} {"Type":"TOSS-UP\n\nPhysics - Short Answer","Prompt":"Identify all of the following four forces that are real in inertial reference frames: Centrifugal; Centripetal; Lorentz; Coriolis.","Completion":"2, 3 (ACCEPT: CENTRIPETAL, LORENTZ)"} {"Type":"TOSS-UP\n\nEarth and Space - Short Answer","Prompt":"What volcano hosts the world's primary benchmark site for atmospheric carbon dioxide measurements and the observatory that produced the original Keeling curve?","Completion":"MAUNA LOA"} {"Type":"TOSS-UP\n\nBiology \\(-\\) Multiple Choice","Prompt":"Which of the following vitamins is NOT synthesized by plants or animals?\n\nW) B12\n\nX) B6\n\nY) B2\n\nZ) C","Completion":"W) B12"} {"Type":"TOSS-UP\n\nMath \\(-\\) Multiple Choice","Prompt":"With respect to \\(x\\), which of the following has \\(-3x^{2}\\) as an antiderivative?\n\nW) \\(-6x\\)\n\nX) \\(6x\\)\n\nY) \\(-x^{3}\\)\n\nZ) \\(x^{3}\\)","Completion":"W) \\(-6x\\)"} {"Type":"TOSS-UP\n\nChemistry - Multiple Choice","Prompt":"Nitrogen monoxide is an example of which of the following?\n\nW) An expanded octet\n\nX) A resonance molecule\n\nY) A dimerizing molecule\n\nZ) A free radical","Completion":"Z) A FREE RADICAL"} {"Type":"TOSS-UP\n\nEnergy - Multiple Choice","Prompt":"One 42-gallon barrel of crude oil produces approximately how many US gallons of gasoline?\n\nW) 10\n\nX) 20\n\nY) 30\n\nZ) 40","Completion":"X) 20"} {"Type":"TOSS-UP\n\nBiology \\(-\\) Short Answer","Prompt":"What form of active transport involves moving two or more different molecules across a membrane in the same direction, coupling the favorable transport of one molecule to the unfavorable transport of the other?","Completion":"SYMPORT"} {"Type":"BONUS\n\nBiology \\(-\\) Multiple Choice","Prompt":"Mycobacterium [MY-koh-bacterium] is a genus of slow-growing and difficult-to-eradicate bacteria responsible for what two human diseases?\n\nW) Mononucleosis and pertussis [per-TUH-sis]\n\nX) Shigella [shih-GEHL-ah]and malaria\n\nY) Gingivitis and impetigo [im-puh-tie-goh]\n\nZ) Leprosy and tuberculosis","Completion":"Z) LEPROSY AND TUBERCULOSIS"} {"Type":"TOSS-UP\n\nPhysics \\(-\\) Short Answer","Prompt":"Identify all of the following three statements that are always true about resistors in parallel: The equivalent resistance is greater than any individual resistance; The voltage across each resistor is the same; The current through each resistor is the same.","Completion":"2"} {"Type":"BONUS\n\nPhysics \\(-\\) Short Answer","Prompt":"An object is thrown with a horizontal velocity of 20 meters per second from a height of 125 meters above level ground. If air resistance is negligible, to the nearest whole number, how many seconds does it take the object to fall to the ground?","Completion":"5"} {"Type":"TOSS-UP\n\nChemistry - Multiple Choice","Prompt":"What is the coordination number of a body-centered cubic lattice?\n\nW) 2\n\nX) 4\n\nY) 6\n\nZ) 8","Completion":"Z) 8"} {"Type":"TOSS-UP\n\nBiology - Multiple Choice","Prompt":"Bacteria are able to become resistant to antibiotics by increasing their genetic variability through all of the following mechanisms EXCEPT:\n\nW) Conjugation\n\nX) Transduction\n\nY) Binary fission\n\nZ) Transformation","Completion":"Y) BINARY FISSION"} {"Type":"TOSS-UP\n\nPhysics - Short Answer","Prompt":"In volts per meter, what is the electric field at the center of a hollow metallic sphere with a radius of 10 centimeters when the center of the sphere is 3 meters from a particle with a charge of 5 coulombs?","Completion":"0"} {"Type":"TOSS-UP\n\nMath - Short Answer","Prompt":"How many exterior angles does a parallelogram have?","Completion":"8"} {"Type":"BONUS\n\nEarth and Space - Short Answer","Prompt":"A stream is referred to as a \"gaining stream\" when its surface is below what groundwater feature?","Completion":"WATER TABLE"} {"Type":"TOSS-UP\n\nEarth and Space - Multiple Choice","Prompt":"Which of the following elements most often limits primary production in freshwater lakes?\n\nW) Oxygen\n\nX) Calcium\n\nY) Carbon\n\nZ) Phosphorus","Completion":"Z) PHOSPHORUS"} {"Type":"BONUS\n\nEarth and Space - Short Answer","Prompt":"Identify all of the following three statements that are true of wooly mammoths: They once lived in North America; They had ivory tusks; They evolved in the Devonian Period.","Completion":"1 AND 2"} {"Type":"TOSS-UP\n\nBiology - Multiple Choice","Prompt":"One species resembles another species that is poisonous. What is this phenomenon called?\n\nW) Commensalism [kah-MEN-sul-ism]\n\nX) Mutualism\n\nY) Batesian mimicry [BAIT-see-in MIM-ih-kree]\n\nZ) Mullerian mimicry [muhl-AIR-ee-an MIM-ih-kree]","Completion":"Y) BATESIIAN MIMICRY"} {"Type":"BONUS\n\nBiology - Multiple Choice","Prompt":"What human organ utilizes the most glucose in the body?\n\nW) Liver\n\nX) Heart\n\nY) Lungs\n\nZ) Brain","Completion":"Z) BRAIN"} {"Type":"TOSS-UP\n\nPhysics - Short Answer","Prompt":"To three significant figures, at what temperature, in kelvins, does pure water boil at sea level?","Completion":"373"} {"Type":"TOSS-UP\n\nMath - Short Answer","Prompt":"What is the degree measure of each interior angle in a regular hexagon?","Completion":"120"} {"Type":"TOSS-UP\n\nChemistry - Multiple Choice","Prompt":"What is the maximum number of electrons that the \\(3d\\) level can hold?\n\nW) 2\n\nX) 6\n\nY) 10\n\nZ) 14","Completion":"Y) 10"} {"Type":"TOSS-UP\n\nEnergy - Short Answer","Prompt":"What refinery process is used to separate components of petroleum based on their boiling points?","Completion":"DISTILLATION (ACCEPT: FRACTIONAL DISTILLATION)"} {"Type":"TOSS-UP\n\nBiology - Multiple Choice","Prompt":"Plant cells are distinguished from animal cells by having:\n\nW) A large central vacuole, chloroplasts, and a cell wall made of cellulose\n\nX) A large central vacuole, mitochondria, and a nucleus\n\nY) A cell wall made of cellulose, Golgi apparatus, and chloroplasts\n\nZ) Chloroplasts, mitochondria, and a nucleus","Completion":"W) A LARGE CENTRAL VACUOLE, CHLOROPLASTS, AND A CELL WALL"} {"Type":"TOSS-UP\n\nMath - Multiple Choice","Prompt":"Which of the following equations is that of a line perpendicular to the line with equation \\(4x-5y=10\\)?\n\nW) \\(4x-5y=12\\)\n\nX) \\(4x+5y=12\\)\n\nY) \\(5x-4y=10\\)\n\nZ) \\(5x+4y=10\\)","Completion":"Z) \\(5x+4y=10\\)"} {"Type":"## BONUS\n\nEarth and Space - Multiple Choice","Prompt":"Addition of which of the following compounds may promote carbon sequestration in the ocean?\n\nW) Potassium hydroxide\n\nX) Carbonic acid\n\nY) Hydrochloric acid\n\nZ) Sulfur","Completion":"W) POTASSIUM HYDROXIDE"} {"Type":"## BONUS\n\nEnergy - Multiple Choice","Prompt":"Which of the following is NOT a type of nuclear reactor currently used in the United States?\n\nW) Boiling water\n\nX) Solid sodium\n\nY) Pressurized water\n\nZ) Liquid metal breeder","Completion":"X) SOLID SODIUM"} {"Type":"TOSS-UP\n\nBiology \\(-\\) Short Answer","Prompt":"Liverwords are a part of which nonvascular plant phylum?","Completion":"Marchantiophytta (ACCEPT: HEPATOPHYTA or HEPATICOPHYTA)"} {"Type":"TOSS-UP\n\nMath \\(-\\) Short Answer","Prompt":"What is the degree of the polynomial \\(x^{2}+3x^{8}+7x\\)?","Completion":"8"} {"Type":"BONUS\n\nEarth and Space - Multiple Choice","Prompt":"Which of the following is NOT true of tropical forest soils in relation to temperate forest soils?\n\nW) Tropical forest soils are richer in nutrients\n\nX) Tropical forest soils experience more intense leaching\n\nY) Temperate forest soils retain more organic matter\n\nZ) Temperate forest soils are thicker","Completion":"W) Tropical FOREST Soils ARE RICHER IN NUTRIENTS"} {"Type":"TOSS-UP\n\nChemistry - Multiple Choice","Prompt":"Under which of the following conditions does the behavior of real gases deviate most from that predicted by the Ideal Gas Law?\n\nW) Low pressure, low temperature\n\nX) High pressure, low temperature\n\nY) Low pressure, high temperature\n\nZ) High pressure, high temperature","Completion":"X) HIGH PRESSURE, LOW TEMPERATURE"} {"Type":"BONUS\n\nChemistry - Multiple Choice","Prompt":"Which of the following best explains why NaCl has a higher melting point than HCl?\n\nW) HCl can form hydrogen bonds, while NaCl cannot\n\nX) HCl has stronger London dispersion forces\n\nY) HCl is a molecular compound, while NaCl is an ionic compound\n\nZ) HCl is a network compound, while NaCl is a molecular compound","Completion":"Y) HCl is A MOLECULAR COMPOUND, WHILE NaCl IS AN IONIC COMPOUND17) Physics - Short Answer A block rests upon an incline of an unknown angle. If the coefficient of static friction between the block and the incline is one-third times the square root of 3, and the block is just barely held by friction, what is the angle of the incline to the horizontal, to the nearest degree?"} {"Type":"TOSS-UP\n\nEnergy - Multiple Choice","Prompt":"Demand Side Management, as part of smart grids, is a combination of programs that allows which of the following?\n\nW) Customers to change their energy consumption patterns based on a varying rate of the cost of electricity\n\nX) Generators to change their production patterns based on a varying rate of the cost of electricity\n\nY) Utilities to refund money to customers\n\nZ) Meter readers to be replaced by automated smart electricity meters","Completion":"W) CUSTOMERS TO CHANGE THEIR ENERGY CONSUMPTION PATTERNS BASED ON A VARYING RATE OF THE COST OF ELECTRICITY"} {"Type":"TOSS-UP\n\nEarth and Space \\(-\\) Short Answer","Prompt":"In what constellation is our Galactic center located?","Completion":"SAGITTARIUS"} {"Type":"BONUS\n\nEarth and Space \\(-\\) Multiple Choice","Prompt":"Which of the following best describes the composition of Earth's core?\n\nW) 78% silicon oxide\n\nX) Mainly iron silicates\n\nY) An iron-nickel alloy\n\nZ) Mainly iron and potassium compounds","Completion":"Y) AN IRON-NICKEL ALLOY"} {"Type":"TOSS-UP\n\nBiology \\(-\\) Multiple Choice","Prompt":"If the only method of introducing genetic variation into a sample population of E. coli is by spontaneous mutation, which of the following is the best way to increase the prevalence of beneficial mutations over many generations?\n\nW) Conjugation\n\nX) Binary fission\n\nY) Transduction\n\nZ) Horizontal gene transfer","Completion":"X) BINARY FISSION"} {"Type":"BONUS\n\nBiology \\(-\\) Short Answer","Prompt":"What is the name for skeletal muscle cells containing multiple nuclei?","Completion":"SYNCYTIUM (ACCEPT: SYMPLASM)"} {"Type":"TOSS-UP\n\nPhysics - Short Answer","Prompt":"What is the name given to light produced when a charged particle passes through a material at a speed faster than the phase velocity of light in that material?","Completion":"CHERENKOV RADIATION (ACCEPT: VALIVOV-CHERENKOV RADIATION)"} {"Type":"BONUS\n\nPhysics - Multiple Choice","Prompt":"Ignoring temperature changes, how much of a pressure increase, in pascals, does a diver feel at 100 meters below the surface of a fresh-water lake?\n\nW) 9,800\n\nX) 98,000\n\nY) 980,000\n\nZ) 9,800,000","Completion":"Y) 980,000"} {"Type":"TOSS-UP\n\nChemistry - Multiple Choice","Prompt":"Which type of hybridization is found in the central atom of carbon dioxide?\n\nW) sp [s-p]\n\nX) sp [s-p-2]\n\nY) sp3 [s-p-3]\n\nZ) sp4 [s-p-4]","Completion":"W) SP"} {"Type":"BONUS\n\nChemistry - Short Answer","Prompt":"What type of fluid undergoes large changes in density in response to small changes in pressure and temperature, and can be formed by heating carbon dioxide above 31 degrees Celsius at pressures above 72.8 atmospheres?","Completion":"SUPERCRITICAL FLUID"} {"Type":"TOSS-UP\n\nMath - Short Answer","Prompt":"The complex number -4 lies on the boundary between what two quadrants in the complex plane?","Completion":"2 AND 3 (ACCEPT: SECOND AND THIRD)"} {"Type":"BONUS\n\nMath - Short Answer","Prompt":"Solve the following equation for \\(x\\): negative 8 plus the absolute value of open parenthesis 4 minus \\(3x\\) close parenthesis equals 20.","Completion":"-8, 32\/3 (ACCEPT: 10 2\/3 for 32\/3)"} {"Type":"TOSS-UP\n\nBiology - Multiple Choice","Prompt":"A plant has a somatic chromosome number of 2n = 22. After a cell in this plant undergoes mitosis, how many chromosomes will the daughter cells each have?\n\nW) 11\n\nX) 22\n\nY) 33\n\nZ) 44","Completion":"X) 22"} {"Type":"BONUS\n\nBiology - Multiple Choice","Prompt":"SDS is a detergent. Which of the following explains how SDS\n\ndenatures proteins?\n\nW) Interferes with the hydrophobic interactions that normally stabilize the proteins\n\nX) Interferes with the hydrophillic interactions that normally stabilize the proteins\n\nY) Interferes with the amphiphillic [amfih-FILL-ic] interactions that normally stabilize the proteins\n\nZ) Breaks the peptide bonds","Completion":"W) INTERFERES WITH THE HYDROphobic INTERACTIONS THAT"} {"Type":"TOSS-UP\n\nChemistry - Short Answer","Prompt":"What is the only alkali metal that forms a stable nitride at room\n\ntemperature and standard pressure?","Completion":"LITHIUM"} {"Type":"BONUS\n\nChemistry - Multiple Choice","Prompt":"Which of the following biological macromolecules does NOT contain nitrogen?\n\nW) DNA\n\nX) RNA\n\nY) Insulin\n\nZ) Amylose [AM-il-ohse]","Completion":"Z) AMYLOSE"} {"Type":"TOSS-UP\n\nEnergy - Short Answer","Prompt":"What is the name of the program that the Environmental Protection Agency maintains in an effort to identify and promote energy-efficient products and buildings, thus reducing pollution and waste?","Completion":"ENERGY STAR"} {"Type":"TOSS-UP\n\nMath - Short Answer","Prompt":"What is binary one-zero-zero-one in decimal form?","Completion":"9"} {"Type":"TOSS-UP\n\nPhysics - Multiple Choice","Prompt":"What is the name for a particle that carries a half-integer spin?\n\nW) Hadron\n\nX) Boson\n\nY) Fermion\n\nZ) Proton","Completion":"Y) FERMION"} {"Type":"TOSS-UP\n\nEarth and Space - Multiple Choice","Prompt":"The majority of Earth's freshwater is found in which of the following?\n\nW) The Pacific Ocean\n\nX) Ice and snow\n\nY) Subsurface streams\n\nZ) The atmosphere as water vapor","Completion":"X) ICE AND SNOW"} {"Type":"TOSS-UP\n\nChemistry - Multiple Choice","Prompt":"Which of the following atoms has at least one completely filled p-subshell?\n\nW) Sulfur\n\nX) Oxygen\n\nY) Boron\n\nZ) Helium","Completion":"W) SULFUR"} {"Type":"TOSS-UP\n\nEnergy - Multiple Choice","Prompt":"Which of the following procedures is NOT commonly used during drilling to recover more oil from the ground?\n\nW) Filling the well with water\n\nX) Pumping nitrogen or carbon dioxide into the well\n\nY) Injecting polymers into the well\n\nZ) Creating a vacuum in the well","Completion":"Z) CREATING A VACUUM IN THE WELL"} {"Type":"TOSS-UP\n\nEarth and Space - Short Answer","Prompt":"The precession of the equinoxes is due principally to the torque induced on the Earth by what celestial body?","Completion":"THE MOON"} {"Type":"TOSS-UP\n\nMath - Multiple Choice","Prompt":"For a twice-differentiable function f(x), f(is 4, f prime of 3 is 0 and f double prime of 3 is 5. Which of the following is true of the graph of \\(y\\) =f(x) at the point (3, ?\n\nW) There is a local minimum\n\nX) There is a local maximum\n\nY) There is an inflection point\n\nZ) The graph is concave down","Completion":"W) THERE IS A LOCAL MINIMUM"} {"Type":"TOSS-UP\n\nBiology - Multiple Choice","Prompt":"The water-proof layer within the endodermis of the root that filters substances entering a plant is called the:\n\nW) Pericycle\n\nX) Hypodermis\n\nY) Casparian strip\n\nZ) Vascular cambium","Completion":"Y) CASPARIAN STRIP"} {"Type":"TOSS-UP\n\nPhysics - Short Answer","Prompt":"Fiber optic cables have revolutionized the information age with the ability to transfer data at much higher rates than conventional electric cables. Fiber optic cables work because of total internal reflection, which results from a difference in what optical property between the cable and surrounding medium?","Completion":"REFRACTIVE INDEX (ACCEPT: INDEX OF REFRACTION; DO NOT ACCEPT: SPEED OF LIGHT)"} {"Type":"TOSS-UP\n\nChemistry - Multiple Choice","Prompt":"What type of magnetism opposes an applied magnetic field and is dominant in materials with no unpaired electrons?\n\nW) Diamagnetism\n\nX) Paramagnetism\n\nY) Ferromagnetism\n\nZ) Antiferromagnetism","Completion":"W) DIAMAGENETISM"} {"Type":"TOSS-UP\n\nMath - Short Answer","Prompt":"What is the sine of 270 degrees?","Completion":"-1"} {"Type":"TOSS-UP\n\nBiology \\(-\\) Short Answer","Prompt":"In female mammalian cells, the second X chromosome is generally silenced. What term is used to describe the condensed inactive X chromosome?","Completion":"BARR BODY"} {"Type":"BONUS\n\nBiology \\(-\\) Multiple Choice","Prompt":"If you needed to find a sample of simple cuboidal epithelium [ehiph-THEEL-ee-um] in the human body, you would want tissue from which of the following organs?\n\nW) Kidneys\n\nX) Lungs\n\nY) Esophagus\n\nZ) Stomach","Completion":"W) KIDNEYS"} {"Type":"TOSS-UP\n\nEarth and Space \\(-\\) Multiple Choice","Prompt":"Which of the following cloud types is strongly associated with thunderstorms?\n\nW) Cirrocumulus [seer-oh-KYOOM-you-lus]\n\nX) Cumulonimbus [KYOOM-yoo-loh-NIM-bus]\n\nY) Altostratus\n\nZ) Stratocumulus","Completion":"X) CUMULONIMBUS"} {"Type":"BONUS\n\nEarth and Space \\(-\\) Short Answer","Prompt":"The Hubble Deep Field is an image of a seemingly empty spot in the sky in what constellation?","Completion":"URSA MAJOR (DO NOT ACCEPT: BIG DIPPER)17) Energy - Multiple ChoiceWhich of the following numbers is the best estimate for the number of kilowatt hours of energy used by an average U.S. home in 2014?"} {"Type":"TOSS-UP\n\nChemistry - Multiple Choice","Prompt":"Which of the following bonds has the shortest bond length?\n\nW) H-F\n\nX) H-Cl\n\nY) H-Br\n\nZ) H-I","Completion":"W) H-F"} {"Type":"TOSS-UP\n\nPhysics - Multiple Choice","Prompt":"Which of the following does NOT describe the strong force?\n\nW) It is mediated by the W boson\n\nX) It is responsible for holding together the nucleus\n\nY) It is carried by gluons\n\nZ) It is described with quantum chromodynamics","Completion":"W) IT IS MEDIATED BY THE W BOSON"} {"Type":"TOSS-UP\n\nBiology \\(-\\) Multiple Choice","Prompt":"In regard to human anatomy, with which of the following is Wolff's law concerned?\n\nW) The shape of bone being determined by mechanical and gravitational stresses\n\nX) The negative feedback loops that control production of thyroid hormone\n\nY) The forces generated by isotonic contractions of skeletal muscle\n\nZ) The electrical potential of resting cardiac cells","Completion":"W) THE SHAPE OF BONE BEING DETERMINED BY MECHANICAL AND GRAVITATIONAL Stresses"} {"Type":"TOSS-UP\n\nMath \\(-\\) Short Answer","Prompt":"Giving your answer in standard form, what is the equation of the circle with center at (0, and diameter 10?","Completion":"\\(x^{2}+y^{2}=25\\)"} {"Type":"## BONUS\n\nEarth and Space - Multiple Choice","Prompt":"Which of the following coasts parallels a nearby tectonic plate boundary?\n\nW) East Coast of North America\n\nX) West Coast of Australia\n\nY) West Coast of South America\n\nZ) West Coast of Africa","Completion":"Y) WEST COAST OF SOUTH AMERICA"} {"Type":"TOSS-UP\n\nChemistry - Multiple Choice","Prompt":"Which of the following compounds does NOT have an adequate octet-rule-satisfying Lewis structure representation?\n\nW) Nitrogen dioxide\n\nX) Sulfur dioxide\n\nY) Carbon dioxide\n\nZ) Oxygen difluoride","Completion":"W) NITROGEN DIOXIDE"} {"Type":"BONUS\n\nChemistry - Multiple Choice","Prompt":"Which of the following types of hydrocarbons are saturated?\n\nW) Alkanes\n\nX) Alkenes\n\nY) Alkynes\n\nZ) Aromatic hydrocarbons","Completion":"W) ALKANES"} {"Type":"TOSS-UP\n\nEarth and Space - Multiple Choice","Prompt":"Which of the following gases is most likely to escape from a planet's atmosphere?\n\nW) Hydrogen\n\nX) Helium\n\nY) Methane\n\nZ) Ammonia","Completion":"W) HYDROGEN"} {"Type":"BONUS\n\nEarth and Space - Short Answer","Prompt":"What geologic era began following the Cretaceous-Paleogene [PAY-leo-jeen] extinction?","Completion":"CENOZOIC"} {"Type":"TOSS-UP\n\nEnergy - Short Answer","Prompt":"What initiative, created in 2000, encourages green building by promoting sustainability in construction and utility-efficient operations and maintenance?","Completion":"LEED (ACCEPT: LEADERSHIP IN ENERGY AND ENVIRONMENTAL DESIGN)"} {"Type":"BONUS\n\nEnergy - Short Answer","Prompt":"After about 3 years in a nuclear reactor producing electricity, 1\/3 of the fuel is removed and replaced. What is the term for fuel taken from the reactor?","Completion":"SPENT FUEL"} {"Type":"TOSS-UP\n\nMath - Multiple Choice","Prompt":"Given sets A and B with A intersect B = the set containing the element 5, which of the following must be true?\n\nW) Set A may be the empty set\n\nX) Set A contains at least one element\n\nY) Set B contains at most one element\n\nZ) Sets A and B have the same number of elements","Completion":"X) SET A CONTAINS AT LEAST ONE ELEMENT"} {"Type":"BONUS\n\nMath - Short Answer","Prompt":"Expressing your answer in terms of m, evaluate the function \\(f(x)=4x^{2}-9x+11\\) when \\(x\\) is \\(m-7\\).","Completion":"\\(4\\)m\\({}^{2}-65\\)m + 270"} {"Type":"TOSS-UP\n\nBiology - Short Answer","Prompt":"What term best describes a plant that has parallel leaf veins, fibrous roots, and flowers with parts in threes?","Completion":"MONOCOT (ACCEPT: MONOCOTYLEDON)"} {"Type":"TOSS-UP\n\nPhysics - Short Answer","Prompt":"What is the name for the change in the direction of propagation of light when it passes from one transparent material to another?","Completion":"REFRACTION"} {"Type":"TOSS-UP\n\nMath - Short Answer","Prompt":"What is the sum of the two vectors \\(7i+3j\\) and \\(2i-5j\\)?","Completion":"\\(9i-2j\\)"} {"Type":"TOSS-UP\n\nPhysics - Multiple Choice","Prompt":"Which of the following constants is fundamentally related to the speed of light?\n\nW) Gravitation constant\n\nX) Permeability of free space\n\nY) Planck constant\n\nZ) Boltzmann constant","Completion":"X) PERMEABILITY OF FREE SPACE"} {"Type":"TOSS-UP\n\nBiology - Short Answer","Prompt":"What plant hormone mediates phototropism?","Completion":"AUXIN"} {"Type":"BONUS\n\nBiology - Multiple Choice","Prompt":"Which of the following is NOT a location where bile is either stored or travels to after it is made?\n\nW) Gall bladder\n\nX) Cystic duct\n\nY) Pancreatic duct\n\nZ) Duodenum","Completion":"Y) PANCREATIC DUCT"} {"Type":"TOSS-UP\n\nEarth and Space - Short Answer","Prompt":"Identify all of the following three conditions that can cause metamorphism of rocks: High temperature; High acidity; High pressure.","Completion":"1, 3 (ACCEPT: HIGH TEMPERATURE, HIGH PRESSURE)"} {"Type":"BONUS\n\nEarth and Space - Multiple Choice","Prompt":"The United States has the highest frequency of tornado occurrences of the world's nations. To the nearest order of magnitude, how many tornadoes touch down in the United States each year?\n\nW) One hundred\n\nX) One thousand\n\nY) Ten thousand\n\nZ) One hundred thousand","Completion":"X) ONE THOUSAND"} {"Type":"TOSS-UP\n\nPhysics - Multiple Choice","Prompt":"Which of the following types of electromagnetic energy has the shortest wavelengths?\n\nW) X-rays\n\nX) AM radio\n\nY) Infrared\n\nZ) Visible","Completion":"W) X-RAYS"} {"Type":"TOSS-UP\n\nMath - Short Answer","Prompt":"What is the distance between the points (0, and (2, ?","Completion":"2\\(\\backslash\\)2"} {"Type":"TOSS-UP\n\nChemistry - Multiple Choice","Prompt":"Which of the following molecules can exist in boat and chair configurations?\n\nW) Benzene\n\nX) Cyclohexane [sye-kloh-HEX-ane]\n\nY) Cyclopentadiene [sye-kloh-penta-DYE-een]\n\nZ) Naphthalene [NAP-thah-leen]","Completion":"X) CYCLOHEXANE"} {"Type":"BONUS\n\nChemistry - Multiple Choice","Prompt":"Which of the following correctly explains why concentration cells have a standard cell potential of 0 volts?\n\nW) According to the Nernst equation, the standard cell potential is cancelled out by the concentration dependent term\n\nX) At standard conditions, all cells have a standard cell potential of 0 volts\n\nY) Standard cell potentials are measured with all species at equal concentrations\n\nZ) Concentration cells are used as a standard, so their standard cell potential is arbitrarily set to 0","Completion":"Y) STANDARD CELL POTENTIALS ARE MEASURED WITH ALL SPECIES AT EQUAL CONCENTRATIONS"} {"Type":"TOSS-UP\n\nEnergy - Multiple Choice","Prompt":"The price of which of the following fuels has the greatest impact on the cost of electricity?\n\nW) Nuclear\n\nX) Oil\n\nY) Coal\n\nZ) Natural Gas","Completion":"Z) NATURAL GAS"} {"Type":"BONUS\n\nEnergy - Short Answer","Prompt":"What is the name of the strategy that makes use of both electrical and thermal output from an electricity generator to greatly increase efficiency?","Completion":"COMBINED HEAT AND POWER (ACCEPT: COGENERATION)"} {"Type":"TOSS-UP\n\nEarth and Space - Multiple Choice","Prompt":"Which of the following is the mechanism by which strain energy is released during earthquakes?\n\nW) Elastic rebound\n\nX) Solution-collapse\n\nY) Hydrostatic pressure\n\nZ) Volcanic degassing","Completion":"W) ELASTIC REBOUND"} {"Type":"TOSS-UP\n\nBiology - Short Answer","Prompt":"Ced-3 and ced-4 are genes likely involved in what cell process?","Completion":"APOPTOSIS (ACCEPT: PROGRAMMED CELL DEATH)"} {"Type":"TOSS-UP\n\nEarth and Space - Multiple Choice","Prompt":"Which of the following minerals is a crystalline form of silicon dioxide?\n\nW) Mica\n\nX) Quartz\n\nY) Calcite\n\nZ) Feldspar","Completion":"X) QUARTZ"} {"Type":"TOSS-UP\n\nPhysics - Multiple Choice","Prompt":"Which of the following leptons has the largest mass?\n\nW) Electron neutrino\n\nX) Muon\n\nY) Muon neutrino\n\nZ) Tauon","Completion":"Z) TAUON"} {"Type":"TOSS-UP\n\nChemistry - Multiple Choice","Prompt":"Which of the following is an example of a molecular solid?\n\nW) Silicon\n\nX) Iodine\n\nY) Diamond\n\nZ) Graphite","Completion":"X) IODINE"} {"Type":"TOSS-UP\n\nBiology \\(-\\) Multiple Choice","Prompt":"Which of the following receptors could NOT be found on the surface of a cell?\n\nW) Laminar receptor\n\nX) Ion-channel coupled receptor\n\nY) Tyrosine [TIE-row-seen] kinase receptor\n\nZ) G-protein coupled receptor","Completion":"W) Laminar Receptor"} {"Type":"TOSS-UP\n\nEarth and Space - Multiple Choice","Prompt":"Which of the following minerals has only one plane of cleavage?\n\nW) Hornblende\n\nX) Biotite\n\nY) Potassium feldspar\n\nZ) Augite [4W-jite]","Completion":"X) BIOTITE"} {"Type":"BONUS\n\nEarth and Space - Short Answer","Prompt":"Identify all of the following three statements that are true of the contrails produced by air traffic: They are mostly composed of water ice; They have a high albedo relative to oceans and forests; They form in unstable, convecting layers of the atmosphere.","Completion":"1 AND 2"} {"Type":"TOSS-UP\n\nEnergy - Multiple Choice","Prompt":"Which of the following is NOT considered a practical source of biodiesel?\n\nW) Animal fat\n\nX) Restaurant oils\n\nY) Vegetable oils\n\nZ) Essential oils","Completion":"Z) ESSENTIAL OILS"} {"Type":"BONUS\n\nEnergy - Multiple Choice","Prompt":"Which of the following countries had the largest wind energy capacity in 2014?\n\nW) India\n\nX) United States\n\nY) China\n\nZ) Germany","Completion":"Y) CHINA3) Physics - Short AnswerA pilot is in a recoverable spin with her engine turned off, and with the plane's nose directed toward the Earth's center. What term describes the limit of her rate of descent?"} {"Type":"TOSS-UP\n\nMath - Short Answer","Prompt":"What is the integral from -6 to 6 of 6 dx?","Completion":"72"} {"Type":"TOSS-UP\n\nChemistry - Multiple Choice","Prompt":"An aqueous solution of sodium sulfate is electrolyzed using platinum electrodes. If phenolphthalein [free-nawf-THAY-leen] indicator is added to the solution, which of the following will be observed?\n\nW) The solution is colorless everywhere except at the cathode, where it is pink\n\nX) The solution is colorless everywhere except at the anode, where it is pink\n\nY) The solution is pink everywhere except at the cathode, where it is colorless\n\nZ) The solution is pink everywhere except at the anode, where it is colorless","Completion":"W) THE SOLUTION IS COLORLESS EVERYWHERE EXCEPT THE CATHODE, WHERE IT IS PINK"} {"Type":"TOSS-UP\n\nBiology - Short Answer","Prompt":"Trans fats are produced from unsaturated fats in what chemical\n\nprocess?","Completion":"HYDROGENATION (ACCEPT: ISOMERIZATION)"} {"Type":"TOSS-UP\n\nChemistry - Short Answer","Prompt":"Identify all of the following three compounds that have only London dispersion forces operating between the molecules: Diffuoromethane [dye-floor-oh-MEH-thane]; Hydrogen chloride; Tetrachloromethane [tetrah-kloor-oh-MEH-thane].","Completion":"3 (ACCEPT: TETRACHLOROMETHANE)"} {"Type":"TOSS-UP\n\nPhysics - Multiple Choice","Prompt":"Which of the following is NOT responsible for the failure of a pendulum to reach the height from which it was released as time progresses?\n\nW) Friction\n\nX) Gravity\n\nY) Air resistance\n\nZ) Damping","Completion":"X) GRAVITY"} {"Type":"TOSS-UP\n\nEarth and Space - Multiple Choice","Prompt":"Which of the following is a fine-grained, typically buffer-colored sediment deposited by wind?\n\nW) Evaporite\n\nX) Loess [LOH-ess]\n\nY) Glaze\n\nZ) Humus [HYOO-muss]","Completion":"X) LOESS"} {"Type":"TOSS-UP\n\nMath - Short Answer","Prompt":"How many terms does the binomial expansion of open parenthesis \\(x^{2}+3y^{3}\\) close parenthesis to \\(13^{\\rm th}\\) power contain?","Completion":"14"} {"Type":"TOSS-UP\n\nEnergy - Short Answer","Prompt":"Commercial nuclear power reactors in the U.S. include a material that reduces the speed of neutrons to permit fission to occur in the fuel more readily. What is the general term for this material?","Completion":"MODERATOR"} {"Type":"BONUS\n\nEnergy - Short Answer","Prompt":"Energy can be stored in batteries by converting electrical energy into chemical energy. In what device is electrical energy stored as separations of large amounts of static charge?","Completion":"SUPERCAPACITOR (ACCEPT: CAPACITOR OR ULTRACAPACITOR)"} {"Type":"TOSS-UP\n\nChemistry \\(-\\) Multiple Choice","Prompt":"Which of the following molecules is most polar?\n\nW) Ethane\n\nX) Carbon tetrachloride\n\nY) Hexane\n\nZ) Pyridine","Completion":"Z) PYRIDINE"} {"Type":"TOSS-UP\n\nPhysics \\(-\\) Short Answer","Prompt":"The weber is the SI unit for what quantity?","Completion":"MAGNETIC FLUX (DO NOT ACCEPT: FLUX)"} {"Type":"TOSS-UP\n\nMath - Short Answer","Prompt":"If the graph of \\(y\\) equals open parenthesis \\(x\\) minus two close parenthesis squared minus three is translated 6 units up and 3 units to the right, what are the coordinates of the vertex of the image?","Completion":"(5, 3)"} {"Type":"BONUS\n\nMath - Short Answer","Prompt":"A bag contains 18 socks: 4 are red, 6 are white, and 8 are blue. How many socks must you take out to be certain that you will have at least one pair of blue socks?","Completion":"12"} {"Type":"TOSS-UP\n\nBiology - Multiple Choice","Prompt":"The suprachiasmatic nucleus is involved in controlling which of the following?\n\nW) Circadian rhythms\n\nX) Body temperature\n\nY) Heart rate\n\nZ) Respiration rate","Completion":"W) CIRCADIAN RHYTHMS"} {"Type":"BONUS\n\nBiology - Multiple Choice","Prompt":"Which of the following disorders is due to a dominant trait?\n\nW) Cystic fibrosis\n\nX) Hemophilia [HEE-moh-FEEL-ee-ah]\n\nY) Albinism [AL-bah-nih-zim]\n\nZ) Polydactyly [polly-DAK-til-ee]","Completion":"Z) POLYDACTYLY"} {"Type":"TOSS-UP\n\nEnergy - Multiple Choice","Prompt":"Enhanced oil recovery allows residual oil to be produced. Which of the following substances allows more oil to be produced when it is injected into a well to lower the oil viscosity?\n\nW) A thinner form of oil\n\nX) Solvents\n\nY) Steam\n\nZ) Saltwater","Completion":"Y) STEAM"} {"Type":"BONUS\n\nEnergy - Multiple Choice","Prompt":"Recently, two wind turbines were installed inside the Eiffel Tower. The energy produced will be enough to power which of the following?\n\nW) The tower's first floor\n\nX) The entire tower, except for lighting\n\nY) The lighting for the entire tower\n\nZ) The entire tower including the lighting","Completion":"W) THE TOWER'S FIRST FLOOR"} {"Type":"TOSS-UP\n\nEarth and Space - Short Answer","Prompt":"What country has emitted the most carbon dioxide to Earth's atmosphere during the past five years?","Completion":"CHINA"} {"Type":"BONUS\n\nEarth and Space - Multiple Choice","Prompt":"Which of the following is NOT characteristic of currents\n\nfound on the western boundary of oceanic gyres, relative to other ocean currents?\n\nW) Warm\n\nX) Deep\n\nY) Narrow\n\nZ) Slow-moving","Completion":"Z) SLOW-MOVING"} {"Type":"TOSS-UP\n\nChemistry \\(-\\) Multiple Choice","Prompt":"The oxidation of elemental lithium with which of the following is a redox reaction that is MOST favorable?\n\nW) Fluorine gas\n\nX) Hydrogen gas\n\nY) Iron (II)\n\nZ) Copper (I)","Completion":"W) FLUORINE GAS"} {"Type":"TOSS-UP\n\nEarth and Space \\(-\\) Multiple Choice","Prompt":"Which of the following laws expresses the increase in\n\nredshift of a galaxy's spectral lines with distance?\n\nW) Doppler's Law\n\nX) Cepheid's Law\n\nY) Hubble's Law\n\nZ) Kepler's Law","Completion":"Y) HUBBLE'S LAW"} {"Type":"TOSS-UP\n\nMath - Short Answer","Prompt":"What is the slope of the line with equation \\(5x+3y=8\\)?","Completion":"-5\/3 (ACCEPT: -1 2\/3)"} {"Type":"BONUS\n\nMath - Short Answer","Prompt":"The three sides of a triangle have lengths 4 centimeters, 6 centimeters, and 7 centimeters. What is the cosine of the smallest angle?","Completion":"23\/28"} {"Type":"TOSS-UP\n\nPhysics - Short Answer","Prompt":"A standard spring mass oscillator on a frictionless surface oscillates at amplitude 2 meters. At what displacement from equilibrium does the spring store zero potential energy?","Completion":"0"} {"Type":"BONUS\n\nPhysics - Short Answer","Prompt":"Identify all of the following four particles that obey Bose-Einstein statistics: Super-cooled helium atoms; Beta particles; Gamma rays; Neutrons.","Completion":"1 AND 3"} {"Type":"TOSS-UP\n\nEarth and Space - Short Answer","Prompt":"Two stars have the same absolute magnitude. One is at a distance of 10 parsecs and the other is at a distance of 100 parsecs. What is the positive difference in their apparent magnitudes?","Completion":"5"} {"Type":"TOSS-UP\n\nBiology - Multiple Choice","Prompt":"Steroids can be classified as which of the following types of biomacromolecule?\n\nW) Carbohydrate\n\nX) Protein\n\nY) Lipid\n\nZ) Nucleic Acid","Completion":"Y) LIPID"} {"Type":"TOSS-UP\n\nEnergy - Multiple Choice","Prompt":"What gas is produced by bacterial decay in landfills?\n\nW) Neon\n\nX) Oxygen\n\nY) Carbon monoxide\n\nZ) Methane","Completion":"Z) METHANE"} {"Type":"TOSS-UP\n\nEarth and Space - Multiple Choice","Prompt":"What type of lava can be expected to erupt from a shield volcano, like Kilauea?\n\nW) Andesite\n\nX) Basalt\n\nY) Dacite\n\nZ) Rhyolite","Completion":"X) BASALT"} {"Type":"TOSS-UP\n\nPhysics - Multiple Choice","Prompt":"Which of the following is equivalent to 10 centimeters?\n\nW) 10\\({}^{8}\\) nanometers\n\nX) 10\\({}^{3}\\) millimeters\n\nY) 10\\({}^{7}\\) micrometers\n\nZ) 10\\({}^{2}\\) decimeters","Completion":"W) 10\\({}^{8}\\) NANOMETERS"} {"Type":"TOSS-UP\n\nBiology - Multiple Choice","Prompt":"Which of the following best describes why a complete lack of cholesterol is dangerous for cells?\n\nW) Cholesterol maintains membrane fluidity\n\nX) Cholesterol is necessary for all signal transduction\n\nY) Cholesterol is required for ATP production\n\nZ) Cholesterol protects against DNA degradation","Completion":"W) CHOLESTEROL MAINTAINS MEMBRANE FLUIDITY"} {"Type":"TOSS-UP\n\nChemistry - Short Answer","Prompt":"What type of covalent bond, found in amine [ah-MEEN] oxides, is formed by two electrons from the same atom?","Completion":"DATIVE (ACCEPT: DIPOLAR OR COORDINATE)"} {"Type":"TOSS-UP\n\nPhysics - Short Answer","Prompt":"What is the adjective for a process that occurs at constant pressure?","Completion":"ISOBARIC"} {"Type":"TOSS-UP\n\nMath - Multiple Choice","Prompt":"Which of the following numbers has a remainder of 1 when it is divided by 7?\n\nW) 62\n\nX) 82\n\nY) 99\n\nZ) 103","Completion":"Y) 99"} {"Type":"BONUS\n\nEarth and Space - Short Answer","Prompt":"Which of the seven spectral classes includes our Sun?","Completion":"G"} {"Type":"TOSS-UP\n\nEnergy - Short Answer","Prompt":"Scientists at Oak Ridge National Lab are using the Titan supercomputer in order to study actinide [AK-tin-ide] chemistry at a molecular level. Identify all of the following three elements that are being studied:\n\nUranium; Thorium [THOR-ee-un]; Neptunium [hep-TOO-nee-un].","Completion":"1, 2, 3 (ACCEPT: ALL OF THEM)"} {"Type":"BONUS\n\nEnergy - Short Answer","Prompt":"DOE researchers, as part of the Daya [DIE-ah] Bay Collaboration, use sophisticated experiments to observe the reactions going on in nuclear reactors. Other than a proton and an electron, what particle that is produced by beta minus decay are they observing?","Completion":"ANTINEUTRINO"} {"Type":"TOSS-UP\n\nEarth and Space - Multiple Choice","Prompt":"Walden Pond in Massachusetts is a famous example of a kettle lake. What created Walden pond?\n\nW) Earthquake\n\nX) Glacial activity\n\nY) A meander cut off to form a lake\n\nZ) Volcanic activity","Completion":"X) GLACIAL ACTIVITY"} {"Type":"BONUS\n\nEarth and Space - Multiple Choice","Prompt":"During what eon did visible life start to appear and resulted in abundant fossils?\n\nW) Hadean\n\nX) Archaean\n\nY) Proterozoic\n\nZ) Phanerozoic","Completion":"Z) PHANEROZOIC15) Math - Multiple ChoiceWhich of the following best describes the relationship between the lines with equations \\(\\frac{5x+7y=35}{\\rm\\ and\\ \\ }\\frac{28x-20y=35}{\\rm?}\\)"} {"Type":"## BONUS\n\nMath - Short Answer","Prompt":"A regular hexagon is inscribed in a circle of diameter 16. What is the area of the hexagon?","Completion":"96\\(\\backslash\\)3"} {"Type":"TOSS-UP\n\nPhysics - Short Answer","Prompt":"The liquefaction of what gas was recognized in the 1913 Nobel Prize?","Completion":"HELIUM"} {"Type":"## BONUS\n\nPhysics - Short Answer","Prompt":"A 2-liter container is filled with 3 moles of an ideal gas at 150 kelvins. What is the pressure of the gas, in atmospheres, to two significant figures?","Completion":"1817) Chemistry - Multiple Choice Consider the covalent bonding in the molecule acetylene [ah-SET-ihl-een], and assume that the carbon-carbon axis is the x-axis. What two orbitals on each carbon atom participate in pi bonding?"} {"Type":"## BONUS\n\nChemistry - Short Answer","Prompt":"Rank the following three barium salts in terms of increasing water solubility:\n\nBarium chlorate; Barium sulfate; Barium hydroxide.","Completion":"2, 3, 1"} {"Type":"## BONUS\n\nBiology - Multiple Choice Which of the following muscles adducts and rotates the scapula and tilts the chin?\n\nW) Latissimus [hal-TIH-sih-mus] dorsi\n\nX) Rhombicaeus [rom-BOY-dee-us] major\n\nY) Subscapularis [sub-scap-yoo-LAIR-us]\n\nZ) Trapezius [trah-PEE-zee-us]Physics - Short Answer","Prompt":"Identifify all of the following 3 forms of heat transfer for which the rate of heat transfer is independent of the temperature gradient: Conduction; Convection; Radiation.","Completion":"NONE"} {"Type":"TOSS-UP\n\nBiology - Short Answer","Prompt":"Dental cavities in humans are primarily caused by what genus [JEE-nus] of bacteria?","Completion":"STREPTOCOCCUS"} {"Type":"TOSS-UP\n\nEarth and Space - Short Answer","Prompt":"After a long period of deflation by the wind, a desert will be reduced to a layer of cobble that is resistant to further deflation. What is the term for this layer of cobble?","Completion":"DESERT PAVEMENT"} {"Type":"TOSS-UP\n\nChemistry - Short Answer","Prompt":"Identify all of the following three elements that, when inserted into a silicon crystal lattice, would result in a p-type, doped semiconductor: Carbon; Boron; Nitrogen.","Completion":"2 (ACCEPT: BORON)"} {"Type":"TOSS-UP\n\nMath - Short Answer","Prompt":"The mean of a set of 5 numbers is 5. Four of the numbers are 1, 2, 5, and 8. What is the fifth number?","Completion":"9"} {"Type":"TOSS-UP\n\nChemistry - Multiple Choice","Prompt":"Trans fats necessarily contain what functional group?\n\nW) Ester\n\nX) Alkene\n\nY) Alcohol\n\nZ) Amine [ah-MEEN]","Completion":"X) ALKENE"} {"Type":"BONUS\n\nChemistry - Multiple Choice","Prompt":"Tollens' test can detect aldehyde [AL-deh-hide]-containing compounds. To perform Tollens' test, an organic compound is mixed with a solution of silver ions. Which of the following indicates that the compound of interest contains an aldehyde?\n\nW) A silver mirror is formed\n\nX) Insoluble silver oxide precipitates from solution\n\nY) The reaction mixture turns yellow\n\nZ) The aldehyde is reduced to an alcohol","Completion":"W) A SILVER MIRRO IS FORMED"} {"Type":"TOSS-UP\n\nBiology - Short Answer","Prompt":"What type of leukocyte [LOO-koh-site] is involved in phagocytosis [flag-oh-sye-TOW-sis] of antigen-antibody complexes and parasites?","Completion":"EOSINOPHIL"} {"Type":"BONUS\n\nBiology - Short Answer","Prompt":"Angiotensin [an-jee-oh-TEN-sin] acts on the adrenal [Ah-DREE-null] cortex to promote secretion of what hormone?","Completion":"ALDOSTERONE"} {"Type":"TOSS-UP\n\nEarth and Space - Short Answer","Prompt":"What is the name for the air-lifting process that forces an air mass up and over a mountain or upland, causing the moisture to rain out on the windward slope and leaving a rainshadow effect on the leeward slope?","Completion":"OROGRAPHIC LIFTING"} {"Type":"TOSS-UP\n\nPhysics - Short Answer","Prompt":"For a particle in an infinite square potential well, how many nodes exist in the wavefunction within the well, not including the walls, in the third excited state?","Completion":"3"} {"Type":"TOSS-UP\n\nMath - Short Answer","Prompt":"Find the value of \\(x\\) for which the following vectors are parallel:\n\n\\(\\textbf{4i}-\\textbf{3j}\\) and \\(\\textbf{x}i+12j\\).","Completion":"-16"} {"Type":"TOSS-UP\n\nEnergy - Short Answer","Prompt":"What US state is responsible for generating the most total net electricity?","Completion":"TEXAS"} {"Type":"TOSS-UP\n\nChemistry - Multiple Choice","Prompt":"Consider ice at 0 degrees Celsius that is heated until it becomes water at 80 degrees Celsius. Which of the following statements is TRUE concerning its volume?\n\nW) It monotonically [mono-TAWN-ik-lee] decreases\n\nX) It decreases until 4 degrees Celsius, then increases\n\nY) It increases until 4 degrees Celsius, then remains constant\n\nZ) It remains constant","Completion":"X) IT DECREASES UNTIL 4 DEGREES CELSIUS, THEN INCREASES"} {"Type":"","Prompt":"TOSS-UP","Completion":"8) Math - Short Answer If \\(f\\) of \\(x\\) equals the cube root of open parenthesis \\(x-6\\) close parenthesis, what is \\(f\\)inverse of 5? 131"} {"Type":"TOSS-UP\n\nEarth and Space - Short Answer","Prompt":"The leading edge of the bowl of the Big Dipper, from bottom to top, points to what important aid to celestial navigation?","Completion":"POLARIS (ACCEPT: NORTH STAR)"} {"Type":"TOSS-UP\n\nPhysics - Short Answer","Prompt":"Identify all of the following three statements that could be true of a particle undergoing constant acceleration: Zero instantaneous velocity; Quadratic increase in velocity; Quadratic increase in position.","Completion":"1 AND 3"} {"Type":"TOSS-UP\n\nEnergy - Short Answer","Prompt":"What is the name of the process by which biomass is thermally decomposed in the absence of oxygen?","Completion":"PYROLYSIS"} {"Type":"TOSS-UP\n\nBiology - Multiple Choice","Prompt":"Which of the following organisms is characterized as a parazzoan [para-ZOH-an]?\n\nW) Sponge\n\nX) Sea cucumber\n\nY) Jellyfish\n\nZ) Tapeworm","Completion":"W) SPONGE"} {"Type":"TOSS-UP\n\nMath - Short Answer","Prompt":"What is the partial derivative with respect to \\(y\\) of the expression \\(x^{5}y^{4}\\)?","Completion":"\\(4x^{5}y^{3}\\)"} {"Type":"TOSS-UP\n\nPhysics - Multiple Choice","Prompt":"Which of the following laws is NOT always true in electrical circuits?\n\nW) Ohm's Law\n\nX) Coulomb's Law\n\nY) Kirchoff's 1st Law\n\nZ) Faraday's Law of Induction","Completion":"W) OHM'S LAW"} {"Type":"## BONUS\n\nChemistry - Short Answer","Prompt":"Identify all of the following three diatomic [dye-ah-TOM-ik] compounds that MO theory predicts to be paramagnetic: B\\({}{2}\\); C\\({}{2}\\); N\\({}{2}\\).","Completion":"1 (ACCEPT: B\\({}{2}\\))"} {"Type":"## BONUS\n\nEarth and Space - Short Answer","Prompt":"What is the name for the circular ocean currents that pinch off from ocean currents in sections, causing drastic changes in sound velocity at the interface?","Completion":"EDDIES"} {"Type":"TOSS-UP\n\nBiology - Multiple Choice","Prompt":"Zinc finger motifs are commonly found in proteins that do which of the following?\n\nW) Hydrolyze [HIGH-droh-lyze] sugars\n\nX) Sequester calcium\n\nY) catalyze polymerization [pawl-ih-mer-eh-ZAY-shun]\n\nZ) Bind to DNA","Completion":"Z) BIND TO DNA"} {"Type":"TOSS-UP\n\nEnergy - Short Answer","Prompt":"DOE scientists at SLAC have previously shown that electrons can be accelerated by riding a wave of plasma. Recently, they showed that what similar particles, products of beta [BAY-tah] plus decay, can also be accelerated in the same manner?","Completion":"POSITRONS (ACCEPT: ANTI-ELECTRON)"} {"Type":"TOSS-UP\n\nEarth and Space - Short Answer","Prompt":"In what constellation can the Crab Nebula be found?","Completion":"TAURUS"} {"Type":"BONUS\n\nEarth and Space - Short Answer","Prompt":"What is the name for a front that is formed when a cold front overtakes a warm front and warm air is separated from the cyclonic center?","Completion":"OCCLUDED FRONT"} {"Type":"TOSS-UP\n\nPhysics - Short Answer","Prompt":"A car brakes to a stop over 15 meters, applying a constant force of 800 newtons to the car. In joules, what was the initial kinetic energy of the car?","Completion":"12,000"} {"Type":"BONUS\n\nPhysics - Short Answer","Prompt":"A block attached to a spring is experiencing simple harmonic motion with a period of 4.0 seconds. What would the period be, in seconds to two significant figures, if the mass of the block was halved?","Completion":"2.8"} {"Type":"TOSS-UP\n\nChemistry - Short Answer","Prompt":"Rank the pH of the following three solutions, from lowest to highest: One molar solution of hydrochloric acid; One molar solution of acetic acid; One molar solution of ammonium perchlorate.","Completion":"1, 2, 3"} {"Type":"BONUS\n\nChemistry - Short Answer","Prompt":"Identify all of the following three molecules that are likely to act as electrophiles in a substitution reaction: Ammonia; Bromide; Methyl bromide.","Completion":"3"} {"Type":"TOSS-UP\n\nMath - Short Answer","Prompt":"In base 7, what is 4 + 6?","Completion":"ONE-THREE"} {"Type":"BONUS\n\nMath - Short Answer","Prompt":"Evaluate the integral from \\(x=\\)-8 to 0 of the square root of open parenthesis 64 - \\(x^{2}\\) close parenthesis d\\(x\\).","Completion":"16\\(\\pi\\)"} {"Type":"TOSS-UP\n\nBiology - Short Answer","Prompt":"What is the net ATP gain during glycolysis [glove-K4WL-eh-sis], per molecule of glucose?","Completion":"TWO"} {"Type":"BONUS\n\nBiology - Short Answer","Prompt":"In a young stem, what is the name for the immature tissues that will specifically develop into xylem and phloem?","Completion":"PROCAMBIUM"} {"Type":"TOSS-UP\n\nEarth and Space - Multiple Choice","Prompt":"The Black Hills of South Dakota is a large dome that has been exposed due to upwarping followed by erosion. Which of the following rocks would we expect to find in the innermost core of the Black Hills?\n\nW) Shale\n\nX) Gypsum\n\nY) Pegmatite\n\nZ) Limestone","Completion":"Y) PEGMATITE"} {"Type":"TOSS-UP\n\nBiology - Multiple Choice","Prompt":"Which of the following correctly explains why fish cannot breathe outside of water?\n\nW) Air contains less oxygen than water\n\nX) Their gas exchange systems are tuned to dissolved oxygen\n\nY) In air, their gills collapse\n\nZ) Air provides more structural support to the fish","Completion":"Y) IN AIR, THEIR GILLS COLLAPSE"} {"Type":"TOSS-UP\n\nChemistry - Multiple Choice","Prompt":"Which of the following is the most plausible first step in the solvolysis [sawl-VAWL-eh-sis] of tert-butyl [turt-BYU-til] iodide in water?\n\nW) Concerted substitution\n\nX) Attack of water to form a pentavalent [penta-VAY-lent] intermediate\n\nY) Decomposition to form positively-charged intermediate\n\nZ) Decomposition to form negatively-charged intermediate","Completion":"Y) DECOMPOSITION TO FORM POSITIVELY-CHARGED INTERMEDIATE"} {"Type":"BONUS\n\nChemistry - Short Answer","Prompt":"Rank the following three acids in terms of increasing pKa: Chlorous [KLOR-us] acid; Hydroiodic [HIGH-droh-eye-AW-dikj] acid; Acetic [ah-SEE-tik] acid.","Completion":"2, 1, 3"} {"Type":"TOSS-UP\n\nMath - Short Answer","Prompt":"What is the area of a kite with diagonals of lengths 5 and 8?","Completion":"20"} {"Type":"BONUS\n\nMath - Short Answer","Prompt":"If \\(a\\) sub one = 3 and, for \\(n\\) greater than one, \\(a\\) sub \\(n\\) = 2 \\(a\\) sub n-1 minus 1, what is the value of \\(a\\) sub five?","Completion":"33"} {"Type":"TOSS-UP\n\nBiology - Multiple Choice","Prompt":"Which of the following is an example of a Chrysophytan [Cris-AW-fih-tan]?\n\nW) Amoeba [ah-MEE-bah]\n\nX) Trypanoma [trih-pan-OH-ma]\n\nY) Plasmodium [plazz-MOH-dee-tun]\n\nZ) Diatom [DYE-ah-ton]","Completion":"Z) DIATOM"} {"Type":"BONUS\n\nBiology - Short Answer","Prompt":"Given that hemoglobin is 144 amino acids long, identify all of the following three numbers of nucleotides that can be in the primary mRNA transcript of hemoglobin at different stages of processing:\n\n144; 432; 1356.","Completion":"JUST 3 (ACCEPT: 1356)"} {"Type":"TOSS-UP\n\nMath - Multiple Choice","Prompt":"The graph of a continuous function \\(f\\) contains the points (4, -and (7, -. The intermediate value theorem guarantees an x-value such that \\(f\\) of \\(x\\) equals which of the following?\n\nW) -6 X) -2 Y) 2 Z) 6","Completion":"X) -2"} {"Type":"BONUS\n\nMath - Short Answer","Prompt":"How many positive integral factors does 936 have?","Completion":"24"} {"Type":"TOSS-UP\n\nChemistry - Multiple Choice","Prompt":"Which of the following compounds has the lowest boiling point?\n\nW) Water\n\nX) Hydrogen sulfide\n\nY) Hydrogen selenide [SELL-en-ide]\n\nZ) Hydrogen telluride [TELL-ur-ide]","Completion":"X) HYDROGEN Sulfide"} {"Type":"TOSS-UP\n\nEarth and Space - Multiple Choice","Prompt":"The Global Positioning System consists of several operational satellites in spatially separated orbits around the earth. How many different satellite signals are required for accurate position, velocity, and time data?\n\nW) 2\n\nX) 3\n\nY) 4\n\nZ) 6","Completion":"Y) 4"} {"Type":"TOSS-UP\n\nEnergy - Short Answer","Prompt":"Renewable diesel is produced via multiple methods. Biodiesel, however, is only produced via what chemical process?","Completion":"TRANSESETRIFICATION (ACCEPT: METHANOLYSIS)"} {"Type":"TOSS-UP\n\nMath - Short Answer","Prompt":"What is the dot product of the vectors \\(4i-2j+k\\) and \\(3i-k\\)?","Completion":"11"} {"Type":"TOSS-UP\n\nPhysics - Short Answer","Prompt":"What dimensionless physical constant is equal to the square of the ratio of the charge of the electron to the Planck charge?","Completion":"FINE STRUCTURE CONSTANT"} {"Type":"TOSS-UP\n\nEnergy - Short Answer","Prompt":"DOE scientists at Brookhaven National Lab are using the Relativistic Heavy Ion Collider to study color confinement within protons. What group of elementary particles are they studying?","Completion":"QUARKS (ACCEPT: GLUONS AND QUARKS)"} {"Type":"TOSS-UP\n\nPhysics - Multiple Choice","Prompt":"Which of the following circuits with two components will behave as an electrical resonator?\n\nW) resistor and capacitor\n\nX) resistor and inductor\n\nY) capacitor and inductor\n\nZ) capacitor and capacitor","Completion":"Y) CAPACITOR AND INDUCTOR"} {"Type":"TOSS-UP\n\nChemistry - Short Answer","Prompt":"Identify all of the following three compounds that are a stronger base than sodium hydride: T-butyl [t-byu-til] lithium; Ammonia; Sodium tert-butoxide [tert-byu-tax-eyed].","Completion":"JUST 1"} {"Type":"TOSS-UP\n\nBiology - Short Answer","Prompt":"In the spinal cord, cell bodies of sensory neurons are grouped together to form what structures?","Completion":"DORSAL ROOT GANGLIA (ACCEPT: DORSAL ROOTS)"} {"Type":"TOSS-UP\n\nMath - Short Answer","Prompt":"If the difference between two prime numbers is 27, what is their sum?","Completion":"31"} {"Type":"TOSS-UP\n\nEarth and Space - Multiple Choice","Prompt":"At which of the following locations on the HR diagram would one find Betelgeuse?\n\nW) Top right\n\nX) Top left\n\nY) Bottom right\n\nZ) Bottom left","Completion":"W) TOP RIGHT"} {"Type":"TOSS-UP\n\nPhysics - Short Answer","Prompt":"A car is traveling around a circular track in uniform circular motion. Identify all of the following three quantities that are constant for the car: Angular velocity; Acceleration; Radial velocity.","Completion":"1"} {"Type":"TOSS-UP\n\nEnergy - Short Answer","Prompt":"What U.S. university was the site of the first ever sustained nuclear reaction?","Completion":"UNIVERSITY OF CHICAGO (ACCEPT: CHICAGO)"} {"Type":"BONUS\n\nEnergy - Short Answer","Prompt":"Recent experiments regarding antimatter at the Relativistic Heavy Ion Collider determined that what fundamental force demonstrates no asymmetry within nuclei [NEW-klee-eye]?","Completion":"STRONG NUCLEAR FORCE (ACCEPT: STRONG, STRONG NUCLEAR)"} {"Type":"TOSS-UP\n\nMath - Multiple Choice","Prompt":"Consider the polynomial equation \\(6x^{3}-20x+15=0\\). According to the rational root theorem, which of the following is NOT a possible rational root of this equation?\n\nW) -5\n\nX) 1\/3\n\nY) 1\/2\n\nZ) 6\/5","Completion":"Z) 6\/5"} {"Type":"BONUS\n\nMath - Short Answer","Prompt":"If the sides of a triangle have lengths 5, 7, and 8, what is the cosine of the largest angle?","Completion":"1\/7"} {"Type":"TOSS-UP\n\nBiology - Multiple Choice","Prompt":"What aspect of a reaction does an enzyme change?\n\nW) Free energy\n\nX) Entropy [EN-trouh-pee]\n\nY) Enthalpy [EN-thul-pee]\n\nZ) Activation energy","Completion":"Z) ACTIVATION ENERGY"} {"Type":"TOSS-UP\n\nChemistry - Multiple Choice","Prompt":"Which of the following statements concerning crystal structure is NOT true?\n\nW) HCP lattices are more space-filling than FCC lattices\n\nX) Cesium [SEEZ-ee-um] chloride lattices form when anions [AN-eye-ons] and cations [CAT-eye-onz] are of roughly equal size\n\nY) Rock-salt lattices have (6,-coordination\n\nZ) The crystal structure of a metal is dependent on pressure and temperature","Completion":"W) HCP LATTICES ARE MORE SPACE-FILLING THAN FCC LATTICES"} {"Type":"TOSS-UP\n\nEnergy - Short Answer","Prompt":"Identify all of the following three options that would increase the resistance of a wire:\n\nIncreasing the temperature; Increasing the radius; Increasing the length.","Completion":"1 AND 3"} {"Type":"TOSS-UP\n\nMath - Short Answer","Prompt":"If angle ABC is inscribed in a circle and measures 15 degrees, what is the degree measure of arc ABC?","Completion":"330"} {"Type":"TOSS-UP\n\nEnergy - Multiple Choice","Prompt":"What is the typical co-electrolyte in Vanadium [vah-NAY-dee-tun] redox [REE-day] flow batteries?\n\nW) Zinc nitrate\n\nX) Sulfuric acid\n\nY) Sodium hydroxide\n\nZ) Copper chloride","Completion":"X) SULFURIC ACID"} {"Type":"BONUS\n\nEnergy - Short Answer","Prompt":"Biodiesel produces less particulates, carbon oxides, sulfur dioxide, and unburnt carbons than fossil-based diesel. However, biodiesel does result in greater emissions of what ozone-generating pollutant?","Completion":"NITROUS OXIDES (ACCEPT: NOX)"} {"Type":"TOSS-UP\n\nBiology - Multiple Choice","Prompt":"Which of the following statements about monocots and dicots is NOT correct?\n\nW) Monocots have three holes in their pollen, while dicots only have one\n\nX) Monocots have scattered vascular bundles, while dicot bundles are in a ring\n\nY) Monocots lack a path region in the stem, while dicots have a pit\n\nZ) Monocots have parallel venation in their leaves while dicots have netted veins","Completion":"W) MONOCOTS HAVE THREE HOLES IN THEIR POLLEN, WHILE DICOTS ONLY HAVE ONE"} {"Type":"BONUS\n\nBiology - Multiple Choice","Prompt":"Which of the following pairs of terms referring to the life cycle of land plants is mismatched with the correct ploidy?\n\nW) Megasporanguium [mega-spor-AYN-gee-um] and diploid\n\nX) Sporocyte and diploid\n\nY) Egg and haploid\n\nZ) Integument and haploid","Completion":"Z) INTEGUMENT AND HAPLOID"} {"Type":"TOSS-UP\n\nMath - Short Answer","Prompt":"To the nearest whole number, what is the slant height of a right circular cone with radius 6 and height 9?","Completion":"11"} {"Type":"BONUS\n\nMath - Short Answer","Prompt":"How many zeros occur at the right of the number of permutations of 100 objects taken 80 at a time?","Completion":"5"} {"Type":"TOSS-UP\n\nBiology - Short Answer","Prompt":"What virus causes mononucleosis [mono-new-klee-OH-sis] in humans?","Completion":"EPSTEIN-BARR VIRUS"} {"Type":"BONUS\n\nBiology - Short Answer","Prompt":"In a developing root, there are three regions: the region of cell division, the region of cell elongation, and the region of maturation. What external feature of the epidermis allows one to know they have located the region of maturation?","Completion":"ROOT HAIRS15) Earth and Space - Multiple Choice Which variable affecting the speed of sound in the ocean dominates at very large depths?"} {"Type":"## BONUS\n\nEarth and Space - Short Answer","Prompt":"Plutinos are Kuiper belt objects whose orbits are locked in a 3-to-2 resonance with what planet?","Completion":"NEPTUNE"} {"Type":"TOSS-UP\n\nMath - Short Answer","Prompt":"In base 8, what is 6 times 7?","Completion":"FIVE-TWO"} {"Type":"## BONUS\n\nMath - Short Answer","Prompt":"Rounded to the nearest integer, what is the area of a circle of circumference 15?","Completion":"1817) Physics - Multiple Choice A swinging door is opened to +80 degrees, and released. It swings to -40 degrees, then back to +20 degrees and so on before eventually reaching rest at 0 degrees. This oscillatory behavior can be best described as which of the following?"} {"Type":"## BONUS\n\nPhysics - Short Answer","Prompt":"A square conducting loop that has sides of length 10 centimeters is pulled out of a 5-tesla magnetic field at 1 meter per second. What is the magnitude of the induced EMF, in volts, in the loop while it is leaving the magnetic field?","Completion":"0.521) Earth and Space - Multiple Choice A necessary conclusion of Hubble's law is that very-distant galaxies are moving away from us at speeds faster than the speed of light. Which of the following correctly explains this observation?"} {"Type":"TOSS-UP\n\nChemistry - Short Answer","Prompt":"Identify all of the following three quantites in reaction kinetics that are dimensionless:\n\nRate constant; Steric [steh-rik] factor; Collision frequency.","Completion":"2"} {"Type":"TOSS-UP\n\nChemistry - Multiple Choice","Prompt":"Which of the following statements is TRUE concerning SN\\({}{2}\\) reactions?\n\nW) Pentavalent [penta-vay-lent] transition state\n\nX) Pentavalent intermediate\n\nY) Trivalent [try-vay-lent] transition state\n\nZ) Trivalent intermediate","Completion":"W) Pentavalent transition state"} {"Type":"TOSS-UP\n\nEnergy - Multiple Choice","Prompt":"Which of the following solar cell systems has the highest efficiency?\n\nW) Silicon\n\nX) Gallium [GAL-ee-um] Arsenide [AHR-sin-ide]\n\nY) Thin Film Photovoltaic [photo-vanyl-TAY-ik]\n\nZ) Polymeric [paw-lih-MARE-ik] Photovoltaic","Completion":"X) GALLIUM ARSENIDE"} {"Type":"TOSS-UP\n\nMath - Multiple Choice","Prompt":"Given that the function \\(f\\) of \\(x\\) has a zero at \\(x\\) = 6, at what x-value must 2 +f of \\(x\\) have a zero?\n\nW) 3\n\nX) 4\n\nY) 8\n\nZ) It need not have a zero","Completion":"Z) IT NEED NOT HAVE A ZERO"} {"Type":"TOSS-UP\n\nEarth and Space - Multiple Choice","Prompt":"What type of feature is the Moon's Rupes Recta?\n\nW) Fault\n\nX) Ridge\n\nY) Bay\n\nZ) Valley","Completion":"W) FAULT"} {"Type":"TOSS-UP\n\nBiology - Short Answer","Prompt":"What is the adjective for the type of nephron [NEF-ron] that possesses long loops of Henley [HEN-lee] in order to promote water reabsorption?","Completion":"JUXATADULLARY"} {"Type":"TOSS-UP\n\nPhysics - Short Answer","Prompt":"Modulo a dimensionless constant, the Heisenberg uncertainty principle relates the products of momentum and positional uncertainties to what constant?","Completion":"PLANCK'S CONSTANT (ACCEPT: REDUCED PLANCK'S)"} {"Type":"TOSS-UP\n\nEnergy - Short Answer","Prompt":"Rank the following three substances in order of increasing energy density: Bituminous coal; Gasoline; TNT.","Completion":"3, 1, 2"} {"Type":"TOSS-UP\n\nMath - Short Answer","Prompt":"What is the greatest integer that is less than or equal to the log base 6 of 975?","Completion":"3"} {"Type":"TOSS-UP\n\nEarth and Space - Multiple Choice","Prompt":"Which of these is true regarding the Sargasso sea?\n\nW) It has relatively low biological productivity\n\nX) It is adjacent to the Bay of Bengal\n\nY) It is primarily a hypoxic dead zone\n\nZ) It has the saltiest ocean water","Completion":"W) IT HAS RELATIVELY LOW BIOLOGICAL PRODUCTIVITY"} {"Type":"TOSS-UP\n\nChemistry - Multiple Choice","Prompt":"Which of the following compounds would have the largest retention factor in a TLC experiment with a silica stationary phase and hexane mobile phase?\n\nW) Benzene\n\nX) Benzoic acid [ben-ZOH-ik]\n\nY) Phenylacetone [fennel-ASS-eh-tone]\n\nZ) Benzyl [BEN-zil] methyl ether","Completion":"X) BENZOIC ACID"} {"Type":"TOSS-UP\n\nBiology - Multiple Choice","Prompt":"Which of the following scientific names correctly describes a family of plants?\n\nW) Quercus rubra\n\nX) Liliaceae [Iilly-AY-see-eye]\n\nY) Poales [Poh-AY-leez]\n\nZ) Magnoliophyta [mag-NO-lee-oh-FYE-tah]","Completion":"X) LILIaceaeAE"} {"Type":"BONUS\n\nBiology - Short Answer","Prompt":"Identify all of the following three statements that are true of second messengers:\n\nPhospholipase [faws-fo-LYE-pace] C catalyzes the release of IP3; Cells store calcium in the nucleus;\n\nCalmodulin [kal-MOD-u-lin] binds to cyclic GMP.","Completion":"1"} {"Type":"TOSS-UP\n\nChemistry - Short Answer","Prompt":"What is the empirical formula for the acid anhydride of phosphoric [faws-FOR-ik] acid?","Completion":"P2O5"} {"Type":"BONUS\n\nChemistry - Short Answer","Prompt":"Identify all of the following four compounds in which London dispersion forces dominate intermolecular interactions: Dichloromethane; Neon; Ammonia; Hydrogen.","Completion":"2 AND 417) Energy - Multiple Choice DOE scientists at SLAC [slack] have been using the Linac Coherent Light Source to study SNARE proteins in the brain. Which of the following processes are SNARE proteins involved with?"} {"Type":"## BONUS\n\nEnergy - Multiple Choice","Prompt":"Researchers at Lawrence Berkeley National Lab are currently working on optimizing plants to produce less lignin. For which of the following are these genetically modified plants likely to be used?\n\nW) Construction materials\n\nX) Ethanol production\n\nY) Insulation\n\nZ) Food","Completion":"X) ETHANOL PRODUCTION"} {"Type":"## BONUS\n\nEarth and Space - Multiple Choice","Prompt":"The orbital period of Pluto is 248 years. For what percentage of Pluto's orbital period is it closer to the Sun than Neptune is?\n\nW) 8\n\nX) 24\n\nY) 37\n\nZ) 48","Completion":"W) 8"} {"Type":"TOSS-UP\n\nChemistry - Short Answer","Prompt":"What is the major acidic species that forms when an acid is dissolved in water?","Completion":"HYDRONIUM (ACCEPT: H3O+)"} {"Type":"BONUS\n\nChemistry - Short Answer","Prompt":"Identify all of the following three compounds that have a higher lattice energy than sodium fluoride: Sodium chloride; Aluminum fluoride; Sodium oxide.","Completion":"2 AND 3"} {"Type":"TOSS-UP\n\nBiology - Short Answer","Prompt":"Which animal phylum [FYE-lum] contains the most extant species?","Completion":"ARTHROPODA"} {"Type":"BONUS\n\nBiology - Short Answer","Prompt":"Identify all of the following three nucleobases that are purines [PURE-eenzf]: Guanine [GWAH-neen]; Uracil [YUR-ah-sil]; Thymine [THIGH-men].","Completion":"1 (ACCEPT: GUANINE)"} {"Type":"TOSS-UP\n\nPhysics - Short Answer","Prompt":"What is the name for the device Michelson and Morley used to disprove the existence of luminiferous aether?","Completion":"INTERFROMETER"} {"Type":"TOSS-UP\n\nEarth and Space - Short Answer","Prompt":"What is the general term for the formation of mountains?","Completion":"OROGENY"} {"Type":"TOSS-UP\n\nMath \\(-\\) Short Answer","Prompt":"The point (5, is reflected across the \\(y\\)-axis and the resulting point is then translated 4 units down. What are the coordinates of the final image point?","Completion":"(-5, 2)"} {"Type":"TOSS-UP\n\nChemistry - Multiple Choice","Prompt":"An alkyne[AL-kine] is added to a reaction vessel containing pressurized hydrogen gas and a catalyst consisting of palladium [pah-LAID-ee-um] poisoned by traces of lead(II) ions and quinoline [KWIN-oh-lin]. Which of the following is the major product of this reaction?\n\nW) Alkane\n\nX) Cis-alkene [sis-al-KEEN\n\nY) Trans-alkene\n\nZ) Alcohol","Completion":"X) CIS-ALKENE"} {"Type":"BONUS\n\nChemistry - Short Answer","Prompt":"Identify all of the following three statements that are true concerning elimination reactions: E2 is a concerted reaction; E1 is substantially accelerated by strong, unhindered bases; Cis-alkenes are often thermodynamically preferred to trans-alkenes.","Completion":"1"} {"Type":"TOSS-UP\n\nBiology - Short Answer","Prompt":"What protein hormone, missing in obese mice with mutations in the ob [O.B.J] gene, is responsible for controlling satiety [SAY-sha-teef]?","Completion":"LEPTIN"} {"Type":"BONUS\n\nBiology - Short Answer","Prompt":"When fasting, the liver will convert amino acids to glucose. What is the name of this process?","Completion":"GLUCONEOGENESIS"} {"Type":"TOSS-UP\n\nEnergy - Multiple Choice","Prompt":"Which of the following percentages is closest to the level of engine efficiency in a modern gasoline-burning automobile?\n\nW) 10%\n\nX) 25%\n\nY) 50%\n\nZ) 90%","Completion":"X) 25%"} {"Type":"TOSS-UP\n\nPhysics - Short Answer","Prompt":"A Wheatstone Bridge is used to measure what property of electrical circuit components?","Completion":"RESISTANCE"} {"Type":"TOSS-UP\n\nMath - Short Answer","Prompt":"Givenf inverse of \\(x\\) equals \\(2-x\\), what is f(x)?","Completion":"\\(2-x\\)"} {"Type":"TOSS-UP\n\nEarth and Space - Multiple Choice","Prompt":"What radioactive element was produced during nuclear-weapons testing in the mid 1900s and is commonly used by oceanographers to date the age of water masses, but will decay to normal background levels in the coming decades.\n\nW) Carbon-14\n\nX) Radon-222\n\nY) Uranium-238\n\nZ) Tritium","Completion":"Z) TRITIUM"} {"Type":"TOSS-UP\n\nEnergy - Multiple Choice","Prompt":"Why is solar power underutilized in American households?\n\nW) It increases the price of a home by about 5-10%\n\nX) North America is mostly cloudy\n\nY) Solar technology is currently not very effective\n\nZ) Solar cells are not effective in cold climates","Completion":"W) IT INCREASES THE PRICE OF A HOME BY ABOUT 5-10%"} {"Type":"TOSS-UP\n\nChemistry - Multiple Choice","Prompt":"Which of the following methods is LEAST helpful in identifying chiral [KYE-ruhl] isomers?\n\nW) X-ray diffraction\n\nX) Quadrupole [QUAD-roo-pole] mass spectrometry [spek-TRAW-metry]\n\nY) Nuclear magnetic resonance spectroscopy [spek-TRAW-scuh-pee]\n\nZ) Raman [RAH-min] spectroscopy","Completion":"X) QUADRUPOLE MASS SPECTROMETRY"} {"Type":"TOSS-UP\n\nPhysics - Multiple Choice","Prompt":"The photoelectric effect reveals what property of photons?\n\nW) Wave-like nature\n\nX) Particle-like nature\n\nY) Wave-particle duality\n\nZ) Masslessness","Completion":"X) PARTICLE-LIKE NATURE"} {"Type":"TOSS-UP\n\nBiology - Short Answer","Prompt":"Identify all of the following three genera [JEN-er-ahl] of bacteria that are Gram positive:\n\nEscherichia [eh-sher-EE-kee-ahl]; Salmonella; Vibrio [VIH-bree-oh].","Completion":"NONE"} {"Type":"TOSS-UP\n\nMath - Short Answer","Prompt":"What is the absolute value of the complex conjugate of -4 + 5i?","Completion":"\\(\\surd\\)41"} {"Type":"TOSS-UP\n\nPhysics - Multiple Choice","Prompt":"A 70-kilogram student is walking at 1 meter per second. Which of the following is closest to her de Broglie wavelength, in meters?\n\nW) 10\\({}^{\\text{-}34}\\)\n\nX) 10\\({}^{\\text{-}35}\\)\n\nY) 10\\({}^{\\text{-}36}\\)\n\nZ) 10\\({}^{\\text{-}37}\\)","Completion":"X) 10\\({}^{\\text{-}35}\\)"} {"Type":"TOSS-UP\n\nEarth and Space - Short Answer","Prompt":"How many planes of cleavage does biotite possess?","Completion":"ONE"} {"Type":"## BONUS\n\nChemistry - Short Answer","Prompt":"How many unpaired electrons does high-spin open bracket CoF\\({}{6}\\) close bracket to the 3 negative have?","Completion":"4"} {"Type":"## BONUS\n\nEarth and Space - Short Answer","Prompt":"Prometheus and Pan both create gaps in Saturn's rings due to their gravitational influence. What is the term for this type of satellite?","Completion":"SHEPHERD"} {"Type":"TOSS-UP\n\nMath - Short Answer","Prompt":"An octagon is dilated with scale factor 7\/4. What is the ratio of the areas of the original octagon to the image?","Completion":"16\/49"} {"Type":"TOSS-UP\n\nBiology - Short Answer","Prompt":"What enzyme is the major regulation site of glycolysis [glye-KAWL-eh-sis]?","Completion":"PHOSPHOFRUCTOKINASE (ALSO ACCEPT: PFK, PFK-1)"} {"Type":"TOSS-UP\n\nBiology - Multiple Choice","Prompt":"Which of the following cell types is associated with haversian [ha-VER-zhen] canals?\n\nW) Osteocytes [AW-steo-sites]\n\nX) Chondrocytes [KON-droh-sites]\n\nY) Fibroblasts\n\nZ) Leukocytes [LOO-koh-sites]","Completion":"W) OSTEOCYTES"} {"Type":"TOSS-UP\n\nEarth and Space - Short Answer","Prompt":"The greenhouse effect on Venus is primarily attributed to what gas?","Completion":"CARBON DIOXIDE (ACCEPT: CO2)"} {"Type":"TOSS-UP\n\nMath - Short Answer","Prompt":"If 6 dice are rolled and the numbers on the top faces are added together, how many different sums are possible?","Completion":"31"} {"Type":"BONUS\n\nMath - Short Answer","Prompt":"What is the largest prime factor of the quantity\n\n\\(4^{2014}+4^{2015}+4^{2016}+4^{2017}\\)?","Completion":"17"} {"Type":"TOSS-UP\n\nEnergy - Multiple Choice","Prompt":"The Advanced Light Source can be used to identify elements in batteries and fuel cells. Which of the following tools is most likely used for these purposes at the ALS?\n\nW) Fluorescence microscopy\n\nX) X-ray absorption spectroscopy [spek-TRAW-scuh-pee]\n\nY) NMR spectroscopy [spek-TRAW-scuh-pee]\n\nZ) Confocal microscopy","Completion":"X) X-RAY ABSORPTION SPECTROSCOPY"} {"Type":"BONUS\n\nEnergy - Multiple Choice","Prompt":"Researchers at the Joint Genome [JEE-nome] Institute have been using new types of fungi [fun-jij] to produce biofuels from plant material. Where are these fungi naturally found?\n\nW) Forest soils\n\nX) Coral reefs\n\nY) Animal guts\n\nZ) Swamps","Completion":"Y) ANIMAL GUTS"} {"Type":"TOSS-UP\n\nPhysics - Short Answer","Prompt":"Compton scattering is the scattering of X-ray photons from what particle?","Completion":"ELECTRON"} {"Type":"BONUS\n\nPhysics - Multiple Choice","Prompt":"Which of the following is closest to the half-life of a free neutron?\n\nW) 1 second\n\nX) 10 minutes\n\nY) 100 hours\n\nZ) 1000 years","Completion":"X) 10 MINUTES"} {"Type":"TOSS-UP\n\nEnergy - Short Answer","Prompt":"In fluidized bed combustors used for clean coal, what is added to the coal to keep pollutants in check?","Completion":"LIMESTONE (ACCEPT: CALCIUM CARBONATE)"} {"Type":"BONUS\n\nEnergy - Short Answer","Prompt":"The great majority of the transistors in integrated circuits are of the CMOS type. What do the letters of the acronym represent?","Completion":"COMPLEMENTARY METAL OXIDE SEMICONDUCTOR"} {"Type":"TOSS-UP\n\nChemistry - Multiple Choice","Prompt":"Consider the hypothetical reaction 2A, aqueous, plus 2B, gaseous, yields 3C, aqueous, plus D, gaseous, where the change in enthalpy of the reaction is -22 kilojules. Which of the following will drive the reaction toward the products?\n\nW) Increasing the temperature\n\nX) Increasing the volume of the reaction vessel at constant temperature\n\nY) Increasing the pressure\n\nZ) Increasing the amount of D","Completion":"Y) INCREASING THE PRESSURE"} {"Type":"BONUS\n\nChemistry - Multiple Choice","Prompt":"Which of the following solvents would most accelerate an SN2 reaction?\n\nW) Hexane\n\nX) Dichloromethane\n\nY) DMSO\n\nZ) Ethyl acetate","Completion":"Y) DMSO"} {"Type":"TOSS-UP\n\nMath - Short Answer","Prompt":"In the integers mod 12, what is 5 times 8?","Completion":"4"} {"Type":"BONUS\n\nMath - Short Answer","Prompt":"\\(f\\) of \\(x\\) is a twice differentiable function such that \\(f\\) prime of \\(x\\) and \\(f\\) double prime of \\(x\\) are both less than 0 for all real numbers. If \\(f\\) of 3 equals 12 and \\(f\\) of 5 equals 8, what is the largest possible integer value for \\(f\\) of 7?","Completion":"311) Biology - Short Answer What is the genus [JEE-nus] of the harmless gut bacterium that can be transformed by a bacteriophage [bak-TER-ee-oh-fayj] into a virulent [VEGF-yoo-lent] form associated with cholera [CALL-ur-ah]?"} {"Type":"TOSS-UP\n\nBiology - Short Answer","Prompt":"What viral reproductive cycle kills the infected host cell while the virus is dividing?","Completion":"LYTIC CYCLE"} {"Type":"BONUS\n\nBiology - Short Answer","Prompt":"What is the number of cells present in a mature male angiosperm gametophyte?","Completion":"3"} {"Type":"TOSS-UP\n\nEnergy - Multiple Choice","Prompt":"Brookhaven National Lab scientists recently made strides on understanding the nature of Cooper pairs. To which of the following disciplines is this discovery most relevant?\n\nW) Particle physics\n\nX) Superconductor physics\n\nY) Nuclear physics\n\nZ) Computational chemistry","Completion":"X) SUPERCONDUCTOR PHYSICS"} {"Type":"BONUS\n\nEnergy - Multiple Choice","Prompt":"Scientists at Argonne National Lab are using x-rays to probe porosity in 3D-printed titanium alloys. Which of the following best explains why porosity is undesirable?\n\nW) Porosity decreases resistance to fatigue\n\nX) Porosity decreases flexibility\n\nY) A large range of pore sizes reduces printing resolution\n\nZ) Porosity limits the size of the structures that can be printed","Completion":"W) Porosity decreases resistance to Fatigue15) Earth and Space - Short Answer Identify all of the following three statements that are true of the inner planets:"} {"Type":"## BONUS\n\nEarth and Space - Multiple Choice","Prompt":"Which of the following is most directly responsible for ocean water\n\ndownwelling in the mid-ocean gyres?\n\nW) Ekman transport\n\nX) High surface water density\n\nY) High organic carbon content\n\nZ) Sea floor spreading","Completion":"W) EKMAN TRANSPORT"} {"Type":"## TOSS-UP\n\nPhysics - Short Answer","Prompt":"By what phenomenon can materials emit electrons in response to incident light?","Completion":"PHOTOELECTRIC EFFECT"} {"Type":"## BONUS\n\nPhysics - Short Answer","Prompt":"CPT symmetry is a fundamental concept of nuclear science and theoretical physics. What do the C, P, and T stand for?","Completion":"CHARGE CONJUGATION (ACCEPT: CHARGE), PARITY INVERSION (ACCEPT: PARITY), AND TIME REVERSAL (ACCEPT: TIME)17) Chemistry - Short Answer Rank the following three elements in terms of increasing electron affinities: 1) Carbon; 2) Nitrogen; 3) Oxygen."} {"Type":"## BONUS\n\nChemistry - Short Answer","Prompt":"What effect, which applies to the chemistry of real gases and is exploited in the liquefaction of air, causes a gas's temperature to decrease in an adiabatic [ad-ee-ah-BAT-ik] isenthalpic expansion?","Completion":"JOULE-THOMSON EFFECT (ACCEPT: JOULE-THOMSON EXPANSION, JOULE-KELVIN EFFECT)"} {"Type":"## BONUS\n\nMath - Short Answer","Prompt":"The sums of three whole numbers taken in pairs are 11, 13, and 14. What is the product of these three numbers?","Completion":"24019) Earth and Space - Short Answer What type of extrusive igneous rock is considered intermediate between basalt and dacite and possesses approximately 60% silica content?"} {"Type":"TOSS-UP\n\nBiology - Short Answer","Prompt":"The Ti (tie) plasmid can be used to stably transfect some plants with recombinant DNA. What is the genus of the bacterium that is a vector for this plasmid?","Completion":"AGROBACTERIUM"} {"Type":"BONUS\n\nBiology - Short Answer","Prompt":"Identify all of the following four cell types involved in plant transport physiology that are non-living yet functional when mature: Parenchyma [pah-REN-kimma!; Sclereid [SKLAIR-ee-id!; Sieve tube; Tracheid [TRAY-kee-id]","Completion":"2 AND 4"} {"Type":"TOSS-UP\n\nPhysics - Short Answer","Prompt":"Identify all of the following three particles that, according to the standard model, can be affected by gravity: Electron; Pion; Gluon.","Completion":"1 AND 2"} {"Type":"BONUS\n\nPhysics - Short Answer","Prompt":"In fluorescence microscopy, energy from absorbed photons is re-emitted as new photons. If the new photons have only two-thirds of the momentum of the absorbed photons, what is the ratio of the wavelength of the new photons to that of the absorbed photons?","Completion":"1.5 (ACCEPT: 3\/2, 1 %, 3 TO 2)23) Chemistry - Short Answer What is the IUPAC name for the cyclic saturated hydrocarbon containing four carbon atoms?"} {"Type":"## BONUS\n\nChemistry - Short Answer","Prompt":"Identify all of the following three functional groups that commonly participate in addition reactions: Aldehyde [AL-deh-hide]; Alkene ; Alkyl [AL-kill] halide [HAY-lide].","Completion":"1 AND 2"} {"Type":"TOSS-UP\n\nPhysics - Short Answer","Prompt":"Kirchhoff's loop rule is a consequence of what conservation principle?","Completion":"CONSERVATION OF ENERGY (ACCEPT: ENERGY)"} {"Type":"BONUS\n\nPhysics - Short Answer","Prompt":"A spacecraft approaches Jupiter head-on for a gravitational slingshot maneuver. Jupiter has an orbital velocity of 13 kilometers per second. If the spacecraft was originally moving at 20 kilometers per second, what is the maximal final speed, in kilometers per second, it can attain?","Completion":"46"} {"Type":"TOSS-UP\n\nMath - Short Answer","Prompt":"What is the distance from the point (1, 3, to the origin?","Completion":"\\(\\surd{35}\\)"} {"Type":"BONUS\n\nMath - Short Answer","Prompt":"Consider an infinite geometric series with sum equal to 8\/3 and first term 7\/5. What is the value of the common ratio \\(r\\) for this series?","Completion":"19\/40"} {"Type":"TOSS-UP\n\nBiology - Multiple Choice","Prompt":"Which of the following organisms possesses enidocytes [NYE-doh-sites]?\n\nW) Tapeworm\n\nX) Jellyfish\n\nY) Starfish\n\nZ) Sea cucumber","Completion":"X) JELLYFISH"} {"Type":"TOSS-UP\n\nEnergy - Multiple Choice","Prompt":"Which of the following sources of electricity had the largest increase in capacity in the US between 1990 and 2013?\n\nW) Nuclear\n\nX) Coal\n\nY) Natural gas\n\nZ) Petroleum","Completion":"Y) NATURAL GAS"} {"Type":"","Prompt":"TOSS-UP\n\nEarth and Space - Multiple Choic Saturn's rings are mostly composed of which of the following materials?\n\nW) Ice\n\nX) Silicates\n\nY) Carbonates\n\nZ) Iron","Completion":"W) ICE"} {"Type":"TOSS-UP\n\nChemistry - Multiple Choice","Prompt":"Which of the following is not true about early transition metals in comparison to late transition metals?\n\nW) Easier to oxidize\n\nX) More electropositive\n\nY) Higher oxidation states\n\nZ) Larger radii","Completion":"W) EASIER TO OXIDIZE"} {"Type":"TOSS-UP\n\nBiology - Short Answer","Prompt":"What is the term for cancers that arise in epithelial [eh-pih-THEEL-ee-ul] tissues?","Completion":"CARCINOMA"} {"Type":"BONUS\n\nBiology - Short Answer","Prompt":"Identify all of the following three statements that are true regarding tendrils in plants:\n\nThey can only be found in dicot vines; They can be modified stems, leaves, or flower stalks; They are primarily used for climbing.","Completion":"2 AND 3"} {"Type":"TOSS-UP\n\nPhysics - Short Answer","Prompt":"What is the minimum number of photons created in an electron-positron annihilation in free space?","Completion":"2"} {"Type":"BONUS\n\nPhysics - Short Answer","Prompt":"Because of observed neutrino oscillation, what property of the neutrino must be non-zero?","Completion":"MASS9) Chemistry - Short Answer Consider a single-reactant reaction that has a rate constant equal to 24 seconds to the negative 1, molar to the negative 2. What is the order of this reaction?"} {"Type":"TOSS-UP\n\nPhysics - Short Answer","Prompt":"In an RLC circuit, the resonance frequency is defined to be at the point at which what quantity is minimized?","Completion":"IMPEDENCE"} {"Type":"TOSS-UP\n\nEarth and Space - Short Answer","Prompt":"Eventually, some stars will begin to fuse helium into carbon. What is the name of this process?","Completion":"TRIPLE ALPHA PROCESS"} {"Type":"TOSS-UP\n\nChemistry - Short Answer","Prompt":"Identify all of the following three species that are Lewis acids:\n\nZn\\({}^{2+}\\); Cl'; BH\\({}{3}\\).","Completion":"1 AND 3"} {"Type":"TOSS-UP\n\nMath - Short Answer","Prompt":"The line with equation \\(y=-2x+5\\) is reflected across the \\(y\\)-axis. What is the slope-intercept equation of its image?","Completion":"\\(y=2x+5\\)"} {"Type":"## BONUS\n\nEnergy - Short Answer","Prompt":"Scientists at Brookhaven National Lab discovered that layering graphene on top of soda-line glass resulted in a large change in the graphene's electronic properties. What specific element in the glass was found to be responsible for these changes?","Completion":"SODIUM"} {"Type":"### TOSS-UP\n\nBiology - Short Answer","Prompt":"From what group of organisms are restriction enzymes isolated?","Completion":"BACTERIA (ACCEPT: EUBACTERIA)"} {"Type":"## BONUS\n\nBiology - Short Answer","Prompt":"What storage polysaccharide that is found in animals contains highly branched chains of sugars containing 1, 4 and 1, 6 linkages?","Completion":"GLYCOGEN19) Earth and Space - Short Answer What term is given to the layer of an ocean or lake that extends from the surface to the depth at which 99 percent of the surface light has been absorbed?"} {"Type":"## BONUS\n\nEarth and Space - Short Answer","Prompt":"What is the name for a cloud that looks like a pouch and often forms hanging down from the underside of the anvil of a cumulonimbus cloud?","Completion":"MAMMATUS"} {"Type":"### TOSS-UP\n\nPhysics - Short Answer","Prompt":"Charged-coupled devices and complementary metal oxide sensors are devices typically used to detect what?","Completion":"LIGHT (ACCEPT: ELECTROMAGNETIC RADIATION)"} {"Type":"## BONUS\n\nPhysics - Multiple Choice","Prompt":"The resulting kinetic energy of an ejected photo-electron is NOT determined by which of the following?\n\nW) Speed of light\n\nX) Energy of light\n\nY) Work function of the metal\n\nZ) Light intensity","Completion":"Z) LIGHT INTENSITY"} {"Type":"TOSS-UP\n\nMath - Short Answer","Prompt":"Using the point names \\(A\\), \\(B\\), \\(C\\), \\(D\\), and \\(E\\), how many different names are there for pentagon ABCDE?","Completion":"10"} {"Type":"TOSS-UP\n\nChemistry - Short Answer","Prompt":"What is the bond-order of an oxygen-oxygen bond in ozone?","Completion":"1.5"} {"Type":"TOSS-UP\n\nBiology - Short Answer","Prompt":"What technique would scientists use if they wanted to amplify a small region of DNA?","Completion":"PCR (ACCEPT: POLYMERASE CHAIN REACTION)"} {"Type":"TOSS-UP\n\nBiology - Multiple Choice","Prompt":"The majority of speciation f[spee-see-AY-shun] occurs under which of the following circumstances?\n\nW) When populations undergo sexual selection X) When populations become geographically separated Y) When populations are R selected Z) When populations reach carrying capacity","Completion":"X) WHEN POPULATIONS BECO REGGRAPHICALLY SEPARATED"} {"Type":"TOSS-UP\n\nChemistry - Multiple Choice Which of the following molecular geometries can exhibit meridional isomerism?\n\nW) Linear\n\nX) Square planar\n\nY) Tetrahedral\n\nZ) Octahedral\n\nBonus\n\nChemistry - Short Answer","Prompt":"What is the fewest number of carbonyl [carbon-EEL] stretching bands that can be seen in the infrared spectrum of a metal atom that is bound to four CO ligands [LIH-gunz]?","Completion":"1"} {"Type":"TOSS-UP\n\nEarth and Space - Multiple Choice","Prompt":"Comet McNaught visited in 2007 but is now on a path to leave the solar system forever. Which of the following best describes its orbit?\n\nW) Circular\n\nX) Elliptical\n\nY) Parabolic\n\nZ) Hyperbolic","Completion":"Z) HYPERBOLIC"} {"Type":"TOSS-UP\n\nMath - Short Answer","Prompt":"Consider a triangle with two sides with lengths of 4 and 9. How many different integers could be the length of the third side?","Completion":"7"} {"Type":"TOSS-UP\n\nEnergy \\(-\\) Multiple Choice","Prompt":"The energy price spikes in the 1970s and in 2008 suggested which of the following about U.S. energy usage?\n\nW) Usage rises in response to rising prices\n\nX) Usage stays the same when prices rise\n\nY) Usage drops in response to rising prices\n\nZ) There was no consistent trend about U.S. energy usage","Completion":"Y) USAGE DROPS IN RESPONSE TO RISING PRICES"} {"Type":"TOSS-UP\n\nPhysics \\(-\\) Short Answer","Prompt":"What is the name of the low-temperature state characterized by zero resistance transport of electron pairs?","Completion":"SUPERCONDUCTIVITY"} {"Type":"TOSS-UP\n\nChemistry - Short Answer","Prompt":"Identify all of the following three fatty acids that are unsaturated: C\\({}{13}\\)H\\({}{27}\\)COOH; C\\({}{17}\\)H\\({}{35}\\)COOH; C\\({}{16}\\)H\\({}{29}\\)COOH.","Completion":"JUST 3"} {"Type":"TOSS-UP\n\nEnergy - Short Answer","Prompt":"In a junction transistor used as a switch, which terminal controls the current through the other two terminals?","Completion":"BASE"} {"Type":"BONUS\n\nEnergy - Multiple Choice","Prompt":"Oil shale is a type of organic-rich sedimentary rock that can be used to produce shale oil, also known as tight oil, which is a substitute for conventional crude oil. Which of the following varieties of oil shale is NOT classified as a marine shale?\n\nW) Kukersite\n\nX) Torbanite\n\nY) Tasminite\n\nZ) Marinite","Completion":"X) TORBANITE11) Physics - Short Answer How much work, in joules foolsly, is done on a particle of charge 2 coulombs as it accelerates over a distance of 1 meter in an electric field of strength 9 volts per meter?"} {"Type":"TOSS-UP\n\nMath - Short Answer","Prompt":"Evaluate the following expression: -i[33].","Completion":"-i"} {"Type":"TOSS-UP\n\nEnergy - Multiple Choice","Prompt":"The further advancement of rechargeable batteries is currently hindered by unavoidable growth of what during battery cell recharge?\n\nW) Lithium dendrites\n\nX) Potassium salts\n\nY) Nickel deposits\n\nZ) Lead precipitates","Completion":"W) LITHIUM DENDRITES"} {"Type":"BONUS\n\nEnergy - Multiple Choice","Prompt":"Lawrence Livermore National Laboratory's National Ignition Facility is using what technology to further nuclear fusion energy research?\n\nW) Magnetic plasma confinement\n\nX) Toroidal plasma confinement\n\nY) Laser inertial confinement\n\nZ) Electromagnetic implosion","Completion":"Y) LASER INERTIAL CONFINEMENT"} {"Type":"TOSS-UP\n\nPhysics - Short Answer","Prompt":"The precession of Mercury's orbit about the sun presented an anomaly for the scientists that observed it. What theory correctly predicted the magnitude of precession each year?","Completion":"GENERAL RELATIVITY (ACCEPT: GENERAL THEORY OF RELATIVITY)"} {"Type":"BONUS\n\nPhysics - Short Answer","Prompt":"A charged conducting sphere is connected by a long, thin conducting wire to an uncharged conducting sphere that has half the radius of the first sphere. After some time has passed, what fraction of the original charge remains on the first sphere?","Completion":"2\/315) Earth and Space - Short Answer In 1967, Jocelyn Bell detected a regularly pulsing radio signal, which she called LGM. What type of celestial object produced this signal?"} {"Type":"TOSS-UP\n\nPhysics - Short Answer","Prompt":"What quark was most recently discovered?","Completion":"TOP QUARK"} {"Type":"BONUS\n\nPhysics - Short Answer","Prompt":"If an electron is moving in the positive-\\(x\\) direction with increasing speed, identify all of the following three scenarios that could be responsible for the driving force: A magnetic field pointing in the negative-\\(x\\) direction; An electric field pointing in the negative-\\(x\\) direction; A magnetic field pointing in the negative-\\(z\\) direction.","Completion":"2"} {"Type":"TOSS-UP\n\nMath - Short Answer","Prompt":"What base-10 numeral does the following Roman numeral represent: CMXLVI?","Completion":"946"} {"Type":"BONUS\n\nMath - Short Answer","Prompt":"What is the following cross product of vectors: \\(5i-3j+k\\) cross \\(4j\\)?","Completion":"-\\(4i+20k\\)"} {"Type":"TOSS-UP\n\nBiology - Short Answer","Prompt":"What is the term for mobile elements of DNA that can cut and paste themselves into new locations?","Completion":"TRANSPOSONS"} {"Type":"TOSS-UP\n\nPhysics - Multiple Choice","Prompt":"Which of the following is characterized by the lack of a distinct melting point?\n\nW) Sodium bicarbonate\n\nX) Quartz\n\nY) Stainless steel of a known composition\n\nZ) Glass","Completion":"Z) GLASS"} {"Type":"TOSS-UP\n\nChemistry - Multiple Choice","Prompt":"In degrees, which of the following is closest to the bond angle between hydrogen atoms in ammonia?\n\nW) 92\n\nX) 108\n\nY) 123\n\nZ) 180","Completion":"X) 108"} {"Type":"TOSS-UP\n\nMath - Multiple Choice","Prompt":"As \\(x\\) increases from -\\(\\pi\\) to \\(\\pi\\), how does the value of cosine \\(x\\) change?\n\nW) always decreases\n\nX) always increases\n\nY) decreases then increases\n\nZ) increases then decreases","Completion":"Z) INCREASES THEN DECREASES"} {"Type":"TOSS-UP\n\nBiology - Multiple Choice","Prompt":"Which of the following types of bonds link monomers together in a polypeptide?\n\nW) Glycosidic [Iglye-koh-SHH-dikJ\n\nX) Ester\n\nY) Amide [AM-ide]\n\nZ) Ether [EE-thur]","Completion":"Y) AMIDE"} {"Type":"TOSS-UP\n\nChemistry - Multiple Choice","Prompt":"Which of the following compounds is INSOLUBLE in water?\n\nW) Copper(I) chloride\n\nX) Ammonium chromate\n\nY) Sodium silicate\n\nZ) Strontium [STRAWNCH-ee-um] chloride","Completion":"W) COPPER(I) CHLORIDE"} {"Type":"BONUS\n\nChemistry - Short Answer","Prompt":"Consider a metal with atomic radius 1 angstrom, which crystallizes in a conventional fcc unit cell. To one decimal place and in angstroms, what is the unit cell edge length of the fcc lattice?","Completion":"2.8"} {"Type":"TOSS-UP\n\nBiology - Multiple Choice","Prompt":"The thumb's opposability can be attributed to what type of synovial joint?\n\nW) Pivot joint\n\nX) Saddle joint\n\nY) Hinge joint\n\nZ) Condyloid joint","Completion":"X) SADDLE JOINT"} {"Type":"BONUS\n\nBiology - Multiple Choice","Prompt":"Oswald Avery found that an \"infectious principle\" could be transferred from dead, virulent streptococcus bacteria to living, avirulent bacteria, causing them to become virulent. Which of the following is NOT true of this infectious principle?\n\nW) This material can be separated from protein in an ultracentrifuge [ultra-SEN-trih-fyoojl]\n\nX) RNA-digesting enzymes have no effect on the activity of this material\n\nY) Performing a hydrophobic [high-droh-FOH-bik] extraction removes the material from water\n\nZ) DNAse [D.N. ace] destroys all activity of this material","Completion":"Y) PERFORMING A HYDROphobic EXTRACTION REMOVES THE MATERIAL FROM WATER"} {"Type":"TOSS-UP\n\nPhysics - Short Answer","Prompt":"What quantum mechanical principle states that no two fermions can possess the same quantum numbers?","Completion":"PAULI EXCLUSION PRINCIPLE"} {"Type":"BONUS\n\nPhysics - Multiple Choice","Prompt":"A sample of an ideal gas has a volume of 1 cubic meter at standard temperature and pressure. Which of the following is closest to the number of molecules in the sample?\n\nW) 2.5 times 10 to the 24th\n\nX) 2.7 times 10 to the 25th\n\nY) 3.0 times 10 to the 25th\n\nZ) 3.7 times 10 to the 25th","Completion":"X) 2.7 TIMES 10 TO THE 25TH"} {"Type":"TOSS-UP\n\nMath - Short Answer","Prompt":"In what logarithm base is the log of 1\/27 equal to -3?","Completion":"3"} {"Type":"BONUS\n\nMath - Short Answer","Prompt":"Evaluate the integral from 0 to \\(\\pi\\)\/3 of sine \\(3x\\,dx\\).","Completion":"2\/3"} {"Type":"TOSS-UP\n\nEnergy - Multiple Choice","Prompt":"Which of the following is closest to the percent of the world's electricity that is supplied by nuclear power plants?\n\nW) 2\n\nX) 15\n\nY) 30\n\nZ) 50","Completion":"X) 15"} {"Type":"TOSS-UP\n\nEarth and Space - Multiple Choice","Prompt":"What type of high strain metamorphism occurs when two fault blocks rub together, breaking up rock during shearing?\n\nW) Hydrothermal\n\nX) Regional\n\nY) Contact\n\nZ) Cataclastic","Completion":"Z) CATACLASTIC"} {"Type":"TOSS-UP\n\nEnergy - Short Answer","Prompt":"DOE researchers have been studying an allotrope of phosphorus with regard to electrical and thermal conductivity and have found that it demonstrates properties similar to graphite. What is the common name of this allotrope?","Completion":"BLACK PHOSPHORUS (ACCEPT: PHOSPHORENE)"} {"Type":"BONUS\n\nEnergy - Short Answer","Prompt":"The Spallation Neutron Source is a user facility at Oak Ridge National Lab that can be used to investigate molecular structure. One method of increasing contrast in neutron spallation experiments is by replacing some protium hydrogen atoms with what?","Completion":"DEUTERIUM (ACCEPT: HEAVY HYDROGEN)"} {"Type":"TOSS-UP\n\nBiology - Short Answer","Prompt":"Sea urchins and sand dollars belong to what phylum [FYE-lum]?","Completion":"ECHINODERMATA"} {"Type":"BONUS\n\nBiology - Multiple Choice","Prompt":"Cryptonephridia [kripto-neh-FRID-ee-ah] can be found in the excretory systems of which of the following organisms?\n\nW) Desert rats\n\nX) Mealworm beetle larvae\n\nY) Horned lizards\n\nZ) Goldfish","Completion":"X) MEALWORM BEETLE LARVAE"} {"Type":"TOSS-UP\n\nMath - Short Answer","Prompt":"The sum of two consecutive prime numbers is 30. What is the larger of the two numbers?","Completion":"17"} {"Type":"TOSS-UP\n\nChemistry - Multiple Choice","Prompt":"Which of the following statements concerning hydrogen bonding is NOT true?\n\nW) Amines [ah-MEENS] can act as hydrogen bond donors and acceptors\n\nX) Ethers can form hydrogen bonds with each other\n\nY) The salicylate [sah-LIH-sil-atel] anion [AN-eye-on] is stabilized by intramolecular hydrogen bonding\n\nZ) Hydrogen sulfide cannot engage in hydrogen bonding","Completion":"X) Ethers CAN FORM HYDROGEN BONDS WITH EACH OTHER"} {"Type":"### 11.1 TOSS-UP\n\nEarth and Space - Short Answer","Prompt":"Identify all of the following four rocks that would be classified as foliated metamorphic rocks: Geniss; Marble; Quartzite; Phyllite.","Completion":"1, 4 (ACCEPT: GREISS AND PHYLLITE)"} {"Type":"TOSS-UP\n\nEarth and Space - Short Answer","Prompt":"Rank the following three magma types from least to greatest in terms of silica content: Andesitic; Rhyolitic; Basaltic.","Completion":"3, 1, 2"} {"Type":"TOSS-UP\n\nChemistry - Short Answer","Prompt":"What is the IUPAC name of the smallest aromatic alcohol?","Completion":"PHENOL"} {"Type":"ANSWER: 1 AND 2TOSS-UP\n\nBiology - Short Answer","Prompt":"To what phylum [FYE-lum] do most mushrooms belong?","Completion":"BASIDIOMYCOTA"} {"Type":"## BONUS\n\nBiology - Short Answer","Prompt":"Identify all of the following three organisms that are platyhelminths [plat-in-HELL-minths]: Octopus; Hookworm; Liver fluke.","Completion":"3"} {"Type":"TOSS-UP\n\nEnergy - Multiple Choice","Prompt":"Into which of the following products is the largest percentage of a barrel of oil refined?\n\nW) Gasoline\n\nX) Heating oil\n\nY) Jet fuel\n\nZ) Diesel","Completion":"W) GASOLINE"} {"Type":"TOSS-UP\n\nEarth and Space - Multiple Choice","Prompt":"Which of the following earthquake waves travels the slowest?\n\nW) Body\n\nX) Primary\n\nY) Secondary\n\nZ) Rayleigh","Completion":"Z) RAYLEIGH"} {"Type":"TOSS-UP\n\nChemistry - Multiple Choice","Prompt":"Which of the following molecules is nonpolar?\n\nW) Acetone\n\nX) Carbon tetrachloride\n\nY) Sulfur dioxide\n\nZ) Iodine chloride","Completion":"X) CARBON TETRACHLORIDE"} {"Type":"TOSS-UP\n\nMath - Short Answer","Prompt":"Giving your answer as a decimal, increasing a number by 250% is equivalent to multiplying it by what?","Completion":"3.5"} {"Type":"## BONUS\n\nEarth and Space - Short Answer","Prompt":"A star has a parallax of 0.4 arcseconds. To the nearest whole light year, how far away is the star?","Completion":"8"} {"Type":"## BONUS\n\nPhysics - Multiple Choice","Prompt":"Which of the following is closest to the amount of energy, in joules [jools], carried by a mole of photons with a wavelength of one micrometer?\n\nW) 12,000\n\nX) 24,000\n\nY) 120,000\n\nZ) 240,000","Completion":"Y) 120,000"} {"Type":"TOSS-UP\n\nEnergy - Multiple Choice","Prompt":"Most lignite mined in the United States comes from what state?\n\nW) California\n\nX) Wisconsin\n\nY) Texas\n\nZ) Pennsylvania","Completion":"Y) Texas"} {"Type":"TOSS-UP\n\nMath - Short Answer","Prompt":"What is the distance between the points (-1, and (-10, ?","Completion":"\\(\\surd\\)130"} {"Type":"TOSS-UP\n\nEarth and Space - Multiple Choice","Prompt":"Which of the following is NOT true regarding white dwarfs?\n\nW) They shrink in radius as they get cooler\n\nX) They do not perform nuclear fusion\n\nY) Their mass is mostly composed of carbon and oxygen\n\nZ) They can have atmospheres of helium","Completion":"W) They shrink in radius as they get cooler"} {"Type":"BONUS\n\nChemistry - Short Answer","Prompt":"Consider the reaction of 1 mole of nitrogen gas and 1 mole of oxygen gas to generate 2 moles of NO gas. In air at 20 degrees Celsius, the equilibrium abundance of NO is about one part in 10\\({}^{16}\\). To one significant figure and in scientific notation, what is the equilibrium constant of this reaction?","Completion":"6 TIMES 10\\({}^{32}\\)"} {"Type":"TOSS-UP\n\nEnergy - Short Answer","Prompt":"Researchers at Lawrence Berkeley National Lab are currently using statistical methods to determine the destiny of every cell in the Drosophila [droh-SAWF-il-ah] zygote [ZYE-goat] as it becomes an adult, as has been previously done for C. elegans. What is the term for this methodology?","Completion":"FATE MAPPING (ACCEPT: CELL LINEAGE TRACING)"} {"Type":"TOSS-UP\n\nBiology - Short Answer","Prompt":"What human organ produces fibrinogen [fye-BRIN-oh-jen]?","Completion":"LIVER"} {"Type":"TOSS-UP\n\nPhysics - Multiple Choice","Prompt":"Which of the following is not a logical corollary to special relativity?\n\nW) Time dilation\n\nX) Relativity of simultaneity\n\nY) Curvature of spacetime\n\nZ) Length contraction","Completion":"Y) CURVATURE OF SPACETIME"} {"Type":"TOSS-UP\n\nMath - Short Answer","Prompt":"Solve the following equation for \\(x\\): log base 3 of\n\nopen parenthesis \\(x+5\\) close parenthesis = 4?","Completion":"76"} {"Type":"TOSS-UP\n\nBiology - Short Answer","Prompt":"In what human organ can one find cells possessing intercalated [in-TER-cah-lated] disks?","Completion":"HEART"} {"Type":"TOSS-UP\n\nChemistry - Multiple Choice","Prompt":"Which of the following graphs would produce a straight line if plotted for a zero-order irreversible reaction?\n\nW) Concentration versus time\n\nX) Inverse concentration versus time\n\nY) Log of concentration versus time\n\nZ) Inverse concentration squared versus time","Completion":"W) CONCENTRATION VERSUS TIME"} {"Type":"TOSS-UP\n\nPhysics - Short Answer","Prompt":"The maximal increase in the velocity of an ideal rocket is linearly proportional to what property of the rocket?","Completion":"EXHAUST VELOCITY (ACCEPT: EXHAUST SPEED)"} {"Type":"TOSS-UP\n\nBiology - Multiple Choice","Prompt":"The three-dimensional or folded shape of a protein is what hierarchical structure level?\n\nW) Primary\n\nX) Secondary\n\nY) Tertiary\n\nZ) Quaternary","Completion":"Y) TERTIARY"} {"Type":"BONUS\n\nBiology - Multiple Choice","Prompt":"Which of the following is NOT a basis for assays to measure cell numbers or cell proliferation?\n\nW) Presence of cell proliferation antigens\n\nX) Rate of DNA replication\n\nY) Rate of RNA replication\n\nZ) Measurement of ATP concentration","Completion":"Y) RATE OF RNA REPLICATION"} {"Type":"BONUS\n\nEnergy - Multiple Choice","Prompt":"Which of the following best explains why propane is stored in liquid form rather than as a gas?\n\nW) It is less flammable as a liquid\n\nX) It takes up much less space as a liquid\n\nY) It is easier to burn as a liquid\n\nZ) It is a liquid at room temperature","Completion":"X) IT TAKES UP MUCHLES SPACE AS A LIQUID"} {"Type":"TOSS-UP\n\nPhysics - Multiple Choice","Prompt":"Which of the following thermodynamic properties is not a state function?\n\nW) Energy\n\nX) Enthalpy [EN-thul-pee]\n\nY) Heat\n\nZ) Entropy [EN-troth-pee]","Completion":"Y) HEAT"} {"Type":"TOSS-UP\n\nEarth and Space - Multiple Choice","Prompt":"What type of metamorphism occurs when magma is injected into crustal rock and the heat of the intrusion causes the crustal rock to metamorphose?\n\nW) Contact\n\nX) Regional\n\nY) Burial\n\nZ) Hydrothermal","Completion":"W) CONTACT"} {"Type":"TOSS-UP\n\nChemistry - Multiple Choice","Prompt":"Which of the following elements is the most electronegative?\n\nW) Beryllium\n\nX) Sodium\n\nY) Calcium\n\nZ) Cesium","Completion":"W) BERYLLIUM"} {"Type":"TOSS-UP\n\nMath - Short Answer","Prompt":"In a room of 12 people, two meet for a handshake. How many different such meetings could take place?","Completion":"66"} {"Type":"TOSS-UP\n\nEnergy - Multiple Choice","Prompt":"What nonrenewable energy source represents the greatest percentage of US energy consumption?\n\nW) Coal\n\nX) Natural gas\n\nY) Propane\n\nZ) Petroleum","Completion":"Z) PETROLEUM"} {"Type":"TOSS-UP\n\nMath - Short Answer","Prompt":"If \\(a\\) equals \\(6\\) cosine theta and \\(b\\) equals \\(6\\) sine theta, what is the square root of open parenthesis \\(a^{2}+b^{2}\\) close parenthesis?","Completion":"6"} {"Type":"TOSS-UP\n\nChemistry - Short Answer","Prompt":"How many d electrons are present in a cobalt ion in the +2 state?","Completion":"7"} {"Type":"BONUS\n\nChemistry - Short Answer","Prompt":"Rank the following four elements in terms of increasing melting point:\n\nBoron; Sodium; Lithium; Tungsten.","Completion":"2, 3, 1, 4 (ACCEPT: SODIUM, LITHIUM, BORON, TUNGSTEN)"} {"Type":"TOSS-UP\n\nBiology - Multiple Choice","Prompt":"In what part of a plant is the casparian strip located?\n\nW) The bark\n\nX) The stomatal guard cells\n\nY) The endodermis of the root\n\nZ) The stem cortex","Completion":"Y) THE ENDODERMIS OF THE ROOT"} {"Type":"BONUS\n\nBiology - Short Answer","Prompt":"What type of symmetry do platyhelminths [plat-ih-HELL-minths] possess?","Completion":"BILATERAL11) Physics - Short Answer Given a graph with acceleration on the \\(y\\)-axis and time on the \\(x\\)-axis, what physical quantity does the area under the curve represent?"} {"Type":"TOSS-UP\n\nMath - Short Answer","Prompt":"How many faces does a hexagonal pyramid have?","Completion":"7"} {"Type":"TOSS-UP\n\nEnergy - Multiple Choice","Prompt":"DOE researchers are studying unique chemical bonds formed by interactions between lithium cations [CAT-eye-onz] and organo-aluminum compounds. Which of the following bonds are formed by these interactions?\n\nW) Two-center one-electron bonds\n\nX) Two-center two-electron bonds\n\nY) Three-center two-electron bonds\n\nZ) Four-center one-electron bonds","Completion":"Y) THREE-CENTER TWO-ELECTRON BONDS"} {"Type":"TOSS-UP\n\nEarth and Space - Multiple Choice","Prompt":"Which of the following materials would make the best aquitard?\n\nW) Sand\n\nX) Limestone\n\nY) Clay\n\nZ) Gravel","Completion":"Y) CLAY"} {"Type":"TOSS-UP\n\nBiology - Short Answer","Prompt":"What type of muscle is found in the intestinal wall?","Completion":"SMOOTH (ACCEPT: SMOOTH MUSCLE)"} {"Type":"## BONUS\n\nPhysics - Short Answer","Prompt":"Identify all of the following three statements that are true of light: Diffraction is an example of the wave-nature of light; Interference is an example of the wave-nature of light; The energy carried by light is directly proportional to frequency.","Completion":"ALL (ACCEPT: 1, 2, 3)"} {"Type":"## BONUS\n\nChemistry - Short Answer","Prompt":"Consider the SN2 reaction of 1-chloropentane with the azide [AY-zide] ion. What is the LUMO [LOO-mo] of the electrophile in this reaction?","Completion":"SIGMA-STAR ORBITAL (ACCEPT SIGMA-STAR C-CI ORBITAL, SIGMA ANTIBONDING C-CI ORBITAL)"} {"Type":"TOSS-UP\n\nEarth and Space - Short Answer","Prompt":"Which of the four terrestrial planets is most unlike the others in density?","Completion":"MARS"} {"Type":"BONUS\n\nEarth and Space - Multiple Choice","Prompt":"What phenomenon is believed to be the main driver in the accelerated expansion of the universe?\n\nW) Dark matter\n\nX) Black hole evaporation\n\nY) Dark energy\n\nZ) Quantum gravity","Completion":"Y) DARK ENERGY"} {"Type":"TOSS-UP\n\nChemistry - Short Answer","Prompt":"Identify all of the following three statements concerning enthalpy [EN-thul-pee] that are TRUE: The enthalpy [EN-thul-pee] of formation of diamond is equal to zero; The enthalpy change of an isothermal contraction is less than zero; The enthalpy of solution of ammonium nitrate is greater than zero.","Completion":"3"} {"Type":"BONUS\n\nChemistry - Short Answer","Prompt":"Identify all of the following three compounds that have a standard enthalpy of formation of zero: Ozone; Diamond; Rhombic sulfur.","Completion":"3"} {"Type":"TOSS-UP\n\nPhysics - Short Answer","Prompt":"In a viscous fluid, what is the type of flow characterized by having layers of currents that are all parallel to the flow?","Completion":"LAMINAR (ACCEPT: STREAMLINE)"} {"Type":"TOSS-UP\n\nMath - Short Answer","Prompt":"What is 4 times 3 squared?","Completion":"36"} {"Type":"TOSS-UP\n\nBiology - Short Answer","Prompt":"What type of cells are found in the lacunae [lah-KOO-nay] of cartilage?","Completion":"CHONDROCYTES"} {"Type":"BONUS\n\nBiology - Short Answer","Prompt":"Identify all of the following three cell types that are leukocytes [LOO-koh-sites]:\n\nEosinophil [ee-oh-SIN-oh-fill]; Basophil [BAY-zoh-fill]; Erythrocyte [eh-RITH-roh-site].","Completion":"1 AND 2"} {"Type":"TOSS-UP\n\nBiology - Multiple Choice","Prompt":"A deficiency of what trace element can result in goiter?\n\nW) Sodium\n\nX) Chlorine\n\nY) Potassium\n\nZ) Iodine","Completion":"Z) Iodine"} {"Type":"BONUS\n\nBiology - Multiple Choice","Prompt":"Which of the following is NOT a symptom or sign of a person suffering from diabetes mellitus?\n\nW) Increase in protein breakdown\n\nX) Presence of sugar in urine\n\nY) Decrease in the amount of urine production\n\nZ) Increased levels of blood glucose","Completion":"Y) DECREASE IN THE AMOUT OF Urine PRODUCTION"} {"Type":"TOSS-UP\n\nMath - Multiple Choice","Prompt":"The natural log of which of the following is irrational?\n\nW) 1\n\nX) \\(\\surd e\\)\n\nY) \\(e\\)\n\nZ) 10","Completion":"Z) 10"} {"Type":"BONUS\n\nMath - Short Answer","Prompt":"If two sides of a right triangle have lengths 13 and 7, then, rounded to the nearest whole numbers, what are the possible lengths of the third side?","Completion":"11 and 15"} {"Type":"TOSS-UP\n\nEnergy - Multiple Choice","Prompt":"Which of the following energy sources produces the least carbon dioxide per unit of energy?\n\nW) Oil\n\nX) Coal\n\nY) Natural gas\n\nZ) Nuclear","Completion":"Z) NUCLEAR"} {"Type":"TOSS-UP\n\nChemistry - Multiple Choice","Prompt":"Which of the following atoms is commonly found in a +3 oxidation state?\n\nW) Calcium\n\nX) Nickel\n\nY) Iron\n\nZ) Zirconium","Completion":"Y) IRON"} {"Type":"TOSS-UP\n\nEarth and Space - Multiple Choice","Prompt":"What type of earthquake wave is associated with ground roll?\n\nW) Love wave\n\nX) P-wave\n\nY) Rayleigh wave\n\nZ) S-wave","Completion":"Y) RAYLEIGH WAVE"} {"Type":"TOSS-UP\n\nPhysics - Multiple Choice","Prompt":"If a constant force is applied to an object, which of the following quantities must also be constant?\n\nW) Position\n\nX) Acceleration\n\nY) Velocity\n\nZ) Speed","Completion":"X) ACCELERATION"} {"Type":"TOSS-UP\n\nMath - Short Answer","Prompt":"During football season, a player catches passes for 687 yards and averages 14.6 yards per catch, rounded to the nearest tenth. How many catches did the player make?","Completion":"47"} {"Type":"TOSS-UP\n\nPhysics - Short Answer","Prompt":"A circuit contains a 6-volt battery and a 2-ohm resistor. What is the power, in watts, of this circuit?","Completion":"18"} {"Type":"## BONUS\n\nChemistry - Short Answer","Prompt":"Rank the boiling points of the following three hydrocarbons, from lowest to highest:\n\nIsobutane; Butane; Pentane.","Completion":"1, 2, 3"} {"Type":"## BONUS\n\nEnergy - Multiple Choice","Prompt":"Which of the following is derived from kerosene?\n\nW) Gasoline\n\nX) Paraffin wax\n\nY) Jet fuel\n\nZ) Propane","Completion":"Y) JET FUEL13) Chemistry - Multiple Choice Consider an ideal gas initially occupying a volume of 5 liters and at a pressure of 5 atmospheres, that undergoes an adiabatic [ad-ee-ah-BAT-ik] expansion to a final pressure of 1 atmosphere. Which of the following is TRUE concerning the final volume of the gas?"} {"Type":"TOSS-UP\n\nPhysics - Multiple Choice","Prompt":"For which of the following is Erwin Schrodinger most famous?\n\nW) Describing how wavefunctions change over time\n\nX) Discovering that electrons occupy energy levels in atoms\n\nY) Discovering that atoms contain a positively charged nucleus\n\nZ) Discovering nuclear fission","Completion":"W) DESCRIBING HOW WAVEFUNCTIONS CHANGE OVER TIME"} {"Type":"TOSS-UP\n\nEarth and Space - Multiple Choice","Prompt":"In what cloud type does hail generally form?\n\nW) Cirrocumulus\n\nX) Cirrostratus\n\nY) Altocumulus\n\nZ) Cumulonimbus","Completion":"Z) CUMULONIMBUS"} {"Type":"TOSS-UP\n\nMath - Short Answer","Prompt":"What is 145 squared?","Completion":"21,025"} {"Type":"## BONUS\n\nPhysics - Short Answer","Prompt":"Identify all of the following 3 classes of levers that are capable of producing a mechanical advantage greater than unity: Class 1; Class 2; Class 3.","Completion":"1 AND 2"} {"Type":"## BONUS\n\nEarth and Space - Multiple Choice","Prompt":"What type of faulting is defined by strike-slip offset along an essentially vertical fault plane?\n\nW) Normal\n\nX) Reverse\n\nY) Thrust\n\nZ) Transcurrent","Completion":"Z) TRANSCURRENT21) Chemistry - Short Answer Identify all of the following three solutions that would be acidic: 1) A solution of iron(III) atoms hexacoordinated by water molecules; 2) A solution of ammonium sulfate; 3) A solution of sodium cyanide."} {"Type":"## BONUS\n\nChemistry - Short Answer","Prompt":"The compound sulfurous [sul-FYUR-us] acid has a first pKa of approximately 1.8 and a second pKa of approximately 7.2. To one decimal place, what is the pH of a solution of 0.1 molar sodium bisulfite?","Completion":"4.5"} {"Type":"TOSS-UP\n\nMath - Short Answer","Prompt":"3 is one-third percent of what number?","Completion":"900"} {"Type":"## BONUS\n\nMath - Short Answer","Prompt":"For real numbers \\(a\\) and \\(b\\), define the binary operation \"star\" by\n\n\"star\"\\(b\\) is equal to one less than the average of \\(a\\) and \\(b\\). What is 3 \"star\" open parenthesis 4 \"star\" 5 close parenthesis?","Completion":"9\/4 (ACCEPT: 2\/4, 2.25)"} {"Type":"TOSS-UP\n\nBiology - Short Answer","Prompt":"The hamstrings allow flexion of what joint?","Completion":"KNEE"} {"Type":"BONUS\n\nBiology - Short Answer","Prompt":"In humans, what group of muscles produce extension of the knee joint, such as during jumping?","Completion":"QUADRICEPS"} {"Type":"TOSS-UP\n\nEnergy - Multiple Choice","Prompt":"Wind, moving water, sunlight, and heat from Earth's interior are sources of what type of energy?\n\nW) Renewable\n\nX) Fossil\n\nY) Alternative\n\nZ) Reusable","Completion":"W) RENEWABLE"} {"Type":"TOSS-UP\n\nBiology - Multiple Choice","Prompt":"Which chamber in the heart has the highest systolic pressure?\n\nW) Right atrium\n\nX) Right ventricle\n\nY) Left atrium\n\nZ) Left ventricle","Completion":"Z) LEFT VENTRICLE"} {"Type":"TOSS-UP\n\nEarth and Space - Multiple Choice","Prompt":"What type of rock is found in most cave systems?\n\nW) Sandstone\n\nX) Granite\n\nY) Basalt\n\nZ) Limestone","Completion":"Z) LIMESTONE"} {"Type":"TOSS-UP\n\nEnergy - Multiple Choice","Prompt":"At the DOE's Advanced Photon Source, researchers are studying industrial catalysts used in petrochemical processing. Which of the following correctly describes most of these catalysts?\n\nW) Organic molecules\n\nX) Enzymes\n\nY) Homogeneous inorganic\n\nZ) Heterogeneous inorganic","Completion":"Z) HETEROGENEOUS INORGANIC"} {"Type":"TOSS-UP\n\nMath - Short Answer","Prompt":"A one-to-one function has its graph in the second quadrant. In which quadrant will the graph of its inverse lie?","Completion":"FOURTH (ACCEPT: 4)"} {"Type":"BONUS\n\nMath - Short Answer","Prompt":"If the line tangent to the graph of the differentiable function \\(f\\) of \\(x\\) at the point (-2, passes through the point (3.5, -, then what is value of the derivative of \\(f\\) of \\(x\\) when \\(x=-2\\)?","Completion":"-16\/11 (ACCEPT: -1 5\/11)"} {"Type":"TOSS-UP\n\nChemistry - Short Answer","Prompt":"What gas, whose methylated derivative is used in rocket fuels, is produced by bubbling ammonia through a sodium hypochlorite solution?","Completion":"HYDRAZINE (ACCEPT: N\\({}{2}\\)H\\({}{4}\\))"} {"Type":"BONUS\n\nChemistry - Short Answer","Prompt":"The isomerization reaction of a particular cis-alkene to its trans-isomer is first order. In terms of liters, moles, and seconds, what are the units for the rate constant of this reaction?","Completion":"INVERSE SECONDS (ACCEPT: SECONDS to THE NEGATIVE ONE, 1 OVER SECONDS)"} {"Type":"TOSS-UP\n\nBiology - Multiple Choice","Prompt":"Grasses provide an example of a root system that is best described as which of the following?\n\nW) Monocot tap\n\nX) Dicot tap\n\nY) Monocot fibrous\n\nZ) Dicot fibrous","Completion":"Y) MONOCOT FIBROUS"} {"Type":"TOSS-UP\n\nPhysics - Short Answer","Prompt":"On a standing wave, what term refers to the points at which the amplitude is maximized?","Completion":"ANTI-NODE (DO NOT ACCEPT: PEAK)"} {"Type":"TOSS-UP\n\nEnergy - Short Answer","Prompt":"While fluorescent lamps have a much higher efficiency than incandescent bulbs, they require special disposal concerns due to their construction involving what heavy metal vapor?","Completion":"MERCURY"} {"Type":"TOSS-UP\n\nBiology - Short Answer","Prompt":"What is the repeating unit found in skeletal muscles?","Completion":"SARCOMERE"} {"Type":"TOSS-UP\n\nPhysics - Multiple Choice","Prompt":"What thermodynamic cycle describes a jet turbine?\n\nW) Rankine\n\nX) Carnot\n\nY) Brayton\n\nZ) Otto","Completion":"Y) Brayton"} {"Type":"TOSS-UP\n\nMath - Short Answer","Prompt":"What is 4\/9 divided by 8\/15?","Completion":"5\/6"} {"Type":"TOSS-UP\n\nChemistry - Short Answer","Prompt":"Identify all of the following three compounds that can crystallize as network solids:\n\nCarbon dioxide; Silicon dioxide; Silicon carbide.","Completion":"2 AND 3"} {"Type":"TOSS-UP\n\nPhysics - Short Answer","Prompt":"What is the term for the quasiparticles present in p-type semiconductors that represent the absence of electrons?","Completion":"HOLES (ACCEPT: ELECTRON HOLES)"} {"Type":"TOSS-UP\n\nEarth and Space - Short Answer","Prompt":"What is the general term for the data-gathering technique by which the measurements of several smaller telescopes are superimposed to create a higher-resolution and more detailed image?","Completion":"INTERFEROMETRY"} {"Type":"TOSS-UP\n\nMath - Short Answer","Prompt":"Name one of the linear factors of \\(x^{2}+14x-72\\).","Completion":"\\(x-4\\) OR \\(x+18\\) (must give at least one)"} {"Type":"TOSS-UP\n\nBiology - Short Answer","Prompt":"In the sarcomere, what protein is found as filaments twisted into a double helix?","Completion":"ACTIN"} {"Type":"BONUS\n\nBiology - Short Answer","Prompt":"Identify all of the following three types of muscle that can contract strongly even when stretched : Cardiac; Skeletal; Smooth.","Completion":"3"} {"Type":"TOSS-UP\n\nEnergy - Multiple Choice","Prompt":"Which of the following forms of energy has the lowest average cost of operation and maintenance?\n\nW) Solar\n\nX) Wind\n\nY) Geothermal\n\nZ) Hydro","Completion":"W) SOLAR"} {"Type":"TOSS-UP\n\nChemistry - Multiple Choice","Prompt":"What element has the highest ionization energy?\n\nW) Hydrogen\n\nX) Helium\n\nY) Lithium\n\nZ) Beryllium","Completion":"X) HELIUM"} {"Type":"TOSS-UP\n\nEarth and Space - Multiple Choice","Prompt":"What is the term for a fluidized mixture of solid to semi-solid fragments and hot expanding gases that flows under gravity down the flanks of a volcano during an eruption?\n\nW) Pyroelastic surge\n\nX) Lahar\n\nY) Pyroelastic flow\n\nZ) Lava flow","Completion":"Y) PYROCLASTIC FLOW"} {"Type":"TOSS-UP\n\nMath - Multiple Choice","Prompt":"Two coplanar circles have a common chord. Which of the following best describes the quadrilateral that has as opposite vertices the centers of the two circles and the endpoints of the common chord?\n\nW) Kite\n\nX) Parallelogram\n\nY) Rhombus\n\nZ) Square","Completion":"W) KITE"} {"Type":"TOSS-UP\n\nEnergy - Multiple Choice","Prompt":"Immediately after being removed from a fission reactor, spent fuel rods will be placed where?\n\nW) Underground\n\nX) Underwater\n\nY) Reprocessing plants\n\nZ) MOX factories","Completion":"X) UNDERWATER"} {"Type":"BONUS\n\nEnergy - Multiple Choice","Prompt":"What are the primary mobile charge carriers in the sulfonated\n\nfluoropolymer [floor-oh-PAWL-ih-mur] ion-exchange membranes used in commercial fuel cells?\n\nW) Hydroxide ions\n\nX) Sodium cations [CAT-eye-onz]\n\nY) Water\n\nZ) Protons","Completion":"Z) PROTONS"} {"Type":"TOSS-UP\n\nMath - Short Answer","Prompt":"A prism with 7 faces has how many edges?","Completion":"15"} {"Type":"BONUS\n\nMath - Short Answer","Prompt":"Evaluate the summation, from \\(n=0\\) to infinity, of the fraction with numerator 12 and denominator \\(4^{n}\\).\n\nAnswer: 1613) Earth and Space - Multiple Choice Greenhouse gases build up in the atmosphere of Earth and limit the amount of heat escaping back into space. Which of the following is not a greenhouse gas for Earth?\n\nW) Carbon dioxide\n\nX) Argon\n\nY) Nitrous oxide\n\nZ) Methane","Completion":"X) ARGON"} {"Type":"TOSS-UP\n\nMath - Short Answer","Prompt":"What is the log base 3 of 243?","Completion":"5"} {"Type":"BONUS\n\nChemistry - Short Answer","Prompt":"What is the name for the wavelength at which a sample's absorbance does not change during a reaction?","Completion":"ISOSBestic POINT"} {"Type":"TOSS-UP\n\nEnergy - Short Answer","Prompt":"Moderators in nuclear reactors are used because of their ability to absorb energy from what particles?","Completion":"NEUTRONS (ACCEPT: FAST NEUTRONS)"} {"Type":"BONUS\n\nEnergy - Short Answer","Prompt":"DOE researchers recently published about the development of a complex oxide alloy that, rather than being primarily stabilized by chemical bonding, is stabilized by what state function?","Completion":"ENTROPY"} {"Type":"TOSS-UP\n\nPhysics - Multiple Choice","Prompt":"Capillary action refers to which of the following?\n\nW) The tendency for objects to flee the center of circular motion\n\nX) The change in frequency of a wave depending on the relative motion of the source\n\nY) The property by which liquids can flow against gravity\n\nZ) The tendency for fluids to dissipate energy as heat","Completion":"Y) THE PROPERT BY WHICH LIQUIDS CAN FLOW AGAINST GRAVITY"} {"Type":"TOSS-UP\n\nBiology - Short Answer","Prompt":"Calcium ions activate muscle contraction because they move what protein from thin filaments?","Completion":"TROPOMYOSIN"} {"Type":"TOSS-UP\n\nMath - Short Answer","Prompt":"What is the slope of the line with equation \\(3x-8y=-24\\)?","Completion":"3\/8 (ACCEPT: 0.375)"} {"Type":"TOSS-UP\n\nBiology - Short Answer","Prompt":"What two blood-sugar-relevant hormones are secreted by the islets [EYE-lets] of Langerhans [LAYN-gur-honz]?","Completion":"GLUCAGON AND INSULIN"} {"Type":"TOSS-UP\n\nChemistry - Short Answer","Prompt":"For the tri-iodide ion, what is the hybridization of the central iodine atom?","Completion":"DSP3 (ACCEPT: SP3D)"} {"Type":"BONUS\n\nChemistry - Short Answer","Prompt":"Identify all of the following three compounds with dsp3 hybridized central atoms:\n\nSilicon tetrafluoride [tetrah-FLOOR-ide]; Sulfur hexafluoride [hex-ah-FLOOR-ide]; Sulfur tetrafluoride.","Completion":"3"} {"Type":"TOSS-UP\n\nEarth and Space - Short Answer","Prompt":"What type of tide is caused when the gravitational forces of the Sun and Moon coincide?","Completion":"SPRING TIDES"} {"Type":"BONUS\n\nEarth and Space - Short Answer","Prompt":"Pahoehoe and a'a are lava flows associated with what type of magma?","Completion":"BASALTIC (ACCEPT: MAFIC)23) Physics - Short Answer A 1000-kilogram car traveling at 15 meters per second takes a turn on a flat surface with a radius of 50 meters. In kilo-newtons, what is the force of friction on the car?"} {"Type":"TOSS-UP\n\nBiology - Multiple Choice","Prompt":"In green plants, energy is stored mainly in which of the following forms?\n\nW) Cellulose\n\nX) Starch\n\nY) Glucose\n\nZ) ATP","Completion":"X) STARCH"} {"Type":"BONUS\n\nBiology - Short Answer","Prompt":"What sheet of muscle separates the thoracic and abdominopelvic cavities?","Completion":"DIAPHRAGM"} {"Type":"TOSS-UP\n\nEarth and Space - Multiple Choice","Prompt":"The earliest known single-celled life forms on Earth belong to which of the following taxonomic groups?\n\nW) Fungi\n\nX) Protoctista\n\nY) Plantae\n\nZ) Monera","Completion":"Z) MONERA"} {"Type":"BONUS\n\nEarth and Space - Short Answer","Prompt":"What is the chief commercial ore of aluminum?","Completion":"BAUXITE"} {"Type":"TOSS-UP\n\nMath - Multiple Choice","Prompt":"A differentiable function \\(f\\) has domain the closed interval from -3 to 4 and range the closed interval from 5 to 12. If \\(f\\) prime of \\(x\\) is greater than 0 for all \\(x\\) in the domain, what is \\(f\\) of -3?\n\nW) -3\n\nX) 4\n\nY) 5\n\nZ) 12","Completion":"Y) 5"} {"Type":"TOSS-UP\n\nEnergy - Short Answer","Prompt":"Methyl-tertiary-butyl-ether [methil-ter-shee-air-ee- byu-til-eethur], abbreviated MTBE, replaced lead in gasoline due to several environmental benefits. In order to raise octane rating in current gasoline, what has subsequently replaced MTBE?","Completion":"ETHANOL"} {"Type":"TOSS-UP\n\nChemistry - Multiple Choice","Prompt":"Which of the following is an example of an ionic compound?\n\nW) Methane\n\nX) Chlorine dioxide\n\nY) Dintitrogen trioxide\n\nZ) Calcium sulfide","Completion":"Z) CALCIUM SULFIDE"} {"Type":"TOSS-UP\n\nPhysics - Short Answer","Prompt":"A proton is traveling counterclockwise in a circle in the \\(x=0\\) plane. In what direction is its magnetic field vector?","Completion":"POSITIVE X"} {"Type":"TOSS-UP\n\nBiology - Multiple Choice","Prompt":"Which of the following types of tissue do monocot roots possess at their center that dicot roots do not?\n\nW) Xylem\n\nX) Pith\n\nY) Collenchyma [koh-LEN-kimma]\n\nZ) Trichomes","Completion":"X) PITH"} {"Type":"TOSS-UP\n\nPhysics - Short Answer","Prompt":"What type of circuits, which include types such as high-pass and low-pass filters, are composed of a resistor and a capacitor?","Completion":"RC CIRCUIT"} {"Type":"TOSS-UP\n\nEnergy - Multiple Choice","Prompt":"Which of the following weather conditions and times of year is best for achieving efficient energy conversion in a solar thermal plant?\n\nW) Partly cloudy day in fall\n\nX) Rainy day in spring\n\nY) Partly sunny day in winter\n\nZ) Overcast day in summer","Completion":"W) PARTLY CLOUDY DAY IN FALL"} {"Type":"TOSS-UP\n\nMath - Short Answer","Prompt":"What is the period of the function with equation \\(y=4\\) sine open parenthesis \\(7x+3\\) close parenthesis \\(+\\) 2?","Completion":"2\\(\\pi\\)\/7 (ACCEPT: (2\/7)\\(\\pi\\))"} {"Type":"TOSS-UP\n\nPhysics - Multiple Choice","Prompt":"A mass on a spring oscillates at a frequency of 200 hertz, creating a low-pitch sound. Which of the following can be done to increase the frequency of the sound produced?\n\nW) Increase the mass\n\nX) Use a stiffer spring\n\nY) Approach the mass-spring system at a high speed\n\nZ) Remove the air in the room","Completion":"X) USE A STIFFER SPRING"} {"Type":"BONUS\n\nPhysics - Short Answer","Prompt":"Identify all of the following three particles that are leptons:\n\nNeutrino; Gluon; Positron.","Completion":"1 AND 3"} {"Type":"TOSS-UP\n\nBiology - Short Answer","Prompt":"In collies, what neurotransmitter is released by somatic motor neurons to induce muscle contraction?","Completion":"ACETYLCHOLINE"} {"Type":"BONUS\n\nBiology - Multiple Choice","Prompt":"With which of the following is protein binding to a TATA [TAH-tah] box activity associated?\n\nW) DNA replication\n\nX) Transcription\n\nY) Translation\n\nZ) Elongation","Completion":"X) TRANSCRIPTION"} {"Type":"TOSS-UP\n\nMath - Short Answer","Prompt":"In what quadrants does the graph of the equation \\(y=4x-3\\) lie?","Completion":"1, 3, AND 4 (ACCEPT: FIRST, THIRD, AND FOURTH or ALL BUT SECOND)"} {"Type":"TOSS-UP\n\nEnergy - Short Answer","Prompt":"DOE scientists recently published a paper in which nanoscale circuitry [sur-cah-tree] was developed by printing a carbon-based semiconductor on a germanium [jer-MAYN-ee-un] surface. What allotrope of carbon are the semiconductors based on?","Completion":"GRAPHENE"} {"Type":"TOSS-UP\n\nChemistry - Multiple Choice","Prompt":"Consider the reaction of two moles of hydrogen gas and one mole of oxygen gas to generate two moles of steam. Which of the following statements is TRUE?\n\nW) Delta H is greater than delta U\n\nX) Delta H is less than delta U\n\nY) Delta H is equal to delta U\n\nZ) Delta H is greater than delta U at high temperatures, but less at low temperatures","Completion":"X) DELTA H IS LESS THAN DELTA U"} {"Type":"BONUS\n\nChemistry - Short Answer","Prompt":"Identify all of the following three diatomic [dye-ah-TOM-ik] molecules that MO theory predicts to be stable: Li\\({}{2}\\); Be\\({}{2}\\); B\\({}{2}\\).","Completion":"1 AND 3"} {"Type":"TOSS-UP\n\nEarth and Space - Multiple Choice","Prompt":"We see the same side of the moon here on Earth all the time due to what effect?\n\nW) Lagrange point\n\nX) Gravitational lensing\n\nY) Tidal locking\n\nZ) Asynchronous spin","Completion":"Y) TIDAL LOCKING"} {"Type":"BONUS\n\nEarth and Space - Multiple Choice","Prompt":"What layer of the Earth's atmosphere varies in depth with the time of day and reflects radio waves long distances?\n\nW) Photosphere\n\nX) Stratosphere\n\nY) Troposphere\n\nZ) Ionosphere","Completion":"Z) IONOSPHERE19) Chemistry - Short Answer The hormone estrogen contains five stereocenters. How many stereoisomers of estrogen are possible?"} {"Type":"TOSS-UP\n\nMath - Short Answer","Prompt":"What is \\(6^{2}\\) - \\(2^{6}\\)?","Completion":"-28"} {"Type":"BONUS\n\nMath - Short Answer","Prompt":"Solve the following equation for x over the negative integers: \\(x^{3}+5x^{2}-4x-20=0\\)","Completion":"-2 AND -5 (must give both answers)"} {"Type":"TOSS-UP\n\nBiology - Short Answer","Prompt":"Rickets is caused by a deficiency of what vitamin?","Completion":"VITAMIIN D"} {"Type":"BONUS\n\nBiology - Short Answer","Prompt":"Identify all of the following three molecules that are exocrine [EX-oh-krin] products: EGF; Amylase; ADH.","Completion":"223) Physics - Short Answer An engine does 15 joules of work while exhausting 35 joules of waste heat. What is the percent efficiency of the engine?"} {"Type":"TOSS-UP\n\nBiology - Short Answer","Prompt":"What leukocyte [LOO-koh-site] can have up to a five-lobed nucleus and plays a role in phagocytosis [flag-oh-sye-TOW-sis] of bacteria?","Completion":"NEUTROPHIL"} {"Type":"BONUS\n\nBiology - Short Answer","Prompt":"What type of immune cells use performs [PUR-fur-ins] to induce cell death in infected body cells?","Completion":"NK CELLS (ACCEPT: NATURAL KILLER CELLS, CYTOTOXIC T CELLS, CTLs)"} {"Type":"TOSS-UP\n\nEnergy - Short Answer","Prompt":"Biogas is a renewable energy source that is typically produced as landfill gas, caused by the breakdown of biodegradable waste via anaerobic microbes. What two gases are the primary constituents of biogas?","Completion":"METHANE AND CARBON DIOXIDE"} {"Type":"BONUS\n\nEnergy - Short Answer","Prompt":"Based on average operating conditions, rank the following household appliances in order of increasing electricity usage per unit time: LCD TV; Toaster oven; Hair dryer; Curling iron.","Completion":"1, 4, 2, 3 (LCD TV, CURLING IRON, TOASTER OVEN, HAIR DRYER)"} {"Type":"TOSS-UP\n\nMath - Short Answer","Prompt":"If \\(f\\) is a continuous function such that the definite integral from 3 to 7 of \\(f\\) of \\(x\\) d\\(x\\) equals 14, what is the average value of \\(f\\) on the closed interval from 3 to 7?","Completion":"7\/2 (ACCEPT: 3%, 3.5)"} {"Type":"TOSS-UP\n\nEarth and Space - Short Answer","Prompt":"What term, combining the Greek words for blanket and rock, describes the loose, incoherent material of any origin on the surface of a planet or satellite?","Completion":"REGOLITH"} {"Type":"TOSS-UP\n\nPhysics - Short Answer","Prompt":"What is the largest resonant wavelength, in meters, for sound waves in a 1-meter tube that is closed on one end?","Completion":"4"} {"Type":"TOSS-UP\n\nChemistry - Multiple Choice","Prompt":"Which of the following is the correct definition of molality?\n\nW) Kilograms of solute per liter of solvent\n\nX) Moles of solute per liter of solution\n\nY) Moles of solute per liter of solvent\n\nZ) Moles of solute per kilogram of solvent","Completion":"Z) MOLES OF SOLUTE PER KILOGRAM OF SOLVENT"} {"Type":"TOSS-UP\n\nPhysics - Multiple Choice","Prompt":"Which of the following correctly describes a magnifying glass?\n\nW) Single convex lens\n\nX) Two convex lenses\n\nY) Single concave lens\n\nZ) Two concave lenses","Completion":"W) SINGLE CONVEX LENS"} {"Type":"TOSS-UP\n\nMath - Short Answer","Prompt":"A triangle has two sides of lengths 38 and 54. What is the largest possible integer length of the third side?","Completion":"91"} {"Type":"TOSS-UP\n\nBiology - Multiple Choice","Prompt":"Sutures are an example of what type of joint?\n\nW) Synovial [sin-OH-vee-ul]\n\nX) Saddle\n\nY) Disk\n\nZ) Immovable","Completion":"Z) IMMOVABLE"} {"Type":"TOSS-UP\n\nChemistry - Multiple Choice","Prompt":"A student obtained a low value when he attempted to measure the number of waters of hydration in a sample of calcium sulfate. Which of the following mistakes could lead to the erroneous result?\n\nW) The sample was not heated enough\n\nX) The sample was heated too much\n\nY) A balance that gave uniformly high readings was used\n\nZ) A balance that gave uniformly low readings was used","Completion":"W) THE SAMPLE WAS NOT HEATED ENOUGH"} {"Type":"TOSS-UP\n\nPhysics - Multiple Choice","Prompt":"Which of the following are proper units for entropy [EN-trou-peer]?\n\nW) Joules [jools]\n\nX) Kelvins per joule\n\nY) Joules per kelvin\n\nZ) Joule-kelvins","Completion":"Y) JOULES PER KELVIN"} {"Type":"TOSS-UP\n\nChemistry - Multiple Choice","Prompt":"Which of the following electrons would be most effective at shielding the nuclear charge seen by a 3s electron in an aluminum atom?\n\nW) 1s\n\nX) 2s\n\nY) 2p\n\nZ) 3s","Completion":"W) 1s"} {"Type":"TOSS-UP\n\nBiology - Multiple Choice","Prompt":"Which of the following organelles would likely contain a high concentration of catalase [CAT-ah-lace]?\n\nW) Glycosome [glye-OX-ih-sowm]\n\nX) Mitochondndrion [my-tow-KON-dree-on]\n\nY) Peroxisome [per-OX-ih-zohm]\n\nZ) Chloroplast [KLOR-oh-plast]","Completion":"Y) PEROXISOME"} {"Type":"TOSS-UP\n\nEarth and Space - Short Answer","Prompt":"List the following four minerals in order of increasing metamorphic grade:\n\nStarolite; Muscovite; Garnet; Chlorite.","Completion":"4, 2, 3, 1 (ACCEPT: CHLORITE, MUSCOVITE, GARNET, STAUROLITE)"} {"Type":"TOSS-UP\n\nMath - Short Answer","Prompt":"What is the -4\/3 power of -27?","Completion":"1\/81"} {"Type":"TOSS-UP\n\nEnergy - Short Answer","Prompt":"What treaty, signed by the United States, banned the usage of CFCs?","Completion":"MONTREAL TREATY (ACCEPT: MONTREAL PROTOCOL)"} {"Type":"TOSS-UP\n\nPhysics - Short Answer","Prompt":"What is the mechanical advantage of a 3.5-foot lever if the fulcrum is located 6 inches from one end?","Completion":"SIX"} {"Type":"TOSS-UP\n\nEarth and Space - Short Answer","Prompt":"What type of supernova can leave behind a black hole?","Completion":"TYPE II"} {"Type":"TOSS-UP\n\nMath - Short Answer","Prompt":"A pyramid with 14 edges has how many vertices?","Completion":"8"} {"Type":"TOSS-UP\n\nMath - Short Answer","Prompt":"If the line tangent to the graph of the function f(x) at the point (1, passes through the point (-1.5, -, then what is value of the derivative of f(x) when \\(x\\) = 1?","Completion":"2"} {"Type":"TOSS-UP\n\nEnergy - Multiple Choice","Prompt":"If the frequency of a wave is decreased, which of the following changes must occur for the wave to maintain a constant velocity?\n\nW) The period must decrease\n\nX) The wavelength must increase\n\nY) The amplitude must increase\n\nZ) The amplitude must decrease","Completion":"X) THE WAVELENGTH MUST INCREASE"} {"Type":"BONUS\n\nEarth and Space - Short Answer","Prompt":"What greenhouse gas has the greatest abundance in the atmosphere?","Completion":"WATER VAPOR"} {"Type":"TOSS-UP\n\nBiology - Short Answer","Prompt":"When a phylogenetic tree has the fewest number of evolutionary events possible, it is said to embody what principle of simplicity?","Completion":"PARSIMONY"} {"Type":"BONUS\n\nBiology - Short Answer","Prompt":"What phenomenon involves the injection of double-stranded RNAs turning off the expression of genes with the same sequence as the RNA?","Completion":"RNA INTERFERENCE"} {"Type":"TOSS-UP\n\nEnergy - Short Answer","Prompt":"What is the SI unit commonly used to measure electrical resistance?","Completion":"OHM"} {"Type":"TOSS-UP\n\nMath - Short Answer","Prompt":"If one side of a right triangle is 14 inches and its opposite angle is 30 degrees, what is the diameter of the circle circumscribed around this triangle?","Completion":"28"} {"Type":"TOSS-UP\n\nChemistry - Multiple Choice","Prompt":"Ceramic materials typically belong to what class of solid structure?\n\nW) Network solid\n\nX) Ionic solid\n\nY) Molecular solid\n\nZ) Metallic solid","Completion":"W) NETWORK SOLID"} {"Type":"TOSS-UP\n\nPhysics - Multiple Choice","Prompt":"What is the name of the principle that states that an object, partly or completely, submerged in a fluid will experience a buoyant force equal to the weight of the fluid the object displaces?\n\nW) Newton\n\nX) Bernoulli\n\nY) Pascal\n\nZ) Archimedes","Completion":"Z) ARCHIMEDES"} {"Type":"TOSS-UP\n\nChemistry - Multiple Choice","Prompt":"Which of the following elemental substances is not at standard state?\n\nW) Diamond at 25 degrees Celsius and one atmosphere\n\nX) Rhombic sulfur at 25 degrees Celsius and one atmosphere\n\nY) Gaseous dioxygen at 25 degrees Celsius and one atmosphere\n\nZ) Gaseous neon at 25 degrees Celsius and one atmosphere","Completion":"W) DIAMOND AT 25 DEGrees CELSIUS AND ONE ATMOSPHERE"} {"Type":"TOSS-UP\n\nBiology - Short Answer","Prompt":"What neurotransmitter is released by motor neurons at the neuromuscular junction?","Completion":"ACETYLCHOLINE [uh-see-tul-koe-leen]"} {"Type":"TOSS-UP\n\nEarth and Space - Short Answer","Prompt":"What prefix describes clouds found in the middle altitude range?","Completion":"ALTO"} {"Type":"BONUS\n\nEarth and Space - Short Answer","Prompt":"What gravitationally bound systems of matter, classified by the Hubble sequence, are the basic units of the Universe?","Completion":"GALAXIES"} {"Type":"TOSS-UP\n\nBiology - Short Answer","Prompt":"What immunoglobulin class is the largest antibody present in the human circulatory system?","Completion":"IgM"} {"Type":"BONUS\n\nBiology - Multiple Choice","Prompt":"A scientist isolates a small amount of DNA from a Tyrannosaur fossil. Which of the following would be most useful for increasing the amount of DNA with which the scientist has to work?\n\nW) RFLP analysis\n\nX) Polymerase chain reaction\n\nY) Southern blotting\n\nZ) SDS-PAGE","Completion":"X) POLYMERASE CHAIN REACTION"} {"Type":"TOSS-UP\n\nMath - Short Answer","Prompt":"If f(x) equals the cube root of open parenthesis \\(x+8\\) close parenthesis, what is \\(f\\) inverse of 6?","Completion":"208"} {"Type":"BONUS\n\nMath - Short Answer","Prompt":"What is the sum of all the distinct prime factors of 15 factorial?","Completion":"41"} {"Type":"TOSS-UP\n\nChemistry - Short Answer","Prompt":"What laboratory instrument, containing gradations that increase from top to bottom, is used to deliver precise quantities of titrant to the analyte in a titration?","Completion":"BURETTE (ACCEPT: BURET)"} {"Type":"BONUS\n\nChemistry - Short Answer","Prompt":"If you have 50 milliliters of a 0.75 molar solution of NaCl, how much water, in milliliters, should you add to dilute the NaCl concentration to 0.25 molar?","Completion":"10023) Physics - Short Answer A 100 milliwatt laser delivers \\(2.50\\times 10^{17}\\) photons per second. Expressing your answer in scientific notation with one significant figure, how much energy in joules does each photon deliver?"} {"Type":"TOSS-UP\n\nChemistry - Short Answer","Prompt":"What is the hybridization of the central iodine atom in iodine pentafluoride?","Completion":"sp\\({}^{3}\\)d\\({}^{2}\\) [s-p-3-d-2]"} {"Type":"BONUS\n\nChemistry - Short Answer","Prompt":"What class of organic molecules has the formula R-COOH and can react with alcohols to produce esters?","Completion":"CARBOXYLIC ACID"} {"Type":"TOSS-UP\n\nEarth and Space - Multiple Choice","Prompt":"Which of the following planets is mostly composed of hydrogen, helium, and methane?\n\nW) Uranus\n\nX) Mercury\n\nY) Mars\n\nZ) Venus","Completion":"W) URANUS"} {"Type":"BONUS\n\nEarth and Space - Multiple Choice","Prompt":"Which of the following provides the most accurate description of carrying capacity?\n\nW) Population below the carrying capacity will increase primary succession\n\nX) Population above the carrying capacity cannot be sustained indefinitely\n\nY) Population above the carrying capacity will always run out of space\n\nZ) Population above the carrying capacity will always result in excessive CO2 levels","Completion":"X) POPULATION ABOVE THE CARRYING CAPACITY CANNOT BE SUSTAINED"} {"Type":"TOSS-UP\n\nEnergy - Multiple Choice","Prompt":"What is the primary advantage of using cellulosic feedstocks over crops like corn to produce ethanol?\n\nW) The enzymes that produce cellulosic ethanol are cheap and plentiful\n\nX) There are more cellulosic ethanol plants than corn ethanol plants\n\nY) Waste products can be used as cellulosic feedstocks\n\nZ) Cellulosic ethanol production does not require government subsidies to be economically competitive","Completion":"Y) WASTE PRODUCTS CAN BE USED AS CELLULulosic FEEDSTOCKS"} {"Type":"BONUS\n\nEnergy - Multiple Choice","Prompt":"Which of the following countries generates the highest percentage of its electricity from nuclear power?\n\nW) Japan\n\nX) France\n\nY) United States\n\nZ) Russia","Completion":"X) FRANCE"} {"Type":"TOSS-UP\n\nPhysics - Short Answer","Prompt":"A certain planet has four times the mass of Earth, but the acceleration due to gravity on the planet's surface is the same as that of Earth. What is the ratio of the planet's radius to the Earth's radius?","Completion":"2 (ACCEPT: 2:1)"} {"Type":"BONUS\n\nPhysics - Short Answer","Prompt":"A ball of mass M and speed V collides with a ball of mass 2M moving toward the first ball with a speed of \\( icefrac{{1}}{{2}}\\) V. If the two balls stick together, what will their speed after the collision be?","Completion":"0"} {"Type":"TOSS-UP\n\nBiology - Short Answer","Prompt":"Recombinant DNA can be introduced to eukaryotic cells by briefly shocking them. What is this technique called?","Completion":"ELECTROPORATION (DO NOT ACCEPT: TRANSFECTION, TRANSFORMATION)"} {"Type":"BONUS\n\nBiology - Multiple Choice","Prompt":"Which of the following statements is true regarding fungal mutualism?\n\nW) Most endophytes are basidiomycetes [buh-sid-ee-oh-mye-seets]\n\nX) Many species of ants culture fungi inside of their bodies\n\nY) Lichens are capable of nitrogen fixation\n\nZ) Plants can generally grow normally without endophytes","Completion":"Y) LICHENS ARE CAPABLE OF NITROGEN FIXATION"} {"Type":"TOSS-UP\n\nMath - Short Answer","Prompt":"For a normal distribution with mean of 94 and standard deviation of 4, for a score of 88, what is the corresponding Z-score?","Completion":"-1.5 (ACCEPT -3\/2 or -1%)"} {"Type":"BONUS\n\nMath - Multiple Choice","Prompt":"For the arithmetic series with first three terms 3, 5, and 7, what is the sum of the first 10 terms of the series?\n\nW) 100\n\nX) 120\n\nY) 140\n\nZ) 160","Completion":"X) 120"} {"Type":"TOSS-UP\n\nChemistry - Multiple Choice","Prompt":"A catalyst speeds up a chemical reaction by:\n\nW) Shifting the equilibrium\n\nX) Increasing the activation energy\n\nY) Decreasing the reaction enthalpy\n\nZ) Providing an alternate reaction pathway","Completion":"Z) PROVIDING AN ALTERNATE REACTION PATHWAY"} {"Type":"TOSS-UP\n\nEnergy - Multiple Choice","Prompt":"Two resistors, R1 and R2, are identical, but the potential difference across R1 is half the potential difference across R2. What is the ratio of the current in R1 to the current in R2?\n\nW) 1\/2\n\nX) 1\/4\n\nY) 2\n\nZ) 4","Completion":"W) 1\/2"} {"Type":"TOSS-UP\n\nEarth and Space - Short Answer","Prompt":"What types of streams, usually found in deserts, only carry water in response to specific episodes of rainfall?","Completion":"EPHEMERAL STREAMS"} {"Type":"BONUS\n\nEarth and Space - Short Answer","Prompt":"Identify all of the following three geological features that are formed by the wind: Yardangs; Roche moutonnee [moot-an-ay]; Blowouts.","Completion":"1 AND 3"} {"Type":"TOSS-UP\n\nPhysics - Short Answer","Prompt":"A neutron, an electron, a proton, and an alpha particle are all traveling at the same speed. Which of these particles has the shortest de Broglie wavelength?","Completion":"ALPHA PARTICLE"} {"Type":"BONUS\n\nPhysics - Short Answer","Prompt":"Identify all of the following 3 statements that are true of equipotential surfaces: Field lines and equipotential surfaces are always mutually perpendicular; When all charges are at rest, the surface of a conductor is always an equipotential surface; The work done to move a charge from any point on an equipotential surface to any other point on the equipotential surface is zero.","Completion":"1, 2, 3"} {"Type":"TOSS-UP\n\nBiology - Multiple Choice","Prompt":"There are three checkpoints during normal cell division. In what stages in the life of a cell are these checkpoints found?\n\nW) Growth phase I, Growth Phase II, and mitosis\n\nX) Growth phase I, S phase, and mitosis\n\nY) S phase, telophase, and anaphase\n\nZ) S phase, growth phase II, and mitosis","Completion":"W) GROWTH PHASE I, GROWTH PHASE II, AND MITOSIS"} {"Type":"BONUS\n\nBiology - Multiple Choice","Prompt":"Which of the following is not true regarding arthropods?\n\nW) Arthropods have an open circulatory system\n\nX) Molting occurs because arthropods cannot grow without shedding their exoskeleton\n\nY) Book lungs are found in arachnids\n\nZ) Cheliceriformes [kuh-liss-er-uh-forms] are characterized by 5 pairs of legs and two pairs of antennae","Completion":"Z) CHELICERiformes ARE CHARACTERIZED BY 5 PAIRS OF LEGS AND TWO PAIRS OF ANTENNAE"} {"Type":"TOSS-UP\n\nMath - Multiple Choice","Prompt":"Divide a unit segment into 3 congruent segments and remove the middle\n\nsegment. Do this process a second time on each of the remaining two segments. If this process is carried out a total of n times, what is the sum of the lengths of the remaining segments?\n\nW) One-third to the nth power\n\nX) One minus one-third to the nth power\n\nY) Two-thirds to the nth power\n\nZ) One minus two-thirds to the nth power","Completion":"Y) TWO-THIRDS TO THE nth POWER"} {"Type":"BONUS\n\nMath - Short Answer","Prompt":"If a\\({}{1}=4\\) and a\\({}{n}=2\\)a\\({}{n-1}+5\\), what is the value of a\\({}{4}\\)?","Completion":"6713) Chemistry - Short Answer For an electron in a 3d subshell, what is the smallest value the magnetic quantum number can take?"} {"Type":"BONUS\n\nEarth and Space - Multiple Choice","Prompt":"Areas of low pressure exist over the equatorial regions due to which of the following?\n\nW) Temperature\n\nX) Salinity\n\nY) Coriolis forces\n\nZ) Circulation","Completion":"W) TEMPERATURE"} {"Type":"TOSS-UP\n\nPhysics - Short Answer","Prompt":"Identify all of the following three types of collisions that conserve kinetic\n\nenergy: Elastic collision; Partial inelastic collision; Completely inelastic collision.","Completion":"1"} {"Type":"BONUS\n\nPhysics - Short Answer","Prompt":"A 2.0 meter long, 100 kilogram horizontal uniform steel beam is supported 0.6\n\nmeters from the right end. To the nearest whole newton-meter, what is the gravitational torque about the\n\nsupport?","Completion":"392"} {"Type":"TOSS-UP\n\nBiology - Multiple Choice","Prompt":"Which of the following cleavage patterns is characteristic of deuterostomes [doo-tuhr-oh-stohms]?\n\nW) Spiral cleavage\n\nX) Determinate cleavage\n\nY) Bilateral cleavage\n\nZ) Radial cleavage","Completion":"Z) RADIAL CLEAVAGE"} {"Type":"BONUS\n\nBiology - Multiple Choice","Prompt":"Which of the following enzymes would be required for the formation of cDNA, a genetic transcript from eukaryotic cells that has had the introns removed?\n\nW) Excinuclease [ex-ih-noo-clee-ayse]\n\nX) AP Endonuclease\n\nY) Reverse transcriptase\n\nZ) DNase I","Completion":"Y) REVERSE TRANSCRIPTASE"} {"Type":"TOSS-UP\n\nMath - Multiple Choice","Prompt":"Which of the following best describes the following system of two linear equations: \\(y-2x=\\) - 2, \\(y+x=\\) 4?\n\nW) Consistent, Dependent\n\nX) Inconsistent, Independent\n\nY) Consistent, Independent\n\nZ) Inconsistent, Dependent","Completion":"Y) CONSISTENT, INDEPENDENT"} {"Type":"BONUS\n\nMath - Multiple Choice","Prompt":"Telesha and Chrissy each have a bag that contains one ball of each of the colors blue, green, red, purple, and pink. If Telesha takes one ball at random out of her bag and places it in Chrissy's bag and then Chrissy takes one ball at random out of her bag and places it in Telesha's bag, what is the probability that the two bags have the same contents?\n\nW) 1\/10\n\nX) 1\/6\n\nY) 1\/5\n\nZ) 1\/3","Completion":"Z) 1\/3"} {"Type":"TOSS-UP\n\nChemistry - Multiple Choice","Prompt":"Macrophages use NADPH oxidase to generate what chemical, which is then used to destroy invading bacteria?\n\nW) Hypochlorite ions\n\nX) Superoxide ions\n\nY) Hydroxide ions\n\nZ) Chloric acid","Completion":"X) SUPEROXIDE IONS"} {"Type":"BONUS\n\nChemistry - Short Answer","Prompt":"Assume that heptane boils at a temperature of 97 degrees Celsius and has a molar enthalpy of vaporization of 37 kilojoules per mole. What is the delta G, to two significant figures and in joules, for the vaporization of one mole of heptane at 100 degrees Celsius?","Completion":"-300 (DO NOT ACCEPT: 300)"} {"Type":"TOSS-UP\n\nEarth and Space - Multiple Choice","Prompt":"Which of the following minerals is the most economically important ore for mining and extraction of the element mercury?\n\nW) Calcite\n\nX) Cinnabar\n\nY) Chabazite\n\nZ) Cristobalite","Completion":"X) CINNABAR"} {"Type":"BONUS\n\nEarth and Space - Multiple Choice","Prompt":"How fast, in miles per hour, can a tsunami travel over the open ocean?\n\nW) 20\n\nX) 100\n\nY) 500\n\nZ) 1000","Completion":"Y) 500"} {"Type":"TOSS-UP\n\nPhysics - Multiple Choice","Prompt":"The gravitational potential energy of an object is often approximated as mgh, for which \\(m\\) is its mass, \\(h\\) is the height above the Earth's surface, and g is the acceleration due to gravity. What assumption is required in order to approximate \\(g\\) as a constant?\n\nW) Wind resistance is negligible\n\nX) The object has no rotational inertia\n\nY) \\(m\\) is very small compared to the mass of the Earth\n\nZ) \\(h\\) is very small compared to the radius of the Earth","Completion":"Z) \\(h\\) is very small compared to the radius of the Earth"} {"Type":"BONUS\n\nPhysics - Short Answer","Prompt":"How many total degrees of freedom does a diatomic gas have?","Completion":"SIX"} {"Type":"TOSS-UP\n\nBiology - Short Answer","Prompt":"Identify all of the following three choices that represent correct reasons why the eradication of polio is possible: Humans are the only reservoir of polio; Transmission of the polio virus is very difficult; Vaccination generates herd immunity.","Completion":"1 AND 3"} {"Type":"BONUS\n\nBiology - Short Answer","Prompt":"Identify all of the following three species that are part of the phylum Mollusca: Blue-ringed octopus; Giant clam; Oyster.","Completion":"1, 2, 3"} {"Type":"TOSS-UP\n\nMath - Short Answer","Prompt":"Evaluate the following definite integral: the integral from 0 to \\(\\pi\\)\/2 of sinx cosx dx.","Completion":"\\(\\%\\) (ACCEPT: 0.5)"} {"Type":"BONUS\n\nMath - Short Answer","Prompt":"The sums of three whole numbers taken in pairs are 11, 13, and 16. What is the product of these three numbers?","Completion":"252"} {"Type":"TOSS-UP\n\nEarth and Space - Multiple Choice","Prompt":"A line with alternating blue triangles and red semicircles on a weather map indicates:\n\nW) A warm front\n\nX) A cold front\n\nY) A stationary front\n\nZ) An occluded front","Completion":"Y) A stationary front"} {"Type":"BONUS\n\nEarth and Space - Multiple Choice","Prompt":"Which of the following is closest to the average pH of marine water?\n\nW) 4.18\n\nX) 6.19\n\nY) 7.56\n\nZ) 8.16","Completion":"Z) 8.16"} {"Type":"TOSS-UP\n\nBiology - Multiple Choice","Prompt":"Which of the following types of epithelial tissue would most likely be found lining the lungs and blood vessels?\n\nW) Stratified columnar\n\nX) Simple squamous\n\nY) Simple cuboidal\n\nZ) Simple columnar","Completion":"X) SIMPLE SQUAMOUS"} {"Type":"BONUS\n\nBiology - Multiple Choice","Prompt":"Which of the following could occur due to a mutation in a Hox gene in a human embryo?\n\nW) Polydactyly [poly-dak-til-ee]\n\nX) Sickle cell anemia\n\nY) Phenylkotunria [free-nil-kee-toe-noo-ree-uh]\n\nZ) Chronic myelogenous [my-uhl-aw-jen-us] leukemia","Completion":"W) POLYDACTYLY"} {"Type":"TOSS-UP\n\nMath - Short Answer","Prompt":"What is the limit as x approaches zero of the fraction with numerator x + 1 and denominator x?","Completion":"DOES NOT EXIST (ACCEPT: DNE, NON-EXistant; DO NOT ACCEPT: INFINITY, MINUS INFINITY, PLUS or MINUS INFINITY)"} {"Type":"BONUS\n\nMath - Short Answer","Prompt":"Two integers between 1 and 10, inclusive, are selected at random, noting that the two numbers may be the same. What is the probability that both numbers selected are prime, and their product is odd?","Completion":"9\/100 (ACCEPT: 0.09 OR 9%)"} {"Type":"TOSS-UP\n\nPhysics - Multiple Choice","Prompt":"In a graph of stopping potential versus frequency, what is the slope of the straight line obtained?\n\nW) Work function over Planck's constant\n\nX) Planck's constant over work function\n\nY) Elementary charge over Planck's constant\n\nZ) Planck's constant over elementary charge","Completion":"Z) PLANCK'S CONSTANT OVER ELEMENTARY CHARGE"} {"Type":"BONUS\n\nPhysics - Short Answer","Prompt":"Identify all of the following three statements that are true regarding electric motors: Back EMF in a motor arises because of Lenz's law; Commutators periodically reverse the direction of current in an electric motor; In an electromechanical machine, the armature generates the EMF.","Completion":"1, 2, AND 3"} {"Type":"TOSS-UP\n\nChemistry - Multiple Choice","Prompt":"A supersaturated solution of sodium nitrate was prepared at 80 degrees Celsius but cooled to room temperature. Which of the following statements describes the addition of a seed crystal?\n\nW) Delta H is negative and Delta S is negative\n\nX) Delta H is positive and Delta S is negative\n\nY) Delta H is negative and Delta S is positive\n\nZ) Delta H is positive and Delta S is positive","Completion":"W) DELTA H IS NEGATIVE AND DELTA S IS NEGATIVE"} {"Type":"BONUS\n\nChemistry - Short Answer","Prompt":"Rank the following three 0.1 molar solutions from the least basic to most basic: Urea; Ammonia; Barium hydroxide.","Completion":"1, 2, 3"} {"Type":"TOSS-UP\n\nMath - Short Answer","Prompt":"How many sides does a regular polygon have if each interior angle equals 172 degrees?","Completion":"45"} {"Type":"BONUS\n\nMath - Short Answer","Prompt":"Simplify the fraction with numerator 3 raised to the \\(2k+1\\) power and denominator 243 raised to the negative \\(k\\) power.","Completion":"3 raised to the \\((7k+1)\\) power7) Earth and Space - Short Answer What is the term for the vertical motions that occur when the atmosphere is unstable?"} {"Type":"BONUS\n\nEarth and Space - Multiple Choice","Prompt":"An isobar on a weather map is a line that connects equal points of:\n\nW) Wind speed\n\nX) Pressure\n\nY) Temperature\n\nZ) Relative humidity","Completion":"X) PRESSURE"} {"Type":"TOSS-UP\n\nBiology - Short Answer","Prompt":"What pentameric antibody is generally the first to be secreted during primary immune response and is responsible for promoting agglutination reactions?","Completion":"IgM (ACCEPT: IMMUNOGLOBLIN M)"} {"Type":"BONUS\n\nBiology - Multiple Choice","Prompt":"Which of the following roles does creatine play in the human body?\n\nW) Intermediate in protein synthesis\n\nX) Coenzyme associated with purine breakdown\n\nY) Energy source in muscles\n\nZ) Enzyme in the urea cycle","Completion":"Y) ENERGY SOURCE IN MUSCLES"} {"Type":"TOSS-UP\n\nEnergy - Multiple Choice","Prompt":"Which of the following thermal reactors was found in the Three Mile Island reactor complex?\n\nW) Breeder reactor\n\nX) Fast neutron reactor\n\nY) Pressurized water reactor\n\nZ) Liquid fluoride thorium reactor","Completion":"Y) PRESSURIZED WATER REACTOR"} {"Type":"BONUS\n\nEnergy - Multiple Choice","Prompt":"Which of the following is not true of the Advanced Light Source at Lawrence Berkeley National Lab?\n\nW) It uses accelerated protons to produce light\n\nX) It primarily produces x-rays\n\nY) It is a cyclotron\n\nZ) Photons from it can be used to solve protein crystal structures","Completion":"W) IT USES ACCELERATED PROTONS TO PRODUCE LIGHT"} {"Type":"TOSS-UP\n\nPhysics - Short Answer","Prompt":"What type of scattering generally involves the elastic scattering of\n\nelectromagnetic radiation by a free charged particle and generally occurs as long as the incident photon\n\nenergy is much less than the mass energy of the particle?","Completion":"THOMSON SCATTERING"} {"Type":"BONUS\n\nPHYSICS - Multiple Choice","Prompt":"A spherical grain of sand falls into a pool. As the grain falls through the water, its terminal velocity is independent of which of the following?\n\nW) The temperature of the water\n\nX) The radius of the grain\n\nY) The density of the grain\n\nZ) The height above the pool from which it fell","Completion":"Z) The height above the pool from which it fell"} {"Type":"TOSS-UP\n\nChemistry - Multiple Choice","Prompt":"Which of the following molecules is NOT matched correctly to its central atom hybridization?\n\nW) Xenon tetrafluoride and d\\({}{2}\\)sp\\({}{3}\\)\n\nX) Phosphorous pentachloride and d\\({}{2}\\)sp\\({}{3}\\)\n\nY) Krypton difluoride and dsp\\({}{3}\\)\n\nZ) Bromine trifluoride and dsp\\({}{3}\\)","Completion":"X) PHOSPHOROUS PENTACHLORIDE AND D\\({}{2}\\)SP\\({}{3}\\)"} {"Type":"BONUS\n\nChemistry - Short Answer","Prompt":"Identify all of the following three statements that characterize SN1\n\nreactions: They have single-step mechanisms; Their rate is strongly dependent on nucleophile strength;\n\nThey are faster in polar, protic solvents like water or ethanol.","Completion":"3"} {"Type":"TOSS-UP\n\nPhysics - Multiple Choice","Prompt":"In which of the following ranges of the electromagnetic spectrum are the Paschen series hydrogen lines found?\n\nW) Ultraviolet\n\nX) Visible\n\nY) Infrared\n\nZ) X-Ray","Completion":"Y) INFRARED"} {"Type":"BONUS\n\nPhysics - Multiple Choice","Prompt":"Which of the following is NOT true regarding electric fields?\n\nW) The strength of an electric field associated with a point charge varies with the inverse square of distance.\n\nX) The strength of an electric field associated with a dipole varies with the inverse cube of distance\n\nY) If a point charge is placed inside a hollow, uncharged, conducting shell, there will be no net electric field outside of the shell\n\nZ) The electric field inside of a hollow, uncharged conductor is always zero","Completion":"Y) IF A POINT CHARGE IS PLACED INSIDE A HOLLOW, UNCHARGED CONDUCTING SHELL, THERE WILL BE NO NET ELECTRIC FIELD OUTSIDE OF THE SHELL13) Earth and Space - Short Answer What is the name of the graben [grah-ben] that has the highest recorded temperature on Earth?"} {"Type":"BONUS\n\nEarth and Space - Multiple Choice","Prompt":"When using a psychrometer and the two temperatures read are nearly identical, what conclusion would you reach?\n\nW) Your instrument reading is accurate\n\nX) A change in temperature is forecast\n\nY) The air has a high relative humidity\n\nZ) The dew point temperature is very low","Completion":"Y) THE AIR HAS A HIGH RELATIVE HUMIDITY"} {"Type":"TOSS-UP\n\nMath - Short Answer","Prompt":"The inverse of the function \\(f(x)=x^{4}+x^{3}+x^{2}+x+1\\) is not itself a function, but how many distinct zeroes does the inverse have?","Completion":"1"} {"Type":"BONUS\n\nMath - Multiple Choice","Prompt":"Consider a 2 by 2 matrix with first row 2, A and second row B, 3, where A and B are integers. How many different combinations exist of the values of A and B so that the matrix is singular?\n\nW) 4\n\nX) 6\n\nY) 8\n\nZ) 10","Completion":"Y) 815) Chemistry - Short AnswerIdentifify all of the following three statements regarding galvanic cells that are true: 1) The anode is the site of oxidation; 2) A salt bridge or semipermeable membrane is required to complete the circuit; 3) Batteries consist of multiple galvanic cells connected in a series."} {"Type":"## BONUS\n\nChemistry - Short Answer","Prompt":"Identifify all of the following three processes that would have a negative change in Gibbs free energy when conducted at atmospheric pressure: Boiling of water at 95 degrees Celsius; Freezing of water at 5 degrees Celsius; Sublimation of ice at 105 degrees Celsius.","Completion":"3"} {"Type":"## BONUS\n\nBiology - Multiple Choice","Prompt":"The terms frustule [fruh-stool], epitheleca [epi-thek-uh], and silica apply to which of the following organisms?\n\nW) Diatoms\n\nX) Cocolithophers\n\nY) Dinoflagellates\n\nZ) Foraminifera","Completion":"W) DIATOMS"} {"Type":"TOSS-UP\n\nMath - Short Answer","Prompt":"Classify the roots of the equation \\(2x^{2}-4x-3=0\\) as imaginary, rational, or irrational.","Completion":"IRRATIONAL"} {"Type":"BONUS\n\nMath - Short Answer","Prompt":"Determine the limit as \\(x\\) approaches negative \\(\\pi\\) of the fraction with numerator cos of the fraction \\(x\/2\\) and denominator \\(\\pi\\) plus x.","Completion":"\\( u{2}\\)"} {"Type":"TOSS-UP\n\nPhysics - Short Answer","Prompt":"What is the name of the force that accounts for the deflection of a moving object due to the Earth's rotation?","Completion":"Coriolis"} {"Type":"BONUS\n\nPhysics - Short Answer","Prompt":"Identify all of the following three statements that are true regarding charged coupled devices: Charged coupled devices can often be found in cameras; Charged coupled devices operate via the photoelectric effect; Pixels in charged coupled devices are made of metal conductors.","Completion":"1 AND 2"} {"Type":"TOSS-UP\n\nEnergy - Short Answer","Prompt":"What is the change in internal energy, delta U, in joules, of a system that absorbs 4000 joules of heat and that does 2000 joules of work on the surroundings?","Completion":"2000"} {"Type":"BONUS\n\nEnergy - Short Answer","Prompt":"What is the emerging class of fuels produced by non-photosynthetic microorganisms that directly harness chemical and electrical energy to turn carbon dioxide into fuels?","Completion":"ELECTROFUELS"} {"Type":"TOSS-UP\n\nBiology - Multiple Choice","Prompt":"Which of the following vitamins is not fat soluble?\n\nW) Tocopherol [toh-cof-uh-rol]\n\nX) Phylloquinone [fj-loh-kwih-nohn]\n\nY) Pyridoxine [pie-rih-dox-een]\n\nZ) Calciferol [cal-sift-uh-rol]","Completion":"Y) PYRIDOXINE"} {"Type":"BONUS\n\nBiology - Short Answer","Prompt":"In C4 photosynthesis, carbon dioxide is fixed to phosphoenolpyruvate [fos-foh-ee-nol-pie-roo-vate] instead of RuBisCo, which forms oxaloacetate. Oxaloacetate is then converted into what organic compound before it is transported to the bundle-sheath cells?","Completion":"MALATE (ACCEPT: MALIC ACID)"} {"Type":"TOSS-UP\n\nEnergy - Short Answer","Prompt":"What is the type of tissue found in the neck and between the shoulders of some mammals that specializes in rapid heat production by producing a molecule that uncouples the ATP synthase, and causes cells to produce heat instead of ATP?","Completion":"BROWN FAT"} {"Type":"BONUS\n\nEnergy - Short Answer","Prompt":"What country is the world's top ethanol fuel producer?","Completion":"UNITED STATES"} {"Type":"TOSS-UP\n\nChemistry - Short Answer","Prompt":"When methyl propanoate [pro-pah-noh-ate] reacts with sodium hydroxide and the reaction is heated, sodium propanoate [pro-pah-noh-ate] and methanol form. What is the name of this type of reaction?","Completion":"SAPONIFICATION (ACCEPT: BASE HYDROLYSIS OF AN ESTER; BASIC HYDROLYSIS)"} {"Type":"BONUS\n\nChemistry - Short Answer","Prompt":"How many vibrational degrees of freedom does a molecule of ethyne have?","Completion":"7"} {"Type":"TOSS-UP\n\nPhysics - Short Answer","Prompt":"What interpretation of quantum mechanics states that quantum mechanics does not provide objective descriptions, but instead probabilities of observing specific quantum phenomena?","Completion":"COPENHAGEN INTERPRETATION"} {"Type":"BONUS\n\nPhysics - Short Answer","Prompt":"A 2.5 Volt battery and a 1.5 Volt battery, each with an internal resistance of 1 Ohm, are connected in parallel such that their positive terminals are connected by a wire and their negative terminals are connected by a wire. What is the terminal voltage of the 1.5 Volt battery?","Completion":"2"} {"Type":"TOSS-UP\n\nMath - Short Answer","Prompt":"If the radius of a right circular cylinder is doubled, by what factor must the height be multiplied in order to keep the volume the same?","Completion":"\\( icefrac{{\\gamma}}{{4}}\\)"} {"Type":"BONUS\n\nMath - Short Answer","Prompt":"Find an antiderivative with respect to \\(x\\) for the fraction with \\(\\underline{\\text{numerator}}\\) e\\({}^{2\\pi}\\) and \\(\\underline{\\text{denominator }}1+\\text{e}^{4\\pi}\\).","Completion":"\\( icefrac{{\\gamma}}{{2}}\\) arctan e\\({}^{2\\pi}\\) (ACCEPT: \\( icefrac{{\\gamma}}{{2}}\\) arctan e\\({}^{2\\pi}+\\text{C}\\) for any constant C)"} {"Type":"TOSS-UP\n\nPhysics - Multiple Choice","Prompt":"In order for a process to have no change in entropy, which of the following properties must it possess?\n\nW) Spontaneous\n\nX) Reversible\n\nY) Endothermic\n\nZ) Exothermic","Completion":"X) Reversible"} {"Type":"BONUS\n\nPhysics - Short Answer","Prompt":"A circular coil consisting of 100 loops has a cross-sectional area of 0.005 meters squared and a resistance of 3 ohms. If a time-varying magnetic field B is oriented parallel to the axis of the coil, what is the magnitude of the rate of change of B that induces a 2.0 ampere current along the wire?","Completion":"12"} {"Type":"TOSS-UP\n\nEarth and Space - Multiple Choice","Prompt":"Which of the following is a cloud type that, by definition, produces precipitation?\n\nW) Stratocumulus\n\nX) Altostratus\n\nY) Nimbostratus\n\nZ) Altocumulus","Completion":"Y) NIMBOSTRATUS"} {"Type":"BONUS\n\nEarth and Space - Multiple Choice","Prompt":"Which of the following explains why the number and intensity of wave cyclones is greatest during the late fall, winter and spring months?\n\nW) Air is drier\n\nX) Density of the air is greatest\n\nY) Temperature contrasts are greater\n\nZ) Temperatures are below freezing","Completion":"Y) TEMPERATURE CONTRASTS ARE GREATER"} {"Type":"TOSS-UP\n\nChemistry - Multiple Choice","Prompt":"Which of the following is not true of oxalic acid?\n\nW) It is a diprotic acid\n\nX) It can be oxidized to carbon dioxide\n\nY) It contains three oxygen atoms per molecule\n\nZ) It contains two carbon atoms per molecule","Completion":"Y) IT CONTAINS THREE OXYGEN ATOMS PER MOLECULE"} {"Type":"BONUS\n\nChemistry - Short Answer","Prompt":"If the equilibrium constant, K, for the reaction A yields B is 2.0 x 10\\({}^{-5}\\), then what is the equilibrium constant for the reaction 2B yields 2A?","Completion":"\\(2.5\\times 10^{9}\\)"} {"Type":"TOSS-UP\n\nBiology - Multiple Choice","Prompt":"Which of the following is NOT a characteristic of chordates?\n\nW) Notochord\n\nX) Pharyngeal slits [fahr-in-jee-uh]\n\nY) Radial symmetry\n\nZ) Post-anal tail","Completion":"Y) RADIAL SYMMETRY"} {"Type":"BONUS\n\nBiology - Short Answer","Prompt":"What type of enzyme, produced as part of bacterial immunity against viruses, recognizes palindromic sites on foreign DNA and cleaves them?","Completion":"RESTRICTION ENZYME"} {"Type":"TOSS-UP\n\nEnergy - Short Answer","Prompt":"What is the thermodynamic cycle that describes the functioning of a sparkless fuel injection piston engine and can be found in certain types of automobiles?","Completion":"DIESEL CYCLE"} {"Type":"BONUS\n\nEnergy - Short Answer","Prompt":"What is the name given to organisms that can use either fermentation or respiration to produce enough ATP to survive?","Completion":"FACULTATIVE ANAEROBES7) Math - Short Answer A Diophantine equation is a polynomial equation in two or more variables such that the solutions are what type of number?"} {"Type":"BONUS\n\nMath - Short Answer","Prompt":"If \\(x\\) is measured in radians, find all values of \\(x\\) between zero and \\(2\\pi\\) that satisfy the equation \\(\\sin^{2}\\!x=2\\)\\(\\cos^{4}\\!x\\).","Completion":"\\(\\pi\/4\\), \\(3\\pi\/4\\), \\(5\\pi\/4\\), \\(7\\pi\/4\\)"} {"Type":"TOSS-UP\n\nPhysics - Multiple Choice","Prompt":"The half-life of substance X is 45 years and decomposes to substance Y. A sample from substance Z was taken which contained 1.5% of X and 13.5% of Y by mass. If substance Y is not normally found in substance Z, which of the following is the best estimate of the age, in years, of substance Z?\n\nW) 45\n\nX) 100\n\nY) 140\n\nZ) 270","Completion":"Y) 140"} {"Type":"BONUS\n\nPhysics - Short Answer","Prompt":"What substance, first postulated by Fritz Zwicky, is thought to be responsible for aberrations between theorized and observed orbital velocities of galaxies?","Completion":"DARK MATTER"} {"Type":"TOSS-UP\n\nEarth and Space - Short Answer","Prompt":"What is the name for the non-dimensional, unitless quantity that indicates how well a surface reflects solar energy?","Completion":"ALBEDO"} {"Type":"BONUS\n\nEarth and Space - Multiple Choice","Prompt":"Which of the following is NOT true regarding the ocean floor?\n\nW) Continental shelves generally overlay oceanic crust\n\nX) The continental shelf tends to be fairly featureless\n\nY) Continental shelves represent less than 10% of the total ocean area\n\nZ) Continental shelves are saturated with water","Completion":"W) CONTINENTAL SHELVES GENERALLY OVERLAY OCEANIC CRUST"} {"Type":"TOSS-UP\n\nChemistry - Multiple Choice","Prompt":"Which of the following statements is true for an ideal solution?\n\nW) The enthalpy of solution is exothermic\n\nX) The vapor pressure of the solution is directly proportional to the molality of the solvent\n\nY) Solvent particles interact more strongly with solute particles than other solvent particles\n\nZ) Raoult's [rah-oolt's] Law is obeyed","Completion":"Z) RAOULT's LAW IS OBEYED"} {"Type":"BONUS\n\nChemistry - Multiple Choice","Prompt":"The rate-limiting step for a reaction involving Reactant X is X + X yields\n\nY, where Y is a product. Which of the following quantities plotted against time would produce a straight line?\n\nW) One over molarity of X\n\nX) Molarity of X\n\nY) Natural log of the molarity of X\n\nZ) Molarity of X squared","Completion":"W) ONE OVER MOLARITY OF X"} {"Type":"TOSS-UP\n\nBiology - Short Answer","Prompt":"Identify all of the following three choices that represent dangers to biodiversity: Allowing fallow farmland to regrow with weeds; Importing an Australian moth to deal with pests on farmland; Relocating mountain lions that have moved into human habitats.","Completion":"2"} {"Type":"BONUS\n\nBiology - Multiple Choice","Prompt":"In which of the following organisms would you most likely be able to find contractile vacuoles?\n\nW) Cytomegalovirusolvirus\n\nX) E. coli\n\nY) Paramecium\n\nZ) Xenopus [zee-noh-pus]","Completion":"Y) PARAMETICUM"} {"Type":"TOSS-UP\n\nMath - Short Answer","Prompt":"For what values of \\(x\\) does the function \\(f(x)=\\) the fraction with numerator \\(x-2\\) and denominator \\(x^{2}-4\\) have a vertical asymptote?","Completion":"-2"} {"Type":"BONUS\n\nMath - Short Answer","Prompt":"Evaluate the following definite integral: the integral from \\(x=\\) -3 to 0 of the square root of open parenthesis \\(9-x^{2}\\) close parenthesis dx.","Completion":"(9\\(\\pi\\))\/4 (ACCEPT: 2.25\\(\\pi\\))"} {"Type":"TOSS-UP\n\nPhysics - Short Answer","Prompt":"What type of early particle accelerator, invented in the 1930s, consists of an evacuated chamber inside of a uniform magnetic field in which particles orbit circularly in and out of D-shaped conductors?","Completion":"CYCLOTRON"} {"Type":"BONUS\n\nPhysics - Short Answer","Prompt":"An ideal refrigerator absorbs heat at -23\\({}^{\\circ}\\) Celsius and exhausts it at 27\\({}^{\\circ}\\) Celsius. Find its coefficient of performance to the nearest integer.","Completion":"5"} {"Type":"TOSS-UP\n\nEarth and Space - Short Answer","Prompt":"Detecting signatures of gravitational waves on the cosmic microwave background would lend support to what theory first proposed by Alan Guth?","Completion":"INFLATION"} {"Type":"BONUS\n\nEarth and Space - Multiple Choice","Prompt":"What causes clouds of matter to flatten during the development of a protostar?\n\nW) Rotation\n\nX) Energy release\n\nY) Gravity\n\nZ) Diffusion","Completion":"W) ROTATION"} {"Type":"TOSS-UP\n\nEnergy - Short Answer","Prompt":"What is the energy range in a solid where no electron states can exist called?","Completion":"BAND GAP (ACCEPT: ENERGY GAP)"} {"Type":"BONUS\n\nEnergy - Short Answer","Prompt":"What type of compression, found in the Rankine and Otto cycles, compresses the gas without removing any heat from it?","Completion":"ADIABATIC"} {"Type":"TOSS-UP\n\nChemistry - Multiple Choice","Prompt":"Which of the following is true concerning a chemical system at a state of dynamic equilibrium?\n\nW) Forward and reverse rate constants are both equal to one\n\nX) Change in enthalpy for the reaction at standard state is zero\n\nY) Change in Gibbs free energy is zero\n\nZ) Molar entropy of products minus molar entropy of reactants is zero","Completion":"Y) CHANGE IN GIBBS FREE ENERGY IS ZERO"} {"Type":"BONUS\n\nChemistry - Short Answer","Prompt":"Approximating copper (II) chloride's molar mass as 133 grams per mole, and assuming that the boiling point elevation constant for water is 0.5 degrees Celsius per molal, calculate the boiling point in degrees Celsius to one decimal place of a solution prepared by dissolving 80 grams of copper (II) chloride in 1 kilogram of water.","Completion":"100.9"} {"Type":"TOSS-UP\n\nMath - Short Answer","Prompt":"If the difference between two prime numbers is 11, what is their sum?","Completion":"15"} {"Type":"BONUS\n\nMath - Short Answer","Prompt":"What is the largest prime factor of the quantity \\(4^{2013}+4^{2014}+4^{2015}\\)?","Completion":"7"} {"Type":"TOSS-UP\n\nBiology - Short Answer","Prompt":"Complete beta oxidation of odd-numbered saturated fatty acids requires what vitamin that is not required for the beta oxidation of even-numbered saturated fatty acids?","Completion":"VITAMIN B-12 (Accept: COBALAMIN)"} {"Type":"BONUS\n\nBiology - Short Answer","Prompt":"Identify all of the following three choices that are pieces of evidence that lend credence to the endosymbiotic theory: Chloroplasts contain their own genomes; Mitochondria contain bacteria-like ribosomes; All mitochondrial proteins are synthesized in the nucleus.","Completion":"1 AND 219) Physics - Short AnswerIdentify all of the following three statements that explain why the actual acceleration due to gravity at any point on the Earth's surface would be different from the value calculated for that point via Newton's Law of Gravitation: 1) The Earth's mass is not uniformly distributed; 2) The Earth is not perfectly spherical; 3) The Earth is rotating."} {"Type":"TOSS-UP\n\nBiology - Short Answer","Prompt":"What are the mating structures of basidiomycetes [bah-sid-ee-oh-mye-seets] called?","Completion":"MUSHROOMS (ACCEPT: BASIDIOCARPS)"} {"Type":"BONUS\n\nBiology - Short Answer","Prompt":"What largely parasympathetic nerve innervates the heart and is responsible for slowing down heart rate and lowering ventricular contractility when in a \"rest and digest\" state?","Completion":"VAGUS NERVE (ACCEPT: CN X, CRANIAL NERVE TEN)"} {"Type":"TOSS-UP\n\nEarth and Space - Short Answer","Prompt":"Pompeii was destroyed by what volcanic phenomenon, sometimes called \"glowing avalanches?\"","Completion":"PYROCLASTIC FLOW (ACCEPT: NUEE ARDENTE)"} {"Type":"BONUS\n\nEarth and Space - Short Answer","Prompt":"What soil horizon is the loosest and is characterized by partly decayed organic matter?","Completion":"O HORIZON"} {"Type":"TOSS-UP\n\nChemistry - Short Answer","Prompt":"What group 14 element has the highest melting point?","Completion":"CARBON"} {"Type":"BONUS\n\nChemistry - Short Answer","Prompt":"Identify all of the following three statements concerning electrochemistry that are true: A Faraday is the number of coulombs of charge generated by 1 mole of electrons; The oxidation of copper into copper(II) ions can spontaneously drive the reduction of protons into hydrogen gas at standard state; The oxidation of zinc into zinc(II) ions can spontaneously drive the reduction of protons into hydrogen gas at standard state.","Completion":"1 AND 3"} {"Type":"TOSS-UP\n\nBiology - Short Answer","Prompt":"What photosynthetic pathway, named after its discoverers, is also known as the light-independent pathway?","Completion":"CALVIN-BENSON CYCLE (ACCEPT: CALVIN CYCLE)"} {"Type":"BONUS\n\nBiology - Multiple Choice","Prompt":"Which of the following would be the most likely explanation for\n\nhypothyroidism in a patient whose iodine level is normal?\n\nW) Disproportionate production of T3 to T4\n\nX) Hyposecretion of TSH\n\nY) Hypersecretion of TSH\n\nZ) Hypersecretion of MSH","Completion":"X) HYPOSECRETION OF TSH"} {"Type":"TOSS-UP\n\nEnergy - Short Answer","Prompt":"What is the name of the compound in which methane is sequestered into water-\n\nice, forming a crystal structure in which water molecules create a cage for a single methane molecule?","Completion":"METHANE CLATHRATE (ACCEPT: METHANE HYDRATE, HYROMETHANE,"} {"Type":"BONUS\n\nEnergy - Short Answer","Prompt":"In fluorescent tube lamps, what element forms a vapor inside of the tube that\n\nproduces the short-wave ultraviolet light when excited by electricity?","Completion":"MERCURY"} {"Type":"TOSS-UP\n\nMath - Short Answer","Prompt":"What adjective describes two angles that sum to 180 degrees?","Completion":"SUPPLEMENTARY"} {"Type":"BONUS\n\nMath - Multiple Choice","Prompt":"What transformation occurs in going from the graph of \\(f(x)=0.5x^{2}+x\\) to the graph of \\(g(x)=0.5x^{2}-x\\)?\n\nW) Horizontal shift right by 1 unit\n\nX) Horizontal shift left by 1 unit\n\nY) Horizontal shift right by 2 units\n\nZ) Horizontal shift left by 2 units","Completion":"Y) Horizontal SHIFT RIGHT BY 2 UNITS"} {"Type":"TOSS-UP\n\nChemistry - Multiple Choice","Prompt":"Consider a reaction vessel at equilibrium where gaseous oxygen and fluorine are reacting to generate liquid oxygen difluoride. The enthalpy of formation of oxygen difluoride is positive. Which of the following pertubations would increase the yield of oxygen difluoride?\n\nW) Increasing the volume of the reaction vessel\n\nX) Increasing the temperature of the reaction vessel\n\nY) Increasing the pressure of the reaction vessel by adding helium gas\n\nZ) Removing oxygen gas from the reaction vessel","Completion":"X) Increasing the temperature of the reaction vessel"} {"Type":"BONUS\n\nChemistry - Multiple Choice","Prompt":"Which of the following statements concerning lattice enthalpies is not true?\n\nW) Increasing ion size will produce a larger lattice enthalpy\n\nX) Cations and anions that are more highly charged will produce larger lattice enthalpies\n\nY) Lattice enthalpy correlates with the percentage ionic character of a compound\n\nZ) Lattice enthalpies can be calculated by summing up electrostatic interactions between all pairs of ions","Completion":"W) Increasing ION SIZE WILL PRODUCE A LARGER LATTICE ENTHALPY"} {"Type":"TOSS-UP\n\nEarth and Space - Multiple Choice","Prompt":"Which of the following is a reasonable conclusion from the observation that particles found in the Sun's chromosphere are more ionized than ions found in the photosphere?\n\nW) The photosphere and the chromosphere are composed of different ions\n\nX) The chromosphere is hotter than the photosphere\n\nY) The photosphere is magnetically opposite the chromosphere\n\nZ) The chromosphere is cooler than the photosphere","Completion":"X) THE CHROMOSPHERE IS HOTTER THAN THE PHOTOSPHERE"} {"Type":"BONUS\n\nEarth and Space - Multiple Choice","Prompt":"What forces maintain neutron stars against gravitational collapse?\n\nW) Electromotive forces\n\nX) Frictional forces\n\nY) Intermolecular forces\n\nZ) Nuclear forces","Completion":"Z) NUCLEAR FORCES"} {"Type":"TOSS-UP\n\nPhysics - Short Answer","Prompt":"What scientist disproved the corpuscular theory of light with his 1801 double-slit experiment, lending credence to the wave model of light?","Completion":"THOMAS YOUNG"} {"Type":"BONUS\n\nPhysics - Short Answer","Prompt":"Four 3-ohm resistors are connected to a 9-volt battery. Three of the resistors are connected in series, while the fourth is connected parallel to the other three. How many cumulative watts of power are dissipated by the four resistors at a given instant?","Completion":"36"} {"Type":"TOSS-UP\n\nEnergy - Multiple Choice","Prompt":"Which of the following is a notorious poison for the platinum catalyst in a polymer electrolyte membrane fuel cell?\n\nW) Carbon monoxide\n\nX) Carbon dioxide\n\nY) Carbonate\n\nZ) Nitrogen","Completion":"W) CARBON MONOXIDE"} {"Type":"TOSS-UP\n\nBiology - Multiple Choice","Prompt":"Which of the following techniques would one use to separate proteins based on size?\n\nW) Polymerase chain reaction\n\nX) E. coli transduction\n\nY) SDS-polyacrylamideamide gel electrophoresis fel-ek-roth-fer-ee-silhs]\n\nZ) Enzyme-linked immunoprecipitation","Completion":"Y) SDS-POLYACRYLAMIDE GEL ELECTPROHRESISIS"} {"Type":"TOSS-UP\n\nChemistry - Short Answer","Prompt":"Identify all of the following three compounds that have more ionic character than lithium chloride: Lithium fluoride; Rubidium fluoride; Lithium bromide.","Completion":"1 AND 2"} {"Type":"BONUS\n\nChemistry - Multiple Choice","Prompt":"Which of the following is the name for the type of isomerism, which is a subset of stereoisomerism, describing compounds that interconvert by rotation of single bonds?\n\nW) Constitutional isomerism\n\nX) Cis-trans isomerism\n\nY) Enantiomerism\n\nZ) Conformational isomerism","Completion":"Z) CONFORMATIONAL ISOMERISM"} {"Type":"TOSS-UP\n\nMath - Multiple Choice","Prompt":"Set \\(q\\) contains all irrational numbers \\(r\\) such that \\(0s\\)) orbital and two 'p' (read as: two \\(<\\)pause\\(>p\\)) orbitals creates how many sp\\({}^{2}\\) hybrid orbitals?","Completion":"3"} {"Type":"BONUS\n\n16CHEMISTRY Multiple Choice","Prompt":"A molecule whose geometry of electron domains is tetrahedral and whose shape is bent or angular is:\n\nW) water\n\nX) sulfur dioxide\n\nY) carbon dioxide\n\nZ) ammonium","Completion":"W) WATER"} {"Type":"TOSS-UP\n\nPHYSICS Multiple Choice","Prompt":"Thomas Young's double slit experiments give the most direct and strongest support for which of the following:\n\nW) the wave nature of light\n\nX) the constancy of the speed of light\n\nY) the existence of the photon\n\nZ) the particle nature of light","Completion":"W) THE WAVE NATURE OF LIGHT"} {"Type":"TOSS-UP\n\nMATH Multiple Choice","Prompt":"Which of the following is NOT a geometric sequence:\n\nW) 30, 6, \\(\\frac{6}{5}\\ldots\\)\n\nX) \\(a^{2}\\), \\(a^{3}b\\), \\(a^{4}b^{2}\\ldots\\)\n\nY) 6, 13, 20\\(\\ldots\\)\n\nZ) \\(\\frac{3}{2}\\), \\(\\frac{3}{4}\\), \\(\\frac{3}{8}\\ldots\\)","Completion":"Y) 6, 13, 20\\(\\ldots\\)"} {"Type":"TOSS-UP\n\nEARTH SCIENCE Multiple Choice","Prompt":"Which of the following is TRUE:\n\nW)deserts often occur on the windward sides of mountains\n\nX)taiga biomes are characterized by long severe winters and short summers\n\nY)the most fertile soils are typically found in tropical forests\n\nZ)pedocal soil types are most likely be found in wet, humid environments","Completion":"X) TAIGA BIOMES ARE CHARACTERZED BY LONG SEVERE WINTERS AND SHORT SUMMERS"} {"Type":"TOSS-UP\n\nGENERAL SCIENCE Multiple Choice","Prompt":"As an alternative name, galvanic corrosion is MOST\n\ncommonly called:\n\nW)pitting corrosion\n\nX)intergranular corrosion\n\nY)stress corrosion\n\nZ)two-metal corrosion","Completion":"Z)TWO-METAL CORROSION"} {"Type":"TOSS-UP\n\nASTRONOMY Short Answer","Prompt":"What astronomical constant has the value of 70 kilometers per second per megaparsec?","Completion":"HUBBLE CONSTANT (ACCEPT: HUBBLE)"} {"Type":"TOSS-UP\n\nBIOLOGY Short Answer","Prompt":"What is the final electron acceptor for electrons passing down the electron transport system?","Completion":"OXYGEN (ACCEPT: MOLECULAR OXYGEN or O\\({}{2}\\))"} {"Type":"","Prompt":"TOSS-UP","Completion":"23) CHEMISTRY Short Answer What are the bond angles around each carbon in a benzene ring? 120\\({}^{\\rm o}\\)"} {"Type":"","Prompt":"TOSS-UP","Completion":"24) PHYSICS Short Answer A boy gets a sled for his birthday in July. He exerts a 50-newton horizontal force as he pulls the 80-newton sled across the cement driveway at a constant speed. What is the coefficient of kinetic friction, to the second decimal place, between the driveway and the metal blades of the sled? 0.63"} {"Type":"TOSS-UP\n\nBIOLOGY Short Answer","Prompt":"What base triplet is the start codon for methionine (read as: methylene-oh-neen)?","Completion":"AUG"} {"Type":"TOSS-UP\n\nBIOLOGY Multiple Choice","Prompt":"If a person has consumed too much alcohol and is showing signs of ataxia [uh-TAK-see-uh], what portion of the brain is being inhibited?\n\nW) Left hemisphere of the cerebrum\n\nX) Tectum\n\nY) Thalamus\n\nZ) Cerebellum [ser-uh-BEL-uhm]","Completion":"Z) CEREBELLUM"} {"Type":"TOSS-UP\n\nCHEMISTRY Multiple Choice","Prompt":"Which of the following molecules exhibits intermolecular\n\nattractions commonly referred to as hydrogen bonding?\n\nW) H\\({}{2}\\)\n\nX) HBr [H-B-R]\n\nY) HCN [H-C-N]\n\nZ) HCOOH [H-C-O-O-H]","Completion":"Z) HCOOH"} {"Type":"BONUS\n\nCHEMISTRY Multiple Choice","Prompt":"The Heisenberg Uncertainty Principle implies that it is\n\nimpossible to simultaneously obtain exact measurements for what two properties of a particle?\n\nW) Mass & velocity\n\nX) Charge & spin\n\nY) Energy & wavelength\n\nZ) Position & momentum","Completion":"Z) POSITION & MOMENTUM"} {"Type":"TOSS-UP\n\nMATH Multiple Choice","Prompt":"Which of the following functions has the greatest number of discontinuities?\n\nW) \\(\\frac{3x+5}{x-4}\\) [quantity three x plus five, divided by quantity x minus 4]\n\nX)(\\(\\frac{1}{(x-})+x\\) [ open parenthesis one, divided by quantity x minus 1, close parenthesis, plus x]\n\nY) \\(\\frac{1}{x^{2}}\\) [one divided by x squared]\n\nZ) \\(\\frac{1}{x^{2}-1}\\) [one, divided by quantity x squared minus one]","Completion":"Z) \\(\\frac{1}{x^{2}-1}\\)"} {"Type":"BONUS\n\nMATH Short Answer","Prompt":"Compute the probability of getting a \"blackjack\" (two cards totaling 21 points) from a standard deck of cards, given that an ace counts as 11, a face card counts as 10, and all others count as their numerical values.","Completion":"32\/663"} {"Type":"TOSS-UP\n\nPHYSICS Multiple Choice","Prompt":"The electric field of a given charge is measured to be 1 newton per coulomb at a distance 10 centimeters away from the charge. According to Gauss's Law, what would the field be in newtons per coulomb at the same location if we were to double the charge?\n\nW) 0.1\n\nX) 2\n\nY) 10\n\nZ) 20","Completion":"X) 2"} {"Type":"BONUS\n\nPHYSICS Short Answer","Prompt":"A uniform magnetic field is applied perpendicularly to the velocity vector of a moving charged particle. What shape will the path of the charged particle be?","Completion":"CIRCLE (DO NOT ACCEPT: HELICAL)"} {"Type":"TOSS-UP\n\nEARTH AND SPACE Multiple Choice","Prompt":"Which of the following volcanic features would be most closely associated with a high rating on the Volcanic Explosivity Index scale?\n\nW) Pahoehoe lava [pah-HOH-ee-hoh-ee]\n\nX) High temperature flows\n\nY) Flood basalts\n\nZ) Widespread ash fallout","Completion":"Z) WIDESPREAD ASH FALLOUT"} {"Type":"BONUS\n\nEARTH AND SPACE Short Answer","Prompt":"What is the currently most accepted hypothesis for the observed acceleration of the expansion of the universe?","Completion":"DARK ENERGY"} {"Type":"TOSS-UP\n\nCHEMISTRY Short Answer","Prompt":"Identify all of the following three compounds that are capable of hydrogen bonding with themselves: hydrogen bromide; hydrogen cyanide; butanol.","Completion":"3 (ACCEPT: BUTANOL)"} {"Type":"BONUS\n\nCHEMISTRY Short Answer","Prompt":"For a particular reaction, \\(\\Delta\\)H = +10 [delta H equals positive ten] kilojules per mole and \\(\\Delta\\)S = +40 [delta S equals positive forty] joules per mole Kelvin. At what temperature, in Kelvin, does the reaction transition from being spontaneous to nonspontaneous?","Completion":"250 Kelvin"} {"Type":"TOSS-UP\n\nBIOLOGY Multiple Choice","Prompt":"What are the most important sensory receptors for the monitoring and regulation of blood pressure in mammals?\n\nW) Statocysts\n\nX) Baroreceptors\n\nY) Nocireceptors [NOH-si-SEP-tuhrs]\n\nZ) Chemoreceptors","Completion":"X) BARORECEPTORS"} {"Type":"BONUS\n\nBIOLOGY Short Answer","Prompt":"Early in the evolution of modern day fish, the superclass\n\nOsteichthyes [os-tee-IK-thee-eez] split into two groups, one of which includes fish such as the\n\nCoelocanth [see-LUH-kahnth] and lungfish. What is the name for this group of fish?","Completion":"SARCOPTERYGII (ACCEPT: LOBE FINNED FISH)"} {"Type":"TOSS-UP\n\nENERGY Multiple Choice","Prompt":"How much energy in tons of coal equivalent can be released by\n\nusing deuterium in 50 cups of sea water in a deuterium - tritium fusion reactor?\n\nW) 2\n\nX) 200\n\nY) 2,000\n\nZ) 2,000,000","Completion":"W) 2"} {"Type":"BONUS\n\nENERGY Short Answer","Prompt":"Fluorescence can be understood as the emission of a photon as a\n\nmolecule transitions from a singlet excited state to a singlet ground state. What phenomenon is\n\nunderstood as a molecule transitioning from a triplet excited state to a singlet ground state and\n\ngenerally takes place on a much longer time scale than fluorescence?","Completion":"PHOSPHORESCENCE"} {"Type":"TOSS-UP\n\nEARTH AND SPACE Short Answer","Prompt":"The Milky Way and the Andromeda Galaxy are the primary constituents of what structure?","Completion":"THE LOCAL GROUP"} {"Type":"BONUS\n\nEARTH AND SPACE Multiple Choice","Prompt":"Which of the following statements about the moon is NOT correct?\n\nW) Moonquakes are due to tidal stresses from the Earth and meteorite impacts\n\nX) The moon formed during the late heavy bombardment\n\nY) Lunar basalts contain fewer volatiles than basalts on Earth\n\nZ) The moon has no global magnetic field","Completion":"X) THE MOON FORMED DURING THE LATE HEAVY BOMBARDMENT"} {"Type":"TOSS-UP\n\nMATH Short Answer","Prompt":"What is the polar form of the equation: \\(y=x\\)?","Completion":"\\(\\theta=45^{\\circ}\\) or \\(\\theta=\\frac{\\pi}{4}\\)"} {"Type":"BONUS\n\nMATH Short Answer","Prompt":"What is the sum of the numbers of faces of each of the unique Platonic solids?","Completion":"50"} {"Type":"BONUS\n\nBIOLOGY Multiple Choice","Prompt":"Which of the following eye adaptations allows birds of prey like\n\nraptors to maintain high visual sensitivity when diving for prey?\n\nW) Presence of a tapetum lucidum [tuh-PEE-tuhm loo-SEE-duhm]\n\nX) Retina with two foveas [FOH-vee-uhs]\n\nY) Ability to change relative amounts of cis and trans retinal\n\nZ) Monocular vision","Completion":"X) RETINA WITH TWO FOVES"} {"Type":"TOSS-UP\n\nCHEMISTRY Multiple Choice","Prompt":"If a molecule is known to have a trigonal bipyramidal shape such that \"M\" is the central atom and \"L\" is each of the five substituents, which of the following statements is NOT true about this particular molecule?\n\nW) It has a 3-fold rotation axis\n\nX) It has at least two mirror planes\n\nY) It has an inversion center\n\nZ) It has at least one improper rotation possible","Completion":"Y) IT HAS AN INVERSION CENTER"} {"Type":"BONUS\n\nCHEMISTRY Short Answer","Prompt":"What is the theoretical yield, in moles, of ammonia when 4\n\nmoles of nitrogen are reacted with 9 moles of hydrogen in the Haber process?","Completion":"6"} {"Type":"TOSS-UP\n\nPHYSICS Short Answer","Prompt":"What is the name for the resistive force that is related to Reynolds number and acts on objects in the direction of relative fluid flow velocity?","Completion":"DRAG FORCE"} {"Type":"TOSS-UP\n\nEARTH AND SPACE Multiple Choice","Prompt":"What are the bright outbursts in flare stars due to?\n\nW) Raleigh-Taylor instabilities\n\nX) Magnetic field reconnections\n\nY) Unstable nuclear reactions\n\nZ) Convective overshoots","Completion":"X) MAGNETIC FIELD RECONNECTIONS"} {"Type":"TOSS-UP\n\nBIOLOGY Multiple Choice","Prompt":"Within the human forebrain, which of the following structures does not belong to the teleccephalon [tel-en-SEF-uh-lon]?\n\nW) Basal nuclei\n\nX) Hippocampus [hip-uh-KAM-puhs]\n\nY) Hypothalamus [hy-puh-THAL-uh-muhs]\n\nZ) Cerebral cortex","Completion":"Y) HYPOTHALAMUS"} {"Type":"BONUS\n\nBIOLOGY Multiple Choice","Prompt":"Which of the following pairs of echinoderms [ih-KY-nuh-durms]\n\nhave respiratory structures that could be characterized as lungs rather than gills, since they are formed from invaginations of the body surface?\n\nW) Sea star and sea urchin\n\nX) Sea urchin and brittle star\n\nY) Sea star and brittle star\n\nZ) Sea cucumber and brittle star","Completion":"Z) SEA CUCUMBER AND BRITTLE STAR"} {"Type":"TOSS-UP\n\nMATH Short Answer","Prompt":"If it takes you 20 seconds to solve a math problem and your friend 30 seconds to solve a math problem, how many can the two of you solve in 6 minutes?","Completion":"30"} {"Type":"BONUS\n\nMATH Short Answer","Prompt":"Two integers between 1 and 10, inclusive, are selected at random with replacement. What is the probability that both numbers selected are prime and their product is odd?","Completion":"9\/100 (ACCEPT: 0.09 or 9%)"} {"Type":"TOSS-UP\n\nENERGY Multiple Choice","Prompt":"According to the U.S. Energy Information Administration, energy consumption per capita in the U.S. is expected to change at what annual percentage from now until 2035?\n\nW) -0.12\n\nX) -0.5\n\nY) +0.5\n\nZ) +0.12","Completion":"X) -0.5"} {"Type":"BONUS\n\nENERGY Short Answer","Prompt":"What crystalline solid consists of a methane molecule surrounded by a cage of interlocking water molecules and is typically found on sea floors and under permafrost?","Completion":"METHANE HYDRATE (ACCEPT: METHANE CLATHRAE)"} {"Type":"TOSS-UP\n\nCHEMISTRY Multiple Choice","Prompt":"London dispersion forces increase as which of the following molecular properties also increases?\n\nW) Dipole moment\n\nX) Polarizability\n\nY) Bond angle\n\nZ) Ionization energy","Completion":"X) POLARIZABILITY"} {"Type":"BONUS\n\nCHEMISTRY Short Answer","Prompt":"Suppose the atoms of a polar diatomic molecule have partial charges of \\(\\pm 0.25\\) e [plus or minus zero point two five ej], equivalent to \\(\\pm 4.0x10^{-20}\\)[plus or minus four point zero times ten to the negative twentieth] coulombs. If the bond length is 1.5 angstroms, to two significant figures, what is the dipole moment of the molecule in coulombmeters?","Completion":"6.0x10\\({}^{-30}\\)"} {"Type":"TOSS-UP\n\nBIOLOGY Multiple Choice","Prompt":"Which of the following is NOT a direct requirement for dinoflagellates [din-uh-FLAJ-uh-layts] to carry out bioluminescence?\n\nW) Luciferly adenylate [foo-sif-UH-ril uh-de-ni-layt]\n\nX) Oxygen\n\nY) Inorganic pyrophosphate\n\nZ) Carbon dioxide","Completion":"Z) CARBON DIOXIDE"} {"Type":"TOSS-UP\n\nPHYSICS Multiple Choice","Prompt":"Which of the following is a consequence of the exclusion principle?\n\nW) Electrons bound in an atom can only occupy orbits for which the angular momentum is quantized\n\nX) An electron in a stationary state emits no radiation\n\nY) No two electrons in a single atom can have an identical set of quantum numbers\n\nZ) Only when an electron jumps to a lower state does it emit a photon","Completion":"Y) NO TWO ELECTRONS IN A SINGLE ATOM CAN HAVE AN IDENTICAL SET OF QUANTUM NUMBERS"} {"Type":"TOSS-UP\n\nENERGY Short Answer","Prompt":"Most companies pursuing algae as a source of biofuels are pumping nutrient-laden water through plastic or borosilicate glass tubes that are exposed to sunlight. What is the name of the device being used in this algae fuel production process?","Completion":"BIOREACTOR (ACCEPT: PHOTOBIOREACTOR, ALGAE BIOREACTOR)"} {"Type":"BONUS\n\nENERGY Multiple Choice","Prompt":"Which of the following is NOT true regarding catalysis?\n\nW) Biological catalysts are called enzymes\n\nX) Catalysts lower the activation energy of chemical reactions\n\nY) A catalyzed reaction always has the same transition state as the uncatalyzed reaction\n\nZ) Catalysts have no effect on the equilibrium constant","Completion":"Y) A CATALYZED REACTION ALWAYS HAS THE SAME TRANSITION STATE AS THE UNCATALYZED REACTION"} {"Type":"TOSS-UP\n\nMATH Short Answer","Prompt":"What is the two-digit square number with the largest sum of its own digits?","Completion":"49"} {"Type":"BONUS\n\nMATH Short Answer","Prompt":"Determine \\(\\lim\\limits{x\\rightarrow\\pi}\\frac{(\\cos(x\/)}{\\pi+x}\\)[the limit as x approaches minus pi of the fraction with numerator cosine of the quantity x divided by two and denominator pi plus x].","Completion":"\\(\\frac{1}{2}\\)"} {"Type":"TOSS-UP\n\nCHEMISTRY Short Answer","Prompt":"A particular reaction involves only one reactant, which decomposes into products. This reactant is also a strong absorber of yellow light, which allows a scientist to use a spectrometer to measure the reactant concentration as a function of time. The scientist then plots the natural logarithm of the reactant concentration as a function of time, and notices that the plot is linear. What is the order of the kinetics of this reaction?","Completion":"FIRST (ACCEPT: ONE)"} {"Type":"TOSS-UP\n\nMATH Short Answer","Prompt":"Find the sum of the infinite geometric series with first three terms 1\/4, 1\/16, and 1\/64.","Completion":"1\/3"} {"Type":"TOSS-UP\n\nENERGY Multiple Choice","Prompt":"Volatile fossil fuels tend to have low carbon to hydrogen ratios.\n\nWhich of the following fossil fuels is the most volatile?\n\nW) Coal\n\nX) Ethane\n\nY) Methane\n\nZ) Petroleum","Completion":"Y) METHANE"} {"Type":"TOSS-UP\n\nEARTH AND SPACE Multiple Choice","Prompt":"Which of the following statements is NOT true about the Gulf of Maine?\n\nW) It is highly fertile\n\nX) A gyre [JYUHRJ circulates nutrients and pollution through the Gulf\n\nY) It is among the deepest gulfs in the world\n\nZ) It is sub-arctic, but has a wide range in water temperature","Completion":"Y) IT IS AMONG THE DEEPEST GULFS IN THE WORLD"} {"Type":"TOSS-UP\n\nCHEMISTRY Multiple Choice","Prompt":"Which of the following is NOT a step in balancing a redox\n\nreaction in an acidic solution?\n\nW) Write the unbalanced net ionic equation\n\nX) Determine which atoms are oxidized and which atoms are reduced\n\nY) Balance each half-reaction for O by adding hydroxide (OH') to the side with less oxygen\n\nZ) Balance each half-reaction for H by adding H' to the side with less H","Completion":"Y) BALANCE EACH HALF-REACTION FOR O BY ADDING HYDROXIDE (OH') TO THE SIDE WITH LESS OXYGEN"} {"Type":"TOSS-UP\n\nBIOLOGY Multiple Choice","Prompt":"Which of the following is NOT an example of substrate-level phosphorylation [FOS-for-uh-lay-shuhn] in cellular respiration?\n\nW) ATP synthase phosphorylates [FOS-for-uh-layts] ADP during chemiosmosis [Ke-mee-uh-SMOH-sis]\n\nX) GDP is phosphorylated [fos-fer-uh-LAY-ted] as succinyl [SUHK-suh-nil] and CoA is converted to succinate [SUHK-suh-nayt]\n\nY) Phosphofructokinase phosphorylates ADP\n\nZ) Pyruvate [py-ROO-vayt] kinase phosphorylates ADP","Completion":"W) ATP SYNTHASE PHOSPHORYLATES ADP DURING CHEMIOSMOSIS"} {"Type":"BONUS\n\nBIOLOGY Short Answer","Prompt":"Beta-lactams, the class of antibiotics to which penicillin belongs, interfere with bacterial synthesis of what cellular structure in bacteria?","Completion":"CELL WALL"} {"Type":"TOSS-UP\n\nPHYSICS Multiple Choice","Prompt":"Two cars of unequal mass crash and stick together in a head on collision. Which of the following statements is true?\n\nW) The car with the larger mass experiences the larger magnitude impulse\n\nX) The car with the smaller mass experiences the larger magnitude impulse\n\nY) Both cars experience the same magnitude impulse in the same direction\n\nZ) Both cars experience the same magnitude impulse in opposite directions","Completion":"Z) BOTH CARS EXPERIENCE THE SAMe MAGNITUDE IMPULSE IN OPPOSITE DIRECTIONS"} {"Type":"BONUS\n\nPHYSICS Multiple Choice","Prompt":"Two balls are thrown perfectly horizontally off a cliff on Mars,\n\nwhere the acceleration due to gravity is 3.7 meters per second squared. If ball 1 lands 30 meters further than ball 2, which of the following statements is correct about their initial\n\nvelocities?\n\nW) The velocity of ball 1 is 30 meters per second greater than the velocity of ball 2\n\nX) The velocity of ball 2 is 30 meters per second greater than the velocity of ball 1\n\nY) The velocity of ball 1 is 30 times larger than the velocity of ball 2\n\nZ) The exact relationship cannot be determined without knowing the height of the cliff","Completion":"Z) THE EXACT RELATIONSHIP CANNOT BE DETERMINED WITHOUT KNOWING THE HEIGHT OF THE CLIFF"} {"Type":"","Prompt":"TOSS-UP","Completion":"9) MATH Short Answer What is the area of a rhombus with diagonals of lengths 6 and 8? 24"} {"Type":"TOSS-UP\n\nENERGY Short Answer","Prompt":"In which direction do protons drag water in a polymer electrolyte membrane fuel cell?","Completion":"ANODE TO CATHODE (ACCEPT: NEGATIVE ELECTRODE TO POSITIVE ELECTRODE)"} {"Type":"TOSS-UP\n\nEARTH AND SPACE Multiple Choice","Prompt":"Which of the following statements is NOT true about upwelling?\n\nW) It is associated with good fishing\n\nX) Water that rises to the surface as a result of upwelling is typically colder and richer in nutrients\n\nY) It is caused when offshore wind causes surface water to build up along a coastline\n\nZ) It occurs in the open ocean and along coastlines","Completion":"Y) IT IS CAUSED WHEN OFFSHORE WIND CAUSES SURFACE WATER TO BUILD UP ALONG A COASTLINE"} {"Type":"BONUS\n\nEARTH AND SPACE Short Answer","Prompt":"What law mathematically expresses the amount of radiation emitted by a blackbody at a given temperature?","Completion":"STEFAN-BOLTZMANN (ACCEPT: STEFAN'S LAW)"} {"Type":"TOSS-UP\n\nCHEMISTRY Multiple Choice","Prompt":"Which of the following is NOT a property of all amino acids?\n\nW) They contain both carboxylic [Kahr-bok-SIL-ik] acid and amine [uh-MEEN] functional groups\n\nX) They are chiral [KY-ruhl]\n\nY) They are considered organic molecules\n\nZ) They are generally referred to by their common name","Completion":"X) THEY ARE CHIRAL"} {"Type":"BONUS\n\nCHEMISTRY Short Answer","Prompt":"What is the pH of a 0.001 molar solution of hydrochloric acid?","Completion":"3"} {"Type":"TOSS-UP\n\nBIOLOGY Multiple Choice","Prompt":"If you are doing a western blot, what kind of macromolecule are you trying to detect?\n\nW) DNA\n\nX) RNA\n\nY) Protein\n\nZ) Carbohydrate","Completion":"Y) PROTEIN"} {"Type":"BONUS\n\nBIOLOGY Short Answer","Prompt":"In adult humans, how many bones articulate with the occipital bone?","Completion":"6"} {"Type":"TOSS-UP\n\nPHYSICS Multiple Choice","Prompt":"An Atwood's machine is built with a massless and frictionless pulley with a mass of 1 kilogram on one side and a mass of 4 kilograms on the other side. Assuming that the acceleration due to gravity is 10 meters per second squared, which is the speed of the masses in meters per second three seconds after being released?\n\nW) 5\n\nX) 6\n\nY) 15\n\nZ) 18","Completion":"Z) 18"} {"Type":"BONUS\n\nPHYSICS Multiple Choice","Prompt":"You sit on a pier and look straight down into the pond below. The apparent depth of the pond from the surface of the water is 5.0 meters. Given that the refractive indexes of air and water are 1.0 and 1.3, respectively, what is the real depth of the pond in meters?\n\nW) 0.3\n\nX) 3.8\n\nY) 5.0\n\nZ) 6.7","Completion":"Z) 6.7"} {"Type":"TOSS-UP\n\nMATH Short Answer","Prompt":"A point rotates around a circle at the rate of 40\\({}^{\\circ}\\) per second. Convert this to revolutions per minute.","Completion":"\\(\\frac{20}{3}\\) (ACCEPT: 6 2\/3)"} {"Type":"TOSS-UP\n\nENERGY Multiple Choice","Prompt":"Acid rain is unusually acidic precipitation containing elevated levels of hydrogen ions. Which of the following emissions is NOT an emission from a power plant that would contribute to acid rain?\n\nW) Carbon dioxide\n\nX) Hydroxide\n\nY) Nitrogen oxide\n\nZ) Sulfur dioxide","Completion":"X) HYDROXIDE"} {"Type":"TOSS-UP\n\nEARTH AND SPACE Multiple Choice","Prompt":"Pangaea [pan-JEE-uh] formed by the end of what geologic period?\n\nW) Paleocene [pay-lee-uh-SEEN]\n\nX) Paleozoicic\n\nY) Permian\n\nZ) Pennsylvania","Completion":"Y) PERMIAN"} {"Type":"BONUS\n\nEARTH AND SPACE Short Answer","Prompt":"Which of the terrestrial planets in our solar system has the strongest magnetic field?","Completion":"EARTH"} {"Type":"TOSS-UP\n\nCHEMISTRY Multiple Choice","Prompt":"Which of the following best explains the difference between real gases and ideal gases?\n\nW) At low pressure, intermolecular attractions become significant\n\nX) At high pressure, intermolecular attractions become significant\n\nY) As pressure decreases, the volume of the gas itself also decreases\n\nZ) As pressure increases, the volume of the gas itself also increases","Completion":"X) AT HIGH PRESSURE, INTERROLOECULAR ATTRACTIONS BECOME"} {"Type":"BONUS\n\nCHEMISTRY Multiple Choice","Prompt":"For a d orbital, which of the following is NOT an allowed value for the magnetic quantum number m\\({}{\\ell}\\)?\n\nW) -1\n\nX) 0\n\nY) 2\n\nZ) 3","Completion":"Z) 3"} {"Type":"TOSS-UP\n\nBIOLOGY Multiple Choice","Prompt":"Which of the following is an example of catabolism in the cell?\n\nW) Energy is extracted from the fat in avocado\n\nX) New DNA is synthesized\n\nY) Glutamic acid is made from alpha-ketoglutarate [Kee-toh-GLOO-tay-rayt] in the citric acid cycle\n\nZ) Dihydroxyacetone [dy-hy-drok-see-AS-uh-tohn] phosphate from glycolysis [gly-KOL-uh-sis] is converted to a precursor for lipid synthesis","Completion":"W) ENERGY IS EXTRACTED FROM THE FAT IN AVOCADO"} {"Type":"TOSS-UP\n\nPHYSICS Short Answer","Prompt":"Providing your answer in scientific notation with one significant digit, what is the value of 1 erg in joules?","Completion":"1 X 10\\({}^{-7}\\)"} {"Type":"TOSS-UP\n\nMATH Multiple Choice","Prompt":"A hypersphere is described by a set of points such that \\(w^{2}+x^{2}+y^{2}+z^{2}=36\\). Which of the following statements is NOT true?\n\nW) (1, 2, 3, lies inside the hypersphere\n\nX) (1, 5, 5, lies outside the hypersphere\n\nY) (3, 3, 3, lies on the hypersphere\n\nZ) (1, 2, 5, lies outside the hypersphere","Completion":"Z) (1, 2, 5, 1) LIES OUTSIDE THE HYPERSPHERE"} {"Type":"TOSS-UP\n\nEARTH AND SPACE Multiple Choice","Prompt":"Which of the following is the most abundant sediment in the oceans?\n\nW) Hydrogenous [hy-DROJ-uh-nuhs]\n\nX) Terrigenous\n\nY) Biogenous\n\nZ) Cosmogenous","Completion":"X) TERRIGENOUS"} {"Type":"TOSS-UP\n\nCHEMISTRY Multiple Choice","Prompt":"What is the hybridization of the central oxygen atom in water?\n\nW) sp\n\nX) sp\\({}^{2}\\)\n\nY) sp\\({}^{3}\\)\n\nZ) sp\\({}^{3}\\)d","Completion":"Y) sp\\({}^{3}\\)"} {"Type":"TOSS-UP\n\nBIOLOGY Short Answer","Prompt":"The extracellular matrix protein, collagen, is composed of intertwined helices. How many protein helices come together to form collagen?","Completion":"3"} {"Type":"TOSS-UP\n\nPHYSICS Multiple Choice","Prompt":"If a light ray passes from air (n = 1.to acetone (n = 1., what is the critical angle?\n\nW) 47\\({}^{\\circ}\\)\n\nX) 67\\({}^{\\circ}\\)\n\nY) 87\\({}^{\\circ}\\)\n\nZ) There is no total internal reflection","Completion":"Z) THERE IS NO TOTAL INTERNAL REFLECTION"} {"Type":"TOSS-UP\n\nCHEMISTRY Multiple Choice","Prompt":"Which of the following strong acids, all at 0.1 molar concentration, will have the LOWEST pH value:\n\nW) H\\({}{2}\\)SO\\({}{4}\\)\n\nX) HNO\\({}{3}\\)\n\nY) HCl\n\nZ) HClO\\({}{4}\\)","Completion":"W) H\\({}{2}\\)SO\\({}{4}\\)"} {"Type":"BONUS\n\nCHEMISTRY Short Answer","Prompt":"Giving its proper charge, what is the oxidation number of the manganese atom in the following compound: MnO\\({}{4}\\)\\({}^{-}\\) (read as: M, N, O, 4, minus)","Completion":"+7 (ACCEPT: 7)"} {"Type":"TOSS-UP\n\nPHYSICS Multiple Choice","Prompt":"A field emission microscope requires which of the following:\n\nW) a conductor with a very fine point\n\nX) high energy alpha particles\n\nY) a light source with a wavelength shorter that the smallest observable feature size\n\nZ) a low amplitude, time-varying magnetic field","Completion":"W) A CONDUCTOR WITH A VERY FINE POINT"} {"Type":"BONUS\n\nPHYSICS Short Answer","Prompt":"Rounded to the nearest thousand, how many joules of work are done in pulling a wagon with a force of 2000 newtons over a horizontal surface of 10 meters, with a chain that makes a 45\\({}^{\\circ}\\) angle with the surface?","Completion":"14,000"} {"Type":"TOSS-UP\n\nBIOLOGY Multiple Choice","Prompt":"The genetic material of different viruses can be:\n\nW) only double-stranded DNA\n\nX) only single-stranded DNA or double-stranded DNA\n\nY) only double-stranded DNA or double-stranded RNA\n\nZ) either single-stranded DNA and RNA or double-stranded DNA and RNA","Completion":"Z) EITHER SINGLE-STANDED DNA AND RNA OR DOUBLE-STRANDED DNA AND RNA"} {"Type":"BONUS\n\nBIOLOGY Multiple Choice","Prompt":"Which of the following is NOT characteristic of retroviruses:\n\nW) they have single-stranded RNA as their genetic material\n\nX) they make mRNA from their original RNA\n\nY) they incorporate their genome into the DNA of the host cell using reverse transcriptase\n\nZ) the host cell polymerase transcribes viral genes","Completion":"X) THEY MAKE mRNA FROM THEIR ORIGINAL RNA"} {"Type":"TOSS-UP\n\nMATH Short Answer","Prompt":"A circle has a radius of 8 centimeters. Find the central angle, in degrees, of a sector of a circle if its arc length is \\(2\\pi\\) centimeters:","Completion":"45"} {"Type":"BONUS\n\nMATH Multiple Choice","Prompt":"Which of the following is closest to the radius of a sphere, in meters, given that its volume is 400 cubic meters:\n\nW) 2.15\n\nX) 4.57\n\nY) 6.28\n\nZ) 9.76","Completion":"X) 4.57"} {"Type":"TOSS-UP\n\nEARTH SCIENCE Multiple Choice","Prompt":"Which of the following typically requires warm, clear, shallow water characterized by low nutrient concentration and a hard coastal substrate to develop:\n\nW) barrier island\n\nX) estuary\n\nY) lagoon\n\nZ) coral reef","Completion":"Z) CORAL REFF"} {"Type":"TOSS-UP\n\nGENERAL SCIENCE Multiple Choice","Prompt":"Rooted plants that grow below the surface of the water are called submerged plants. Which of the following is BEST classified as a freshwater submerged plant:\n\nW) common reed\n\nX) sedges\n\nY) water lilies\n\nZ) water milfoil","Completion":"Z) WATER MILFOIL"} {"Type":"TOSS-UP\n\nASTRONOMY Multiple Choice","Prompt":"Recent analysis of Cassini spacecraft data has provided convincing evidence that Saturn's Moon Titan has:\n\nW) large seas of liquid water\n\nX) methane lakes on its surface\n\nY) a moon of its own\n\nZ) a large active volcano","Completion":"X) METHANE LAKES ON ITS SURFACE"} {"Type":"TOSS-UP\n\nCHEMISTRY Short Answer","Prompt":"In which positions on the benzene ring are the methyl groups found in the compound o-xylene or ortho-xylene (read as: ORTHO-ZIE-lean):","Completion":"1 AND 2 (ACCEPT: 2 AND 1)"} {"Type":"TOSS-UP\n\nPHYSICS Multiple Choice","Prompt":"The scattering of alpha particles by a metal foil was first studied by:\n\nW) Compton\n\nX) Rutherford\n\nY) Bragg\n\nZ) Rayleigh","Completion":"X) RUTHERFORD"} {"Type":"BONUS\n\nPHYSICS Short Answer","Prompt":"A pole vaulter with a mass of 150 kilograms is traveling at 25 meters per second just prior to landing on an air mattress which collapses and allows the vaulter to come to rest over a period of 2.0 seconds. What was the force of the impact between the vaulter and the air mattress, in newtons?","Completion":"1875"} {"Type":"TOSS-UP\n\nBIOLOGY Multiple Choice","Prompt":"TSH causes the thyroid to produce more T3. When T3 rises, it flows through the blood to the brain to decrease the release of TSH. Which of the following is the MOST accurate and common term for this physiological phenomenon:\n\nW) metabolic reaction\n\nX) hormonal balance\n\nY) negative feedback\n\nZ) endocrine action","Completion":"Y) NEGATIVE FEEDBACK"} {"Type":"BONUS\n\nBIOLOGY Multiple Choice","Prompt":"In mammalian kidneys, reabsorption of water occurs mostly in which of the following:\n\nW) glomerulus\n\nX) descending limb of the loop of Henle (read as: HEN-lee)\n\nY) proximal convoluted tubule\n\nZ) Bowman's capsule","Completion":"X) DESCENDING LIMB OF THE LOOP OF HENLE"} {"Type":"TOSS-UP\n\nMATH Multiple Choice If a population distribution is illustrated as a symmetrical bell shape curve, then the median and the mean are:\n\nW) not the same\n\nX) uncertain\n\nY) variable\n\nZ) the same\n\nAnswer: Z) THE SAME\n\nBonus\n\nMATH Short Answer","Prompt":"Giving your answer in standard form, what is the fourth term in the binomial expansion of the following: \\(\\left(4x+y\\right)^{4}\\)","Completion":"\\(16xy^{3}\\) (ACCEPT: \\(16y^{3}x\\))"} {"Type":"TOSS-UP\n\n13 GENERAL SCIENCE Multiple Choice","Prompt":"Which of the following is NOT true:\n\nW) nanometer-thick films that roll themselves into tubes and other useful shapes have been recently discovered\n\nX) counting annual growth lines in dinosaur bones can be used to estimate their age at death\n\nY) amphibian population declines and extinctions are global and rapid\n\nZ) epiphyic plants such as many orchids are considered parasitic plants","Completion":"Z) EPIPHYTIC PLANTS SUCH AS MANY ORCHIDS ARE CONSIDERED"} {"Type":"TOSS-UP\n\nASTRONOMY Multiple Choice","Prompt":"Microlensing is BEST explained by which of the following theories or laws:\n\nW) Quantum theory\n\nX) General Relativity\n\nY) Kirchhoff's laws\n\nZ) Maxwell's law","Completion":"X) GENERAL RELATIVITY"} {"Type":"BONUS\n\nCHEMISTRY Multiple Choice","Prompt":"Avogadro's gas law concerns which of the following two variables:\n\nW) temperature and pressure\n\nX) pressure and volume\n\nY) temperature and volume\n\nZ) moles and volume","Completion":"Z) MOLES AND VOLUME"} {"Type":"TOSS-UP\n\nPHYSICS Multiple Choice","Prompt":"Which of the following could most practically be used as a detector for the automatic door opening system you find at your local grocery store:\n\nW) infrared camera system\n\nX) ultrasonic motion detector\n\nY) radar system\n\nZ) optical camera system","Completion":"Y) RADAR SYSTEM"} {"Type":"BONUS\n\nPHYSICS Multiple Choice","Prompt":"The radar system associated with the automatic doors at a local grocery store uses 10.5 gigahertz radiation. If the door is to open for a person walking towards it at 1 meter per second, which Doppler shift must the radar system be able to detect:\n\nW) a red shift of 2 hertz\n\nX) a blue shift of 35 hertz\n\nY) a red shift of 160 hertz\n\nZ) a blue shift of 1.8 hertz","Completion":"X) A BLUE SHIFT OF 35 HERTZ"} {"Type":"TOSS-UP\n\nBIOLOGY Multiple Choice","Prompt":"During embryogenesis, which of the following is MOST accurate concerning the blastopore:\n\nW) it does not exist in protostomes\n\nX) it originates before the blastula stage\n\nY) it forms the anus in deuterostomes (read as: DOO-ter-oh-stomes)\n\nZ) it is the hollow center of the gastrula","Completion":"Y) IT FORMS THE ANUS IN DEUTEROSTOMES"} {"Type":"BONUS\n\nBIOLOGY Multiple Choice","Prompt":"Which of the following is a primary function of histones H2A, H2B, H3 and H4:\n\nW) preventing damage to DNA from ionizing radiation\n\nX) densely packaging DNA into nucleosomes\n\nY) holding DNA in the double helical configuration\n\nZ) directing the packaging of DNA into looped domains of about 300 nanometers","Completion":"X) DENSELY PACKAGING DNA INTO NUCLEOSOMES"} {"Type":"TOSS-UP\n\nMATH Multiple Choice","Prompt":"Which of the following has a graph that is a parabola that opens downward:\n\nW) f(x) = 3x + 1\n\nX) f(x) = x^2 - 2\n\nY) f(x) = -x^2\n\nZ) f(x) = x^3","Completion":"Y) f(x) = -x^2"} {"Type":"BONUS\n\nMATH Short Answer","Prompt":"Find the 101st term in the arithmetic sequence whose first three terms are 5, 11, and 17:","Completion":"605"} {"Type":"TOSS-UP\n\nEARTH SCIENCE Multiple Choice","Prompt":"Which of the following rocks typically has the SMALLEST grains:\n\nW) shale\n\nX) conglomerate\n\nY) breccia\n\nZ) sandstone","Completion":"W) SHALE"} {"Type":"TOSS-UP\n\nGENERAL SCIENCE Multiple Choice","Prompt":"What type of fluorescence is produced in \"Glow Sticks\" that are widely found on the market today and which glow for hours after the stick is \"broken\" to allow for the mixture of different substances:\n\nW) thermoluminescence\n\nX) triboluminescence\n\nY) photoluminescence\n\nZ) chemiluminescencecence","Completion":"Z) CHEMILUMINESCENCE"} {"Type":"TOSS-UP\n\nASTRONOMY Short Answer","Prompt":"Name all of the following 4 celestial objects that are typically considered minor planets: Ceres; Titan; Mercury; Juno","Completion":"CERES AND JUNO"} {"Type":"BONUS\n\nASTRONOMY Short Answer","Prompt":"How many times as great will light-gathering power of a reflector telescope be if the radius of its primary mirror is doubled?","Completion":"4"} {"Type":"TOSS-UP\n\nCHEMISTRY Short Answer","Prompt":"What is the systematic IUPAC name for the compound Fe(NO\\({}{2}\\))\\({}{3}\\)?","Completion":"IRON (III) NITRITE (read as: iron three nitrite) (MUST BE EXACT)"} {"Type":"BONUS\n\nCHEMISTRY Short Answer","Prompt":"To the second decimal place, what is the concentration of nitrate ions, in moles per liter, in a 0.26 molar Al(NO\\({}{3}\\))\\({}{3}\\) solution?","Completion":"0.78"} {"Type":"TOSS-UP\n\nPHYSICS Short Answer","Prompt":"Billy drops his teddy bear out of the car window 2 meters from the ground while his father is driving down Main Street at 20 meters per second. In seconds to the first decimal place, how long does it take for the bear to hit the ground, assuming air resistance is neglected?","Completion":"0.6"} {"Type":"TOSS-UP\n\nBIOLOGY Short Answer","Prompt":"The corpus luteum in humans primarily secretes what hormone?","Completion":"PROGESTERONE"} {"Type":"TOSS-UP\n\nPHYSICS Short Answer","Prompt":"What is the underwater pressure in atmospheres in a fresh water lake at a depth of 20 meters?","Completion":"3"} {"Type":"TOSS-UP\n\nCHEMISTRY Multiple Choice","Prompt":"Which of the following gases has the slowest rate of effusion?\n\nW) Hydrogen\n\nX) Helium\n\nY) Oxygen\n\nZ) Neon","Completion":"Y) OXYGEN"} {"Type":"BONUS\n\nCHEMISTRY Short Answer","Prompt":"The electrolysis of water is endothermic, requiring 286 kilojoules of energy to decompose one mole of water at 25\\({}^{\\circ}\\)C. Given that the enthalpy of formation of hydrogen peroxide is -136 kilojoules per mole, what is the standard enthalpy in kilojoules for the decomposition of one mole of hydrogen peroxide into water and oxygen?","Completion":"-150"} {"Type":"TOSS-UP\n\nBIOLOGY Multiple Choice","Prompt":"The arpili [pee-LEE] muscle has what function when a mammal is cold?\n\nW) Inducing shivering\n\nX) Speeding metabolism\n\nY) Increasing the depth of the air insulation barrier\n\nZ) Vasoconstriction","Completion":"Y) INCREASING THE DEPTH OF THE AIR INSULATION BARRIER"} {"Type":"BONUS\n\nBIOLOGY Multiple Choice","Prompt":"Someone has carelessly tossed several sets of human vertebrae into a single box. You are trying to sort out the mess and have counted 20 lumbar, 48 thoracic, and 26 cervical vertebrae. It's now clear you are missing some bones. How many bones are you missing, and what type of vertebrae are they?\n\nW) 4 lumbar\n\nX) 2 thoracic\n\nY) 2 cervical\n\nZ) 4 thoracic","Completion":"Y) 2 CERVICAL"} {"Type":"TOSS-UP\n\nENERGY Short Answer","Prompt":"Indicate, by number, all of the following statements that must be true for an object to be in static equilibrium [ee-kwuh-LIB-ree-uhm]: All forces are balanced; All torques are balanced; Their pressures are the same.","Completion":"1 AND 2"} {"Type":"BONUS\n\nENERGY Multiple Choice","Prompt":"The buildup of heavier elements in the nuclear fusion processes in stars is limited to elements below what isotope?\n\nW) Iron-56\n\nX) Iron-58\n\nY Nickel-62\n\nZ) Lead-82","Completion":"W) IRON-56"} {"Type":"TOSS-UP\n\nEARTH AND SPACE Multiple Choice","Prompt":"Which of the following best describes the accretion properties of the supermassive black hole at the center of the Milky Way?\n\nW) Not accreting a detectable amount of matter\n\nX) Accreting an unusually small amount of matter\n\nY) Accreting an average amount of matter\n\nZ) Accreting an unusually large amount of matter","Completion":"X) ACCRETING AN UNUSUALLY SMALL AMOUNT OF MATTER"} {"Type":"BONUS\n\nEARTH AND SPACE Multiple Choice","Prompt":"Which of the following mountain ranges in the state of Oregon is an accreted terrain?\n\nW) Wallowa Mountains\n\nX) Cascade Volcano Range\n\nY) Steens Mountains\n\nZ) Hart Mountains","Completion":"W) WALLOWA MOUNTAINS"} {"Type":"TOSS-UP\n\nMATH Short Answer","Prompt":"How many ways are there to get exactly 25 heads results from 26 consecutive fair coin tosses?","Completion":"26"} {"Type":"TOSS-UP\n\nBIOLOGY Short Answer","Prompt":"Name the type of reproduction in which the eggs hatch inside the female body and the embryo is retained for protection?","Completion":"OVOVIVPARITY (ACCEPT: OVOVIVIPAROUS; DO NOT ACCEPT: VIVPARY)"} {"Type":"TOSS-UP\n\nCHEMISTRY Multiple Choice","Prompt":"What choice best describes the type of reaction that is catalyzed by lyases [Jy-AYS-es]?\n\nW) Transfer of functional groups\n\nX) Mediate isomerization [eye-so-muh-ri-ZAY-shuhn]\n\nY) Addition to double bonds\n\nZ) Formation of bonds with ATP cleavage","Completion":"Y) ADDITION TO DOUBLE BONDS"} {"Type":"BONUS\n\nCHEMISTRY Short Answer","Prompt":"A neutral coordination compound is composed of a complex ion bound to a potassium cation [KAT-eye-uhn]. The complex ion contains a central cobalt ion bound to two ammonia ligands and four chloride ligands. What is the oxidation number of the cobalt ion?","Completion":"THREE PLUS"} {"Type":"TOSS-UP\n\nPHYSICS Short Answer","Prompt":"What is the resistance of a resistor that has a 20 milliampere current when connected across the terminals of a 1.5 volt battery?","Completion":"75 OHMS"} {"Type":"BONUS\n\nPHYSICS Multiple Choice","Prompt":"A light ray passes from air (n = 1.to the cornea (n = 1.of the eye. If the angle of incidence is 30\\({}^{\\circ}\\), which of the following is true of the angle of refraction?\n\nW) Less than 30\\({}^{\\circ}\\)\n\nX) Equal to 30\\({}^{\\circ}\\)\n\nY) Between 30\\({}^{\\circ}\\) and 45\\({}^{\\circ}\\)\n\nZ) Equal to 45\\({}^{\\circ}\\)","Completion":"W) LESS THAN 30\\({}^{\\circ}\\)"} {"Type":"TOSS-UP\n\nEARTH AND SPACE Short Answer","Prompt":"What is the name of the landform found west of the current Yellowstone Hotspot that was made as the North American Plate moved southwest over the hotspot?","Completion":"SNAKE RIVER PLAIN"} {"Type":"BONUS\n\nEARTH AND SPACE Multiple Choice","Prompt":"Which of the following best approximates the number of separate cinder cones located on the flanks of Newberry Volcano?\n\nW) 4\n\nX) 40\n\nY) 400\n\nZ) 4000","Completion":"Y) 400"} {"Type":"TOSS-UP\n\nBIOLOGY Short Answer","Prompt":"Identify the food test used to detect the presence of a reducing sugar by the addition of a solution containing sodium carbonate, sodium citrate, and copper sulphate.","Completion":"BENEDICT'S TEST"} {"Type":"BONUS\n\nBIOLOGY Multiple Choice","Prompt":"You are conducting an experiment that involves incubating a heat-insensitive enzyme with its substrate, while slowly increasing the temperature of the test tube. The concentration of the substrate is much higher than the concentration of the enzyme. Can you use the Michaelis-Menten [mi-KAY-lis MEN-tuhn] equation to model the enzyme kinetics of this reaction, and why or why not?\n\nW) Yes, because the conditions are changing\n\nX) No, because the conditions are changing\n\nY) Yes, because the enzyme activity remains unchanged\n\nZ) No, because the enzyme activity remains unchanged","Completion":"X) NO, BECAUSE THE CONDITIONS ARE CHANGING"} {"Type":"TOSS-UP\n\nMATH Multiple Choice","Prompt":"If A and B are integers, consider the 2 by 2 matrix with first row 2, A and second row B, 3. For how many different values of A can the matrix be singular?\n\nW) 2\n\nX) 4\n\nY) 8\n\nZ) 10","Completion":"Y) 8"} {"Type":"TOSS-UP\n\nENERGY Short Answer","Prompt":"In the U.S., ethanol is produced by a fermentation process involving corn. What is the cool season legume that can be used to produce biodiesel?","Completion":"SOYBEANS"} {"Type":"TOSS-UP\n\nCHEMISTRY Short Answer","Prompt":"Identify all of the following three techniques that could be used to determine the size of an unknown compound: affinity chromatography; mass spectrometry; electron paramagnetic resonance.","Completion":"2 (ACCEPT: MASS SPECTROMETRY)"} {"Type":"BONUS\n\nCHEMISTRY Multiple Choice","Prompt":"Which of the following qualitative organic tests is NOT matched correctly to a functional group for which it predominantly tests?\n\nW) Biuret test and amides [by-yuh-RET]\n\nX) Lucas reagent and alcohols\n\nY) Tollens' reagent and carboxylic acids\n\nZ) Benedict's test and reducing sugars","Completion":"Y) TOLLENS' REAGENT AND CARBOXYLIC ACIDS"} {"Type":"TOSS-UP\n\nBIOLOGY Short Answer","Prompt":"Sodium dodecyl [duh-DES-uh] sulfate is a detergent often used to dissolve molecules. In SDS-PAGE, what type of molecule is SDS used to dissolve?","Completion":"PROTEINS"} {"Type":"BONUS\n\nBIOLOGY Multiple Choice","Prompt":"Beriberi is caused by deficiency in which of the following vitamins?\n\nW) Thiamine [THY-uh-min]\n\nX) Retinol\n\nY) Ascorbic Acid\n\nZ) Cobalamin [koh-BAL-uh-min]","Completion":"W) THIAMINE"} {"Type":"TOSS-UP\n\nEARTH AND SPACE Short Answer","Prompt":"What is the form of equilibrium [ee-kwuh-LIB-ree-uhm] between pressure and gravity in a stellar interior?","Completion":"HYDROSTATIC EQUILIBRIUM (ACCEPT: HYDROSTATIC)"} {"Type":"TOSS-UP\n\nENERGY Multiple Choice","Prompt":"Catalysts do not get consumed in reactions; however, platinum in hydrogen fuel cells can become inactive when poisoned by which of the following compounds?\n\nW) Nitrogen\n\nX) Hydrogen\n\nY) Hydrogen peroxide\n\nZ) Carbon monoxide","Completion":"Z) CARBON MONOXIDE"} {"Type":"TOSS-UP\n\nMATH Short Answer","Prompt":"What is the absolute value of the difference between the sum of the first 2014 even counting numbers and the first 2014 odd counting numbers?","Completion":"2014"} {"Type":"TOSS-UP\n\nPHYSICS Multiple Choice","Prompt":"If Z = atomic number and A = mass number, the radius of an approximately spherical nucleus [NOO-klee-uhs] is proportional to which of the following?\n\nW) Z3\n\nX) A1\/3\n\nY) A3\n\nZ) Z1\/3","Completion":"X) A1\/3"} {"Type":"TOSS-UP\n\nBIOLOGY Multiple Choice","Prompt":"Moths are nocturnal, have a good sense of smell, and are also pollinators of a variety of plant species, many of which have evolved to better suit the needs of their primary pollinators. Which of the following traits would you expect to find in flowers of moth-pollinated plants?\n\nW) Heavy, musky odors\n\nX) Dangling flowers\n\nY) Large, red flowers\n\nZ) Blue or purple flowers","Completion":"W) HEAVY, MUSKY ODORS"} {"Type":"BONUS\n\nBIOLOGY Multiple Choice","Prompt":"If you lower the intracellular levels of fructose 2, 6 biphosphate, what will happen to the rate of gluconeogenesis [gloo-koh-nee-uh-JEN-uh-sis] and glycolysis [gly-KOL-uh-sis], respectively?\n\nW) Gluconeogenesis speeds up, glycolysis slows down\n\nX) Gluconeogenesis slows down, glycolysis speeds up\n\nY) Gluconeogenesis and glycolysis both slow down\n\nZ) Glycolysis speeds up and gluconeogenesis halts","Completion":"W) GLUCONEOGENESIS SPEEDS UP, GLYCOLYSIS SLOWS DOWN"} {"Type":"TOSS-UP\n\nCHEMISTRY Multiple Choice","Prompt":"Which of the following statements concerning chemical kinetics is NOT true?\n\nW) Catalysts of a reaction can appear in the reaction's rate law\n\nX) The reciprocal of concentration versus time will be linear for a first order reaction\n\nY) For a reaction, the equilibrium constant is the forward rate constant over the reverse rate constant\n\nZ) The half-life of a zero-th order reaction is proportional to initial concentration of the reactant","Completion":"X) THE RECIPROCAL OF CONCENTRATION VERSUS TIME WILL BE LINEAR FOR A FIRST ORDER REACTION"} {"Type":"BONUS\n\nCHEMISTRY Multiple Choice","Prompt":"Which of the following species has an unpaired electron, according to the VSEPR [ves-per] model?\n\nW) |F\\({}{3}\\)[I-F-3]\n\nX) IF\\({}{5}\\)[I-F-5]\n\nY) [BrF\\({}{5}\\)] [negative charge on B-R-F-5]\n\nZ) [BrF\\({}{3}\\)]\\({}^{2}\\)[negative two charge on B-R-F-3]","Completion":"Y) [BrF\\({}{5}\\)]\\({}^{\\cdot}\\)"} {"Type":"TOSS-UP\n\nMATH Multiple Choice","Prompt":"In order to find the average size of families in a particular town, a surveyor took a simple random sample of individuals and asked each of them how many people were in his or her family. Which of the following is TRUE regarding the surveyor's findings?\n\nW) The surveyor's calculated average will likely be greater than the actual average\n\nX) The surveyor's calculated average will likely be less than the actual average\n\nY) The surveyor's calculated average will likely be equal to the actual average\n\nZ) It is equally likely that the surveyor's calculated average will be less than or greater than the actual average","Completion":"W) THE SURVEYOR'S CALCULATED AVERAGE WILL LIKELY BE GREATER THAN THE ACTUAL AVERAGE"} {"Type":"TOSS-UP\n\nPHYSICS Short Answer","Prompt":"What group of subatomic particles, known to be emitted by the\n\nreactions that take place in the Sun, were recently confirmed to oscillate between three flavors,\n\nexplaining the discrepancy between expected and observed numbers of these particles emitted by the Sun?","Completion":"NEUTRINOS"} {"Type":"TOSS-UP\n\nEARTH AND SPACE Multiple Choice","Prompt":"Which of the following minerals are present in politic rocks, resulting from the metamorphism of aluminous sedimentary rocks such as shales and mudrocks?\n\nW) Sphalerite [SFAL-uh-ryt]\n\nX) Calcite\n\nY) Quartz\n\nZ) Garnet","Completion":"W) SPHALERITE"} {"Type":"BONUS\n\nEARTH AND SPACE Multiple Choice","Prompt":"Which of the following best describes cataclastic metamorphism?\n\nW) Ultrahigh pressures generated by impact with an extra-terrestrial body\n\nX) High temperature alteration accompanied by moderate pressures in the presence of\n\nhydrothermal fluids\n\nY) Mechanical deformation that can result from rocks sliding along a shear zone\n\nZ) Shallow burial with temperatures exceeding 300\\({}^{\\circ}\\)C, similar to diagenesis [dy-uh-JEN-uh-sis]","Completion":"Y) MECHANICAL DEFORMATION THAT CAN RESULT FROM ROCKS SLIDING"} {"Type":"TOSS-UP\n\nCHEMISTRY Multiple Choice","Prompt":"Which of the following properties is generally characteristic of metals?\n\nW) High heat conductivity\n\nX) Low electrical conductivity\n\nY) High first ionization energies\n\nZ) Tendency to form negative ions","Completion":"W) HIGH HEAT CONDUCTIVITY"} {"Type":"BONUS\n\nCHEMISTRY Short Answer","Prompt":"What separation technique would effectively separate a 50:50 mixture of ethanol and water?","Completion":"FRACTIONAL DISTILLATION (ACCEPT: DISTILLATION)"} {"Type":"TOSS-UP\n\nBIOLOGY Multiple Choice","Prompt":"A man with an X-linked dominant trait has children with a female who is homozygous [hoh-muh-ZY-guhs] recessive for the trait. Which of the following proportions of his offspring will be affected by the trait?\n\nW) All of his children, male or female\n\nX) All his male children\n\nY) All his female children\n\nZ) Half of his female children","Completion":"Y) ALL HIS FEMALE CHILDREN"} {"Type":"BONUS\n\nBIOLOGY Short Answer","Prompt":"Name four of the five structural characteristics shared by all Chordata at some point in their life cycle?","Completion":"any 4 of DORSAL HOLLOW NERVE CORD (ACCEPT: DORSAL NERVE CORD), NOTOCHORD, GILL-SLITS, POST ANAL FAIL (ACCEPT: TAIL), ENDOSTYLE"} {"Type":"TOSS-UP\n\nMATH Multiple Choice","Prompt":"How many two digit positive integers have at least one 6 as a digit?","Completion":"W) 17 X) 18 Y) 19 Z) 20 X) 18"} {"Type":"TOSS-UP\n\nEARTH AND SPACE Short Answer","Prompt":"Which element was first discovered by observing its spectral lines in the solar atmosphere?","Completion":"HELIUM"} {"Type":"TOSS-UP\n\nPHYSICS Multiple Choice","Prompt":"A field emission microscope requires which of the following?\n\nW) A conductor with a very fine point\n\nX) High energy alpha particles\n\nY) A light source with a wavelength shorter that the smallest observable feature size\n\nZ) A low amplitude, time-varying magnetic field","Completion":"W) A CONDUCTOR WITH A VERY FINE POINT"} {"Type":"TOSS-UP\n\nCHEMISTRY Multiple Choice","Prompt":"If an endothermic reaction is at equilibrium [ee-kwwth-LIB-reet-uhm], how will the equilibrium shift if the temperature is raised?\n\nW) Toward the reactants\n\nX) Toward the products\n\nY) Toward both the reactants and the products\n\nZ) It will not shift in either direction","Completion":"X) TOWARD THE PRODUCTS"} {"Type":"TOSS-UP\n\nBIOLOGY Multiple Choice","Prompt":"In the lab, you engineer a gene so that it is specifically expressed in pancreatic cells. Which of the following aspects of the gene did you likely adjust?\n\nW) Intron sequences\n\nX) 5'[5 prime] UTR sequences\n\nY) Translational start sequences\n\nZ) Promoter sequences","Completion":"Z) PROMOTER SEQUENCES"} {"Type":"TOSS-UP\n\nMATH Short Answer","Prompt":"How many edges does a hexagonal prism have?","Completion":"18"} {"Type":"TOSS-UP\n\nEARTH AND SPACE Multiple Choice","Prompt":"Which of the following aberrations only occurs in refracting telescopes?\n\nW) Chromatic aberration\n\nX) Spherical aberration\n\nY) Coma\n\nZ) Astigmatism","Completion":"W) CHROMATIC ABERRATION"} {"Type":"BONUS\n\nEARTH AND SPACE Multiple Choice","Prompt":"Which of the following is NOT true about tides?\n\nW) They are long-period waves that move through the oceans in response to forces exerted by the Moon and Sun\n\nX) They originate in the oceans and progress toward the coastlines\n\nY) A horizontal movement of water often accompanies the rising and falling of the tide\n\nZ) The weakest currents occur before or near the time of the high and low tides","Completion":"X) THEY ORIGINATE IN THE OCEANS AND PROGRESS TOWARD THE COASTLINES"} {"Type":"TOSS-UP\n\nPHYSICS Short Answer","Prompt":"What are the units of the dielectric constant?","Completion":"UNIT-LESS (ACCEPT: HAS NO UNIT)"} {"Type":"BONUS\n\nPHYSICS Short Answer","Prompt":"A parallel plate capacitor has an area of 2 square meters and plate separation of 1 millimeter. If the electric field between the plates is 10 volts per meter, and given 9 x 10\\({}^{\\text{-}12}\\) coulombs squared over newtons meter squared as \\(\\varepsilon{0}\\)[epsilon naught], providing your answer in scientific notation with one significant figure, what is the energy in joules stored by the capacitor?","Completion":"9 x 10\\({}^{\\text{-}13}\\)"} {"Type":"TOSS-UP\n\nENERGY Multiple Choice","Prompt":"Growth in the production of what commodity has resulted in an excess of glycerine [GLIS-er-in]?\n\nW) Gasoline\n\nX) Natural gas\n\nY) Ethanol\n\nZ) Biodiesel","Completion":"Z) BIODIESEL"} {"Type":"TOSS-UP\n\nCHEMISTRY Short Answer","Prompt":"Give the name of the group of elements that have high melting points, are electropositive, and are frequently used in lasers and television sets.","Completion":"LANTHANIDES"} {"Type":"TOSS-UP\n\nBIOLOGY Multiple Choice","Prompt":"Which of the following is NOT a common method by which ion channels are gated?\n\nW) Osmolarity\n\nX) Voltage\n\nY) Mechanical\n\nZ) Ligand","Completion":"W) OSMOLARITY"} {"Type":"TOSS-UP\n\nMATH Short Answer","Prompt":"The graph of the following equation is an example of what conic section: \\(2x^{2}=2y^{2}+13\\)?","Completion":"HYPERBOLA"} {"Type":"TOSS-UP\n\nEARTH AND SPACE Short Answer","Prompt":"In hydrostatic equilibrium [ee-kwuh-LIB-ree-uhm], which two forces are balanced against each other?","Completion":"PRESSURE AND GRAVITY"} {"Type":"BONUS\n\nEARTH AND SPACE Multiple Choice","Prompt":"Estuaries off the coast of which of the following states demonstrate the greatest nutrient pollution?\n\nW) Louisiana\n\nX) Oregon\n\nY) Maine\n\nZ) Texas","Completion":"W) LOUISIANA"} {"Type":"TOSS-UP\n\nBIOLOGY Multiple Choice","Prompt":"Which of the following best describes actin treadmilling?\n\nW) New actin polymers attach to the minus end of the actin filament\n\nX) The actin filament remains static with no addition or loss of subunits\n\nY) One end of an actin filament has a net addition of actin polymers while the other end has a net loss of actin polymers\n\nZ) One end of the actin filament adds and loses polymers at an equal rate","Completion":"Y) ONE END OF THE ACTIN FILAMENT HAS A NET ADDITION OF ACTIN"} {"Type":"BONUS\n\nBIOLOGY Multiple Choice","Prompt":"When actin reaches its steady state in a cell, which of the following is occurring?\n\nW) The actin filament is no longer adding new subunits or losing existing subunits\n\nX) The actin filament is constantly adding new and losing existing subunits\n\nY) The actin filament forms new disulfide bonds within itself\n\nZ) The actin filament forms new alpha helix structures to maintain its shape","Completion":"X) THE ACTIN FILAMENT IS CONSTATLY ADDING NEW AND LOSING"} {"Type":"TOSS-UP\n\nCHEMISTRY Multiple Choice","Prompt":"A reaction is carried out in a constant pressure calorimeter. Appropriate measurements are taken and the energy for the change is calculated. The quantity of heat exchanged under these conditions leads to the calculation of which of the following state functions?\n\nW) \\(\\Delta q\\) [defta Q]\n\nX) \\(\\Delta E\\)\n\nY) \\(\\Delta H\\)\n\nZ) \\(\\Delta T\\)","Completion":"Y) \\(\\Delta H\\)"} {"Type":"BONUS\n\nCHEMISTRY Short Answer","Prompt":"Identify the number of pi bonds, respectively, in the following:\n\nformaldehyde [for-MAL-duh-hyd], sulfate ion, acetylene [uh-SET-l-een].","Completion":"1) ONE, 2) TWO, 3) TWO"} {"Type":"TOSS-UP\n\nENERGY Multiple Choice","Prompt":"Which of the following materials is a common catalyst in commercial polymer electrolyte membrane fuel cells?\n\nW) Platinum\n\nX) Carbon\n\nY) Copper\n\nZ) Silver","Completion":"W) PLATINUM"} {"Type":"BONUS\n\nENERGY Short Answer","Prompt":"In September 2011, an experiment at CERN appeared to show neutrinos traveling at a speed faster than light. In March of 2012, these results were disproved. However, if the experiment had been accurate, it would have violated what physical principle?","Completion":"LORENTZ INVARIANCE"} {"Type":"TOSS-UP\n\nCHEMISTRY Multiple Choice","Prompt":"Assuming a temperature where the net magnetic moment is nonzero, which of the following types of magnetism will have the weakest external field?\n\nW) Antiferromagnetism\n\nX) Ferromagnetism\n\nY) Paramagnetism\n\nZ) Diamagnetism","Completion":"W) ANTIFERROMAGNETISM"} {"Type":"BONUS\n\nCHEMISTRY Short Answer","Prompt":"Given the molar mass of silver as 107.87 grams per mole, how many moles of silver are contained in 1.806 x 10\\({}^{24}\\) atoms?","Completion":"3"} {"Type":"TOSS-UP\n\nBIOLOGY Multiple Choice","Prompt":"Which of the following is NOT an example of a human disease caused by a recessive allele that fails to produce a viable cellular protein?\n\nW) Turner's syndrome\n\nX) Tay-sachs [day-SAKS]\n\nY) Cystic fibrosis\n\nZ) Phenylketonuria [fen-I-kee-toh-NOOR-ee-uh]","Completion":"W) TURNER'S SYNDROME"} {"Type":"BONUS\n\nBIOLOGY Short Answer","Prompt":"What are two specific types of glycocalyx [gly-koh-KAY-liks] in bacteria?","Completion":"CAPSULE, SLIME LAYER"} {"Type":"TOSS-UP\n\nMATH Short Answer","Prompt":"If a family has four children, what is the probability of having exactly 3 girls?","Completion":"\\(\\%\\) (ACCEPT: 0.25 OR 25%)"} {"Type":"TOSS-UP\n\nEARTH AND SPACE Short Answer","Prompt":"What is the term for radiation emitted by an electron undergoing relativistic helical motion around a magnetic field?","Completion":"SYNCHROTRON"} {"Type":"TOSS-UP\n\nPHYSICS Multiple Choice","Prompt":"As an object falls to the Earth with air resistance present, what happens to the acceleration of the object?\n\nW) It remains a constant 9.8 meters per second squared X) It increases from 0 to 9.8 meters per second squared Y) It remains at a constant 0 meters per second squared Z) It decreases from 9.8 to 0 meters per second squared","Completion":"Z) IT DECREASES FROM 9.8 TO 0 METERS PER SECOND SQUARED"} {"Type":"TOSS-UP\n\nPHYSICS Multiple Choice","Prompt":"Often some of the strongest spectral lines in cold interstellar gas are produced by:\n\nW) neutral lithium\n\nX) ionized iron\n\nY) carbon and iron\n\nZ) calcium and sodium","Completion":"Z) CALCIUM AND SODIUM"} {"Type":"BONUS\n\nPHYSICS Short Answer","Prompt":"The cosmic background radiation discovered by Penzias and Wilson closely matches the radiation from a black body with what temperature Kelvin, to the nearest whole number?","Completion":"3"} {"Type":"TOSS-UP\n\nCHEMISTRY Multiple Choice","Prompt":"Which of the following salt solutions, all at 1.0 molar concentration, will have a pH value less than 7:\n\nW) NH\\({}{4}\\)Cl\n\nX) NaC\\({}{2}\\)H\\({}{3}\\)O\\({}{2}\\)\n\nY) KCl\n\nZ) LiBr","Completion":"W) NH\\({}{4}\\)Cl"} {"Type":"BONUS\n\nCHEMISTRY Short Answer","Prompt":"Name all of the following 4 compounds that exhibit hydrogen bonding: CH\\({}{4}\\); H\\({}{2}\\)S; NH\\({}{3}\\); CH\\({}{3}\\)OH","Completion":"NH\\({}{3}\\); CH\\({}{3}\\)OH"} {"Type":"TOSS-UP\n\nBIOLOGY Multiple Choice","Prompt":"In early experiments in the mid-1900's, the source and fate of oxygen involved in photosynthesis was mostly determined through experiments that used:\n\nW) animals in closed containers that would die unless a plant was introduced into the container\n\nX) a heavy isotope of oxygen as a tracer\n\nY) fluorescent labeling of various enzymes and organic products\n\nZ) simple accounting of the atomic mass of all the reactants and products","Completion":"X) A HEAVY ISOTOPE OF OXYGEN AS A TRACER"} {"Type":"TOSS-UP\n\nMATH Short Answer","Prompt":"If you roll 2 fair dice simultaneously, what is the probability, given as a fraction, that you will roll the number 6 on AT LEAST one die?","Completion":"11\/36"} {"Type":"TOSS-UP\n\nEARTH SCIENCE Multiple Choice","Prompt":"Which of the following is the BEST example of a non-point source pollution:\n\nW) leaching from a landfill\n\nX) a leaking septic tank\n\nY) fertilizer runoff from agricultural fields\n\nZ) chemical spills at a mining site","Completion":"Y) FERTILIZER RUNOFF FROM AGRICULTURAL FIELDS"} {"Type":"TOSS-UP\n\nGENERAL SCIENCE Multiple Choice","Prompt":"Some vitamin tablets contain high concentrations of zinc and must be formulated to include high levels of which of the following minerals in order to prevent an induced anemia:\n\nW) copper\n\nX) calcium\n\nY) magnesium\n\nZ) boron","Completion":"W) COPPER"} {"Type":"TOSS-UP\n\nCHEMISTRY Multiple Choice","Prompt":"Which of the following is typically TRUE of organic compounds:\n\nW) acetylene contains delocalized pi bonds\n\nX) the simplest carboxylic acid is ethanoic acid\n\nY) ketones have no hydrogen atoms directly attached to the carbonyl group\n\nZ) alcohols contain nonpolar hydroxyl groups","Completion":"Y) KETONES HAVE NO HYDROGEN ATOMS DIRECTLY ATTACHED TO THE CARBONYL GROUP"} {"Type":"BONUS\n\nCHEMISTRY Short Answer","Prompt":"Rounded to the nearest whole number, what is the percent of magnesium in magnesium carbonate, given the atomic mass of magnesium is 24 grams per mole?","Completion":"29"} {"Type":"TOSS-UP\n\nBIOLOGY Multiple Choice","Prompt":"In the famous experiment by Hershey and Chase that showed DNA was the genetic material, what did they use to transmit the DNA into E. coli:\n\nW) other E. coli\n\nX) electric shocks\n\nY) a blender\n\nZ) bacteriophage","Completion":"Z) BACTERIOPHAGE"} {"Type":"BONUS\n\nBIOLOGY Short Answer","Prompt":"In the Amanita mushroom, what is the name of the cell in which karyogamy and meiosis occur and what is the product of meiosis, respectively?","Completion":"CELL = BASIDIUM; PRODUCT = BASIDIOSPORE"} {"Type":"TOSS-UP\n\nMATH Multiple Choice","Prompt":"Which of the following are coterminal angles in standard position (read slowly):\n\nW) 550\\({}^{\\rm o}\\) and 185\\({}^{\\rm o}\\)\n\nX) 80\\({}^{\\rm o}\\) and -190\\({}^{\\rm o}\\)\n\nY) 1160\\({}^{\\rm o}\\) and 800\\({}^{\\rm o}\\)\n\nZ) 181\\({}^{\\rm o}\\) and -118\\({}^{\\rm o}\\)","Completion":"Y) 1160\\({}^{\\rm o}\\) AND 800\\({}^{\\rm o}\\)"} {"Type":"BONUS\n\nMATH Short Answer","Prompt":"Solve the following equation for \\(x\\), expressing your answer in simplest radical form over the real numbers: \\(x^{4}-9x^{2}+8=0\\)","Completion":"\\(\\pm\\,2\\sqrt{2}\\) AND \\(\\pm\\)1 (ACCEPT ALL: +2\\(\\sqrt{2}\\), -2\\(\\sqrt{2}\\), +1, -1)"} {"Type":"TOSS-UP\n\nEARTH SCIENCE Multiple Choice","Prompt":"Which of the following is the BEST example of an ocean current found in the intertidal littoral zone that flows seaward:\n\nW) rip current\n\nX) tidal bore\n\nY) the Antarctic Circumpolar Current\n\nZ) equatorial tidal current","Completion":"W) RIP CURRENT"} {"Type":"BONUS\n\nEARTH SCIENCE Multiple Choice","Prompt":"Which of the following is NOT true of isotherms:\n\nW) they are more widely spaced over the equatorial regions\n\nX) they are more closely spaced over North America in the summer than in the winter\n\nY) they tend to be linear in form in the Southern Hemisphere south of 40\\({}^{\\rm o}\\) latitude\n\nZ) they are lines that connect places with the same average temperature","Completion":"X) THEY ARE MORE CLOSELY SPACED OVER NORTH AMERICA IN THE SUMMER THAN IN THE WINTER"} {"Type":"TOSS-UP\n\nGENERAL SCIENCE Multiple Choice","Prompt":"Which of the following instruments is BEST used to image small, transparent and fluorescent specimens and obtain optical sections without the need for mechanical sectioning:\n\nW) X-ray microanalyzer\n\nX) scanning electron microscope\n\nY) transmission electron microscope\n\nZ) confocal laser scanning microscope","Completion":"Z) CONFOCAL LASER SCANNING MICROSCOPE"} {"Type":"TOSS-UP\n\nASTRONOMY Short Answer","Prompt":"A sidereal drive is a motor on a telescope that turns it in a certain direction to keep the telescope aimed at a certain celestial object. Giving your answer as north, south, east or west, in what direction does it turn?","Completion":"WEST"} {"Type":"TOSS-UP\n\nPHYSICS Multiple Choice","Prompt":"In a compound gearing arrangement where a 40-tooth gear drives an 80-tooth gear which is directly connected to a 50-tooth gear that drives a 100-tooth gear, the total reduction is a factor of:\n\nW) 2 X) 2.5 Y) 4 Z) 5","Completion":"Y) 4"} {"Type":"BONUS\n\nPHYSICS Multiple Choice","Prompt":"Elastic deformation is BEST described as:\n\nW) the linear portion of a stress\/strain curve X) the ultimate strength of a material Y) 0.02% elongation Z) permanent deformation","Completion":"W) THE LINEAR PORTION OF A STRESS\/STRAIN CURVE"} {"Type":"TOSS-UP\n\nCHEMISTRY Multiple Choice","Prompt":"Which of the following elements has the largest second ionization energy:\n\nW) fluorine X) nitrogen Y) boron Z) lithium","Completion":"Z) LITHIUM (Solution: core electrons must be removed)"} {"Type":"","Prompt":"BONUS","Completion":"16) CHEMISTRY Short Answer Arrange the following compounds in order of INCREASING ionic characteristic: HCl; LiBr; CO; KF CO; HCl; LiBr; KF"} {"Type":"TOSS-UP\n\nBIOLOGY Multiple Choice","Prompt":"Which of the following are MOST often used to introduce foreign genes into mammalian genomes:\n\nW) sliceosomes (read as: sly-see-oh-somz)\n\nX) zinc fingers\n\nY) plasmids\n\nZ) telomeres","Completion":"Y) PLASMIDS"} {"Type":"BONUS\n\nBIOLOGY Multiple Choice","Prompt":"Which of the following regulate development in animals, plants and fungi, and were first discovered in Drosophila:\n\nW) homeobox (read as: HOME-ee-oh-box) genes\n\nX) p53\n\nY) shock genes\n\nZ) oncogenes","Completion":"W) HOMEOBOX GENES"} {"Type":"TOSS-UP\n\nMATH Short Answer","Prompt":"What function is the inverse of \\(y=3x+4\\)?","Completion":"\\(y=\\dfrac{x-4}{3}\\)"} {"Type":"BONUS\n\nMATH Short Answer","Prompt":"Giving your answer as an ordered pair, find the solution of the system of the following 2 equations: \\(2x+3y=8\\), and \\(3x-2y=-1\\)","Completion":"\\((1,2)\\) (ACCEPT: 1 AND 2)"} {"Type":"TOSS-UP\n\nEARTH SCIENCE Multiple Choice","Prompt":"Orthoclase feldspar primarily weathers by which of the following processes to form clay minerals:\n\nW) abrasion\n\nX) hydrolysis\n\nY) exfoliation\n\nZ) dissolution","Completion":"X) HYDROLYSIS"} {"Type":"TOSS-UP\n\nGENERAL SCIENCE Short Answer","Prompt":"Name all of the following 4 reagents that would NOT be labeled as a highly flammable substance:\n\nmethanol; diethyl ether; 30% hydrogen peroxide; benzene","Completion":"30% HYDROGEN PEROXIDE"} {"Type":"TOSS-UP\n\nASTRONOMY Short Answer","Prompt":"If the surface temperature of a given star increased from 20,000K to 60,000K, how many times as much energy would the star release?","Completion":"81 (ACCEPT: 3\\({}^{4}\\))"} {"Type":"TOSS-UP\n\nPHYSICS Short Answer","Prompt":"What specific type of semiconductor is produced by adding phosphorus to silicon?","Completion":"N-TYPE"} {"Type":"TOSS-UP\n\nBIOLOGY Short Answer","Prompt":"Calculate the atom percent composition, rounded to the nearest whole number, for each atom in acetic acid:","Completion":"\\[\\text{}\\ \\ \\text{C}=25\\%;\\ \\ \\text{H}=50\\%;\\ \\ \\text{O}=25\\%\\]"} {"Type":"TOSS-UP\n\nMATH Multiple Choice","Prompt":"In 2014, January 1st fell on a Wednesday. What is the next year that January 1st will fall on a Wednesday?\n\nW) 2015\n\nX) 2019\n\nY) 2020\n\nZ) 2025","Completion":"Y) 2020"} {"Type":"BONUS\n\nMATH Short Answer","Prompt":"The sums of three whole numbers taken in pairs are 11, 13, and 16. What is the product of these three numbers?","Completion":"252"} {"Type":"TOSS-UP\n\nBIOLOGY Multiple Choice","Prompt":"During which of the following phases of an action potential do voltage-gated sodium channels begin to close?\n\nW) Rising phase\n\nX) Falling phase\n\nY) After hyperpolarization\n\nZ) Refractory period","Completion":"X) FALLING PHASE"} {"Type":"BONUS\n\nBIOLOGY Multiple Choice","Prompt":"In humans, in addition to transferring antibodies, colostrum also stimulates the development of the neonatal gut through the transference of which of the following?\n\nW) Pheromones\n\nX) Non-heritable mutations\n\nY) Beta-galactose\n\nZ) Growth factors","Completion":"Z) GROWTH FACTORS"} {"Type":"TOSS-UP\n\nCHEMISTRY Multiple Choice","Prompt":"Halogenation [hal-uh-juh-NAY-shuhn] will occur fastest between methane and the radical of which of the following elements?\n\nW) Chlorine\n\nX) lodine\n\nY) Fluorine\n\nZ) Bromine","Completion":"Y) FLUORINE"} {"Type":"BONUS\n\nCHEMISTRY Short Answer","Prompt":"In terms of the ideal gas constant R, what is the constant-pressure heat capacity, per mole, of a monatomic ideal gas?","Completion":"5\/2 R (ACCEPT: 2.5 R)"} {"Type":"TOSS-UP\n\nPHYSICS Multiple Choice","Prompt":"An antihydrogen atom is made of what two components?\n\nW) An antiproton and a positron\n\nX) An antiproton and an electron\n\nY) A proton and a positron\n\nZ) A proton and a neutrino","Completion":"W) AN ANTIPROTON AND A POSITRON"} {"Type":"BONUS\n\nPHYSICS Multiple Choice","Prompt":"A capillary tube with an inner radius of 0.1 millimeter is partially dipped in a liquid with a surface tension of 0.1 newtons per meter, with approximately 0 degrees of contact angle. The density of the liquid is approximately the same as that of water. Calculate the liquid rise in the tube.\n\nW) 2 millimeter\n\nX) 2 centimeter\n\nY) 20 centimeter\n\nZ) 2 meter","Completion":"Y) 20 cm"} {"Type":"TOSS-UP\n\nEARTH AND SPACE Multiple Choice","Prompt":"At a latitude of 45 degrees, which of the following is closest to the percentage of astronomical sources that are circumpolar?\n\nW) 0%\n\nX) 30%\n\nY) 50%\n\nZ) 100%","Completion":"X) 30%"} {"Type":"BONUS\n\nEARTH AND SPACE Multiple Choice","Prompt":"A poikiloblastic [poi-kil-uh-BLAS-tik] texture is characterized by which of the following?\n\nW) Idioblasts embedded within a fine matrix\n\nX) Homogeneously [hoh-muh-JEE-nee-uhs-il] xenoblastic with little to no evidence of additional alteration minerals\n\nY) Fine, phyllitic [fi-LIT-ik] foliation accompanied by porphyroblasts [por-Fl-roh-blasts]\n\nZ) Inclusions from other minerals enveloped during porphyroblast growth","Completion":"Z) INCLUSIONS FROM OTHER MINERALS ENVELOPED DURING"} {"Type":"TOSS-UP\n\nBIOLOGY Multiple Choice","Prompt":"Which of the following phases of meiosis [my-OH-sis] is longest?\n\nW) Propphase I\n\nX) Propphase II\n\nY) Metaphase I\n\nZ) Anaphase II","Completion":"W) PROPHASE I"} {"Type":"BONUS\n\nBIOLOGY Short Answer","Prompt":"Arrange the following three steps of cell respiration from lowest to highest in terms of net NADH molecules produced per one molecule of glucose entering respiration: glycolysis [gly-KOL-uh-sis], electron transport chain, Krebs cycle.","Completion":"2, 1, 3 (ACCEPT: ELECTRON TRANSPORT CHAIN, GLYCOYSIS, KREBS CYCLE)"} {"Type":"TOSS-UP\n\nENERGY Multiple Choice","Prompt":"What major rock type is the medium for fracking?\n\nW) Schist [SHIFT]\n\nX) Shale\n\nY) Basalt\n\nZ) Limestone","Completion":"X) SHALE"} {"Type":"TOSS-UP\n\nCHEMISTRY Multiple Choice","Prompt":"Which of the following is the full list of products produced by the mild heating of sodium bicarbonate at temperatures of about 80 degrees Celsius?\n\nW) Sodium oxide and carbon dioxide\n\nX) Sodium peroxide and carbon dioxide\n\nY) Sodium carbonate, water, and carbon dioxide\n\nZ) Sodium carbonate and carbon dioxide","Completion":"Y) SODIUM CARBONATE, WATER, AND CARBON DIOXIDE"} {"Type":"TOSS-UP\n\nPHYSICS Multiple Choice","Prompt":"Which of the following does not interact with a magnetic field?\n\nW) A magnet at rest\n\nX) Moving electric charges\n\nY) A moving magnet\n\nZ) Electric charges at rest","Completion":"Z) ELECTRIC CHARGES AT REST"} {"Type":"TOSS-UP\n\nBIOLOGY Multiple Choice","Prompt":"Which of the following structures would have been most likely absent from the last shared common ancestor of osteichthyans [os-tee-IK-thee-uhns] and chondrichthyans [kan-DRIK-thee-ez]?\n\nW) Teeth\n\nX) Gills\n\nY) Scales\n\nZ) Swim bladder","Completion":"Z) SWIM BLADDER"} {"Type":"TOSS-UP\n\nEARTH AND SPACE Multiple Choice","Prompt":"Which of the following composes most of the insoluble residues of limestones?\n\nW) Calcite matrix\n\nX) Crystalline quartz\n\nY) Chert nodules [churt]\n\nZ) Clay minerals","Completion":"Y) CHERT NODULES"} {"Type":"TOSS-UP\n\nCHEMISTRY Short Answer","Prompt":"The van der Waals equation is an equation of state for real gases that contains two gas-dependent parameters, \\(a\\) and \\(b\\). What state function from the ideal gas equation does the \\(a\\) parameter modify in the van der Waals equation?","Completion":"PRESSURE (ACCEPT: P)"} {"Type":"## BONUS\n\nMATH Short Answer","Prompt":"What is the smallest positive integer that can be added to 2500 to get a perfect square?","Completion":"101"} {"Type":"## TOSS-UP\n\nENERGY Multiple Choice","Prompt":"A 100 mile 765 kilovolt line carrying 1000 megawatts of energy can have losses ranging from 1.1% to 0.5%. Which of the following best approximates the losses as a ratio to the 1.0% of a 345 kilovolt line carrying the same load across the same distance?\n\nW) 1\/10\n\nX) 1\/4\n\nY) 4\/1\n\nZ) 10\/1","Completion":"Y) 4\/1"} {"Type":"## BONUS\n\nENERGY Short Answer","Prompt":"If a single electric distribution line has a resistance of 1 ohm per 1000 meters and current at 50 amps, what will be the voltage reduction over a 10 kilometer run?","Completion":"500"} {"Type":"TOSS-UP\n\nBIOLOGY Short Answer","Prompt":"Identify all of the following three neurotransmitters that are considered to have only excitatory actions: acetylcholine [uh-seet-I-KOH-Ieen], GABA, glutamate.","Completion":"3 (ACCEPT: GLUTAMATE)"} {"Type":"BONUS\n\nBIOLOGY Short Answer","Prompt":"What specific cell type in the stomach is responsible for production of hydrochloric acid in response to stimulation by gastrin?","Completion":"PARIETAL CELLS"} {"Type":"TOSS-UP\n\nCHEMISTRY Multiple Choice","Prompt":"Which of the following organic functional groups does NOT exhibit sp\\({}^{2}\\)[s-p-2] hybridization?\n\nW) Ketones\n\nX) Aldehydes [AL-duh-hyds]\n\nY) Amides\n\nZ) Amines","Completion":"Z) AMINES"} {"Type":"BONUS\n\nCHEMISTRY Multiple Choice","Prompt":"Which of the following statements is true regarding the chemistry of the alkaline earth elements?\n\nW) Compounds of beryllium have the most covalent character\n\nX) Melting point increases as you go down the group\n\nY) Atomic radius decreases as you go down the group\n\nZ) Barium has a distinct red flame test color","Completion":"W) COMPOUNDS OF BERYLLIUM HAVE THE MOST COVALENT CHARACTER"} {"Type":"TOSS-UP\n\nMATH Multiple Choice","Prompt":"Which of the following statements is TRUE?\n\nW) \\(-7\\) is not a prime number\n\nX) \\(e^{i\\pi}=-e^{-i\\pi}\\) [e to the power of quantity I times pi equals negative e to the power of quantity negative I times pi]\n\nY) All pairs of perpendicular lines in the Cartesian plane have slopes that are negative reciprocals of each other\n\nZ) There is no number \\(a\\) such that \\(\\log(-a)\\)[log of negative a] exists.","Completion":"W) \\(-7\\) IS NOT A PRIME NUMBER"} {"Type":"BONUS\n\nMATH Short Answer","Prompt":"If a circle is divided into 8 sectors whose central degree angles are integers that form an arithmetic sequence with first term 3, what is the common difference for the sequence?","Completion":"12"} {"Type":"TOSS-UP\n\nPHYSICS Short Answer","Prompt":"Assuming the heat of fusion for water is 80 calories per gram, what is the increase in entropy, in calories per Kelvin, for a 273 gram block of ice at 0 degrees\n\nCelsius when it melts?","Completion":"80"} {"Type":"BONUS\n\nPHYSICS Multiple Choice","Prompt":"An \\(\\alpha\\)-particle [alpha particle] and a proton, both initially at rest, are accelerated through a potential difference of \\(10^{5}\\) kilovolts. Find the ratio (K\\({}{\\alpha}\\)\/K\\({}{\\rho}\\)) [k alpha over k proton] of their final kinetic energies.\n\nW) 1\/4\n\nX) 1\/2\n\nY) 2\n\nZ) 4","Completion":"Y) 2"} {"Type":"TOSS-UP\n\nEARTH AND SPACE Multiple Choice","Prompt":"What will occur if a white dwarf in a binary exceeds its Roche [ROSH] lobe?\n\nW) Its companion will transfer mass to it until it shrinks below its Roche lobe\n\nX) It will form a planetary nebula\n\nY) It will continue to expand and ablate onto its companion\n\nZ) It will transfer matter to its companion until it shrinks below its Roche lobe","Completion":"Z) IT WILL TRANSFER MATTER TO ITS COMPANION UNTIL IT SHRINKS BELOW ITS ROCHE LOBE"} {"Type":"BONUS\n\nEARTH AND SPACE Multiple Choice","Prompt":"Which of the following describes the carbonate\n\ncompensation depth (CCD)?\n\nW) Occurring between 2,000-3,000 m\n\nX) Varying by latitude: deeper near the equator, shallower near the poles\n\nY) Restricted to waters near the poles in both hemispheres\n\nZ) The depth at which carbonates start to form at nearly two times the rate of formation in shallower water","Completion":"X) VARYING BY LATTITUDE: DEEPER NEAR THE EQUATOR, SHALLOWER NEAR THE POLES"} {"Type":"TOSS-UP\n\nCHEMISTRY Multiple Choice","Prompt":"Which of the following best describes the elementary kinetic\n\nstep where a proton is transferred directly between an ammonium ion and water?\n\nW) Unimolecular\n\nX) Bimolecular\n\nY) Trimolecular\n\nZ) Tetramolecular","Completion":"X) BIMOLECTULAR"} {"Type":"BONUS\n\nCHEMISTRY Short Answer","Prompt":"One liter of a saturated solution of silver chromate contains\n\n0.0002 moles of the silver cation [KAT-eye-uhn] and 0.00003 moles of the chromate ion. To two significant figures and in scientific notation, what is the Ksp of silver chromate?","Completion":"1.2 x 10\\({}^{\\text{-}12}\\)"} {"Type":"TOSS-UP\n\nBIOLOGY Multiple Choice","Prompt":"Which of the following vitamin deficiencies causes the condition known as \"Pellagra\"?\n\nW) Vitamin B\\({}{3}\\)\n\nX) Vitamin B\\({}{6}\\)\n\nY) Vitamin B\\({}{2}\\)\n\nZ) Vitamin B\\({}{1}\\)","Completion":"W) VITAMIN B\\({}{3}\\)"} {"Type":"TOSS-UP\n\nENERGY Multiple Choice","Prompt":"Energy radiates from a surface as a function of temperature to what power?\n\nW) 1\n\nX) 3\n\nY) 2\n\nZ) 4","Completion":"Z) 4"} {"Type":"TOSS-UP\n\nEARTH AND SPACE Short Answer","Prompt":"A torus of sulfur and sodium has been detected along the orbit of which of Jupiter's moons?","Completion":"IO"} {"Type":"TOSS-UP\n\nMATH Short Answer","Prompt":"Let \\(y=e^{x^{2}}\\)[e to the power of x squared]. Find \\(dy\/dx\\).","Completion":"\\(2xe^{x^{2}}\\)"} {"Type":"TOSS-UP\n\nCHEMISTRY Multiple Choice","Prompt":"What is the local VSEPR geometry of a fully substituted carbonyl carbon?\n\nW) Tetrahedral\n\nX) Trigonal pyramidal\n\nY) Trigonal planar\n\nZ) T-shaped","Completion":"Y) TRIGONAL PLANAR"} {"Type":"TOSS-UP\n\nPHYSICS Short Answer","Prompt":"A ball is dropped from rest at a height of 60 meters above the ground. If the potential energy is zero at the ground, then at what height in meters is the potential energy 60% of its initial potential energy?","Completion":"36"} {"Type":"TOSS-UP\n\nEARTH AND SPACE Multiple Choice","Prompt":"During an El Nifio event, the thermocline [THUR-muh-klyn] in the eastern equatorial Pacific is which of the following, compared to normal?\n\nW) Thicker\n\nX) Thinner\n\nY) The same\n\nZ) Either thicker or thinner depending on the salinity","Completion":"W) THICKER"} {"Type":"TOSS-UP\n\nBIOLOGY Short Answer","Prompt":"What wasteful process occurs when rubisco acts as an oxygenase [OK-si-juh-nays] instead of a carboxylase [kahr-BOK-suh-layz] during the Calvin cycle, decreasing the efficiency of photosynthesis and producing carbon dioxide?","Completion":"PHOTORESPIRATION"} {"Type":"BONUS\n\nBIOLOGY Multiple Choice","Prompt":"In contrast to the lysogenic [ly-suh-JEN-ik] cycle of a bacteriophage [bak-TEER-ee-uh-fayjl], the lytic cycle does which of the following?\n\nW) Integrates phage DNA into the host chromosome\n\nX) Involves the dissolution of host cells to release new phages\n\nY) Features prophages that can be excised from the host chromosome\n\nZ) Copies phage DNA when the cell divides","Completion":"X) INVOLVES THE DISSOLUTION OF HOST CELLS TO RELEASE NEW PHAGES"} {"Type":"TOSS-UP\n\nENERGY Multiple Choice","Prompt":"The Marcellus shale gas formation is located in which of the following states?\n\nW) Pennsylvania\n\nX) Texas\n\nY) Oklahoma\n\nZ) Wyoming","Completion":"W) PENNSYLVANIA"} {"Type":"BONUS\n\nENERGY Multiple Choice","Prompt":"Compounds of bismuth telluride are used in which of the following types of devices?\n\nW) Photovoltaic devices\n\nX) Computer chips\n\nY) Energy control devices\n\nZ) Thermoelectric devices","Completion":"Z) THERMOELECTRIC DEVICES7) MATH Short Answer Solve for \\(x\\): \\(\\ln(x+1)\\) - \\(1=0\\) [the natural log of open parenthesis x plus 1 close parenthesis minus 1 equals 0]."} {"Type":"TOSS-UP\n\nCHEMISTRY Multiple Choice","Prompt":"What are the products of the complete combustion of cyclohexane [sy-kluh-HEK-sayn]?\n\nW) Carbon monoxide, carbon trioxide and water\n\nX) Carbon dioxide and water\n\nY) Carbon monoxide and water\n\nZ) A variety of small hydrocarbons","Completion":"X) CARBON DIOXIDE AND WATER"} {"Type":"TOSS-UP\n\nPHYSICS Short Answer","Prompt":"What is the efficiency of a Carnot heat engine in which 500 joules of heat enters from the hot reservoir and 300 joules exits into the cold reservoir?","Completion":"40%"} {"Type":"BONUS\n\nPHYSICS Short Answer","Prompt":"A typical incandescent light bulb wastes 90% of its power as heat. Ignoring the contribution of the walls and any furniture, if you left a 100 watt incandescent bulb on in a 5 by 5 by 5 meter room for 100,000 seconds, how much would the temperature of the air in the room increase, in degrees Celsius? Assume, for simplicity, that the room is a closed thermodynamic system, the heat capacity of air is 1 joule per gram per degree, and the mass density of air in the room is 1.2 kilograms per cubic meter.","Completion":"60"} {"Type":"TOSS-UP\n\nEARTH AND SPACE Short Answer","Prompt":"Which class of supernova exhibits strong silicon lines but no hydrogen or helium lines?","Completion":"TYPE IA"} {"Type":"BONUS\n\nEARTH AND SPACE Multiple Choice","Prompt":"How does the Sound Fixing and Ranging (SOFAR) channel support our study of climate change?\n\nW) The deep SOFAR channel is populated with organisms extremely sensitive to ocean temperature change\n\nX) World War II submarines gathered rich and extensive data sets about the SOFAR channel that can be compared with current data\n\nY) Video monitoring of marine mammal mating behavior is useful because mating behavior in this region is impacted by temperature\n\nZ) Speed of sound is temperature-dependent in the SOFAR channel","Completion":"Z) SPEED OF SOUND IS TEMPERATURE-DEPENDENT IN THE SOFAR CHANNEL"} {"Type":"TOSS-UP\n\nBIOLOGY Multiple Choice","Prompt":"Which of the following organisms stores major food reserves as laminin and oil instead of starch?\n\nW) Green algae\n\nX) Brown algae\n\nY) Red algae\n\nZ) Flowering plants","Completion":"X) BROWN ALGAE"} {"Type":"TOSS-UP\n\nENERGY Multiple Choice","Prompt":"Which of the following explanations is used by scientists to justify the observed accelerating expansion of the universe?\n\nW) Dark matter\n\nX) Fast moving neutrinos\n\nY) Dark energy\n\nZ) Supernovae [soo-per-NOH-ve]","Completion":"Y) DARK ENERGY"} {"Type":"TOSS-UP\n\nPHYSICS Multiple Choice","Prompt":"A small rubber duck is held underwater at a depth of 10 meters, then released. When the duck reaches a depth of 5 meters, how much has the buoyant force on the duck changed?\n\nW) It has decreased by half\n\nX) It is unchanged\n\nY) It has doubled\n\nZ) It has quadrupled","Completion":"X) IT IS UNCHANGED"} {"Type":"BONUS\n\nPHYSICS Short Answer","Prompt":"Charged particles in a uniform magnetic field will undergo circular motion with a period of T = 2\\(\\pi\\)m\/(qB). The magnetic field of the Earth is roughly 10\\({}^{-4}\\) tesla, assuming it is uniform. Given the mass of a proton to be 1.6 x 10\\({}^{-27}\\) kilograms and its electric charge to be 1.6 x 10\\({}^{-19}\\) coulombs, providing your answer in scientific notation to one significant figure, what period in seconds would you measure for a proton?","Completion":"6 x 10\\({}^{-4}\\)"} {"Type":"TOSS-UP\n\nEARTH AND SPACE Multiple Choice","Prompt":"For the mass-radius relation of white dwarfs, M is proportional to R to what power?\n\nW) -1\/3\n\nX) 1\/3\n\nY) 1\n\nZ) 3","Completion":"W) -1\/3"} {"Type":"BONUS\n\nEARTH AND SPACE Multiple Choice","Prompt":"What is the primary mode of heat transport in a white dwarf?\n\nW) Convection\n\nX) Conduction\n\nY) Radiation\n\nZ) Advection","Completion":"X) CONDUCTION"} {"Type":"TOSS-UP\n\nBIOLOGY Multiple Choice","Prompt":"Which of the following marine fish groups reproduces ovoviviparovously [oh-voh-vy-vip-ER-uhs-lee]?\n\nW) Coelocanth [SEE-luh-kanth]\n\nX) Ratfish\n\nY) Ray\n\nZ) Lamprey","Completion":"W) COELOCANTH"} {"Type":"BONUS\n\nBIOLOGY Short Answer","Prompt":"What is the process in which a protein inhibits the function of a second protein by binding to the regulatory site of that second protein, thus altering its functionality?","Completion":"ALLOSTERIC REGULATION (ACCEPT: ALLOSTERIC INHIBITION)"} {"Type":"TOSS-UP\n\nCHEMISTRY Multiple Choice","Prompt":"Dr. Ecriecs, the evil genius, has released multiple dangerous weather balloons into the atmosphere, filled with his new concoction - Boomgas! If the density of Boomgas drops below one third of its density at room temperature, it will explode with the force of 1\/10\\({}^{\\rm th}\\) of an atomic bomb. Which of the following disposal methods would force the balloons to explode?\n\nW) Throwing weighted balloons into the sea\n\nX) Releasing the balloons into space\n\nY) Cryogenically storing the balloons in liquid nitrogen\n\nZ) Venting Boomgas from the balloons","Completion":"X) RELEASING THE BALLOONS INTO SPACE"} {"Type":"BONUS\n\nCHEMISTRY Multiple Choice","Prompt":"Consider the following reaction: 5H\\({}{2}\\)O\\({}{(y)}\\)+ S\\({}{2}\\)O\\({}{3}\\)\\({}^{2}\\)+ 4Cl\\({}{2}\\)\\({}{(aq)}\\)\\(\\rightarrow\\)[yields] 8Cl\\({}{(aq)}\\)+ 2SO\\({}{4}\\)\\({}^{2}\\)\\({}{(aq)}\\)+ 10H\\({}^{+}\\). What is the oxidizing agent in this reaction?\n\nW) Cl\n\nX) S\\({}{2}\\)O\\({}{3}\\)\\({}^{2}\\)-\n\nY) Cl\\({}{2}\\)\n\nZ) H\\({}{2}\\)O","Completion":"Y) Cl\\({}{2}\\)"} {"Type":"TOSS-UP\n\nENERGY Multiple Choice","Prompt":"How much energy is required to run a device with compressed air compared to running the same device with electricity?\n\nW) An equal amount\n\nX) Twice as much\n\nY) Four times as much\n\nZ) Six times as much","Completion":"Y) FOUR TIMES AS MUCH"} {"Type":"TOSS-UP\n\nPHYSICS Short Answer","Prompt":"What law describes the relationship between the direction of incident and refracted light rays?","Completion":"SNELL'S LAW"} {"Type":"TOSS-UP\n\nEARTH AND SPACE Multiple Choice","Prompt":"According to the United Nations, what is the global warming potential of methane on a 100 year time scale?\n\nW) 1\n\nX) 21\n\nY) 150\n\nZ) 310","Completion":"X) 21"} {"Type":"TOSS-UP\n\nBIOLOGY Multiple Choice","Prompt":"In one form of the disease called macular degeneration, eyesight decreases due to overproduction of new blood vessels (angiogenesis [an-jee-oh-JEN-uh-sis]) in the eye. Inhibition of a gene involved in angiogenesis can be achieved by introducing which of the following forms of complementary RNA molecule into the eye?\n\nW) siRNA\n\nX) Messenger RNA\n\nY) lncRNA\n\nZ) Transfer RNA","Completion":"W) siRNA"} {"Type":"BONUS\n\nBIOLOGY Multiple Choice","Prompt":"Enzyme A has a high K\\({}{m}\\) value and low V\\({}{max}\\) value relative to Enzyme B. This means Enzyme A, relative to Enzyme B has which of the following?\n\nW) Higher substrate affinity and a lower reaction rate\n\nX) Lower substrate affinity and a higher reaction rate\n\nY) Higher substrate affinity and higher reaction rate\n\nZ) Lower substrate affinity and lower reaction rate","Completion":"Z) LOWER SUBSTRATE AFFINITY AND LOWER REACTION RATE"} {"Type":"TOSS-UP\n\nCHEMISTRY Short Answer","Prompt":"What is the oxidation number of tin in SnS\\({}{2}\\)?","Completion":"+4"} {"Type":"BONUS\n\nCHEMISTRY Short Answer","Prompt":"What term describes the general process by which a racemate [ray-SEE-mayt] is separated into its component enantiomers [I-NAN-tee-uh-mers]?","Completion":"RESOLUTION (ACCEPT: CHEMICAL RESOLUTION)"} {"Type":"TOSS-UP\n\nBIOLOGY Multiple Choice","Prompt":"You suspect that Cylon, an alien, may be the parent of a human-alien hybrid baby. The alien's blood type happens to be homozygous [hoh-muh-ZY-guhs] for blood type P for purple, the allele of which is symbolized by I\\({}^{\\text{P}}\\)[feye superscript P]. As with other blood types, the P protein is co-dominant. If the human parent's blood type is A with a homozygous genotype, what blood phenotype would the baby have to be in order to be a confirmed human-alien hybrid baby?\n\nW) O\n\nX) AP\n\nY) P\n\nZ) A","Completion":"X) AP"} {"Type":"BONUS\n\nBIOLOGY Multiple Choice","Prompt":"How do bacteria protect their own genomic DNA from being cut by their restriction endonucleases [en-duh-NOO-klee-ays-is]?\n\nW) Genomic DNA of a bacterial species does not contain the recognition sequences for its own restriction endonucleases\n\nX) Bacteria apply protective methyl groups to DNA at sites recognized by their restriction endonucleases\n\nY) Bacteria have proteins associated with their genomic DNA, preventing the endonucleases from getting close enough to cut\n\nZ) Sequestration of chromosomal DNA in the nucleoid region of a bacterial cell protects it from being cut by restriction endonucleases","Completion":"X) BACTERIA APPLY PROTECTIVE METHYL GROUPS TO DNAAT SITES RECOGNIZED BY THEIR RESTRICTION ENDONUCLEASES"} {"Type":"TOSS-UP\n\nBIOLOGY Multiple Choice","Prompt":"Which of the following is the form of highly condensed chromatin that is not transcribable in its present form and typical of the type of chromatin visible in the light microscope when the cell is in interphase:\n\nW) Z DNA\n\nX) euchromatin\n\nY) heterochromatin\n\nZ) histochromatin","Completion":"Y) HETEROCHROMATIN"} {"Type":"TOSS-UP\n\nCHEMISTRY Short Answer","Prompt":"What type of magnetism exists in substances in which electrons are unpaired and are very weakly attracted by magnetic fields?","Completion":"PARAMAGNETISM (ACCEPT: PARAMAGNETIC)"} {"Type":"TOSS-UP\n\nPHYSICS Multiple Choice","Prompt":"What percent of energy in our Universe is believed to consist of dark matter and normal matter:","Completion":"W) 4.4% X) 16% Y) 27% Z) 73% Y) 27%"} {"Type":"","Prompt":"BONUS","Completion":"3) PHYSICS Short Answer Order the following 4 substances from the one with the LOWEST dielectric constant to the HIGHEST at \\(20^{\\circ}\\)C: mica; paraffin; vacuum; pure water VACUUM; PARAFFIN; MICA; PURE WATER (Solution: vacuum = 1.00; paraffin = 2.2; mica = 7.0; water = 80.0)"} {"Type":"","Prompt":"TOSS-UP","Completion":"4) MATH Short Answer Solve the following trigonometric equation for \\(x\\), giving your answer in degrees such that \\(0^{\\circ}\\leq x<360^{\\circ}\\): \\(2\\ \\sin\\ x-1=0\\) \\(30^{\\circ}\\) AND \\(150^{\\circ}\\)"} {"Type":"BONUS\n\nMATH Short Answer","Prompt":"Solve the following logarithmic equation in base 10 for \\(x\\) over the real numbers:","Completion":"log \\((2x-1)=\\log\\ (x+2)+\\log\\ (x-2)\\) 3 (DO NOT ACCEPT 3 and -1) (Solution: \\(2x-1=(x+2)(x-2)\\); \\((x-3)(x+1)=0\\); \\(x=3\\), \\(x eq\\) -1 - the log(-1) does not exist in the real number system)"} {"Type":"TOSS-UP\n\nEARTH SCIENCE Multiple Choice","Prompt":"Which of the following BEST describes the largest size of sediment particle that can be transported by a particular flow in a river:","Completion":"W) discharge X) competency Y) capacity Z) bed load X) COMPETENCY"} {"Type":"TOSS-UP\n\nGENERAL SCIENCE Multiple Choice","Prompt":"Which of the following is a mint-flavoring used in chewing gum, toothpaste, and mouthwashes produced naturally in fruits and vegetables:\n\nW) wintergreen X) xylitol Y) winterol Z) arabin","Completion":"X) XYLITOL"} {"Type":"TOSS-UP\n\nASTRONOMY Multiple Choice","Prompt":"Which of the following is LEAST accurate:\n\nW) magnetic fields can be strongly amplified when 2 neutron stars merge\n\nX) Saturn's moon Titan has been found to have large dunes\n\nY) hydrogen and nitrogen isotopes have been found in organic matter of primitive meteorites\n\nZ) interplanetary dust particles are typically nitrogen-rich and carbon-poor","Completion":"Z) INTERPLANETARY DUST PARTICLES ARE TYPICALLY NITROGEN-RICH AND CARBON-POOR"} {"Type":"TOSS-UP\n\nBIOLOGY Multiple Choice","Prompt":"Which of the following is a primary target of certain HIV drugs, such as AZT, specifically because HIV is a retrovirus:\n\nW) reverse transcriptase\n\nX) HIV receptors\n\nY) CD 4 cells\n\nZ) CD 8 cells","Completion":"W) REVERSE TRANSCRIPTASE"} {"Type":"TOSS-UP\n\nCHEMISTRY Multiple Choice","Prompt":"Which of the following is a term used to describe the electronic geometry around a central atom that has 5 regions of high electron density:\n\nW) tetrahedral\n\nX) trigonal pyramidal\n\nY) trigonal bipyramidal\n\nZ) octahedral","Completion":"Y) TRIGONAL Bipyramidal"} {"Type":"TOSS-UP\n\nPHYSICS Short Answer","Prompt":"Giving your answer in hertz, what is the lowest frequency possible for a string that is 90-centimeters long and whose wave speed is 360 meters per second?","Completion":"200"} {"Type":"TOSS-UP\n\nMATH Short Answer","Prompt":"Simplify the following radical expression, giving your answer in standard bi form: \\(\\sqrt{\\frac{-144}{49}}\\)","Completion":"\\(\\frac{12}{7}i\\)"} {"Type":"TOSS-UP\n\nASTRONOMY Multiple Choice","Prompt":"Which of the following was the primary instrument used to gather 5 to 35 micrometer spectra data from the Deep Impact mission:\n\nW) Chandra X-ray observatory\n\nX) Arecibo radio telescope\n\nY) Hubble gamma-ray telescope\n\nZ) Spitzer infrared spectrograph","Completion":"Z) SPITZER INFRARED SPECTROGRAPH"} {"Type":"TOSS-UP\n\nEARTH SCIENCE Multiple Choice","Prompt":"Which of the following glacial features are deposited in annual layers and can be used by geologists to determine seasonal variations during past ice ages:\n\nW) drumlins X) moraines Y) varves Z) eskers","Completion":"Y) VARVES"} {"Type":"TOSS-UP\n\nASTRONOMY Short Answer","Prompt":"If the temperature of an object is doubled, and all other things remain the same, how many times as fast will it radiate energy?","Completion":"16 (ACCEPT: 2\\({}^{4}\\))"} {"Type":"TOSS-UP\n\nBIOLOGY Multiple Choice","Prompt":"During the electron transport system, a store of potential energy is created that will be converted into high energy phosphates. The potential energy is best described as:\n\nW) ADP\n\nX) ATP\n\nY) the hydrogen ion gradient\n\nZ) the sodium differential","Completion":"Y) THE HYDROGEN ION GRADIENT"} {"Type":"BONUS\n\nBIOLOGY Multiple Choice","Prompt":"What is the first place in the breakdown of glucose, starting in glycolysis and through the Krebs cycle, where CO\\({}{2}\\) is released:\n\nW) pyruvate yielding Acetyl Co-A\n\nX) 3-phosphoglycerate (read as: 3-foss-foe-GLISS-err-ate) yielding 2-phosphoglycerate\n\nY) isocitrate yielding alpha ketogluterate\n\nZ) oxaloacetate (read as: ox-AL-oh-ACETATE) yielding citrate","Completion":"W) PYRUVATE YIELDING ACETYL Co-A"} {"Type":"TOSS-UP\n\nCHEMISTRY Multiple Choice","Prompt":"Which of the following molecules has the highest bond enthalpy, or energy it would take to break apart the bond:\n\nW) F\\({}{2}\\)\n\nX) H\\({}{2}\\)\n\nY) HCl\n\nZ) CO","Completion":"Z) CO"} {"Type":"BONUS\n\nCHEMISTRY Multiple Choice","Prompt":"Silver iodide, or AgI, is LEAST soluble in which of the following aqueous solutions:\n\nW) NaOH\n\nX) distilled water\n\nY) HNO\\({}{3}\\)\n\nZ) KI","Completion":"Z) KI"} {"Type":"Z) MORE ELECTRONS WILL BE EJECTED FROM THE METAL BONUS\n\nPHYSICS Multiple Choice","Prompt":"Which of the following is NOT true according to the Standard Model of Particle Physics:\n\nW) quarks carry color charge\n\nX) hadrons are color-neutral particles\n\nY) quarks can be isolated from their hadrons\n\nZ) the carrier particle for the electromagnetic interaction has zero charge","Completion":"Y) QUARKS CAN BE ISOLATED FROM THEIR HADRONS"} {"Type":"TOSS-UP\n\nMATH Short Answer","Prompt":"In degrees, what is the least positive angle that is coterminal with 650\\({}^{\\circ}\\)?","Completion":"290\\({}^{\\circ}\\)"} {"Type":"BONUS\n\nMATH Short Answer","Prompt":"Given the function f(\\(x\\)) = \\(x^{3}+3x-k\\) and f(\\(3\\)) = 31, solve for \\(k\\):","Completion":"5"} {"Type":"TOSS-UP\n\nEARTH SCIENCE Multiple Choice","Prompt":"It is estimated that the supercontinent Pangaea began to break up at the end of the Triassic Period about how many years ago:\n\nW) 2.5 billion years\n\nX) 225 million years\n\nY) 5 million years\n\nZ) 150,000 years","Completion":"X) 225 MILLION YEARS"} {"Type":"BONUS\n\nEARTH SCIENCE Multiple Choice","Prompt":"A group of small volcanoes, called the petit spot volcanoes, was recently found. What makes this find so unusual is they are believed to be formed:\n\nW) from asthenosphere material squeezing through cracks caused by bending in Earth's crust\n\nX) from plumes rising rapidly from the boundary between Earth's core and mantle\n\nY) at a convergent plate boundary\n\nZ) at a mid-ocean ridge","Completion":"W) FROM ASTHENOSPHERE MATERIAL SQUEEZING THROUGH CRACKS CAUSED BY BENDING IN EARTH'S CRUST"} {"Type":"TOSS-UP\n\nGENERAL SCIENCE Multiple Choice","Prompt":"Which of the following is NOT true:\n\nW) long-term variation in Earth's insolation is partially caused by precession\n\nX) there has yet to be any scientifically well-documented case of a human whose immune system has eradicated HIV from their body\n\nY) Candida albicans is a common fungal infective agent in humans\n\nZ) most scientific evidence supports the belief that C3 plants evolved from C4 plants","Completion":"Z) MOST SCIENTIFIC EVIDENCE SUPPORTS THE BELEF THAT C3 PLANTS EVOLVED FROM C4 PLANTS"} {"Type":"BONUS\n\nGENERAL SCIENCE Short Answer","Prompt":"By number or name, identify all of the following blood measures that would suggest a woman is deficient in iron: low hematocrit; larger than average size of erythrocytes; high levels of TSH; increased transferrin level","Completion":"1 AND 4 (ACCEPT: 1 - LOW HEMATOCRIT; 4 - INCREASED TRANSFERIN LEVEL"} {"Type":"TOSS-UP\n\nASTRONOMY Short Answer","Prompt":"Name all of the following 4 objects that would NOT be found on Earth: meteors; craters; asteroid impact remnants; bolides","Completion":"METEORS; BOLIDES"} {"Type":"TOSS-UP\n\nBIOLOGY Short Answer","Prompt":"How many codons in the entire genetic code call for amino acids?","Completion":"61"} {"Type":"TOSS-UP\n\nCHEMISTRY Short Answer","Prompt":"Order the following 3 ionic solids in terms of their lattice energies or enthalpies, going from the HIGHEST to the LOWEST: LiBr; KBr; NaBr","Completion":"LiBr; NaBr; KBr"} {"Type":"TOSS-UP\n\nPHYSICS Short Answer","Prompt":"If a rock is spun on a sling with a frequency of 100 revolutions per minute, what is the rock's period, in seconds?","Completion":"0.6"} {"Type":"TOSS-UP\n\nBIOLOGY Short Answer","Prompt":"What part of the human brain releases the most corticotropin?","Completion":"PITUITARY"} {"Type":"BONUS\n\nBIOLOGY Short Answer","Prompt":"What is the average yield of ATP for every NADH and for every FADH\\({}{2}\\), respectively, that pass through the electron transport system?","Completion":"NADH = 3; FADH\\({}{2}\\) = 2"} {"Type":"TOSS-UP\n\nEARTH AND SPACE Multiple Choice","Prompt":"Which of the following is characteristic of orthochemical carbonate rocks but not of allochemical carbonate rocks?\n\nW) Fossils from other locations\n\nX) Intraclasts within a micrite matrix\n\nY) Ooids in a sparry calcite matrix\n\nZ) Clear, granular carbonate","Completion":"Z) CLEAR, GRANULAR CARBONATE"} {"Type":"BONUS\n\nEARTH AND SPACE Short Answer","Prompt":"Sirius is the brightest star in the sky at night, with an apparent magnitude of about -1, and the Sun is the brightest star in the sky during the day, with an apparent magnitude of about -26. Expressing your answer as a power of 10, approximately how many times as bright does the Sun appear in the sky compared to Sirius?","Completion":"10\\({}^{10}\\) (ACCEPT: 10 BILLION)"} {"Type":"TOSS-UP\n\nBIOLOGY Multiple Choice","Prompt":"Which of the following plant hormones is manufactured primarily in the shoot tips, in embryos, and in parts of developing flowers and seeds?\n\nW) Auxins [AWK-sins]\n\nX) Cytokins [sy-tuh-KY-nins]\n\nY) Gibberellins [jib-uh-REL-ins]\n\nZ) Ethylene","Completion":"W) AUXINS"} {"Type":"BONUS\n\nBIOLOGY Multiple Choice","Prompt":"What is the most immediate role for the protons produced from water photolysis [foh-TOL-uh-sis] during photosynthesis?\n\nW) React with oxygen to make water\n\nX) Acidify the mitochondrial matrix and provide fuel for chemiosmosis\n\nY) Contribute to the proton gradient in the thylakoid and ultimately provide energy to make ATP\n\nZ) Combine with carbon dioxide during sugar synthesis","Completion":"Y) CONTRIBUTE TO THE PROTON GRADIENT IN THE THYLAKOID AND ULTIMATELY PROVIDE ENERGY TO MAKE ATP"} {"Type":"TOSS-UP\n\nMATH Multiple Choice","Prompt":"While eating out, you and your friend both tip the server 2 dollars. Your tip is 10% of your bill, while your friend's is 20%. What is the difference, in dollars, between your bills?\n\nW) 2\n\nX) 4\n\nY) 5\n\nZ) 10","Completion":"Z) 10"} {"Type":"BONUS\n\nMATH Short Answer","Prompt":"What is the smallest positive number other than 3 that, when divided by 4, 5, 6, or 7, has a remainder of 3?","Completion":"423"} {"Type":"TOSS-UP\n\nENERGY Short Answer","Prompt":"What form of petroleum deposit, abundant in Canada and Venezuela, contains naturally occurring mixtures of sand, clay, water, and a dense and extremely viscous form of petroleum?","Completion":"OIL SANDS"} {"Type":"BONUS\n\nENERGY Multiple Choice","Prompt":"According to a study at Argonne National Laboratories, what is the net energy gain or loss, expressed as a ratio, from producing a gallon of ethanol from corn?\n\nW) -0.95\n\nX) + 1.0\n\nY) +1.1\n\nZ) +1.3","Completion":"Z) +1.3"} {"Type":"TOSS-UP\n\nPHYSICS Short Answer","Prompt":"What fundamental physical constant is a baseline that gives the value of the absolute dielectric [dy-i-LEK-trik] constant of a classical vacuum?","Completion":"PERMITTIVITY CONSTANT (ACCEPT: PERMITTIVITY OF FREE SPACE OR VACUUM PERMITTIVITY)"} {"Type":"BONUS\n\nPHYSICS Multiple Choice","Prompt":"A cord making a 60\\({}^{\\circ}\\) angle with the floor pulls on a crate with a tension of 100 newtons along a 10 meter distance. How much work, in joules, is done on the crate by the pulling cord?\n\nW) 0\n\nX) 50\n\nY) 500\n\nZ) 5000","Completion":"Y) 500"} {"Type":"TOSS-UP\n\nCHEMISTRY Multiple Choice","Prompt":"Consider the dissociation of weak hydrofluoric acid into fluoride and hydronium ions, and a solution in which this reaction is at equilibrium [ee-kwuh-LIB-ree-uhm]. Which of the following additions would shift the equilibrium to the right?\n\nW) Calcium fluoride\n\nX) Sulfuric acid\n\nY) Ammonia\n\nZ) Ammonium chloride","Completion":"Y) AMMONIA"} {"Type":"BONUS\n\nCHEMISTRY Short Answer","Prompt":"Place the following four molecules in order from lowest to highest melting point: NaF, CH\\({}{4}\\), F\\({}{2}\\), Ca.","Completion":"F\\({}{2}\\), CH\\({}{4}\\), Ca, NaF (ACCEPT: 3, 2, 4, 1)"} {"Type":"TOSS-UP\n\nEARTH AND SPACE Short Answer","Prompt":"The most distant quasar known has a redshift of z = 6. Its emission is dominated by the Lyman alpha hydrogen line, which has a laboratory wavelength of 1216 angstroms. To two significant figures, and in angstroms, what is the wavelength of the Lyman alpha line observed in the quasar?","Completion":"7300"} {"Type":"BONUS\n\nEARTH AND SPACE Multiple Choice","Prompt":"Which of the following statements about limestone and dolostone is true?\n\nW) The primary cation [KAT-eye-uhn] present in dolostone is magnesium, while in limestone, it is calcium\n\nX) Both limestone and dolostone have high solubility in HCL, making them difficult to distinguish in the field\n\nY) Dolostone's primary mineral compound has two CO\\({}{3}\\)\\({}^{2}\\)-anions while limestone's primary mineral compound has only one CO\\({}{3}\\)\\({}^{2}\\)- anion\n\nZ) Unlike limestone, which is composed primarily of the mineral calcite, dolostone is considered both a rock and a mineral","Completion":"Y) DOLOSTONE'S PRIMARY MINERAL COMPOUND HAS TWO CO\\({}{3}\\)\\({}^{2}\\)-ANIONS"} {"Type":"TOSS-UP\n\nENERGY Multiple Choice","Prompt":"Which of the following elements is part of the cladding and used as a corrosion-resistant alloy to coat pellets of uranium dioxide prior to fuel rod production?\n\nW) Cadmium\n\nX) Zirconium\n\nY) Titanium\n\nZ) Silver","Completion":"X) ZIRCONIUM"} {"Type":"BONUS\n\nENERGY Short Answer","Prompt":"Indicate all of the following three statements that are true of\n\n biodiesel: Biodiesel has better lubricating properties and much higher cetane [SEE-tayn]\n\nratings than today's lower sulfur diesel fuels; Animal fats, algae, and soybean oil are all used to produce biofuels; A biodiesel blend with 80% petroleum diesel is labeled B80.","Completion":"1 AND 2 (ACCEPT: BIODIESEL HAS BETTER LUBRICATIONS PROPERTIES AND MUCH HIGHER CETANE RATINGS THAN TODAY'S LOWER SULFUR DIESEL FUELS AND ANIMAL FATS, ALGAE, AND SOYBEAN OIL ARE ALL USED TO PRODUCE BIOFUELS)"} {"Type":"TOSS-UP\n\nMATH Short Answer","Prompt":"Identify all of the following three logical conclusions that follow from the statement \"all A are B\": All B are A; If C is A, then C is also B; If C is not A, then C is not B.","Completion":"2 ONLY (ACCEPT: IF C IS A, THEN C IS ALSO B)"} {"Type":"TOSS-UP\n\nPHYSICS Short Answer","Prompt":"How many seconds does it take for a solid cylinder to go from 10 radians per second to 70 radians per second if the moment of inertia is 50 kg meters squared and a torque of 200 newton meters is applied?","Completion":"15"} {"Type":"TOSS-UP\n\nBIOLOGY Multiple Choice","Prompt":"Which of the following is NOT an example of negative feedback?\n\nW) Excess ATP inhibiting phosphofructokinase [fohs-foh-frook-toh-KY-nays]\n\nX) High blood glucose causing insulin release\n\nY) Secretion of cortisol inhibiting ACTH release\n\nZ) Release of platelet activating factor during clotting","Completion":"Z) RELEASE OF PLATELET ACTIVATING FACTOR DURING CLOTTING"} {"Type":"TOSS-UP\n\nCHEMISTRY Multiple Choice","Prompt":"In chemical kinetics, which of the following types of reactions has a half-life that is directly proportional to the initial reactant concentration?\n\nW) Zero order\n\nX) First order\n\nY) Second order\n\nZ) Third order","Completion":"W) ZERO ORDER"} {"Type":"TOSS-UP\n\nMATH Short Answer","Prompt":"What is the maximum possible number of intersection points of a circle and a square?","Completion":"EIGHT"} {"Type":"TOSS-UP\n\nPHYSICS Short Answer","Prompt":"What is the principle quantum number of the only orbital of the hydrogen atom that has no angular dependence?","Completion":"1"} {"Type":"TOSS-UP\n\nEARTH AND SPACE Multiple Choice","Prompt":"Which of the following metamorphic rocks results from shallow depth, low pressure contact metamorphism affiliated with a large igneous [IG-nee-uhs] intrusion?\n\nW) Slate\n\nX) Gneiss [NYS]\n\nY) Hornfels\n\nZ) Blueschist","Completion":"Y) HORNFELS"} {"Type":"BONUS\n\nEARTH AND SPACE Multiple Choice","Prompt":"The ocean floor near a midocean ridge has which of the following patterns of magnetization?\n\nW) The same constant magnetization on both sides\n\nX) Strips of alternate magnetization on both sides\n\nY) It is magnetized in one direction on one side and in the opposite direction on the other side\n\nZ) It has no pattern of magnetization","Completion":"X) STRIPS OF ALTERNATE MAGNETIZATION ON BOTH SIDES"} {"Type":"TOSS-UP\n\nCHEMISTRY Multiple Choice","Prompt":"Which of the following statements portrays waxes\n\ninaccurately?\n\nW) Many waxes are natural products\n\nX) Waxes help plants eliminate excess water\n\nY) Paraffin waxes do not contain ester functional groups\n\nZ) Ear wax contains both phospholipids and esters of cholesterol","Completion":"X) WAXES HELP PLANTS Eliminate EXCESS WATER"} {"Type":"BONUS\n\nCHEMISTRY Short Answer","Prompt":"Assume that 1 mole of each of the following three salts is dissolved in 1 liter of water. By name or number, give the three solutions from the one with the lowest freezing point to the one with the highest freezing point: aluminum sulfate; calcium nitrate; sodium fluoride.","Completion":"1, 2, 3 (ACCEPT: IN THE ORDER THEY WERE PRESENTED)"} {"Type":"TOSS-UP\n\nBIOLOGY Multiple Choice","Prompt":"Which of the following is an organic dietary requirement that is only needed in small amounts?\n\nW) Vitamins\n\nX) Minerals\n\nY) Enzymes\n\nZ) Coenzymes","Completion":"W) VITAMINS"} {"Type":"TOSS-UP\n\nENERGY Multiple Choice","Prompt":"Which of the following types of fuel cell is most commonly used in vehicles?\n\nW) Alkaline\n\nX) Polymer electrolyte membrane\n\nY) Solid oxide\n\nZ) Phosphoric acid","Completion":"X) POLYMER ELECTROLYTE MEMBRANE"} {"Type":"TOSS-UP\n\nEARTH AND SPACE Multiple Choice","Prompt":"Why does dolostone weather to a brownish colored rock?\n\nW) Iron occurs in small amounts, replacing some of the magnesium in dolomite\n\nX) Copper ions replace magnesium in dolostone as groundwater permeates the rock\n\nY) Aluminum ions replace the calcium in the dolomite compounds, and there is a reaction\n\nbetween aluminum and the remaining magnesium ions\n\nZ) Magnesium ions replace the calcium ions in dolostone compounds","Completion":"W) IRON OCCURS IN SMALL AMOUNTS, REPLACING SOME OF THE"} {"Type":"BONUS\n\nEARTH AND SPACE Multiple Choice","Prompt":"Blue-green algae produces mats that trap and bind\n\nmicrocrystalline carbonates as incoming tides sweep over the sand, producing what kind of\n\ncarbonate rock structure?\n\nW) Crystalline veins\n\nX) Laminations\n\nY) Ripples and cross-bedding\n\nZ) Styloites","Completion":"X) LAMINATIONS"} {"Type":"TOSS-UP\n\nMATH Short Answer","Prompt":"If you think of the fingers on your hand as being 10 binary digits (0 if the finger is down and 1 if the finger is up), then what is the biggest base 10 number you can count to on your fingers?","Completion":"1023"} {"Type":"BONUS\n\nMATH Short Answer","Prompt":"Which of the following is not an antiderivative for sin x cos x [ sine of x times cosine of x]\n\nW) \\(\\frac{\\sin^{2}x}{2}\\) [one-half sine squared of x]\n\nX) \\(-\\frac{(\\cos^{2}x)}{2}\\) [negative one-half cosine squared of x]\n\nY) \\(\\frac{(sin\\,2x)}{4}\\) [one-fourth sine of two x]\n\nZ) \\(-\\frac{(cos\\,2x)}{4}\\) [negative one-fourth cosine of two x]","Completion":"Y) \\(\\frac{sin\\,2x}{4}\\)"} {"Type":"TOSS-UP\n\nBIOLOGY Short Answer","Prompt":"What hormone, secreted from pinealocytes [pin-ee-A-loh-syt], is thought to be responsible for the regulation of the body's circadian [sur-KAY-ee-uhn] rhythms?","Completion":"MELATONIN"} {"Type":"TOSS-UP\n\nCHEMISTRY Short Answer","Prompt":"What type of reduction reaction generally employs a metal such as ruthenium, platinum, nickel, or palladium as a catalyst, and is used to transform alkenes [AL-kayns] to alkanes in a chamber pressurized with hydrogen gas?","Completion":"HYDROGENATION"} {"Type":"TOSS-UP\n\nPHYSICS Multiple Choice","Prompt":"Unstable particles muons [MYOO-ons], pions [PY-ons], kaons, and sigmas have half-life times in the range of which of the following?\n\nW) 10\\({}^{\\text{-}6}\\) to 10\\({}^{\\text{-}23}\\) seconds\n\nX) 10\\({}^{\\text{-}1}\\)to 10\\({}^{\\text{-}3}\\) seconds\n\nY) 10 to 1 seconds\n\nZ) 10\\({}^{\\text{-}1}\\) to 10\\({}^{\\text{-}8}\\) seconds","Completion":"W) 10\\({}^{\\text{-}6}\\) TO 10\\({}^{\\text{-}23}\\) SECONDS"} {"Type":"TOSS-UP\n\nEARTH AND SPACE Multiple Choice","Prompt":"Isotelus rex [fi-SOH-te-luhs REKS] was discovered in 1998 and is the largest found fossil of what organism?\n\nW) Dinosaur\n\nX) Whale\n\nW) Fish\n\nZ) Tribotite","Completion":"Z) TRILOBITE"} {"Type":"BONUS\n\nEARTH AND SPACE Short Answer","Prompt":"Winds aloft tend to form into longwaves. Assuming there is no divergence or convergence and the column of air cannot stretch or contract, as we follow the air at a constant speed and a constant 500 millibar level, it begins to flow southeastward, where the value of Earth's vorticity decreases. The conservation of what factor then causes the air to head northeastward?","Completion":"ABSOLUTE VORTICITY"} {"Type":"TOSS-UP\n\nBIOLOGY Multiple Choice","Prompt":"Which of the following fish is endemic to the island of Cozumel?\n\nW) Splendid toadfish\n\nX) Coelacanth [SEE-luh-kanth]\n\nY) Mola mola\n\nZ) Orange roughly","Completion":"W) SPLENDID TOADFISH"} {"Type":"BONUS\n\nBIOLOGY Short Answer","Prompt":"Arrange proximally to distally the following four components of a bird's contour feather: superior umbilicus [uhm-BIL-i-kuhs], rachis [RAY-kis], calamus, inferior umbilicus.","Completion":"4, 3, 1, 2 (ACCEPT: INFERIOR UMBILICUS, CALAMUS, SUPERIOR UMBILICUS, RACHIS)"} {"Type":"TOSS-UP\n\nMATH Short Answer","Prompt":"Given \\(g(x)=5+\\frac{x^{2}}{\\sin(x^{2})}\\)[g of x equals 5 plus the fraction with numerator x squared and denominator sine of the quantity x squared], find \\(\\lim{x\\to 0}g(x)\\) [the limit as x approaches zero of g of x].","Completion":"6"} {"Type":"TOSS-UP\n\nENERGY Multiple Choice","Prompt":"Judged to be an intervention that delivers high impact per development dollar, improved cooking stoves aim to reduce greenhouse gas contribution by reducing the amount of what substance?","Completion":"W) Carbon dioxide X) Black carbon Y) Methane Z) Nitro oxide X) BLACK CARBON"} {"Type":"TOSS-UP\n\nCHEMISTRY Short Answer","Prompt":"What simple system is used to describe molecular vibrations in quantum mechanics?","Completion":"HARMONIC OSCILLATOR (ACCEPT: BALL AND SPRING)"} {"Type":"TOSS-UP\n\nPHYSICS Multiple Choice","Prompt":"Which of the following quark compositions does NOT have a total of zero strangeness quantum number?\n\nW) Up, down\n\nX) Up, strange\n\nY) Strange, antiparticle strange\n\nZ) Down, down","Completion":"X) UP, STRANGE"} {"Type":"TOSS-UP\n\nEARTH AND SPACE Multiple Choice","Prompt":"Which of the following organisms were NOT extinct by the end of the Cretaceous [Kri-TAY-shuhs] period?\n\nW) Ichthyosaurs [IK-thee-uh-sawrs]\n\nX) Cycliophora [sy-KLEEUH-fruh]\n\nY) Quetzalcoatlus [ket-sahl-koh-AHT-luhs]\n\nZ) Plesiosaurs [PLEE-see-uh-sawrs]","Completion":"X) CYCLIOPHORA"} {"Type":"BONUS\n\nEARTH AND SPACE Short Answer","Prompt":"The orbit of an asteroid in our solar system has a semi-major axis of 4 AU. To the nearest integer, and in years, what is its period?","Completion":"8"} {"Type":"TOSS-UP\n\nBIOLOGY Multiple Choice","Prompt":"The following are examples of proteins that are not functional because of some problem. In which of the following examples does the problem result from damage primarily to the secondary structure of the protein?\n\nW) The protein has been exposed to urea, which has disrupted its hydrogen bonds\n\nX) A mutation is at the active site of a protein and the protein has lost catalytic activity\n\nY) A mutation is at the dimer [DY-mer] interface of a protein and prevents its dimerization\n\nZ) A protein has been exposed to an oily chemical that disrupts its hydrophobic core","Completion":"W) THE PROTEIN HAS BEEN EXPOSED TO UREA, WHICH HAS DISRUPTED ITS HYDROGEN BONDS"} {"Type":"BONUS\n\nBIOLOGY Short Answer","Prompt":"Arrange the following six layers found in a one-year old woody\n\nsapling from most lateral to most medial: Cortex, Phloem [FLOH-em] fibers, Pith, Secondary phloem, Periderm, Secondary xylem [ZY-luhm].","Completion":"5, 1, 2, 4, 6, 3 (ACCEPT: PERIDERM, CORTEX, PHLOEM FIBERS, SECONDARY PHLOEM, SECONDARY XYLEM, PITH)"} {"Type":"TOSS-UP\n\nMATH Multiple Choice","Prompt":"What is the principal value in radians for \\(cos^{-1}(-\\frac{\\sqrt{3}}{2})\\) [inverse cosine of negative square root of 3 over 2]?\n\n\\(\\mbox{W})-\\frac{\\pi}{6}\\)\n\n\\(\\mbox{X})-\\frac{\\pi}{3}\\)\n\n\\(\\mbox{Y})\\frac{5\\pi}{6}\\)\n\n\\(\\mbox{Z})\\frac{11\\pi}{6}\\)","Completion":"Y)\\(\\frac{5\\pi}{6}\\)"} {"Type":"BONUS\n\nMATH Short Answer","Prompt":"Find tan(arccos (\\(2x\\))) [the tangent of the arc-cosine of 2x] in terms of \\(x\\).","Completion":"\\(\\sqrt{1-4x^{2}}\/2x\\)"} {"Type":"TOSS-UP\n\nENERGY Multiple Choice","Prompt":"Roughly what percentage of the energy density of the universe is dark energy?\n\nW) 13%\n\nX) 35%\n\nY) 72%\n\nZ) 99%","Completion":"Y) 72%"} {"Type":"BONUS\n\nENERGY Multiple Choice","Prompt":"Which of the following does NOT help explain why the theoretical yield of cellular respiration is never actually achieved?\n\nW) Pyruvate [py-ROO-vayt] must be actively transported out of the mitochondria [my-tuh-KON-dree-uh]\n\nX) Phosphate is actively transported into the mitochondria\n\nY) ADP and ATP are actively exchanged across the inner mitochondrial membrane\n\nZ) The inner mitochondrial membrane leaks protons","Completion":"W) PYRUVATE MUST BE ACTIVELY TRANSPORTED OUT OF THE MITOCHONDRIA"} {"Type":"TOSS-UP\n\nCHEMISTRY Short Answer","Prompt":"What is the relationship of the organic compounds diethyl ether, n-butyl alcohol, and methyl propyl ether?","Completion":"STRUCTUREI ISOMERS (ACCEPT: CONSTITUTIONAL ISOMERS)"} {"Type":"TOSS-UP\n\nPHYSICS Multiple Choice","Prompt":"A body with a 2 kilogram mass has position vector \\(r=3i+2\\,t^{2}j\\) meters. What is the velocity at \\(t=1\\) second?\n\nW) 1\\(j\\)\n\nX) 4\\(j\\)\n\nY) 3\\(i+4j\\)\n\nZ) 1\\(i+1\\)j","Completion":"X) 4\\(j\\)"} {"Type":"TOSS-UP\n\nEARTH AND SPACE Multiple Choice","Prompt":"Where do the strongest winds in Antarctica, which can reach 200 miles per hour, originate?\n\nW) Antarctic east coast\n\nX) Antarctic polar plateau\n\nY) Antarctic circumpolar wave\n\nZ) Weddell Sea","Completion":"X) ANTRACTIC POLAR PLATEAU"} {"Type":"TOSS-UP\n\nBIOLOGY Short Answer","Prompt":"In muscle cells, the sarcoplasmic reticulum [sahr-kuh-PLAHZ-mik ri-TIK-yuh-luhm] is an important organelle that permits the muscle cell to contract. It acts to store a particular chemical until it is electrically excited. What chemical is stored in the sarcoplasmic reticulum?","Completion":"CALCIUM (ACCEPT: CALCIUM ION OR CA+2)"} {"Type":"TOSS-UP\n\nCHEMISTRY Multiple Choice","Prompt":"Which of the following molecules has a permanent dipole moment?\n\nW) Carbon dioxide\n\nX) Methane\n\nY) Hydrogen sulfide\n\nZ) Molecular nitrogen","Completion":"Y) HYDROGEN SULFIDE"} {"Type":"BONUS\n\nCHEMISTRY Short Answer","Prompt":"List the five halogens in order of increasing first ionization energy.","Completion":"ASTATINE, IODINE, BROMINE, CHLORINE, FLUORINE (DO NOT ACCEPT ANOTHER ORDER)"} {"Type":"TOSS-UP\n\nMATH Short Answer","Prompt":"Solve for \\(x\\): \\(14-\\sqrt{3-x}=9\\)[14 minus the square root of the quantity 3 minus x equals 9].","Completion":"-22"} {"Type":"BONUS\n\nMATH Short Answer","Prompt":"A circle of radius 2 is centered at the point (2, . What is the slope of the line tangent to the circle at the point where \\(x=3\\) and \\(y\\) is greater than 3?","Completion":"\\(\\frac{-\\sqrt{3}}{3}\\) (DO NOT ACCEPT: \\(\\frac{-1}{\\sqrt{3}}\\))"} {"Type":"TOSS-UP\n\nCHEMISTRY Multiple Choice","Prompt":"What is the change in the parent atom atomic number upon alpha particle emission?\n\nW) It decreases by 2\n\nX) It decreases by 1\n\nY) It increases by 1\n\nZ) It increases by 2","Completion":"W) IT DECREASES BY 2"} {"Type":"TOSS-UP\n\nPHYSICS Multiple Choice","Prompt":"If the tension in a stretched string is multiplied by a factor of 9, the speed of the transverse wave is multiplied by what factor?\n\nW) 1\/9\n\nX) 1\/3\n\nY) 3\n\nZ) 9","Completion":"Y) 3"} {"Type":"TOSS-UP\n\nBIOLOGY Multiple Choice","Prompt":"When two pyrimidine [py-RIM-i-deen] bases are adjacent on a DNA strand, exposure to UV light will commonly lead to formation of a cyclobutane ring between them, resulting in a bulky lesion in the DNA. The normal replicative polymerases [pol-uh-muh-RAYS-uhs] cannot use this as a template, and the lesion often results in mutation if it cannot be repaired. Which of the following is an example of this lesion formed by UV exposure of neighboring pyrimidines [py-RIM-i-deens]?\n\nW) Transversion\n\nX) Cytosine deamination [SY-tuh-seen]\n\nY) Thymine dimer [THY-meen DY-mer]\n\nZ) Intercalation","Completion":"Y) THYMINE DIMER"} {"Type":"BONUS\n\nBIOLOGY Multiple Choice","Prompt":"Which of the following statements is true regarding the mechanism of infection in prion diseases?\n\nW) Prions invade the cell nucleus, altering DNA transcription processes\n\nX) Prions induce abnormal folding in normal protein molecules\n\nY) Prions bind to specific sites on a ribosome [RY-buh-sohm], rendering them useless\n\nZ) Prions bind to mRNA, leaving the nucleus, inducing post-transcriptional changes before translation","Completion":"X) PRIONS INDUCE ABNORMAL FOLDING IN NORMAL PROTEIN MOLECULES"} {"Type":"TOSS-UP\n\nEARTH AND SPACE Multiple Choice","Prompt":"Which of the following features forms because parts of a glacier move at varying speeds and in different directions?\n\nW) Moraines\n\nX) Moulan\n\nY) Crevasses\n\nZ) Aretes [uh-REYTS]","Completion":"Y) CREVASSES"} {"Type":"BONUS\n\nEARTH AND SPACE Short Answer","Prompt":"Order the following four steps that are theorized to have occurred in the formation of our atmosphere: plants evolve and oxygen increases\n\ndramatically; the atmosphere gradually becomes rich in nitrogen; outgassing leads to\n\nabundant water vapor; photodissociation increases oxygen gradually.","Completion":"3, 2, 4, 1"} {"Type":"","Prompt":"TOSS-UP","Completion":"21) MATH Short Answer What is the scalar product of the vectors \\(2i\\) - \\(3j+2k\\) and \\(i+2j+2k\\)? 0"} {"Type":"TOSS-UP\n\nENERGY Multiple Choice","Prompt":"When dealing with a tank of 150 pounds of water on a platform, it is most energy intensive to do which of the following?\n\nW) Lift the 150 pounds of water 2 feet in the air\n\nX) Lower the 150 pounds of water by 2 feet\n\nY) Decrease the temperature of the water by 2\\({}^{\\circ}\\) Fahrenheit\n\nZ) Increase the temperature of the water by 1\\({}^{\\circ}\\) Fahrenheit","Completion":"Y) DECREASE THE TEMPERATURE OF THE WATER BY 2\\({}^{\\circ}\\) FAHRENHEIT"} {"Type":"TOSS-UP\n\nBIOLOGY Multiple Choice","Prompt":"Which of the following is NOT a point mutation?\n\nW) Missense\n\nX) Transversion\n\nY) Inversion\n\nZ) Transition","Completion":"Y) INVERSION"} {"Type":"BONUS\n\nBIOLOGY Multiple Choice","Prompt":"The cotyledon [kot-uhl-EED-uhn] is which of the following?\n\nW) Primary embryonic leaf\n\nX) Primary embryonic petal\n\nY) Secondary embryonic leaf\n\nZ) Secondary embryonic petal","Completion":"W) PRIMARY EMBRYONIC LEAF"} {"Type":"TOSS-UP\n\nCHEMISTRY Multiple Choice","Prompt":"Who was the inventor of the first electrochemical cell?\n\nW) Michael Faraday\n\nX) Alessandro Volta\n\nY) James Maxwell\n\nZ) Benjamin Franklin","Completion":"X) ALESSANDRO VOLTA"} {"Type":"BONUS\n\nCHEMISTRY Short Answer","Prompt":"A plastic undergoes deformation at 740\\({}^{\\circ}\\) Celsius. Given that its original cross-sectional area is 70 centimeters, its cross-sectional area after deformation is 62 centimeters, and its yield strength is 300 megapascals, determine the percent cold work to the nearest whole number.","Completion":"11%"} {"Type":"TOSS-UP\n\nPHYSICS Multiple Choice","Prompt":"What physical unit is the product of velocity, magnetic field, and length?\n\nW) Volt\n\nX) Newton meter per ampere second squared\n\nY) Joule per second squared\n\nZ) Newton per ampere","Completion":"W) VOLT"} {"Type":"TOSS-UP\n\nCHEMISTRY Multiple Choice","Prompt":"Which of the following molecules has an electron pair geometry that matches its molecular geometry:\n\nW) \\(\\rm H{2}O\\)\n\nX) \\(\\rm NH{3}\\)\n\nY) \\(\\rm SO{2}\\)\n\nZ) \\(\\rm PCl{5}\\)","Completion":"Z) \\(\\rm PCl{5}\\)"} {"Type":"BONUS\n\nCHEMISTRY Multiple Choice","Prompt":"Which of the following is NOT a constitutional isomer of the organic compound whose molecular formula is \\(\\rm C{4}H{8}\\):\n\nW) 1-butene\n\nX) 2-butene\n\nY) 2-methyl-1-butene\n\nZ) 2-methyl-1-propene","Completion":"Y) 2-METHYL-1-BUTENE"} {"Type":"TOSS-UP\n\nPHYSICS Short Answer","Prompt":"Within the Standard Model of Particle Physics what particle is responsible for carrying the strong nuclear force?","Completion":"GLUON"} {"Type":"BONUS\n\nPHYSICS Short Answer","Prompt":"What is the principle or law that fundamentally limits the accuracy of specifying at the same time the momentum and position of a particle?","Completion":"HEISENBERG UNCERTAINTY PRINCIPLE (ACCEPT: UNCERTAINTY PRINCIPLE)"} {"Type":"TOSS-UP\n\nBIOLOGY Multiple Choice","Prompt":"Which of the following occurs during synapsis of meiosis:\n\nW) non-sister chromatids of homologous chromosomes cross over\n\nX) sister chromatids of homologous chromosomes cross over\n\nY) non-sister chromatids of non-homologous chromosomes cross over\n\nZ) sister chromatids of non-homologous chromosomes cross over","Completion":"W) NON-SISTER CHROMATIDS OF HOMologous CHROMOSOMES CROSS OVER"} {"Type":"TOSS-UP\n\nMATH Short Answer","Prompt":"In simplest radical form, give the cosecant of a \\(135^{\\circ}\\) angle:","Completion":"\\(\\sqrt{2}\\)"} {"Type":"TOSS-UP\n\nEARTH SCIENCE Multiple Choice","Prompt":"Groundwater flows according to what law that, in its simplest form, is given by the equation Q = kIA:\n\nW) Darcy's Law\n\nX) Law of Superposition\n\nY) Law of Original Horizontality\n\nZ) Ekman's Law","Completion":"W) DARCY'S LAW"} {"Type":"TOSS-UP\n\nGENERAL SCIENCE Multiple Choice","Prompt":"A primary consequence of acid rain in the northeast U.S. has been:\n\nW) salt-water intrusion in coastal wells\n\nX) the leaching out of toxic heavy metals in lakes and streams\n\nY) increase in radioactive decay of parent bedrock\n\nZ) increased algal growth to cause eutrophication in lakes","Completion":"X) THE LEACHING OUT OF TOXIC HEAVY METALS IN LAKES AND STREAMS"} {"Type":"TOSS-UP\n\nASTRONOMY Multiple Choice","Prompt":"If an astronomer photographed a massive cloud of ionized hydrogen in space that was emitting all of the Balmer wavelengths, what color would it typically appear to us:\n\nW) white X) green Y) pink Z) blue","Completion":"Y) PINK"} {"Type":"TOSS-UP\n\nCHEMISTRY Multiple Choice","Prompt":"For a given chemical reaction at constant temperature and pressure, if the value for Gibbs free-energy change is a negative, the reaction is always:\n\nW) endothermic X) exothermic Y) spontaneous Z) nonspontaneous","Completion":"Y) SPONTANEOUS"} {"Type":"TOSS-UP\n\nMATH Short Answer","Prompt":"Giving your answer as one, two, three, or four, in what quadrant will theta terminate if sine \\(\\theta\\) and secant \\(\\theta\\) are both negative?","Completion":"3"} {"Type":"TOSS-UP\n\nEARTH SCIENCE Multiple Choice","Prompt":"Which of the following is the global wind system that most influences weather in the West Indies and Central America:\n\nW) northeast trade winds\n\nX) prevailing westerlies\n\nY) southeast trade winds\n\nZ) polar westerlies","Completion":"W) NORTHEAST TRADE WINDS"} {"Type":"TOSS-UP\n\nCHEMISTRY Multiple Choice","Prompt":"In order to find the heat of reaction, or delta H naught, for a reaction using standard enthalpies of formation, the working formula is:\n\nW) delta H products minus delta H reactants\n\nX) delta H reactants minus delta H products\n\nY) delta H reactants plus delta H products\n\nZ) delta H products plus delta H reactants","Completion":"W) DELTA H PRODUCTS MINUS DELTA H REACTANTS"} {"Type":"","Prompt":"BONUS\n\nCHEMISTRY Short answer In crystallography, what is the coordination number, or number of nearest neighbors, of an atom in the simple cubic unit cell structure?","Completion":"6"} {"Type":"TOSS-UP\n\nPHYSICS Multiple Choice","Prompt":"Which of the following is NOT true:\n\nW) the hydrogen nucleus has a lower binding energy per nucleon than the helium nucleus\n\nX) the tritium nucleus contains 3 nucleons\n\nY) it is typically easier to fission U-238 than U-235 with slow neutrons\n\nZ) fission fragments of a nucleus are more often about 60% and 40% of the original mass of the atom than 50% and 50%","Completion":"Y) IT IS TYPICALLY EASIER TO FISION U-238 THAN U-235 WITH SLOW NEUTRONS"} {"Type":"BONUS\n\nPHYSICS Short Answer","Prompt":"In the Standard Model of particle physics, all forces arise because of the exchange of what fundamental class of particles?","Completion":"BOSONS"} {"Type":"TOSS-UP\n\nBIOLOGY Multiple Choice","Prompt":"If the human SRY gene is inactivated in the first week of gestation, which of the following will occur:\n\nW) the embryo will die\n\nX) the embryo will remain female\n\nY) the embryo will continue as a male\n\nZ) the embryo will remain in a quiescent state","Completion":"X) THE EMBRYO WILL REMAIN FEMALE"} {"Type":"BONUS\n\nBIOLOGY Multiple Choice","Prompt":"Which of the following functions primarily by cutting RNA strands at specific sites:\n\nW) micro RNA\n\nX) episome\n\nY) ribozyme\n\nZ) ligase","Completion":"Y) RIBOZYME"} {"Type":"TOSS-UP\n\nMATH Short Answer","Prompt":"Simplify the following trigonometric expression completely: \\(\\cos\\,\\theta\\,\\tan\\,\\theta\\) (read as: cosine theta tangent theta)","Completion":"\\(\\sin\\,\\theta\\)"} {"Type":"BONUS\n\nMATH Short Answer","Prompt":"In how many ways can 4 green, 2 yellow, and 2 red tee-shirts be given to 8 soccer players standing in a row if each player receives only one tee-shirt?","Completion":"420"} {"Type":"TOSS-UP\n\nEARTH SCIENCE Multiple Choice","Prompt":"Which of the following is a glacial feature that is NOT caused by the deposition of sediments:\n\nW) kame\n\nX) kettle\n\nY) drumlin\n\nZ) bergshrund","Completion":"Z) BERGSHRUND"} {"Type":"TOSS-UP\n\nGENERAL SCIENCE Short Answer","Prompt":"What is the SI unit for absorbed dose of ionizing radiation?","Completion":"GRAY"} {"Type":"TOSS-UP\n\nASTRONOMY Short Answer","Prompt":"What is the most common astronomical term for the geometrical configuration where Venus is between the Earth and the Sun?","Completion":"INFERIOR CONJUNCTION"} {"Type":"TOSS-UP\n\nPHYSICS Short Answer","Prompt":"What is the focal length, in centimeters and giving the proper sign, of a convex mirror that has a radius of curvature of 12 centimeters?","Completion":"-6"} {"Type":"TOSS-UP\n\nCHEMISTRY Short Answer","Prompt":"Predict the bond angles, in degrees, for atoms in a molecule of boron trichloride:","Completion":"120\\({}^{\\rm o}\\) (ACCEPT: THREE 120\\({}^{\\rm o}\\))"} {"Type":"BONUS\n\nCHEMISTRY Short Answer","Prompt":"Consider a flexible balloon filled with 3.0 moles of helium at a temperature of 27\\({}^{\\rm o}\\)C and at a pressure of 101.3 kilopascals. What is the volume in the balloon, in liters to the nearest whole number?","Completion":"74"} {"Type":"","Prompt":"TOSS-UP","Completion":"24) BIOLOGY Short Answer In which of the following 4 phases of cell cycle would a chemical that causes mispairing of bases during replication do the most damage: G-1; G-2; S; M S"} {"Type":"BONUS\n\nBIOLOGY Short Answer","Prompt":"For a dihybrid cross between parent plants that are each heterozygous for both traits, if there is complete dominance and independent assortment, what is the total fraction of the offspring that will have a recessive phenotype for either trait and the dominant for the other?","Completion":"6\/16 (ACCEPT: 3\/8)"} {"Type":"TOSS-UP\n\nPHYSICS Short Answer","Prompt":"What is the angular velocity, in radians per second, of a wagon wheel with a diameter of 2 meters and a surface velocity of 3 meters per second?","Completion":"3"} {"Type":"TOSS-UP\n\nCHEMISTRY Multiple Choice","Prompt":"Which of the following ionic salts has the LARGEST lattice dissociation energy?\n\nW) Sodium chloride\n\nX) Magnesium oxide\n\nY) Magnesium chloride\n\nZ) Lithium fluoride","Completion":"X) MAGNESIUM OXIDE"} {"Type":"BONUS\n\nCHEMISTRY Short Answer","Prompt":"If 30 milliliters of 0.10 molar sulfuric acid is titrated with 0.03 molar sodium hydroxide, how many milliliters of sodium hydroxide are required to reach the equivalence point?","Completion":"200"} {"Type":"TOSS-UP\n\nMATH Short Answer","Prompt":"In a group of 16 students, 12 take science and 8 take math. One student takes neither science nor math. How many students take both?","Completion":"5"} {"Type":"BONUS\n\nMATH Short Answer","Prompt":"If \\(a^{\\star}b=ab^{\\div}(a+b)\\) [a star b equals a times b divided by the quantity a plus b], then find the value of \\(4^{\\star}\\) (\\(4^{\\star}\\) [four star open parenthesis four star four close parenthesis]","Completion":"4\/3 (ACCEPT: 1 1\/3)"} {"Type":"TOSS-UP\n\nPHYSICS Multiple Choice","Prompt":"An electromagnetic wave goes into a block of glass with an index of refraction of n = 1.1. In terms of the speed of light C, which of the following is closest to the speed at which the wave propagates inside the glass medium?\n\nW) C\n\nX) 0.1C\n\nY) 0.9C\n\nZ) 1.1C","Completion":"Y) 0.9C"} {"Type":"BONUS\n\nPHYSICS Multiple Choice","Prompt":"Water is warmed by an immersed coil that generates 1000 joules of heat per second when an electric potential difference of 10 volts is applied across its ends. What is the resistance, in ohms, of the heater wire?\n\nW) 0.1\n\nX) 1\n\nY) 10\n\nZ) 100","Completion":"W) 0.1"} {"Type":"TOSS-UP\n\nEARTH AND SPACE Short Answer","Prompt":"If a galaxy is observed to have a recessional velocity of 150 km\/s, what is its distance, in megaparsecs, to one significant figure?","Completion":"2"} {"Type":"BONUS\n\nEARTH AND SPACE Multiple Choice","Prompt":"Which of the following best represents the maximum possible change in thickness of a single clay particle between its condition during hydration compared to dehydration?\n\nW) Less than 20%\n\nX) Approximately 50%\n\nY) Approximately 70%\n\nZ) Nearly 100%","Completion":"Z) NEARLY 100%"} {"Type":"TOSS-UP\n\nCHEMISTRY Multiple Choice","Prompt":"Which of the following statements does NOT describe a molecule in an octahedral arrangement?\n\nW) Two-fold, three-fold, and four-fold rotation axes can all be found\n\nX) Two different types of improper rotations can be found\n\nY) Exactly eight different mirror planes can be found\n\nZ) An inversion center is present","Completion":"Y) EXACTLY EIGHT DIFFERENT MIRROR PLANES CAN BE FOUND"} {"Type":"TOSS-UP\n\nBIOLOGY Multiple Choice","Prompt":"Which of the following is an example of commensalism?\n\nW) A hummingbird drinking from a flower\n\nX) A tapeworm living in a host intestine\n\nY) A clown fish living in a sea anemone [uh-NEM-uh-nee]\n\nZ) Egrets foraging among cattle fields","Completion":"Z) EGRETS FORAGING AMONG CATILE FIELDS"} {"Type":"TOSS-UP\n\nENERGY Multiple Choice","Prompt":"Which of the following is NOT an oil product?\n\nW) Gasoline\n\nX) Syngas\n\nY) Kerosene\n\nZ) Asphalt","Completion":"X) SYNGAS"} {"Type":"TOSS-UP\n\nEARTH AND SPACE Multiple Choice","Prompt":"Which of the following volcanoes in the Cascade Volcanic Arc is a shield volcano?\n\nW) Mt Rainier\n\nX) Mt St Helens\n\nY) Newberry Volcano\n\nZ) Glacier Peak","Completion":"Y) NEWBERRY VOLCANO"} {"Type":"TOSS-UP\n\nMATH Multiple Choice","Prompt":"For the expression \\(5^{x}=4^{x}\\), [five to the power of x equals four to the power of z] if x and z are positive, which of the following is true?\n\nW) \\(x^{x}=\\log\\frac{5}{4}\\)[x to the power of z equals log of five-fourths]\n\nX) \\(\\frac{x}{4}=\\log{5}(x)\\)[z divided by four equals log base five of x]\n\nY) \\(z-7\\) (read as: the absolute value of, open parenthesis, \\(x+5\\), close parenthesis, is greater than \\(-7\\))","Completion":"ALL REAL NUMBERS (ACCEPT: \\(+\\) INFINITY TO -INFINITY)"} {"Type":"TOSS-UP\n\nEARTH SCIENCE Short Answer","Prompt":"What is the name for the lowermost 12 to 18 kilometers of Earth's atmosphere?","Completion":"TROPOSPHERE"} {"Type":"TOSS-UP\n\nGENERAL SCIENCE Multiple Choice","Prompt":"An isotonic solution of sodium chloride, also known as normal saline, is what percent by mass:\n\nW) 0.045% X) 0.9% Y) 2.5% Z) 10%","Completion":"X) 0.9%"} {"Type":"TOSS-UP\n\nASTRONOMY Multiple Choice","Prompt":"Our Sun is a:\n\nW) A-type star\n\nX) B-type star\n\nY) G-type star\n\nZ) K-type star","Completion":"Y) G-TYPE STAR"} {"Type":"TOSS-UP\n\nCHEMISTRY Short Answer","Prompt":"What element is central to synthetic chemistry because of the unique ability of its atoms to bind to one another with single, double and triple bonds and form chains, branches and rings of varying sizes?","Completion":"CARBON"} {"Type":"TOSS-UP\n\nPHYSICS Multiple Choice","Prompt":"Which of the following is the SI unit for the product of newtons and meters:\n\nW) henry X) watt Y) erg Z) joule","Completion":"Z) JOULE"} {"Type":"TOSS-UP\n\nBIOLOGY Multiple Choice","Prompt":"The type of muscle that is in the walls of human arteries is:\n\nW) smooth X) striated Y) skeletal Z) fast twitch","Completion":"W) SMOOTH"} {"Type":"TOSS-UP\n\nMATH Short Answer","Prompt":"Simplify the following, giving your answer in proper scientific notation: \\(\\left(2.0\\times 10^{-6}\\right)^{4}\\)","Completion":"\\(1.6\\times 10^{-23}\\)"} {"Type":"BONUS\n\nMATH Short Answer","Prompt":"What is the sum of the first 5 terms of the geometric sequence in which the first term is 2 and the common ratio is 2?","Completion":"62"} {"Type":"TOSS-UP\n\nEARTH SCIENCE Multiple Choice","Prompt":"Which of the following is the largest global reserve of carbon on Earth:\n\nW) oceans\n\nX) soils\n\nY) vegetation\n\nZ) atmospheric CO\\({}{2}\\)","Completion":"W) OCEANS"} {"Type":"BONUS\n\nEARTH SCIENCE Short Answer","Prompt":"What moon of Saturn was shown by the Cassini space-craft to be spewing a watery, gassy plume from its geothermally heated south pole?","Completion":"ENCELADUS"} {"Type":"TOSS-UP\n\nGENERAL SCIENCE Multiple Choice","Prompt":"Which of the following is a trace element in coal that is a pollutant and neurotoxin released into the atmosphere when coal is burned, for which some states are currently developing technologies to reduce these emissions by 70 to 90%:\n\nW) sulfur\n\nX) mercury\n\nY) carbon\n\nZ) chromium","Completion":"X) MERCURY"} {"Type":"","Prompt":"TOSS-UP\n\nASTRONOMY Multiple choice Main sequence stars are found in a region on the H-R diagram that is oriented on the diagram from:\n\nW) upper left to lower left\n\nX) upper left to lower right\n\nY) upper right to lower left\n\nZ) upper right to lower right","Completion":"X) UPPER LEFT TO LOWER RIGHT"} {"Type":"TOSS-UP\n\nCHEMISTRY Short Answer","Prompt":"What is the MOST common method for separating a mixture of liquids based on the differences in component volatilities?","Completion":"DISTILLATION (ACCEPT: FRACTIONAL or SIMPLE DISTILLATION)"} {"Type":"BONUS\n\nCHEMISTRY Short Answer","Prompt":"What type of interaction occurs between molecules of H\\({}{2}\\)O that causes it to have a higher boiling point than H\\({}{2}\\)S?","Completion":"HYDROGEN BONDING"} {"Type":"TOSS-UP\n\nPHYSICS Short Answer","Prompt":"Giving your answer as north, south, east, or west relative to a point on the Equator, in what direction is the Earth's angular velocity as it rotates daily about its axis?","Completion":"NORTH"} {"Type":"BONUS\n\nPHYSICS Multiple Choice","Prompt":"Ignoring air resistance, wind velocity, and the rotation of the Earth, if a projectile is fired directly upwards, the horizontal component of the projectile's velocity:\n\nW) is zero\n\nX) remains a non zero constant\n\nY) continuously increases\n\nZ) continuously decreases","Completion":"W) IS ZERO"} {"Type":"TOSS-UP\n\nBIOLOGY Multiple Choice","Prompt":"Which of the following is a homologous structure to a whale flipper:\n\nW) insect wing\n\nX) human arm\n\nY) a fish operculum\n\nZ) shark fin","Completion":"X) HUMAN ARM"} {"Type":"BONUS\n\nBIOLOGY Short Answer","Prompt":"A sarcomere is a basic unit in what specific tissue of mammals?","Completion":"MUSCLE"} {"Type":"TOSS-UP\n\nMATH Short Answer","Prompt":"Evaluate the following radical expression: \\(5\\sqrt{3}-\\sqrt{75}\\)","Completion":"0"} {"Type":"BONUS\n\nMATH Short Answer","Prompt":"Find the positive geometric mean between the numbers 1 and 10,000:","Completion":"100"} {"Type":"TOSS-UP\n\nEARTH SCIENCE Multiple Choice","Prompt":"In which of the following locations is oceanic crust converging with oceanic crust:","Completion":"W) the Aleutian Islands X) the Himalayan Mountains Y) the Mid-Atlantic Ridge Z) the East Pacific Rise W) THE ALEUTIAN ISLANDS"} {"Type":"","Prompt":"BONUS","Completion":"19) EARTH SCIENCE Multiple Choice About what percent of all glaciers in the world are retreating today: W) 25% X) 50% Y) 75% Z) 100% Z) 100%"} {"Type":"","Prompt":"TOSS-UP","Completion":"20) GENERAL SCIENCE Multiple Choice The process in which fat globules in milk are broken up into smaller, more evenly dispersed particles is BEST described as: W) lactation X) pasteurization Y) fermentation Z) homogenization Z) HOMOGENIZATION"} {"Type":"","Prompt":"BONUS","Completion":"20) GENERAL SCIENCE Multiple Choice Which of the following is LEAST accurate regarding non-fuel petroleum products: W) petroleum jelly is often used to make household toiletries X) asphalt is often used in making roof shingles Y) petrochemicals are used to make latex Z) printing inks and paint solvents are often petroleum-based Y) PETROCHEMICALS ARE USED TO MAKE LATEX"} {"Type":"TOSS-UP\n\nASTRONOMY Short Answer","Prompt":"What scientific term is given to the apparent path of the Sun around the celestial sphere?","Completion":"THE ECLIPTIC"} {"Type":"TOSS-UP\n\nCHEMISTRY Short Answer","Prompt":"What general term BEST describes the use of electrical current to cause a normally non-spontaneous reaction to occur?","Completion":"ELECTROLYSIS"} {"Type":"TOSS-UP\n\nPHYSICS Multiple Choice","Prompt":"Neglecting air resistance, at what angle above the horizontal would you aim your garden hose nozzle to obtain the greatest horizontal distance if the nozzle is at ground level:\n\nW) 0\\({}^{\\circ}\\)\n\nX) 30\\({}^{\\circ}\\)\n\nY) 45\\({}^{\\circ}\\)\n\nZ) 60\\({}^{\\circ}\\)","Completion":"Y) 45\\({}^{\\circ}\\)"} {"Type":"TOSS-UP\n\nBIOLOGY Multiple Choice","Prompt":"The lambdoid suture is found in which of the following parts of the human body:\n\nW) scapula\n\nX) pelvis\n\nY) skull\n\nZ) knee","Completion":"Y) SKULL"} {"Type":"TOSS-UP\n\nPHYSICS Short Answer","Prompt":"What is the SI unit given to a volt per ampere?","Completion":"OHM"} {"Type":"TOSS-UP\n\nMATH Multiple Choice","Prompt":"If it takes you 50 minutes to walk uphill 2 miles to school but only 30 minutes to walk back, which of the following is your average speed, in miles per hour, for the round trip?\n\nW) 3\n\nX) 3.5\n\nY) 4\n\nZ) 4.5","Completion":"W) 3"} {"Type":"BONUS\n\nMATH Short Answer","Prompt":"If \\(a{1}=3\\) [a sub one equals three] and \\(a{n}=5+2a{n-1}\\) [a sub n equals five plus two a sub quantity n minus 1] for all integers n greater than 1, what is the value of \\(a{3}\\)[a sub three]?","Completion":"27"} {"Type":"TOSS-UP\n\nBIOLOGY Multiple Choice","Prompt":"Which of the following groups of organisms would contain the most biomass productivity in a trophic pyramid?\n\nW) Primary Producers\n\nX) Primary Consumers\n\nY) Detritivores [de-TRY-tuh-vohrs]\n\nZ) Tertiary Consumers","Completion":"W) PRIMARY PRODUCERS"} {"Type":"BONUS\n\nBIOLOGY Short Answer","Prompt":"What is the only multicellular animal without a permanent gut cavity?","Completion":"SPONGE (ACCEPT: PORIFERAN, PORIFERA, or PLACOZOA)"} {"Type":"TOSS-UP\n\nCHEMISTRY Multiple Choice","Prompt":"Which of the following scientists is credited with discovering electrons and calculating their mass to charge ratio?\n\nW) J.J. Thomson\n\nX) Robert Millikan\n\nY) Ernest Rutherford\n\nZ) Neils Bohr","Completion":"W) J.J. THOMSON"} {"Type":"TOSS-UP\n\nPHYSICS Short Answer","Prompt":"What physical constant is the magnitude of electric charge per mole of electrons?","Completion":"FARADAY CONSTANT"} {"Type":"TOSS-UP\n\nEARTH AND SPACE Short Answer","Prompt":"Joints are often formed in igneous [IG-nee-uhs] rocks, such as basalt, during what stage of the rock's development?","Completion":"COOLING"} {"Type":"BONUS\n\nEARTH AND SPACE Multiple Choice","Prompt":"What type of volcanic vent is Wizard Island, the only resurgent dome of Crater Lake that breaks the surface of the water line?\n\nW) Shield volcano\n\nX) Fissure\n\nY) Cinder cone\n\nZ) Stratovolcano","Completion":"Y) CINDER CONE"} {"Type":"TOSS-UP\n\nBIOLOGY Multiple Choice","Prompt":"Which of the following forms of nitrogen cannot be directly utilized by plants?\n\nW) Ammonia\n\nX) Ammonium\n\nY) Nitrogen gas\n\nZ) Nitrite","Completion":"Y) NITROGEN GAS"} {"Type":"BONUS\n\nBIOLOGY Multiple Choice","Prompt":"Which of the following was NOT a component of the Miller-Urey [MILL-er YOO-ree] experiment test conditions for the chemical origins of life?\n\nW) Carbon dioxide\n\nX) Hydrogen\n\nY) Ammonia\n\nZ) Water","Completion":"W) CARBON DIOXIDE"} {"Type":"TOSS-UP\n\nENERGY Multiple Choice","Prompt":"At what point in time is a CFL's [C-F- elz] energy consumption greatest, compared to its light output?\n\nW) Right after being turned on\n\nX) After being on for 15 minutes\n\nY) Right after being turned off\n\nZ) After being turned off for 15 minutes","Completion":"W) RIGHT AFTER BEING TURNED ON"} {"Type":"TOSS-UP\n\nCHEMISTRY Short Answer","Prompt":"Name the two common crystalline forms of carbon.","Completion":"GRAPHITE AND DIAMOND (ACCEPT: AND CARBON (IV))"} {"Type":"TOSS-UP\n\nPHYSICS Short Answer","Prompt":"Unpolarized light is passed through a single polarizing filter. What fraction of the intensity of the light emerges from the filter?","Completion":"1\/2"} {"Type":"TOSS-UP\n\nBIOLOGY Short Answer","Prompt":"Huntington disease is caused by the sequence CAG [C-A-G]\n\nrepeated between 37-100 times, rather than the normal 9-35 times. What type of mutation is this?","Completion":"INSERTION"} {"Type":"TOSS-UP\n\nEARTH AND SPACE Multiple Choice","Prompt":"Which of the following rocks can be biogenic?\n\nW) Schist [SHIFT]\n\nX) Basalt\n\nY) Limestone\n\nZ) Gabbro","Completion":"Y) LIMESTONE"} {"Type":"TOSS-UP\n\nCHEMISTRY Multiple Choice","Prompt":"Lipstick is known to be a suspension of high molecular weight hydrocarbons, waxes, and other ingredients. Which of the following choices does NOT describe lipstick composition?\n\nW) Castor oil may be a dispersing agent\n\nX) Beeswax is sometimes used as a stiffening agent\n\nY) Metal ions such as Nickel(ll) [nickel two] are the basis for lipstick dyes\n\nZ) Carbon soot is typically used as a coloring agent","Completion":"Z) CARBON SOOT IS TYPICALLY USED AS A COLORING AGENT"} {"Type":"TOSS-UP\n\nMATH Multiple Choice","Prompt":"Which of the following statements is true for the radius \\(r\\), height \\(h\\), and volume \\(V\\) of a right circular cylinder?\n\nW) If both \\(r\\) and \\(h\\) are doubled, \\(V\\) is doubled\n\nX) If \\(r\\) is increased by 2, \\(V\\) is increased by 4\n\nY) If \\(r\\) is doubled and \\(h\\) is halved, \\(V\\) is quadrupled\n\nZ) If \\(r\\) is doubled and \\(h\\) is divided by 4, \\(V\\) remains the same","Completion":"Z) IF \\(r\\) IS DOUBLED AND \\(h\\) IS DIVIDED BY 4, \\(V\\) REMAINS THE SAME"} {"Type":"TOSS-UP\n\nENERGY Short Answer","Prompt":"What is the term used to describe the medium in nuclear reactors that is used to reduce the speed of fast neutrons?","Completion":"MODERATOR"} {"Type":"TOSS-UP\n\nBIOLOGY Short Answer","Prompt":"What is a form of asexual reproduction in which young develop from unfertilized eggs?","Completion":"PARTHENOGENESIS"} {"Type":"TOSS-UP\n\nCHEMISTRY Short Answer","Prompt":"What is the chemical formula of the most stable ionic compound that forms between aluminum and sulfur?","Completion":"Al\\({}{2}\\)S\\({}{3}\\) (DO NOT ACCEPT: ALUMINNUM SULFIDE)"} {"Type":"TOSS-UP\n\nMATH Short Answer","Prompt":"In statistical testing, what term describes the default position, that there is no relationship between the two measured phenomena?","Completion":"NULL HYPOTHESIS"} {"Type":"TOSS-UP\n\nPHYSICS Short Answer","Prompt":"To the nearest whole decibel, a doubling of sound intensity is equal to an increase of how many decibels?","Completion":"3"} {"Type":"TOSS-UP\n\nEARTH ANDE SPACE Multiple Choice","Prompt":"Which of the following major U.S. rivers flows north?\n\nW) Snake River\n\nX) Connecticut River\n\nY) Missouri River\n\nZ) Sacramento River","Completion":"W) SNAKE RIVER"} {"Type":"TOSS-UP\n\nCHEMISTRY Short Answer","Prompt":"Gypsum wallboard, commonly known as drywall, is a laminar, composite material composed of front and back paper with a hydrated gypsum core. What is the chemical formula for the hydrated gypsum?","Completion":"CaSO\\({}{4}\\cdot 2\\) H\\({}{2}\\)O (DO NOT ACCEPT: CALCIUM SULFATE AND 2 MOLECULES OF WATER)"} {"Type":"TOSS-UP\n\nBIOLOGY Short Answer","Prompt":"In prokaryotic [proh-Kr-ee-OT-ik] DNA replication, what enzyme is responsible for unwinding the DNA double helix?","Completion":"HELICASE"} {"Type":"TOSS-UP\n\nENERGY Multiple Choice","Prompt":"Which of the following regions of the United States has the greatest amount of installed wind power capacity?\n\nW) West coast\n\nX) Midwest\n\nY) New England\n\nZ) Southeast","Completion":"X) MIDWEST"} {"Type":"TOSS-UP\n\nEARTH AND SPACE Multiple Choice","Prompt":"Which of the following lava characteristics does NOT affect the lava's viscosity?\n\nW) Gas amount\n\nX) Temperature\n\nY) Gas composition\n\nZ) Silica content","Completion":"Y) GAS COMPOSITION"} {"Type":"BONUS\n\nEARTH AND SPACE Short Answer","Prompt":"Anticlines are usually found in conjunction with what other type of fold?","Completion":"SYNCLINE"} {"Type":"TOSS-UP\n\nMATH Short Answer","Prompt":"If \\(log100^{2x}=2\\) [log base 10 of 100 to the power of 2x equals 2], what is the value of \\(x\\)?","Completion":"\\(\\frac{1}{2}\\) (ACCEPT: 0.5)"} {"Type":"BONUS\n\nMATH Short Answer","Prompt":"Simplify the following expression: \\(\\sec^{2}\\frac{3\\pi}{4}-\\csc^{2}\\frac{\\pi}{4}-\\cos^{2}4\\pi\\) [secant squared of 3 pi over 4 minus cosecant squared of pi over 4 minus cosine squared of 4 pi].","Completion":"-1"} {"Type":"TOSS-UP\n\nCHEMISTRY Short Answer","Prompt":"What is the electron configuration of chromium?","Completion":"1s\\({}^{2}\\) 2s\\({}^{2}\\) 2p\\({}^{6}\\) 3s\\({}^{2}\\) 3p\\({}^{6}\\) 4s\\({}^{1}\\) 3d\\({}^{5}\\) (ACCEPT: [Ar (argon)] 4s\\({}^{1}\\) 3d\\({}^{5}\\))"} {"Type":"BONUS\n\nCHEMISTRY Short Answer","Prompt":"Baeyer-Villiger oxidation is used to transform a ketone into what functional group?","Completion":"ESTER"} {"Type":"TOSS-UP\n\nPHYSICS Multiple Choice","Prompt":"A box is sitting on a ramp with a coefficient of static friction of 0.7. Which of the following is closest to the smallest ramp angle for which the box will begin to slide down the ramp?\n\nW) 30\\({}^{\\circ}\\)\n\nX) 35\\({}^{\\circ}\\)\n\nY) 45\\({}^{\\circ}\\)\n\nZ) 60\\({}^{\\circ}\\)","Completion":"X) 35\\({}^{\\circ}\\)"} {"Type":"BONUS\n\nPHYSICS Multiple Choice","Prompt":"Assuming that 1 atmosphere = 1.01 x 10\\({}^{5}\\) pascals and the density of water is 1000 kilograms per cubic meter, at what depth of water will the absolute pressure be 10.8 times the atmospheric pressure?\n\nW) 10.1 meters\n\nX) 101 meters\n\nY) 1,010 meters\n\nZ) 10.1 kilometers","Completion":"X) 101 METERS"} {"Type":"TOSS-UP\n\nEARTH AND SPACE Multiple Choice","Prompt":"Which of the following is closest to the percentage of incoming solar radiation that is absorbed by the ocean and land?\n\nW) 5%\n\nX) 15%\n\nY) 20%\n\nZ) 50%","Completion":"Z) 50%"} {"Type":"BONUS\n\nEARTH AND SPACE Multiple Choice","Prompt":"In which of the following silicate groups of minerals does olivine belong?\n\nW) Nessilicates [nee-soh-SIL-i-kayts]\n\nX) Inosilicates\n\nY) Sorosilicates\n\nZ) Phylosilicates [fil-oh-SIL-i-kayts]","Completion":"W) NESOSILICATES"} {"Type":"TOSS-UP\n\nBIOLOGY Multiple Choice","Prompt":"Eukaryotic [fvo-KAR-ee-oh-tik] genes are composed of introns and exons. Which of the following eukaryotes [fvo-KAR-ee-ohts] would have the shortest intron size?\n\nW) Mice\n\nX) Fruit flies\n\nY) Humans\n\nZ) Yeast","Completion":"Z) YEAST"} {"Type":"BONUS\n\nBIOLOGY Short Answer","Prompt":"In the production of mammalian proteins, the first amino acid in the polypeptide sequence is almost always the same amino acid. What is this amino acid that is prevalent in almost all proteins?","Completion":"METHIONINE (ACCEPT: MET AND M)"} {"Type":"TOSS-UP\n\nENERGY Multiple Choice","Prompt":"Which of the following has long been considered the upper theoretical percentage limit for solar cell efficiency?\n\nW) 24.5%\n\nX) 33.7%\n\nY) 41.2%\n\nZ) 62.8%","Completion":"X) 33.7%"} {"Type":"BONUS\n\nENERGY Multiple Choice","Prompt":"What property of hydrogen makes water a good neutron moderator?\n\nW) It has a small neutron scattering cross section\n\nX) It has a large neutron scattering cross section\n\nY) It is fissile\n\nZ) It evaporates at 373 Kelvin","Completion":"X) IT HAS A LARGE NEUTRON SCATTERING CROSS SECTION"} {"Type":"TOSS-UP\n\nMATH Short Answer","Prompt":"What is the probability that at least one head appears in 4 tosses of a fair coin?","Completion":"15\/16 (ACCEPT: 0.9375)"} {"Type":"TOSS-UP\n\nCHEMISTRY Multiple Choice","Prompt":"Which of the following is true of spontaneous processes at constant temperature, volume, and pressure?\n\nW) Gibbs and Helmholtz free energies increase\n\nX) Gibbs free energy increases; Helmholtz free energy decreases\n\nY) Gibbs and Helmholtz free energies decrease\n\nZ) Gibbs free energy decreases; Helmholtz free energy increases","Completion":"Y) GIBBS AND HELMHOLTZ FREE ENERGIES DECREASE"} {"Type":"TOSS-UP\n\nPHYSICS Short Answer","Prompt":"In order to play a low C followed by a high C, one octave up, on a violin string, the violinist must increase the frequency of the string by what factor?","Completion":"2"} {"Type":"TOSS-UP\n\nEARTH AND SPACE Multiple Choice","Prompt":"Most solar radiation is emitted in short wavelengths. Visible wavelengths fall in which of the following ranges in micrometers?\n\nW) 0.1 - 0.4\n\nX) 0.4 - 0.7\n\nY) 0.7 - 1.0\n\nZ) 1.0 - 1.3","Completion":"X) 0.4 - 0.7"} {"Type":"TOSS-UP\n\nBIOLOGY Short Answer","Prompt":"During which phase of interphase does expression of proteins needed to prepare for DNA replication increase?","Completion":"G1"} {"Type":"TOSS-UP\n\nENERGY Short Answer","Prompt":"In an isothermal system, what is one way that one can increase the number of interactions of a gas with a solid surface?","Completion":"INCREASE PRESSURE (ACCEPT: DECREASE VOLUME)"} {"Type":"TOSS-UP\n\nMATH Multiple Choice","Prompt":"What is the general solution to the differential equation:\n\ndy\/dx = \\(2x+1\\) [dy dx equals 2x plus 1]?\n\nW) \\(y=x^{2}+x+c\\)\n\nX) \\(y=1\/2\\)\\(x^{2}+c\\)\n\nY) \\(y=2+c\\)\n\nZ) \\(y=2x+c\\)","Completion":"W) \\(y=x^{2}+x+c\\)"} {"Type":"BONUS\n\nMATH Short Answer","Prompt":"Simplify: \\(x^{1\/2}\\) (\\(x^{1\/2}+x^{1\/2}\\)) [x to the power of one half times open parenthesis x to the power of one half plus x to the power of one half close parenthesis].","Completion":"\\(2x\\)"} {"Type":"TOSS-UP\n\nCHEMISTRY Short Answer","Prompt":"In lab, you have patented the creation of a metallic radioactive isotope called Sciencebowlium-23. But, after a period of time, the sample radioactively decays into oxygen-16, where it becomes a gas and dissipates into the atmosphere. You create a sample of Sciencebowlium and weigh it as 500 grams. Exactly four days later, you weigh the sample at 62.5 grams. How many hours long is the half-life of Sciencebowlium-23?","Completion":"32"} {"Type":"BONUS\n\nCHEMISTRY Short Answer","Prompt":"What is the bond order of the nitrogen-oxygen bond in the nitrate anion?","Completion":"1.33 (ACCEPT: 4\/3 or 1 1\/3)"} {"Type":"TOSS-UP\n\nPHYSICS Multiple Choice","Prompt":"Two satellites of masses \\(m{1}\\) and \\(m{2}\\), where \\(m{1}=4m{2}\\), are deployed in circular orbits of the same radius in the gravitational field of Earth. What is the ratio of their linear speeds \\( u{1}\\)\/\\( u{2}\\)?","Completion":"W) 1\/4 X) 1 Y) 2 Z) 4 X) 1"} {"Type":"BONUS\n\nPHYSICS Multiple Choice","Prompt":"Assuming h = 6.63 x 10\\({}^{-34}\\) joule seconds, what is the approximate energy of a photon in joules in a 1000 nanometer light beam?\n\nW) 2 x 10\\({}^{-19}\\) X) 2 x 10\\({}^{-18}\\) Y) 2 x 10\\({}^{-17}\\) Z) 4 x 10\\({}^{-18}\\)","Completion":"W) 2 x 10\\({}^{-19}\\)"} {"Type":"TOSS-UP\n\nEARTH AND SPACE Short Answer","Prompt":"What type of front travels at about 30 miles per hour and has a slope of about 1:100?","Completion":"COLD"} {"Type":"BONUS\n\nEARTH AND SPACE Short Answer","Prompt":"What is the name of the deepest lake in Africa that was formed from continental rifting?","Completion":"LAKE TANGANYIKA"} {"Type":"TOSS-UP\n\nBIOLOGY Multiple Choice","Prompt":"A type of albinism in the Hopi Native American population is caused by mutation of the OCA2 gene. If the dominant form of the gene gives a person normal pigmentation, then which of the following genotypes would result in albinism?\n\nW) Heterozygous co-dominance [het-er-uh-ZY-guhs]\n\nX) Homozygous dominant [hoh-muh-ZY-guhs]\n\nY) Homozygous recessive\n\nZ) Heterozygous incomplete dominance","Completion":"Y) HOMOZYGOUS RECESSIVE"} {"Type":"BONUS\n\nBIOLOGY Short Answer","Prompt":"Identify all of the following five choices that are diploid: spermatagonium [spur-mat-uh-GOH-nee-uhm], spermatids, secondary spermatocyte [spuhr-MAT-uh-syt], primary spermatocyte, mature sperm cells.","Completion":"1, 4 (ACCEPT: SPECTMATOGONIUM, PRIMARY SPECTMATOCTYE)"} {"Type":"TOSS-UP\n\nPHYSICS Multiple Choice","Prompt":"In order to see a full height image of your body, the vertical size of a mirror should be at least which of the following?\n\nW) One-fourth of your height\n\nX) Half of your height\n\nY) Your height\n\nZ) Twice your height","Completion":"X) HALF OF YOUR HEIGHT"} {"Type":"BONUS\n\nPHYSICS Multiple Choice","Prompt":"A mass spectrometry experiment is performed with a magnetic field of 0.2 newtons per ampere meter and an electric field of 3.0 x 10\\({}^{5}\\) volts per meter. What is the speed of the ions?\n\nW) 750 meters per second\n\nX) 1500 meters per second\n\nY) 150 kilometers per second\n\nZ) 1500 kilometers per second","Completion":"Z) 1500 KILOMETERS PER SECOND"} {"Type":"TOSS-UP\n\nMATH Multiple Choice","Prompt":"What is the derivative of \\(y=(3x+^{2}\\)Jy equals 3x plus 1 quantity squared?\n\nW) \\(18x+6\\)\n\nX) \\(18x\\)\n\nY) \\(6x+1\\)\n\nZ) \\(6x+2\\)","Completion":"W) \\(18x+6\\)"} {"Type":"BONUS\n\nMATH Short Answer","Prompt":"What is the equation, in slope-intercept form, of the perpendicular bisector of the line segment with endpoints (4, and (-6, ?","Completion":"\\(y=\\) -5\\(x+4\\)"} {"Type":"TOSS-UP\n\nCHEMISTRY Multiple Choice","Prompt":"At high temperatures, nitrogen (N\\({}{2}\\)) and oxygen (O\\({}{2}\\)) gases react to form nitric oxide (NO) in a highly endothermic process. In a closed vessel, the three compounds form an equilibrium [ee-kwuh-LIB-ree-uhm]. Which of the following changes to the vessel will result in the production of more NO?\n\nW) Addition of helium at constant volume\n\nX) Adding a catalyst\n\nY) Increasing the temperature of the vessel\n\nZ) Removing oxygen from the vessel","Completion":"Y) INCREASING THE TEMPERATURE OF THE VESSEL"} {"Type":"BONUS\n\nCHEMISTRY Short Answer","Prompt":"Dinitrogen tetroxide is a powerful oxidizer and can be used with hydrazine fuels to propel rockets. What is the point group of dinitrogen tetroxide?","Completion":"\\(D{2h}\\)"} {"Type":"TOSS-UP\n\nPHYSICS Multiple Choice","Prompt":"A light ray passes from air (\\(n{1}=1.00\\)) to three layers of the human eye: cornea (\\(n{2}=1.33\\)), lens (\\(n{3}=1.41\\)), and vitreous humor (\\(n{4}=1.33\\)). The speeds of light in these mediums are, respectively, \\( u{1}\\), \\( u{2}\\), \\( u{3}\\), \\( u{4}\\). Which of the following must be true about the relationships between the speeds of light in the different media?\n\nW) \\( u{1}< u{2}< u{3}< u{4}\\)\n\nX) \\( u{4}< u{3}< u{2}< u{1}\\)\n\nY) \\( u{3}< u{2}\\) and \\( u{4}< u{1}\\)\n\nZ) \\( u{2}< u{3}\\) and \\( u{1}< u{4}\\)","Completion":"Y) \\( u{3}< u{2}\\) AND \\( u{4}< u{1}\\)"} {"Type":"BONUS\n\nPHYSICS Short Answer","Prompt":"Two satellites are orbiting in circular orbits of radii \\(r{1}\\) and \\(r{2}\\) around a planet, where \\(r{1}=4r{2}\\). What is the ratio of their periods of revolutions \\(T{1}\/T{2}\\)?","Completion":"8"} {"Type":"TOSS-UP\n\nEARTH AND SPACE Short Answer","Prompt":"What international agreement, initially drafted in 1987, was designed to phase out production of chlorofluorocarbons [klohr-oh-floor-oh-KAHR-bunhs] that cause damage to the stratospheric ozone layer?","Completion":"MONTREAL PROTOCOL"} {"Type":"BONUS\n\nEARTH AND SPACE Multiple Choice","Prompt":"The shells of which of the following zooplankton groups are used in carbon-14 dating of ocean sediments?\n\nW) Polycistina\n\nX) Foraminifera [fuh-ram-uh-NIF-er-uh]\n\nY) Acantheria [uh-kan-THUH-ree-uh]\n\nZ) Ciliata [sil-ee-AY-tuh]","Completion":"X) FORAMIINFERA"} {"Type":"TOSS-UP\n\nBIOLOGY Short Answer","Prompt":"If there are two alleles for the gene encoding size, and if the frequency of the big size allele, B [upper case B], in a population of warthogs is 0.4, then what is the frequency of the small size allele, b [lower case b], assuming Hardy Weinberg equilibrium [ee-kwwth-LIB-ree-uhm]?","Completion":"0.6"} {"Type":"BONUS\n\nBIOLOGY Short Answer","Prompt":"Arrange the following four groups from the taxa containing the largest number of known living species to the group containing the least: Amphibians, Mammals, Fishes, Birds.","Completion":"3, 4, 1, 2 (ACCEPT: FISHES, BIRDS, AMPHIBIANS, MammALS)"} {"Type":"TOSS-UP\n\nENERGY Multiple Choice","Prompt":"The Bakken Oil\/Gas field is estimated to contain up to 24 billion barrels of oil. In which of the following states, among others, is it located?\n\nW) North Dakota\n\nX) Oklahoma\n\nY) Pennsylvania\n\nZ) Texas","Completion":"W) NORTH DAKOTA"} {"Type":"BONUS\n\nENERGY Multiple Choice","Prompt":"A DOE geothermal research project has established the effectiveness of enhanced geothermal systems (EGS) for utilizing geothermal energy for the generation of electricity. Using this approach, which of the following percentages of total U.S. current generating capacity could be generated with EGS?\n\nW) 5%\n\nX)15%\n\nY) 25%\n\nZ) 50%","Completion":"Z) 50%"} {"Type":"TOSS-UP\n\nBIOLOGY Multiple Choice","Prompt":"Which of the following animals would have the highest levels of myoglobin [my-uh-GLOH-bin] in their muscle?\n\nW) American pronghorn\n\nX) Human being\n\nY) Weddell seal\n\nZ) Kangaroo rat","Completion":"Y) WEDDELL SEAL"} {"Type":"BONUS\n\nBIOLOGY Multiple Choice","Prompt":"Some virus species can infect an individual host more than once because they change the shape of molecules exposed on the surface of their virions [VY-re-ons] so that they are no longer recognized by the host's immunological memory. The influenza virus can make sudden and drastic changes in these molecules due to a phenomenon called reassortment. Which of the following contributes to the process of reassortment?\n\nW) Widely varying strains of influenza\n\nX) The diversity of animals infected by influenza\n\nY) The segmented genome found in influenza\n\nZ) The lack of effective antivirals against influenza until recently","Completion":"Y) THE SEGMENTED GENOME FOUND INFLUENZA"} {"Type":"TOSS-UP\n\nPHYSICS Multiple Choice","Prompt":"The slope of a velocity versus time graph represents:\n\nW) acceleration\n\nX) velocity\n\nY) position\n\nZ) displacement","Completion":"W) ACCELERATION"} {"Type":"BONUS\n\nPHYSICS Multiple Choice","Prompt":"Under which of the following conditions is the average velocity equal to the average of an object's initial and final velocity:\n\nW) always\n\nX) never\n\nY) only when the acceleration changes\n\nZ) only when the acceleration is constant","Completion":"Z) ONLY WHEN THE ACCELERATION IS CONSTANT"} {"Type":"TOSS-UP\n\nCHEMISTRY Multiple Choice","Prompt":"Which of the following is an EXTENSIVE property of matter:\n\nW) temperature\n\nX) density\n\nY) mass\n\nZ) viscosity","Completion":"Y) MASS"} {"Type":"BONUS\n\nCHEMISTRY Short Answer","Prompt":"What is the name for the phenomenon that occurs when a light beam becomes visible after passing through a colloid, an example of which is a beam of light passing through fog at night?","Completion":"THE TYNDALL EFFECT"} {"Type":"TOSS-UP\n\nBIOLOGY Short Answer","Prompt":"How many autosomes are in a normal nucleated human nerve cell in G-zero of the cell cycle?","Completion":"44"} {"Type":"BONUS\n\nBIOLOGY Multiple Choice","Prompt":"Which of the following is NOT true of the human body:\n\nW) there are substantial numbers of lymph nodes in the wall of the large intestine\n\nX) veins have comparatively thick walls with more muscle than arteries\n\nY) there are two layers of smooth muscle in the walls of the small intestine\n\nZ) the small intestine is longer than the large intestine","Completion":"X) VEINS HAVE COMPARATIVELY THICK WALLS WITH MORE MUSCLE THAN ARTERIES"} {"Type":"TOSS-UP\n\nMATH Short Answer","Prompt":"What is the side of a square, in centimeters to the nearest whole number, given that its diagonal is 16 centimeters?","Completion":"11"} {"Type":"BONUS\n\nMATH Short Answer","Prompt":"Solve the following equation for \\(x\\): \\(-10\\mid(x+\\mid=-50\\) (read as: negative 10 times the absolute value of, open parenthesis, \\(x+2\\), close parenthesis, equals negative","Completion":"3 AND -7"} {"Type":"TOSS-UP\n\nCHEMISTRY Multiple Choice","Prompt":"Which of the following techniques employs a moving phase and a stationary phase to separate components of a mixture:\n\nW) filtration\n\nX) centrifugation\n\nY) distillation\n\nZ) chromatography","Completion":"Z) CHROMATOGRAHY"} {"Type":"TOSS-UP\n\nBIOLOGY Multiple Choice","Prompt":"What specific part of the human brain has the most influence on heart rate, sleep, body temperature, and metabolism, and is a large part of the ventral portion of the diencephalon (read as: die-en-SEFF-ah-lon):\n\nW) hypothalamus (read as: hypo-THAL-ah-mus)\n\nX) cerebellum\n\nY) pituitary\n\nZ) sella turcica (read as: TURR-sik-ah)","Completion":"W) HYPOTHALAMUS"} {"Type":"TOSS-UP\n\nMATH Short Answer","Prompt":"Solve the following inequality for \\(x\\): \\(x^{2}<3x\\)","Completion":"\\(04\\)[g of x is greater than 4]?","Completion":"\\(x<\\) -1 OR \\(x>3\\) (MUST GIVE BOTH ANSWERS)"} {"Type":"TOSS-UP\n\nENERGY Multiple Choice","Prompt":"Which of the following metals reacts violently with water?\n\nW) Gold\n\nX) Lithium\n\nY) Iron\n\nZ) Mercury","Completion":"X) LITHIUM"} {"Type":"BONUS\n\nENERGY Multiple Choice","Prompt":"How many electrons are transferred for the complete electrochemical oxidation of one hydrogen molecule in a fuel cell?\n\nW) 1\n\nX) 2\n\nY) 4\n\nZ) 6","Completion":"X) 2"} {"Type":"TOSS-UP\n\nEARTH AND SPACE Multiple Choice","Prompt":"Which of the following factors is most important in controlling the density of seawater?\n\nW) Temperature\n\nX) Salinity\n\nY) Pressure\n\nZ) Amount of oxygen in water","Completion":"W) TEMPERATURE"} {"Type":"TOSS-UP\n\nCHEMISTRY Multiple Choice","Prompt":"Which of the following statements is NOT true of polyunsaturated fatty acids?\n\nW) They are highly concentrated in cell membranes of the nervous system\n\nX) They are the most prevalent fatty acid found in the human body\n\nY) They contain two or more double bonds\n\nZ) They play a crucial role in brain function","Completion":"X) THEY ARE THE MOST PREVALENT FATTY ACID FOUND IN THE HUMAN BODY"} {"Type":"TOSS-UP\n\nBIOLOGY Multiple Choice","Prompt":"Which of the following portions of a gene could ultimately be translated to protein?\n\nW) Exon\n\nX) Terminator\n\nY) Intron\n\nZ) Promoter","Completion":"W) EXON"} {"Type":"TOSS-UP\n\nPHYSICS Multiple Choice","Prompt":"A projectile is launched tangentially to the Earth's surface with a speed such that it will neither collide with the Earth nor escape its gravity. Ignoring air resistance, this projectile's trajectory will have what shape?\n\nW) Parabolic\n\nX) Circular\n\nY) Elliptical\n\nX) Hyperbolic","Completion":"Y) ELLIPTICAL"} {"Type":"## BONUS\n\nMATH Short Answer","Prompt":"Given \\(f(x)=\\sqrt{2-x}\\) [If of \\(x\\) equals the square root of the quantity 2 minus x], for what value or values of \\(x\\) does \\(f(x+=1\\) [If of the quantity \\(x\\) plus 2 equals 1]?","Completion":"\\(x\\) = -1"} {"Type":"## BONUS\n\nENERGY Short Answer","Prompt":"In megajoules, how much energy does a 2.5 kilowatt stove use if turned on for two hours?","Completion":"18"} {"Type":"TOSS-UP\n\nEARTH AND SPACE Multiple Choice","Prompt":"The Coriolis effect causes objects moving in the Northern Hemisphere to veer in which of the following ways?\n\nW) To the right, or clockwise when viewed from above\n\nX) To the left, or counterclockwise when viewed from above\n\nY) In an upward direction\n\nZ) In a downward direction","Completion":"W) TO THE RIGHT, OR CLOCKWISE WHEN VIEWED FROM ABOVE"} {"Type":"TOSS-UP\n\nCHEMISTRY Multiple Choice","Prompt":"Which of the following is generally NOT considered to be an appropriate S\\({}{N}\\)2 electrophile?\n\nW) Benzyl bromide\n\nX) Allyl bromide\n\nY) Propargyl bromide\n\nZ) Vinyl bromide","Completion":"Z) VINYL BROMIDE"} {"Type":"TOSS-UP\n\nBIOLOGY Multiple Choice","Prompt":"In a landmark 1958 study by researchers Meselson and Stahl involving nitrogen isotope labeling, which of the following important DNA replication mechanisms was supported by their data?\n\nW) Semiconservative\n\nX) Dispersive\n\nY) Conservative\n\nZ) Transformative","Completion":"W) SEMICONSERVATIVE"} {"Type":"TOSS-UP\n\nPHYSICS Short Answer","Prompt":"Ryan, who weighs 150 pounds, and Kate, who weighs 50 pounds, are standing on slippery ice and push off from each other. Ignoring friction, if Ryan moves off at 1 meter per second, Kate moves off with what speed in meters per second?","Completion":"3"} {"Type":"TOSS-UP\n\nMATH Short Answer","Prompt":"The IQ of a Martian varies directly with the square of the number of tentacles it has. If a Martian with 5 tentacles has an IQ of 75, what is the IQ of a Martian having 8 tentacles?","Completion":"192"} {"Type":"BONUS\n\nMATH Multiple Choice","Prompt":"If \\(y=2\\)cos\\({}^{2}\\)x [y equals 2 cosine squared of x], what is dy\/dx at \\(x=\\pi\/4\\)?","Completion":"W) -2 X) -1 Y) 1 Z) 2 W) -2"} {"Type":"TOSS-UP\n\nEARTH AND SPACE Short Answer","Prompt":"What two underwater events can cause tsunamis?","Completion":"EARTHQUAKES AND VOLCANIC ERUPTIONS (ACCEPT: SUBMARINE"} {"Type":"BONUS\n\nEARTH AND SPACE Multiple Choice","Prompt":"Even if nothing was done to clean up the 2010\n\nDeepwater Horizon oil spill in the Gulf of Mexico, which of the following areas would likely remain unaffected by the oil floating on the surface?\n\nW) Brazil\n\nX) Florida\n\nY) Cape Hatteras\n\nZ) Yucatan Peninsula","Completion":"W) BRAZIL"} {"Type":"TOSS-UP\n\nCHEMISTRY Multiple Choice","Prompt":"According to the integrated form of the Clausius [KLOU-zee-uhs]-Clapeyron equation solved for the natural logarithm of vapor pressure, vapor pressure and absolute temperature data can be plotted to give a straight line. Which of the following corresponds to the slope?\n\nW) -\\(\\Delta\\)H\\({}{\\rm vap}\\)\/R [negative delta H of vaporization over R]\n\nX) \\(\\Delta\\)H\\({}{\\rm vap}\\)\/R [delta H of vaporization over R]\n\nY) -R\/\\(\\Delta\\)H\\({}{\\rm vap}\\) [negative R over delta H of vaporization]\n\nZ) R\/\\(\\Delta\\)H\\({}{\\rm vap}\\) [R over delta H of vaporization]","Completion":"W) -\\(\\Delta\\)H\\({}{\\rm vap}\\)\/R"} {"Type":"BONUS\n\nCHEMISTRY Short Answer","Prompt":"Place the following four substances in order from lowest to highest boiling point: carbon tetrachloride, ethane, propane, water.","Completion":"2, 3, 1, 4 (ACCEPT: ETHANE, PROPANE, CARBON TETRACHLORIDE, WATER)"} {"Type":"TOSS-UP\n\nBIOLOGY Short Answer","Prompt":"What is the one type of infectious agent that contains no nucleic acid?","Completion":"PRION"} {"Type":"BONUS\n\nBIOLOGY Multiple Choice","Prompt":"Loss of the retinoblastoma gene product, Rb, causes cancer by affecting which of the following portions of the cell cycle?\n\nW) G1\/S checkpoint\n\nX) S\/G2 checkpoint\n\nY) G2\/M checkpoint\n\nZ) M\/G1 checkpoint","Completion":"W) G1\/S CHECKPOINT"} {"Type":"TOSS-UP\n\nPHYSICS Multiple Choice","Prompt":"Which of the following quantities has an inverse-cubic relationship with distance?\n\nW) The magnitude of the gravitational force between two point masses\n\nX) The electric potential energy for a point charge\n\nY) The magnetic field strength at an axial point from the center of a current loop\n\nZ) The speed of a satellite in circular orbit around the Earth","Completion":"Y) THE MAGNETIC FIELD STRENGTH AT AN AYIAL POINT FROM THE CENTER OF A CURRENT LOOP"} {"Type":"BONUS\n\nPHYSICS Short Answer","Prompt":"A horizontal water pipe tapers so that the cross-sectional area of the pipe decreases. If the water moves at 3 meters per second in the larger portion of the pipe and at 15 meters per second in the smaller portion, what is the change in pressure, in pascals, as the water moves from the larger portion to the smaller portion of the pipe? Assume the density of water is 1000 kilograms per cubic meter and incompressible.","Completion":"-108000 (DO NOT ACCEPT: 108000)"} {"Type":"TOSS-UP\n\nPHYSICS Short Answer","Prompt":"What is the resulting electrical potential, in volts, when a charge of 12 coulombs is applied to a 1 farad capacitor?","Completion":"12"} {"Type":"BONUS\n\nPHYSICS Multiple Choice","Prompt":"Which of the following is NOT true:\n\nW) the magnetic field associated with the current on a straight long wire is inversely proportional to the distance from the wire\n\nX) Kirchhoff's first rule has to do with the accounting of total charges entering and leaving a junction per unit time\n\nY) superconducting quantum interference devices, or SQUID's, are based on the Josephson Effect\n\nZ) resistance and voltage are the two most common parameters used to characterize a resistor","Completion":"Z) RESISTANCE AND VOLTAGE ARE THE TWO MOST COMMON"} {"Type":"TOSS-UP\n\nCHEMISTRY Multiple Choice","Prompt":"Which of the following is NOT a colligative property:\n\nW) vapor pressure lowering\n\nX) osmotic pressure\n\nY) boiling point elevation\n\nZ) color of solution","Completion":"Z) COLOR OF SOLUTION"} {"Type":"BONUS\n\nCHEMISTRY Multiple Choice","Prompt":"Which of the following is a type of colloid that is classified as a sol at room temperature:\n\nW) gelatin\n\nX) milk\n\nY) whipped cream\n\nZ) marshallow","Completion":"W) GELATIN"} {"Type":"TOSS-UP\n\nBIOLOGY Multiple Choice","Prompt":"Which of the following tissues is MOST directly responsible for the increased diameter of a tree trunk from year to year:\n\nW) apical meristem\n\nX) pericycle\n\nY) vascular cambium\n\nZ) pith","Completion":"Y) VASCULAR CAMBIUM"} {"Type":"TOSS-UP\n\nMATH Short Answer","Prompt":"Convert log base 4 of 53 into a base 10 expression:","Completion":"\\(\\frac{\\log 53}{\\log 4}\\) (ACCEPT: \\(\\frac{\\log{10}53}{\\log{10}4}\\) )"} {"Type":"TOSS-UP\n\nEARTH SCIENCE Multiple Choice","Prompt":"Which of the following is a man-made coastal structure that causes coastal erosion by blocking oceanic longshore drift:","Completion":"W) jetty X) barrier island Y) atoll Z) fringing reef W) JETTY"} {"Type":"BONUS\n\nEARTH SCIENCE Short Answer","Prompt":"Used as a way to categorize rivers and streams, what is the term for the volume of water flowing through the cross-section of a channel over a given period of time?","Completion":"DISCHARGE (ACCEPT: DISCHARGE RATE)"} {"Type":"TOSS-UP\n\nGENERAL SCIENCE Multiple Choice","Prompt":"Case hardening is a process through which:\n\nW) the surface of a metal is hardened X) the metal is hardened completely through Y) the hardness of a metal is determined Z) metals are shaped or formed","Completion":"W) THE SURFACE OF A METAL IS HARDENED"} {"Type":"BONUS\n\nGENERAL SCIENCE Multiple Choice","Prompt":"The oil used with an oil-immersion objective in compound light microscopes takes advantage of what optical phenomenon to improve magnification:\n\nW) chromatic coherence X) index of refraction Y) polarization Z) dispersion","Completion":"X) INDEX OF REFRACTION"} {"Type":"TOSS-UP\n\nPHYSICS Multiple Choice","Prompt":"Which of the following has the highest vapor pressure at standard temperature and pressure:\n\nW)mercury\n\nX)ethyl alcohol\n\nY)methyl alcohol\n\nZ)acetone","Completion":"Z)ACETONE"} {"Type":"TOSS-UP\n\nCHEMISTRY Multiple Choice","Prompt":"According to the Arrhenius theory, an Arrhenius acid is any substance that does which of the following in a chemical reaction:\n\nW) donates a pair of electrons\n\nX) accepts a pair of electrons\n\nY) produces H\\({}^{+}\\) ions in aqueous solution\n\nZ) produces OH\\({}^{-}\\) ions in aqueous solution","Completion":"Y) PRODUCES H\\({}^{+}\\) IONS IN AQUEOUS SOLUTION"} {"Type":"BONUS\n\nCHEMISTRY Short Answer","Prompt":"If you were to evaporate 1 liter or kilogram of water, how many calories of latent heat are stored in the vapor?","Completion":"540,000 (ACCEPT: \\(5.4\\times 10^{5}\\))"} {"Type":"","Prompt":"TOSS-UP","Completion":"10) BIOLOGY Short Answer During meiosis, how many chromatids are in a chromosomal tetrad? 4"} {"Type":"BONUS\n\nBIOLOGY Multiple Choice","Prompt":"Which of the following is NOT true of nucleated mammalian cells:\n\nW) mast cells contain large amounts of histamine\n\nX) the golgi apparatus processes proteins for exocytosis\n\nY) microtubules are components in cilia\n\nZ) all the DNA in a nucleated cell is in the nucleus","Completion":"Z) ALL THE DNA IN A HUMAN NUCLEATED CELL IS IN THE NUCLEUS"} {"Type":"TOSS-UP\n\nMATH Short Answer","Prompt":"Solve the following equation for \\(x\\):\n\n\\(x^{2}-20x+19=0\\)","Completion":"19 AND 1"} {"Type":"BONUS\n\nMATH Short Answer","Prompt":"Express the following radical in its simplest radical form:\n\n\\(-2\\sqrt{(80a^{2}b^{4}c^{3})}\\)","Completion":"\\(-8ab^{2}c\\sqrt{5c}\\)"} {"Type":"TOSS-UP\n\nEARTH SCIENCE Multiple Choice","Prompt":"Which of the following is NOT true about Earth's history:\n\nW) the Ordovician Period predates the Silurian Period\n\nX) we currently live in the Cenozoic Era\n\nY) we currently live in the Pleistocene Epoch\n\nZ) the Holocene Epoch began about 10,000 years ago","Completion":"Y) WE CURRENTLY LIVE IN THE PLEISTOCENE EPOCH"} {"Type":"BONUS\n\nEARTH SCIENCE Multiple Choice","Prompt":"Which of the following is the global wind system that most influences weather in the U.S.:\n\nW) prevailing westerlies\n\nX) tropical easterlies\n\nY) trade winds\n\nZ) polar easterlies","Completion":"W) PREVAILING WESTERLIES"} {"Type":"TOSS-UP\n\nGENERAL SCIENCE Short Answer","Prompt":"One metric ton equals how many grams?","Completion":"ONE MILLION (ACCEPT: 1 \\(\\times\\) 10\\({}^{6}\\) or 10\\({}^{6}\\))"} {"Type":"TOSS-UP\n\nASTRONOMY Short Answer","Prompt":"With reference to the Sun's position in the sky, when does a full Moon set?","Completion":"WHEN THE SUN RISES"} {"Type":"TOSS-UP\n\nCHEMISTRY Short Answer","Prompt":"How many carbon and hydrogen atoms are in a cyclopentane molecule?","Completion":"CARBON = 5; HYDROGEN = 10"} {"Type":"BONUS\n\nCHEMISTRY Short Answer","Prompt":"What is the systematic IUPAC name for the following acid:","Completion":"HCIO HYPOCHLORUS ACID"} {"Type":"TOSS-UP\n\nBIOLOGY Short Answer","Prompt":"What specific part of any member of the Anthophyta produces pollen?","Completion":"STAMEN (ACCEPT: ANTHER)"} {"Type":"TOSS-UP\n\nMATH Short Answer","Prompt":"Subtract the following complex numbers, giving your answer in standard form: \\((11-4i)-(-3+3i)\\)","Completion":"\\(14-7i\\)"} {"Type":"TOSS-UP\n\nEARTH SCIENCE Multiple Choice","Prompt":"The time interval between the generation of P and S waves from its arrival at a seismometer is BEST translated as:\n\nW): the magnitude of the earthquake\n\nX): the distance to the epicenter\n\nY): the wave height of a potential tsunami\n\nZ): the distance to the nearest fault zone","Completion":"X) THE DISTANCE TO THE EPICENTER"} {"Type":"TOSS-UP\n\nGENERAL SCIENCE Short Answer","Prompt":"What substance made of several fibrous materials was widely used in the 20th century for its heat resistant properties but is now banned because it can cause lung cancer when inhaled?","Completion":"ASBESTOS"} {"Type":"TOSS-UP\n\nASTRONOMY Short Answer","Prompt":"What planet is never more than 28\\({}^{\\rm o}\\) from the Sun in the evening sky?","Completion":"MERCURY"} {"Type":"BONUS\n\nASTRONOMY Multiple Choice","Prompt":"At 8:00 AM in early August of 2006 in New York, the Moon could be seen low in the western sky. What phase was it most likely in:\n\nW) waxing crescent\n\nX) just past new\n\nY) a few days past full\n\nZ) first quarter","Completion":"Y) A FEW DAYS PAST FULL"} {"Type":"TOSS-UP\n\nPHYSICS Short Answer","Prompt":"How many nodes are there in the first overtone in a standing wave of a vibrating guitar string?","Completion":"3"} {"Type":"BONUS\n\nPHYSICS Short Answer","Prompt":"If g = 9.8 meters per second squared, to the first decimal place and in newtons, how many newtons of force are required to keep a 500 kilogram slab of granite moving across a horizontal surface at constant speed if there is a kinetic frictional coefficient of 0.10 between the surface and the slab?","Completion":"490"} {"Type":"TOSS-UP\n\nCHEMISTRY Multiple Choice","Prompt":"Organic compounds that contain double bonds are said to be:\n\nW) saturated\n\nX) unsaturated\n\nY) unable to undergo cis-trans isomerization\n\nZ) unable to undergo addition reactions","Completion":"X) UNSATURATED"} {"Type":"TOSS-UP\n\nBIOLOGY Short Answer","Prompt":"Which one of the following 4 animal groups is LEAST closely related to the others: brachiopods; gastropods; cephalopods; bivalves","Completion":"BRACHIOPODS"} {"Type":"TOSS-UP\n\nCHEMISTRY Multiple Choice","Prompt":"Which of the following is NOT true:\n\nW) actinides are characterized by occupying the f-block of the periodic table\n\nX) lanthanides are characterized by occupying the d-block of the periodic table\n\nY) alkali metals are the most reactive of all metallic elements\n\nZ) actinides are metals","Completion":"X) LANTHANIDES ARE CHARACTERIZED BY OCCUPYING THE D-BLOCK OF THE PERIODIC TABLE"} {"Type":"BONUS\n\nCHEMISTRY Short Answer","Prompt":"In standard chemical nomenclature, what is the complete electron configuration for the sodium ion, Na\\({}^{+}\\)?","Completion":"1s\\({}^{2}\\)2s\\({}^{2}\\)2p\\({}^{6}\\) (ACCEPT: 1s\\({}^{2}\\)2s\\({}^{2}\\)2p\\({}^{6}\\)3s\\({}^{0}\\))"} {"Type":"TOSS-UP\n\nENERGY Short Answer","Prompt":"What is the SI [S-I] unit used to measure capacitance, or the ability of a body to store an electrical charge?","Completion":"FARAD"} {"Type":"TOSS-UP\n\nEARTH AND SPACE Short Answer","Prompt":"Over a 720 minute period, through what angle in degrees does the Earth rotate?","Completion":"180"} {"Type":"TOSS-UP\n\nMATH Short Answer","Prompt":"What is the smallest possible value for the sum of a positive number and its own reciprocal?","Completion":"2"} {"Type":"TOSS-UP\n\nBIOLOGY Short Answer","Prompt":"What general class of animal tissue is characterized by the presence of collagen?","Completion":"CONNECTIVE"} {"Type":"TOSS-UP\n\nCHEMISTRY Multiple Choice","Prompt":"You take an IR [I-R] spectrum that has a long, broad peak just below 3000 inverse centimeters. Which of the following is the likely identification of the molecule?\n\nW) Nonane [NO-nayn]\n\nX) Methylcyclopentane [meth-uhl-sy-kluh-PEN-tan]\n\nY) Hexene\n\nZ) Pentanol","Completion":"Z) PENTANOL"} {"Type":"BONUS\n\nCHEMISTRY Multiple Choice","Prompt":"Which of the following compounds is most thermodynamically unstable?\n\nW) Nitrogen trifluoride\n\nX) Nitrogen trichloride\n\nY) Nitrogen tribromide\n\nZ) Nitrogen triiodide","Completion":"Z) NITROGEN TRIIDIDIDE"} {"Type":"TOSS-UP\n\nPHYSICS Short Answer","Prompt":"How much energy, in joules, does a circuit containing a 9 volt battery and a 6 ohm resistor dissipate over 8 seconds?","Completion":"108"} {"Type":"BONUS\n\nPHYSICS Short Answer","Prompt":"If the radius of a body is smaller than its Schwarzschild radius, the body becomes what?","Completion":"BLACK HOLE"} {"Type":"TOSS-UP\n\nEARTH AND SPACE Multiple Choice","Prompt":"Which of the following statements is true about kaolinite [KAY-uh-luh-nyt]?\n\nW) It has an inosilicate structure [in-uh-SIL-i-kayt]\n\nX) It has a high cation exchange capacity [KAT-eye-uhn]\n\nY) It is a magnesium-rich silicate\n\nZ) It is a non-expanding clay","Completion":"Z) IT IS A NON-EXPANDING CLAY"} {"Type":"TOSS-UP\n\nBIOLOGY Multiple Choice","Prompt":"Which of the following animal cell junctions is most similar in function to plant cell plasmodesmata [plaz-muh-des-MUH-tuh]?\n\nW) Gap junction\n\nX) Tight junction\n\nY) Desmosome\n\nZ) Adherens junction","Completion":"W) GAP JUNCTION"} {"Type":"TOSS-UP\n\nMATH Short Answer","Prompt":". Determine \\(\\lim\\limits{x\\to 0}\\frac{x+1}{x}\\). [the limit as x approaches zero of quantity x plus one divided by x]","Completion":"THE LIMIT DOES NOT EXIST (ACCEPT: NON-EXISTANT; DO NOT ACCEPT: INFINITY)"} {"Type":"BONUS\n\nMATH Multiple Choice","Prompt":"Consider a circle in which a square is inscribed. Inscribed within that square is another circle. If the innermost circle has circumference of 12\\(\\pi\\), what is the area of the outermost circle?\n\nW) 64\\(\\pi\\)\n\nX) 72\\(\\pi\\)\n\nY) 88\\(\\pi\\)\n\nZ) 96\\(\\pi\\)","Completion":"X) 72\\(\\pi\\)"} {"Type":"TOSS-UP\n\nENERGY Multiple Choice","Prompt":"What is the power generated by a circuit with a total current of 6 amperes running through two resistors in parallel, of 2 and 6 ohms?\n\nW) 40 Watts\n\nX) 54 Watts\n\nY) 66 Watts\n\nZ) 70 Watts","Completion":"X) 54 WATTS"} {"Type":"BONUS\n\nENERGY Short Answer","Prompt":"Consider a Carnot engine operating between two reservoirs [REZ-er-wawhrs] of different temperatures. If the temperatures in Kelvin of both reservoirs are doubled, by what factor is the efficiency multiplied?","Completion":"ONE"} {"Type":"TOSS-UP\n\nCHEMISTRY Multiple Choice","Prompt":"Which of the following would be the most appropriate S\\({}{\\rm N}\\)2 [S- N- two] leaving group?\n\nW) Hydroxyl\n\nX) O-Trifluoromethanesulfonate [try-FLOOR-oh-METH-ayn-SUHL-fuh-nayt]\n\nY) O-Ethyl\n\nZ) O-Acyl","Completion":"X) O-TRIFLUOROMETHANESULFORNATE"} {"Type":"TOSS-UP\n\nBIOLOGY Multiple Choice","Prompt":"Which of the following animals transfers oxygen to blood via a process called countercurrent exchange?\n\nW) Goldfish\n\nX) Tarantula\n\nY) Hummingbird\n\nZ) Mole","Completion":"W) GOLDFISH"} {"Type":"TOSS-UP\n\nEARTH AND SPACE Multiple Choice","Prompt":"Which of the following is responsible for auroras occurring mainly in the polar regions?\n\nW) The moon\n\nX) Low temperatures\n\nY) The geomagnetic field\n\nZ) The earth's equatorial bulge","Completion":"Y) THE GEOMAGNETIC FIELD"} {"Type":"BONUS\n\nEARTH AND SPACE Short Answer","Prompt":"Below the carbonate compensation depth, what sort of deep ocean ooze can no longer form?","Completion":"CALCAREOUS OOZE (ACCEPT: CALCIUM CARBONATE OOZE)"} {"Type":"TOSS-UP\n\nENERGY Multiple Choice","Prompt":"In an adiabatic [ad-ee-uh-BAT-iK] process, in which direction does heat flow between the system and surroundings?\n\nW) Out of the system\n\nX) Into the system\n\nY) No heat flow into or out of the system\n\nZ) Through the system's boundaries","Completion":"Y) NO HEAT FLOW INTO OR OUT OF THE SYSTEM"} {"Type":"BONUS\n\nENERGY Short Answer","Prompt":"In a solar power tower system, what are the names of the mirrors that focus sunlight onto a pipe located at the focal line, heating up fluid inside and producing steam that drives a turbine?","Completion":"PARABOLIC TROUGH"} {"Type":"TOSS-UP\n\nMATH Short Answer","Prompt":"A car leaves its original location traveling due west at a speed of 60 miles per hour and arrives at its destination in 3 hours. Later that day, the car returns to its original location. At what speed, in miles per hour, did it travel to cover the same distance in 2 hours?","Completion":"90 MILES PER HOUR"} {"Type":"TOSS-UP\n\nPHYSICS Multiple Choice","Prompt":"Two objects collide in a perfectly inelastic collision. Which of the following quantities is not conserved in the collision?\n\nW) Kinetic energy\n\nX) Momentum\n\nY) Potential energy\n\nZ) Total energy","Completion":"W) KINETIC ENERGY"} {"Type":"TOSS-UP\n\nBIOLOGY Multiple Choice","Prompt":"What disadvantage does the presence of introns and exons give eukaryotes [foo-KAR-ee-ohts] when compared to other organisms?\n\nW) The ability to produce more proteins from the same primary transcript\n\nX) Greater stability during reproduction\n\nY) Lower error rate during replication\n\nZ) Mutations are harder to detect","Completion":"Z) Mutations ARE HARDER TO DETECT"} {"Type":"TOSS-UP\n\nCHEMISTRY Multiple Choice","Prompt":"Lithium dialkylcuprates [dy-AL-kuh-KUHP-rayts] can be used in carbon-carbon bond-forming reactions and are usually known as what named reagent?\n\nW) Grignard\n\nX) Gilman\n\nY) Wittig\n\nZ) Birch","Completion":"X) GILMAN"} {"Type":"TOSS-UP\n\nMATH Short Answer","Prompt":"Differentiate the following expression with respect to \\(x\\), provided \\(x\\) is not 0: \\(\\frac{x}{|x|}\\)[x over the quantity the absolute value of x].","Completion":"0"} {"Type":"TOSS-UP\n\nPHYSICS Short Answer","Prompt":"The statement that all points on a wavefront will serve as point sources for secondary wavelets is based on what principle?","Completion":"HUYGENS PRINCIPLE (ACCEPT: HUYGENS-FRESNEL PRINCIPLE)"} {"Type":"TOSS-UP\n\nEARTH AND SPACE Multiple Choice","Prompt":"Which of the following is true about the Labrador Sea Water mass (LSW)?\n\nW) It is formed in the western Labrador Sea in the late spring\n\nX) It travels south along the western edge of the North Atlantic Basin\n\nY) It has decreased in temperature in recent decades\n\nZ) It does not enter the subtropical gyre of the North Atlantic","Completion":"X) IT TRAVELS SOUTH ALONG THE WESTERN EDGE OF THE NORTH"} {"Type":"BONUS\n\nEARTH AND SPACE Multiple Choice","Prompt":"Which of the following volcanic hazards occurred from Mount Nyiragongo [nee-ir-uh-GON-goh] in 1977, killing 70 people in nearby villages?\n\nW) Eruption of a pyroclastic flow [py-ruh-KLAS-tik]\n\nX) Draining of a lava lake\n\nY) Eruption of a noxious gas cloud\n\nZ) Creation of a large lahar","Completion":"X) DRAINING OF A LAVA LAKE"} {"Type":"TOSS-UP\n\nCHEMISTRY Short Answer","Prompt":"Dr. Eeniecs, who loves riddles, gave the following riddle about the periodic table: Element A is left-right adjacent to element B. Element A never shares what it holds. Element B is always taking from other elements. Both elements are a clear gas. Both elements are nonmetals. Combined, they have a mass less than that of gallium. What are the identities of elements A and B?","Completion":"NEON AND FLUORINE"} {"Type":"TOSS-UP\n\nBIOLOGY Multiple Choice","Prompt":"Which of the following is typically derived from ectoderm [EK-tuh-durm]?\n\nW) Areolar connective tissue\n\nX) Nervous tissue\n\nY) Blood\n\nZ) Simple squamous epithelium [SKWAY-muhs ep-uh-THEE-lee-uhm]","Completion":"X) NERVOUS TISSUE"} {"Type":"TOSS-UP\n\nMATH Short Answer","Prompt":"Given that \\(t eq 0\\), find \\(f(\\frac{1}{t})\\) if \\(f(t)=\\frac{1}{t}\\).","Completion":"\\(t\\)"} {"Type":"TOSS-UP\n\nEARTH AND SPACE Short Answer","Prompt":"What is the hottest layer of the Sun's atmosphere?","Completion":"CORONA"} {"Type":"TOSS-UP\n\nPHYSICS Multiple Choice","Prompt":"Two spheres, each with a charge of 3 microcoulombs, are separated by 9 meters. Given that \\(k\\) = 9 x 10\\({}^{9}\\) meters per farad, how much electric potential energy in joules is stored in this combination of charges?\n\nW) 9 x 10\\({}^{-6}\\)\n\nX) 1 x 10\\({}^{-3}\\)\n\nY) 9 x 10\\({}^{-3}\\)\n\nZ) 9 x 10\\({}^{9}\\)","Completion":"Y) 9 x 10\\({}^{3}\\)"} {"Type":"BONUS\n\nPHYSICS Short Answer","Prompt":"In 1955, a soldier's parachute failed to deploy, causing him to fall 1200 feet from the plane to the ground. However, he managed to land in a snow bank, surviving the fall and suffering only minor injuries. Assuming the paratrooper weighed 100 kilograms, that he had reached terminal velocity of approximately 50 meters per second, and that the snow exerts an average force of 120,000 newtons, what minimum depth of snow, in meters to one decimal place, was necessary to stop him?","Completion":"1.0"} {"Type":"TOSS-UP\n\nCHEMISTRY Multiple Choice","Prompt":"Which of the following is NOT true in the titration of a weak monoprotic acid with a strong base?\n\nW) The starting pH will be lower than 7\n\nX) The final pH will be higher than 7\n\nY) Methyl red would be a good indicator to use\n\nZ) Phenolphthalein [free-knol-THAY-teen] would be a good indicator to use","Completion":"Y) METHYL RED WOULD BE A GOOD INDICATOR TO USE"} {"Type":"BONUS\n\nCHEMISTRY Short Answer","Prompt":"Since internal alkynes [AL-kyns] have an additional pi bond, they do not form the same products as alkenes [AL-kyns] upon cleavage by ozone or potassium permanganate [per-MANG-guh-nayt]. What functional group does this cleavage produce for internal alkynes?","Completion":"CARBOXYLIC ACID"} {"Type":"TOSS-UP\n\nBIOLOGY Multiple Choice","Prompt":"After a volcanic eruption wipes out the organisms on an island, which of the following would be the first to recolonize?\n\nW) Large carnivore\n\nX) Wind-dispersed grass\n\nY) Fruqivorous bird [froo-JIV-er-uhs]\n\nZ) Forest tree","Completion":"X) WIND-DISPERSED GRASS"} {"Type":"TOSS-UP\n\nMATH Short Answer","Prompt":"Evaluate: \\(\\int{0}^{4}4\\ dx\\)[the integral from 0 to 4 of 4 dx].","Completion":"16"} {"Type":"TOSS-UP\n\nEARTH AND SPACE Multiple Choice","Prompt":"Which of the following statements is NOT true about shallow living coral reefs?\n\nW) They serve as centers of biodiversity\n\nX) They are generally found in temperate waters\n\nY) They are threatened by climate change\n\nZ) They can be assessed using towed diver surveys","Completion":"X) THEY ARE GENERALLY FOUND IN TEMPERATE WATERS"} {"Type":"TOSS-UP\n\nPHYSICS Multiple Choice","Prompt":"A wire carries a current of 2 amperes vertically upward. What is the magnitude and direction of the magnetic field in teslas produced by the wire 4 millimeters to the North of the wire?\n\nW) 1 x 10\\({}^{-4}\\), West\n\nX) 1 x 10\\({}^{-4}\\), East\n\nY) 2 x 10\\({}^{-4}\\), West\n\nZ) 2 x 10\\({}^{-4}\\), East","Completion":"W) 1 X 10\\({}^{-4}\\), WEST"} {"Type":"TOSS-UP\n\nENERGY Short Answer","Prompt":"What is the name for the practice of reducing or shifting electricity usage during peak periods in response to time-based rates or other forms of financial incentives?","Completion":"DEMAND RESPONSE (ACCEPT: DYNAMIC DEMAND)"} {"Type":"TOSS-UP\n\nCHEMISTRY Multiple Choice","Prompt":"Which of the following is NOT a proposed step in the Suzuki reaction catalytic cycle?\n\nW) Oxidative addition\n\nX) Transmetalation\n\nY) Beta-hydride elimination\n\nZ) Reductive elimination","Completion":"Y) BETA-HYDRIDE ELIMINATION"} {"Type":"TOSS-UP\n\nBIOLOGY Multiple Choice","Prompt":"Which of the following is NOT a tool to develop homeothermry [hoh-mee-uh-THERM-ee] in baby birds?\n\nW) Large brood size\n\nX) Large toes\n\nY) Down\n\nZ) Large leg muscles","Completion":"X) LARGE TOES"} {"Type":"TOSS-UP\n\nMATH Short Answer","Prompt":"A drawer contains 30 socks: 10 brown, 9 black, 6 blue, and 5 red. What is the fewest number of socks I must pick from the drawer without looking to be absolutely certain I have 2 socks of the same color?","Completion":"5"} {"Type":"TOSS-UP\n\nEARTH AND SPACE Short Answer","Prompt":"What discovery in the second half of the 20th century led to the acceptance of the Big Bang theory and the abandonment of the steady state model?","Completion":"COSMIC MICROWAVE BACKGROUND"} {"Type":"TOSS-UP\n\nBIOLOGY Multiple Choice","Prompt":"Which of the following is a major benefit of membrane-bound cells and organelles?\n\nW) Compartmentalization of biochemical reactions\n\nX) Protection from UV radiation\n\nY) Thermal isolation\n\nZ) Membranes conduct electrical signals","Completion":"W) COMPARTMENTALIZATION OF BIOCHEMICAL REACTIONS"} {"Type":"TOSS-UP\n\nPHYSICS Multiple Choice","Prompt":"A 500 kilogram roller coaster car rounds the top of a hill with a radius of 25 meters at a speed of 15 meters per second. Given that the acceleration due to gravity is 10 meters per second squared, what is the normal force in newtons on the car at this point?\n\nW) 500\n\nX) 4500\n\nY) 5000\n\nZ) 5500","Completion":"W) 500"} {"Type":"BONUS\n\nPHYSICS Short Answer","Prompt":"The coefficient of kinetic friction between a car's tires and a flat road is 0.8. The car is travelling at 40 meters per second when the driver slams on the brakes, causing the tires to lock. Assuming the acceleration due to gravity is 10 meters per second squared, how far will the car travel in meters after the driver hits the brakes?","Completion":"100"} {"Type":"TOSS-UP\n\nENERGY Multiple Choice","Prompt":"Which of the following industries has the largest water withdrawal?\n\nW) Agriculture\n\nX) Public drinking water\n\nY) Thermoelectric power\n\nZ) Industrial and mining","Completion":"Y) THERMOELECTRIC POWER"} {"Type":"BONUS\n\nENERGY Short Answer","Prompt":"Your mother has a fluorescent light in her office, and you notice a hum coming from it. Which property, that governs the changing of a material's physical dimensions in response to a change in magnetization, causes this humming sound?","Completion":"MAGNETOSTRICION"} {"Type":"TOSS-UP\n\nCHEMISTRY Multiple Choice","Prompt":"Which of the following pairs would most likely undergo a Heck reaction?\n\nW) Styrene and iodobenzene\n\nX) Ethylbenzene and phenol [FEE-noI]\n\nY) Vinyl bromide and benzene\n\nZ) Cyclohexane [sy-kluh-HEK-sayn] and allyl bromide","Completion":"W) STYRENE AND IODOBENZENE"} {"Type":"BONUS\n\nCHEMISTRY Short Answer","Prompt":"In semicrystalline polymers, the ordered regions will break up and become disordered at the melting temperature. However, amorphous regions soften over a wider range. What is the name of this range?","Completion":"GLASS TRANSITION"} {"Type":"TOSS-UP\n\nBIOLOGY Multiple Choice","Prompt":"Restriction endonucleases [en-duh-noo-klee-AY-sis] are found in bacteria and are thought to provide protection from viruses by cutting viral DNA into small fragments. Molecular biologists have used these enzymes to do which of the following?\n\nW) Fight viral infections in eukaryotes [yoo-KAR-ee-ohts]\n\nX) Clone specific genes into plasmids for expression by bacteria\n\nY) Restrict the production of telomeres in yeast\n\nZ) Amplify desired DNA molecules in a controlled reaction","Completion":"X) CLONE SPECIFIC GENES INTO PLASMIDS FOR EXPRESSION BY BACTERIA"} {"Type":"BONUS\n\nBIOLOGY Multiple Choice","Prompt":"What component of the nephron establishes and maintains a salt gradient used to concentrate urine?\n\nW) Bowman's capsule\n\nX) Proximal convoluted tubule\n\nY) Distal convoluted tubule\n\nZ) Loop of Henle","Completion":"Z) LOOP OF HENLE"} {"Type":"TOSS-UP\n\nMATH Short Answer","Prompt":"Given that \\(f(x)=\\frac{2x}{x-1}\\)If of x equals the fraction with numerator 2x and denominator x minus 1] is a one-to-one function, find \\(x\\) so that \\(f^{\\mbox{-1}}(x)=5\\)[f inverse of x equals 5].","Completion":"5\/2 (ACCEPT: 2.5 OR 2 1\/2)"} {"Type":"TOSS-UP\n\nEARTH AND SPACE Short Answer","Prompt":"What is the region around a spinning black hole in which the frame dragging effect is so strong that all objects are forced to rotate around the black hole?","Completion":"ERGOSPHERE"} {"Type":"TOSS-UP\n\nPHYSICS Multiple Choice","Prompt":"A 3 kilogram mass, a 4 kilogram mass, and a 5 kilogram mass are placed in a line at positions of 3 meters, 4 meters, and 5 meters, respectively, from the origin. Which of the following is nearest the location of the center of mass of this set of masses in meters from the origin?\n\nW) 0.24\n\nX) 1.00\n\nY) 4.17\n\nZ) 12.0","Completion":"Y) 4.17"} {"Type":"BONUS\n\nPHYSICS Short Answer","Prompt":"A 10 ohm resistor, a 20 ohm resistor, and a 30 ohm resistor are connected in series to a 12 volt battery. How much energy in joules is dissipated in heat through the 30 ohm resistor in 60 seconds?","Completion":"72"} {"Type":"TOSS-UP\n\nBIOLOGY Multiple Choice","Prompt":"In Barbara McClintock's famous experiments revealing the existence of transposable genes, she primarily used which of the following as her experimental organism:\n\nW) drosophila\n\nX) E. coli\n\nY) lambda phage\n\nZ) corn plants","Completion":"Z) CONN PLANTS"} {"Type":"TOSS-UP\n\nCHEMISTRY Multiple Choice","Prompt":"Which of the following would be the MOST soluble in water at room temperature:\n\nW) Aluminum sulfate\n\nX) Barium sulfate\n\nY) Lead sulfate\n\nZ) Calcium sulfate","Completion":"W) ALUMINUM SULFATE"} {"Type":"TOSS-UP\n\nPHYSICS Multiple Choice","Prompt":"Astrophysicists, when referring to interstellar gases, typically use the Roman numeral I to indicate that an atom is:\n\nW) singly ionized\n\nX) doubly ionized\n\nY) contains one proton\n\nZ) neutral","Completion":"Z) NEUTRAL"} {"Type":"TOSS-UP\n\nMATH Short Answer","Prompt":"If a 6-foot tall person who is standing next to a vertical pole casts a shadow of 8 feet and the pole casts a shadow of 40 feet, how many feet tall is the pole?","Completion":"30"} {"Type":"TOSS-UP\n\nEARTH SCIENCE Multiple Choice","Prompt":"Which of the following is NOT a type of meteorite:\n\nW) chondrites\n\nX) achondrites\n\nY) stoney-irons\n\nZ) andesites","Completion":"Z) ANDESITES"} {"Type":"TOSS-UP\n\nGENERAL SCIENCE Multiple Choice","Prompt":"As a close relative to the bacteria that cause botulism and gas gangrene, which of the following bacteria are a growing concern in hospitals, killing patients who have been treated with antibiotics:\n\nW) Bacillus anthracis (read as: an-THRAY-sis)\n\nX) Clostridium difficile\n\nY) Treponema pallidium (read as: Trep-oh-KNEE-mah pal-LID-ee-um)\n\nZ) Vibrio cholera","Completion":"X) CLOSTRIDIUM DIFFICILE"} {"Type":"TOSS-UP\n\nASTRONOMY Multiple Choice","Prompt":"Which of the following sets of constellations would be found nearest the ecliptic to an observer in mid-northern latitudes:\n\nW) Leo, Virgo, and Scorpius\n\nX) Pisces, Aries, and Draco\n\nY) Cancer, Gemini, and Cassiopeia\n\nZ) Taurus, Ursa Minor, and Pisces","Completion":"W) LEO, VIRGO, AND SCORPIUS"} {"Type":"BONUS\n\nASTRONOMY Multiple Choice","Prompt":"Which of the following will MOST likely occur if the Earth crosses the orbit of a comet:\n\nW) a meteor shower could be seen from Earth\n\nX) an asteroid will likely strike the Earth\n\nY) red sprites and bolides will be seen both day and night\n\nZ) the Earth would temporarily lose its Van Allen belts","Completion":"W) A METEOR SHOWER COULD BE SEEN FROM EARTH"} {"Type":"TOSS-UP\n\nBIOLOGY Multiple Choice","Prompt":"During photosynthesis, from what substance does the oxygen that is liberated originate:\n\nW) glucose\n\nX) air\n\nY) water\n\nZ) carbon dioxide","Completion":"Y) WATER"} {"Type":"BONUS\n\nBIOLOGY Short Answer","Prompt":"The oxidized form of nicotinamide adenine dinucleotide has what specific charge?","Completion":"+1 (ACCEPT: 1 PLUS or ONE POSITIVE)"} {"Type":"TOSS-UP\n\nCHEMISTRY Multiple Choice","Prompt":"Which of the following is NOT true of halogens:\n\nW) iodine is the least reactive of the stable halogens\n\nX) elemental halogens exist as diatomic molecules containing single covalent bonds\n\nY) their outermost electron configuration is represented by ns\\({}^{2}\\)np\\({}^{6}\\)\n\nZ) halides of typical metals are ionic","Completion":"Y) THEIR OUTERMOST ELECTRON CONFIGATION IS REPRESENTED BY NS\\({}^{2}\\)NP\\({}^{6}\\)"} {"Type":"TOSS-UP\n\nPHYSICS Multiple Choice","Prompt":"In the ray approximation in optics, which of the following is NOT true for a converging lens:\n\nW) a ray parallel to the optical axis passes through the focus\n\nX) a ray passing through the center of the lens is bent parallel to the optical axis\n\nY) an object at infinity is imaged at the focus\n\nZ) a ray passing through the focus is bent parallel to the optical axis","Completion":"X) A RAY PASSING THROUGH THE CENTER OF THE LENS IS BENT PARALLEL TO THE OPTICAL AXIS"} {"Type":"TOSS-UP\n\nMATH Short Answer","Prompt":"Remove all negative exponents and simplify the following expression for all non-zero values of x, y, and z: \\(\\dfrac{(2x^{2}y^{-1}z^{3})^{0}}{(2x^{-3}y^{-4}z^{-5})}\\)","Completion":"\\(\\dfrac{x^{3}y^{4}z^{5}}{2}\\) (ACCEPT: \\(\\dfrac{1}{2}x^{3}y^{4}z^{5}\\))"} {"Type":"TOSS-UP\n\nEARTH SCIENCE Multiple Choice","Prompt":"Which of the following is a brief yet destructive windstorm marked by cool moist air descending from a thunderstorm that is sometimes mistaken for a tornado because of the damage it produces:\n\nW) Chinook\n\nX) zephyr\n\nY) dust devil\n\nZ) microburst","Completion":"Z) MICRBURST"} {"Type":"TOSS-UP\n\nGENERAL SCIENCE Multiple Choice","Prompt":"In a recent breakthrough announcement by the CDC, which of the following diseases has been shown to have both a genetic and physiological component:\n\nW) malaria\n\nX) chronic fatigue syndrome\n\nY) Tay Sach's disease\n\nZ) sleeping sickness","Completion":"X) CHRONIC FATIGUE SYNDROME"} {"Type":"TOSS-UP\n\nASTRONOMY Multiple Choice","Prompt":"Which of the following BEST describes the location of the heliosphere:\n\nW) the region that extends a few thousand kilometers outside of the Sun's corona\n\nX) the region that extends from the Sun's corona to the end of the solar system\n\nY) the region that extends from the Sun's core to the end of the solar system\n\nZ) the region of the expanding solar wind between the sun and the heliopause","Completion":"Z) THE REGION OF THE EXPANDING SOLAR WIND BETWEEN THE SUN AND THE HELIOPAUSE"} {"Type":"TOSS-UP\n\nBIOLOGY Short Answer","Prompt":"Bacterial transduction is primarily caused by the transference of what specific biological substance?","Completion":"DNA"} {"Type":"TOSS-UP\n\nCHEMISTRY Multiple Choice","Prompt":"When iron becomes the ion Fe\\({}^{2+}\\), it\n\nW) loses two, 3\\(d\\) electrons\n\nX) loses two, 4\\(s\\) electrons\n\nY) gains two, 3\\(d\\) electrons\n\nZ) gains two, 4\\(s\\) electrons","Completion":"X) LOSES TWO, 4\\(s\\) ELECTRONS"} {"Type":"TOSS-UP\n\nPHYSICS Multiple Choice","Prompt":"Huygen's principle can be used to most directly derive which of the following:\n\nW) the speed of light\n\nX) the law of diffraction\n\nY) the laws of reflection and refraction\n\nZ) Fermat's Principle","Completion":"Y) THE LAWS OF REFLECTION AND REFRACTION"} {"Type":"TOSS-UP\n\nMATH Short Answer","Prompt":"Remove all negative exponents and simplify the following expression for all non-zero values of \\(x\\): \\(\\left(-32x^{5}\\right)^{\\frac{2}{5}}\\)","Completion":"\\(\\frac{1}{4x^{2}}\\)"} {"Type":"TOSS-UP\n\nEARTH SCIENCE Short Answer","Prompt":"What type of volcano is made mostly of andesite and typically erupts violently and dangerously with pyroclastic flows, an example of which is Mount Fuji in Japan?","Completion":"STRATOVOLCANO (ACCEPT: COMPOSITE VOLCANO)"} {"Type":"TOSS-UP\n\nGENERAL SCIENCE Short Answer","Prompt":"How many carbon and hydrogen atoms are in an octane molecule?","Completion":"CARBON = 8; HYDROGEN = 18"} {"Type":"TOSS-UP\n\nASTRONOMY Short Answer","Prompt":"At how many parsecs is a star's absolute magnitude the same as its apparent magnitude?","Completion":"10"} {"Type":"TOSS-UP\n\nBIOLOGY Short Answer","Prompt":"In plants, water molecules are generally protected from water loss to the outside environment by what lipid-based structure covering the epidermis?","Completion":"CUTICLE"} {"Type":"TOSS-UP\n\nCHEMISTRY Multiple Choice","Prompt":"The principle quantum number '\\(n\\)' of an electron largely determines its:\n\nW) energy\n\nX) spin\n\nY) shape\n\nZ) orientation","Completion":"W) ENERGY"} {"Type":"BONUS\n\nCHEMISTRY Short Answer","Prompt":"Name all of the following compounds that exhibit ONLY London forces: CH\\({}{4}\\); N\\({}{2}\\); BaCl\\({}{2}\\); NH\\({}{3}\\)","Completion":"CH\\({}{4}\\); N\\({}{2}\\)"} {"Type":"TOSS-UP\n\nPHYSICS Short Answer","Prompt":"How many times as intense is 60 decibels than 30 decibels?","Completion":"1,000"} {"Type":"BONUS\n\nPHYSICS Short Answer","Prompt":"What is the minimal force, in newtons to the first decimal place, that a mother must exert to vertically lift her 4 kilogram baby out of her play pen?","Completion":"39.2"} {"Type":"TOSS-UP\n\nBIOLOGY Short Answer","Prompt":"What biological substance is the SECOND MOST abundant organic compound on Earth after cellulose, and is especially abundant in sclereids, xylem fibers and tracheids?","Completion":"LIGNIN"} {"Type":"TOSS-UP\n\nMATH Multiple Choice","Prompt":"Which of the following best describes the system of linear equations: \\(y-2x=\\) - 2, \\(y+x=4\\) If minus two x equals negative two and y plus x equals four!\n\nW) Consistent, Dependent\n\nX) Consistent, Independent\n\nY) Inconsistent, Dependent\n\nZ) Inconsistent, Independent","Completion":"X) CONSISTENT, INDEPENDENT"} {"Type":"BONUS\n\nMATH Short Answer","Prompt":"What is the largest possible sum of the digits on a 12-hour digital clock that displays the hours and minutes?","Completion":"23"} {"Type":"TOSS-UP\n\nBIOLOGY Multiple Choice","Prompt":"Which of the following instruments would be used to sterilize glassware for use in a biology lab?\n\nW) Microtome\n\nX) Autoclave\n\nY) Spectrometer\n\nZ) Thermocycler","Completion":"X) AUTOCLAVE"} {"Type":"BONUS\n\nBIOLOGY Multiple Choice","Prompt":"In dragons, the alleles [uh-LEELS] for yellow flame color and blue flame color are incompletely dominant. If a homozygous [hoh-muh-ZY-guhs] yellow-flame dragon mates with a homozygous blue-flame dragon, all of the brood will have green flame. Which of the following choices represents an expected probability for a brood produced by two green-flame dragons?\n\nW) 75% green flame\n\nX) 75% blue flame\n\nY) 25% yellow flame\n\nZ) 25% green flame","Completion":"Y) 25% YELLOW FLAME"} {"Type":"TOSS-UP\n\nCHEMISTRY Multiple Choice","Prompt":"Which of the following molecules is NOT an exception to the octet rule?\n\nW) BF\\({}{3}\\)[B-F three]\n\nX) NH\\({}{3}\\)[N-H three]\n\nY) SF\\({}{4}\\)[S-F four]\n\nZ) NO [N-O]","Completion":"X) NH\\({}{3}\\)"} {"Type":"TOSS-UP\n\nPHYSICS Short Answer","Prompt":"Millikan's oil-drop experiment was used to calculate what physical value?","Completion":"CHARGE OF THE ELECTRON (ACCEPT: CHARGE-TO-MASS RATIO)"} {"Type":"TOSS-UP\n\nEARTH AND SPACE Short Answer","Prompt":"The Sun's energy is generated from the fusion of what element to form what other element?","Completion":"HYDROGEN TO FORM HELIUM"} {"Type":"TOSS-UP\n\nBIOLOGY Short Answer","Prompt":"The Bowman's capsule and the glomerulus [glob-MER-yuh-luhs]\n\ncompose what part of the mammalian kidney?","Completion":"MALPIGHIAN BODY (ACCEPT: MALPIGHIAN CORPUSCLE, RENAL CORPUSCLE, RENAL CORTEX; DO NOT ACCEPT: MALPIGHIAN)"} {"Type":"TOSS-UP\n\nEnergy Multiple Choice","Prompt":"Which of the following is true regarding capacity factor for an electric power producer?\n\nW) Can be any number\n\nX) Always greater than 1\n\nY) Always between 1 and 0\n\nZ) Always below 0","Completion":"Y) ALWAY's BETWEEN 1 AND 0"} {"Type":"TOSS-UP\n\nCHEMISTRY Multiple Choice","Prompt":"If ( = 3, what are the minimum and maximum allowed values of the magnetic quantum number (m\\({}{i}\\)) [M-L], respectively?\n\nW) 1, infinity\n\nX) 0, 2\n\nY) -3, 3\n\nZ) -%, %","Completion":"Y) -3, 3"} {"Type":"TOSS-UP\n\nPHYSICS Multiple Choice","Prompt":"An object is placed 20 centimeters in front of a thin convex lens of focal length 10 centimeters. What must be the image distance in centimeters?\n\nW) 2 X) 5 Y) 10 Z) 20","Completion":"Z) 20"} {"Type":"BONUS\n\nPHYSICS Short Answer","Prompt":"Two people slide a 22 kilogram box across a frictionless floor a distance of 8 meters. Person 1 applies a force of 10 newtons to the box, while Person 2 applies a force of 12 newtons to the box in the same direction as Person 1. What is the velocity of the box, in meters per second, when it reaches the end of the 8 meters?","Completion":"4"} {"Type":"TOSS-UP\n\nBIOLOGY Multiple Choice","Prompt":"You are surfing at the beach and see a shark nearby, activating your vertebrate stress response. Levels of which of the following would decrease during this response?\n\nW) Cortisol X) Glucagon [GLOO-kuh-gon] Y) Insulin Z) Epinephrine [e-puh-NE-frin]","Completion":"Y) INSULIN"} {"Type":"BONUS\n\nBIOLOGY Multiple Choice","Prompt":"If your parietal [puh-RAY-i-tl] cells were to stop functioning, which of the following would you need to stop consuming?\n\nW) Soda pop X) Steak Y) Bread Z) Salad greens","Completion":"X) STEAK"} {"Type":"TOSS-UP\n\nEARTH AND SPACE Short Answer","Prompt":"Carbon dioxide was released into the atmosphere by volcanic activity and the resulting increase in carbon dioxide levels raised world temperatures by 6 degrees Celsius. The resulting phenomenon, known as The Great Dying, occurred between what two eras?","Completion":"PALEOZOIC AND MESOZOIC"} {"Type":"BONUS\n\nEARTH AND SPACE Multiple Choice","Prompt":"The red shift in the spectral lines reaching us from other galaxies implies that these galaxies are which of the following?\n\nW) In rapid rotation\n\nX) Moving farther apart from one another\n\nY) Predominantly of red giants\n\nZ) Moving closer to one another","Completion":"X) MOVING FARTHER APART FROM ONE ANOTHER"} {"Type":"TOSS-UP\n\nCHEMISTRY Multiple Choice","Prompt":"With respect to magnesium and oxygen, respectively, how do the radii of the constituent ions compare to the corresponding neutral atoms?\n\nW) Larger and larger\n\nX) Larger and smaller\n\nY) Smaller and larger\n\nZ) Smaller and smaller","Completion":"Y) SMALLER AND LARGER"} {"Type":"BONUS\n\nCHEMISTRY Short Answer","Prompt":"Given that the pKa [P-K-A] of acetic acid is 4.8, find the pH to one decimal place of a solution of acetic acid that contains a 10:1 [ten to one] ratio of acetic acid to acetate.","Completion":"3.8"} {"Type":"TOSS-UP\n\nMATH Short Answer","Prompt":"If \\(\\frac{1}{x}-\\frac{1}{y}=\\frac{1}{z}\\), [one over x minus one over y equals one over z] what does z equal in terms of \\(x\\) and \\(y\\)?","Completion":"\\(\\frac{xy}{y-x}\\)"} {"Type":"BONUS\n\nMATH Short Answer","Prompt":"Find the inverse of the functionf(\\(x)=2\/(x-\\)[If of x equals two divided by quantity x minus one].","Completion":"\\(f^{-1}(x)=(2+x)\/x\\) (ACCEPT: (2+x)\/x, 1 + 2\/x)"} {"Type":"TOSS-UP\n\nENERGY Multiple Choice","Prompt":"Carnot efficiency is limited by what physical quantity?\n\nW) Pressure\n\nX) Entropy\n\nY) Flow rate\n\nZ) Temperature","Completion":"Z) TEMPERATURE"} {"Type":"BONUS\n\nENERGY Short Answer","Prompt":"What type of biofuel is produced by the anaerobic digestion or fermentation of biodegradable materials, primarily producing methane and carbon dioxide?","Completion":"BIOGAS"} {"Type":"TOSS-UP\n\nBIOLOGY Multiple Choice","Prompt":"Which of the following fish groups is most likely to utilize tidal ventilation when feeding?\n\nW) Shark\n\nX) Tuna\n\nY) Lamprey\n\nZ) Eel","Completion":"Y) LAMPREY"} {"Type":"BONUS\n\nBIOLOGY Multiple Choice","Prompt":"Duchenne [duh-SHEEN] muscular dystrophy is the result of a recessive disease allele [uh-LEEL] for a gene on the X-chromosome. Which of the following is true regarding inheritance of the disease?\n\nW) Females have twice the chance of developing it, because they have two X chromosomes X) Mothers with the disease allele can pass it with equal probability to sons or daughters Y) Inbreeding has no effect on the likelihood of inheriting this disease Z) An affected son is always produced if his father has the disease allele","Completion":"X) MOTHERS WITH THE DISEASE ALLELE CAN PASS IT WITH EQUAL PROBABILITY TO SONS OR DAughtERS"} {"Type":"TOSS-UP\n\nCHEMISTRY Short Answer","Prompt":"In sulfur dioxide, what is the VSEPR [ves-per] theory prediction for the bond angle between sulfur-oxygen bonds?","Completion":"120"} {"Type":"BONUS\n\nCHEMISTRY Short Answer","Prompt":"A reaction is dependent upon the concentrations of L, M, and N. You find that keeping the other reactants constant and doubling L quadruples the reaction rate; halving M has no effect on the reaction rate; halving N reduces the reaction rate to 1\/8 of its original value. Provide the reaction orders for L, M, and N, as well as the overall order for the reaction.","Completion":"L: 2; M: 0; N: 3 Overall Order: 5"} {"Type":"TOSS-UP\n\nMATH Short Answer","Prompt":"How many points of intersection do the lines \\(2x-3y=5\\) and \\(x=1.5y+2\\) [two x minus three y equals five and x equals one point five y plus two] have?","Completion":"ZERO"} {"Type":"TOSS-UP\n\nPHYSICS Short Answer","Prompt":"A horizontally polarized electromagnetic wave is travelling in the positive \\(x\\) direction. When the electric field is pointing in the negative \\(z\\) direction, what direction is the magnetic field pointing?","Completion":"POSITIVE Y DIRECTION"} {"Type":"TOSS-UP\n\nEARTH ANDE SPACE Multiple Choice","Prompt":"Present evidence suggests that, by mass, most of the matter in the universe is in the form of what material?\n\nW) Cosmic rays\n\nX) Dark matter\n\nY) Luminous matter\n\nZ) Black holes","Completion":"X) DARK MATTER"} {"Type":"BONUS\n\nEARTH AND SPACE Short Answer","Prompt":"Indirect evidence for the existence of gravitational waves has been found by observing the orbital decay of the Hulse-Taylor binary. A pair of what object makes up the binary?","Completion":"NEUTRON STARS (ACCEPT: PULSARS)"} {"Type":"TOSS-UP\n\nCHEMISTRY Multiple Choice","Prompt":"Which of the following statements about Fischer projections is false?\n\nW) The structure may be rotated by 90 degrees or 180 degrees\n\nX) Four bonds to a chiral [KY-ruhl] carbon make a cross with the carbon at the intersection of horizontal and vertical lines\n\nY) Two horizontal bonds are directed toward the viewer\n\nZ) Two vertical bonds are directed behind the stereogenic carbon","Completion":"W) THE STRUCTURE MAY BE ROTATED BY 90 DEGREES OR 180 DEGREES"} {"Type":"BONUS\n\nCHEMISTRY Short Answer","Prompt":"Identify, respectively, the type of strain, if any, for both the antiperiplanar and synclinal [sin-KLYN-I] conformations of butane.","Completion":"ANTIPERIPLANAR = NONE; SYNCLINAL = STERIC"} {"Type":"TOSS-UP\n\nBIOLOGY Multiple Choice","Prompt":"Which of the following trace nutrients is most important for proper function of cytochrome c [SY-tuh-krohm CJ?\n\nW) Magnesium\n\nX) Copper\n\nY) Iron\n\nZ) Chromium","Completion":"Y) IRON"} {"Type":"TOSS-UP\n\nENERGY Multiple Choice","Prompt":"Assuming that one maintained a calorie deficit of 500 calories per day, approximately how long would it take to bum one pound of fat?\n\nW) 1 day\n\nX) 1 week\n\nY) 1 month\n\nZ) 1 year","Completion":"X) 1 WEEK"} {"Type":"TOSS-UP\n\nEARTH AND SPACE Multiple Choice","Prompt":"Which of the following is primarily responsible for the production of spectral absorption lines in the Sun?\n\nW) Cool gas in the photosphere\n\nX) Cool gas in the chromosphere\n\nY) Hot gas in the photosphere\n\nZ) Hot gas in the chromosphere","Completion":"W) COOL GAS IN THE PHOTOSPHERE"} {"Type":"TOSS-UP\n\nPHYSICS Multiple Choice","Prompt":"A 20 kilogram mass is suspended from the end of a spring with a spring constant of 5 newtons per meter. If the mass is displaced 10 centimeters from equilibrium [ee-kwuh-LIB-ree-uhm], what is the maximum speed of the mass in meters per second?\n\nW) 0.05\n\nX) 0.25\n\nY) 0.5\n\nZ) 2","Completion":"W) 0.05"} {"Type":"BONUS\n\nPHYSICS Short Answer","Prompt":"A distant planet has a 3:2 spin-orbit resonance due to tidal influences from its nearby host star. If this planet's sidereal day is 50 Earth days, how long, respectively, in Earth days, are its year and its solar day?","Completion":"YEAR=75, SOLAR DAY=150"} {"Type":"TOSS-UP\n\nEARTH AND SPACE Short Answer","Prompt":"Which two spectral classes of main sequence stars have the smallest vertical velocity dispersion about the plane of the galaxy?","Completion":"O AND B"} {"Type":"BONUS\n\nEARTH AND SPACE Multiple Choice","Prompt":"Which of the following two organisms that still exist today first appeared in the recovery period directly after the Permian-Triassic extinction?\n\nW) Marsupial wombats and wallabies\n\nX) Sea urchins and sea stars\n\nY) Sharks and rays\n\nZ) Magnolias and grasses","Completion":"X) SEA UROHINS AND SEA STARS"} {"Type":"TOSS-UP\n\nBIOLOGY Short Answer","Prompt":"What is the most common autosomal recessive lethal disease in the U.S. among Caucasians?","Completion":"CYSTIC FIBROSIS"} {"Type":"TOSS-UP\n\nENERGY Short Answer","Prompt":"Air source heat pumps rely on an evaporative and condensing intermediate fluid. What is the general term for this fluid?","Completion":"REFRIGERANT"} {"Type":"TOSS-UP\n\nPHYSICS Multiple Choice","Prompt":"Which of the following light movement directions is NOT able to produce total reflection from a surface perpendicular to a light beam?\n\nW) From water to air\n\nX) From air to water\n\nY) From glass to air\n\nZ) From oil to air","Completion":"X) FROM AIR TO WATER"} {"Type":"TOSS-UP\n\nEARTH AND SPACE Multiple Choice","Prompt":"Hurricanes are rated on the Saffir-Simpson Scale.\n\nTornadoes are rated on the Enhanced Fujita Scale. How are these ratings determined?\n\nW) Saffir-Simpson is based on central pressure and Enhanced Fujita is based on damage\n\nX) Saffir-Simpson is based on wind speed and Enhanced Fujita is based on damage\n\nY) Both are based on damage\n\nZ) Both are based on vorticity","Completion":"X) SAFFIR-SIMPSON IS BASED ON WIND SPEED AND ENHANCED FUJITA IS BASED ON DAMAGE"} {"Type":"TOSS-UP\n\nBIOLOGY Short Answer","Prompt":"What is the unit in which the lengths of gene-coding DNA sequences are measured?","Completion":"BASE PAIR (ACCEPT: BP)"} {"Type":"BONUS\n\nBIOLOGY Multiple Choice","Prompt":"While on holiday in Mexico, you note a local vendor selling arrow plant seed pods that jump due the presence of an immature insect that is violently throwing itself from side to side in the seed pod. The insect responsible for this would be a member of what insect order?\n\nW) Lepidoptera [Rep-i-DOP-ter-uh]\n\nX) Diptera\n\nY) Coleoptera [koh-lee-OP-ter-uh]\n\nZ) Hymenoptera [hy-muh-NOP-ter-uh]","Completion":"W) LEPIDOPTERA"} {"Type":"TOSS-UP\n\nENERGY Short Answer","Prompt":"What is the term for a rate structure where the cost of electricity is based on the time of day you use it?","Completion":"TIME OF USE (ACCEPT: REAL-TIME PRICING, PEAK USE)"} {"Type":"BONUS\n\nENERGY Multiple Choice","Prompt":"Enhanced Geothermal Systems, developed by the DOE, increased the availability of geothermal generated electricity in the U.S. by what multiple?\n\nW) 3\n\nX) 5\n\nY) 10\n\nZ) 40","Completion":"Z) 4013) MATH Multiple Choice Given \\(f(x)=\\sqrt{7x^{2}-3}\\) [f of x equals the square root of the quantity 7 x squared minus 3], which of the following are \\(u(x)\\) and \\(g(u)\\) such that \\(f(x)=g(u(x))\\) [f of x equals g of u of x]?"} {"Type":"TOSS-UP\n\nPHYSICS Multiple Choice","Prompt":"Three unequal forces act on an object. If the forces have magnitudes of 30 newtons, 50 newtons, and 70 newtons, what is the minimum net force in newtons that can act on this object?\n\nW) 0\n\nX) 10\n\nY) 50\n\nZ) 90","Completion":"W) 0"} {"Type":"BONUS\n\nPHYSICS Short Answer","Prompt":"If we could capture one second of the total energy output of the Sun, store it, and use it with just 20% efficiency, how long, in years and to two significant digits, would this energy last based upon current global energy consumption? Assume the current global energy consumed per year is about 500,000,000 terajoules and the current luminosity of the Sun is 4 x 10\\({}^{26}\\) watts.","Completion":"160,000"} {"Type":"TOSS-UP\n\nEARTH AND SPACE Multiple Choice","Prompt":"If a climate report shows the difference in surface air pressure between the Azores and Iceland, what phenomenon is it measuring?\n\nW) El Nino Southern Oscillation\n\nX) Pacific Decadal Oscillation\n\nY) North Atlantic Oscillation\n\nZ) Arctic Oscillation","Completion":"Y) NORTH ATLANIC OSCILLATION"} {"Type":"BONUS\n\nEARTH AND SPACE Multiple Choice","Prompt":"Detrital rocks are classified based on which of the following?\n\nW) Mineral composition\n\nX) Grain size only\n\nY) The conditions in which they formed\n\nZ) Grain shape, grain size, and sorting","Completion":"Z) GRAIN SHAPE, GRAIN SIZE, AND SORTING"} {"Type":"TOSS-UP\n\nBIOLOGY Short Answer","Prompt":"Barbara McClintock studied variation in maize kernel coloration. She found that different patterns of coloration were correlated with chromosome structural changes. She was awarded the 1983 Nobel Prize in physiology or medicine for her discovery. What were the genetic elements she discovered?","Completion":"MOBILE GENETIC ELEMENTS (ACCEPT: TRANSPOSONS, JUMPING GENES OR MOBILE CONTROLLING ELEMENTS, TRANSPOSABLE ELEMENTS)"} {"Type":"BONUS\n\nBIOLOGY Multiple Choice","Prompt":"Which of the following processes is NOT correctly matched with the role it plays in the global nitrogen cycle?\n\nW) Nitrogen fixation with the Haber-Bosch process\n\nX) Nitrogen-fixing bacteria with ammonia produced from nitrogen gas\n\nY) Legume root nodules with nitrate produced from ammonia\n\nZ) Denitrification [dee-ny-truth-fi-KAY-shuhn] with nitrogen gas produced from nitrates","Completion":"Y) LEGUME ROOT NODLES WITH NITRATE PRODUCED FROM AMMONIA"} {"Type":"TOSS-UP\n\nPHYSICS Multiple Choice","Prompt":"A 2 kilogram piece of clay is moving with a velocity of 4 meters per second to the north when it collides perfectly inelastically with another 2 kilogram piece of clay that is moving with a speed of 3 meters per second to the east. Which of the following is nearest the angle in degrees that the combined clay's velocity will make with the east after the collision?\n\nW) 30\\({}^{\\circ}\\)\n\nX) 37\\({}^{\\circ}\\)\n\nY) 53\\({}^{\\circ}\\)\n\nZ) 60\\({}^{\\circ}\\)","Completion":"Y) 53\\({}^{\\circ}\\)"} {"Type":"BONUS\n\nPHYSICS Short Answer","Prompt":"Given h = 6.6 x 10\\({}^{\\text{-}}\\)34 joule seconds and providing your answer in scientific notation to one significant figure, what is the energy of a photon in joules with a wavelength of 0.1 micrometers?","Completion":"2 x 10\\({}^{\\text{-}}\\)18"} {"Type":"TOSS-UP\n\nMATH Multiple Choice","Prompt":"If X and Y are bounded sets, then which of the following is true?\n\nW) Both the union and the intersection of sets X and Y are bounded\n\nX) Both the union and the intersection of sets X and Y are unbounded\n\nY) The union of sets X and Y is bounded and the intersection of sets X and Y is unbounded\n\nZ) The union of sets X and Y is unbounded and the intersection of sets X and Y is bounded","Completion":"W) BOTH THE UNION AND THE INTERSECTION OF SETS X AND Y ARE BOUNDED"} {"Type":"TOSS-UP\n\nCHEMISTRY Multiple Choice","Prompt":"The Michaelis-Menten assumptions are integral to\n\nunderstanding the kinetics of enzyme-based reactions. Which of the following is NOT one of\n\nthese assumptions?\n\nW) Substrate is present in slightly greater quantities than the enzyme\n\nX) In a steady state, the concentration of the intermediate is always constant\n\nY) Rate is highest when enzyme catalytic sites are saturated with substrate\n\nZ) The reaction between the substrate and the enzyme is reversible","Completion":"W) SUBSTRATE IS PRESENT IN SLIGHTLY GREATER QUANTITIES THAN THE ENZYME"} {"Type":"TOSS-UP\n\nPHYSICS Multiple Choice","Prompt":"If the proper energy of 15 grams of matter is 1.35 x 10\\({}^{n}\\) joules, what is the value of n?\n\nW) 9\n\nX) 12\n\nY) 15\n\nZ) 18","Completion":"Y) 15"} {"Type":"BONUS\n\nPHYSICS Short Answer","Prompt":"Given that the elementary charge is 1.6 x 10\\({}^{\\text{-}19}\\) coulombs and providing your answer in scientific notation to one significant digit, how many excess electrons would be present in an object with a net negative charge of 2.5 coulombs?","Completion":"2 x 10\\({}^{19}\\)"} {"Type":"TOSS-UP\n\nEARTH AND SPACE Multiple Choice","Prompt":"What is the shape of the orbit of a long period comet that has exactly as much kinetic energy as the absolute value of its gravitational potential energy?\n\nW) Circle\n\nX) Parabola\n\nY) Hyperbola\n\nZ) Ellipse","Completion":"X) PARABOLA"} {"Type":"BONUS\n\nEARTH AND SPACE Short Answer","Prompt":"Order the following four index minerals according to increasing grade of metamorphism of shale: Sillimanite [SIL-uh-muh-nyt], Biotite, Chlorite, Garnet.","Completion":"3, 2, 4, 1 (ACCEPT: CHLORITE, BIOTITE, GARNET, SILLIMANITE)"} {"Type":"TOSS-UP\n\nBIOLOGY Short Answer","Prompt":"What is the name of the phenomenon whereby microorganisms detect and respond to the microbial population density in their immediate surroundings?","Completion":"QUORUM SENSING"} {"Type":"TOSS-UP\n\nENERGY Short Answer","Prompt":"What is the name of the energy storage method where water is pumped from a lower elevation reservoir to a higher elevation during off-peak periods, then released through turbines to produce electric power during peak periods?","Completion":"PUMPED-STORAGE HYDROELECTRICITY (ACCEPT: HYDRO-STORAGE, PUMPED HYDRO)"} {"Type":"TOSS-UP\n\nBIOLOGY Multiple Choice","Prompt":"Which of the following sea turtle species is phylogenetically distinct from the others and can maintain body temperature elevated above ambient levels?\n\nY) Kemp's Ridley\n\nX) Leatherback\n\nY) Green\n\nZ) Loggerhead","Completion":"X) LEATHERBACK"} {"Type":"TOSS-UP\n\nCHEMISTRY Multiple Choice","Prompt":"According to the Bronsted-Lowry system, a base is defined as which of the following:\n\nW) electron pair acceptor\n\nX) proton acceptor\n\nY) proton donor\n\nZ) hydroxide donor","Completion":"X) PROTON ACCEPTOR"} {"Type":"BONUS\n\nCHEMISTRY Short Answer","Prompt":"What is the molecular formula for the conjugate base of the following Bronsted-Lowry acid: H\\({}{2}\\)CO\\({}{3}\\)","Completion":"HCO\\({}{3}\\)-"} {"Type":"TOSS-UP\n\nPHYSICS Multiple Choice","Prompt":"In response to the development of incandescent lights, gas lights were vastly improved by soaking the mantles in what substance to help convert heat and UV energy into visible light:\n\nW) thorium oxide\n\nX) titanium dioxide\n\nY) uranium oxide\n\nZ) tungsten","Completion":"W) THORIUM OXIDE"} {"Type":"BONUS\n\nPHYSICS Short Answer","Prompt":"A hunter uses a 30-gram arrow that is fired by a bowstring at a velocity of 56 meters per second. What is the impulse on the arrow, to the second decimal place in kilogram meters per second?","Completion":"1.68"} {"Type":"TOSS-UP\n\nBIOLOGY Short Answer","Prompt":"What specific area of the human diencephalon is responsible for producing factors that regulate the release of hormones of the pituitary?","Completion":"HYPOTHALAMUS"} {"Type":"TOSS-UP\n\nMATH Short Answer","Prompt":"Given that tangent \\(\\theta=-\\dfrac{3}{4}\\), find the possible values of sine \\(\\theta\\):","Completion":"\\(\\dfrac{3}{5}\\) AND \\(-\\dfrac{3}{5}\\)"} {"Type":"TOSS-UP\n\nEARTH SCIENCE Multiple Choice","Prompt":"Which of the following was the evidence that Alfred Wegener did not have that would have convinced the scientific community that his theory of continental drift was correct:\n\nW) paleomagnetic studies\n\nX) apparent fit of the continents\n\nY) locations of ancient glacial deposits\n\nZ) comparative fossil studies","Completion":"W) PALEOMAGNETIC STUDIES"} {"Type":"TOSS-UP\n\nGENERAL SCIENCE Multiple Choice","Prompt":"Which of the following is the BEST description of an ecological niche:\n\nW) a situation in which a species is well adapted to a specific set of environmental conditions\n\nX) the biomass load of a given environment\n\nY) a pollutant that becomes concentrated in organisms as it moves up through the food chain\n\nZ) all the environmental conditions, plants, and animals of an isolated area","Completion":"W) A SITUATION IN WHICH A SPECIES IS WELL ADAPTED TO A SPECIFIC SET OF ENVIRONMENTAL CONDITIONS"} {"Type":"TOSS-UP\n\nASTRONOMY Multiple Choice","Prompt":"Which of the following is the MAIN advantage of space-based telescopes over typical ground-based telescopes:\n\nW) they are not affected by radiation\n\nX) their mirrors can be any size because of zero gravity\n\nY) they are cheaper to maintain\n\nZ) they experience no atmospheric distortion","Completion":"Z) THEY EXPERIENCE NO ATMOSPHERIC DISTORTION"} {"Type":"TOSS-UP\n\nCHEMISTRY Multiple Choice","Prompt":"The reaction Ca(OH)\\({}{2}+2\\)HCl \\(\\rightarrow\\)\\(2\\)H\\({}{2}\\)O + CaCl\\({}{2}\\), is an example of what class of chemical reaction:\n\nW) synthesis\n\nX) single replacement\n\nY) double replacement\n\nZ) decomposition","Completion":"Y) DOUBLE REPLACEMENT"} {"Type":"TOSS-UP\n\nBIOLOGY Multiple Choice","Prompt":"Which of the following is NOT true:\n\nW): comb jellies or ctenophores (read as: ten-OH-fours) have radial symmetry\n\nX): lobsters have bilateral symmetry\n\nY): protostomes have no true cooloms (read as: SEE-loams)\n\nZ): flatworms are acoelomates (read as: ah-SEA-low-mates)","Completion":"Y) PROTOSTOMES HAVE NO TRUE COELOMS"} {"Type":"BONUS\n\nBIOLOGY Short Answer","Prompt":"Name all of the following 4 choices that are compound fruits:\n\ntomato; raspberry; grape; pineapple","Completion":"RASPBERRY AND PINEAPPLE"} {"Type":"TOSS-UP\n\n11. MATH Multiple Choice","Prompt":"Which of the following is NOT a regular polyhedron:\n\nW) regular tetrahedron\n\nX) cube\n\nY) regular icosahedron\n\nZ) regular hexadecagon","Completion":"Z) REGULAR HEXADECAGON"} {"Type":"TOSS-UP\n\n12. EARTH SCIENCE Multiple Choice","Prompt":"Which of the following is a terminal moraine that resulted at the climax of the Wisconsin ice sheet:\n\nW) Long Island, New York\n\nX) the northern Bahamas Islands\n\nY) Death Valley, California\n\nZ) Hudson Bay, Canada","Completion":"W) LONG ISLAND, NEW YORK"} {"Type":"TOSS-UP\n\nASTRONOMY Multiple Choice","Prompt":"The Schmidt telescope would BEST be considered:\n\nW) an advanced adaptive optics telescope with x-ray detection capabilities\n\nX) a wide field telescope\n\nY) a radio telescope\n\nZ) an orbital x-ray telescope","Completion":"X) A WIDE FIELD TEELSCOPE"} {"Type":"TOSS-UP\n\nCHEMISTRY Multiple Choice","Prompt":"Which of the following is NOT true regarding general trends for the representative elements in the Periodic Table:\n\nW) simple positive ions are always smaller than the neutral atoms from which they are formed\n\nX) elements with low ionization energies lose electrons easily\n\nY) the size of anions increases from left to right\n\nZ) electronegativities increase from left to right","Completion":"Y) THE SIZE OF ANIONS INCREASES FROM LEFT TO RIGHT"} {"Type":"TOSS-UP\n\nPHYSICS Multiple Choice","Prompt":"The index of refraction for a medium is:\n\nW) the ratio of the angles of refraction of red light and blue light in the medium\n\nX) the angle of refraction for monochromatic light in the medium\n\nY) the ratio of the speed of light in the medium to the speed of light in a vacuum\n\nZ) the ratio of the speed of light in a vacuum to the speed of light in the medium","Completion":"Z) THE RATIO OF THE SPEED OF LIGHT IN A VACUUM TO THE SPEED OF LIGHT IN THE MEDIUM"} {"Type":"BONUS\n\nBIOLOGY Short Answer","Prompt":"Name all of the following 3 classes of biological molecules whose immediate breakdown product can generate energy by feeding into metabolism at the level of pyruvate or acetyl Co-A: lipids; proteins; carbohydrates","Completion":"LIPIDS; PROTEINS; CARBOHYDRATES (ACCEPT: ALL)"} {"Type":"TOSS-UP\n\nMATH Short Answer","Prompt":"Solve the following logarithmic equation for \\(x\\): \\(\\log{4}x=4\\) (read as: log base 4 of \\(x\\) equals","Completion":"256"} {"Type":"BONUS\n\nMATH Short Answer","Prompt":"Find the 9th term of the geometric sequence whose first 3 terms are 3, 6, and 12:","Completion":"768"} {"Type":"TOSS-UP\n\nEARTH SCIENCE Multiple Choice","Prompt":"The catastrophic dust bowl of the 1930's was caused by which of the following processes:\n\nW) isostatic adjustment\n\nX) basal slip\n\nY) desertification\n\nZ) mass wasting","Completion":"Y) DESERTIFICATION"} {"Type":"BONUS\n\nEARTH SCIENCE Multiple Choice","Prompt":"Which of the following is an example of a U-shaped valley that is partially submerged:\n\nW) fjord\n\nX) arete\n\nY) paternoster lake\n\nZ) hanging valley","Completion":"W) FJORD"} {"Type":"TOSS-UP\n\nGENERAL SCIENCE Short Answer","Prompt":"What is the medical name for the disease caused by Clostridium bacteria that is prevalent in soils and commonly named lockjaw?","Completion":"TETANUS (ACCEPT: TETANY)"} {"Type":"BONUS\n\nGENERAL SCIENCE Short Answer","Prompt":"What is the common name for vitamin B3 that is a central component of coenzymes NAD and NADP\\({}^{+}\\)?","Completion":"NIACIN (ACCEPT: NICOTINIC ACID)"} {"Type":"TOSS-UP\n\nASTRONOMY Short Answer","Prompt":"Which one of the following 4 planets has the smallest orbital eccentricity: Mercury; Jupiter; Venus; Saturn","Completion":"VENUS"} {"Type":"TOSS-UP\n\nCHEMISTRY Short Answer","Prompt":"What is the pH of a 1 \\(\\times\\) 10\\({}^{-4}\\) molar HCl solution?","Completion":"4"} {"Type":"TOSS-UP\n\nPHYSICS Short Answer","Prompt":"How many 20-ohm resistors connected in parallel will produce 5 ohms of equivalent resistance?","Completion":"4"} {"Type":"BONUS\n\nPHYSICS Short Answer","Prompt":"Assuming g = 10 meters per second squared, how many total newtons of thrust are needed to accelerate a 5,000-kilogram rocket vertically at 6 meters per second squared?","Completion":"80,000"} {"Type":"TOSS-UP\n\nBIOLOGY Short Answer","Prompt":"The pincers of scorptions are a modification of what arachnid appendage?","Completion":"PEDIPALPS"} {"Type":"BONUS\n\nBIOLOGY Short Answer","Prompt":"Name all of the following 4 organisms that are cephalopods (read as: SEFF-al-low-pods): squid; mussel; chambered nautilus; slug","Completion":"SQUID; CHAMBERED NAUTILUS"} {"Type":"TOSS-UP\n\nPHYSICS Short Answer","Prompt":"How many watts are equivalent to 1 joule of energy expended per minute of time?","Completion":"1\/60 (ACCEPT: 0.016 or 0.017 or 0.0166 or 0.0167)"} {"Type":"BONUS\n\nPHYSICS Short Answer","Prompt":"Michael, who has a mass of 100 kilograms, tries to ski jump with a velocity of 15 meters per second. What is Michael's momentum, in proper SI units, as he leaves the ski jump?","Completion":"1500 KILOGRAM METERS PER SECOND"} {"Type":"TOSS-UP\n\nEARTH AND SPACE Multiple Choice","Prompt":"The velocity of particles in the solar wind is closest to which of the following?\n\nW) 300 meters per second\n\nX) 3 kilometers per second\n\nY) 30 kilometers per second\n\nZ) 300 kilometers per second","Completion":"Z) 300 KILOMETERS PER SECOND"} {"Type":"BONUS\n\nEARTH AND SPACE Multiple Choice","Prompt":"Which of the following has a shale protoolith?\n\nW) Quartzite\n\nX) Serpentinite [suhr-pen-TUH-nyd]\n\nY) Gneiss [NYS]\n\nZ) Gabbro","Completion":"Y) GNEISS"} {"Type":"TOSS-UP\n\nBIOLOGY Multiple Choice","Prompt":"In a particular bird population, individuals that lay too many or too few eggs are less likely to survive and reproduce. Which of the following terms best describes this pattern of natural selection?\n\nW) Directional selection\n\nX) Stabilizing selection\n\nY) Disruptive selection\n\nZ) Negative-frequency-dependent selection","Completion":"X) STABILIZING SELECTION"} {"Type":"BONUS\n\nBIOLOGY Short Answer","Prompt":"Spongiform encephalopathies [en-sef-uh-LOP-uh-thees], such as scrapie [SKKAY-pee], made cow disease, and kuru [KOO-roo], are produced after exposure to what general group of infectious agents?","Completion":"PRIONS"} {"Type":"","Prompt":"TOSS-UP","Completion":"3) MATH Short Answer How many 4-digit numbers contain the same digits as 2014? 18"} {"Type":"TOSS-UP\n\nENERGY Multiple Choice","Prompt":"What would be the total kinetic energy of an object rolling down a ramp with no slippage?\n\nW) \\(\\%\\) MV\\({}^{2}\\)[half M-V squared]\n\nX) \\(\\%\\) I omega\\({}^{2}\\)[half I omega squared]\n\nY) \\(\\%\\) MV\\({}^{2}\\) + \\(\\%\\) I omega\\({}^{2}\\)[half M-V- squared plus half I omega squared]\n\nZ) 0","Completion":"Y) \\(\\%\\) MV\\({}^{2}\\) + \\(\\%\\) I OMEGA\\({}^{2}\\)"} {"Type":"TOSS-UP\n\nPHYSICS Short Answer","Prompt":"What values in quantum systems are used to describe conserved quantities that tend to take on integer or half-integer values, examples of which are spin, azimuthal, and parity?","Completion":"QUANTUM NUMBERS"} {"Type":"BONUS\n\nPHYSICS Short Answer","Prompt":"A cannonball is shot vertically upward from ground level with an initial speed of 100 meters per second. Approximating \\(g\\) as 10 meters per second squared, and ignoring air resistance, what is the maximum height, in meters, that the cannonball reaches?","Completion":"500"} {"Type":"TOSS-UP\n\nCHEMISTRY Multiple Choice","Prompt":"Which of the following acts as a Lewis acid in the presence of water to form carbonic acid?\n\nW) Carbon dioxide\n\nX) Methane\n\nY) Methanol\n\nZ) Formic acid","Completion":"W) CARBON DIOXIDE"} {"Type":"BONUS\n\nCHEMISTRY Short Answer","Prompt":"Identify all of the following four compounds that would function as soluble electrolytes when added to water: sodium chloride; decane; acetic acid; barium sulfate.","Completion":"1 AND 3 (ACCEPT: SODIUM CHLORIDE AND ACETIC ACID)"} {"Type":"TOSS-UP\n\nEARTH AND SPACE Multiple Choice","Prompt":"Spectroscopic binaries are easiest to detect when\n\nW) The stars are of the same spectral class\n\nX) The orbit is viewed edge-on\n\nY) The orbit is wide\n\nZ) The orbit is viewed face-on","Completion":"X) THE ORBIT IS VIEWED EDGE-ON"} {"Type":"TOSS-UP\n\nENERGY Multiple Choice","Prompt":"Which of the following is an isotope that has attracted recent interest in terms of developing a new generation of cleaner, proliferation-resistant nuclear power?\n\nW) Uranium-235\n\nX) Thorium-232\n\nY) Plutonium-239\n\nZ) Uranium-238","Completion":"X) THORIUM-232"} {"Type":"TOSS-UP\n\nMATH Short Answer","Prompt":"A basketball player makes 6 baskets during a game, each worth 2 or 3 points. How many different possible values exist for the total points scored by the player?","Completion":"SEVEN"} {"Type":"TOSS-UP\n\nPHYSICS Short Answer","Prompt":"Venturi tubes measure the speed of gas flow via application of what rule or principle of fluid dynamics?","Completion":"BERNOULLI'S PRINCIPLE"} {"Type":"TOSS-UP\n\nBIOLOGY Multiple Choice","Prompt":"Which of the following would be least likely to be a life history trait of an r-selected organism?\n\nW) Strong competitive ability\n\nX) Rapid development\n\nY) Early reproductive age\n\nZ) Found in disturbed habitats","Completion":"W) STRONG COMPETITIVE ABILITY"} {"Type":"TOSS-UP\n\nCHEMISTRY Short Answer","Prompt":"Rank, by name or number, the following three ionic compounds in terms of increasing lattice energy, from the lowest to the highest: Magnesium oxide; Sodium chloride; Sodium fluoride.","Completion":"2, 3, 1 (ACCEPT: SODIUM CHLORIDE, SODIUM FLUORIDE, MAGNESIUM OXIDE)"} {"Type":"TOSS-UP\n\nMATH Short Answer","Prompt":"What is the name of the type of curve that is formed when a piece of rope is hung between two points?","Completion":"CATENARY CURVE (ACCEPT: FUNICULAR)"} {"Type":"TOSS-UP\n\nPHYSICS Short Answer","Prompt":"A truck moving at 50 meters per second begins to brake, which applies a constant force of 1000 newtons to the truck. The truck rolls 20 meters before stopping. In joules, what is the initial kinetic energy of the truck?","Completion":"20,000"} {"Type":"TOSS-UP\n\nEARTH AND SPACE Multiple Choice","Prompt":"Which of the following is not a polymorph of silica?\n\nW) Cristobalite [ki-STOH-buh-lyt]\n\nX) Stishovite [STISH-uh-vyt]\n\nY) Wadsleyite [washd-ZLEE-ahyt]\n\nZ) Quartz","Completion":"Y) WADSLEYITE"} {"Type":"BONUS\n\nEARTH AND SPACE Multiple Choice","Prompt":"Clays such as kaolinite [KAY-uh-luh-nyt] form when H+ [H plus] ions combine with which of the following?\n\nW) Sulfur compounds\n\nX) Pre-existing minerals, such as feldspars\n\nY) Carbon dioxide\n\nZ) Primordial water emitted directly from volcanic eruptions","Completion":"X) PRE-EXISTING MINERALS, SUCH AS FELDSPARS"} {"Type":"TOSS-UP\n\nCHEMISTRY Multiple Choice","Prompt":"The solubility of a gaseous solute in a liquid solvent is typically expressed in units of concentration. Which of the following factors does NOT affect the solubility of a gas in a liquid?\n\nW) Volume of the liquid\n\nX) Temperature of the liquid\n\nY) Pressure of the gas\n\nZ) Polarity of solvent and solute molecules","Completion":"W) VOLUME OF THE LIQUID"} {"Type":"BONUS\n\nCHEMISTRY Short Answer","Prompt":"How many moles of O\\({}{2}\\)[O two] are required to completely\n\ncombust 1 mole of pentane?","Completion":"EIGHT"} {"Type":"TOSS-UP\n\nBIOLOGY Multiple Choice","Prompt":"Which of the following animal respiratory pigments does not contain iron?\n\nW) Hemoglobin [HEE-muh-glob-bin]\n\nX) Hemoerythrin [bee-muh- ih-RITH-rin]\n\nY) Myoglobin [my-uh-GLOH-bin]\n\nZ) Hemocyanin [bee-muh-SAHY-uh-neen]","Completion":"Z) HEMOCYANIN"} {"Type":"BONUS\n\nBIOLOGY Multiple Choice","Prompt":"Which of the following functions is least important in the open circulatory system of insects?\n\nW) Delivery of nutrients\n\nX) Delivery of oxygen\n\nY) Circulation of immune cells\n\nZ) Movement of signaling molecules","Completion":"X) DELIVERY OF OXYGEN"} {"Type":"TOSS-UP\n\nENERGY Multiple Choice","Prompt":"In general, there are three types of compensation agreements for electricity generated using distributed generation, such as solar photovoltaic. All of the following are examples, except:\n\nW) Rate allocation\n\nX) Feed-in Tariff (FIT)\n\nY) Net metering\n\nZ) Power purchase agreement","Completion":"W) RATE ALLOCATION"} {"Type":"BONUS\n\nENERGY Short Answer","Prompt":"Quartz clocks use a crystal oscillator that generates regularly timed electric pulses to keep track of time. What is the name of the effect through which crystals such as quartz can accumulate electric charges?","Completion":"PIEZOELECTRICITY"} {"Type":"TOSS-UP\n\nEARTH AND SPACE Multiple Choice","Prompt":"Which of the following is NOT true about geothermal gradients?\n\nW) The gradient beneath continent crust is less steep than under oceanic crust\n\nX) They vary with tectonic environments\n\nY) They are largely determined by the way that pressure and temperature vary in the Earth\n\nZ) The \"normal\" geothermal gradient in the upper part of Earth is 25'C\/km [w twenty-five degrees Celisus per kilometer]","Completion":"W) THE GRADIENT BENEATH CONTINENT CRUST IS LESS STEEP THAN"} {"Type":"BONUS\n\nEARTH AND SPACE Short Answer","Prompt":"The blue color found in a blue schist [SHIST] rock\n\noccurs due to what mineral?","Completion":"GLAUCOPHANE (ACCEPT: SODIC AMPHIBOLITE OR CROSSITE)"} {"Type":"TOSS-UP\n\nMATH Short Answer","Prompt":"If the equations \\(2x+8=4\\) and \\(ax-30=\\) -22 [two x plus eight equals four and a x minus thirty equals negative twenty-two] have the same solution for \\(x\\), what is the value of \\(a\\)?","Completion":"-4"} {"Type":"BONUS\n\nMATH Short Answer","Prompt":"What value of \\(x\\) satisfies the equation \\(27^{6\\cdot x}=9^{x-1}\\)[27 to the power of quantity six minus x equals nine to the power of quantity x minus one]?","Completion":"4"} {"Type":"TOSS-UP\n\nBIOLOGY Multiple Choice","Prompt":"What is the name of the enzyme that catalyzes the formation of bicarbonate and hydronium ions from CO\\({}{2}\\) and H\\({}{2}\\)O [C-O two and H two O]?\n\nW) Carbonic anhydrase [an-HY-days]\n\nX) Carbonic carboxyylase [Kahr-BOK-suh-layz]\n\nY) Carbonic decarboxylase\n\nZ) Carbonic dehydrogenase [dee-HY-druh-juh-nays]","Completion":"W) CARBONIC ANHYDRASE"} {"Type":"TOSS-UP\n\nCHEMISTRY Multiple Choice","Prompt":"The standard enthalpy and entropy of a hypothetical reaction are both positive. Assuming these values are constant with temperature, which of the following statements describes the temperature dependence of the spontaneity of the reaction?\n\nW) Spontaneous at all temperatures\n\nX) Nonspontaneous at all temperatures\n\nY) Spontaneous at high temperatures but nonspontaneous at low temperatures\n\nZ) Spontaneous at low temperatures but nonspontaneous at high temperatures","Completion":"Y) SPONTANEOUS AT HIGH TEMPERATURES BUT NONSPONTANEOUS AT LOW TEMPERATURES"} {"Type":"TOSS-UP\n\nPHYSICS Multiple Choice","Prompt":"A gold leaf electroscope is negatively charged by touching it with a negatively charged conductor. You then bring a positively charged glass rod close to, but not touching, the top of the electroscope. Which of the following occurs?\n\nW) The leaves do not move\n\nX) The leaves spread further apart\n\nY) The leaves get closer together\n\nZ) The leaves first spread further apart, then move closer together","Completion":"Y) THE LEAVES GET CLOSER TOGETHER"} {"Type":"BONUS\n\nPHYSICS Short Answer","Prompt":"Indicate, by number, all of the following four particles that are acted upon by gravity: electron; photon; neutrino; top quark.","Completion":"1, 2, 3 AND 4 (ACCEPT: ALL OF THEM)"} {"Type":"TOSS-UP\n\nEARTH AND SPACE Short Answer","Prompt":"What is the spectral type of the Sun?","Completion":"G (ACCEPT: G2, G2V)"} {"Type":"BONUS\n\nEARTH AND SPACE Multiple Choice","Prompt":"When a strong south wind occurs along the west coast of North America, Ekman transport causes water to move to which of the following?\n\nW) Onshore\n\nX) Offshore\n\nY) Alongshore to the south\n\nZ) Alongshore to the north","Completion":"W) ONSHORE"} {"Type":"TOSS-UP\n\nBIOLOGY Multiple Choice","Prompt":"Which of the following marine microbe groups has a cell membrane mainly comprised of lipids?\n\nW) Foraminiferans [for-uh-MIN-uh-fer-uhns]\n\nX) Radiolarians [ray-dee-oh-LAIR-ee-uhns]\n\nY) Ciliates [SIL-ee-ayts]\n\nZ) Titaxis","Completion":"Y) CILIATES"} {"Type":"BONUS\n\nBIOLOGY Multiple Choice","Prompt":"Which of the following habitat and life history combinations would select for the production of a large brood size?\n\nW) Arctic, precocial young [pri-KOH-shuh]\n\nX) Arctic, altricial young [al-TRISH-uh]\n\nY) Tropics, precocial young\n\nZ) Tropics, altricial young","Completion":"W) ARCTIC, PRECOCIAL YOUNG"} {"Type":"TOSS-UP\n\nMATH Short Answer","Prompt":"If f(x) = 3'[f of x equals 3 to the power of x], find the value of f(log\\({}{3}\\)[f of log base 3 of 7].","Completion":"7"} {"Type":"TOSS-UP\n\nENERGY Multiple Choice","Prompt":"What percentage of the world's production of rare earth metals is produced by China?\n\nW) 57%\n\nX) 68%\n\nY) 83%\n\nZ) 95%","Completion":"Z) 95%"} {"Type":"TOSS-UP\n\nCHEMISTRY Multiple Choice","Prompt":"In atomic orbital overlap, which of the following is true of destructive interference?\n\nW) It gives a bonding orbital and no nodes\n\nX) It gives an antibonding orbital and a node parallel to the internuclear axis\n\nY) It gives an antibonding orbital and a node perpendicular to the internuclear axis\n\nZ) It gives an antibonding orbital and no nodes","Completion":"Y) IT GIVES AN ANTIBONDING ORBITAL AND A NODE PERPENDICUULAR TO THE INTERNUCLEAR AXIS"} {"Type":"BONUS\n\nCHEMISTRY Multiple Choice","Prompt":"In order to enhance the activity of the catalyst, a textural or chemical promoter may be used in ammonia synthesis. Which of the following is an example of a chemical promoter?\n\nW) Calcium oxide\n\nX) Potassium oxide\n\nY) Aluminum oxide\n\nZ) Silicon dioxide","Completion":"X) POTASSIUM OXIDE"} {"Type":"TOSS-UP\n\nPHYSICS Multiple Choice","Prompt":"How many types of quarks are there in nature?\n\nW) 3\n\nX) 5\n\nY) 6\n\nZ) 10","Completion":"Y) 6"} {"Type":"BONUS\n\nPHYSICS Multiple Choice","Prompt":"Which of the following is the approximate distance in kilometers covered by a photon in one year?\n\nW) 9 thousand\n\nX) 9 million\n\nY) 9 billion\n\nZ) 9 trillion","Completion":"Z) 9 TRILLION"} {"Type":"TOSS-UP\n\nEARTH AND SPACE Multiple Choice","Prompt":"On July 4, under modern conditions, the planet is at which of the following in relation to the Sun?\n\nW) Perihelion [per-uh-HEE-lee-uhn]\n\nX) Aphelion [a-FEE-lee-uhn]\n\nY) Perigee [PER-i-jee]\n\nZ) Apogee [A-puh-jee]","Completion":"X) APPLIION"} {"Type":"BONUS\n\nEARTH AND SPACE Multiple Choice","Prompt":"During La Ni\\(\\tilde{\\rm n}\\)e events, which of the following is most likely?\n\nW) Drought in northeast Brazil\n\nX) Drought in most of Australia\n\nY) Drought in the southwestern U.S.\n\nZ) Wetter than normal in Oregon and Washington","Completion":"Y) DROUGHT IN THE SOUTHWESTERN U.S."} {"Type":"TOSS-UP\n\nBIOLOGY Multiple Choice","Prompt":"During meiosis [my-OH-sis], when a cell has lost both members of a homologous [huh-MOL-uh-guhs] pair of chromosomes, which of the following types of aneuploidy [an-yoo-PLOI-deep] would apply to the cell?\n\nW) Tetrasomy\n\nX) Nullisomy\n\nY) Monosomy\n\nZ) Trisomy","Completion":"X) NULLISOMY"} {"Type":"BONUS\n\nBIOLOGY Short Answer","Prompt":"By number, place the following steps necessary for cloning a gene into bacteria from genomic DNA in order from first to last: Cut the target gene and the cloning vector with restriction enzymes; Transform bacteria with the plasmid; Ligate the target gene and the cloning vector with DNA ligase; Amplify the target gene using PCR.","Completion":"4, 1, 3, 2"} {"Type":"TOSS-UP\n\nMATH Multiple Choice","Prompt":"In how many ways can a committee of 7 be seated along one side of a table if the chair of the committee must sit in the middle?\n\nW) 49\n\nX) 120\n\nY) 720\n\nZ) 5040","Completion":"Y) 720"} {"Type":"BONUS\n\nMATH Short Answer","Prompt":"How many three-digit numbers can be made using digits 0 through 4, provided the first digit is not zero?","Completion":"100"} {"Type":"TOSS-UP\n\nENERGY Multiple Choice","Prompt":"From 2001 to 2011, the installed wind power in the U.S. in megawatts has increased by approximately what factor?\n\nW) 3\n\nX) 6\n\nY) 10\n\nZ) 15","Completion":"Y) 10"} {"Type":"BONUS\n\nENERGY Short Answer","Prompt":"The characteristics of heating oil and diesel fuel are similar, except for which of the following?\n\nW) Evaporation\n\nX) Sulfur composition\n\nY) Volatility\n\nZ) BTU","Completion":"X) SULFUR COMPOSITION"} {"Type":"TOSS-UP\n\nCHEMISTRY Multiple Choice","Prompt":"Which of the following statements is true of the compression of a system?\n\nW) Work is done on the system, and dV and dw are greater than 0\n\nX) Work is done on the system, dV is less than 0, and dw is greater than 0\n\nY) Work is done by the system, dV is greater than 0, and dw is less than 0\n\nZ) Work is done by the system, and dV and dw are less than 0","Completion":"X) WORK IS DONE ON THE SYSTEM, dV IS LESS THAN 0, AND dw IS GREATER THAN 0"} {"Type":"TOSS-UP\n\nPHYSICS Short Answer","Prompt":"You are traveling down a level highway in your car with mass 1000 kilograms at 30 meters per second when you see a deer and hit the brakes. How much time, in seconds, would it take to stop fully if the applied braking force is 5000 newtons?","Completion":"6"} {"Type":"TOSS-UP\n\nEARTH AND SPACE Multiple Choice","Prompt":"Which of the following rocks is most effective at neutralizing acid rain?\n\nW) Limestone\n\nX) Shale\n\nY) Granite\n\nZ) Sandstone","Completion":"W) LIMESTONE"} {"Type":"TOSS-UP\n\nBIOLOGY Multiple Choice","Prompt":"If you have a white horse mate with a white horse that has many spots called leopard spots and half of their offspring have a little bit of leopard spotting, which of the following is your likely conclusion about the leopard-spotting allele?\n\nW) Co-dominant\n\nX) Low penetrance\n\nY) Incompletely dominant\n\nZ) Low expressivity","Completion":"Y) INCOMPLETELY DOMINANT"} {"Type":"TOSS-UP\n\nPHYSICS Multiple Choice","Prompt":"The power input in a perfectly efficient step-up transformer is which of the following?\n\nW) Larger than the power output\n\nX) Equal to the power output\n\nY) Smaller than the power output\n\nZ) Not related to the power output","Completion":"X) EQUAL TO THE POWER OUTPUT"} {"Type":"BONUS\n\nPHYSICS Multiple Choice","Prompt":"A high energy photon near a nucleus can produce an electron-positron pair. What is the minimum energy a photon must have for this to happen?\n\nW) Half of the proper energy of the electron\n\nX) The proper energy of the electron\n\nY) Two times the proper energy of the electron\n\nZ) Ten times the proper energy of the electron","Completion":"Y) TWO TIMES THE PROPER ENERGY OF THE ELECTRON"} {"Type":"TOSS-UP\n\nMATH Multiple Choice","Prompt":"What is the coefficient of the \\(xy^{2}\\) term in the expansion of \\((x+2y)^{3}\\)[open parenthesis x plus 2y close parenthesis to the power of 3]?\n\nW) 3\n\nX) 8\n\nY) 12\n\nZ) 18","Completion":"Y) 12"} {"Type":"BONUS\n\nMATH Short Answer","Prompt":"What is the area of a square inscribed in a circle of radius K?","Completion":"\\(2k^{2}\\)"} {"Type":"TOSS-UP\n\nCHEMISTRY Multiple Choice","Prompt":"Which of the following is an example of an A+1 isotope?\n\nW) Nitrogen\n\nX) Oxygen\n\nY) Sulfur\n\nZ) Fluorine","Completion":"W) NITROGEN"} {"Type":"BONUS\n\nPHYSICS Short Answer","Prompt":"In neutron decay, a neutron decays into a proton, an electron, and what else?","Completion":"AN ELECTRON ANTI-NEUTRINO (DO NOT ACCEPT: NEUTRINO, ELECTRON NEUTRINO, OR ANTI-NEUTRINO)"} {"Type":"TOSS-UP\n\nBIOLOGY Multiple Choice","Prompt":"The absence of which of the following transcript components would decrease eukaryotic [Joo-KAR-ee-oh-tik] translational initiation?\n\nW) Poly(A) tail\n\nX) Intron\n\nY) Stop codon\n\nZ) Internal exon","Completion":"W) POLY(A) TAIL"} {"Type":"BONUS\n\nBIOLOGY Multiple Choice","Prompt":"Leghemoglobin [leg-HEE-muh-gloh-bin] functions as an oxygen binding protein in which of the following?\n\nW) Plant roots\n\nX) Bacteria\n\nY) Human legs\n\nZ) Lactating mothers","Completion":"W) PLANT ROOTS"} {"Type":"TOSS-UP\n\nEARTH AND SPACE Multiple Choice","Prompt":"Which of the following stream types best describes the Columbia River along the Columbia River Gorge?\n\nW) Meandering\n\nX) Braided\n\nY) Ephemeral\n\nZ) Antecedent","Completion":"Z) ANTECEDENT"} {"Type":"BONUS\n\nEARTH AND SPACE Multiple Choice","Prompt":"The Columbia River Gorge primarily cuts through what type of rock?\n\nW) Granite\n\nY) Sandstone\n\nX) Basalt\n\nZ) Schist [SHIFT]","Completion":"X) BASAL"} {"Type":"TOSS-UP\n\nMATH Multiple Choice","Prompt":"Find the radius of a circle in which a \\(30^{\\circ}\\) arc is \\(2\\pi\\) units long.\n\nW) 12\n\nX) 15\n\nY) 15 \\(\\pi\\)\n\nZ) 60 \\(\\pi\\)","Completion":"W) 12"} {"Type":"TOSS-UP\n\nENERGY Multiple Choice","Prompt":"If my current annual heating bill of natural gas is $800 with a furnace that is 70% efficient, approximately what will be my new bill if I install a 95% efficient furnace?\n\nW) $500\n\nX) $600\n\nY) $700\n\nZ) $800","Completion":"X) $600"} {"Type":"TOSS-UP\n\nBIOLOGY Multiple Choice","Prompt":"A child who excretes black urine is born to two normal parents. The child has a recessive homozygous genotype. What is the most likely explanation for the genotype?\n\nW) Spontaneous mutations\n\nX) Consanguinity [kon-sang-GWIN-i-tee]\n\nY) Anticipation\n\nZ) Environmental exposure","Completion":"X) CONSANGUNITY"} {"Type":"BONUS\n\nBIOLOGY Multiple Choice","Prompt":"Which of the following situations might cause disruptive selection to occur in a population of seed-eating finches on an island?\n\nW) A hurricane that wipes out plants that produce medium-sized seeds\n\nX) A hurricane that wipes out plants that produce small and large seeds\n\nY) A hurricane that wipes out plants that produce small seeds\n\nZ) A hurricane that reduces equally plants that produce all sizes of seeds","Completion":"W) A HURRICANE THAT WIPES OUT PLANTS THAT PRODUCE MEDIUM-SIZEED SEEDS"} {"Type":"TOSS-UP\n\nCHEMISTRY Short Answer","Prompt":"According to valence bond theory, we assume that in methane, each pair of electrons is confined to the region between the carbon and hydrogen nuclei, where the bond is strongest. What is the term used to describe this regional confinement?","Completion":"LOCALIZED (ACCEPT: LOCALIZED BOND AND LOCALIZED BONDING)"} {"Type":"BONUS\n\nCHEMISTRY Multiple Choice","Prompt":"Steam cracking is one process used in the manufacture of lighter olefins. Which of the following is NOT true of steam cracking?\n\nW) Increasing temperature shifts cracking to the ends of molecules\n\nX) Short residence times cause more olefin formation\n\nY) The reaction is favored by high pressure\n\nZ) To minimize coke formation, steam may be added","Completion":"Y) THE REACTION IS FAVORED BY HIGH PRESSURE"} {"Type":"TOSS-UP\n\nPHYSICS Multiple Choice","Prompt":"Which of the following is true when light enters a denser medium and is refracted?\n\nW) Wavelength remains unchanged\n\nX) Wave speed increases\n\nY) Wavelength increases\n\nZ) Frequency remains unchanged","Completion":"Z) FREQUENCY REMAINS UNCHANGED"} {"Type":"TOSS-UP\n\nPHYSICS Multiple Choice","Prompt":"Resonance in a musical instrument is MOST directly exemplified by:\n\nW) sympathetic harmonic vibration\n\nX) destructive interference\n\nY) dissonance in the standing wave\n\nZ) diffraction","Completion":"W) SYMPATHETIC HARMONIC VIBRATION"} {"Type":"BONUS\n\nPHYSICS Short Answer","Prompt":"The specific heat of a certain metal is 0.750 joules per gram kelvin. How much heat, in joules to the nearest whole number, will it take to increase the temperature of 0.50 kilograms of the metal from 25\\({}^{\\rm o}\\)C to 50\\({}^{\\rm o}\\)C?","Completion":"9,375"} {"Type":"TOSS-UP\n\nCHEMISTRY Multiple Choice","Prompt":"Solvents, such as water, that can both donate and accept protons are typically described as:\n\nW) resonant\n\nX) metalloid\n\nY) amphoteric\n\nZ) polyprotic","Completion":"Y) AMPHOTERIC"} {"Type":"BONUS\n\nCHEMISTRY Multiple Choice","Prompt":"When going from left to right, elements in the same period of the Periodic Table have progressively smaller atomic radii because of:\n\nW) increased effective nuclear charge\n\nX) decreased effective nuclear charge\n\nY) addition of electrons\n\nZ) addition of neutrons","Completion":"W) INCREASED EFFECTIVE NUCLEAR CHARGE"} {"Type":"TOSS-UP\n\nBIOLOGY Multiple Choice","Prompt":"Which of the following is missing in bryophytes and is an evolutionary advancement of higher plants:\n\nW) vascular tissue\n\nX) epidermal tissue\n\nY) chloroplasts\n\nZ) plasmodesmata (read as: PLAS-mo-dez-MAH-tah)","Completion":"W) VASCULAR TISSUE"} {"Type":"BONUS\n\nBIOLOGY Short Answer","Prompt":"If the flower color red is dominant and white is recessive, what will be the expected phenotypic ratio of red to white flowers in the first generation of a cross between parent plants both heterozygous for flower color?","Completion":"3 TO 1"} {"Type":"TOSS-UP\n\nMATH Short Answer","Prompt":"What is the domain and range for the function defined by the following set of three ordered pairs: {(-2, , (0, , (3, }","Completion":"DOMAIN = -2, 0, 3; RANGE = 6, 9, 7"} {"Type":"BONUS\n\nMATH Short Answer","Prompt":"Giving your answer in centimeters to the nearest whole number, what is the perimeter of a closed quarter-circle whose radius is 12 centimeters?","Completion":"43"} {"Type":"TOSS-UP\n\nEARTH SCIENCE Multiple Choice","Prompt":"What type of stream only bears water during and immediately after a rainstorm:\n\nW) ephemeral\n\nX) intermittent\n\nY) perennial\n\nZ) dendritic","Completion":"W) EPHEMERAL"} {"Type":"BONUS\n\nEARTH SCIENCE Multiple Choice","Prompt":"Which of the following is NOT true about ocean tides:\n\nW) the Moon's influence on Earth's tides is about 2-times greater than the Sun's influence\n\nX) a complete tidal day is about 24 hours 50 minutes long\n\nY) some coastlines have 2 high tides and 2 low tides each lunar day whereas some have only have 1 high tide and 1 low tide each lunar day\n\nZ) spring high tides typically have greater ranges in the Northern Hemisphere during summer than during winter","Completion":"Z) SPRING HIGH TIDES TYPICALLY HAVE GREATER RANGES IN THE NORTHERN HEMISPHERE DURING SUMMER THAN DURING WINTER"} {"Type":"TOSS-UP\n\nGENERAL SCIENCE Short Answer","Prompt":"What neurotransmitter is deficient in Parkinson's disease?","Completion":"DOPAMINE"} {"Type":"BONUS\n\nGENERAL SCIENCE Multiple Choice","Prompt":"Which of the following is the BEST reason why Parkinson's disease cannot be treated by simply administering dopamine into the blood stream:\n\nW) dopamine's blood half-life is too short\n\nX) dopamine will not cross the blood brain barrier\n\nY) dopamine is the precursor to the real effective transmitter in Parkinson's disease\n\nZ) it is not known why dopamine will not work if administered directly into the blood","Completion":"X) DOPAMINE WILL NOT CROSS THE BLOOD BRAIN BARRIER"} {"Type":"TOSS-UP\n\nASTRONOMY Multiple Choice","Prompt":"Which of the following was the primary instrument used to gather 5 to 35 micrometer spectra data from the Deep Impact mission:\n\nW) Chandra X-ray observatory\n\nX) Arecibo radio telescope\n\nY) Hubble gamma-ray telescope\n\nZ) Spitzer infrared spectrograph","Completion":"Z) SPITZER INFRARED SPECTROGRAPH"} {"Type":"","Prompt":"BONUS\n\nASTRONOMY Multiple choice By what multiple does the brightness of two stars differ with apparent magnitudes of \\(-\\)3 and \\(+\\)4:\n\nW) 2.5\n\nX) 25\n\nY) 250\n\nZ) 625","Completion":"Z) 625"} {"Type":"TOSS-UP\n\nPHYSICS Short Answer","Prompt":"What is the acceleration, in meters per second squared, of a 10-kilogram snow sled that is on a frictionless horizontal surface if a horizontal force of 500 newtons is applied to it?","Completion":"50"} {"Type":"BONUS\n\nPHYSICS Multiple Choice","Prompt":"What is the speed of light in meters per second for monochromatic light of 780 nanometers in glass with a refractive index of 1.55:\n\nW) \\(1.5\\times 10^{8}\\)\n\nX) \\(1.9\\times 10^{8}\\)\n\nY) \\(4.65\\times 10^{7}\\)\n\nZ) \\(4.65\\times 10^{8}\\)","Completion":"X) \\(1.9\\times 10^{8}\\)"} {"Type":"TOSS-UP\n\nCHEMISTRY Multiple Choice","Prompt":"Which of the following is a nonpolar molecule:\n\nW) HCN\n\nX) HBr\n\nY) CHCl\\({}{3}\\)\n\nZ) CO\\({}{2}\\)","Completion":"Z) CO\\({}{2}\\)"} {"Type":"TOSS-UP\n\nBIOLOGY Short Answer","Prompt":"In photosystem one, when NADP\\({}^{+}\\) forms NADPH with the catalytic help of NADP\\({}^{+}\\) reductase, from what specific place in the chloroplast are the hydrogen ions derived?","Completion":"STROMA"} {"Type":"TOSS-UP\n\nMATH Short Answer","Prompt":"What is the degree of the following monomial: \\(-6x^{2}y^{4}z^{3}\\)","Completion":"9"} {"Type":"BONUS\n\nMATH Short Answer","Prompt":"Find the number of sides in a regular polygon if each interior angle measures 176 degrees:","Completion":"90"} {"Type":"TOSS-UP\n\nEARTH SCIENCE Multiple Choice","Prompt":"Which of the following is LEAST accurate regarding paleontology:\n\nW) endothermic mammals thrived during the Cenozoic Era\n\nX) the fossil records of birds is poor because of their thin bones and lack of teeth\n\nY) the mastodon was a Proboscidian mammal of the Cenozoic Era\n\nZ) trilobites are an important index fossil of the Mesozoic Era","Completion":"Z) TRIILOBITES ARE AN IMPORTANT INDEX FOSSIL OF THE MESOZOIC ERA"} {"Type":"BONUS\n\nEARTH SCIENCE Multiple Choice","Prompt":"Deposited by subglacial streams, ridges of sand and gravel that form beneath the ablation zone of a glacier are called:\n\nW) ciroques\n\nX) eskers\n\nY) horns\n\nZ) kames","Completion":"X) ESKERS"} {"Type":"TOSS-UP\n\nGENERAL SCIENCE Multiple Choice","Prompt":"Which of the following is MOST commonly used for removing many organic compounds in water in municipal water treatment plants:\n\nW) activated charcoal\n\nX) precipitation reactions\n\nY) reverse osmosis\n\nZ) dredging","Completion":"W) ACTIVATED CHARCOAL"} {"Type":"TOSS-UP\n\nASTRONOMY Multiple Choice","Prompt":"Which of the following is LEAST accurate:\n\nW) recent evidence suggests a subsurface salty ocean is present on the Moon Europa\n\nX) spiral galaxies are actively forming stars\n\nY) Earth's aphelion occurs on the summer solstice\n\nZ) Julian days are often used as the time reference on light curves of stars","Completion":"Y) EARTH'S APHELION OCCURS ON THE SUMMER SOLSTICE"} {"Type":"TOSS-UP\n\nPHYSICS Multiple Choice","Prompt":"Which of the following is generally NOT true regarding diffraction of light:\n\nW) when white light is passed through a diffraction grating, shorter wavelengths are diffracted the most\n\nX) diffraction is wavelength dependent\n\nY) if laser light is passed through a diffraction grating it will produce an interference pattern as a series of dots on a screen\n\nZ) white light passing through a diffraction grating is spread into a rainbow of colors","Completion":"W) WHEN WHITE LIGHT IS PASSED THROUGH A DIFFRACTION GRATING, SHORTER WAVELENGTHS ARE DIFFRACTED THE MOST"} {"Type":"TOSS-UP\n\nCHEMISTRY Multiple Choice","Prompt":"Dalton's gas law applies primarily to:\n\nW) non-ideal gases\n\nX) gas mixtures\n\nY) density of gases\n\nZ) molar mass of gas","Completion":"X) GAS MIXTURES"} {"Type":"TOSS-UP\n\nBIOLOGY Multiple Choice","Prompt":"According to the 3 domain system of classification with the 3 domains being, Archaea, Bacteria and Eukarya, which of the following would be LEAST likely to belong to the domain Bacteria:\n\nW) chlamydia\n\nX) spirochetes (read as: spy-row-keets)\n\nY) gram positive bacteria\n\nZ) non-nucleated thermophiles","Completion":"Z) NON-NUCLEATED THERMOPHILES"} {"Type":"BONUS\n\nBIOLOGY Multiple Choice","Prompt":"In which of the following does the RUBISCO enzyme play a central role:\n\nW) Photosystem One\n\nX) Electron Transport\n\nY) Photosystem Two\n\nZ) Calvin Cycle","Completion":"Z) CALVIN CYCLE"} {"Type":"TOSS-UP\n\nMATH Short Answer","Prompt":"Giving your answer in simplest radical form, rationalize the denominator of the following expression: \\(\\dfrac{1}{(3-\\sqrt{x})}\\)","Completion":"\\(\\dfrac{3+\\sqrt{x}}{9-x}\\)"} {"Type":"BONUS\n\nMATH Short Answer","Prompt":"Solve the following equation for \\(x\\) over the set of complex numbers, giving your answer in simplest form: \\(x^{2}-10x+34=0\\)","Completion":"\\(5\\pm 3i\\) (ACCEPT: \\(5+3i\\) AND \\(5-3i\\))"} {"Type":"TOSS-UP\n\nEARTH SCIENCE Multiple Choice","Prompt":"A hurricane originating in the Eastern tropical Atlantic is driven westward by:\n\nW): the intertropical convergence zone\nX): the prevailing Westerlies\nY): the easterly trade winds\nZ): the doldrums","Completion":"Y) THE EASTERLY TRADE WINDS"} {"Type":"TOSS-UP\n\nGENERAL SCIENCE Short Answer","Prompt":"What technique, used by the U.S. Navy in military exercises and suspected of harming many marine species, is the focus of a recent agreement to not run these exercises within 40 kilometers of the new Hawaiian Islands Marine Reserve?","Completion":"SONAR (ACCEPT: MIN-RANGE SONAR)"} {"Type":"TOSS-UP\n\nCHEMISTRY Short Answer","Prompt":"A molecule that contains 4 carbon-hydrogen single bonds and one carbon-carbon double bond contains how many sigma and pi bonds, respectively?","Completion":"SIGMA = 5; PI = 1"} {"Type":"TOSS-UP\n\nBIOLOGY Short Answer","Prompt":"Animal tissues are generally divided into four main types. If three of them are, connective, nervous, and muscle, what is the fourth tissue?","Completion":"EPITHELIAL (ACCEPT: EPITHELIUM)"} {"Type":"TOSS-UP\n\nPHYSICS Short Answer","Prompt":"If a particle moves from a position of x = 5 meters out to x = 200 meters and back again to x = 5 meters, what is its displacement in meters?","Completion":"0"} {"Type":"BONUS\n\nPHYSICS Short Answer","Prompt":"What are the two features of displacement?","Completion":"MAGNITUDE AND DIRECTION"} {"Type":"TOSS-UP\n\nENERGY Multiple Choice","Prompt":"Which trophic level has the most energy?\n\nW) Consumers\n\nX) Decomposers\n\nY) Producers\n\nZ) Scavengers","Completion":"Y) PRODUCERS"} {"Type":"BONUS\n\nENERGY Multiple Choice","Prompt":"Which is the approximate percentage of energy transfer from one trophic level to the next?\n\nW) 10%\n\nX) 50%\n\nY) 75%\n\nZ) 100%","Completion":"W) 10%"} {"Type":"TOSS-UP\n\nMATH Short Answer","Prompt":"What is the greatest common factor of the terms in the expression \\(3x^{5}y^{2}+6x^{3}y^{5}\\) (read as: 3 \\(x\\) to the fifth \\(y\\) squared plus 6 \\(x\\) cubed \\(y\\) to the fifth)?","Completion":"\\(3x^{3}y^{2}\\)"} {"Type":"TOSS-UP\n\nENERGY Short Answer","Prompt":"If a 250 acre farm were leased for wind development with income of $50 per acre per year, how much income would be produced each year?","Completion":"$12,500"} {"Type":"TOSS-UP\n\nEARTH AND SPACE Multiple Choice","Prompt":"A compressional tectonic environment can result in all of the following, except:\n\nW) Folded mountains\n\nX) Basin and range\n\nY) Volcanoes\n\nZ) Reverse faults","Completion":"X) BASIN AND RANGE"} {"Type":"BONUS\n\nEARTH AND SPACE Multiple Choice","Prompt":"The source of energy that drives a tsunami wave onto a shore originated:\n\nW) From the gravitational attraction between the Earth and the Moon\n\nX) Along the ocean floor\n\nY) From tidal waves\n\nZ) In Earth's interior","Completion":"Z) IN EARTH'S INTERIOR"} {"Type":"TOSS-UP\n\nBIOLOGY Multiple Choice","Prompt":"When sound waves travelling through the air reach the human\n\near, the cochlea (read as: KOE-klee-ah) converts the sound wave into:\n\nW) A chemical response transmitted through the blood stream\n\nX) An electrical impulse sent to the brain\n\nY) A lower frequency wave, which resonates within the ear\n\nZ) Many small wave vectors that are separately communicated to the brain","Completion":"X) AN ELECTRICAL IMPULSE SENT TO THE BRAIN"} {"Type":"BONUS\n\nBIOLOGY Multiple Choice","Prompt":"How does neural transmission across a synaptic gap occur?\n\nW) Neural impulses cause the release of neurotransmitters that diffuse across the gap\n\nX) Neural impulses open potassium-gated channels, allowing sodium to move across the gap\n\nY) Neural impulses travel across the gap as unidirectional electrical currents\n\nZ) Neural impulses travel across the gap as multidirectional electrical currents","Completion":"W) NEURAL IMPULSES CAUSE THE RELEASE OF NEUROTRANSMITTERS"} {"Type":"TOSS-UP\n\nMATH Short Answer","Prompt":"Name a real number that does not have a multiplicative inverse.","Completion":"0"} {"Type":"TOSS-UP\n\nEARTH AND SPACE Multiple Choice","Prompt":"Which of the following statements is true about the Andes mountains?\n\nW) They are folded mountains\n\nX) They are volcanic mountains\n\nY) They are near a spreading center\n\nZ) They are found along a transform boundary","Completion":"X) THEY ARE VOLCANIC MOUNTAINS"} {"Type":"TOSS-UP\n\nMATH Short Answer","Prompt":"As a fraction in lowest terms, what is the probability of rolling 1 or 3 on a standard 6-sided die?","Completion":"1\/3"} {"Type":"TOSS-UP\n\nCHEMISTRY Multiple Choice","Prompt":"Which of the following units can be used to express the volume of a liquid?\n\nW) Micrometers\n\nX) Milligrams\n\nY) Grams per cubic centimeter\n\nZ) Nanoliters","Completion":"Z) NANOLITERS"} {"Type":"TOSS-UP\n\nPHYSICS Multiple Choice","Prompt":"Which of the following choices is NOT an example of a rigid body?\n\nW) Slinky\n\nX) Wheel\n\nY) Pool ball\n\nZ) CD","Completion":"W) SLINKY"} {"Type":"BONUS\n\nPHYSICS Short Answer","Prompt":"Which of the following are transmitted by electromagnetic waves:\n\nradio, television, Rayleigh, or P-waves?","Completion":"RADIO, TELEVISION (ACCEPT: 1, 2)"} {"Type":"TOSS-UP\n\nEARTH AND SPACE Multiple Choice","Prompt":"Which of the following is the best way to get a good view of a meteor shower?\n\nW) Watch it during a full moon\n\nX) Watch it from a city\n\nY) Get as far away from urban light pollution as possible\n\nZ) Wear light clothing","Completion":"Y) GET AS FAR AWAY FROM URBAN LIGHT POLLUUTION AS POSSIBLE"} {"Type":"BONUS\n\nEARTH AND SPACE Short Answer","Prompt":"When a meteoroid enters the earth's atmosphere, it glows. Sometimes, the earth meets a number of meteoroids, creating a meteor shower. Most streams and swarms of meteoroids are believed to be fragments of what celestial body?","Completion":"COMET"} {"Type":"TOSS-UP\n\nBIOLOGY Multiple Choice","Prompt":"The combined genetic information of all the members of a particular population is known as the:\n\nW) Allele (read as: ah-LEEL) frequency\n\nX) Genome\n\nY) Directional selection\n\nZ) Gene pool","Completion":"Z) GENE POOL"} {"Type":"BONUS\n\nBIOLOGY Multiple Choice","Prompt":"The black fur allele B (read as: big b), is dominant over the\n\nwhite fur allele b (read as: little b). Two mice with black fur mate, producing four offspring: one is white and three are black. What are the likely genotypes of the two parents?\n\nW) Bb and Bb\n\nX) BB and Bb\n\nY) BB and BB\n\nZ) BB and bb","Completion":"W) Bb AND Bb"} {"Type":"TOSS-UP\n\nBIOLOGY Short Answer","Prompt":"What is the process by which particles move from an area of\n\nhigher concentration to an area of lower concentration, not requiring cellular energy?","Completion":"DIFFUSION"} {"Type":"BONUS\n\nBIOLOGY Multiple Choice","Prompt":"Which of the following substances is transported into cells via\n\ncarrier-assisted transport?\n\nW) Water\n\nX) Carbon dioxide\n\nY) Oxygen\n\nZ) Sodium ion","Completion":"Z) SODIUM ION"} {"Type":"TOSS-UP\n\nCHEMISTRY Multiple Choice","Prompt":"For a series of chemical reactions, Hess's law provides a method for the summation of which property?\n\nW) Heat of formation\n\nX) Entropy\n\nY) Equilibrium constant\n\nZ) Reaction rate","Completion":"W) HEAT OF FORMATION"} {"Type":"BONUS\n\nCHEMISTRY Multiple Choice","Prompt":"What is the difference between heat and temperature?\n\nW) Temperature is a form of heat, while heat is a measurement\n\nX) Temperature is a measurement of molecular kinetic energy, while heat is a form of energy\n\nY) Heat and temperature have different wavelengths\n\nZ) Heat and temperature have similar wavelengths but different amplitudes","Completion":"X) TEMPERATURE IS A MEASUREM OF MOLECULAR KINETIC ENERGY,"} {"Type":"TOSS-UP\n\nPHYSICS Short Answer","Prompt":"If a car is accelerating along a road, the force acting on the car that causes it to move forward is the reaction force from the car pushing on the ground. What is this reaction force due to?","Completion":"FRICION BETWEEN THE TIRE AND THE GROUND (ACCEPT: FRICION)"} {"Type":"BONUS\n\nPHYSICS Short Answer","Prompt":"If a car is accelerating along a road, what force due to the air does the frictional force have to oppose?","Completion":"DRAG"} {"Type":"TOSS-UP\n\nEARTH AND SPACE Short Answer","Prompt":"What is a Robonaut?","Completion":"ROBOTIC ASTRONAUT"} {"Type":"TOSS-UP\n\nMATH Short Answer","Prompt":"Given the equation \\(2x-y+5=0\\), find the slope and \\(y\\)-intercept of the line it represents.","Completion":"SLOPE = 2; Y-INTERCEPT = 5 OR (0, 5)"} {"Type":"TOSS-UP\n\nBIOLOGY Multiple Choice","Prompt":"Some plants spread their pollen via the wind while others use animals to spread their pollen. When a plant uses an animal to spread its pollen, this type of pollination is called:\n\nW) Vector pollination\n\nX) Seed dispersal\n\nY) Evolutionary adaptation\n\nZ) Reproductive guarantee","Completion":"W) VECTOR POLLINATION"} {"Type":"BONUS\n\nBIOLOGY Short Answer","Prompt":"What is the most common term for the stage of photosynthesis (read as: foe-toe-SIN-teh-sis) during which carbon fixation occurs?","Completion":"CALVIN CYCLE"} {"Type":"TOSS-UP\n\nCHEMISTRY Short Answer","Prompt":"What is the oxidation state of Hydrogen in H\\({}{2}\\)?","Completion":"0"} {"Type":"BONUS\n\nCHEMISTRY Short Answer","Prompt":"What is the oxidation state of Hydrogen in LiAlH\\({}{4}\\)?","Completion":"-1"} {"Type":"TOSS-UP\n\nPHYSICS Short Answer","Prompt":"What are the two forms of electrical current?","Completion":"ALTERNATING AND DIRECT CURRENT (ACCEPT: AC AND DC)"} {"Type":"BONUS\n\nPHYSICS Multiple Choice","Prompt":"Which of the following is the most common method of producing alternating current?\n\nW) Circuits varying from parallel to series\n\nX) Switches automatically turning on and off\n\nY) Turbines rotating backwards and forwards\n\nZ) Magnets spinning in a coil of wire","Completion":"Z) MAGNETS SPINNING IN A COIL OF WIRE"} {"Type":"TOSS-UP\n\nCHEMISTRY Multiple Choice","Prompt":"Which of the following relationships is true?\n\nW) 1 food calorie is equal to 4.18 kiljoules\n\nX) 1 joule is equal to 1 food calorie\n\nY) 1 joule is equal to 4.18 food calories\n\nZ) 1 food calorie is equal to 4.18 joules","Completion":"W) 1 FOOD CALORIE IS EQUAL TO 4.18 KILOJOULES"} {"Type":"BONUS\n\nCHEMISTRY Multiple Choice","Prompt":"In order to calculate the heat of fusion for a piece of ice placed in warm water, you must know:\n\nW) The energy released by the warm water and the mass of the water\n\nX) The energy released by the warm water and the mass of the ice\n\nY) The energy released by the ice and the mass of the warm water\n\nZ) The energy released by the ice and the mass of ice","Completion":"X) THE ENERGY RELEASED BY THE WARM WATER AND THE MASS OF THE ICE"} {"Type":"TOSS-UP\n\nMATH Short Answer","Prompt":"If the distribution graph of a set of data has a positive skew, then which tail of the graph is longer?","Completion":"RIGHT"} {"Type":"TOSS-UP\n\nENERGY Multiple Choice","Prompt":"A kilowatt-hour is a unit of:\n\nW) Charge\n\nX) Power\n\nY) Energy\n\nZ) Electric potential","Completion":"Y) ENERGY"} {"Type":"TOSS-UP\n\nEARTH AND SPACE Multiple Choice","Prompt":"Consider two beaches, both with source material from the same granitic mountains. The beach located further from the mountains would have:\n\nW) Dark, more angular particles\n\nX) Dark, rounder particles\n\nY) Pink, more poorly sorted sediments\n\nZ) Lighter-colored, more well-rounded, finer particles","Completion":"Z) LIGHTER-COLORED, MORE WELL-ROUNDED, FINER PARTICLES"} {"Type":"BONUS\n\nEARTH AND SPACE Multiple Choice","Prompt":"Which of the following is NOT a mechanism of physical and chemical weathering?\n\nW) Mass wasting\n\nX) Glaciation\n\nY) Frost wedging\n\nZ) Groundwater","Completion":"W) MASS WASTING"} {"Type":"TOSS-UP\n\nENERGY Multiple Choice","Prompt":"Biodiesel generally outperforms regular petro diesel in all of the following categories, EXCEPT:\n\nW) Cetane rating\n\nX) Flash point\n\nY) Calorific value\n\nZ) Sulfur content","Completion":"Y) CALORIFIC VALUE"} {"Type":"TOSS-UP\n\nPHYSICS Short Answer","Prompt":"A 4-kilogram ball is tossed at 5 meters per second. How fast, in meters per second, must a 1-kilogram ball be thrown to have the same kinetic energy?","Completion":"10"} {"Type":"TOSS-UP\n\nBIOLOGY Short Answer","Prompt":"What is the term for the pastelike substance, which is also the least soluble nitrogenous waste, excreted by birds, reptiles, and insects?","Completion":"URIC ACID"} {"Type":"TOSS-UP\n\nMATH Short Answer","Prompt":"Consider the polar equation \\(r=\\sin(\\) (read as: \\(r\\) equals the sine of 2 theta). Its graph looks like a rose. How many petals would there be on the \"rose\" in this graph?","Completion":"4"} {"Type":"TOSS-UP\n\nCHEMISTRY Multiple Choice","Prompt":"In which of the following compounds does nitrogen have an unpaired electron?\n\nW) Ammonia\n\nX) Nitrous oxide\n\nY) Sodium nitrate\n\nZ) Nitric oxide","Completion":"Z) NITRIC OXIDE"} {"Type":"BONUS\n\nCHEMISTRY Short Answer","Prompt":"A saturated solution of calcium hydroxide is titrated with aqueous hydrochloric acid. Using proper chemical nomenclature and indicating the state of each substance, such as solid, liquid, aqueous, or gas, what is the balanced molecular equation for this reaction?","Completion":"Ca(OH)\\({}{2(\\mathrm{aq})}\\) + 2HCl\\({}{(\\mathrm{aq})}\\)\\(\\rightarrow\\) CaCl\\({}{2(\\mathrm{aq})}\\) + 2H\\({}{2}\\)O\\({}{(0)}\\)"} {"Type":"TOSS-UP\n\nEARTH AND SPACE Multiple Choice","Prompt":"Which of the following times of day BEST corresponds to a decrease in ionization of particles in the ionosphere?\n\nW) Morning\n\nX) Noon\n\nY) Afternoon\n\nZ) Night","Completion":"Z) NIGHT"} {"Type":"BONUS\n\nEARTH AND SPACE Short Answer","Prompt":"Earth has a prograde rotation. Which two planets in our Solar System exhibit retrograde rotation?","Completion":"VENUS AND URANUS"} {"Type":"TOSS-UP\n\nCHEMISTRY Multiple Choice","Prompt":"Which of the following is NOT a colligative property of a solution?\n\nW) Freezing point depression\n\nX) Osmotic pressure\n\nY) Percent by mass\n\nZ) Boiling point elevation","Completion":"Y) PERCENT BY MASS"} {"Type":"BONUS\n\nCHEMISTRY Short Answer","Prompt":"How many liters of 2 molar sulfuric acid are required to titrate three litters of 0.4 molar nitric acid?","Completion":"0.3"} {"Type":"TOSS-UP\n\nBIOLOGY Multiple Choice","Prompt":"Which of the following is NOT true about snowy plovers?\n\nW) When fighting, they may grab their enemy's tail with their bills\n\nX) They nest on sandy beaches\n\nY) They are endemic to New Zealand\n\nZ) Males nurture their chicks","Completion":"Y) THEY ARE ENDEMIC TO NEW ZEALAND"} {"Type":"BONUS\n\nBIOLOGY Short Answer","Prompt":"What protein, released by cells in the ileum and colon, appears to inhibit appetite in humans?","Completion":"PEPTIDE YY"} {"Type":"TOSS-UP\n\nCHEMISTRY Short Answer","Prompt":"When the equation \\(\\mathrm{O}{2}+\\mathrm{C}{6}\\mathrm{H}{14}\\rightarrow\\) (read as: yields) \\(\\mathrm{CO}{2}+\\mathrm{H}{2}\\mathrm{O}\\) is balanced with the lowest whole-number coefficients, what is the coefficient of \\(\\mathrm{O}{2}\\)?","Completion":"19"} {"Type":"TOSS-UP\n\nMATH Short Answer","Prompt":"Evaluate \\(\\lim{\\chi\\rightarrow\\mathrm{m}3}\\frac{\\sin\\chi}{x}\\frac{\\frac{\\sqrt{3}}{2}}{ \\frac{n}{3}}\\) (read as: the limit as \\(x\\) approaches pi over three of open parenthesis sine of \\(x\\) [pause] minus the square root of 3 over 2 close parenthesis over open parenthesis \\(x\\) minus pi over 3 close parenthesis).","Completion":"1\/2"} {"Type":"TOSS-UP\n\nPHYSICS Short Answer","Prompt":"If in a 2-piston hydraulic device, a 100-newton force is applied to a piston with an area of 0.01 square meters, what will be the resulting force, in newtons, on a piston with an area of 0.5 square meters?","Completion":"5000"} {"Type":"BONUS\n\nPHYSICS Short Answer","Prompt":"What is the centripetal acceleration in meters per second squared of a point on a bicycle tire of radius 0.81 meters when the bike is moving at 9 meters per second?","Completion":"100"} {"Type":"TOSS-UP\n\nBIOLOGY Short Answer","Prompt":"What is the general term for scientific research that looks for a useful application, process, or product in nature?","Completion":"BIOPROSPECTING"} {"Type":"BONUS\n\nBIOLOGY Short Answer","Prompt":"What is the term in immunobiology that describes a lack of reaction by the body's defense mechanisms for foreign substances, and consists of a direct induction of peripheral lymphocyte tolerance?","Completion":"ANERGY"} {"Type":"TOSS-UP\n\nCHEMISTRY Multiple Choice","Prompt":"Which of the following species is NOT considered an electrolyte?\n\nW) Sodium hydroxide\n\nX) Hydrogen chloride\n\nY) Aluminum nitrate\n\nZ) Diethyl ether","Completion":"Z) DIETHYL ETHER"} {"Type":"BONUS\n\nCHEMISTRY Short Answer","Prompt":"By name or number, identify all of the following four substances that would show a positive result in a Benedict's test: simple sugars, starches, proteins, lipids.","Completion":"SIMPLE SUGARS (ACCEPT: 1)"} {"Type":"TOSS-UP\n\nPHYSICS Short Answer","Prompt":"How many layers of doped silicon are needed to make a transistor?","Completion":"3"} {"Type":"BONUS\n\nPHYSICS Short Answer","Prompt":"A thin copper rod 1 meter long has a mass of 0.05 kilograms and is in a magnetic field of 0.10 teslas. Given 10 meters per second squared for g, what is the minimum current in amperes needed in the rod for the magnetic force to cancel the weight of the rod?","Completion":"5"} {"Type":"TOSS-UP\n\nEARTH AND SPACE Short Answer","Prompt":"About 3.9 billion years ago, evidence suggests that the Earth was undergoing a cataclysmic convulsion in which the surfaces of the Moon, the Earth, Mercury, Venus, and Mars were heavily impacted by fragments left over from formation of the planets. What was that period called?","Completion":"LATE HEAVY BOMBARDMENT (ACCEPT: ERA OF HEAVY BOMBARDMENT, LHB, LUNAR CATACLYSM)"} {"Type":"BONUS\n\nEARTH AND SPACE Short Answer","Prompt":"Conceptually, solar sail propulsion is based upon the transmission of the momentum of photons as they bounce off mirror-like aluminized Mylar sails. By number, identify all of the following four options that are advantages of solar sailing as compared to a conventional rocket: no need to carry fuel, quicker acceleration, constant acceleration, constant velocity.","Completion":"1 AND 3"} {"Type":"TOSS-UP\n\nPHYSICS Multiple Choice","Prompt":"On a game show, contestants spin a large wheel to determine the prize associated with correct answers. How should a contestant apply force to spin the wheel as rapidly as possible?\n\nW) Tangential to the rim of the wheel\n\nX) At a 45 degree angle to the rim of the wheel\n\nY) Away from the center of the wheel\n\nZ) Towards the center of the wheel","Completion":"W) TANGENTIAL TO THE RIM OF THE WHEEL"} {"Type":"BONUS\n\nPHYSICS Short Answer","Prompt":"Nitrogen molecules of mass 4.65 x 10\\({}^{.26}\\) kilograms are contained in a cube with sides that have an area of 6 x 10\\({}^{.4}\\) square meters. If 5.0 x 10\\({}^{.23}\\) molecules collide head on elastically each second with one wall at a speed of 300 meters per second, what is the pressure exerted on the wall to the nearest kilopascal?","Completion":"23"} {"Type":"TOSS-UP\n\nENERGY Short Answer","Prompt":"A 5-kilogram mass is moving along a frictionless surface with a velocity of 10 meters per second toward a horizontal spring with a spring constant of 500 newtons per meter. What is the maximum compression of the spring in meters?","Completion":"1"} {"Type":"TOSS-UP\n\nMATH Multiple Choice","Prompt":"The last two digits, in order, of a positive integer \\(n\\) are 1 and 8. Which of the following statements about \\(n\\) can be TRUE?\n\nW) \\(n\\) is divisible by 7\n\nX) \\(n\\) is divisible by 5\n\nY) \\(n\\) is divisible by 4\n\nZ) \\(n\\) is a perfect square","Completion":"W) \\(n\\) IS DIVISIBLE BY 7"} {"Type":"TOSS-UP\n\nEARTH AND SPACE Multiple Choice","Prompt":"Which of the following happens to liquid water suspended in an air mass descending on the leeward side of a mountain range?\n\nW) It condenses\n\nX) It precipitates\n\nY) It freezes\n\nZ) It evaporates","Completion":"Z) IT EVAPORATES"} {"Type":"BONUS\n\nEARTH AND SPACE Short Answer","Prompt":"By name or number, identify all of the following four factors that contribute to global wind patterns: solar radiation, friction, pressure gradient, Coriolis force.","Completion":"SOLAR RADIATION, FRICTION, PRESSURE GRADIENT, AND CORIOLIS FORCE (ACCEPT: 1, 2, 3, 4, OR ALL)"} {"Type":"TOSS-UP\n\nPHYSICS Multiple Choice","Prompt":"If \\(g{E}\\) and \\(g{M}\\) are the accelerations due to gravity on the surfaces of Earth and the Moon respectively, and if Millikan's oil drop experiment could be performed on the two surfaces, the ratio of the electronic charge on the Moon to the electronic charge on the Earth would be which of the following?\n\nW) \\(g{E}\\)\/\\(g{M}\\)\n\nX) 1\n\nY) \\(g{M}\\)\/\\(g{E}\\)\n\nZ) (\\(g{M}\\)\/\\(g{E}\\))\\({}^{2}\\)","Completion":"X) 1"} {"Type":"BONUS\n\nPHYSICS Short Answer","Prompt":"Providing your answer in scientific notation with two significant digits, how much energy in joules is required to heat a clay pizza baking stone with mass of 4.8 kilograms and a specific heat of 860 joules per kilogram Kelvin from 25\\({}^{\\circ}\\) Celsius to 235\\({}^{\\circ}\\) Celsius?","Completion":"8.7 \\(\\times\\) 10\\({}^{5}\\)"} {"Type":"TOSS-UP\n\nEARTH AND SPACE Multiple Choice","Prompt":"Which of the following U.S. coastlines would change the LEAST as sea level rises?\n\nW) West coast\n\nX) Gulf coast\n\nY) Northeast coast\n\nZ) Florida coast","Completion":"W) WEST COAST"} {"Type":"TOSS-UP\n\nENERGY Short Answer","Prompt":"Which hydroelectric dam has the greatest power capacity in the world?","Completion":"THREE GORGES DAM"} {"Type":"TOSS-UP\n\nBIOLOGY Multiple Choice","Prompt":"Of the seven species of sea turtles, six are found in U.S.\n\nwaters. Which of the following species of sea turtles is NOT found in U.S. waters?\n\nW) Flatback\n\nX) Green\n\nY) Kemp's ridley\n\nZ) Leatherback","Completion":"W) FLATBACK"} {"Type":"BONUS\n\n1. CHEMISTRY Multiple Choice","Prompt":"Which of the following substances is soluble in carbon tetrachloride?\n\nW) Formaldehyde [for-MAL-duh-hyd]\n\nX) Bromine\n\nY) Ammonia\n\nZ) Sodium chloride","Completion":"X) BROMINE"} {"Type":"BONUS\n\n2. BIOLOGY Multiple Choice","Prompt":"Which of the following human vitamin or mineral requirement is NOT matched correctly with its symptoms of deficiency?\n\nW) Vitamin B12 and beriberi [BER-ee-BER-ee]\n\nX) Vitamin C and scurvy\n\nY) Vitamin A and blindness, impaired immunity, and skin disorders\n\nZ) Vitamin K and defective blood clotting","Completion":"W) VITAMIN B12 AND BERIBERI"} {"Type":"BONUS\n\n3. ENERGY Multiple Choice","Prompt":"If my current annual heating bill is $900 and my old furnace has an efficiency rating of 75%, which of the following best approximates my new bill if I install a 95% efficient furnace?","Completion":"W) $675 X) $690 Y) $710 Z) $840 Y) $710"} {"Type":"BONUS\n\n4. EARTH AND SPACE Multiple Choice","Prompt":"The metallicity of globular clusters is generally which of the following?\n\nW) Less than that of the Sun X) Comparable to that of the Sun Y) Greater than that of the Sun Z) Too variable between globular clusters to make any definitive conclusions","Completion":"W) LESS THAN THAT OF THE SUN"} {"Type":"BONUS\n\n5. PHYSICS Short Answer","Prompt":"A force displaces an object. The force vector is 3i plus 2j plus k newtons and the displacement vector is 2i plus 6j minus 5k meters. Find the work done in joules.","Completion":"13"} {"Type":"BONUS\n\n6. MATH Short Answer","Prompt":"Giving your answer with \\(r\\) positive and theta in radians, what are the equivalent polar coordinates to the rectangular coordinates (-3\\(\\sqrt{3}\\), -?","Completion":"(6,7\\(\\pi\\)\/6)"} {"Type":"BONUS\n\n9. ENERGY Multiple Choice","Prompt":"At 120 volts, which of the following is the approximate emergence through a circuit that consists of resistors of 5 ohms, 10 ohms, and 20 ohms in series, followed by the same resistors in parallel?\n\nW) 3.16\n\nX) 3.42\n\nY) 4.66\n\nZ) 35","Completion":"W) 3.16"} {"Type":"BONUS\n\n10. EARTH AND SPACE Short Answer","Prompt":"What is the coolest spectral class of stars around which H II regions are generally seen?","Completion":"B"} {"Type":"BONUS\n\n11. PHYSICS Short Answer","Prompt":"An ambulance is approaching the scene of an accident with a speed of 30 meters per second. Its siren has a frequency of 1500 hertz. Providing your answer with two significant figures, find the frequency in hertz of the siren heard by a stationary person waiting for the ambulance, given that the speed of sound is 340 meters per second.","Completion":"1600"} {"Type":"BONUS\n\n12. MATH Short Answer","Prompt":"Solve the system of equations: \\(x+y=1\\), and \\(x^{2}+3y^{2}=21\\).","Completion":"(-3\/2, 5\/2), (3, -2)"} {"Type":"BONUS\n\n13. CHEMISTRY Short Answer","Prompt":"What is the electron configuration of an atom of gallium in the ground state?","Completion":"1s\\({}^{2}\\)2s\\({}^{2}\\)2p\\({}^{6}\\)3s\\({}^{2}\\)3p\\({}^{6}\\)4s\\({}^{2}\\)3d\\({}^{10}\\)4p\\({}^{1}\\)(ACCEPT: THE LAST THREE IN ANY ORDER; [Ar] 4s\\({}^{2}\\)3d\\({}^{10}\\)4p\\({}^{1}\\))"} {"Type":"BONUS\n\n14. BIOLOGY Multiple Choice","Prompt":"Which of the following statements is NOT true regarding ticks and transmitted diseases?\n\nW) Female black legged ticks are only slightly better at transmitting Lyme disease than their male counterparts\n\nX) The disease-causing agent in Lyme disease is the bacterium Borrelia burgdorferi [buh-REL-ee-uh boorg-dor-FE-ree]\n\nY) The dog tick serves as a vector for Ricketssia ricketssia, the causative agent of Rocky Mountain Spotted Fever\n\nZ) The Lone Star Tick serves as a vector for disease-causing Rickettsia","Completion":"W) FEMALE BLACK LEGGED TICKS ARE ONLY SLIGHTLY BETTER AT"} {"Type":"## 17 BONUS\n\n### 17. MATH Short Answer","Prompt":"In square centimeters and in terms of pi, find the area of a circle with an inscribed triangle with side lengths of 10 centimeters, 24 centimeters, and 26 centimeters.","Completion":"169\\(\\pi\\)"} {"Type":"BONUS\n\n19. BIOLOGY Multiple Choice","Prompt":"While looking through a microscope at a few drops of water squeezed out of a fist full of moss, you notice an eight-legged animal that has claws. Which of the following is the most likely identity of this organism?\n\nW) Daphnia\n\nX) Rotifer\n\nY) Tardigrade\n\nZ) Collembola","Completion":"Y) TARDIGRADE"} {"Type":"BONUS\n\n20. ENERGY Short Answer","Prompt":"By number, identify all of the following four statements that accurately describe the processes involved in aerobically producing ATP: Although prokaryotic [proh-KAR-ee-oh-tik] cells lack mitochondria [my-tuh-KON-dree-uh] and cristae [KRIS-tuh], they use a proton pump to produce ATP; Energy is liberated when the chemical bond between ATP and its phosphate subgroup is formed; Oxygen is used in the last step of aerobic respiration to produce water; Chemiosmosis produces more ATP molecules than oxidative phosphorylation [FOS-fer-uh-lay-shuhn].","Completion":"1 AND 3"} {"Type":"BONUS\n\n25. BIOLOGY Multiple Choice","Prompt":"If you have an alien population and the green skin phenotype is always inherited with the menacing horns phenotype, the genes that code for these phenotypes would be classified as which of the following?\n\nW) Recombinant\n\nX) Hardy-Weinberg\n\nY) Mendelian [men-DEE-lee-uhn]\n\nZ) Linked","Completion":"Z) LINKED"} {"Type":"TOSS-UP\n\nPHYSICS Multiple Choice","Prompt":"The Meissner effect is most directly associated with which of the following:\n\nW) superconductors\n\nX) rare earth magnets\n\nY) very strong electromagnets\n\nZ) Tesla coils","Completion":"W) SUPERCONDUCTORS"} {"Type":"TOSS-UP\n\nCHEMISTRY Multiple Choice","Prompt":"Which of the following is NOT generally true of activation energies in chemical reactions:\n\nW) activation energies vary from reaction to reaction\n\nX) the Arrhenius equation can be used to determine the activation energy of a reaction\n\nY) the lower the activation energy is, the faster the reaction\n\nZ) increasing the temperature of a reaction decreases the activation energy","Completion":"Z) INCREASING THE TEMPERATURE OF A REACTION DECREASES THE ACTIVATION ENERGY"} {"Type":"TOSS-UP\n\nBIOLOGY Multiple Choice","Prompt":"Which of the following was first proposed by Peter Mitchell as a potential energy ion gradient across a selectively permeable membrane that is responsible for the production of ATP:\n\nW) osmosis\n\nX) chemiosmosis (read as: chem-ee-oz-MOE-sis):\n\nY) electro-phosphorylation\n\nZ) photosynthesis","Completion":"X) CHEMIOSMOSIS"} {"Type":"TOSS-UP\n\nMATH Multiple Choice","Prompt":"What is the product of the following 2 values: (the greatest common divisor of 7 and 14; and (the least common multiple of 7 and 14:\n\nW) 21\n\nX) 49\n\nY) 98\n\nZ) 196","Completion":"Y) 98"} {"Type":"TOSS-UP\n\nEARTH SCIENCE Multiple Choice","Prompt":"Which of the following BEST determines the maximum height of open ocean waves and is the point where breaking white caps form in a fully developed sea:\n\nW) the cohesive forces in water molecules are overcome and the waves break apart\n\nX) the waves are losing as much energy as breaking white caps as they are receiving from the wind\n\nY) the trough moves a little bit faster than the crest and the wave becomes unstable\n\nZ) the waves have reached a maximum period","Completion":"X) THE WAVES ARE LOSING AS MUCH ENERGY AS BREAKING WHITE CAPS AS THEY ARE RECEIVING FROM THE WIND"} {"Type":"TOSS-UP\n\nGENERAL SCIENCE Multiple Choice","Prompt":"Which of the following is NOT true of coelacanths (read as: SEE-laa-kanths):\n\nW) the coelacanth is considered a living fossil\n\nX) the coelacanth is a bottom dweller fish and uses its 4 limbs for walking\n\nY) the coelacanth is essentially inedible to humans\n\nZ) the coelacanth gives birth to live young","Completion":"X) THE COELACANTH IS A BOTTOM DWELLER FISH AND USES ITS 4 LIMBS FOR WALKING"} {"Type":"TOSS-UP\n\nASTRONOMY Multiple Choice","Prompt":"Which of the following is a space observatory, launched in 2004 through the combined effort of Italy, the United Kingdom, and the United States, for the detection and study of gamma-ray bursts:\n\nW) SOHO\n\nX) SWIFT\n\nY) MOC\n\nZ) international GRB","Completion":"X) SWIFT"} {"Type":"BONUS\n\nASTRONOMY Multiple Choice","Prompt":"How were the rings of the planet Uranus discovered:\n\nW) astronomers in 1977 noticed stars were occulted by Uranus before the planet passed in front of them and reappeared afterwards\n\nX) the Magellan 1 space probe took pictures of them\n\nY) the Hubble space telescope detected them with its infrared detector\n\nZ) the rings are made of gases with certain signatures revealed in absorption spectra","Completion":"W) ASTRONOMERS IN 1977 NOTICED STARS WERE OCCULTED BY URANUS BEFORE THE PLANET PASSED IN FRONT OF THEM AND REAPPEARED AFTERWARDS"} {"Type":"TOSS-UP\n\nPHYSICS Multiple Choice","Prompt":"Which of the following BEST describes Fermat's principle:\n\nW) light disperses in a specific medium depending on the medium's refractive index\n\nX) light follows the path of least time between two points\n\nY) the angle of incidence can be greater than the angle of reflection\n\nZ) light always travels in the same medium at the same speed","Completion":"X) LIGHT FOLLOWS THE PATH OF LEAST TIME BETWEEN TWO POINTS"} {"Type":"BONUS\n\nPHYSICS Short Answer","Prompt":"What is the ideal mechanical advantage of a wheel and axle, if the diameter of the wheel is 500 centimeters and the radius of the axle is 2.5 centimeters?","Completion":"100"} {"Type":"TOSS-UP\n\nCHEMISTRY Multiple Choice","Prompt":"A lead chromate solution is typically what color:","Completion":"W) blue X) red Y) yellow Z) green Y) YELLOW"} {"Type":"TOSS-UP\n\nBIOLOGY Multiple Choice","Prompt":"From which of the following does the seed coat of angiosperm seeds typically develop:\n\nW) integuments X) polar nuclei Y) nucleus Z) cotyledons","Completion":"W) INTEGUMENTS"} {"Type":"TOSS-UP\n\n11. MATH Multiple Choice","Prompt":"Which of the following is NOT true of matrices:\n\nW) a square matrix that has an inverse is called invertible\n\nX) a matrix does not have to have an inverse\n\nY) a matrix that does not have an inverse is called non-singular\n\nZ) a matrix must be square in order to have an inverse","Completion":"Y) A MATRIX THAT DOES NOT HAVE AN INVERSE IS CALLED NON-SINGULAR"} {"Type":"TOSS-UP\n\n12. EARTH SCIENCE Multiple Choice","Prompt":"Which of the following are thought to form the mantle of the Earth:\n\nW) ultramafic rocks\n\nX) felsic rocks\n\nY) silicic rocks\n\nZ) mafic rocks","Completion":"W) ULTRAMAFIC ROCKS"} {"Type":"TOSS-UP\n\nGENERAL SCIENCE Multiple Choice","Prompt":"Which of the following is the p value for F = 0 in an ANOVA F test:","Completion":"W) 0.00 X) 0.50 Y) 0.95 Z) 1.00 Z) 1.00"} {"Type":"BONUS\n\nGENERAL SCIENCE Short Answer","Prompt":"Name all of the following 4 choices that are characteristic of scientific publications:\n\nscientific papers are peer reviewed\n\nscientific papers present more detailed data\n\nconclusions in scientific papers are supported by original data in the publication\n\nscientific papers have data that has been proven to be true","Completion":"1; 2; 3"} {"Type":"TOSS-UP\n\nASTRONOMY Multiple Choice","Prompt":"Which of the following is NOT true:\n\nW) an eclipse of the Moon only occurs during a full Moon\n\nX) Mercury will appear to be in its full phase when it is farthest from Earth\n\nY) an event horizon is associated with black holes\n\nZ) no more than 50% of the Moon's surface can be seen over any one year","Completion":"Z) NO MORE THAN 50% OF THE MOON'S SURFACE CAN BE SEEN OVER ANY ONE YEAR (Solution: libration will allow more than 55% to be seen over a year)"} {"Type":"BONUS\n\nASTRONOMY Multiple Choice","Prompt":"Which of the following is the most common location of young blue and white stars in a spiral galaxy:\n\nW) halo\n\nX) spiral arms\n\nY) nucleus\n\nZ) throughout the entire galaxy","Completion":"X) SPIRAL ARMS"} {"Type":"TOSS-UP\n\nPHYSICS Short Answer","Prompt":"What physical property of neutrinos, which was confirmed in the past decade, contradicts earlier assumptions that neutrinos travel at the speed of light?","Completion":"MASS (ACCEPT: NEUTRINOS HAVE MASS)"} {"Type":"TOSS-UP\n\nCHEMISTRY Multiple Choice","Prompt":"When dissociated in water, which of the following acids has its dissociation equilibrium shifted farthest to the right:\n\nW) HClO\n\nX) HClO\\({}{2}\\)\n\nY) HClO\\({}{3}\\)\n\nZ) HClO\\({}{4}\\)","Completion":"Z) HClO\\({}{4}\\)"} {"Type":"TOSS-UP\n\nBIOLOGY Multiple Choice","Prompt":"Of which of the following is endosperm in wheat primarily composed:\n\nW) protein\n\nX) nucleic acid\n\nY) carbohydrate\n\nZ) lipid","Completion":"Y) CARBOHYDREATE"} {"Type":"TOSS-UP\n\nMATH Short Answer","Prompt":"By words or number, name all of the following 3 statements that are TRUE for the function, \\(f(x)=-3x^{2}-2x-2=0\\):\n\nthere are no real zeros\n\nthe graph is a parabola opening downward\n\nthe graph has no \\(x\\)-intercepts","Completion":"ALL"} {"Type":"TOSS-UP\n\nEARTH SCIENCE Multiple Choice","Prompt":"Halos and rings around the Moon in the night sky are most often caused by which of the following:\n\nW) nucleation particles in the stratosphere\n\nX) small water droplets in nimbostratus clouds\n\nY) temperature inversions\n\nZ) ice crystals in cirrus clouds","Completion":"Z) ICE CRYSTALS IN CIRRUS CLOUDS"} {"Type":"TOSS-UP\n\nGENERAL SCIENCE Multiple Choice","Prompt":"Which of the following plastics is very tough, optically transparent, and often used in the manufacture of CD's:\n\nW) polypropylene\n\nX) polycarbonate\n\nY) low density polyethylene\n\nZ) polyvinyl chloride","Completion":"X) POLYCARBONATE"} {"Type":"TOSS-UP\n\nCHEMISTRY Short Answer","Prompt":"Name the LARGEST atom or ion in EACH of the following 3 pairs:\n\nan iron 3 plus ion or an iron 2 plus ion\n\nan oxygen atom or an oxygen 2 minus ion\n\na gallium 3 plus ion or a gallium atom","Completion":"1) IRON 2 PLUS ION; 2) OXYGEN 2 MINUS ION; 3) GALLIUM ATOM"} {"Type":"TOSS-UP\n\nBIOLOGY Short Answer","Prompt":"From the FIRST to the LAST, what are the stages into which interphase is most commonly divided?","Completion":"G-1; S; G-2 (ACCEPT: GAP ONE, DNA SYNTHESIS, GAP 2)"} {"Type":"TOSS-UP\n\nCHEMISTRY Short Answer","Prompt":"Giving your answer to the first decimal place and in liters, one mole of gas at 1.00 atmosphere pressure and a temperature of 273.15 kelvin occupies what volume?","Completion":"22.4"} {"Type":"BONUS\n\nCHEMISTRY Short Answer","Prompt":"A sample of gas occupies a volume of 300 cubic centimeters at a pressure of 2 atmospheres. Calculate what the pressure would have to be, in atmospheres to the first decimal place, in order for the gas to have a volume of 500 cubic centimeters at constant temperature:","Completion":"1.2"} {"Type":"TOSS-UP\n\nCHEMISTRY Short Answer","Prompt":"What is the van 't Hoff factor for glucose?","Completion":"1"} {"Type":"TOSS-UP\n\nEARTH AND SPACE Short Answer","Prompt":"What is the name for a spherical collection of stars that orbits a galactic core as a satellite?","Completion":"GLOBULAR CLUSTER"} {"Type":"TOSS-UP\n\nBIOLOGY Multiple Choice","Prompt":"Which period involved a major extinction event that included the dinosaurs?\n\nW) Between the Paleozoicic and Mesozoic eras\n\nX) Between the Phanerozoic and the Precambrian eras\n\nY) Between the Mesozoic and Cenozoic eras\n\nZ) Between the Precambrian and Paleozoic eras","Completion":"Y) BETWEEN THE MESOZOIC AND CENOZOIC ERAS"} {"Type":"BONUS\n\nBIOLOGY Multiple Choice","Prompt":"If a 24 hour clock was made to model the major events of the evolution of life on Earth, humans would appear:\n\nW) Close to the beginning of the 24 hour period\n\nX) About 12 hours through the 24 hour period\n\nY) About 2 hours before the 24 hours were up\n\nZ) Less than a minute before the 24 hours were up","Completion":"Z) LESS THAN A MINUTE BEFORE THE 24 HOURS WERE UP"} {"Type":"TOSS-UP\n\nPHYSICS Multiple Choice","Prompt":"Which of the following does a Carnot engine convert?\n\nW) Energy to heat\n\nX) Pressure to volume\n\nY) Heat to work\n\nZ) Work to pressure","Completion":"Y) HEAT TO WORK"} {"Type":"BONUS\n\nPHYSICS Short Answer","Prompt":"A heat engine operates with 60 kilocalories of heat supplied, and exhausts 40 kilocalories of heat. How much work, in joules, did the engine do? Provide your answer to the nearest thousand.","Completion":"84,000"} {"Type":"TOSS-UP\n\nBIOLOGY Multiple Choice","Prompt":"Flowers are actually miniature stems that produce four kinds of specialized:\n\nW) Roots\n\nX) Leaves\n\nY) Vascular tissue\n\nZ) Cones","Completion":"X) LEAVES"} {"Type":"TOSS-UP\n\nPHYSICS Short Answer","Prompt":"Careful measurements have shown that the mass of a particular atom is always slightly less than the sum of the masses of the individual neutrons, protons, and electrons of which the atom consists. What is the difference between the mass of the atom and the sum of the masses of its parts called?","Completion":"MASS DEFECT"} {"Type":"TOSS-UP\n\nBIOLOGY Short Answer","Prompt":"What are the most abundant cells in bones?","Completion":"OSTEOCYTES (read as: OHS-tee-oh-syts)"} {"Type":"TOSS-UP\n\nCHEMISTRY Short Answer","Prompt":"What is the formula for sulfurous acid?","Completion":"H\\({}{2}\\)SO\\({}{3}\\)"} {"Type":"TOSS-UP\n\nPHYSICS Multiple Choice","Prompt":"Which of the following explains why extremely high energy is needed for fusion to work?\n\nW) To break the nuclei into two parts\n\nX) To bind the protons and electrons\n\nY) To overcome the electrical repulsion between two positively charged nuclei\n\nZ) To create the necessary supercold environment","Completion":"Y) TO OVERCOME THE ELECTRICAL REPUSSION BETWEEN TWO"} {"Type":"TOSS-UP\n\nCHEMISTRY Multiple Choice","Prompt":"Which of the following chemical bonds is not important in polymer formation?\n\nW) Glycosidic (read as: gly-koh-SYDIK)\n\nX) Hydrogen\n\nY) Peptide\n\nZ) Phosphodiester (read as: FOHS-foh-DY-ehs-tehr)","Completion":"X) HYDROGEN"} {"Type":"BONUS\n\nCHEMISTRY Short Answer","Prompt":"What relationship exists between bond length and bond order?","Completion":"INVERSE RELATIONSHIP (ACCEPT: BONDS OF HIGHER ORDER ARE"} {"Type":"TOSS-UP\n\nMATH Multiple Choice","Prompt":"Which of the following radical expressions is in simplest form?\n\nW) \\(\\frac{\\sqrt{31}}{3}\\) (read as: the square root of 31 over X) -5 \\(\\sqrt{175}\\) (read as: negative 5 times the square root of 175)\n\nY) 6 \\(\\sqrt{\\frac{x^{2}}{4}}\\) (read as: 6 times the square root of the quantity \\(x\\) squared over Z) - \\(\\sqrt{153}\\ x\\) (read as: the negative square root of 153 \\(x\\))","Completion":"W) \\(\\frac{\\sqrt{31}}{3}\\)"} {"Type":"BONUS\n\nMATH Short Answer","Prompt":"If \\(x\\) is negative, simplify \\(7\\sqrt{5x}\\ \\times\\sqrt{180\\ x^{5}}\\) (read as: 7 times the square root of 5x times the square root of, open parenthesis, 180x to the fifth, close parenthesis)","Completion":"-210x\\({}^{3}\\)"} {"Type":"TOSS-UP\n\nENERGY Short Answer","Prompt":"What is the term used to describe the total amount of energy used in the production of a product, including raw material extraction, manufacture, transportation, assembly, and all other aspects of production?","Completion":"EMBODIED ENERGY"} {"Type":"BONUS\n\nENERGY Multiple Choice","Prompt":"In 2009, demand for small wind systems for residential and small business use grew 15%, adding 20 megawatts of wind-generating capacity. What was the approximate capacity of small wind systems in 2008 in megawatts?\n\nW) 17\n\nX) 35\n\nY) 115\n\nZ) 133","Completion":"Z) 133"} {"Type":"TOSS-UP\n\nEARTH AND SPACE Multiple Choice","Prompt":"Which of the following is NOT a metamorphic rock?\n\nW) Schist (read as: SHIST)\n\nX) Gneiss (read as: NYS)\n\nY) Basalt\n\nZ) Marble","Completion":"Y) BASALT"} {"Type":"TOSS-UP\n\nPHYSICS Multiple Choice","Prompt":"An electron-volt is the:\n\nW) Energy gained by 1 electron falling through a 1 meter vertical distance\n\nX) Energy of a visible light photon\n\nY) Energy of 1 electron accelerated through a 1 volt potential difference\n\nZ) Energy of a typical electron orbiting a nucleus","Completion":"Y) ENERGY OF 1 ELECTRON ACCELERATED THROUGH A 1 VOLT POTENTIAL DIFFERENCE"} {"Type":"TOSS-UP\n\nENERGY Short Answer","Prompt":"During oxidative phosphorylation, the proton gradient is used to drive which enzyme?","Completion":"ATP SYNTHASE"} {"Type":"TOSS-UP\n\nMATH Short Answer","Prompt":"What are the x-intercepts of \\(y=x^{2}-15x+54\\)?","Completion":"6 AND 9"} {"Type":"TOSS-UP\n\nENERGY Short Answer","Prompt":"An engine yields 100 watts of power for every 1000 watts received. What is the efficiency of the engine?","Completion":"10%"} {"Type":"TOSS-UP\n\nEARTH AND SPACE Short Answer","Prompt":"Which of the following molecules is NOT known to destroy ozone molecules in our stratosphere?\n\nW) Chloroflurocarbons\n\nX) Chlorine\n\nY) Bromine\n\nZ) Argon","Completion":"Z) ARGON"} {"Type":"TOSS-UP\n\nBIOLOGY Multiple Choice","Prompt":"Which of the following is NOT an example of the negative effects of bioaccumulation of toxins in the environment?\n\nW) The thinning of bird eggshells due to DDT\n\nX) Inedibility of oysters due to their filter feeding on naturally occurring red tide algae\n\nY) Birth defects in babies exposed to Accutane in utero\n\nZ) Inedibility of fish due to mercury content","Completion":"Y) BIRTH DEFECTS IN BABIES EXPOSED TO ACCUTANE IN UTERO"} {"Type":"BONUS\n\nBIOLOGY Multiple Choice","Prompt":"If a deer population exceeds its carrying capacity, which of the following will likely happen?\n\nW) The birth rate will increase\n\nX) The death rate will increase\n\nY) The death rate will decrease\n\nZ) Carrying capacity will increase","Completion":"X) THE DEATH RATE WILL INCREASE"} {"Type":"TOSS-UP\n\nMATH Short Answer","Prompt":"What is the reference angle for \\(\\frac{7\\pi}{6}\\)?","Completion":"\\(\\frac{\\pi}{6}\\)"} {"Type":"BONUS\n\nMATH Short Answer","Prompt":"Given coordinates A (1,, B (7,, C (3,, and D (4,, find the angle in degrees between \\(\\overline{\\mbox{AB}}\\) and \\(\\overline{\\mbox{CD}}\\).","Completion":"\\(90^{\\circ}\\)"} {"Type":"TOSS-UP\n\nEARTH AND SPACE Multiple Choice","Prompt":"Flood basalts, such as the Cretaceous Deccan\n\nTraps in Siberia, may have had a major impact on which of the following past events?\n\nW) Climate change\n\nX) Formation of island arcs\n\nY) Human development\n\nZ) Interruption in stratospheric circulation","Completion":"W) CLIMATE CHANGE"} {"Type":"BONUS\n\nEARTH AND SPACE Short Answer","Prompt":"The Deccan traps of 66 million years ago may have influenced Earth's climate through the release of which sulfur species common to volcanic gases?","Completion":"SULFUR DIOXIDE"} {"Type":"TOSS-UP\n\nMATH Short Answer","Prompt":"What is the smallest integer greater than 1 that is relatively prime with 210?","Completion":"11"} {"Type":"BONUS\n\nMATH Multiple Choice","Prompt":"Which of the following numbers is not a Mersenne prime?\n\nW) 7\n\nX) 31\n\nY) 61\n\nZ) 127","Completion":"Y) 61"} {"Type":"TOSS-UP\n\nCHEMISTRY Multiple Choice","Prompt":"Which of the following 1.0 molar solutions will have the highest pH?\n\nW) Hydrochloric acid\n\nX) Nitrous acid\n\nY) Sulfuric acid\n\nZ) Hydrobromic acid","Completion":"X) NITROUS ACID"} {"Type":"TOSS-UP\n\nPHYSICS Short Answer","Prompt":"What is the phrase that describes the process of producing an electron and a positron when a high-energy photon travels through matter, directly converting radiant energy into matter?","Completion":"PAIR PRODUCTION"} {"Type":"BONUS\n\nPHYSICS Short Answer","Prompt":"A living room has floor dimensions of 3.5 meters by 4.2 meters and a height of 2.4 meters. Given air density of 1.2 kilograms per cubic meter, g of 10 meters per second squared, and providing your answer to the nearest ten, what does the air in the room weigh in newtons?","Completion":"420"} {"Type":"TOSS-UP\n\nBIOLOGY Multiple Choice","Prompt":"Which of the following are NOT found in the thylakoid membranes of the chloroplast?\n\nW) Carotenoids\n\nX) Rubisco\n\nY) Water\n\nZ) ADP","Completion":"X) RUBISCO"} {"Type":"BONUS\n\nBIOLOGY Short Answer","Prompt":"By name or number, order the following five cell layers in a eudicot (read as: yuh-DY-koht) root from most lateral to most medial: pericycle, epidermis, endodermis, phloem, cortex.","Completion":"EPIDERMIS, CORTEX, ENDODERMIS, PERICYCLE, PHLOEM (ACCEPT: 2, 5, 3, 1, 4)"} {"Type":"TOSS-UP\n\nCHEMISTRY Short Answer","Prompt":"Give the specific name for the following functional group: a nitrogen atom bonded to two carbon chains and a hydrogen atom?","Completion":"SECONDARY AMIINE"} {"Type":"TOSS-UP\n\nPHYSICS Short Answer","Prompt":"The period of revolution of a particle traveling in uniform circular motion is a function of what two variables?","Completion":"VELOCITY (ACCEPT: CENTRIPETAL FORCE) AND RADIUS (ACCEPT: CIRUMERENCE, DIAMETER)"} {"Type":"TOSS-UP\n\nEARTH AND SPACE Short Answer","Prompt":"The freshwater and saltwater zones within coastal aquifers are separated by a transition zone, also called the zone of dispersion, within which there is mixing between freshwater and saltwater. Which state of the Northern Atlantic Coastal Plain Aquifer System has the shallowest depth to the top of the transition zone?","Completion":"NORTH CAROLINA"} {"Type":"BONUS\n\nEARTH AND SPACE Short Answer","Prompt":"In a stratified period of a lake, oxygen supplied directly by the atmosphere and photosynthetic organisms replenishes the dissolved oxygen of which of the following three choices: epilimnion (read as: eh-pi-LIM-nee-ohn), hypolimnion (read as: hy-p6e-LIM-nee-ohn), or thermocine?","Completion":"EPILIMNION (ACCEPT: 2)"} {"Type":"TOSS-UP\n\nPHYSICS Short Answer","Prompt":"Neglecting friction, a 33-kilogram cart rolls down a slope that is 20 meters high and 60 meters long. Using 10 meters per second squared for g, what is the velocity in meters per second at the base of the slope?","Completion":"20"} {"Type":"BONUS\n\nPHYSICS Short Answer","Prompt":"A long straight wire carries a 10 ampere current north. Give the magnitude in teslas and the direction of the magnetic field induced at a point 10 centimeters directly under the wire?","Completion":"2 x 10.5 WEST"} {"Type":"TOSS-UP\n\nENERGY Multiple Choice","Prompt":"Approximately how much energy does the Sun generate in joules per hour if it converts about 4 billion kilograms of mass into pure energy every second?\n\nW) 10\\({}^{9}\\)\n\nX) 10\\({}^{18}\\)\n\nY) 10\\({}^{30}\\)\n\nZ) 10\\({}^{33}\\)","Completion":"Y) 10\\({}^{30}\\)"} {"Type":"BONUS\n\nENERGY Multiple Choice","Prompt":"In the presence of alcohol dehydrogenase, the rate of reduction of acetaldehyde to ethanol increases as you increase the concentration of acetaldehyde. Eventually, the rate of reaction reaches a maximum, where further increases in the concentration of acetaldehyde have no effect. Which of the following BEST explains this result?\n\nW) All of the alcohol dehydrogenase molecules are bound to acetaldehyde molecules\n\nX) At high concentrations of acetaldehyde, the activation energy of the reaction decreases\n\nY) The enzyme is no longer specific for acetaldehyde\n\nZ) At high concentrations of acetaldehyde, the change in free energy of the reaction decreases","Completion":"W) ALL OF THE ALCOHOL DEHYDROGENASE MOLECULES ARE BOUND TO ACETALDEHYDE MOLECULES"} {"Type":"TOSS-UP\n\nMATH Multiple Choice","Prompt":"Populations P1 and P2 are normally distributed and have identical means; however, the standard deviation of P1 is twice the standard deviation of P2. Which of the following is TRUE about the percentage of observations falling within two standard deviations of the mean for each population?\n\nW) The percentage for P1 is twice the percentage for P2\n\nX) The percentage for P1 is greater, but not twice as great, as the percentage for P2\n\nY) The percentage for P2 is twice the percentage for P1\n\nZ) The percentages are identical","Completion":"Z) THE PERCENTAGES ARE IDENTICAL"} {"Type":"BONUS\n\nMATH Short Answer","Prompt":"Find the second derivative of the function \\(f(x)=(3+2x)\\mbox{e}^{\\,3x}\\).","Completion":"\\(3(6x+5)\\mbox{e}^{\\,3x}\\) [ACCEPT: (18x + 15)\\mbox{e}^{\\,3x}\\)]"} {"Type":"TOSS-UP\n\nEARTH AND SPACE Multiple Choice","Prompt":"Which of the following compounds is NOT a common cement in sedimentary rocks?\n\nW) SiO\\({}{2}\\)\n\nX) Fe\\({}{2}\\)O\\({}{3}\\)\n\nY) Mg\\({}{2}\\)SiO\\({}{3}\\)\n\nZ) CaCO\\({}{3}\\)","Completion":"Y) Mg\\({}{2}\\)SiO\\({}{3}\\)"} {"Type":"BONUS\n\nEARTH AND SPACE Short Answer","Prompt":"The Ghyben-Herzberg relation characterizes the relationship between the depth of freshwater occurring above and below sea level in the presence of a saltwater intrusion. What physical property determines this relationship?","Completion":"DENSITY"} {"Type":"TOSS-UP\n\nPHYSICS Multiple Choice","Prompt":"If the coulomb constant is 9 \\(\\times\\) 10\\({}^{9}\\) newton meters squared per coulomb squared, what force, in newtons, will two particles with charges of +2 and -2 coulombs kept 2 meters apart exert on each other?\n\nW) 4.5 \\(\\times\\) 10\\({}^{9}\\) away from each other\n\nX) 4.5 \\(\\times\\) 10\\({}^{9}\\) toward each other\n\nY) 9.0 \\(\\times\\) 10\\({}^{9}\\) toward each other\n\nZ) 1.8 \\(\\times\\) 10\\({}^{10}\\) toward each other","Completion":"Y) 9.0 \\(\\times\\) 10\\({}^{9}\\) TOWARD EACH OTHER"} {"Type":"BONUS\n\nPHYSICS Short Answer","Prompt":"Pam buys a pendulum clock at a discount store and discovers when she gets home that it loses 6 minutes each day. If the pendulum has a period of 2 seconds, by how much must the length be changed, to the nearest centimeter, so that the clock keeps accurate time?","Completion":"1"} {"Type":"TOSS-UP\n\nEARTH AND SPACE Multiple Choice","Prompt":"Which of the following statements is TRUE about Population I stars and II stars?\n\nW) Population I stars tend to be older than Population II stars\n\nX) Population I stars of a given mass tend to be more luminous than Population II stars of the same mass\n\nY) Population I stars tend to be found in the spiral arms of galaxies, while Population II stars tend to be found in globular clusters\n\nZ) Population I stars tend to have fewer heavy elements than Population II stars","Completion":"Y) POPULATION I STARS TEND TO BE FOUND IN THE SPIRAL ARMS OF GALAXIES, WHILE POPULATION II STARS TEND TO BE FOUND IN GLOBULAR CLUSTERS"} {"Type":"TOSS-UP\n\nENERGY Short Answer","Prompt":"What is the term for a kind of landfill technology, currently in research and development, that accelerates the biological decomposition of organic wastes in a landfill by promoting the conditions necessary for the microorganisms that degrade the waste?","Completion":"BIOREACTOR"} {"Type":"TOSS-UP\n\nBIOLOGY Short Answer","Prompt":"Cowbirds, common cuckoos, and African honeyguides lay their eggs in the nests of other birds. As a result, what term is typically used to describe these birds?","Completion":"BROOD PARASITE"} {"Type":"TOSS-UP\n\nMATH Short Answer","Prompt":"If \\(x^{2}+y^{2}=5^{2}\\), find \\(\\frac{dy}{dx}\\) in terms of \\(x\\) and \\(y\\) in simplest form.","Completion":"\\(\\frac{-x}{y}\\)"} {"Type":"TOSS-UP\n\nBIOLOGY Multiple Choice","Prompt":"Which of the following organelles would you expect to be MOST active during apoptosis of the cells that contain them?\n\nW) Endoplasmic reticulum\n\nX) Golgi bodies\n\nY) Lysosomes\n\nZ) Ribosomes","Completion":"Y) LYSOSOMES"} {"Type":"BONUS\n\nBIOLOGY Short Answer","Prompt":"What is the scientific name for a type of skin growth that looks different than a common mole? It is usually larger, with borders that are not easy to see, uneven colors, and is partially raised above the skin surface.","Completion":"DYSPLASTIC NEVUS (ACCEPT: DYSPLASTIC MELANOCYTIC NEVUS)"} {"Type":"TOSS-UP\n\nENERGY Multiple Choice","Prompt":"Roughly, which of the following BEST represents the solar energy intensity in watts per square meter incident on the Earth?\n\nW) 200\n\nX) 1,400\n\nY) 3,000\n\nZ) 10,000","Completion":"X) 1,400"} {"Type":"BONUS\n\nENERGY Short Answer","Prompt":"Companies A, B, and C are forced to pay one dollar of tax per ton of carbon dioxide equivalent emissions. If Company A emits 100 tons of carbon dioxide and 4 tons of nitrous oxide, Company B emits 10 tons of carbon dioxide and 5 tons of nitrous oxide, and Company C emits 200 tons of carbon dioxide and no nitrous oxide, which company has to pay the largest tax?","Completion":"COMPANY B"} {"Type":"TOSS-UP\n\nPHYSICS Short Answer","Prompt":"By number, identify all of the following four situations in which an object is NOT in equilibrium: when moving to the right with a constant velocity; when slowing to a stop; when moving with a constant speed in a circle; when in free fall.","Completion":"2, 3, AND 4"} {"Type":"BONUS\n\nPHYSICS Short Answer","Prompt":"A brass ball with a mass of 200 grams at 20\\({}^{\\circ}\\) Celsius is placed in boiling water until it reaches equilibrium. Given that the specific heat of brass is 0.094 kilocalories per kilogram degree Celsius, and providing your answer as a decimal to the nearest tenth, how much thermal energy in kilocalories is absorbed by the ball?","Completion":"1.5"} {"Type":"TOSS-UP\n\nBIOLOGY Short Answer","Prompt":"What is the name of a highly contagious, infectious viral disease in cows that causes symptoms such as respiratory disease, conjunctivitis, encephalitis, and brain infections?","Completion":"INFECTIONUS BOVINE RHINOTRACHEITIS (ACCEPT: IBR) (DO NOT ACCEPT: MAD COW DISEASE)"} {"Type":"BONUS\n\nBIOLOGY Short Answer","Prompt":"By name or number, identify all of the following five choices that are NOT arthropods: trilobites, chelicerates (read as: ki-LI-keh-rah-tehs), crustaceans, icterids (read as: ik-TEHR-ids), polyplacophores (read as: poh-lee-PLAH-koh-fors).","Completion":"ICTERIDS AND POLYPLACOPHORES (ACCEPT: 4 AND 5)"} {"Type":"TOSS-UP\n\nMATH Multiple Choice","Prompt":"Which of the following series converges?\n\nW) \\(\\sum{n=1}^{\\infty}\\frac{1}{n^{0.5}}\\)\n\nX) \\(\\sum{n=1}^{\\infty}\\frac{1}{n+1}\\)\n\nY) \\(\\sum{n=1}^{\\infty}\\frac{n}{\\mathrm{e}^{n}}\\)\n\nZ) \\(\\sum{n=1}^{\\infty}\\frac{\\mathrm{e}^{n}}{n}\\)","Completion":"Y) \\(\\sum{n=1}^{\\infty}\\frac{n}{\\mathrm{e}^{n}}\\)"} {"Type":"TOSS-UP\n\nCHEMISTRY Short Answer","Prompt":"By name or number, identify all of the following four substances that are substitutional alloys: brass, steel, sterling silver, pewter.","Completion":"BRASS, STERLING SILVER, AND PEWTER (ACCEPT: 1, 3, AND 4)"} {"Type":"TOSS-UP\n\nEARTH AND SPACE Multiple Choice","Prompt":"Which of the following is the closest to the maximum solar altitude for Barrow, Alaska in the course of a year?\n\nW) 90\\({}^{\\circ}\\)\n\nX) 45\\({}^{\\circ}\\)\n\nY) 20\\({}^{\\circ}\\)\n\nZ) 0\\({}^{\\circ}\\)","Completion":"X) 45\\({}^{\\circ}\\)"} {"Type":"TOSS-UP\n\nCHEMISTRY Multiple Choice","Prompt":"A well near the beach is suspected to be contaminated with chloride ions from sea water. If chloride is present, which of the following will produce a cloudy suspension when mixed with the water?\n\nW) Silver nitrate\n\nX) Sodium nitrate\n\nY) Potassium acetate\n\nZ) Calcium bromide","Completion":"W) SILVER NITRATE"} {"Type":"TOSS-UP\n\nBIOLOGY Short Answer","Prompt":"Amebocytes from Limulus organisms are currently used in the medical field to test for the presence of what substances?","Completion":"BACTERIAL TOXINS (ACCEPT: TOXINS)"} {"Type":"TOSS-UP\n\nENERGY Short Answer","Prompt":"When a 10 kilogram object is lifted 10 meters, in a gravitational field of 9.8 meters per second squared, what is the change in potential energy of the object in joules?","Completion":"980"} {"Type":"TOSS-UP\n\nCHEMISTRY Multiple Choice","Prompt":"Which of the following is the name of the compound HClO\\({}{4}\\)?\n\nW) Hydrogen perchlorite\n\nX) Chloric acid\n\nY) Perchloric acid\n\nZ) Hypochloric acid","Completion":"Y) PERCHLORIC ACID"} {"Type":"TOSS-UP\n\nMATH Short Answer","Prompt":"Express \\(\\sum{x=1}^{n}\\log x\\) (read as: the summation of \\(x\\) equals 1 to n of log of \\(x\\)) with a factorial.","Completion":"log \\(n!\\)"} {"Type":"BONUS\n\nMATH Short Answer","Prompt":"How many different letter arrangements can be formed using the six letters in the word \"energy\"?","Completion":"360"} {"Type":"TOSS-UP\n\nPHYSICS Short Answer","Prompt":"If Tajon applies 100 newtons of force on a 2-meter wrench at a right angle to the wrench and parallel to the plane of rotation, how much torque, in newton meters, is he applying to the bolt?","Completion":"200"} {"Type":"BONUS\n\nPHYSICS Multiple Choice","Prompt":"What makes arches and domes more stable than flat roofed structures?\n\nW) They transmit compressive forces along their length to the ground, uniformly bearing the load of gravity\n\nX) Each section is under great tension, using the stronger dimension of the components\n\nY) Torsional resistance to sideways swaying ensures that such structures are stable, even in high winds\n\nZ) The surface tension of the arched shape allows the building components to be hollowed out, saving weight.","Completion":"W) THEY TRANSMIT COMPRESSIVE FORCES ALONG THEIR LENGTH TO THE GROUND, UNIFORMLY BEARING THE LOAD OF GRAVITY"} {"Type":"TOSS-UP\n\nEARTH AND SPACE Multiple Choice","Prompt":"Which of the following is a common mineral that is composed entirely of silicon and oxygen?\n\nW) Pyrite\n\nX) Quartz\n\nY) Bauxite (read as: BOHK-syt)\n\nZ) Feldspar","Completion":"X) QUARTZ"} {"Type":"BONUS\n\nEARTH AND SPACE Multiple Choice","Prompt":"Which of the following minerals is a mafic silicate?\n\nW) Fluorite\n\nX) Diamond\n\nY) Muscovite\n\nZ) Pyroxene (read as: py-ROHK-seen)","Completion":"Z) PYROXENE"} {"Type":"TOSS-UP\n\nMATH Multiple Choice","Prompt":"Which of the following functions is one-to-one?\n\nW) f(x) = |x| + 3 (read as: f of \\(x\\) equals the absolute value of \\(x\\) plus X) f(x) = \\(x^{2}-6x+11\\) (read as: f of \\(x\\) equals \\(x\\) squared minus 6 \\(x\\) plus Y) f(x) = \\(5x^{3}+9\\) (read as: f of \\(x\\) equals 5 \\(x\\) cubed plus Z) f(x) = \\(\\frac{1}{x^{2}}\\) (read as: f of \\(x\\) equals 1 over \\(x\\) squared)","Completion":"Y) f(x) = \\(5x^{3}+9\\)"} {"Type":"BONUS\n\nMATH Short Answer","Prompt":"A triangle has sides of length 5, 7, and 8 meters. In simplest form and in meters squared, what is the area of the triangle?","Completion":"10\\(\\sqrt{3}\\)"} {"Type":"TOSS-UP\n\nENERGY Multiple Choice","Prompt":"Why is riding a bicycle on a well-maintained road more energy efficient than walking or running?\n\nW) Bipedal motion is highly inefficient\n\nX) The coefficient of sliding friction between tires and the ground is less than that between shoes and the ground\n\nY) Gravity does very little work on a bicyclist relative to a walker or runner\n\nZ) Bicycling requires more calories than walking","Completion":"Y) GRAVITY DOES VERY LITTLE WORK ON A BICYCLIST RELATIVE TO A WALKER OR RUNNER"} {"Type":"TOSS-UP\n\nPHYSICS Multiple Choice","Prompt":"Which of the following has NOT been proposed as a way to test string theory?\n\nW) Cosmic ray experiments\n\nX) High energy particle collisions\n\nY) Electron mapping\n\nZ) Precision tests of short-range gravitational forces","Completion":"Y) ELECTRON MAPPING"} {"Type":"TOSS-UP\n\nENERGY Short Answer","Prompt":"On a football team, there is a defensive lineman who weighs 350 pounds and can run at 10 miles per hour. On the same team, the wide receiver weighs 175 pounds and can run 20 miles per hour. At top speed, which player has more kinetic energy?","Completion":"WIDE RECEIVER"} {"Type":"BONUS\n\nENERGY Short Answer","Prompt":"A race car initially moving with a velocity of 50 meters per second produces a skid mark on the road while braking. Using g as 10 meters per second squared, and providing your answer to the nearest integer, how long is the skid mark in meters if the coefficient of kinetic friction is 1.0?","Completion":"125"} {"Type":"TOSS-UP\n\nEARTH AND SPACE Short Answer","Prompt":"Transpolar drift is responsible for the net transport of water and ice in what direction in the Arctic Basin?","Completion":"WEST TO EAST"} {"Type":"BONUS\n\nEARTH AND SPACE Short Answer","Prompt":"Most sea ice exiting the Arctic Basin passes through what large strait between Greenland and Svalbard (read as: SVAHL-bahr)?","Completion":"THE FRAM STRAIT"} {"Type":"TOSS-UP\n\nPHYSICS Multiple Choice","Prompt":"A car of mass 700 kilograms travels at 20 meters per second and collides with a stationary truck of mass 1400 kilograms. The two vehicles interlock as a result of the collision and slide along an icy road. Which of the following is the approximate velocity in meters per second of the combined system?\n\nW) 5.6\n\nX) 6.6\n\nY) 7.6\n\nZ) 8.6","Completion":"X) 6.6"} {"Type":"BONUS\n\nPHYSICS Multiple Choice","Prompt":"Given that the specific heat of iron is 0.12 calories per gram degree Celsius, what will be the final temperature in degrees Celsius of 100 grams of 20 degree Celsius water when 100 grams of 40 degree Celsius iron nails are submerged in it? Provide your answer to the nearest whole number.","Completion":"22"} {"Type":"TOSS-UP\n\nMATH Short Answer","Prompt":"Find the derivative of \\(12x^{3}\\) - \\(6x^{2}+\\frac{3}{x^{3}}-25\\) (read as: 12 \\(x\\) cubed minus 6 \\(x\\) squared plus 3 over \\(x\\) to the fifth minus","Completion":"\\(36x^{2}-12x\\) - \\(\\frac{15}{x^{6}}\\)"} {"Type":"BONUS\n\nMATH Short Answer","Prompt":"A particle's displacement in meters can be described by the function f(x) = \\(2x^{3}+3x^{2}\\) - \\(10x\\) + 30. At \\(x\\) = 5 seconds, what is the acceleration in meters per second squared? Provide your answer to the nearest integer.","Completion":"66"} {"Type":"TOSS-UP\n\nEARTH AND SPACE Short Answer","Prompt":"Besides Earth, what is the only other planet in our solar system known to have water ice on its surface?","Completion":"MARS"} {"Type":"TOSS-UP\n\nMATH Short Answer","Prompt":"What is \\(\\tan\\frac{5\\pi}{4}+\\sin\\frac{\\pi}{6}\\)?","Completion":"\\(\\frac{3}{2}\\)"} {"Type":"TOSS-UP\n\nCHEMISTRY Short Answer","Prompt":"What is the pH of 1.0 mole of sodium hydroxide dissolved in 10 liters of water?","Completion":"13"} {"Type":"TOSS-UP\n\nBIOLOGY Multiple Choice","Prompt":"Which of the following best describes the way in which the summation of action potentials from a motor neuron can bring a muscle fiber to the maximal level of contraction?\n\nW) Summation of contractions\n\nX) Superstimulus\n\nY) Maximal twitch rate\n\nZ) Tetany (read as: TEHT-nee)","Completion":"Z) TETANY"} {"Type":"TOSS-UP\n\nCHEMISTRY Multiple Choice","Prompt":"The ability of geckos to climb smooth surfaces is due to which of the following?\n\nW) Hydrogen bonding\n\nX) Van der Waals forces\n\nY) Ion-dipole forces\n\nZ) Intramolecular forces","Completion":"X) VAN DER WAALS FORCES"} {"Type":"BONUS\n\nCHEMISTRY Multiple Choice","Prompt":"Nanotechnologists are creating butterfly scale patterns on paper money to foil counterfeiters using which of the following substances?\n\nW) Calcium oxide\n\nX) Zinc oxide\n\nY) Magnesium oxide\n\nZ) Aluminum oxide","Completion":"Z) ALUMINUM OXIDE"} {"Type":"TOSS-UP\n\nBIOLOGY Short Answer","Prompt":"Tay Sachs is a fatal genetic disease. Individuals with Tay Sachs are homozygous for the recessive disease allele. If a husband and wife are both heterozygous for the gene, what is the percent chance that they will have a child with the disease?","Completion":"25%"} {"Type":"BONUS\n\nBIOLOGY Multiple Choice","Prompt":"A diploid cell has 32 chromatids in the G2 phase of the cell cycle. If you examine the cell in Propphase II you would find 2 cells each containing which of the following?\n\nW) 16 molecules of DNA\n\nX) 16 chromosomes made of DNA\n\nY) 32 molecules of DNA\n\nZ) 32 chromosomes made of DNA","Completion":"W) 16 MOLECULES OF DNA"} {"Type":"TOSS-UP\n\nCHEMISTRY Multiple Choice","Prompt":"Which of the following bonds should have the longest length?\n\nW) C-C (read as: C single bond C)\n\nX) C=C (read as: C double bond C)\n\nY) C-H (read as: C single bond H)\n\nZ) C=C (read as: C triple bond C)","Completion":"W) C-C"} {"Type":"BONUS\n\nCHEMISTRY Multiple Choice","Prompt":"Which of the following contains polar covalent bonds?\n\nW) F\\({}{2}\\)\n\nX) HF\n\nY) NaH\n\nZ) O\\({}{2}\\)","Completion":"X) HF"} {"Type":"TOSS-UP\n\nBIOLOGY Multiple Choice","Prompt":"Darwin's theory of evolution stated all of the following except:\n\nW) Those organisms best fit for their environment survive and reproduce most successfully\n\nX) Acquired characteristics can be inherited\n\nY) Species change over time through the process of natural selection\n\nZ) More organisms are produced than can survive, so living things compete for resources","Completion":"X) ACOUIRED CHARACTERISTICS CAN BE INHERITED"} {"Type":"BONUS\n\nBIOLOGY Short Answer","Prompt":"Name all of the following in which the embryonic blastopore\n\nbecomes the anus, and the mouth originates after the anus: leopard frog, cockroach, bull shark, earthworm, and\/or human.","Completion":"1) LEOPARD FROG, 3) BULL SHARK AND 5) HUMAN"} {"Type":"TOSS-UP\n\nPHYSICS Multiple Choice","Prompt":"What is one way in which power can be calculated?\n\nW) Subtracting the relative vectors \\(F\\) and \\( u\\)\n\nX) The time derivative of work performed\n\nY) The strength of the force times \\(g\\)\n\nZ) The second derivative of acceleration","Completion":"X) THE TIME DERIVATIVE OF WORK PERFORMED"} {"Type":"TOSS-UP\n\nEARTH AND SPACE Short Answer","Prompt":"If the moon is in a crescent phase and appears in the eastern sky at sunrise, what phase will be next?","Completion":"NEW MOON"} {"Type":"TOSS-UP\n\nMATH Short Answer","Prompt":"If someone has 1 red shirt, 2 blue shirts, 4 red pants, and 3 green pants from which to choose, what is the probability that a randomly selected shirt and pant combination has at least one red item? Provide your answer as a fraction in simplest form.","Completion":"5\/7"} {"Type":"BONUS\n\nMATH Short Answer","Prompt":"If the mean of a certain sample is 8 and the standard deviation is 0.25, what value has a z-score of 12?","Completion":"11"} {"Type":"TOSS-UP\n\nBIOLOGY Multiple Choice","Prompt":"An apple flower is hermaphroditic, it has both a pistil and a stamen, which means it is a type of:\n\nW) Monoecious plant\n\nX) Diecious plant (read as: dy-EE-shehs)\n\nY) Diatomaceous plant (read as: dy-ah-toh-MAY-shehs)\n\nZ) Monostomate plant","Completion":"W) MONOECIOUS PLANT"} {"Type":"BONUS\n\nBIOLOGY Short Answer","Prompt":"Many monoecious plant species avoid self pollination, reducing their likelihood of expressing deleterious recessive alleles in future generations. One strategy used by monoecious plants is to produce pollen long after the carpel of the same flower becomes able to receive it. Give the name for this strategy.","Completion":"PROTOGYNOUS FLOWERING (ACCEPT: PROTOGYNY)"} {"Type":"TOSS-UP\n\nEARTH AND SPACE Multiple Choice","Prompt":"During which of the following two days is the maximum solar altitude less than 90\\({}^{\\circ}\\) at the equator?\n\nW) Winter solstice and vernal equinox\n\nX) Vemal and autumnal equinoxes\n\nY) Winter and summer solstices\n\nZ) Summer solstice and autumnal equinox","Completion":"Y) Winter and SUMMER SOLSTICES"} {"Type":"BONUS\n\nEARTH AND SPACE Short Answer","Prompt":"Put the formation of the following in chronological order from oldest to youngest: Rocky Mountains, Great Lakes, Colorado River, Appalachian Mountains.","Completion":"APPALACHIAN MOUNTAINS, ROCKY MOUNTAINS, COLORADO RIVER, GREAT LAKES"} {"Type":"TOSS-UP\n\nBIOLOGY Multiple Choice","Prompt":"In which of the following wavelengths does DNA absorb the best:","Completion":"W) 3,900 angstroms X) 260 nanometers Y) 340 nanometers Z) 540 nanometers X) 260 NANOMETERS"} {"Type":"BONUS\n\nBIOLOGY Multiple Choice","Prompt":"Which of the following is closest to the diameter of a plasmodesmata (read as: PLAS-mo-dez-MAH-tah):","Completion":"W) 2 angstroms X) 60 nanometers Y) 80 microns Z) 180 microns X) 60 NANOMETERS"} {"Type":"TOSS-UP\n\nCHEMISTRY Short Answer","Prompt":"Name the 2 different bond angles present in a molecule with octahedral geometry, such as in SF\\({}{6}\\):","Completion":"90; 180"} {"Type":"BONUS\n\nCHEMISTRY Short Answer","Prompt":"In a titration experiment, it takes 20.0 milliliters of a 0.05 molar barium hydroxide, Ba(OH)\\({}{2}\\), solution to neutralize 40 milliliters of a nitric acid, or HNO\\({}{3}\\), solution to the equivalence point. Calculate the molarity of HNO\\({}{3}\\), rounded to the second decimal place:","Completion":"0.05"} {"Type":"TOSS-UP\n\nEARTH SCIENCE Multiple Choice","Prompt":"Which of the following is a coarse-grained igneous rock primarily containing plagioclase feldspar, amphibole and biotite:\n\nW) gabbro X) basalt Y) diorite Z) granite","Completion":"Y) DIORITE"} {"Type":"TOSS-UP\n\nGENERAL SCIENCE Multiple Choice","Prompt":"Which of the following is the source of alpha radiation most often used in household ionization smoke detectors:\n\nW) americium-241 X) uranium-238 Y) radium-226 Z) iron-59","Completion":"W) AMERICIUM-241"} {"Type":"TOSS-UP\n\nASTRONOMY Short Answer","Prompt":"Order the following 3 stages from the EARLIEST to the\n\nLATEST in the evolution of a G-type star of 2 solar masses: red giant; T-tauri; white dwarf","Completion":"T-TAURI; RED GIANT; WHITE DWARF"} {"Type":"TOSS-UP\n\nBIOLOGY Short Answer","Prompt":"What reduced coenzyme donates its electrons to complex-one in the electron transport system?","Completion":"NADH"} {"Type":"TOSS-UP\n\nCHEMISTRY Short Answer","Prompt":"What is the bond order for each oxygen-oxygen bond in the ozone or O\\({}{3}\\) molecule?","Completion":"\\(\\frac{3}{2}\\) (ACCEPT: 1.5)"} {"Type":"TOSS-UP\n\nPHYSICS Multiple Choice","Prompt":"Which of the following best describes why the optical spectrum of an atom exists as discrete lines and not as a continuous spectrum:\n\nW) atoms have electrons with unit charge\n\nX) all atoms when excited emit light\n\nY) all energy levels of atoms are quantized\n\nZ) all electrons have spin","Completion":"Y) ALL ENERGY LEVELS OF ATOMS ARE QUANTIZED"} {"Type":"TOSS-UP\n\n11. MATH Multiple Choice","Prompt":"At \\(x=-\\dfrac{1}{2}\\), the graph of \\(y=x^{3}\\) is:\n\nW) increasing and concave up\n\nX) increasing and concave down\n\nY) decreasing and concave up\n\nZ) decreasing and concave down","Completion":"X) INCREASING AND CONCAVE DOWN"} {"Type":"TOSS-UP\n\nGENERAL SCIENCE Multiple Choice","Prompt":"According to EPA nationwide emission trends for common air pollutants and excluding fires and dust, which of the following accounted for the least amount of emissions in the past 5 years:\n\nW) volatile organic compounds\n\nX) carbon monoxide\n\nY) sulfur dioxide\n\nZ) particulate matter","Completion":"Z) PARTICULATE MATTER"} {"Type":"TOSS-UP\n\nEARTH SCIENCE Short Answer","Prompt":"In what layer of Earth's atmosphere do most meteors experience the most heating?","Completion":"MESOSPHERE"} {"Type":"TOSS-UP\n\nASTRONOMY Multiple Choice","Prompt":"Which of the following is LEAST likely to occur when viewed from mid-northern latitudes in the month of August:\n\nW) Venus passing within 2\\({}^{\\rm o}\\) of Saturn X) Neptune at opposition Y) the Moon at apogee on the 1\\({}^{\\rm st}\\) of the month and perigee at the 29\\({}^{\\rm th}\\) of the month Z) the Moon passing within 5\\({}^{\\rm o}\\) of Uranus","Completion":"Y) THE MOON AT APOGEE ON THE 1\\({}^{\\rm ST}\\) OF THE MONTH AND PERIGEE AT THE 29\\({}^{\\rm TH}\\) OF THE MONTH"} {"Type":"BONUS\n\nBIOLOGY Multiple Choice","Prompt":"Which of the following is the RNA complementary sequence of the following sequence: 5'CCGCGA 3' (read as: 5-prime, CCGCGS, 3-prime)\n\nW) 5' UCGCGG 3' (read as: 5-prime, UCGCGG, 3-prime)\n\nX) 5' GGCGCU 3' Y) 5' GGCGCT 3' Z) 5' TCGCGG 3'","Completion":"W) 5' UCGCG 3'"} {"Type":"TOSS-UP\n\nCHEMISTRY Short Answer","Prompt":"For the following 3 processes, predict whether the change in enthalpy, or \\(\\Delta\\)S\\({}^{\\rm o}\\) (read as: delta S naught), is positive or negative, respectively:\n\ndecomposition of ammonium nitrate\n\nsublimation of dry ice\n\ncondensation of gaseous iodine to liquid iodine","Completion":"1) +; 2) +; 3) -"} {"Type":"BONUS\n\nCHEMISTRY Short Answer","Prompt":"Consider the following equilibrium reaction,\n\n\\({\\rm PCL{5(gas)}}\\leftrightarrow{\\rm PCL{3(gas)}}+{\\rm Cl{2(gas)}}\\). If a 1-liter flask contains 0.02 moles of \\({\\rm PCL{3}}\\) and 0.02 moles of \\({\\rm Cl{2}}\\) at equilibrium, how many moles of \\({\\rm PCL{5}}\\) are in the flask. Assume the equilibrium constant is \\(5\\times 10^{-2}\\).","Completion":"\\(8\\times 10^{-3}\\) (ACCEPT: 0.008)"} {"Type":"TOSS-UP\n\n17 PHYSICS Short Answer","Prompt":"During beta minus decay, what nucleon type is reduced by one?","Completion":"NEUTRON"} {"Type":"BONUS\n\n17 PHYSICS Multiple Choice","Prompt":"Which of the following is NOT true of the Hall effect:\n\nW: it can be used to measure the strength of a magnetic field\n\nX: it is a consequence of a current carrying wire in a magnetic field\n\nY: it was one of the first direct indications that negative charges are what move in conductors\n\nZ: it cannot distinguish between positive and negative particles","Completion":"Z) IT CANNOT DISTINGUISH BETWEEN POSITIVE AND NEGATIVE PARTICLES"} {"Type":"TOSS-UP\n\n18 GENERAL SCIENCE Short Answer","Prompt":"What are the Ice Cube observatory under construction at the South Pole and the ANTARES telescope off the coast of France primarily designed to detect?","Completion":"NEUTRINOS"} {"Type":"BONUS\n\n18 GENERAL SCIENCE Multiple Choice","Prompt":"During the winter solstice in the northern hemisphere, which of the following northern latitude positions will have a day length of 10.1 hours:\n\nW: 30\\({}^{\\circ}\\)\n\nX: 40\\({}^{\\circ}\\)\n\nY: 50\\({}^{\\circ}\\)\n\nZ: 60\\({}^{\\circ}\\)","Completion":"W) 30\\({}^{\\circ}\\)"} {"Type":"TOSS-UP\n\nEARTH SCIENCE Multiple Choice","Prompt":"During the past 500,000 years, there have been how many periods of major glaciation on Earth:\n\nW) 2\n\nX) 3\n\nY) 4\n\nZ) 5","Completion":"Z) 5"} {"Type":"TOSS-UP\n\nMATH Short Answer","Prompt":"Of the following 5 functions identify all that are NOT differentiable for all real numbers: \\(\\mid x\\mid\\) (read as: absolute value of \\(x\\)); cosine(\\(x\\)); tangent(\\(x\\)); \\(e^{x}\\); the greatest integer function","Completion":"\\(\\mid x\\mid\\); TAN(\\(x\\)); THE GREATEST INTEGER FUNCTION"} {"Type":"","Prompt":"TOSS-UP","Completion":"21)ASTRONOMY Short Answer What 3 elements are most prominent in the carbon fusion cycle? CARBON; NITROGEN; OXYGEN"} {"Type":"","Prompt":"BONUS","Completion":"21)ASTRONOMY Short Answer If the surface of a star is 15,000 kelvin, at what wavelength, in nanometers rounded to the nearest whole number, does it emit its maximum energy? 200"} {"Type":"","Prompt":"TOSS-UP","Completion":"22)BIOLOGY Short Answer What cell type is MOST directly responsible for secreting the bone matrix in developing humans? OSTEOBLASTS (DO NOT ACCEPT: OSTEOCYTE)"} {"Type":"","Prompt":"BONUS","Completion":"22)BIOLOGY Short Answer What specific human bone cell is antagonistic to osteoblasts and is known to cause apoptosis (read as: A-pop-TOE-sis) of osteoblasts in vitro? OSTEOCLASTS"} {"Type":"TOSS-UP\n\nBIOLOGY Short Answer","Prompt":"Intestinal absorption of what vitamin is most directly dependent on intrinsic factor?","Completion":"B\\({}{12}\\)"} {"Type":"TOSS-UP\n\nCHEMISTRY Multiple Choice","Prompt":"L-alanine and D-alanine are BEST classified as what type of isomers:\n\nW) cis\n\nX) trans\n\nY) structural\n\nZ) optical","Completion":"Z) OPTICAL"} {"Type":"BONUS\n\nCHEMISTRY Short Answer","Prompt":"How many sigma and pi bonds, respectively, are there in a molecule with the following formula: [read slowly] CH\\({}{3}\\)CHCHCH\\({}{2}\\)CH\\({}{3}\\)","Completion":"SIGMA 14; PI = 1"} {"Type":"TOSS-UP\n\nBIOLOGY Multiple Choice","Prompt":"Which of the following BEST explains how cyclic AMP can cause so many different intracellular responses in a mammalian cell:\n\nW) it is produced in large quantities\n\nX) it migrates to the DNA where it binds to a variety of histones\n\nY) it activates a wide variety of specific protein kinases\n\nZ) it prevents the activation of other hormonal signals","Completion":"Y) IT ACTIVATES A WIDE VARIETY OF SPECIFIC PROTEIN KINASES"} {"Type":"BONUS\n\nBIOLOGY Multiple Choice","Prompt":"Which of the following BEST describes the function of a spliceosome (read as: SPLY-see-oh-zome):\n\nW) splicing DNA together during replication\n\nX) splicing out mutations\n\nY) joining multiple copies of structural genes\n\nZ) removing introns","Completion":"Z) REMOVING INTRONS"} {"Type":"TOSS-UP\n\nPHYSICS Multiple Choice","Prompt":"Which of the following BEST explains how a radium isotope with a half-life of about 1,600 years can be found in the crust of the Earth that was formed millions of years ago:\n\nW) its decay is slowed by the pressure of the Earth\n\nX) the radium is replenished by decay of longer lived radioactive elements\n\nY) the radium was replenished by meteors and asteroids over the millennia\n\nZ) the radium is recycled from the Earth's core where magnetic fields slow the decay","Completion":"X) THE RADIUM IS REPLENISHED BY DECAY OF LONGEY LIVED RADIOACTIVE ELEMENTS"} {"Type":"TOSS-UP\n\nMATH Multiple Choice","Prompt":"Which of the following is TRUE about any two successive terms in the Fibonacci sequence:\n\nW) their product is a Fibonacci number\n\nX) they are either both odd or both even\n\nY) they are relatively prime\n\nZ) their quotient is the golden ratio","Completion":"Y) THEY ARE RELATIVELY PRIME"} {"Type":"","Prompt":"TOSS-UP","Completion":"5) EARTH SCIENCE Multiple Choice Most terrestrial plants grow best in soil with a pH range of: W) 4.5 to 5.5 X) 6.0 to 7.0 Y) 7.0 to 8.0 Z) 7.5 to 8.5 X) 6.0 TO 7.0"} {"Type":"","Prompt":"TOSS-UP","Completion":"6) GENERAL SCIENCE Multiple Choice Which of the following best illustrates a limitation of a supercapacitor, or electric double-layer capacitor, versus an electrochemical battery: W) virtually unlimited cycle life X) rapid recharging Y) low impedance Z) high self-discharge rate Z) HIGH SELF-DISCHARGE RATE"} {"Type":"TOSS-UP\n\nASTRONOMY Multiple Choice","Prompt":"Which of the following would astronomers most likely consider emission nebula:\n\nW) Population III (read as: regions\n\nX) Population II (read as: regions\n\nY) glowing H II (read as: H, regions\n\nZ) active regions","Completion":"Y) GLOWING H II REGIONS"} {"Type":"BONUS\n\nASTRONOMY Multiple Choice","Prompt":"In which of the following locations would there typically be more Population Two stars than Population One stars:\n\nW) in globular clusters\n\nX) in spiral arms of galaxies\n\nY) in planetary nebula\n\nZ) in regions actively forming stars","Completion":"W) IN GLOBULAR CLUSTERS"} {"Type":"TOSS-UP\n\nCHEMISTRY Multiple Choice","Prompt":"Which of the following are the units of the rate constant for a 2\\({}^{\\rm nd}\\) order reaction given that concentration is measured in molarity and time in seconds:\n\nW) M\\({}^{-2}\\)s\\({}^{-1}\\)\n\nX) M\\({}^{-1}\\)s\\({}^{-1}\\)\n\nY) Ms\\({}^{-1}\\)\n\nZ) M\\({}^{-2}\\)minutes\\({}^{-1}\\)","Completion":"X) M\\({}^{-1}\\)s\\({}^{-1}\\)"} {"Type":"BONUS\n\nCHEMISTRY Short Answer","Prompt":"1.0 gram of an unknown organic molecule is dissolved in 50.0 grams of benzene. Determine the molecular mass of the unknown if this solution freezes at 4.5\\({}^{\\rm o}\\)C. Assume the freezing point constant of benzene is 5.0\\({}^{\\rm o}\\)C per molal, and the freezing point of benzene is 5.5\\({}^{\\rm o}\\)C. Give your answer to the nearest whole number in grams per mole.","Completion":"100"} {"Type":"TOSS-UP\n\nBIOLOGY Multiple Choice","Prompt":"Which of the following is the target tissue of the hormone leptin where it binds with LepRB (read as: L, E, P, R, B) receptors:","Completion":"W) hypothalamus X) pancreas Y) adipose tissue Z) kidney W) HYPOTHALAMUS"} {"Type":"","Prompt":"TOSS-UP","Completion":"10) PHYSICS Short Answer Name all of the following 3 sub-atomic particles that interact with all 4 of the fundamental forces: electron; quarks; neutrino QUARKS"} {"Type":"TOSS-UP\n\nMATH Multiple Choice","Prompt":"Which of the following is the equation for the tangent line for \\(y=x^{4}\\) at the point \\((1,\\,\\):\n\nW) \\(y=4x-3\\)\n\nX) \\(y=4x-1\\)\n\nY) \\(y=4x+1\\)\n\nZ) \\(y=4x+3\\)","Completion":"W) \\(y=4x-3\\)"} {"Type":"BONUS\n\nMATH Short Answer","Prompt":"What are the coordinates of the hole in the graph of the function,\n\n\\(g(x)=\\dfrac{(2x^{2}-}{(x-}\\,\\gamma\\)","Completion":"\\((2,\\,8)\\)"} {"Type":"TOSS-UP\n\nEARTH SCIENCE Multiple Choice","Prompt":"According to the discontinuous side of Bowen's reaction series, which of the following is likely to first crystallize as basaltic magma cools:\n\nW) quartz\n\nX) olivine\n\nY) pyroxene\n\nZ) hornblende","Completion":"X) OLIVINE"} {"Type":"BONUS\n\nEARTH SCIENCE Short Answer","Prompt":"Of the 3 basic types of magma on Earth, which one generally has a chemical composition intermediate in iron, magnesium, and calcium?","Completion":"ANDESITIC (ACCEPT: ANDESITE)"} {"Type":"TOSS-UP\n\nGENERAL SCIENCE Multiple Choice","Prompt":"Which of the following age groups would have the most brown fat per kilogram of body weight:\n\nW) infant\n\nX) adolescent\n\nY) middle age\n\nZ) elderly","Completion":"W) INFANT"} {"Type":"TOSS-UP\n\nASTRONOMY Short Answer","Prompt":"What element makes up the innermost core of a type II (read as: type supernova just before core collapse?","Completion":"IRON"} {"Type":"TOSS-UP\n\nCHEMISTRY Multiple Choice","Prompt":"Consider the following equilibrium reaction, \\(\\mathrm{FeO{(solid)}+CO{(gas)}\\leftrightarrow\\ Fe{(solid)}+CO{2(gas)}}\\), where \\(\\Delta\\)H (read as: delta H) equals -10 kilojoules. Which of the following would have a similar effect on the equilibrium position as lowering the temperature:\n\n1. adding iron\n2. increasing pressure\n3. removing carbon dioxide\n4. removing carbon monoxide","Completion":"Y) REMOVING CARBON DIOXIDE"} {"Type":"TOSS-UP\n\nBIOLOGY Short Answer","Prompt":"From what specific biological molecule are Barbara McClintock's transposons (read as: trans-POE-zons) made?","Completion":"DNA"} {"Type":"BONUS\n\nBIOLOGY Short Answer","Prompt":"Name all of the following 4 organisms in which transposons (read as: trans-POE-zons) are found: humans; maize; fruit fly; bacteria","Completion":"ALL"} {"Type":"TOSS-UP\n\nEARTH SCIENCE Short Answer","Prompt":"In the southern hemisphere, Ekman transport generated by trade winds over the equatorial Pacific Ocean is how many degrees to the left of the prevailing winds?","Completion":"90"} {"Type":"BONUS\n\nEARTH SCIENCE Multiple Choice","Prompt":"Which of the following BEST describes an air mass that originates in northern Mexico, moves into the Great Plains and stagnates, resulting in severe drought:\n\nW) continental tropical\n\nX) continental polar\n\nY) maritime tropical\n\nZ) maritime polar","Completion":"W) CONTINENTAL Tropical"} {"Type":"TOSS-UP\n\nGENERAL SCIENCE Multiple Choice","Prompt":"Which of the following is NOT true regarding the burning of biodiesel versus gasoline in automobile engines:\n\nW) biodiesel combustion does not emit SO\\({}{2}\\)\n\nX) biodiesel contains more oxygen than gasoline\n\nY) the CO\\({}{2}\\) released when biodiesel is burned is the same CO\\({}{2}\\) the plants removed from the atmosphere as they grew\n\nZ) the energy content per gallon of biodiesel is approximately 28% lower than that of ethanol","Completion":"Z) THE ENERGY CONTENT PER GALLON OF BIODIESEL IS APPROXIMATELY 28% LOWER THAN THAT OF ETHANOL"} {"Type":"BONUS\n\nGENERAL SCIENCE Short Answer","Prompt":"A physician orders 0.1 gram of a drug be administered to a patient. If it is only available as 20 milligrams per 5 milliliters, how many milliliters needs to be administered?","Completion":"25"} {"Type":"TOSS-UP\n\nASTRONOMY Short Answer","Prompt":"From the LEAST to the MOST, what are the 3 most abundant elements in a G-type star?","Completion":"HYDROGEN; HELIUM; CARBON"} {"Type":"BONUS\n\nASTRONOMY Short Answer","Prompt":"If a supernova with an apparent magnitude of -2.0 was observed for the first time in the year 2,000 at 35.00 kiloparsecs from Earth, how many years before observation, rounded to the nearest whole number, did the supernova most likely occur?","Completion":"114"} {"Type":"TOSS-UP\n\nBIOLOGY Short Answer","Prompt":"What is the specific intermediate informational molecule for human retrotransposons (read as: rhet-tro-trans-POE-zons):","Completion":"RNA (ACCEPT: RIBONUCLEIC ACID or mRNA or MESSENGER RNA)"} {"Type":"BONUS\n\nBIOLOGY Short Answer","Prompt":"Order the following 4 processes from the one to typically occur the EARLIEST to the one to occur the LATEST in the production of collagen:\n\n(read slowly) spliceosome (read as: SPLY-see-oh-zome); transcription; translation; golgi modification","Completion":"TRANSCRIPTION; SPLICEOSOME; TRANSLATION; GOLGI MODIFICATION"} {"Type":"TOSS-UP\n\nCHEMISTRY Short Answer","Prompt":"What is the overall reaction order for the following rate law:\n\n\\(\\rm rate=k[NH{4}^{+}][NO{2}^{-}]\\) (read as: \\(k\\) times the concentration of \\(\\rm NH{4}^{+}\\) times the concentration of \\(\\rm NO{2}^{-}\\))","Completion":"2 (ACCEPT: \\(\\rm 2^{nd}\\) ORDER)"} {"Type":"TOSS-UP\n\nPHYSICS Short Answer","Prompt":"What boson is the only particle predicted by the Standard Model of Particle Physics that had not yet been observed as of June 2008?","Completion":"HIGGS BOSON (ACCEPT: HIGGS)"} {"Type":"TOSS-UP\n\nBIOLOGY Short Answer","Prompt":"From what amino acid is the vasodilator histamine primarily synthesized?","Completion":"HISTIDINE"} {"Type":"TOSS-UP\n\nCHEMISTRY Short Answer","Prompt":"What is the scientific term for the ratio between the density of a given substance to that of water when both are at the same temperature?","Completion":"SPECIFIC GRAVITY (ACCEPT: RELATIVE DENSITY)"} {"Type":"BONUS\n\nCHEMISTRY Multiple Choice","Prompt":"Which of the following pure substances has the highest melting point at 1 atmosphere of pressure:\n\nW) magnesium oxide\n\nX) diamond\n\nY) sodium chloride\n\nZ) cesium chloride","Completion":"X) DIAMOND"} {"Type":"TOSS-UP\n\nBIOLOGY Short Answer","Prompt":"What is the MOST common anatomical synonym for ventral, when locating a part of the human body in relation to another part?","Completion":"ANTERIOR"} {"Type":"BONUS\n\nBIOLOGY Short Answer","Prompt":"What are the names for the pyrimidine bases found in DNA?","Completion":"CYTOSINE; THYMINE"} {"Type":"TOSS-UP\n\nPHYSICS Short Answer","Prompt":"According to one of the most common color triangles, if blue and green are primary colors, what is the third primary color?","Completion":"RED"} {"Type":"TOSS-UP\n\nMATH Short Answer","Prompt":"What is the smaller of two integers whose sum is 19 and whose product is 48?","Completion":"3"} {"Type":"TOSS-UP\n\nEARTH SCIENCE Multiple Choice","Prompt":"Which of the following is best classified as a plutonic intrusive rock:","Completion":"W) obsidian X) granite Y) basalt Z) pumice X) GRANITE"} {"Type":"BONUS\n\nEARTH SCIENCE Multiple Choice","Prompt":"The principle constituent of most granites is:","Completion":"W) feldspar X) muscovite Y) calcite Z) dolomite W) FELDSPAR"} {"Type":"TOSS-UP\n\nGENERAL SCIENCE Multiple Choice","Prompt":"The related quantities charted on a line graph are most often called:","Completion":"W) results X) lines Y) sets Z) variables Z) VARIABLES"} {"Type":"","Prompt":"BONUS","Completion":"6) GENERAL SCIENCE Short Answer How many grams of sodium chloride is in 2,000 milliliters of 0.9% NaCl solution? 18"} {"Type":"TOSS-UP\n\nBIOLOGY Short Answer","Prompt":"From what chamber of the human heart does blood carried by the left pulmonary artery leave?","Completion":"RIGHT VENTRICLE"} {"Type":"TOSS-UP\n\nPHYSICS Short Answer","Prompt":"A white-colored object illuminated by a green light will appear as what color to the human eye?","Completion":"GREEN"} {"Type":"TOSS-UP\n\n11. MATH Multiple Choice","Prompt":"One-fifth of 0.04% is equal to:\n\nW) \\(8\\times 10^{-2}\\)\n\nX) \\(8\\times 10^{-3}\\)\n\nY) \\(8\\times 10^{-4}\\)\n\nZ) \\(8\\times 10^{-5}\\)","Completion":"Z) \\(8\\times 10^{-5}\\)"} {"Type":"BONUS\n\n11. MATH Short Answer","Prompt":"Find the prime factorization of 240:","Completion":"\\(2\\times 2\\times 2\\times 2\\times 3\\times 5\\) (ACCEPT: \\(2^{4}\\times 3\\times 5\\))"} {"Type":"TOSS-UP\n\n12. EARTH SCIENCE Multiple Choice","Prompt":"When spreading centers reach above the ocean's surface, they typically form which of the following types of volcanoes:\n\nW) shield\n\nX) composite\n\nY) stratovolcano\n\nZ) cinder cone","Completion":"W) SHIELD"} {"Type":"BONUS\n\n12. EARTH SCIENCE Multiple Choice","Prompt":"Mount Vesuvius in Italy is a typical:\n\nW) mud volcano\n\nX) caldera\n\nY) cinder cone\n\nZ) composite volcano","Completion":"Z) COMPOSITE VOLCANO"} {"Type":"TOSS-UP\n\nGENERAL SCIENCE Short Answer","Prompt":"The fertilizer potash provides plants with what element necessary for healthy growth and which is also the seventh most abundant element on Earth?","Completion":"POTASSIUM (ACCEPT: K)"} {"Type":"BONUS\n\nGENERAL SCIENCE Short Answer","Prompt":"From what protein is gelatin, which is used as a food additive, most commonly derived?","Completion":"COLAGEN"} {"Type":"TOSS-UP\n\nASTRONOMY Short Answer","Prompt":"First proposed by Georges Lemaitre, what is the name, most likely coined by Fred Hoyle, for the theory that the universe originated at a finite time many eons ago from an extremely compressed hot state?","Completion":"BIG BANG"} {"Type":"BONUS\n\nASTRONOMY Short Answer","Prompt":"In what constellation is the star Polaris found?","Completion":"URSA MINOR (DO NOT ACCEPT: LITTLE DIPPER)"} {"Type":"TOSS-UP\n\nCHEMISTRY Short Answer","Prompt":"What is the pH of a solution with a pOH of 6.8 at 25\\({}^{\\rm o}\\)C?","Completion":"7.2"} {"Type":"BONUS\n\nCHEMISTRY Multiple Choice","Prompt":"Which of the following substances is water soluble:\n\nW) ethanol\n\nX) dichloro-methane\n\nY) chloroform\n\nZ) benzene","Completion":"W) ETHANOL"} {"Type":"TOSS-UP\n\nBIOLOGY Short Answer","Prompt":"Yersinia, Drosophila and Homo are all what taxonomical category?","Completion":"GENUS (ACCEPT: GENERA)"} {"Type":"BONUS\n\nBIOLOGY Short Answer","Prompt":"From what monosaccharide is cellulose primarily composed?","Completion":"GLUCOSE (ACCEPT: B-D-GLUCOPYRANOSYL or D-GLUCOSE)17) PHYSICS Short Answer Although in many respects difficult to define, what term is MOST often described as a push or a pull that can cause an object to accelerate?"} {"Type":"TOSS-UP\n\nEARTH SCIENCE Short Answer","Prompt":"What is the oldest Eon on the Geological Time Scale within which the first organisms appeared between 550 million and 3.4 billion years ago?","Completion":"PRECAMBRIAN"} {"Type":"TOSS-UP\n\nGENERAL SCIENCE Multiple Choice","Prompt":"Anthropogenic means:\n\nW) generated by humans\n\nX) created many years ago\n\nY) carried in primate genes\n\nZ) harmful to elderly people","Completion":"W) GENERATED BY HUMANS"} {"Type":"TOSS-UP\n\nASTRONOMY Short Answer","Prompt":"What is the primary elemental product of stellar hydrogen fusion?","Completion":"HELIUM"} {"Type":"TOSS-UP\n\nCHEMISTRY Short Answer","Prompt":"What is the atomic number for the ground state element with the following electron configuration: 1s\\({}^{2}\\)2s\\({}^{2}\\)2p\\({}^{5}\\) (read as: 1, s, 2; 2, s, 2; 2, p,","Completion":"9"} {"Type":"TOSS-UP\n\nMATH Short Answer","Prompt":"Giving your answer in standard form, what is the last term in the binomial expansion of the following: \\(\\left(x+y\\right)^{6}\\)","Completion":"\\(y^{6}\\)"} {"Type":"TOSS-UP\n\nPHYSICS Short Answer","Prompt":"What specific law of motion most directly implies that every action has an equal and opposite reaction?","Completion":"NEWTON'S THIRD LAW (ACCEPT: THIRD LAW)"} {"Type":"TOSS-UP\n\nBIOLOGY Short Answer","Prompt":"The sinoatrial (read as: sine-oh-AYE-tree-al) node is located in what human organ?","Completion":"HEART"} {"Type":"BONUS\n\n1. MATH Short Answer","Prompt":"How many zeroes are at the end of 25! [25 factorial]?","Completion":"6"} {"Type":"BONUS\n\n2. EARTH AND SPACE Multiple Choice","Prompt":"A new study has found that white spruce trees on the edge of the tundra in Alaska's far north have thrived in the past 100 years. Which of the following methodologies was most likely used to determine the age of the trees?\n\nW) Sclerochronology\n\nX) Dendrochronology\n\nY) Radiometric dating\n\nZ) Acanthochronology","Completion":"X) DENDROCHRONOLOGY"} {"Type":"BONUS\n\n7. MATH Short Answer","Prompt":"Find the coordinates of the center and the radius of the circle:\n\n\\(x^{2}+y^{2}\\) - \\(8x+6y\\) - \\(11=0\\)","Completion":"CENTER = (4, -3); RADIUS = 6"} {"Type":"BONUS\n\n8. EARTH AND SPACE Short Answer","Prompt":"Based on Bowen's reaction series, which feldspar forms last as a magma cools?","Completion":"ORTHOCLASE (ACCEPT: POTASSUM FELDSPAR, K-SPAR)"} {"Type":"BONUS\n\n9. BIOLOGY Multiple Choice","Prompt":"Which of the following mechanisms would best explain how bisphenol A causes above average production of yellow agouti [uh-GOO-tee] mice?\n\nW) Decreased methylation [meth-uh-LAY-shuhn] of the agouti [uh-GOO-tee] gene\n\nX) Increased methylation of the agouti gene\n\nY) Decreased phosphorylation [FOS-fer-uh-lay-shuhn] of the agouti gene\n\nZ) Increased phosphorylation of the agouti gene","Completion":"W) DECREASED METHYLATION OF THE AGOUTI GENE"} {"Type":"BONUS\n\n10. PHYSICS Multiple Choice","Prompt":"An electron is accelerated through a potential difference of 5.0 kilovolts. Which of the following is the kinetic energy in joules gained in the process?\n\nW) 8.0 x 10\\({}^{22}\\)\n\nX) 8.0 x 10\\({}^{-16}\\)\n\nY) 1.6 x 10\\({}^{-16}\\)\n\nZ) 1.6 x 10\\({}^{-22}\\)","Completion":"X) 8.0 x 10\\({}^{-16}\\)"} {"Type":"BONUS\n\n11. CHEMISTRY Multiple Choice","Prompt":"Assuming complete dissociation and the absence of ion pairing, which of the following 1.00 molar aqueous solutions would have the lowest freezing point?\n\nW) Sucrose\n\nX) Sodium chloride\n\nY) Calcium hydroxide\n\nZ) Potassium phosphate","Completion":"Z) POTASSIUM PHOSPHATE"} {"Type":"BONUS\n\n12. BIOLOGY Short Answer","Prompt":"During DNA replication, what would be the complement of the template strand: 5[5 prime]-TAT CAG CCC-3' [3 prime], including strand polarity in your answer.","Completion":"3'- ATA GTC GGG-5' (ACCEPT: 5'-GGG CTG ATA-3')"} {"Type":"BONUS\n\n15. BIOLOGY Multiple Choice","Prompt":"Which of the following organisms is triploid?\n\nW) Aphid\n\nX) Escherichia coli [esh-uh-RIK-ee-uh]\n\nY) Seedless watermelon\n\nZ) Wheat","Completion":"Y) SEEDLESS WATERMELON"} {"Type":"BONUS\n\n16. PHYSICS Multiple Choice","Prompt":"Some scientists have predicted the melting of polar ice caps due to climate change. Which of the following changes would be caused by the melting?\n\nW) The moment of inertia of the Earth would increase\n\nX) The moment of inertia of the Earth would decrease\n\nY) The angular momentum associated with the Earth's rotation would increase\n\nZ) The angular momentum associated with the Earth's rotation would decrease","Completion":"W) THE MOMENT OF INERTIA OF THE EARTH WOULD INCREASE"} {"Type":"BONUS\n\n17. ENERGY Multiple Choice","Prompt":"Modern smart grid technology predominantly improves which of the following components of the electricity system?\n\nW) Transmission and distribution\n\nX) Fuel extraction\n\nY) Renewable energy generation\n\nZ) Fossil fuel energy generation","Completion":"W) TRANSMISSION AND DISTRIBUTION"} {"Type":"BONUS\n\n18. CHEMISTRY Short Answer","Prompt":"Providing your answers to the nearest integer, convert the following four pressures to atmospheres: 203 kilopascals, 2660 Torr, 2.2 bar, and 3700 millimeters of mercury.","Completion":"1) 2, 2) 3, 3) 2, 4) 5"} {"Type":"BONUS\n\n21. BIOLOGY Multiple Choice","Prompt":"Which of the following foramina of the human skull houses the spinal cord?\n\nW) Foramen ovale [for-A-men oh-VAL-ay]\n\nX) Foramen incisivum\n\nY) Foramen magnum\n\nZ) Foramen spinosum","Completion":"Y) FORAMEN MAGNUM"} {"Type":"BONUS\n\n22. PHYSICS Multiple Choice","Prompt":"In a photoelectric experiment, if the intensity of the incident light beam is unchanged while the frequency of the light beam is increased, which of the following is true?\n\nW) The number of photoelectrons emitted per second increases\n\nX) The maximum kinetic energy of the photoelectrons emitted increases\n\nY) The maximum kinetic energy of the photoelectrons emitted decreases\n\nZ) The maximum kinetic energy of the photoelectrons emitted remains unchanged","Completion":"X) THE MAXIMUM KINETIC ENERGY OF THE PHOTOELECTRONS EMITTED INCREASES"} {"Type":"BONUS\n\n25. CHEMISTRY Short Answer","Prompt":"Rank the following four substances according to increasing boiling point: helium, water, ammonia, carbon monoxide.","Completion":"1, 4, 3, 2 (ACCEPT: HELIUM, CARBON MONOXIDE, AMMONIA, WATER)"} {"Type":"BONUS\n\n3. CHEMISTRY Multiple Choice","Prompt":"Which of the following is the energy in joules of a photon with a frequency of 1.00 x 10\\({}^{6}\\) gigahertz?\n\nW) 6.63 x 10-19\n\nX) 6.63 x 10-28\n\nY) 6.63 x 10-34\n\nZ) 6.63 x 10-49","Completion":"W) 6.63 X 10-19"} {"Type":"BONUS\n\n4. MATH Short Answer","Prompt":"If six geese in a gaggle produce 66 eggs in 6,666 days, what is the average number of days it takes a single goose to lay an egg?","Completion":"606"} {"Type":"BONUS\n\n5. EARTH AND SPACE Multiple Choice","Prompt":"Which of the following are three general categories of map projections?\n\nW) Cylindrical, conic, azimuthal [az-uh-MUTH-uh]\n\nX) Conformal, spherical, transverse\n\nY) Equidistant, conic, stereographic\n\nZ) Cylindrical, oblique, grid","Completion":"W) CYLINDRICAL, CONIC, AZIMUTHAL"} {"Type":"BONUS\n\n6. BIOLOGY Short Answer","Prompt":"Name all of the following four bones that are part of the human\n\nfoot: stylohyoid [sty-loh-HY-oid], calcaneus [kal-KAY-nee-uhs], metacarpals, femur.","Completion":"2"} {"Type":"BONUS\n\n7. PHYSICS Short Answer","Prompt":"A 200 microfarad parallel plate capacitor is connected to a 6 volt battery. Once the capacitor is fully charged, it is disconnected from the battery and a dielectric is inserted between the plates with a dielectric constant of 5. Expressed as a decimal, what is the new voltage across the capacitor?","Completion":"1.2"} {"Type":"BONUS\n\n8. CHEMISTRY Short Answer","Prompt":"How many milliliters of 0.1 molar sulfuric acid must be titrated with 20 milliliters of 0.5 molar sodium hydroxide in order reach the equivalence point?","Completion":"50"} {"Type":"BONUS\n\n11. BIOLOGY Multiple Choice","Prompt":"Which of the following organisms is NOT matched to its anatomical structure for excreting salt?\n\nW) Fish and gill\n\nX) Shark and rectal gland\n\nY) Whale and ocular salt gland\n\nZ) Seabird and nostril salt gland","Completion":"Y) WHALE AND OCULAR SALT GLAND"} {"Type":"BONUS\n\n12. CHEMISTRY Short Answer","Prompt":"Consider the complete combustion of propane in oxygen. Providing your answer to two significant figures, if 32 grams of oxygen are consumed in the reaction, how many grams of carbon dioxide are produced?","Completion":"26"} {"Type":"BONUS\n\n15. CHEMISTRY Multiple Choice","Prompt":"Which of the following compounds exhibits hydrogen bonding and can act as a Lewis base?\n\nW) Ozone\n\nX) Ammonia\n\nY) Cesium fluoride\n\nZ) Hydrogen fluoride","Completion":"X) AMMONIA"} {"Type":"BONUS\n\n16. MATH Short Answer","Prompt":"Divide the polynomial \\(x^{3}-x-6\\) by \\(x-2\\).","Completion":"\\(x^{2}+2x+3\\)"} {"Type":"BONUS\n\n17. EARTH AND SPACE Short Answer","Prompt":"What is the name of the metamorphic rock that is dominantly composed of hornblende?","Completion":"AMPHIBOLITE"} {"Type":"BONUS\n\n18. BIOLOGY Multiple Choice","Prompt":"Which of the following groups consists only of strictly vegetarian marine mammals?\n\nW) Sirenians [sy-REE-nee-uhns]\n\nX) Pinnipeds [PIN-uh-peds]\n\nY) Cetaceans [si-TAY-shuhns]\n\nZ) Fissipeds [FIS-uh-peds]","Completion":"W) SIRENIANS"} {"Type":"BONUS\n\n25. BIOLOGY Multiple Choice","Prompt":"Which of the following types of lipid modification connects proteins to cytosolic membrane surfaces and is reversible?\n\nW) Palmitoylation [pal-mi-toi-LAY-shuhn]\n\nX) Prenylation [pre-NIL-ay-shuhn]\n\nY) Myristoylation [may-ruh-stoi-LAY-shuhn]\n\nZ) Acylation [as-uh-LAY-shuhn]","Completion":"W) PALMITOYLATION"} {"Type":"BONUS\n\n1. BIOLOGY Short Answer","Prompt":"Arrange the following four choices from proximal to distal of the trachea [truth-KE-uh]: bronchiole [BRONG-kee-ohl], alveolar [al-VEE-uh-ler] sac, primary bronchus, alveolar duct.","Completion":"3, 1, 4, 2 (ACCEPT: PRIMARY BRONCHUS, BRONCHIOLE, ALVEOLAR DUCT, ALVEOLAR SAC)"} {"Type":"BONUS\n\n2. CHEMISTRY Short Answer","Prompt":"Giving your answers to the nearest integers, identify the pressure in atmospheres of a gas that starts at 1 atmosphere and 23\\({}^{\\circ}\\) Celsius at each of the following three temperatures: 52\\({}^{\\rm o}\\), 317\\({}^{\\rm o}\\), 2700\\({}^{\\rm o}\\).","Completion":"1) 1, 2) 2, 3) 10"} {"Type":"BONUS\n\n4. PHYSICS Short Answer","Prompt":"A 2 kilogram box is pushed forward with a 10 newton force while it experiences a 5 newton force of kinetic friction in the opposite direction. Providing your answer to the nearest meter, after 3 seconds, how far has the box moved?","Completion":"11"} {"Type":"BONUS\n\n5. EARTH AND SPACE Multiple Choice","Prompt":"Which of the following volcanoes erupted the most sulfuric acid into the atmosphere?\n\nW) Mount St. Helens\n\nX) Pinatubo\n\nY) Tambora\n\nZ) Toba","Completion":"Z) TOBA"} {"Type":"BONUS\n\n6. ENERGY Short Answer","Prompt":"If a million people leave their cell phone chargers plugged in unnecessarily for 16 hours per day and each charger consumes 1 watt when not in use, providing your answer to the nearest million, how many kilowatt-hours are consumed per year while the chargers are plugged in but not charging?","Completion":"6,000,000"} {"Type":"BONUS\n\n7. BIOLOGY Multiple Choice","Prompt":"Long bristles are found around the bills of insectivorous [in-sex-TV-er-uhsl] birds. How might these bristles help birds when they catch prey?\n\nW) Bristles help birds snap up insects by improving the bird's aerial balance\n\nX) Bristles protect the bird's nares [NAIR-eez] from an insect's legs\n\nY) Bristles function only for attracting a mate, not for catching prey\n\nZ) Bristles act as nets, catching insects that the bird misses with its bill","Completion":"Z) BRISTLES ACT AS NETS, CATCHING INSECTS THAT THE BIRD MISSES WITH ITS BILL"} {"Type":"BONUS\n\n8. CHEMISTRY Short Answer","Prompt":"Assuming that the transition of an electron in the hydrogen atom from the fourth to the second electron energy level is two electron volts, providing your answer to the nearest tenth, how many electronvolts is the transition between the first and third energy level?","Completion":"9.5 (ACCEPT: 9.6)"} {"Type":"BONUS\n\n11. EARTH AND SPACE Multiple Choice","Prompt":"Which of the following mountain ranges exhibits basin and range topography?\n\nW) Cascade Mountains\n\nX) Rocky Mountains\n\nY) Appalachian Mountains\n\nZ) Wasatch Mountains","Completion":"Z) WASTCH MOUNTAINS"} {"Type":"BONUS\n\n12. BIOLOGY Multiple Choice","Prompt":"Different species of hummingbirds have bills that vary in length and shape, matching the structure of the flowers on which they feed. Which of the following evolutionary scenarios best explains this diversity of hummingbird bill morphology?\n\nW) Coevolution of flowers and hummingbirds, leading to specialization of hummingbirds on particular flowers\n\nX) Random mutation of plants to develop varying flower shapes, and subsequent evolution of hummingbird bills to match these shapes\n\nY) Evolutionary race between flowers and hummingbirds, in which flowers become more extreme in shape to prevent nectar feeding\n\nZ) Evolution of various bill shapes, since all hummingbirds opportunistically feed on many different species of flower, regardless of shape","Completion":"W) COEVOLUTION OF FLOWERS AND HUMINGBIRDS, LEADING TO SPECIALIZATION OF HUMINGBIRDS ON PARTICULAR FLOWERS"} {"Type":"BONUS\n\n17. ENERGY Short Answer","Prompt":"The half life of radium is 1600 years. What fraction of the original quantity of radium has decayed after 9600 years?","Completion":"63\/64"} {"Type":"BONUS\n\n18. BIOLOGY Multiple Choice","Prompt":"Compared to the Antarctic region, the Arctic region has which of the following?\n\nW) Fewer endemic species\n\nX) Fewer terrestrial mammal species\n\nY) Less seasonality in pack ice\n\nZ) More flightless birds","Completion":"Y) LESS SEASONALITY IN PACK ICE"} {"Type":"BONUS\n\n21. PHYSICS Short Answer","Prompt":"A wave pulse travels along a string with a speed of 20 meters per second. If the tension in the string is 100 newtons, providing your answer as a decimal, what is the linear density of the string in kilograms per meter?","Completion":"0.25"} {"Type":"BONUS\n\n22. EARTH AND SPACE Short Answer","Prompt":"Imagine that you've just spent a long, hot summer day at the beach. The water was cool and refreshing after being out in the hot sun. As the Sun sets and night falls, you start to get chilled by the drop in air temperature. You decide to get back into the water, and you notice that its temperature doesn't seem to have changed much. In fact, it is warmer than the air. Which property of water best explains your observation?","Completion":"SPECIFIC HEAT"} {"Type":"BONUS\n\n23. ENERGY Multiple Choice","Prompt":"Which of the following has the most kinetic energy?\n\nW) A 1 kilogram mass moving at 10 meters per second\n\nX) A 2 kilogram mass moving at 7 meters per second\n\nY) A 0.5 kilogram mass moving at 14 meters per second\n\nZ) A 0.25 kilogram mass moving at 18 meters per second","Completion":"W) A 1 KILOGRAM MASS MOVING AT 10 METERS PER SECOND"} {"Type":"BONUS\n\n24. BIOLOGY Multiple Choice","Prompt":"Which of the following groups consists only of organisms incapable of photosynthesizing without the aid of symbiotic organisms?\n\nW) Dinoflagellates [dim-uh-FLAJ-uh-layts]\n\nX) Radiolarians [ray-dee-oh-LAIR-ee-uhns]\n\nY) Diatoms\n\nZ) Bacteria","Completion":"X) RADIOLARIANS"} {"Type":"BONUS\n\n1. CHEMISTRY Short Answer","Prompt":"The pressure of a mixture of three gases in a fixed volume container is 300 kilopascals. If the number of nitrogen molecules is three times the number of hydrogen molecules and the number of oxygen molecules is two times the number of hydrogen molecules, what are the partial pressures of the three gases?","Completion":"N\\({}{2}\\) = 150; H\\({}{2}\\) = 50; O\\({}{2}\\) = 100"} {"Type":"BONUS\n\n2. BIOLOGY Multiple Choice","Prompt":"How might fertilizer runoff from a farm affect a nearby lake?\n\nW) The lake will become depleted of oxygen\n\nX) Secchi [SEK-ee] depth will increase\n\nY) Algae will become less abundant\n\nZ) Organic matter will decrease","Completion":"W) THE LAKE WILL BECOME DEPLETED OF OXYGEN"} {"Type":"BONUS\n\n3. ENERGY Multiple Choice","Prompt":"Which of the following are the two primary products of combustion of methane gas in increasing order of molar amount?\n\nW) Water vapor and carbon dioxide\n\nX) Carbon monoxide and water vapor\n\nY) Carbon dioxide and water vapor\n\nZ) Carbon monoxide and carbon dioxide","Completion":"Y) CARBON DIOXIDE AND WATER VAPOR"} {"Type":"BONUS\n\n4. EARTH AND SPACE Short Answer","Prompt":"If a Kuiper [KY-per] belt object has an orbit with a semi-major axis of 64 AU, what is its orbital period in years?","Completion":"512"} {"Type":"BONUS\n\n9. ENERGY Multiple Choice","Prompt":"Which of the following elements has the greatest number of stable isotopes?\n\nW) Lead\n\nX) Plutonium\n\nY) Uranium\n\nZ) Hydrogen","Completion":"W) LEAD"} {"Type":"BONUS\n\n10. EARTH AND SPACE Short Answer","Prompt":"When a strong convective cell forms and there is a sufficient amount of supercooled water and ice crystals, what phenomenon can occur?","Completion":"HAILSTORM (ACCEPT: HAIL)"} {"Type":"BONUS\n\n11. PHYSICS Short Answer","Prompt":"A wheel spinning at a rate of 20 revolutions per minute slows steadily to a stop in a time of 30 seconds. If the wheel has a radius of 0.5 meters, through what distance to the nearest meter does a point on the rim travel during this time?","Completion":"16"} {"Type":"BONUS\n\n12. MATH Short Answer","Prompt":"Evaluate arctan (tan \\(3\\pi\/4\\)) [arctangent of the tangent of the quantity 3 pi over 4].","Completion":"-\\(\\pi\/4\\)"} {"Type":"BONUS\n\n13. CHEMISTRY Multiple Choice","Prompt":"When an aqueous solution of copper (II) bromide is electrolyzed, which of the following colors will occur at the anode?\n\nW) Red brown\n\nX) Greenish red\n\nY) Dark blue\n\nZ) No color change","Completion":"W) RED BROWN"} {"Type":"BONUS\n\n14. BIOLOGY Multiple Choice","Prompt":"Which of the following is NOT a specific adaptation to reduce desiccation in intertidal organisms?\n\nW) Presence of a holdfast\n\nX) Covering with shell fragments\n\nY) Living in a dense group\n\nZ) Highly ridged shell coverings","Completion":"W) PRESENCE OF A HOLDFAST"} {"Type":"BONUS\n\n15. EARTH AND SPACE Multiple Choice","Prompt":"Which of the following was the moment magnitude of the 2010 Chilean earthquake that was caused by the subduction of the Nazca Plate underneath the South American Plate?\n\nW) 6.8\n\nX) 7.8\n\nY) 8.8\n\nZ) 9.8","Completion":"Y) 8.8"} {"Type":"BONUS\n\n16. PHYSICS Short Answer","Prompt":"Vectors F and R are given by, respectively, 2 i minus 3 j and i plus 4 j. What is the cross product R X F [R cross F]?","Completion":"-11k17. MATH Short Answer If \\(\\int{0}^{5}f(x)dx=10\\) [the integral from 0 to 5 of f of x dx equals 10] and \\(\\int{0}^{2}f(x)dx=12\\)[the integral from 0 to 2 of f of x dx equals 12], then what is the value of \\(\\int{2}^{5}f(x)dx\\)[the integral from 2 to 5 of f of x dx]?"} {"Type":"BONUS\n\n18. CHEMISTRY Multiple Choice","Prompt":"At a constant temperature, a syringe containing 20.0 milliliters of air at 1 atmosphere of pressure is pulled out so that the volume inside the syringe is increased to 40.0 milliliters. The total pressure of the gases inside the syringe decreases to approximately 0.5 atmospheres. Which of the following best explains this observation?\n\nW) Gay-Lussac's Law\n\nX) Charles's Law\n\nY) Boyle's Law\n\nZ) Avogadro's Law","Completion":"Y) BOYLE'S LAW"} {"Type":"BONUS\n\n19. BIOLOGY Multiple Choice","Prompt":"Which of the following is NOT an advantage for the coral in the coral-zooxanthellae [zoh-uh-zan-THEL-ee] symbiosis?\n\nW) Increased growth\n\nX) Increasing calcification\n\nY) Protection from predators\n\nZ) Relatively constant supply of reduced carbon","Completion":"Y) PROTECTION FROM PREDATORS"} {"Type":"BONUS\n\n20. ENERGY Multiple Choice","Prompt":"Elements from which of the following sections of the periodic table have the highest ionization energies?\n\nW) Periods 2 and 3 of Group 1\n\nX) Periods 2 and 3 of Group 17\n\nY) Periods 5 and 6 of Group 17\n\nZ) Periods 6 and 7 of Group 1","Completion":"X) PERIODS 2 AND 3 OF GROUP 17"} {"Type":"BONUS\n\n3. BIOLOGY Multiple Choice","Prompt":"Which of the following would be the most direct product of meiosis [my-OH-sis] in a typical plant?\n\nW) Gametophyte [guh-MEE-tuh-fyt]\n\nX) Spore\n\nY) Egg\n\nZ) Sporophyte","Completion":"X) SPORE"} {"Type":"BONUS\n\n4. PHYSICS Short Answer","Prompt":"If it takes 12 seconds total for a ball to travel straight up from the ground and fall back to earth, neglecting air resistance and given g as 10 meters per second squared, what was the maximum height it reached in meters?","Completion":"180"} {"Type":"BONUS\n\n11. CHEMISTRY Multiple Choice","Prompt":"Which of the following statements about hydroboration-oxidation is false?\n\nW) It is a syn addition\n\nX) The borane acts as the electrophile\n\nY) Overall, the reaction converts an alkene to an alcohol\n\nZ) The common oxidizing agent used is sodium borohydride","Completion":"Z) THE COMMON OXIDIZING AGENT USED IS SODIUM BOROHYDRIDE"} {"Type":"BONUS\n\n12. BIOLOGY Multiple Choice","Prompt":"Which of the following populations would be most likely to exhibit logarithmic population growth?\n\nW) Elephants protected from hunting in a national park\n\nX) Sparrows that breed on a small island in northern Canada, with alternating mild and very harsh winters\n\nY) Yeast growing under controlled laboratory conditions\n\nZ) Deer living in Yellowstone National Park with wolf predators present","Completion":"Y) YEAST GROWING IN CONTROLLED LABORATORY CONDITIONS"} {"Type":"BONUS\n\n13. MATH Multiple Choice","Prompt":"For a recursive sequence \\(a{n}\\) defined as follows: \\(a{1}=7,a{2}=3\\), and \\(a{n}=a{n-1}-a{n-2}\\) for \\(n\\geq 3\\), find \\(a{1205}\\)[a sub 1205].\n\nW) -4\n\nX) -3\n\nY) 3\n\nZ) 4","Completion":"X) -3"} {"Type":"BONUS\n\n14. EARTH AND SPACE Short Answer","Prompt":"Of the following four factors, by number, identify those that will increase the rate of light attenuation in a natural water body: decreased particulate turbidity, increased concentrations of plankton, decreased chromophoric compounds, increased dissolved organic carbon.","Completion":"2, 4"} {"Type":"BONUS\n\n15. BIOLOGY Multiple Choice","Prompt":"A new species of fruit fly has been discovered that has wings of two colors, brown or white. Brown-winged females are allowed to mate with white-winged males. In the F1 generation, half of the females are brown and half of the males are brown. Which of the following is the best explanation for these results?\n\nW) White wing color is a recessive X-linked trait\n\nX) Brown wing color is a recessive X-linked trait\n\nY) White wing color is a recessive autosomal trait\n\nZ) White wing color is a lethal mutation","Completion":"W) WHITE WING COLOR IS A RECRESSIVE X-LINKED TRAIT"} {"Type":"BONUS\n\n16. PHYSICS Short Answer","Prompt":"What is the kinetic energy in joules to two significant figures of a car with a mass of 2000 kilograms moving with a speed 100 kilometers per hour?","Completion":"770,000"} {"Type":"BONUS\n\n17. ENERGY Multiple Choice","Prompt":"Approximately how many horsepower are generated by a horse pulling with a force of 180 pounds and turning a 24 foot diameter wheel 72 times in one hour?\n\nW) 1\/2\n\nX) 3\/4\n\nY) 1\n\nZ) 5\/4","Completion":"W) 1\/2"} {"Type":"BONUS\n\n18. CHEMISTRY Multiple Choice","Prompt":"Which of the following is a characteristic of an adiabatic gas expansion?\n\nW) Internal energy increases\n\nX) Temperature of the working gas decreases\n\nY) Kinetic energy increases with average molecular speed\n\nZ) Work done is inversely proportional to temperature difference between initial and final states","Completion":"X) TEMPERATURE OF THE WORKING GAS DECREASES"} {"Type":"BONUS\n\n23. ENERGY Short Answer","Prompt":"Rank the four following forms of coal in order of energy per pound from least to greatest: bituminous [by-TOO-muh-nuhs], anthracite [AN-thruh-syt], lignite, sub-bituminous.","Completion":"3, 4, 1, 2 (ACCEPT: LIGNITE, SUB-BITUMINOUS, BITUMINOUS, ANTHRACITE)"} {"Type":"BONUS\n\n24. CHEMISTRY Multiple Choice","Prompt":"An element's most stable ion forms an ionic compound with iodine having the formula, XI\\({}{2}\\). If the ion of element X has a mass of 89 amu and has 36 electrons, which of the following correctly matches the identity of the element and how many neutrons it has?\n\nW) Strontium and 51\n\nX) Krypton and 55\n\nY) Selenium and 55\n\nZ) Krypton and 53","Completion":"W) STRONTIUM AND 51"} {"Type":"TOSS-UP\n\nCHEMISTRY Multiple Choice","Prompt":"A molecule having a chemical formula of CH\\({}{3}\\)CCCH\\({}{3}\\) would be classified by which of the following?\n\nW) Alkyne\n\nX) Alkene\n\nY) Alkane\n\nZ) Aldehyde (read as: AHL-deh-hyd)","Completion":"W) ALKYNE"} {"Type":"BONUS\n\nCHEMISTRY Multiple Choice","Prompt":"The rare earth elements are primarily composed of elements from which groups of the periodic table?\n\nW) Transition metals and main group\n\nX) Alkali and alkaline\n\nY) Lanthanides (read as: LAHN-thah-nyds) and actinides\n\nZ) Alkaline and transition metals","Completion":"Y) Lanthanides and Actinides"} {"Type":"TOSS-UP\n\nEARTH AND SPACE Short Answer","Prompt":"What substance primarily determines the viscosity of magma?","Completion":"SILICA"} {"Type":"BONUS\n\nEARTH AND SPACE Multiple Choice","Prompt":"Which of the following is NOT a property of pyroclastic (read as: py-r0e-KLAHS-tik) flows?\n\nW) They can flow uphill\n\nX) They move slowly\n\nY) Hot rock and ash flow like a turbulent blanket over the ground\n\nZ) They glow red at night but appear grey in the daytime","Completion":"X) THEY MOVE SLOWLY"} {"Type":"TOSS-UP\n\nBIOLOGY Multiple Choice","Prompt":"During meiosis, haploid cells are created from diploid parent cells. In animals, this process only takes place in:\n\nW) Gametes\n\nX) Reproductive organs\n\nY) Somatic cells\n\nZ) Plasma cells","Completion":"X) REPRODUCTIVE ORGANS"} {"Type":"BONUS\n\nBIOLOGY Short Answer","Prompt":"Certain species of fungi are self-fertile, in which a sexual spore can result from the fusion of nuclei that are genetically distinct but derived from the same body.\n\nWhat do we call these kinds of organisms?","Completion":"HOMOTHALIC"} {"Type":"TOSS-UP\n\nPHYSICS Short Answer","Prompt":"What is the ideal mechanical advantage of a 24 inch lever if the fulcrum is located 6 inches from the resistance, which is at one end of the lever?","Completion":"3"} {"Type":"BONUS\n\nPHYSICS Short Answer","Prompt":"What is the mechanical advantage of a screw with a radius of 1 centimeter and a thread of 0.25 centimeters? Provide your answer to the nearest integer.","Completion":"25"} {"Type":"TOSS-UP\n\nBIOLOGY Multiple Choice","Prompt":"Which of the following organisms is NOT an autotroph?\n\nW) Pigmented sulfur bacteria\n\nX) Palm tree\n\nY) Nitrifying bacteria\n\nZ) Mosquito","Completion":"Z) MOSQUITO"} {"Type":"TOSS-UP\n\nPHYSICS Multiple Choice","Prompt":"Which one of the following statements concerning permanent bar magnets is NOT true?\n\nW) The north pole of a permanent magnet is attracted to a south pole of another magnet\n\nX) The direction of a magnetic field is indicated by the north pole of a compass\n\nY) All permanent magnets are surrounded by a magnetic field\n\nZ) When a permanent magnet is cut in half, one piece will be a north pole and one a south pole","Completion":"Z) WHEN A PERMANENT MAGNET IS CUT IN HALF, ONE PIECE WILL BE A NORTH POLE AND ONE PIECE WILL BE A SOUTH POLE"} {"Type":"TOSS-UP\n\nEARTH AND SPACE Short Answer","Prompt":"The surface of Titan is so cold that water is only stable in ice form. However, Titan still has clouds, rain, and lakes composed of which category of substances that includes oil and gasoline.","Completion":"HYDROCARBONS"} {"Type":"BONUS\n\nEARTH AND SPACE Multiple Choice","Prompt":"A mineral with the chemical formula\n\nCa\\({}{2}\\)Mg\\({}{5}\\)Si\\({}{6}\\)O\\({}{2}\\)2(OH)\\({}{2}\\) belongs to which of the following mineral groups?\n\nW) Serpentines\n\nX) Amphiboles\n\nY) Olivines (read as: OH-li-veens)\n\nZ) Pyroxenes (read as: py-ROHK-seens)","Completion":"X) AMPHIBOLES"} {"Type":"TOSS-UP\n\nCHEMISTRY Multiple Choice","Prompt":"How would you classify a reaction in which solid potassium and chlorine gas react to form the ionic compound potassium chloride?\n\nW) Decomposition\n\nX) Combustion\n\nY) Synthesis\n\nZ) Disproportionation","Completion":"Y) SYNTHESIS"} {"Type":"BONUS\n\nCHEMISTRY Short Answer","Prompt":"Generally, what type of reaction is an acid base reaction?","Completion":"DOUBLE DISPLACEMENT (ACCEPT: DOUBLE REPLACEMENT)"} {"Type":"TOSS-UP\n\nBIOLOGY Multiple Choice","Prompt":"The Spotted Owl is considered an endangered species in the western United States, largely due to competition from the introduced Barred Owl. These two species are genetically similar and are believed to have speciated from a common ancestor after separation of the two populations on opposite coasts of the United States. This is an example of:\n\nW) Allopatric speciation\n\nX) Sympathetic speciation\n\nY) Symatric speciation\n\nZ) Genetic drift","Completion":"W) ALLOPATRIC SPECIATION"} {"Type":"BONUS\n\nBIOLOGY Short Answer","Prompt":"Spotted Owls and Barred Owls are examples of ecotypes that can interbreed to produce Sparred Owls - a hybrid species. Which of the following is NOT true about the sparred owls?\n\nW) They are protected under the Endangered Species Act since they are offspring of spotted owls\n\nX) They may interbreed with spotted owls, reducing future numbers of pure spotted owls\n\nY) They may interbreed among themselves, creating a new subspecies\n\nZ) They could trigger the extinction of the Spotted Owl","Completion":"W) THEY ARE PROTECTED UNDER THE ENDANGERD SPECIES ACT SINCE THEY ARE OFFSPRING OF SPOTTED OWLS"} {"Type":"TOSS-UP\n\nCHEMISTRY Short Answer","Prompt":"Most atoms combine to form molecules in such a way that they have how many electrons in a valence shell?","Completion":"8"} {"Type":"BONUS\n\nCHEMISTRY Multiple Choice","Prompt":"In which of the following groups does the elemental form have a completed valence shell?\n\nW) Group 8\n\nX) Group 10\n\nY) Group 18\n\nZ) Group 28","Completion":"Y) GROUP 18"} {"Type":"TOSS-UP\n\nPHYSICS Short Answer","Prompt":"A truck starts from rest. After 4 seconds of constant acceleration, its instantaneous speed is 8 meters per second. What is the average speed in meters per second over the first four seconds?","Completion":"4"} {"Type":"TOSS-UP\n\nMATH Multiple Choice","Prompt":"If \\(x=4\\) and \\(y=2\\), which of the following expressions is undefined?\n\nW) \\(3x+\\frac{7}{4\\cdot 2y}\\)\n\nX) \\(2y+4x\\)\n\nY) \\(xy-x^{2}\\)\n\nZ) \\(x+\\frac{y\\cdot 4}{2}\\)","Completion":"W) \\(3x+\\frac{7}{4\\cdot 2y}\\)"} {"Type":"TOSS-UP\n\nENERGY Multiple Choice","Prompt":"The typical efficiency of an automobile engine is approximately which of the following percentages?\n\nW) 95 X) 65 Y) 25 Z) 5","Completion":"Y) 25"} {"Type":"TOSS-UP\n\nMATH Short Answer","Prompt":"The area of a square is 36 square centimeters. What is the length of its diagonal in centimeters in simplest radical form?","Completion":"6\\(\\sqrt{2}\\)"} {"Type":"TOSS-UP\n\nEARTH AND SPACE Multiple Choice","Prompt":"Eccentricity of an orbit describes:\n\nW) Its deviation from a circular orbit\n\nX) Its deviation from the ecliptic\n\nY) Its time difference from a standard day\n\nZ) Its tumble ratio","Completion":"W) ITS DEVIATION FROM A CIRULAR ORBIT"} {"Type":"TOSS-UP\n\nPHYSICS Multiple Choice","Prompt":"If a soldier wants to shoot a projectile as far into enemy territory\n\nas possible over level ground, at what angle from the ground should the projectile be shot?\n\nW) 0\\({}^{\\circ}\\)\n\nX) 30\\({}^{\\circ}\\)\n\nY) 45\\({}^{\\circ}\\)\n\nZ) 60\\({}^{\\circ}\\)","Completion":"Y) 45\\({}^{\\circ}\\)"} {"Type":"TOSS-UP\n\nENERGY Multiple Choice","Prompt":"Which of the following types of coal is of high quality and has the highest energy content?\n\nW) Subbituminous (read as: subb-bi-tuh-MI-nehs)\n\nX) Bituminous (read as: bi-TUH-mi-nehs)\n\nY) Lignite (read as: LIG-nyt)\n\nZ) Peat","Completion":"X) BITUMINOUS"} {"Type":"TOSS-UP\n\nMATH Multiple Choice","Prompt":"The radius of a bicycle tire is 10 inches. How far does the wheel travel in inches in 6 revolutions?\n\nW) 45 m\n\nX) 60 m\n\nY) 90 m\n\nZ) 120 m","Completion":"Z) 120 m"} {"Type":"TOSS-UP\n\nENERGY Short Answer","Prompt":"When a ball rolls on a pool table, it eventually stops moving because of friction between the ball and the table. In this case, the kinetic energy of the ball has been converted to what other form of energy?","Completion":"HEAT"} {"Type":"TOSS-UP\n\nEARTH AND SPACE Short Answer","Prompt":"What type of water condition is created by differing water densities resulting from melting ice caps and glaciers?","Completion":"STRATIFIED"} {"Type":"TOSS-UP\n\nBIOLOGY Multiple Choice","Prompt":"According to the Hardy-Weinberg Law, from generation to generation, gene frequencies and genotype frequency in a randomly-breeding population are:\n\nW) Constant\n\nX) Increasing\n\nY) Decreasing\n\nZ) Variable, depending upon habitat limitations","Completion":"W) CONSTANT"} {"Type":"BONUS\n\nBIOLOGY Multiple Choice","Prompt":"Some transposons in bacteria carry genes for proteins as well as the gene for transposase. These proteins often benefit the bacteria by imparting:\n\nW) Antibiotic resistance\n\nX) Growth factors\n\nY) gamete formation\n\nZ) New organelles","Completion":"W) ANTIBIOTIC RESISTANCE"} {"Type":"TOSS-UP\n\nMATH Multiple Choice","Prompt":"Given two parallel lines in a plane, the set of points that lie in that plane and are equidistant from the given lines forms a:\n\nW) Point\n\nX) Segment perpendicular to the lines\n\nY) Circle\n\nZ) Line","Completion":"Z) LINE"} {"Type":"BONUS\n\nMATH Multiple Choice","Prompt":"In a plane, the set of points equidistant from a point and a line form a:\n\nW) Ellipse\n\nX) Parabola\n\nY) Triangle\n\nZ) Tip of a cone","Completion":"X) PARABOLA"} {"Type":"TOSS-UP\n\nEARTH AND SPACE Short Answer","Prompt":"Found on Mars, what is the name of the largest volcano in the Solar System?","Completion":"OLYMPUS MONS"} {"Type":"TOSS-UP\n\nMATH Short Answer","Prompt":"A fair 6-sided die is rolled 2 times. As a fraction in lowest terms, what is probability of rolling a 5 both times?","Completion":"\\(\\frac{1}{36}\\)"} {"Type":"TOSS-UP\n\nCHEMISTRY Short Answer","Prompt":"Determine the oxidation state of each of the elements in the polyatomic ion C\\({}{2}\\)O\\({}{4}\\)\\({}^{2}\\):","Completion":"CARBON = 3+; OXYGEN = 2-"} {"Type":"BONUS\n\nCHEMISTRY Multiple Choice","Prompt":"Which of the following 1.0 molar solutions will have the largest change in boiling point from that of pure water?\n\nW) Sodium bromide\n\nX) Magnesium iodide\n\nY) Sucrose\n\nZ) Potassium phosphate","Completion":"Z) POTASSIUM PHOSPHATE"} {"Type":"BONUS\n\n3. CHEMISTRY Multiple Choice","Prompt":"For a given reaction, \\(\\Delta\\)H [delta H] is positive and \\(\\Delta\\)S [delta S] is negative. In which of the following temperature limits is the reaction spontaneous?\n\nW) Not spontaneous at any temperature\n\nX) Spontaneous at only high temperatures\n\nY) Spontaneous at only low temperatures\n\nZ) Spontaneous at all temperatures","Completion":"W) NOT SPONTANEOUS AT ANY TEMPERATURE"} {"Type":"BONUS\n\n4. MATH Short Answer","Prompt":"Each exterior angle of a regular polygon has a measure of 10\\({}^{\\circ}\\). What is the sum, in degrees, of the measures of the interior angles of the polygon?","Completion":"6120\\({}^{\\circ}\\)"} {"Type":"BONUS\n\n5. EARTH AND SPACE Short Answer","Prompt":"What is the Hubble classification for disk galaxies with very little gas and poorly defined spiral arms?","Completion":"S0 (ACCEPT: SB0)"} {"Type":"BONUS\n\n6. BIOLOGY Multiple Choice","Prompt":"Following a stroke that affects the left cerebral hemisphere in an elderly female patient, the patient has halting speech, poor articulation and loss of the ability to write. The patient can still read and understand speech. A PET scan would reveal damage to which of the following parts of the brain?\n\nW) Wernicke's area\n\nX) Amygdala [uh-MIG-duh-luh]\n\nY) Broca's area\n\nZ) Thalamus [THAL-uh-muhs]","Completion":"Y) BROCA'S AREA"} {"Type":"BONUS\n\n7. PHYSICS Short Answer","Prompt":"A 1000 kilogram car moving with a speed of 6 meters per second\n\ncollides perfectly inelastically with 2000 kilogram truck that is at rest. What is the change in\n\nkinetic energy in joules during this collision?","Completion":"12,000"} {"Type":"BONUS\n\n8. CHEMISTRY Short Answer","Prompt":"According to the VSEPR model, what is the molecular geometry\n\nof xenon [ZEE-non] tetrafluoride?","Completion":"SQUARE PLANAR"} {"Type":"BONUS\n\n9. MATH Short Answer","Prompt":"A regular decagon with a perimeter of 40 units has an area of \\(40k\\) square units. Find its apothem.","Completion":"\\(2k\\)"} {"Type":"BONUS\n\n10. EARTH AND SPACE Short Answer","Prompt":"What is the name of the classification system used to rate a hurricane's intensity?","Completion":"SAFFIR-SIMPSON HURRICANE SCALE (ACCEPT: SAFFIR-SIMPSON)"} {"Type":"BONUS\n\n13. PHYSICS Short Answer","Prompt":"Three 300-microfarads capacitors are connected in series to a 12 volt battery. What is the energy stored in one of these capacitors in microjoules after the circuit has reached steady state?","Completion":"2400"} {"Type":"BONUS\n\n14. ENERGY Multiple Choice","Prompt":"As more zero variable cost generators are added to an electric power market, how would you expect the average cost of power in the entire market to change?\n\nW) It would increase\n\nX) It would decrease\n\nY) There would be no change\n\nZ) It would fluctuate","Completion":"X) IT WOULD DECREASE"} {"Type":"BONUS\n\n15. CHEMISTRY Short Answer","Prompt":"A buffer solution is 0.05 molar in lactic acid and 0.5 molar in sodium lactate. If the pKa of lactic acid is 3.85, what is the pH of this solution?","Completion":"4.85"} {"Type":"BONUS\n\n16. MATH Multiple Choice","Prompt":"Which of the following represents the solution in degrees of the equation \\(4\\sin^{2}\\!x=1\\) in the open interval from \\(0^{\\circ}\\) to \\(180^{\\circ}\\)?\n\nW) \\(30^{\\circ}\\)\n\nX) \\(30^{\\circ}\\), \\(60^{\\circ}\\)\n\nY) \\(30^{\\circ}\\), \\(150^{\\circ}\\)\n\nZ) \\(60^{\\circ}\\), \\(120^{\\circ}\\)","Completion":"Y) \\(30^{\\circ}\\), \\(150^{\\circ}\\)"} {"Type":"BONUS\n\n17. EARTH AND SPACE Short Answer","Prompt":"What is the polymorph of graphite that is formed in Earth's mantle?","Completion":"DIAMOND"} {"Type":"BONUS\n\n18. BIOLOGY Short Answer","Prompt":"What is the name of the gland in insects that produces juvenile hormone?","Completion":"CORPUS ALLATUM (ACCEPT: CORPORA ALLATA)"} {"Type":"BONUS\n\n19. PHYSICS Short Answer","Prompt":"A 5 microcoulomb charge is uniformly distributed in a 1-meter diameter sphere. Given that the coulomb constant is 9 x 10\\({}^{9}\\) newton meters squared per coulomb squared, what is the electric field in newtons per coulomb at a radius of 0.25 from the center of the sphere?","Completion":"0"} {"Type":"BONUS\n\n20. ENERGY Multiple Choice","Prompt":"Assuming similar wind velocity and turbine swept area, of the following places, where would you anticipate the largest power output from a wind turbine?\n\nW) Denver\n\nX) Nebraska\n\nY) Rhode Island\n\nZ) West Texas","Completion":"Y) RHODE ISLAND"} {"Type":"BONUS\n\n23. MATH Multiple Choice","Prompt":"Which of the following represents a simplified form of: \\(sin\\,x+\\frac{cos^{2}x}{sinx}\\) [sine of x plus the fraction with numerator cosine squared of x and denominator sine of x]\n\nW) csc x [cosecant of x]\n\nX) sec x [secant of x]\n\nY) cos^2x [cosine squared of x]\n\nZ) 1","Completion":"W) csc \\(x\\)"} {"Type":"BONUS\n\n24. EARTH AND SPACE Multiple Choice","Prompt":"Which of the following best approximates the percentage of solar-type binary stellar systems?\n\nW) 0.1%\n\nX) 1%\n\nY) 10%\n\nZ) 50%","Completion":"Z) 50%"} {"Type":"BONUS\n\n25. BIOLOGY Short Answer","Prompt":"Name all of the following four amino acids that would have a net positive charge in Tris glycine buffer at pH 8.6: glycine [GLY-seen], alanine [AL-uh-nin], lysine [LY-seen], arginine [AHR-juh-neen].","Completion":"3, 4 (ACCEPT: LYSINE, ARGININE)"} {"Type":"TOSS-UP\n\nBIOLOGY Multiple Choice","Prompt":"Which of the following processes results in a net reduction in the amount of atmospheric carbon dioxide?\n\nW) Photosynthesis [foh-tuh-SIN-thuh-sis]\n\nX) Fossil fuel use\n\nY) Respiration\n\nZ) Decomposition","Completion":"W) PHOTOSYNTHESIS"} {"Type":"BONUS\n\nBIOLOGY Multiple Choice","Prompt":"Photosynthesis requires water as a reactant. Oxygen atoms from the water end up in which of the following product molecules?\n\nW) Molecular oxygen\n\nX) Water\n\nY) Glucose\n\nZ) ATP","Completion":"W) MOLECULAR OXYGEN"} {"Type":"TOSS-UP\n\nEARTH AND SPACE Multiple Choice","Prompt":"Which of the following is NOT an example of a native element?\n\nW) Platinum\n\nX) Ruby\n\nY) Gold\n\nZ) Diamond","Completion":"X) RUBY"} {"Type":"BONUS\n\nEARTH AND SPACE Short Answer","Prompt":"What celestial phenomenon results from the movement of charged particles interacting with the Earth's magnetosphere?","Completion":"AUORA BOREALIS (ACCEPT: AUORA AUSTRALIS, NORTHERN LIGHTS, SOUTHERN LIGHTS, AURORA)"} {"Type":"TOSS-UP\n\nPHYSICS Multiple Choice","Prompt":"If an ideal gas performs work on its surroundings, then which of the following must be true?\n\nW) Its pressure increases\n\nX) Its volume increases\n\nY) Its temperature increases\n\nZ) Its internal energy decreases","Completion":"X) ITS VOLUME INCREASES"} {"Type":"BONUS\n\nPHYSICS Multiple Choice","Prompt":"A 90 kilogram fullback running north at 10 meters per second is tackled by a 120 kilogram linebacker running south at 4 meters per second in a perfectly inelastic collision. Which of the following is the velocity in meters per second of the players after the collision?\n\nW) 3 south\n\nX) 3 north\n\nY) 2 south\n\nZ) 2 north","Completion":"Z) 2 NORTH"} {"Type":"TOSS-UP\n\nMATH Multiple Choice","Prompt":"For all real numbers, the expression \\(\\sqrt[x^{2}-4x-4]\\) [the square root of the quantity x squared minus 4 x plus 4] equals which of the following?\n\nW) \\(x-2\\)\n\nX) \\(x-2x+2\\)\n\nY) \\(|x-2|\\)[the absolute value of the quantity x minus 2]\n\nZ) \\(x+2\\)","Completion":"Y) \\(|x-2|\\)"} {"Type":"BONUS\n\nMATH Short Answer","Prompt":"What are the coordinates of the vertex of the graph of \\(y=2x^{2}+8x-3\\)?","Completion":"(-2,-11)"} {"Type":"TOSS-UP\n\nCHEMISTRY Multiple Choice","Prompt":"Which of the following types of structures describes a beta sheet?\n\nW) Primary\n\nX) Secondary\n\nY) Tertiary [TER-shee-er-ee]\n\nZ) Quaternary [KWOT-er-ner-ree]","Completion":"X) SECONDARY"} {"Type":"BONUS\n\nCHEMISTRY Short Answer","Prompt":"What is the oxidation state of bromine in the compound\n\nbromous acid?","Completion":"+3"} {"Type":"TOSS-UP\n\nENERGY Multiple Choice","Prompt":"When traveling at the same speed, which of the following fission\n\nproducts has the greatest kinetic energy?\n\nW) A neutron\n\nX) An alpha particle\n\nY) A beta particle\n\nZ) A positron","Completion":"X) AN ALPHA PARTICLE"} {"Type":"BONUS\n\nENERGY Short Answer","Prompt":"Name all of the following four choices that are types of coal: lignite, bituminous [by-TOO-muh-nuhs], dolomite, anthrac [AN-thruh-syt].","Completion":"1, 2, 4 (ACCEPT: LIGNITE, BITUMINOUS, ANTHRACITE)"} {"Type":"TOSS-UP\n\nBIOLOGY Short Answer","Prompt":"What is the commercially important gelatinous [ju-LAT-n-uhs] substance that is produced by Geldium amansil?","Completion":"AGAR"} {"Type":"TOSS-UP\n\nEARTH AND SPACE Short Answer","Prompt":"What are the two most abundant dissolved gases released by magma?","Completion":"WATER VAPOR (ACCEPT: WATER) AND CARBON DIOXIDE"} {"Type":"TOSS-UP\n\nPHYSICS Multiple Choice","Prompt":"Which of the following will double the power dissipated by a resistor?\n\nW) Doubling the voltage across the resistor\n\nX) Decreasing the voltage to one half of the original value\n\nY) Increasing the voltage by a factor of the square root of 2\n\nZ) Decreasing the voltage by a factor of the square root of 2","Completion":"Y) INCREASING THE VOLTAGE BY A FACTOR OF THE SQUARE ROOT OF 2"} {"Type":"BONUS\n\nPHYSICS Short Answer","Prompt":"Given \\(c\\) as 3 X 10\\({}^{8}\\) meters per second, what is the wavelength in meters to the nearest tenth of a radio wave with a frequency of 94.7 megahertz?","Completion":"3.2"} {"Type":"TOSS-UP\n\nMATH Multiple Choice","Prompt":"The equation -3 + |-2x - 5| = k [negative 3 plus the absolute value of the quantity negative 2x minus 5 equals kJ has no solution if \\(k\\) equals which of the following?\n\nW) -5\n\nX) -3\n\nY) 0\n\nZ) 7","Completion":"W) -5"} {"Type":"BONUS\n\nMATH Multiple Choice","Prompt":"Which of the following statements is NOT true?\n\nW) cos(-\\(x\\)) = cos(\\(x\\)) [cosine of negative x equals cosine of x]\n\nX) tan(-\\(x\\)) = tan(\\(x\\)) [fangent of negative x equals tangent of x]\n\nY) The amplitude of \\(y\\) = -2 cos(\\(\\delta\\)) is equal to 2\n\nZ) The range of \\(y\\) = -5 sin(\\(\\delta\\)) is [-5, 5] [the closed interval from negative 5 to 5]","Completion":"X) tan(-\\(x\\)) = tan(\\(x\\))"} {"Type":"TOSS-UP\n\nCHEMISTRY Short Answer","Prompt":"How many moles of carbon dioxide are produced from the combustion of 5 moles of hydrogen?","Completion":"0"} {"Type":"TOSS-UP\n\nENERGY Multiple Choice","Prompt":"Which of the following power types demonstrates large scale temporary storage of energy?\n\nW) Coal\n\nX) Chemical\n\nY) Hydroelectric\n\nZ) Wind","Completion":"Y) HYDROELECTRIC"} {"Type":"TOSS-UP\n\nBIOLOGY Short Answer","Prompt":"Motion of the basilar membrane stimulates what cell type that initiates the transmission of a signal to the brain?","Completion":"INNER HAIR (ALSO ACCEPT STEREOCILIA OR HAIR CELLS)"} {"Type":"TOSS-UP\n\nEARTH AND SPACE Multiple Choice","Prompt":"Which of the following is a term specifically given to water that replenishes an aquifer?\n\nW) Recharge\n\nX) Discharge\n\nY) Hydraulic\n\nZ) Permeable","Completion":"W) RECHARGE"} {"Type":"TOSS-UP\n\nPHYSICS Multiple Choice","Prompt":"Which of the following particles has an electric charge of zero?\n\nW) Electron\n\nX) Quark\n\nY) Gluon\n\nZ) Muon","Completion":"Y) GLUON"} {"Type":"BONUS\n\nPHYSICS Short Answer","Prompt":"A swimming pool water pump draws about 4.2 amperes when connected to 240 volts. If energy costs 10 cents per kilowatt hour, what is the bill to the nearest cent for running the pump continuously for 1 day?","Completion":"$2.42 (ACCEPT: 242 CENTS)"} {"Type":"TOSS-UP\n\nMATH Short Answer","Prompt":"If the graph of the function \\(f\\) lies in quadrant IV, then in which quadrant does the graph of the inverse of \\(f\\)lie?","Completion":"II (ACCEPT: 2 OR SECOND)"} {"Type":"BONUS\n\nMATH Short Answer","Prompt":"What is the period of the function \\(f(x)=\\sin((\\pi\/\\,x+\\pi\/\\)[f of x equals the sine of the quantity pi over 6 times x plus pi over 4]?","Completion":"12"} {"Type":"TOSS-UP\n\nCHEMISTRY Multiple Choice","Prompt":"Which of the following metals commonly forms cations [KA7-eye-uhns] with differing charges?\n\nW) Strontium\n\nX) Rubidium\n\nY) Calcium\n\nZ) Chromium","Completion":"Z) CHROMIUM"} {"Type":"BONUS\n\nCHEMISTRY Multiple Choice","Prompt":"Which of the following is the most common linkage between successive monomers in cellulose?\n\nW) Alpha-1, 4\n\nX) Alpha-1, 6\n\nY) Beta-1, 4\n\nZ) Beta-1, 6","Completion":"Y) BETA-1, 4"} {"Type":"TOSS-UP\n\nENERGY Multiple Choice","Prompt":"One of the criticisms of ground-mount solar farms is the amount of space they occupy. A new solar system was recently announced called \"floatovoltaics,\" because it is a floating solar farm. Which of the following is NOT a potential benefit of this system?\n\nW) Reducing evaporation\n\nX) Reducing algae growth\n\nY) Short-term profitability\n\nZ) Increasing solar panel power production","Completion":"Y) SHORT-TERM PROFITABILITY"} {"Type":"BONUS\n\nENERGY Short Answer","Prompt":"When lodine-131 is ingested, which organ most readily absorbs it and can potentially be damaged by it?","Completion":"THYROID"} {"Type":"TOSS-UP\n\nBIOLOGY Multiple Choice","Prompt":"Which of the following is an incorrect descriptor of DNA replication?\n\nW) Semi-conservative\n\nX) Discontinuous on the lagging strand and continuous on the leading strand\n\nY) High-fidelity\n\nZ) Repetitive","Completion":"Z) REPETITIVE"} {"Type":"TOSS-UP\n\nEARTH AND SPACE Multiple Choice","Prompt":"Which of the following factors does NOT directly impact the climate of a geographic location?\n\nW) Elevation\n\nX) Ocean currents\n\nY) Longitude\n\nZ) Latitude","Completion":"Y) LONGITUDE"} {"Type":"TOSS-UP\n\nPHYSICS Multiple Choice","Prompt":"A spinning skater draws her arms in closer to her body to rotate faster. Which of the following conservation principles is being applied?\n\nW) Rotational kinetic energy\n\nX) Torque\n\nY) Angular momentum\n\nZ) Energy","Completion":"Y) ANGULAR MOMENTUM"} {"Type":"BONUS\n\nPHYSICS Short Answer","Prompt":"An object with a moment of inertia of 2 kilogram meters squared about its axis of rotation makes 6 revolutions per minute. A second stationary object is now gently placed on the first so that the combined moment of inertia about the spin axis is 4 kilogram meters squared. What is the spin rate in revolutions per minute of the composite object?","Completion":"3"} {"Type":"TOSS-UP\n\nMATH Multiple Choice","Prompt":"Which of the following is an equivalent form of \\(2\/(3+i)\\) [2 over the quantity 3 plus ij?\n\nW) \\((3-i)\/4\\) [open parenthesis 3 minus i,close parenthesis, over 4]\n\nX) \\((3-i)\/5\\) [open parenthesis 3 minus i,close parenthesis, over 5]\n\nY) \\((4-i)\/4\\) [open parenthesis 4 minus i,close parenthesis, over 4]\n\nZ) \\((4-i)\/5\\) [open parenthesis 4 minus i,close parenthesis, over 5]","Completion":"X) \\((3-i)\/5\\)"} {"Type":"BONUS\n\nMATH Multiple Choice","Prompt":"Which of the following points is on the graph of the inverse of the function \\(f(x)=10^{x\\,*\\,2}\\) [f of x equals 10 to the power of the quantity x plus 2]?\n\nW) \\((100,\\)\n\nX) \\((0,100)\\)\n\nY) \\((10,\\)\n\nZ) \\((0,\\)","Completion":"W) \\((100,0)\\)"} {"Type":"TOSS-UP\n\nCHEMISTRY Short Answer","Prompt":"What is the term for a product generated in one step of a reaction mechanism that is consumed as a reactant in another step?","Completion":"REACTION INTERMEDIATE (ACCEPT: REACTIVE INTERMEDIATE OR INTERMEDIATE)"} {"Type":"BONUS\n\nCHEMISTRY Short Answer","Prompt":"Several years ago, scientists announced that they had succeeded in getting a nucleus of calcium-48 to fuse with a nucleus of plutonium-244. The combined nucleus is a nucleus of what element?","Completion":"UNUNQUADIUM (ACCEPT: ELEMENT 114; UUQ, FLEROVIUM, OR FL)"} {"Type":"TOSS-UP\n\nEARTH AND SPACE Multiple Choice","Prompt":"Which of the following is the definition of dew point?\n\nW) The ratio of actual water vapor compared to saturated water vapor in the air\n\nX) The ratio of liquid water to water vapor in the air\n\nY) The temperature at which the air is saturated with water vapor\n\nZ) The temperature at which water evaporates into the air","Completion":"Y) THE TEMPERATURE AT WHICH THE AIR IS SATURATED WITH WATER VAPOR"} {"Type":"BONUS\n\nEARTH AND SPACE Short Answer","Prompt":"What phenomenon is recorded in the changing orientation of iron-bearing minerals on the surface of the oceanic crust?","Completion":"MAGNETIC POLE REVERSAL (ACCEPT: FLIPPED\/ALTERNATING MAGNETIC FIELD)"} {"Type":"TOSS-UP\n\nBIOLOGY Multiple Choice","Prompt":"In which of the following locations would a spliceosome [SPLY-see-uh-sohm] be found?\n\nW) Nucleus [NOO-klee-uhs]\n\nX) Cytoplasm\n\nY) Plasma membrane\n\nZ) Endosome","Completion":"W) NUCLEUS"} {"Type":"BONUS\n\nBIOLOGY Short Answer","Prompt":"Identify all of the following five structures which can be found on paramecia [par-uh-MEE-see-uh]: oral groove, cilia, contractile vacuoles, flagella, trichocysts [TRIK-uh-sists].","Completion":"1, 2, 3, 5 (ACCEPT: ORAL GROOVE, CILIA, CONTRACTILE VACUOLES, TRICHOCYSTS)"} {"Type":"TOSS-UP\n\nMATH Multiple Choice","Prompt":"In the graph of the function \\(f(x)=\\frac{3}{2}\\sin\\left(\\frac{\\pi}{\\lambda}\\right)\\)If of x equals three halves the sine of the quantity x over 2], at which of the following x coordinates would \\(f(x)=0\\)?\n\nW) \\(\\pi\/2\\)\n\nX) \\(\\pi\\)\n\nY) \\(2\\pi\\)\n\nZ) \\(3\\pi\/2\\)","Completion":"Y) \\(2\\pi\\)"} {"Type":"BONUS\n\nMATH Short Answer","Prompt":"Expressing the answer in simplest form, solve \\(\\left|2r-\\frac{13}{4}\\right|<\\frac{5}{2}\\)the absolute value of the quantity 2r minus 13 fourths is less than 5 halves].","Completion":"\\(3\/81\\)"} {"Type":"TOSS-UP\n\nPHYSICS Multiple Choice","Prompt":"If two inductance coils of inductance \\(L\\) are connected in parallel, but are well separated, the equivalent inductance of the system is which of the following?\n\nW) 0.5\\(L\\)\n\nX) \\(L\\)\n\nY) 2\\(L\\)\n\nZ) \\(L^{2}\\)","Completion":"W) 0.5\\(L\\)"} {"Type":"BONUS\n\nPHYSICS Multiple Choice","Prompt":"Two balls are dropped one second apart from a tall building. Given g as 10 meters per second squared and neglecting air resistance, what is the approximate distance in meters between the two balls three seconds after the first ball is dropped?\n\nW) 20\n\nX) 25\n\nY) 45\n\nZ) Depends on the masses","Completion":"X) 25"} {"Type":"TOSS-UP\n\nEARTH AND SPACE Multiple Choice","Prompt":"Which of the following theories describes the random movement of small particles in the atmosphere?\n\nW) Diffusion\n\nX) Reynolds number\n\nY) Brownian motion\n\nZ) Kinetic","Completion":"Y) BROWNIAN MOTION"} {"Type":"BONUS\n\nEARTH AND SPACE Short Answer","Prompt":"Igneous [IG-nee-uhs] rocks and magmas are classified into four main compositional groups: ultramafic, mafic, intermediate, and felsic, based on the content of what chemical compound?","Completion":"SILICA"} {"Type":"TOSS-UP\n\nBIOLOGY Short Answer","Prompt":"What is the term for the process in which a reptile sheds its outer layer of skin, or an arthropod [AHR-thruh-pod] or crustacean [kruh-STAY-shuhn] sheds its outer shell?","Completion":"ECDYSIS (ACCEPT: MOLTING)"} {"Type":"BONUS\n\nBIOLOGY Short Answer","Prompt":"The acronym NAD stands for what phrase?","Completion":"NICOTINAMIDE ADENINE DINUCLEOTIDE"} {"Type":"TOSS-UP\n\nENERGY Short Answer","Prompt":"Cadmium and boron are used in a nuclear reactor to absorb which subatomic particle?","Completion":"NEUTRON"} {"Type":"BONUS\n\nENERGY Multiple Choice","Prompt":"Which of the elements, used as a nuclear fuel, is most abundant in the Earth's crust?\n\nW) Uranium\n\nX) Plutonium\n\nY) Thorium\n\nZ) Americium","Completion":"Y) THORIUM"} {"Type":"TOSS-UP\n\nCHEMISTRY Multiple Choice","Prompt":"When hydrogen reacts with 2-butanone, which of the following is the primary product?\n\nW) A primary aldehyde [AL-duh-hyd]\n\nX) A secondary aldehyde\n\nY) A primary alcohol\n\nZ) A secondary alcohol","Completion":"Z) A SECONDARY ALCOHOL"} {"Type":"TOSS-UP\n\nMATH Short Answer","Prompt":"If two 6-sided fair dice are rolled, providing your answer as a fraction in lowest terms, what is the probability that both numbers will be even?","Completion":"\\(\\frac{1}{4}\\)"} {"Type":"TOSS-UP\n\nPHYSICS Multiple Choice","Prompt":"White light is incident on a diffraction grating. Which of the following colors will be diffracted through the largest angle?\n\nW) Red\n\nX) Yellow\n\nY) Blue\n\nZ) Green","Completion":"W) RED"} {"Type":"TOSS-UP\n\nEARTH AND SPACE Multiple Choice","Prompt":"Which of the following clockwise currents brings\n\nwarm Caribbean waters into the Gulf of Mexico before joining the Gulf Stream?\n\nW) Caribbean system\n\nX) Loop\n\nY) North Equatorial\n\nZ) Florida","Completion":"X) LOOP"} {"Type":"TOSS-UP\n\nBIOLOGY Short Answer","Prompt":"Who discovered the antibiotic nature of penicillin in 1928?","Completion":"SIR ALEXANDER FLEMING (ACCEPT: ALEXANDER FLEMING, FLEMING)"} {"Type":"TOSS-UP\n\nBIOLOGY Multiple Choice","Prompt":"Female insects usually lay their eggs near which of the following?\n\nW) Light\n\nX) Shade\n\nY) Food\n\nZ) Water","Completion":"Y) FOOD"} {"Type":"BONUS\n\nBIOLOGY Multiple Choice","Prompt":"Which of the following biological molecules most directly enables the structural compartmentalization required for cellular respiration and photosynthesis [foh-tuh-SIN-thuh-sis]?\n\nW) Amino acids\n\nX) Phospholipids [fos-foh-LIP-ids]\n\nY) Glycogen [GLY-kuh-junh]\n\nZ) Cholesterol","Completion":"X) PHOSPHOLIPIDS"} {"Type":"TOSS-UP\n\nPHYSICS Short Answer","Prompt":"Which Greek scientist discovered that, when an object is immersed in a fluid, there is a buoyant force acting upward on the object equal to the weight of the displaced fluid?","Completion":"ARCHIMEDES"} {"Type":"BONUS\n\nPHYSICS Multiple Choice","Prompt":"Which of the following is true for a cyclist applying her brakes while traveling west on a straight path?\n\nW) Her acceleration is zero\n\nX) She accelerates to the east\n\nY) She accelerates to the west\n\nZ) She accelerates both to the east and the west","Completion":"X) SHE ACCELERATES TO THE EAST"} {"Type":"TOSS-UP\n\nCHEMISTRY Multiple Choice","Prompt":"In which of the following ways would oxygen's electrons fill the 3 p subshells?\n\nW) 1 pair, 2 unpaired\n\nX) 1 pair, 1 unpaired\n\nY) 2 pairs\n\nZ) 3 pairs","Completion":"W) 1 PAIR, 2 UNPAIRED"} {"Type":"BONUS\n\nCHEMISTRY Short Answer","Prompt":"What gaseous organic compound is made when ethanol is heated in a concentrated solution of sulfuric acid?","Completion":"ETHYLENE (ACCEPT: ETHENE)"} {"Type":"TOSS-UP\n\nEARTH AND SPACE Short Answer","Prompt":"Mechanical weathering is one type of weathering, what is the other?","Completion":"CHEMICAL (ALSO ACCEPT BIOLOGICAL)"} {"Type":"BONUS\n\nEARTH AND SPACE Multiple Choice","Prompt":"For the June solstice, which of the following statements is NOT true?\n\nW) The northern hemisphere experiences the day with the longest period of sunlight\n\nX) There are 24 hours of sunlight in the Arctic Circle\n\nY) Earth is further from the Sun than it is at the winter solstice\n\nZ) The Sun is directly overhead at noon in the Arctic Circle","Completion":"Z) THE SUN IS DIRECTLY OVERHEAD AT NOON IN THE ARCTIC CIRCLE"} {"Type":"TOSS-UP\n\nMATH Short Answer","Prompt":"How many distinct diagonals does a pentagon have?","Completion":"5"} {"Type":"TOSS-UP\n\nENERGY Multiple Choice","Prompt":"Which of the following describes the purpose of the ground connection on a common power outlet?\n\nW) Provide a zero potential reference\n\nX) Remove excess heat from the transmission system\n\nY) Give a low resistance path for short circuits to travel through\n\nZ) Act as a small current source to prevent corrosion","Completion":"Y) GIVE A LOW RESISTANCE PATH FOR SHORT CIRCUITS TO TRAVEL"} {"Type":"","Prompt":"TOSS-UP","Completion":"7) BIOLOGY Multiple Choice Which of the following statements is NOT true about spike moss? W) They do not have true roots X) They have a well-developed vascular system Y) They are widely distributed in all parts of the world Z) Their leaves are spore-bearing X) THEY HAVE A WELL-DEVELOPED VASCULAR SYSTEM"} {"Type":"","Prompt":"TOSS-UP","Completion":"8) PHYSICS Multiple Choice Pascal's law governs the actions of forces in which of the following? W) Gases X) Solids Y) Incompressible fluids Z) A vacuum Y) INCOMPRESSIBLE FLUIDS"} {"Type":"TOSS-UP\n\nCHEMISTRY Short Answer","Prompt":"In a flame test on an unknown compound, the flame is violet. What element is likely to be in the compound?","Completion":"POTASSIUM (ACCEPT: RUBIDIUM, CESIUM)"} {"Type":"TOSS-UP\n\nEARTH AND SPACE Short Answer","Prompt":"What term is used to describe the advance of desert conditions into formerly non-desert areas?","Completion":"DESERTIFICATION"} {"Type":"TOSS-UP\n\nMATH Short Answer","Prompt":"What is the surface area of a cube with volume 8?","Completion":"24"} {"Type":"TOSS-UP\n\nENERGY Multiple Choice","Prompt":"Which of the following uses stored electricity and is required to get a gasoline-powered car's engine going?\n\nW) Catalytic converter\n\nX) Starter motor\n\nY) Fuel rail\n\nZ) Alternator","Completion":"X) STARTER MOTOR"} {"Type":"TOSS-UP\n\nBIOLOGY Multiple Choice","Prompt":"Which of the following best describes blood?\n\nW) It is a specialized type of cell found in blood vessels\n\nX) It is a type of cell formed in the bone marrow\n\nY) It is a kind of connective tissue in which red and white blood cells are suspended in plasma\n\nZ) It is a kind of epithelial tissue [ep-uh-THEE-lee-al]","Completion":"Y) IT IS A KIND OF CONNECTIVE TISSUE IN WHICH RED AND WHITE BLOOD CELLS ARE SUSPENDED IN PLASMA"} {"Type":"TOSS-UP\n\nEARTH AND SPACE Multiple Choice","Prompt":"Which of the following statements about the Moon best describes why we only see one side of the Moon from Earth?\n\nW) It does not rotate\n\nX) It has a side that is always dark\n\nY) It does not revolve\n\nZ) It has the same period for rotation and revolution","Completion":"Z) IT HAS THE SAME PERIOD FOR ROTATION AND REVOLUTION"} {"Type":"TOSS-UP\n\nBIOLOGY Multiple Choice","Prompt":"Which of the following statements best describes the thermodynamic effect of an enzyme on the reaction it catalyzes?\n\nW) The energy level of the reactants is decreased in the presence of the enzyme\n\nX) The activation energy of the reaction is decreased\n\nY) The activation energy of the reaction is increased\n\nZ) The \\(\\Delta\\)G [delta-G] between products and reactants is decreased","Completion":"X) THE ACTIVATION ENERGY OF THE REACTION IS DECREASED"} {"Type":"TOSS-UP\n\nPHYSICS Short Answer","Prompt":"The two leaves of an electroscope repel each other and stand out at an angle. What force balances the electrical force so the leaves don't stand out farther?","Completion":"GRAVITY"} {"Type":"TOSS-UP\n\nCHEMISTRY Multiple Choice","Prompt":"A nitrogen atom present in the backbone of a polypeptide [pol-ee-PEP-tyd] is part of which of the following functional groups?\n\nW) A primary amine\n\nX) A secondary amine\n\nY) A tertiary amine [TER-shee-er-ee]\n\nZ) An amide","Completion":"Z) AN AMIDE"} {"Type":"BONUS\n\nCHEMISTRY Multiple Choice","Prompt":"Which of the following is the final product of purine metabolism excreted in the urine?\n\nW) Urea\n\nX) Uric acid\n\nY) Ammonia\n\nZ) Ammonium","Completion":"X) URIC ACID"} {"Type":"TOSS-UP\n\nEARTH AND SPACE Short Answer","Prompt":"What is the most common element in Earth's crust?","Completion":"OXYGEN"} {"Type":"BONUS\n\nEARTH AND SPACE Multiple Choice","Prompt":"Which of the following is NOT a characteristic of the mixed layer of the ocean?\n\nW) Relatively stratified in the vertical\n\nX) In direct contact with the atmosphere\n\nY) Stirred by winds and convection\n\nZ) Varied depth with latitude and season","Completion":"W) RELATIVELY STRATIFIED IN THE VERTICAL"} {"Type":"TOSS-UP\n\nMATH Short Answer","Prompt":"For what values of \\(x\\) is the function \\(f(x)=\\frac{x^{2}}{x^{2}-4x+4}\\) [f of x equals the fraction with numerator x squared and denominator x squared minus 4 x plus 4] NOT defined?","Completion":"2"} {"Type":"TOSS-UP\n\nENERGY Short Answer","Prompt":"What energy barrier must a chemical reaction pass in order to proceed?","Completion":"ACTIVATION ENERGY"} {"Type":"TOSS-UP\n\nBIOLOGY Multiple Choice","Prompt":"Which of the following choices is NOT a source of new genetic combinations?\n\nW) Mitosis [my-TOH-sis]\n\nX) Fertilization\n\nY) Mutation\n\nZ) Meiosis [meye-OH-sis]","Completion":"W) MITOSIS"} {"Type":"TOSS-UP\n\nPHYSICS Short Answer","Prompt":"What force does a charged particle moving through a magnetic field experience that is perpendicular to the direction the particle is traveling?","Completion":"LORENTZ (ACCEPT: MAGNETIC FORCE)"} {"Type":"TOSS-UP\n\nCHEMISTRY Short Answer","Prompt":"At -78\\({}^{\\circ}\\) Celsius, dry ice changes directly from a solid to a gas. What is this process called?","Completion":"SUBLIMATION"} {"Type":"TOSS-UP\n\nEARTH AND SPACE Short Answer","Prompt":"In what compass direction would the magnetic north pole of your compass point during a period in which the Earth's magnetic field was reversed?","Completion":"SOUTH"} {"Type":"TOSS-UP\n\nMATH Short Answer","Prompt":"If \\(\\ln(x^{3})=6\\)[natural log of the quantity x cubed equals 6], what is \\(\\ln(x^{2})\\)[the natural log of the quantity x squared]?","Completion":"4"} {"Type":"TOSS-UP\n\nCHEMISTRY Multiple Choice","Prompt":"Which of the following transition metals has the smallest atomic radius?\n\nW) Nickel\n\nX) Iron\n\nY) Scandium\n\nZ) Cadmium","Completion":"W) NICKEL"} {"Type":"BONUS\n\nCHEMISTRY Short Answer","Prompt":"Consisting of a fatty acid chain attached through an amide linkage to sphingosine [SFING-guh-seen], what is the fundamental structural unit common to all sphingolipids?","Completion":"CERAMIDE"} {"Type":"TOSS-UP\n\nEARTH AND SPACE Short Answer","Prompt":"What is the naturally occurring radioactive isotope of carbon called?","Completion":"CARBON-14 (ACCEPT: C-14)"} {"Type":"BONUS\n\nEARTH AND SPACE Short Answer","Prompt":"Based on the Spitzer Space Telescope observatory program, infrared emission from most galaxies comes primarily from three sources. Name two of them.","Completion":"STARS, INTERSTELLAR GAS, DUST (ACCEPT: INTERSTELLAR DUST)"} {"Type":"TOSS-UP\n\nPHYSICS Short Answer","Prompt":"An alpha particle consists of what types and numbers of subatomic particles?","Completion":"TWO PROTONS AND TWO NEUTRONS"} {"Type":"TOSS-UP\n\nENERGY Multiple Choice","Prompt":"In 1859, drilling first began in the U.S. for which of the following energy resources?\n\nW) Coal\n\nX) Oil\n\nY) Natural gas\n\nZ) Geothermal energy","Completion":"X) OIL"} {"Type":"TOSS-UP\n\nBIOLOGY Multiple Choice","Prompt":"Which of the following terms is used to describe a structure that is similar in different species of organisms, because the species share a common ancestor that was the source of the structure?\n\nW) Homologous [huh-MOL-uh-guhs]\n\nX) Convergent\n\nY) Analogous [uh-NAL-uh-guhs]\n\nZ) Divergent","Completion":"W) HOMOLOGOUS"} {"Type":"BONUS\n\nBIOLOGY Multiple Choice","Prompt":"Which of the following is an example of an ectoparasite [ek-toh-PAR-uh-syt]?\n\nW) Tapeworm\n\nX) Hookworm\n\nY) Nematode\n\nZ) Louse","Completion":"Z) LOUSE"} {"Type":"TOSS-UP\n\nMATH Multiple Choice","Prompt":"A certain radioactive element decays over time according to the equation \\(y=R\\ (1\\ \/\\ ^{x\/200}\\)[ly equals R times one half to the power of the quantity x over 300], where \\(R\\) equals the number of grams that were present initially and \\(x\\) equals time in years. If 1000 grams were present initially, how many grams will remain after 900 years?\n\nW) 62.5\n\nX) 125\n\nY) 250\n\nZ) 500","Completion":"X) 125"} {"Type":"BONUS\n\nMATH Short Answer","Prompt":"If -4x - 9y - 3z = -10 and -5a - 9b = -6, then what is -9b - 15z - 45y - 20x - 5a?","Completion":"-56"} {"Type":"TOSS-UP\n\nCHEMISTRY Short Answer","Prompt":"In carbon dioxide, what is the formal charge on the central atom?","Completion":"0"} {"Type":"TOSS-UP\n\nEARTH AND SPACE Short Answer","Prompt":"The presence of what molecule causes temperatures to increase with increasing altitude in the stratosphere?","Completion":"OZONE (ACCEPT: O\\({}{3}\\))"} {"Type":"TOSS-UP\n\nPHYSICS Multiple Choice","Prompt":"Which of the following wave types does not exhibit polarization?\n\nW) Gravitational\n\nX) X-ray\n\nY) Acoustic\n\nZ) Visible light","Completion":"Y) ACOUSTIC"} {"Type":"TOSS-UP\n\nENERGY Multiple Choice","Prompt":"Nuclear power plants and coal fired plants have which of the following operations in common?\n\nW) Both give off smoke from a stack\n\nX) Both heat water to become steam\n\nY) Both use moderators to control energy output\n\nZ) Both use water to control heat build-up","Completion":"X) BOTH HEAT WATER TO BECOME STEAM"} {"Type":"TOSS-UP\n\nBIOLOGY Multiple Choice","Prompt":"The probocsis [proh-BOS-kis] is used primarily by invertebrates to perform which of the following functions?\n\nW) Egg fertilization\n\nX) Excavation\n\nY) Feeding\n\nZ) Defense","Completion":"Y) FEEDING"} {"Type":"BONUS\n\nBIOLOGY Multiple Choice","Prompt":"Which of the following pollinators is a member of the order Lepidoptera [Hep-i-DOP-ter-uh]?\n\nW) Lesser long-nosed bats\n\nX) Fritillary butterflies [FRIT-l-er-ee]\n\nY) Green bottle flies\n\nZ) Blue orchard bees","Completion":"X) FRITILARY BUTTERFLIES"} {"Type":"TOSS-UP\n\nMATH Short Answer","Prompt":"The perimeters of two similar figures have a ratio of 5:7. What is the ratio of their areas?","Completion":"25:49 (ACCEPT: 25\/49)"} {"Type":"BONUS\n\nMATH Short Answer","Prompt":"After 40 times at bat, BJ has 9 hits. What is the least number of hits he would need in his next 30 times at bat so that he has hits in at least 31% of his total at-bats?","Completion":"13"} {"Type":"TOSS-UP\n\nEARTH AND SPACE Short Answer","Prompt":"Which two planets in our solar system are considered ice giants?","Completion":"URANUS AND NEPTUNE"} {"Type":"TOSS-UP\n\nBIOLOGY Multiple Choice","Prompt":"Which of the following organelles within the eukaryotic [yoo-KAR-ee-oh-tik] cell functions as the digestive system of the cell, serving both to degrade material taken up from outside the cell and to digest obsolete components of the cell itself?\n\nW) Lysosome\n\nX) Golgi apparatus\n\nY) Smooth endoplasmic reticulum [EN-duh-plaz-mik ri-TIK-yuh-luhm]\n\nZ) Rough endoplasmic reticulum","Completion":"W) LYSOSOME"} {"Type":"TOSS-UP\n\nCHEMISTRY Multiple Choice","Prompt":"Which of the following carbohydrates will most easily dissolve in water?\n\nW) Glycogen [GLY-kuh-juhn]\n\nX) Starch\n\nY) Maltose [MOHL-tohs]\n\nZ) Cellulose","Completion":"Y) MALTOSE"} {"Type":"BONUS\n\nCHEMISTRY Short Answer","Prompt":"What adjective describes a mixture that has equal amounts of left- and right-handed enantiomers [i-NAN-tee-uh-mers] of a chiral [KY-ruhl] molecule?","Completion":"RACEMIC (ACCEPT: RACEMATES)"} {"Type":"TOSS-UP\n\nEARTH AND SPACE Multiple Choice","Prompt":"Which of the following is NOT necessary for a cloud to form?\n\nW) Saturated air\n\nX) Water vapor\n\nY) High altitude\n\nZ) High humidity","Completion":"Y) HIGH ALTITUDE"} {"Type":"BONUS\n\nEARTH AND SPACE Multiple Choice","Prompt":"Which of the following is NOT a symptom of desertification?\n\nW) Lowering of the water table\n\nX) Decreased rate of erosion\n\nY) Increased salinity in natural waters and soils\n\nZ) Progressive destruction of vegetation","Completion":"X) DECREASED RATE OF EROSION"} {"Type":"TOSS-UP\n\nPHYSICS Short Answer","Prompt":"A person standing on a scaffold lowers an object with a weight of 250 newtons by means of a rope, at constant speed. If the weight of the rope is negligible, what is the force in newtons that the person exerts on the rope?","Completion":"250"} {"Type":"BONUS\n\nPHYSICS Short Answer","Prompt":"Relative to the ground, how many joules of potential energy does a 1000 newton boulder have at the top of a 5 meter ledge, relative to the bottom of the ledge and given g as 10 meters per second squared?","Completion":"5000"} {"Type":"TOSS-UP\n\nENERGY Multiple Choice","Prompt":"Which of the following is a product from a hydrogen fuel reaction?\n\nW) Water\n\nX) Carbon dioxide\n\nY) Methane\n\nZ) Nitrogen","Completion":"W) WATER"} {"Type":"BONUS\n\nENERGY Short Answer","Prompt":"What value is used to describe the heat resistance of home insulation?","Completion":"R-VALUE"} {"Type":"TOSS-UP\n\nBIOLOGY Multiple Choice","Prompt":"Which of the following types of genes violates Mendel's principle of independent assortment?\n\nW) Linked\n\nX) Codominant\n\nY) Dominant\n\nZ) Epistatic","Completion":"W) LINKED"} {"Type":"BONUS\n\nBIOLOGY Short Answer","Prompt":"Name an allosteric [al-uh-STER-ik] inhibitor of pyruvate [py-ROO-vayt] kinase.","Completion":"ATP (ACCEPT: ADENOSINE TRIPHOSPHATE, ALANINE OR ACETYL-CoA)"} {"Type":"TOSS-UP\n\nMATH Short Answer","Prompt":"If a person walks 4 miles due north, 6 miles due east, then 4 miles due north, how far in miles is the person displaced from his or her original position?","Completion":"10"} {"Type":"BONUS\n\nMATH Short Answer","Prompt":"In a standard deck of cards with no jokers, providing your answer as a fraction in lowest terms, what is the probability of picking a card that is black or a number less than 10 (which includes aces)?","Completion":"11\/13"} {"Type":"TOSS-UP\n\nCHEMISTRY Short Answer","Prompt":"According to valence bond theory, what is the hybridization of the central atom of methane?","Completion":"sp\\({}^{3}\\)"} {"Type":"TOSS-UP\n\nEARTH AND SPACE Multiple Choice","Prompt":"Which of the following is NOT evidence that liquid water was once present on Mars?\n\nW) Large flood channels\n\nX) Sedimentary layering\n\nY) Sand dunes\n\nZ) Eroded craters","Completion":"Y) SAND DUNES"} {"Type":"TOSS-UP\n\nPHYSICS Multiple Choice","Prompt":"An unpowered open truck is coasting horizontally with a constant velocity along a frictionless track underneath a grain bin when grain falls vertically into the truck, partly filling it as it moves along. Which of the following will happen to the horizontal momentum of the truck with the accumulating grain?\n\nW) It will decrease\n\nX) It will increase\n\nY) It will stay the same\n\nZ) It will first increase and then decrease","Completion":"Y) IT WILL STAY THE SAME"} {"Type":"BONUS\n\nPHYSICS Short Answer","Prompt":"A mass is suspended by two springs of equal length connected in parallel. The mass stretches the springs by a distance A beyond their equilibrium length. The springs are rearranged to be connected in series, so that now the mass stretches the springs by a distance B beyond their equilibrium length. By number, identify which of the following is true:\n\nA is greater than B, A is less than B, A and B are the same distance.","Completion":"2"} {"Type":"BONUS\n\nMATH Short Answer","Prompt":"For a function f(\\(x\\)), suppose that you know that f(-= -2 and that f(= 2. What would you need to know about \\(f\\) in order to invoke the Intermediate Value Theorem to conclude that there is at least one zero of \\(f\\) on [-1, 3][the closed interval from negative 1 to 3]?","Completion":"\\(f\\) IS CONTINUOUS ON [-1, 3] (ACCEPT: F IS CONTINOUS ON THE INTERVAL)"} {"Type":"TOSS-UP\n\nBIOLOGY Multiple Choice","Prompt":"Which of the following examples describes a postzygotic reproductive barrier that creates or enforces reproductive isolation?\n\nW) Two closely-related species of garter snakes live in the same area, but one lives mainly in water while the other lives mainly on land\n\nX) Two species of closely-related skunks have different mating seasons\n\nY) Two species of closely-related snails have morphological differences that prevent successful inter-species mating\n\nZ) Two species of closely related equines may successfully mate to produce offspring, but the offspring is sterile","Completion":"Z) TWO SPECIES OF CLOSELY RELATED EQUINES MAY SUCCESSFULLY"} {"Type":"TOSS-UP\n\nMATH Short Answer","Prompt":"What is the degree of the polynomial: \\((x-^{3}\\)\\((x-^{4}\\)[open parenthesis x minus 3 close parenthesis cubed times open parenthesis x minus 7 close parenthesis to the power of 4]?","Completion":"7"} {"Type":"BONUS\n\nMATH Multiple Choice","Prompt":"If \\(x\\) is an angle such that \\(\\tan(x)=5\/12\\) and \\(\\pi